You are on page 1of 674

AIIMS

EBD_7100
nd
• Corporate Office : 45, 2 Floor, Maharishi Dayanand Marg, Corner Market,
Malviya Nagar, New Delhi-110017

Tel. : 011-49842349 /50

By
Disha Experts

Typeset by Disha DTP Team

DISHA PUBLICATION
ALL RIGHT RESERVED

© Copyright Publisher
All rights reserved. No part of this publication may be reproduced in any form without prior permission
of publisher. The author and the publisher do not take any legal responsibility for any errors or
misrepresentations that might have crept in. We have tried and made our best efforts to provide
accurate up-to-date information in this book.

For further information about the products from DISHA,


log on to www.dishapublication.com or email to info@dishapublication.com
CONTENTS

1. Units and Measurements 1-6 16. Electrostatic Po tential and
2. Motion in a Straight Line 7-12 Capacitance 117-124

3. Motion in a Plane 13-20 17. Current E lectricity 125-136

4. Laws of Motion 21-28 18. Moving Char ges and Magn etism 137-145

5. Work, Energy and Power 29-36 19. Magnetism and Matter 146-151

6. System of P articles and Rota tional 20. Electromagnetic Induction 152-156


Motion 37-46 21. Alternating Curr ent 157-163
7. Gravitation 47-54 22. Electromagnetic Waves 164-165
8. Mechanical Properties of S olids 55-57 23. Ray Optics and Optical Instruments 166-177
9. Mechanical Properties o f Fl uids 58-67 24. Wave O ptics 178-185
10. Thermal Properties o f M atter 68-75 25. Dual Nat ure o f Rad iation and Matter 186-191
11. Thermodynamics 76-81 26. Atoms 192-196
12. Kinetic Theory 82-86 27. Nuclei 197-206
13. Oscillations 87-95 28. Semi-Conductor, E lectronics :
14. W aves 96-106 Materials, D evices a nd Simple
Circuits 207-214
15. Electric Charges and Fields 107-116
29. Communication Systems 215-216

1. Some B asic Concepts of Ch emistry 1-6 16. Solutions 92-98


2. Structure of Atom 7-13 17. Electrochemistry 99-104
3. Classification of Elemen ts an d 18. Chemical K inetics 105-110
Periodicity in Properties 14-16 19. Surface C hemistry 111-115
4. Chemical Bonding a nd Mo lecular 20. General Pr inciples a nd Processes
Structure 17-22 of Isolation o f Elements 116-118
5. States of Matter 23-28 21. The p-Block Elements (Group 15,
6. Thermodynamics 29-39 16, 17 and 18) 119-128
7. Equilibrium 40-50 22. The d- and f -Block Elements 129-134
8. Redox Reactions 51-54 23. Coordination Compounds 135-144
24. Haloalkanes and Ha loarenes 145-151
9. Hydrogen 55-56
25. Alcohols, Ph enols and Ethers 152-159
10. The s- Block Elements 57-61
26. Aldehydes, Ketones a nd Car boxylic
11. The p-Block Elements 62-67
Acids 160-171
12. Organic C hemistry–Some B asic
27. Amines 172-178
Principles and Techniques 68-76
28. Biomolecules 179-183
13. Hydrocarbons 77-85
29. Polymers 184-185
14. Environmental Chemistry 86-87
30. Chemistry in Everyday Life 186-188
15. The Solid State 88-91
31. Analytical Ch emistry 189-192
EBD_7100

1. The Living W orld 1-4 22. Chemical Co-ordination and
2. Biological C lassification 5-9 Integration 114-120
3. Plant Kingdom 10-17 23. Reproduction i n Organisms 121-123
4. Animal Kingdom 18-27 24. Sexual Reproduction i n
5. Morphology of Flowering Plants 28-34 Flowering Pl ants 124-128
6. Anatomy of Flowering P lants 35-41 25. Human Reproduction 129-135
7. Structural Organisation in Animals 42-45 26. Reproductive Heal th 136-138
8. Cell : The U nit of L ife 46-51 27. Principles o f Inheritance and
9. Biomolecules 52-58 Variation 139-147
10. Cell Cycle and C ell Division 59-62 28. Molecular Basis of I nheritance 148-154
11. Transport i n P lants 63-66 29. Evolution 155-160
12. Mineral Nutrition 67-69 30. Human Health & Diseases 161-168
13. Photosynthesis i n Higher P lants 70-74 31. Strategies for E nhancement i n F ood
Production 169-174
14. Respiration in Pl ants 75-79
32. Microbes i n Human W elfare 175-177
15. Plant Growth and Development 80-83
33. Biotechnology: P rinciples a nd Processes
16. Digestion and A bsorption 84-89
178-182
17. Breathing and Exchange of Gases 90-93
34. Biotechnology & its Applications 183-185
18. Body Fluids a nd Circulation 94-99
35. Organisms and Popul ations 186-190
19. Excretory Products and their
36. Ecosystem 191-195
Elimination 100-103
37. Biodiversity and its Conservation 196-200
20. Locomotion and Mov ement 104-107
38. Environmental Issues 201-208
21. Neural Control and Coordination 108-113


1. History 1-3 5. General Awar eness 15-19
2. Indian Polity 4-6 6. Current A ffairs 21-22
3. Geography 7-9 7. Logical Thinking 24
4. General Science 10-13
1 Units and Measurements

TYPE A : MULTIPLE CHOICE QUESTIONS 8. Velocity of light is equal to [2002]


1. The dimensions of Planck’s constant are [1997] 1
-3 -1 -2 -1 (a) e 0m 0
(b) e0 / m0
(a) [ML T ] (b) [ML T ]
(c) [M 0 L-1T -3 ] (d) [ ML2T -1 ] (c) e 0 / m0 (d) e0 m
2. The dimensional formula of magnetic flux is
9. Using mass (M), length (L), time (T) and current
[1998]
(A) as fundamental quantities, the dimensions
(a) [ML0T -2 A -1 ] (b) [ML2 T -2 A -1 ] of permeability are : [2003]
(c) [ ML2 T -1A 3 ] (d) [ M 0 L-2 T -2 A -2 ] (a) [M–1LT –2A] (b) [ML–2 T–2 A–1 ]
3. The dimensional formula of the constant a in (c) [MLT–2A–2 ] (d) [MLT–1A–1 ]
Vander Waal's gas equation 10. Using mass (M), length (L), time (T) and current
(A) as fundamental quantities, the dimensional
æ a ö formula of permittivity is : [2004]
çè P + 2 ÷ø (V - b) = RT is: [1999]
V (a) [ML–2T2A] (b) [M–1L–3 T4 A2 ]
(a) [ ML4 T - 1 ] (b) [ ML2 T -2 ] (c) [MLT–2A] (d) [ML2T –1A2 ]
11. “Parsec” is the unit of : [2005]
(a) time
(c) [ ML5 T -3 ] (d) [ML5 T -2 ]
(b) distance
4. What is the dimensional formula of gravitational
(c) frequency
constant? [2000]
(d) angular acceleration
(a) [ML2 T - 2 ] (b) [ ML-1T -1 ] 12. Dimensions of electrical resistance are : [2005]

(c) [M -1L3T - 2 ] (d) none of these (a) [ML2 T -3 A -1 ] (b) [ML2 T -3 A -2 ]


5. Which of the following pairs does not have
similar dimensions ? [2001] (c) [ML3 T -3 A -2 ] (d) [ML-1L3 T 3 A 2 ]
(a) tension and surface tension 13. The magnetic moment has dimensions of:[2006]
(b) stress and pressure (a) [LA] (b) [L2A]
(c) Planck’s constant and angular momentum –1
(c) [LT A] (d) [L2T–1A]
(d) angle and strain 14. Which of the following physical quantities do
6. The length and breadth of a metal sheet are not have same dimensions? [2007]
3.124 m and 3.002 m respectively. The area of
(a) pressure and stress
this sheet upto four correct significant figure
is : [2001] (b) tension and surface tension
(a) 9.378 m2 (b) 9.37 m2 (c) strain and angle
(c) 9.378248 m2 (d) 9.3782 m2 (d) energy and work.
7. The dimensions of energy are [2002] 15. What is the dimensions of impedance? [2007]
(a) [ML3T–3] (b) [ML–1T–1] (a) ML2T –3I –2 (b) M –1 L–2 T3 I2
(c) [ML2T–2] (d) [MT–2] (c) ML3T –3I –2 (d) M –1 L–3 T3 I2
EBD_7100
P-2 Topicwise AIIMS Solved Papers – PHYSICS
16. The speed of light (c), gravitational constant 24. The least count of a stop watch is 0.2 second.
(G) and planck's constant (h) are taken as The time of 20 oscillations of a pendulum is
fundamental units in a system. The dimensions measured to be 25 second. The percentage error
of time in this new system should be [2008] in the measurement of time will be [2015]
(a) 8% (b) 1.8%
(a) G1/ 2 h1/ 2 c-5 / 2 (b) G -1/ 2 h1/ 2 c1/ 2 (c) 0.8% (d) 0.1%
(c)G1/ 2 h1/ 2 c-3 / 2 (d) G1/ 2 h1/ 2 c1/ 2 25. If e is the charge, V the potential difference, T the
17. Dimensions of coefficient of viscosity is eV
temperature, then the units of are the same
[2010] T
2 –3 –4
as that of [2016]
(a) [MT ] (b) [ML T ]
(a) Planck’s constant
(b) Stefan’s constant
(c) [ML–1T–2] (d) [ML–1T–1] (c) Boltzmann constant
18. Which of the following pair of quantities do not (d) Gravitational constant
have the same dimensions : [2011] 26. If the capacitance of a nanocapacitor is
(a) Potential gradient, electric field measured in terms of a unit ‘u’ made by
(b) Torque, kinetic energy combining the electric charge ‘e’, Bohr radius
(c) Light year, time period ‘a0’, Planck’s constant ‘h’ and speed of light ‘c’
then [2016]
(d) Impedance, reactance 2
e h hc
19. The dimensional formula for torque is : [2011] (a) u = (b) u = 2
a0 e a0
(a) ML2 T–2 (b) ML–1 T –1
(c) L2T –1 (d) M2 T –2 K –1 e2 c e2 a 0
(c) u= (d) u =
20. What is the fractional error in g calculated from ha 0 hc
27. A force F is applied onto a square plate of side
T = 2 p l / g ? Given fraction errors in T and l L. If the percentage error in determining L is 2%
and that in F is 4%, the permissible percentage
are ± x and ± y respectively? [2012]
error in determining the pressure is [2017]
(a) x + y (b) x – y
(a) 2% (b) 4%
(c) 2x + y (d) 2x – y (c) 6% (d) 8%
21. The dimensional formula of farad is [2012]
TYPE B : ASSERTION REASON QUESTIONS
(a) [M -1L-2 TQ] (b) [M -1L-2T 2Q 2 ]
Directions for (Qs. 28-30) : These questions consist
of two statements, each printed as Assertion and
(c) [M -1L-2 TQ 2 ] (d) [M -1L-2 T 2 Q]
Reason. While answering these questions, you are
22. The density of a cube is measured by measuring required to choose any one of the following five
its mass and length of its sides. If the maximum responses.
error in the measurement of mass and length are (a) If both Assertion and Reason are correct and
4% and 3% respectively, the maximum error in the Reason is a correct explanation of the
the measurement of density will be [2013] Assertion.
(a) 7% (b) 9% (b) If both Assertion and Reason are correct but
(c) 12% (d) 13% Reason is not a correct explanation of the
Assertion.
23. The dimensions of æç 1 ö÷e0E 2 ( e 0 : permittivity of (c) If the Assertion is correct but Reason is
è 2ø incorrect.
free space, E electric field) are [2014] (d) If both the Assertion and Reason are incorrect.
(a) [MLT–1] (b) [ML2T–2] (e) If the Assertion is incorrect but the Reason is
(c) [ML–1T–2] (d) [ML2T–1] correct.
Units and Measurements P-3

28. Assertion : The dimensional formula for relative (b) If both Assertion and Reason are correct, but
velocity is same as that of the change in velocity. Reason is not the correct explanation of
Reason : Relative velocity of P w.r.t. Q is the Assertion.
ratio of velocity of P and that of Q. [2002] (c) If Assertion is correct but Reason is incorrect.
29. Assertion : Specific gravity of a fluid is a (d) If both the Assertion and Reason are incorrect.
dimensionless quantity. 31. Assertion : When percentage errors in the
Reason : It is the ratio of density of fluid to the measurement of mass and velocity are 1% and
density of water. [2005] 2% respectively, the percentage error in K.E.
is 5%.
30. Assertion : The error in the measurement of
radius of the sphere is 0.3%. The permissible DE Dm 2Dv
Reason : = + [2010]
error in its surface area is 0.6% E m v
Reason : The permissible error is calculated by 32. Assertion : The number of significant figures
depends on the least count of measuring
DA 4 Dr
the formula = [2008] instrument.
A r Reason : Significant figures define the accuracy
Directions for (Qs.31-33) : Each of these questions of measuring instrument. [2016]
contains an Assertion followed by Reason. Read them 33. Assertion: In the measurement of physical
carefully and answer the question on the basis of quantities direct and indirect methods are used.
following options. You have to select the one that Reason : The accuracy and precision of
best describes the two statements. measuring instruments along with errors in
(a) If both Assertion and Reason are correct and measurements should be taken into account,
Reason is the correct explanation of Assertion. while expressing the result. [2017]
EBD_7100
P-4 Topicwise AIIMS Solved Papers – PHYSICS

Type A : Multiple Choice Questions


MLT -2
1. (d) E = hn, h is Planck’s constant m=
1
A.AT.LT -1
[E] 2 -2 L
ML T
[h] = = = ML2 T -1 [n is no. of turns per unit length]
[ n] T -1
f = MLA -2T -2
2. (b) F = Bqv Þ F = qv
A 1 qq qq
10. (b) F= . 1 2 Þ Î0 = 1 2
F.A MLT L -2 2 4p Î0 r 2 4pFr 2
f= = = ML+2 A -1T -2
qv ATLT -1 AT.AT A2T2
= = M -1L-3 A 2 T 4
a MLT -2 L2 ML3 T -2
3. (d) Here the dimension of will be equal to
V2 11. (b) Parsec is a unit of length on the
astronomical scale. It is the distance of an
a object that will show a parallax of 1" of arc
pressure so = ML-1T - 2
3 2 from opposite side of a baseline (radius)
(L )
equal to the distance between sun and
a = ML5T - 2 earth.
1 parsec = 3.1 ´ 1016 m
G.M.M
4. (c) F= = MLT - 2 V W æ Wö
L2 12. (b) R= = Work = V ´ q Þ V =
I qI çè q ÷ø
MLT -2 L2
G= = M -1L3 T -2 ML2 T -2
M 2 R= = ML2 A - 2 T -3
AT.A
5. (a) Tension will have dimension of force and
surface tension will have dimension of force 13. (b) Magnetic moment of a coil carrying current
per unit length so they have different is,
dimensions. M = I.A
6. (a) Area of metal sheet = 3.124 × 3.002 [A is area of cross section and i is current]
= 9.378248 Dimension of M = AL2
Now, the result must have significant 14. (b) Tension is a force and surface tension is
figures equal to the least of figure being force per unit area hence their dimensions
multiplied, so, Area of metal sheet = 9.378 are not same.
m2 15. (a) Impedance is same as resistance but in ac
7. (c) Torque = Force × distance = Energy circuit
\ Dimension of impedance
= MLT -2 L = ML2T -2
dimension of voltage
=
1 dimension of current
8. (a) Velocity of light, c =
e0m0
[V] [ML2 T -3 I-1 ]
9. (c) B = mni ; F = Bqv = = = [ML2T–3I–2]
[I] I
F 16. (a) Let time, T µ c x G y h z
F = mni qv Þ m =
niqv Þ T = kc x G y h z
Units and Measurements P-5

Taking dimensions on both sides 2


éQ ù éQ ù -1 -2 2 2
0 0 1 -1 x -1 3 -2 y
[M L T ] = [LT ] [M L T ] [ML T ] 2 -1 z 21. (b) [C] = ê ú = ê ú = [M L T Q ]
ë V û ëê W ûú
i.e.,
Mass
[M 0 L0 T1 ] = [M - y + z Lx + 3y + 2z T - x - 2y - z ] 22. (d) Density =
Volume
Equating power of M, L, T on both sides,
we get M Dr DM DL
r= 3 , = +3
-y + z = 0 … (1) L r M L

x + 3y + 2z = 0 … (2) % error in density = % error in Mass + 3


(% error in length) = 4 + 3(3) = 13%
- x - 2y - z = 1 … (3)
æ1ö
23. (c) Here ç ÷e 0 E 2 represents energy per unit
From (1) Þ z = y
è2ø
Adding (2) and (3) Þ y + z = 1 volume
or 2y = 1 [From (1)]
é ML2 T -2 ù
Energy
1 [ e0 ] éE =2ù
=ë û
i.e., y = ë û volume éL ù
3
2 ë û
1 = ML–1T–2
\z = y =
2 0.2
24. (c) ´ 100 = 0.8
Putting these values in (2) we get 25
3 -5 eV W PV
x+ + 1 = 0 or x= 25. (c) = = =R
2 2 T T T
Hence, [T] = [G1/ 2 h1/ 2 c -5 / 2 ] R
and = Boltzmann constant.
N
F 26. (d) Let unit ‘u’ related with e, a0, h and c as
17. (d) Coefficient of viscosity, h = Adv
follows :
[F] = [Force] = MLT–2 dx [u] = [e]a [a0]b [h]c [C]d
[A] = [Area] = L2 Using dimensional method,
é dv ù LT -1 [M–1L–2T+4A+2]
êë dx úû = [Velocity gradient] = = T –1 = [A1T1]a[L]b[ML2T–1]c[LT–1]d
L
[M–1L–2T+4A+2] = [Mc Lb+2c+d Ta–c–d Aa]
MLT -2
\ [h] = 2 -1 = [ML–1 T –1]. a = 2, b = 1, c = – 1, d = – 1
LT
18. (c) Light year has the dimensions of distance e 2 a0
and time period is time. \ u=
hc
19. (a) t = Fr = MLT–2L = ML2T–2
F F
l l 27. (d) As, pressure P = = 2
20. (c) From T = 2p ; g = 4p 2 A L
g T2
DF DL
% Error = ´100 + 2 ´100
D g Dl 2 DT F L
= + = ( y + 2x )
g l T = 4 + 2 × 2 = 8%
EBD_7100
P-6 Topicwise AIIMS Solved Papers – PHYSICS

Type B : Assertion Reason Questions DA Dr


But =4 is false.
28. (e) Relative velocity which is vector A r
subtraction of two velocities will also be a 31. (a) Both Assertion and Reason are correct and
vector of the form of velocity so, its Reason is the correct explanation of
dimensional formula will remain unchanged. Assertion.
Relative velocity is measured not by
calculating ratio but by calculating 1 2
Kinetic energy, E = mv .
difference. 2
29. (a) Specific gravity of fluid Differenting both side
density of fluid DE Dm 2 Dv
= = +
density of water E m v
It is a ratio. DE 1 2 5
= +2´ = = 5%
30. (c) Area of the sphere, A = 4 pr 2 E 100 100 100
% error in area = 2 × % error in radius 32. (b) Significant figure refers to the accuracy of
measurement and accuracy of measurment
DA Dr
i.e., ´ 100 = 2 ´ ´ 100 also depends upon the least count of
A r measuring instrument.
= 2 ´ 0.3% = 0.6% 33. (a)
2 Motion in a Straight Line

TYPE A : MULTIPLE CHOICE QUESTIONS 7. A body starting from rest moves along straight
line with a constant acceleration. The variation
1. If a car at rest accelerated uniformly to a speed of speed (v) with distance (s) is represented by
of 144 km/hour in 20 second it covers a distance : the graph : [2003]
(a) 400 m (b) 1440 m [1997]
(c) 2880 m (d) 25 m
v v
2. A ball is dropped from a bridge 122.5 m high.
(a) (b)
After the first ball has fallen for 2 second, a
second ball is thrown straight down after it, what s s
must be the initial velocity of the second ball be,
so that both the balls hit the surface of water at v v
the same time? [1997] (c) (d)
(a) 26.1 m/s (b) 9.8 m/s
(c) 55.5 m/s (d) 49 m/s s s
8. A ball is thrown vertically upwards. Which of
3. A particle is thrown vertically upwards. Its
the following plots represents the speed-time
velocity at half of the height is 10 m/s, then
graph of the ball during its flight if the air
the maximum height attained by it will be :
resistance is not ignored? [2003]
(g = 10 m/s2)
(a) 10 m (b) 20 m [1999]
(c) 15 m (d) 25 m s s
4. A body is released from the top of the tower H (a) (b)
metre high. It takes t second to reach the ground. t t
Where is the body after t/2 second of release ?
(a) at 3H/4 metre from the ground [2000]
(b) at H/2 metre from the ground s s
(c) at H/6 metre from the ground (c) (d)
(d) at H/4 metre from the ground t t
5. A body starts from rest with an acceleration a1. 9. Which of the following velocity-time graphs
After two seconds another body B starts from shows a realistic situation for a body in motion?
rest with an acceleration a2. If they travel equal [2004, 2007]
distances in fifth second after the starts of A,
the ratio a1 : a2 will be equal to : [2001]
v v
(a) 9 : 5 (b) 5 : 7
(c) 5 : 9 (d) 7 : 9 (a) (b)
6. Three different objects m 1 , m 2 and m 3 are
allowed to fall from rest and from the same point t t
O along three different frictionless paths. The
speeds of the three objects, on reaching the
ground, will be in the ratio of [2002] v v
(a) m1 : m2 : m3 (b) 1 : 1 : 1 (c) (d)
1 1 1
(c) m1 : 2m2 : 3m3 (d) : :
m1 m 2 m 3 t t
EBD_7100
P-8 Topicwise AIIMS Solved Papers – PHYSICS
10. Two spheres of same size one of mass 2 kg and 16. A ball is thrown upwards. Its height varies with
another of mass 4 kg are dropped simultaneously time as follows :
from the top of Qutab Minar(height = 72m).
When they are 1m above the ground, the two height
(m)
spheres have the same: [2006]
(a) momentum (b) kinetic energy
(c) potential energy (d) acceleration h
11. A particle is thrown vertically upwards with a
velocity of 4ms–1. The ratio of its accelerations time(s)
after 1s and 2s of its motion is [2009] 1 2 5 6
(a) 2 (b) 9.8 If the acceleration due to gravity is 7.5 m/s2,
(c) 1 (d) 4.9 then the height h is : [2011]
12. A body is thrown vertically upwards with a (a) 10 m (b) 15 m
velocity of 19.6 ms–1. The position of the body (c) 20 m (d) 25 m
after 4 s will be [2009] 17. The coordinates of a moving particle at any time
(a) at the highest point t are given by x = a t2 and y = b t2. The speed of
(b) at the mid-point of the line joining the the particle is [2012]
starting point and the highest point
(c) at the starting point (a) 2 t (a + b) (b) 2t (a2 + b2 )
(d) none of the above
13. The position(x) of a particle at any time(t) is (c) 2 t (a 2 - b 2 ) (d) (a 2 + b 2 )
given by [2009] 18. A ball is released from the top of a tower of
x(t) = 4t3 – 3t2 + 2 height h meters. It takes T seconds to reach the
The acceleration and velocity of the particle at
T
any time t = 2 sec are respectively ground. What is the position of the ball at
(a) 16 ms–2 and 22 ms–1 3
second
(b) 42 ms–2 and 36 ms–1
(c) 48 ms–2 and 36 ms–1 8h
(a) meters from the ground [2012]
(d) 12 ms–2 and 25 ms–1 9
14. A stone is projected vertically up from the 7h
bottom of a water tank. Assuming no water (b) meters from the ground
9
resistance it will go up & come down in same h
time but if water drag is present then the time it (c) meters from the ground
9
takes to go up, tup and the time it takes to come
down, tdown are related as [2009] 17 h
(d) meters from the ground
(a) tup > tdown 18
(b) tup = tdown 19. A body starts from rest at time t = 0, the acceleration
time graph is shown in the figure. The maximum
(c) tup < tdown
velocity attained by the body will be [2014]
(d) can not say
Acceleration
15. A student is standing at a distance of 50 metre 2
from the bus. As soon as the bus begins its (m/s ) 10
motion with an acceleration of 1 ms–2, the
student starts running towards the bus with a
uniform velocity u. Assuming the motion to be
along a straight road, the minimum value of u, 11 Time
so that the student is able to catch the bus is (sec.)
(a) 8 ms–1 (b) 5 ms–1 [2010] (a) 110 m/s (b) 55 m/s
(c) 12 ms –1 (d) 10 ms–1 (c) 650 m/s (d) 550 m/s
Motion in a Straight Line P-9

20. Two bodies begin a free fall from the same height 24. Assertion : Retardation is directly opposite to
at a time interval of N s. If vertical separation the velocity.
between the two bodies is 1 after n second from Reason : Retardation is equal to the time rate of
the start of the first body, then n is equal to decrease of speed. [2002]
[2016]
Directions for (Qs.25-28) : Each of these questions
1 contains an Assertion followed by Reason. Read them
(a) nN (b)
gN carefully and answer the question on the basis of
following options. You have to select the one that
1 N 1 N
(c) + (d) - best describes the two statements.
gN 2 gN 4 (a) If both Assertion and Reason are correct and
21. From a balloon moving upwards with a velocity Reason is the correct explanation of Assertion.
of 12 ms–1, a packet is released when it is at a (b) If both Assertion and Reason are correct, but
height of 65 m from the ground. The time taken Reason is not the correct explanation of
by it to reach the ground is (g = 10 ms–2) Assertion.
(a) 5 s (b) 8 s [2017]
(c) If Assertion is correct but Reason is incorrect.
(c) 4 s (d) 7 s
(d) If both the Assertion and Reason are incorrect.
TYPE B : ASSERTION REASON QUESTIONS 25. Assertion : Two balls of different masses are
Directions for (Qs. 22-24) : These questions consist thrown vertically upward with same speed. They
of two statements, each printed as Assertion and will pass through their point of projection in the
Reason. While answering these questions, you are downward direction with the same speed.
required to choose any one of the following five Reason : The maximum height and downward
responses. velocity attained at the point of projection are
(a) If both Assertion and Reason are correct and independent of the mass of the ball. [2013]
the Reason is a correct explanation of the 26. Assertion : The two bodies of masses M and m
Assertion. (M > m) are allowed to fall from the same height
(b) If both Assertion and Reason are correct but if the air resistance for each be the same then
Reason is not a correct explanation of the both the bodies will reach th e earth
Assertion. simultaneously.
(c) If the Assertion is correct but Reason is Reason : For same air resistance, acceleration of
incorrect. both the bodies will be same. [2014]
(d) If both the Assertion and Reason are incorrect. 27. Assertion : In a free fall, weight of a body
(e) If the Assertion is incorrect but the Reason is becomes effectively zero.
correct.
Reason : Acceleration due to gravity acting on
22. Assertion : A body can have acceleration even
a body having free fall is zero. [2014]
if its velocity is zero at a given instant of time.
28. Assertion : Velocity-time graph for an object
Reason : A body is numerically at rest when it
in uniform motion along a straight path is
reverses its direction. [1998]
a straight line parallel to the time axis.
23. Assertion : A body with constant acceleration
always moves along a straight line. Reason : In uniform motion of an object
Reason : A body with constant acceleration may velocity increases as the square of time
not speed up. [1998] elapsed. [2015]
EBD_7100
P-10 Topicwise AIIMS Solved Papers – PHYSICS

Type A : Multiple Choice Questions Let H1 be the height after t/2 secs. So
5 distance of fall = H – H1
1. (a) u = 0, v = 144 km/hour = 144 ´ m / sec
18 2
= 40 m/sec 1 ætö
H – H1 = gç ÷
v = u + at 2 è2ø
v - u 40 - 0 1
Þ a= = = 2 m/sec2 Þ H - H1 = gt 2 .......(ii)
t 20 8
1 Dividing (i) and (ii),
\ s = ut + at 2
2
H - H1 1 2 1
1 = ´ =
= ´ 2 ´ (20) 2 = 400 m H 8 1 4
2
3
2. (a) Time taken by the first object to reach the Þ 4H - 4H1 = H Þ H1 = H
ground = t, so 4
5. (c) Distance travelled in fifth second for first
1
122.5 = ut + gt 2 body = distance travelled in 3rd second for
2 second body,
1 S5 = S3
122 .5 = ´ 10 ´ t 2
2
(2t - 1)a
Þ t = 5 sec( approx) St = u +
2
Time to be taken by the second ball to reach 9
the ground = 5 – 2 = 3 sec. S5 = 0 + a1
If u be its initial velocity then, 2
5
1 2 1 S3 = 0 + a 2
122.5 = u ´ 3 + gt = 3u + ´ 10 ´ 9 2
2 2
9 5 a1 5
3u = 122.5 - 45 = 77.5 a1 = a 2 Þ =
u = 26 (approx.) 2 2 a2 9
3. (a) Let maximum height be H
B
From the formula, v 2 = u 2 - 2gs 6. (b)

(10)2 = u 2 - 2gH / 2 = u 2 - gH .......(1)


For attaining maximum height, v = 0
h
0 = u 2 - 2gH Þ u 2 = 2gH
Putting the value of u2 in (1),
100 = 2gH - gH = gH
A C1 C2 C3
100 100
H= = = 10 metre
g 10 For paths BC1, BC 2 & BC 3 the height is
same that is h. The terminal velocity be v
1
4. (a) Applying S = ut + gt 2 for the Ist case then for all cases, v = 2gh
2
1 2 So, all will have same value of terminal
H= gt .......(i) velocity.
2
Motion in a Straight Line P-11

7. (d) v2 = u2 + 2as
m
v2 = 2as as u = 0 14. (c) While moving up & while
The graph between v and s will be of the mg
form of parabola which will be symmetric Fdrag
with respect to v-axis. So curve (d) is the
right answer. Fdrag
8. (d) For a body going in upward direction
moving down
v = u – gt
mg
dv
The slope of the graph, = -g (constant) \ aup > adown
dt
But when we take into account the effect Hence to cover same distance tup < tdown.
of resistance it will have sharper slope. 15. (d) Let the student travels distance s1 in time t
Curve (d) fits into this result. and catches the bus.
9. (b) Time cannot reverse itself or it can only go s1 = ut
forward. In graph (a), (c) & (d) some portion ...(1)
d = 50m s2
of graph has shown time changing in such
a way or time is going from high value to
low value which is not practical. s1
Distance travelled by the bus in time t
Graph (b) is the answer.
10. (d) Since their masses are different they will 1
s2 = 0 + at2
have different momentum, kinetic energy 2
and potential energy. But their acceleration ...(2)
will be same which will be equal to g. The student is able to catch the bus if,
11. (c) We know that the acceleration in a motion s1 = d + s2
under gravity is constant which is 9.8 ms–2. 1
ut = d + 1 × 12
2
9.8 or, 2ut = 2d + t2
Hence, the required ratio will be =1
9.8 or, t2 – 2ut + 2d = 0
12. (c) Clearly the time taken by the particle to Solving the quadratic equation
reach the highest point is given by t = – 2 ± 4u 2 - 8d
v = u – gt
= – 2 ± 2 u 2 - 2d
u - v 19.6 - 0 For t to be real
or, t = =
g 9.8 u ³ 2d
or, t = 2 s. ³ 2 ´ 50 = 10 m/s.
Therefore, the particle will reach at the 16. (b) Velocity at highest point becomes zero
starting point itself after 4 s.
\ 0 = u – at
13. (c) We have x(t) = 4t3 – 3t2 + 2 or u = at
dx = 7.5 × 3.5 = 62.25 m/s
Þ v= = 12t 2 - 6t
dt 1
y1 = u ´ 1 - ´ 7.5 ´ 12
dv 2
and a = = 24t
dt 1
y2 = u ´ 2 - ´ 7.5 ´ 22
\ v at t = 2s is 12(2)2 – 6(2) i.e., 36 ms–1 2
and a at t = 2s is 24 × 2 i.e., 48 ms–2 h = y2 – y1 = 15
EBD_7100
P-12 Topicwise AIIMS Solved Papers – PHYSICS

dr Type B : Assertion Reason Questions


17. (b) r = i a t2 + j b t2, v= = i2a t + j2b t
dt 22. (a) When a body is thrown upwards vertically,
at the highest point its velocity becomes
\ Magnitude of v = (4 a 2 t 2 + 4 b 2 t 2 ) zero but gravitational force continues to
act on it so it has acceleration in downward
= 2 t (a 2 + b2 ) direction even at the highest point. So
assertion is true.
1
18. (a) h = gT 2 A body is numerically at rest but it reverses
2 its direction due to acceleration present in
now for t = T/3 second vertical distance it. Reason is true & it supports assertion.
moved is given by 23. (e) In case of circular motion, constant
2 2
acceleration creates circular motion. In
1 æ Tö
h¢ = g ç ÷ Þ h¢ = 1 ´ gT = h circular motion (uniform) the body in
2 è 3ø 2 9 9 motion does not speed up inspite of
acceleration.
h
\ position of ball from ground = h - decrease in velocity
9 24. (a) Retardation =
time
8h
=
9 It acts opposite to velocity.
19. (b) Vi = 0, Vf = Vmax
1
11 25. (a) h = ut - gt 2 and v2 = u2 – 2gh;
DV = area under the curve = 10 ´ = 55 2
2
These equations are independent of mass.
or Vf - Vi = 55 m / s since Vi = 0 26. (d) The force acting on the body of mass M
Vf = 55 m / s are its weight Mg acting vertically
downward and air resistance F acting
Vf = Vmax = 55 m / s vertically upward.
1 2 1 F
20. (c) y1 = gn , y 2 = g ( n - N ) 2 \ Acceration of the body , a = g -
2 2 M
1
\ y1 - y 2 = g[n 2 - (n - N) 2 ] Now M > m, therefore, the body with larger
2
mass will have great acceleration and it
g
Þ 1= (2n - N)N will reach the ground first.
2
27. (d)
[Q y1 - y 2 = 1]
28. (c) In uniform motion the object moves with
1 N uniform velocity, the magnitude of its velocity
Þ n= +
gN 2 at different instance i.e., at t = 0, t =1, sec,
t = 2sec ..... will always be constant. Thus
1 2 velocity-time graph for an object in uniform
21. (a) s = ut + at
2 motion along a straight path is a straight line
–65 = 12t – 5t2 on solving we get, t = 5s parallel to time axis.
3 Motion in a Plane

TYPE A : MULTIPLE CHOICE QUESTIONS 8. Two projectiles are projected with the same
velocity. If one is projected at an angle of 30°
1. Rain is falling vertically downwards with a and the other at 60° to the horizontal, the ratio of
velocity of 3 km/hr. A man walks in the rain with maximum heights reached, is [2001]
a velocity of 4 km/hr. The rain drop will fall on (a) 1 : 3 (b) 2 : 1
the man with a velocity of [1997] (c) 3 : 1 (d) 1 : 4
(a) 5 km/hr (b) 4 km/hr
9. A stone tied to a string is rotated with a uniform
(c) 1 km/hr (d) 3 km/hr
speed in a vertical plane . If mass of the stone is
2. A body of mass 5 kg is moving in a circle of
m, the length of the string is r and the linear
radius 1 m with an angular velocity of 2 rad/sec.
Then the centripetal acceleration (in m/s2) will speed of the stone is v, when the stone is at its
be [1998] lowest point, then the tension in the string will
(a) 80 N (b) 30 N be (g = acceleration due to gravity) [2001]
(c) 10 N (d) 20 N mv2 mv2
3. A body is projected at such angle that the (a) + mg (b) - mg
horizontal range is three times the greatest r r
height. The angle of projection is [1998] mv
(a) 42° 8' (b) 53° 7' (c) (d) mg
r
(c) 33° 7' (d) 25° 8'
4. An aeroplane moves 400 m towards the north, 10. At the uppermost point of a projectile, its
300 m towards west and then 1200 m vertically velocity and acceleration are at an angle of
upwards, then its displacement from the initial (a) 180° (b) 90° [2002]
position is [1998] (c) 60° (d) 45°
(a) 1600 m (b) 1800 m 11. If vectors P = aî + aĵ + 3k̂ and Q = aî – 2ˆj – k̂
(c) 1500 m (d) 1300 m

( ) ( )
® ® ® ®
are perpendicular to each other, then the positive
5. The angle between P + Q and P - Q will be value of a is [2002]
(a) 90° only [1999] (a) zero (b) 1
(b) between 0° and 180° (c) 2 (d) 3
(c) 180° only 12. The maximum range of a gun horizontal terrain
(d) none of these is 10 km. If g = 10 m/s2 what must be the muzzle
6. Two equal vectors have a resultant equal to velocity of the shell [2004]
either of them, then the angle between them will (a) 400 m/s (b) 200 m/s
be [2000] (c) 100 m/s (d) 50 m/s
(a) 110° (b) 120° 13. A projectile can have the same range R for two
(c) 60° (d) 150° angles of projection. If t 1 and t2 be the times of
7. A stone tied to the end of a string of 80 cm long, flights in the two cases, then the product of the
is whirled in a horizontal circle with a constant two time of flights is proportional to [2006]
speed. If the stone makes 14 revolutions in 25
1 2
sec, then magnitude of acceleration of the same (a) (b) R
will be [2001] R2
(a) 990 cm/sec2 (b) 680 cm/sec2 1
(c) R (d)
(c) 750 cm/sec2 (d) 650 cm/sec2 R
EBD_7100
P-14 Topicwise AIIMS Solved Papers – PHYSICS
14. A car travels 6 km towards north at an angle of (a) 10.6 km (b) 9.6 km
45° to the east and then travels distance of 4 km (c) 7.4 km (d) 5.8 km
towards north at an angle 135° to east. How far 20. For ordinary terrestrial experiments, the
is the point from the starting point? What angle observer in an inertial frame in the following
does the straight line joining its initial and final cases is [2010]
position makes with the east? [2008] (a) a child revolving in a giant wheel
-1
(a) 50 km and tan (5) (b) a driver in a sports car moving with a
constant high speed of 200 kmh–1 on a
(b) 10 km and tan -1 ( 5)
straight road
-1
(c) 52 km and tan (5) (c) the pilot of an aeroplane which is taking
(d) -1 off
52 km and tan ( 5)
(d) a cyclist negotiating a sharp curve
15. If two forces of equal magnitudes act
21. For a particle in a uniformly accelerated circular
simultaneously on a body in the east and the
motion [2011]
north directions then [2009]
(a) the body will displace in the north direction (a) velocity is radial and acceleration has both
(b) the body will displace in the east direction radial and transverse components
(c) the body will displace in the north-east (b) velocity is transverse and acceleration has
direction both radial and transverse components
(d) the body will remain at the rest. (c) velocity is radial and acceleration is
16. Two vectors having equal magnitudes of x units transverse only
acting at an angle of 45° have resultant (d) velocity is transverse and acceleration is
radial only
(2 + 2) units. The value of x is [2009] 22. For a given angle of the projectile if the initial
(a) 0 (b) 1 velocity is doubled the range of the projectile
(c) 2 (d) 2 2 becomes [2011]
17. If R and H represent the horizontal range and (a) Half (b) One -fourth
the maximum height achieved by a projectile then (c) Two times (d) Four times
which of the relation exists? [2009] 23. If we can throw a ball upto a maximum height H,
H R the maximum horizontal distance to which we
(a) = 4cot q (b) = 4cot q can throw it is [2011]
R H
H R (a) 2H (b) 2H
(c) = 4 tan q (d) = 4 tan q
R H H
18. The acceleration of a body in a non-uniform (c) H (d)
2
circular motion is 5 ms–2. Which one of the 24. The vector sum of two forces is perpendicular
following is correct? [2009] to their vector differences. In that case, the
(a) The radial acceleration and the tangential forces [2012]
accelerations are 3 ms –2 and 4 ms –2 (a) cannot be predicted
respectively. (b) are equal to each other
(b) The radial and the tangential accelerations
(c) are equal to each other in magnitude
are 2 ms–2 and 3 ms–2 respectively.
(d) are not equal to each other in magnitude
(c) The radial and the tangential accelerations
are both 5 ms–2. 25. A projectile can have the same range for two
(d) The radial and the tangential acceleration angles of projection. If h 1 and h 2 are maximum
are 5 ms–2 and 3 ms–2 respectively. heights when the range in the two cases is R,
19. An aircraft executes a horizontal loop with then the relation between R, h 1 and h2 is [2013]
a speed of 150 m/s with its wings banked at (a) R = 4 h1h 2 (b) R = 2 h1h 2
an angle of 12°. The radius of the loop is
(g = 10 m/s2) [2010] (c) R = h1h 2 (d) None of these
Motion in a Plane P-15

26. A projectile thrown with velocity v making angle 31. A boy playing on the roof of a 10 m high building
q with vertical gains maximum height H in the throws a ball with a speed of 10m/s at an angle
time for which the projectile remains in air, the of 30º with the horizontal. How far from the
time period is [2013] throwing point will the ball be at the height of 10
m from the ground ? [2017]
(a) H cos q / g (b) 2 H cos q / g
1 3
[ g = 10m/s2 , sin 30o = , cos 30o = ]
(c) 4H / g (d) 8H / g 2 2
(a) 5 5 (b) 6
27. A bomb is released from a horizontal flying (c) 3 (d) 5 3
aeroplane. The trajectory of bomb is [2013]
(a) a parabola (b) a straight line TYPE B : ASSERTION REASON QUESTIONS
(c) a circle (d) a hyperbola Directions for (Q. 32) : These questions consist of
28. A stone tied to the end of a string of 1 m long is two statements, each printed as Assertion and Reason.
whirled in a horizontal circle with a constant While answering these questions, you are required to
speed. If the stone makes 22 revolution in 44 choose any one of the following five responses.
seconds, what is the magnitude and direction (a) If both Assertion and Reason are correct and
of acceleration of the stone? [2014] the Reason is a correct explanation of the
2 –2 Assertion.
(a) p m s and direction along the radius
(b) If both Assertion and Reason are correct but
towards the centre.
Reason is not a correct explanation of the
(b) p2 m s–2 and direction along the radius Assertion.
away from the centre.
(c) If the Assertion is correct but Reason is
(c) p2 m s–2 and direction along the tangent incorrect.
to the circle. (d) If both the Assertion and Reason are incorrect.
(d) p2/4 m s–2 and direction along the radius (e) If the Assertion is incorrect but the Reason is
towards the centre. correct.
29. Two projectiles are fired from the same point 32. Assertion : If a body is thrown upwards, the
with the same speed at angles of projection 60º distance covered by it in the last second of
and 30º respectively. Which one of the following upward motion is about 5 m irrespective of its
is true? [2014] initial speed
(a) Their maximum height will be same Reason : The distance covered in the last
(b) Their range will be same second of upward motion is equal to that
covered in the first second of downward motion
(c) Their landing velocity will be same when the particle is dropped. [2000]
(d) Their time of flight will be same Directions for (Qs.33-37) : Each of these questions
30. A ball is thrown from a point with a speed ‘v0’ at contains an Assertion followed by Reason. Read them
an elevation angle of θ . From the same point carefully and answer the question on the basis of
and at the same instant, a person starts running following options. You have to select the one that
best describes the two statements.
'v0 '
with a constant speed to catch the ball. (a) If both Assertion and Reason are correct and
2 Reason is the correct explanation of Assertion.
Will the person be able to catch the ball? If yes, (b) If both Assertion and Reason are correct, but
what should be the angle of projection θ ? Reason is not the correct explanation of
[2016] Assertion.
(a) No (b) Yes, 30° (c) If Assertion is correct but Reason is incorrect.
(c) Yes, 60° (d) Yes, 45° (d) If both the Assertion and Reason are incorrect.
EBD_7100
P-16 Topicwise AIIMS Solved Papers – PHYSICS

33. Assertion : The driver in a vehicle moving with Reason : The centripetal acceleration in circular
a constant speed on a straight road is an inertial motion is dependent on angular velocity of the
frame of reference. body. [2010]
36. Assertion : Centripetal and centrifugal forces
Reason : A reference frame in which Newton's
cancel each other.
laws of motion are applicable is non-inertial.
Reason : Centrifugal force is a reaction of
[2009]
centripetal force. [2011]
34. Assertion : A tennis ball bounces higher on 37. Assertion : The magnitude of velocity of two
hills than in plains. boats relative to river is same. Both boats start
Reason : Acceleration due to gravity on the hill simultaneously from same point on one bank
is greater than that on the surface of earth. [2009] may reach opposite bank simultaneously
moving along different paths.
35. Assertion : When a particle moves in a circle
Reason : For boats to cross the river in same
with a uniform speed, its velocity and
time. The component of their velocity relative to
acceleration both changes.
river in direction normal to flow should be same.
[2015]
Motion in a Plane P-17

Type A : Multiple Choice Questions 5. (b)


A® ®
Q P +Q
D 4 km/h C A
1. (a) ®
4 km/h O P
R –Q
3 km/h
B ® ®
B P–Q
In the figure, CB represents velocity of rain, ur ur
CA represents velocity of the man. To find In the figure OA represents ( P + Q ) , OB
relative velocity of the rain with respect to ur ur
man we add a velocity equal to that of man represents ( P - Q) . It is clear from the
in opposite direction to the velocity of rain. uuur uuur
figure that angle between OA and OB
It has been depicted by line CD. Now rain
has two velocities simultaneously. Their may be between 0° and 180°.
resultatnt, 6. (b) Applying the formula,

R 2 = 4 2 + 32 Þ R = 5 R 2 = P 2 + Q 2 + 2PQ cos a
which gives us the value of relative velocity
P 2 = P 2 + P 2 + 2PP cos a
of rain.
= 2P 2 + 2P 2 cos a = 2P 2 (1 + cos a )
2. (d) Centripetal acceleration = w 2 r = 2 ´ 2 ´ 1
= 4 m/s2 1 a 1
2 2 2
1 + cos a = Þ 2 cos 2 =
u sin 2q u sin q 2 2 2
3. (b) R = 3 H ; R = ;H =
g 2g a 1 a 1
cos 2 = Þ cos = = cos 60°
2 2 2 2 4 2 2
u sin 2q 3u sin q
=
g 2g a
= 60° Þ a = 120°
2 2
3 sin q
2 sin q cos q = 7. (a) Centripetal accn =w2r = (2pn)2× r
2
4 14
tan q = Þ q = 53 °7 ' where frequency, n =
3 25
4. (d) D
\ accn = 4 ´ 22 ´ 22 ´ 14 ´ 14 ´ 80
7 7 25 25
1200 m
300 m
C A = 990 cm / sec 2
8. (a) For maximum height
500 m 400 m
u 2 sin 2 a
H=
2g
B
Here CD is perpendicular to the plane of u 2 sin 2 30° u 2 sin 2 60°
paper. Required distance = BD H1 = ; H2 =
2g 2g
BD 2 = CB 2 + CD 2 = 500 2 + 1200 2
1/ 4
H1 : H 2 = sin 2 30° : sin 2 60° = =1 : 3
BD = 5002 + 12002 = 1300m 3/ 4
EBD_7100
P-18 Topicwise AIIMS Solved Papers – PHYSICS
14. (c) Net distance travellel along x-direction,
9. (a)
Sx= 6 cos 45°iˆ - 4cos 45°ˆi
1
= 2´ = 2 km
T 2
N
4 km

m
6k
2 °
mv 135
45°
mg r W E
At the lowest point, as shown in the figure
mv2
both mg and centrifugal force will
r S
act in the same direction so, Net distance travelled along y-direction
S = 6sin 45°ˆj + 4sin 45°ˆj
mv 2 y
T = mg + 1
r = 10 ´ = 5 2 km
2
Velocity \ Net distance travelled from the starting
point,
10. (b)
acceleration
S = Sx 2 + S y 2 = ( 2) 2 + (5 2) 2
As the figure implies, velocity acts in
horizontal direction and acceleration due = 2 + 25 ´ 2 = 52 km
to gravity acts in vertical direction. So, Angle which the resultant makes with the
angle between them is 90°. east direction
uur uur y 5 2
11. (d) If P and Q are perpendicular to each tan q = = or q = tan -1 (5)
ur ur x 2
other then P .Q = 0 r r r
15. (c) The resultant F of two forces F1 and F2
ur
(where vector P = aiˆ + ajˆ + 3kˆ and acting in the east and the north direction
respectively will act in the north- east
Q = aî - 2ˆj - k̂ ) direction as per the parallelogram law of
vector addition.
(aiˆ + ajˆ + 3k)(ai
ˆ ˆ - 2jˆ - k)
ˆ =0
16. (b) Here, P = x units, Q = x units, q = 45°
a 2 - 2a - 3 = 0 Þ (a – 3) (a + 1) = 0
R= (2 + 2) units
a – 3 = 0 Þ a = 3 and a = –1
12. (a) For maximum range We have, R = P 2 + Q2 + 2PQ cos q
u2 x 2 + x 2 + 2.x.x cos 45°
R= Þ u 2 = gR or, R =
g
1
or, (2 + 2) = 2x 2 + 2x 2
u 2 = 16,000 ´10 Þ u = 4 ´ 100 2
u = 400m / sec
= 2x 2 + 2x 2
2R
13. (c) t1t 2 = (It is a formula) or, (2 + 2) = x 2 (2 + 2)
g
t 1t 2 µ R or, (2 + 2) = x (2 + 2) Þ x = 1
Motion in a Plane P-19

r r r
u 2 sin 2q 2u 2 sin q.cos q 24. (c) P = vector sum = A + B
17. (b) R = = r r r
g g Q = vector differences = A - B
u 2 sin 2 q r r
H= Since P and Q are perpendicular
2g r r
\ P.Q=0
H u 2 sin 2 q g r r r r
\ = ´
2 Þ (A + B).(A - B) = 0 Þ A2 = B2
R 2g 2u sin q.cos q
sin q Þ A=B
=
4 cos q u 2 sin 2 q
25. (a) h1 =
R 4 cos q R 2g
Þ = or, = 4cot q
H sin q H
u 2 sin 2 (90 - q) u 2 sin 2q
18. (a) The acceleration of a body in a non- h2 = , R=
uniform circular motion is the resultant of 2g g
the radial and the tangential accelerations. Range R is same for angle q and (90° – q)
If
ar = 3 ms–2 and at = 4 ms–2 u 2 sin 2 q u 2 sin 2 (90 - q)
\ h1h 2 = ´
2g 2g
then, a = a r 2 + a t 2 = (3)2 + (4)2 4 2 2
u (sin q) ´ sin (90 - q)
=
= 9 + 16 = 25 = 5 ms–2 4g 2
19. (a) Using the relation for the radius (r) of loop [Q sin(90 - q) = cos q]
2
v u 4 (sin 2 q) ´ cos 2 q
tan q = =
rg
4g 2
(150)2
or tan 12º = [Q sin 2q = 2sin q cos q]
r ´ 10
2250 u 4 (sin q cos q)2 u 4 (sin 2q)2
or r = = 10.6 × 103 m = 10.6 km. = =
0.2125
20. (b) The car moving with a constant velocity 4g 2 16g2
has no acceleration. Hence, it is an inertial (u 2 sin 2q)2 R2
frame. = =
21. (b) For a uniformly accelerated motion there 16g 2 16
are two acceleration, one along the radius
called radial acceleration and another along or, R2 = 16 h1h2 or R = 4 h1h 2
tangent called tangential acceleration.
v 2 sin 2 (90 - q)
Velocity is directed along the tangent. 26. (d) Max. height = H = .....(i)
2g
u 2 sin 2q
22. (d) R = 2 v sin( 90 - q )
g Time of flight, T = ...(ii)
g
(2u)2 sin 2q
R' = = 4R.
g v
2
u
Vertical

23. (a) H = Þ u 2 = 2gH


2g q
For maximum horizontal distance
u 2 2gH
x max = = = 2H
g g Horizontal
EBD_7100
P-20 Topicwise AIIMS Solved Papers – PHYSICS
Type B : Assertion Reason Questions
v cos q 2H
From (i), = ,From (ii),
g g 32. (a) For the distance covered in the last second,
final velocity becomes zero. So if we drop
2H 8H an object with zero velocity it will cover
T= 2 = . the same distance in one second while
g g
going downwards.
27. (a) A parabola Now distance travelled in the later case
28. (a) a r = w2 R
1 2 1
ar = (2p2)2R s = ut + gt = 0 + ´ 10 ´ 1
2 2
2 é 22 ù s = 5m
æ 22 ö
= 4p222R = 4 p 2 çè ÷ (1)
44 ø êëQ v = 44 úû 33. (c) A vehicle moving with constant speed on
a straight road is an inertial frame. Newton’s
dv laws of motion is applicable only in inertial
at = =0
dt frame.
anet = ar = p2 ms–2 and direction along the 34. (c) Suppose that the tennis ball bounces with
radius towards the centre. a velocity u. It will go up, till its velocity
29. (b) Given, u1 = u2 = u, q1 = 60º, q2 = 30º becomes zero. If h is the height up to which
In Ist case, we know that range it rises on the hill, then
u 2 sin 2(60°) u 2 sin 120° u 2 sin( 90° + 30°) (0)2 – u2 = 2(–g')h
R1 = = =
g g g
where g' is acceleration due to gravity on
u 2 (cos 30 ° ) 3u 2 the hill.
= =
g 2g
u2
In IInd case when q 2 = 30° , then \ h=
2g '
u 2 sin 60° u 2 3 Since, the acceleration due to gravity on
R2 = = Þ R1 = R2
g 2g the hill (g') is less than that on earth (effect
(we get same value of ranges). of height), it follows that tennis ball will
30. (c) Yes, the person can catch the ball when bounce higher on hills than in plains.
horizontal velocity is equal to the horizontal 35. (b) In uniform circular motion, the magnitude
component of ball’s velocity, the motion of of velocity and acceleration remains same,
ball will be only in vertical direction w.r.t but due to change in direction of motion,
v the direction of velocity and acceleration
person for that 0 = v 0 cos θ or θ = 60°
2 changes. Also the centripetal acceleration
31. (d) From the figure it is clear that range is is given by a = w2r.
required 36. (d)
u 2 sin 2q (10)2 sin(2 ´ 30°) 37. (a)
R= = =5 3
g 10
Boat 2 v Boat 1
u
v
30° River
Range R V r,g=5m/s
If component of velocities of boat relative
10m

10m

Tower to river is same normal to river flow (as


shown in figure) both boats reach other
bank simultaneously.
Laws of Motion P-21

4 Laws of Motion

TYPE A : MULTIPLE CHOICE QUESTIONS 7. The velocity of a bullet is reduced from 200 m/s
to 100 m/s while travelling through a wooden
1. A molecule of mass m of an ideal gas collides
block of thickness 10 cm. Assuming it to be
with the wall of the vessel with the velocity v
uniform, the retardation will be : [2001]
and returns back with the same velocity. The
change in the linear momentum of the molecule (a) 15´10 4 m / s 2 (b) 10´10 4 m / s 2
will be : [1997]
(a) 4 mv (b) 8 mv (c) 12´10 4 m / s 2 (d) 14.5 m / s 2
(c) 2 mv (d) – 2mv 8. In the given figure, the position-time graph of a
2. If the force on a rocket, moving with a velocity particle of mass 0.1 kg is shown. The impulse at
500 m/s is 400 N, then the rate of combustion of t = 2 sec is : [2005]
the fuel will be : [1997]
(a) 0.8 kg/sec (b) 10.8 kg/sec
(c) 8 kg/sec (d) 1.6 kg/sec 6
3. The rocket engine lift a rocket from the earth, 4
X(m)
because hot gases : [1998] 2
(a) push it against the air with very high
velocity 2 4 6
t (seconds)
(b) push it against the earth with very high
velocity (a) 0.2 kg m sec -1 (b) - 0.2 kg m sec -1
(c) heat up the air which lifts the rocket with
(c) 0.1 kg m sec -1 (d) - 0.4 kg m sec -1
very high velocity
(d) react against rocket and push it up with 9. A person is standing in an elevator. In which
very high velocity situation he finds his weight less ? [2005]
4. A 1 kg particle strikes a wall with a velocity 1 m/s (a) When the elevator moves upward with
at an angle 30° and reflects at the same wall in constant acceleration
0.1 second then the force will be: [1999] (b) When the elevator moves downward with
(a) 30 3 N (b) 0 constant acceleration
(c) When the elevator moves upward with
(c) 40 3 N (d) 10 3 N
uniform velocity
5. A gun fires a bullet of mass 50 g with a velocity
(d) When the elevator moves downward with
of 30 m/s. Due to this, the gun is pushed back
with a velocity of 1 m/s, then the mass of the uniform velocity
gun is : [2001] 10. A person used force (F), shown in the figure to
(a) 1.5 kg (b) 5.5 kg move a load with a constant velocity on a given
surface.
(c) 0.5 kg (d) 3.5 kg
6. When the two surfaces are coated with the
F
lubricant, then they will : [2001]
(a) slide upon each other
(b) stick to each other O X
(c) roll upon each other
(d) none of these L
EBD_7100
P-22 Topicwise AIIMS Solved Papers – PHYSICS
Identify the correct surface profile: [2006] 16. A gardener holds a hosepipe through which
(a) (b) water is gushing out at a rate of 4kg s–1 with
speed 2 ms–1. The moment the speed of water is
increased to 3 ms–1, the gardener will experience
L a jerk of : [2011]
L (a) 20 Ns in backward direction
(c) (d) (b) 18 Ns in forward direction
(c) 10 Ns in backward direction
(d) 10 Ns in forward direction
L
17. A 150 g tennis ball coming at a speed of 40 m/s
L
is hit straight back by a bat to a speed of 60 m/s.
11. A man of mass 60 kg records his wt. on a The magnitude of the average force F on the
weighing machine placed inside a lift. The ratio ball, when it is in contact for 5 ms, is : [2011]
of wts. of man recorded when lift ascending up (a) 2500 N (b) 3000 N
with a uniform speed of 2 m/s to when it is (c) 3500 N (d) 4000 N
descending down with a uniform speed of 4 m/s 18. A particle moves in a circle of radius 25 cm at
will be [2007] two revolutions per second. The acceleration of
(a) 0.5 (b) 1 the particle in meter per second2 is [2012]
(c) 2 (d) None of these (a) p2 (b) 8 p2
12. A smooth block is released at rest on a 45° incline (c) 4 p2 (d) 2 p2
and then slides a distance ‘d’. The time taken to 19. On a smooth plane surface (figure) two block A
slide is ‘n’ times as much to slide on rough incline and B are accelerated up by applying a force 15
than on a smooth incline. The coefficient of N on A. If mass of B is twice that of A, the force
friction is [2008] on B is [2012]
1 1
(a) m k = 1- (b) m k = 1- 15 N
2 B
n n2 A
1 1
(c) m s = 1- (d) m s = 1- (a) 30 N (b) 15 N
n2 n2 (c) 10 N (d) 5 N
13. A parachutist after bailing out falls 50 m without 20. Two pulley arrangements of figure given are
friction. When parachute opens, it decelerates identical. The mass of the rope is negligible. In
at 2 m/s2. He reaches the ground with a speed of fig (a), the mass m is lifted by attaching a mass
3 m/s. At what height, did he bail out ? [2008] 2m to the other end of the rope. In fig (b), m is
(a) 182 m (b) 91 m lifted up by pulling the other end of the rope
(c) 111 m (d) 293 m with a constant downward force F = 2mg. The
14. An 80 kg person is parachuting and is acceleration of m in the two cases are respectively
experiencing a downward acceleration of 2.8 [2013]
m/s2. The mass of the parachute is 5 kg. The
upward force on the open parachute is
(Take g = 9.8 m/s2) [2009]
(a) 595 N (b) 675 N
(c) 456 N (d) 925 N
15. When a horse pulls a wagon, the force that
causes the horse to move forward is the force
(a) the ground exerts on it [2010] m 2m m F = 2 mg
(b) it exerts on the ground (a) (b)
(c) the wagon exerts on it (a) 3g, g (b) g/3, g
(d) it exerts on the wagon (c) g/3, 2g (d) g, g/3
Laws of Motion P-23

21. A 3 kg ball strikes a heavy rigid wall with a speed 25. A constant force F = m2g/2 is applied on the
of 10 m/s at an angle of 60º. It gets reflected block of mass m1 as shown in fig. The string
with the same speed and angle as shown here. and the pulley are light and the surface of the
If the ball is in contact with the wall for 0.20s, table is smooth. The acceleration of m1 is [2015]
what is the average force exerted on the ball by
the wall? [2013] m1
F

60º m2

m2g
60º (a) towards right
2 (m1 + m 2 )

m 2g
(b) towards left
(a) 150 N (b) Zero 2 (m1 - m 2 )

(c) 150 3N (d) 300 N m 2g


(c) towards right
22. A mass is hanging on a spring balance which is 2 (m 2 - m1 )
kept in a lift. The lift ascends. The spring balance
m 2g
will show in its readings [2014] (d) towards left
(a) an increase 2 (m 2 - m1 )
(b) a decrease 26. A smooth block is released at rest on a 45° incline
(c) no change and then slides a distance ‘d’. The time taken to
(d) a change depending on its velocity slide is ‘n’ times as much to slide on rough incline
23. A ball of mass 0.5 kg moving with a velocity of 2 than on a smooth incline. The coefficient of
m/sec strikes a wall normally and bounces back friction is [2016]
with the same speed. If the time of contact
between the ball and the wall is one millisecond, 1 1
(a) m k = 1- 2 (b) m k = 1-
the average force exerted by the wall on the ball n n2
is : [2015]
(a) 2000 newton (b) 1000 newton 1 1
(c) m s = 1- (d) m s = 1-
(c) 5000 newton (d) 125 newton n 2
n2
24. A smooth inclined plane is inclined at an angle q
with horizontal. A body starts from rest and slides 27. A hockey player is moving northward and
down the inclined surface. [2015] suddenly turns westward with the same speed
to avoid an opponent. The force that acts on
the player is [2017]
(a) frictional force along westward
h (b) muscles force along southward
(c) frictional force along south-west
q (d) muscle force along south-west
28. The retarding acceleration of 7.35 ms–2 due to
Then the time taken by it to reach the bottom is
frictional force stops the car of mass 400 kg
æ 2h ö æ 2l ö travelling on a road. The coefficient of friction
(a) çç ÷÷ (b) çç ÷÷ between the tyre of the car and the road is
è g ø è g ø
[2017]
1 2h ( 2h ) (a) 0.55 (b) 0.75
(c) (d) sin q
sin q g g (c) 0.70 (d) 0.65
EBD_7100
P-24 Topicwise AIIMS Solved Papers – PHYSICS

TYPE B : ASSERTION REASON QUESTIONS Directions for (Qs. 34-36) : Each of these questions
contains an Assertion followed by Reason. Read them
Directions for (Qs. 29-33) : These questions consist carefully and answer the question on the basis of
of two statements, each printed as Assertion and following options. You have to select the one that
Reason. While answering these questions, you are best describes the two statements.
required to choose any one of the following five
(a) If both Assertion and Reason are correct and
responses.
Reason is the correct explanation of Assertion.
(a) If both Assertion and Reason are correct and
the Reason is a correct explanation of the (b) If both Assertion and Reason are correct, but
Assertion. Reason is not the correct explanation of
(b) If both Assertion and Reason are correct but Assertion.
Reason is not a correct explanation of the (c) If Assertion is correct but Reason is incorrect.
Assertion. (d) If both the Assertion and Reason are incorrect.
(c) If the Assertion is correct but Reason is incorrect. 34. Assertion : There is a stage when frictional force
(d) If both the Assertion and Reason are incorrect. is not needed at all to provide the necessary
(e) If the Assertion is incorrect but the Reason is centripetal force on a banked road.
correct. Reason : On a banked road, due to its inclination
29. Assertion : On a rainy day it is difficult to drive the vehicle tends to remain inwards without any
a car or bus at high speed. chances of skidding. [2016]
Reason : The value of coefficient of friction is 35. Assertion : Mountain roads rarely go straight
lowered due to wetting of the surface. [1999] up the slope.
30. Assertion : A rocket moves forward by pushing Reason : Slope of mountains are large, therefore
the surrounding air backwards. more chances of vehicle to slip from roads.
Reason : It derives the necessary thrust to move
[2016]
forward according to Newton’s third law of
motion. [2001] 36. Assertion : A man and a block rest on smooth
31. Assertion : The driver in a vehicle moving with horizontal surface. The man holds a rope which
a constant speed on a straight road is in a non- is connected to block. The man cannot move on
inertial frame of reference. the horizontal surface.
Reason : A reference frame in which Newton's
laws of motion are applicable is non-inertial.
[2004]
32. Assertion : Use of ball bearings between two
moving parts of a machine is a common practice. Reason : A man standing at rest on smooth
Reason : Ball bearings reduce vibrations and horizontal surface cannot start walking due to
provide good stability. [2006] absence of friction (The man is only in contact
33. Assertion : Angle of repose is equal to the angle with floor as shown). [2017]
of limiting friction.
Reason : When the body is just at the point of
motion, the force of friction in this stage is called
limiting friction. [2008]
Laws of Motion P-25

Type A : Multiple Choice Questions 6. (a) If two surfaces are coated with lubricant
then friction will be reduced so they can
1. (c) Initial momentum = mv slide over each other if one is pushed on
Final momentum = – mv
the other. It is friction which prevents
Change in momentum = mv – (– mv) = 2mv
2. (a) We know that relative motion between two surfaces.
dp d(mv) dm 7. (a) Using the formula, v 2 = u 2 + 2as
Force = = =v
dt dt dt 10
[when v is constant] (100) 2 = ( 200) 2 - 2a ´
In the given case force = 400 N; 100
v = 500 m/sec. 10
2a ´ = (200) - (100) 2 = 300 ´ 100
2
dm 100
We are required to calculate or rate of
dt 3 ´ 105
change of mass of the rocket. a= = 15 ´104 m / sec 2
2
In normal cases, force creates change in 8. (a) Impulse = mu – mv
momentum. Here force is created due to
change in momentum of the rocket by 4
= 0 .1´ - m ´ 0 [v = 0 after two seconds]
emission of fuel (a part of rocket). 2
= 0.2 kg m sec–1
So, dm = force = 400 = 0.8 kg / sec 9. (b) Person will feel his weight less when the
dt v 500
lift goes down with some acceleration.
3. (b) When the rocket gas pushes it against the
earth with high velocity, there is production 10. (a) In figure no. (a) and (c), a constant force
of reaction force which creates lift for the equal to mg sinq is required. After reaching
rocket. the highest point, in case of figure (c), no
4. (d) force is required but in case of figure (a),
v
body travels on its own. So a –ve force is
v cos 30° acting on the body. In this way, figure (a)
30° represents the given F – x curve.
30° 11. (b) Net force of reaction acts on a body in a lift
when it is accelerating. If lift moves up or
down with uniform speed then acceleration
Change in momentum of the ball a = 0, \ weight of man = mg is same in
perpendicular to the wall ascending or descending hence ratio = 1.
= m[v cos q - (-v cos q)] 12. (b) g sin q - mg cos q
= 2mv cos q
nq
= 2 ´1´1´ cos 30° = 2 ´
3
= 3, g si d
2 45° 45°
Rate of change of momentum smooth rough
Change of momentum 3 1
= = = 10 3
time 0.1 d= (g sin q) t1 ,
2
This will be equal to force, so force = 10 3 N 1
5. (a) Applying conservation of momentum d = (g sin q - mg cos q) t 2
2
MV = mv
50 3 2d
M´l = ´ 30 = t1 = ,
1000 2 g sin q
M = 1.5 kg
EBD_7100
P-26 Topicwise AIIMS Solved Papers – PHYSICS

2d v 2 (rw) 2
t2 = aC = = = rw2 = r ´ (2p / T ) 2
g sin q - mg cos q r r
According to question, t 2 = nt1 so a c = 0.25 ´ (2 p / 0.5) 2

2d 2d = 16 p2 ´ .25 = 4.0 p 2
n = 19. (c) The acceleration of both the blocks =
g sin q g sin q - mg cos q
15 5
m, applicable here, is kinetic friction as the =
block moves over the inclined plane. 3x x
5
1 æ 1 ö \ Force on B = ´ 2x = 10 N
n= ççQ cos 45° = sin 45° = ÷÷ x
1 - mk è 2ø 20. (b) Let a and a' be the accelerations in both
1 1 cases respectively. Then for fig (a),
n2 = or 1 - m k = 2
1- mk n
1
or m k = 1-
n2
13. (d) v = 2gh = 2 ´ 9.8 ´ 50 = 14 5 a
T T a

v2 - u 2 32 - 980
S= = » 243 m
2´ 2 4
Initially he has fallen 50 m. mg 2mg
\ Total height from where (a)
he bailed out = 244 + 50 = 293 m T – mg = ma … (i)
14. (a) The net upward acceleration is and 2mg – T = 2ma … (ii)
(9.8 – 2.8) = 7 m/sec2 Adding (i) and (ii), we get
Total mass = 80 + 5 = 85 kg mg = 3ma
So, net upward force is
F = 85 × 7 = 595 N g
\a=
15. (a) As per Newton’s third law of motion, when 3
a horse pulls a wagon, the force that causes For fig (b),
the horse to move forward is the force the
ground exerts on it.
dm
16. (a) F = (Dv) = 4(2 + 3) = 20 N.
dt
17. (b) The change in momentum a¢ T¢ T¢ a¢
Dp = m(vf – vi)
= 0.150 (60 – (– 40)] F = 2mg
= 0.150 × 100 = 15 Ns mg
Dp 15 (b)
Thus, F = = = 3 ´ 103 N
Dt 5 ´ 10-3 T' – mg = ma' … (iii)
and 2mg – T' = 0 … (iv)
1 Solving (iii) and (iv)
18. (c) Here T = sec the required centripetal
2 a' = g
acceleration for moving in a circle is g
\ a = and a ' = g
3
Laws of Motion P-27

21. (c) Change in momentum along the wall


q
= mv cos60º – mv cos 60º = 0 sin
g

g cosq
Change in momentum perpendicular to the h
wall g
= mv sin60º – (– mv sin60º)
q
= 2mv sin60º
\ Applied force 25. (a) Let a be the acceleration of mass m2 in the
downward direction. Then
Change in momentum
= T – m2 (g/2) = m1 a ....(i)
Time and m2 g – T = m2 a ....(ii)
2 mv sin 60º Adding eqs. (1) and (2), we get
= (m1 + m2) a = m2g – m2 (g/2) = m2 g/2
0.20
m2 g
2 ´ 3 ´ 10 ´ 3 \a=
= = 50 ´ 3 3 2 (m 1 + m 2 )
2 ´ 0.20 g sin q - mg cos q
26. (b)
= 150 3 newton
d q
in
22. (a) Let acceleration of lift = a and gs d
45° 45°
let reaction at spring balance = R smooth rough
R When surface is When surface is
smooth rough
1
d= (g sin q)t 12 ,
2
1 2
d = (g sin q - mg cos q) t 2
2
mg
2d
Applying Newton’s law t1 = ,
g sin q
R – mg = ma Þ R = m(g + a ) 2d
t2 =
thus net weight increases, so reading of g sin q - mg cos q
spring balance increases.
According to question, t 2 = nt1
mv - (- mv ) 2mv 2 ´ 0.5 ´ 2
23. (a) F= = = 2d 2d
t t 10 -3 n =
g sin q g sin q - mg cos q
= 2 × 103 N m, applicable here, is coefficient of kinetic
24. (c) So by second equation of motion, we get friction as the block moves over the
1
inclined plane.
S = ut + at 2 1
2 n=
here S = l, u = 0, a = g sinq 1- mk
æ 1 ö
2l 2h 1 2h ççQ cos 45° = sin 45° = ÷÷
t= = = è 2ø
a g sin q sin q g
2
1 1
n2 = or 1 - m k = 2
1- mk n
æ hö
çèQ sin q = ÷ø 1
l or m k = 1-
n2
EBD_7100
P-28 Topicwise AIIMS Solved Papers – PHYSICS

27. (c) Frictional force is always opposite to the R F


direction of motion
N a
a
sin mg mg cos a
W E mg

S
Angle of repose is defined as the angle of
F
28. (b) As we know, coefficient of friction m = the inclined plane with horizontal such that
N a body placed on it is just begins to slide.
ma a In limiting condition,
Þm= = (a = 7.35 m s–2 given) F = mg sin a and R = mg cos a
mg g
where a–angle of repose.
7.35
\m= = 0.75 F
9.8 So = tan a
R
Type B : Assertion Reason Questions F
\ = ms = tan q = tan a (Q tan q = ms)
29. (a) On a rainy day, the roads are wet. Wetting R
of roads lowers the coefficient of friction or q = a
between the types and the road. Therefore, i.e., angle of friciton = angle of repose.
grip on a road of car reduces and thus 34. (c) The assertion is true for a reason that when
chances of skidding increases. the car is driven at optimum speed. Then
30. (a) A rocket moves forward taking the help of the normal reaction component is enough
reaction force. For that it has to exert a force to provide the centripetal force.
on the surrounding air so that it receives 35. (a) Both Assertion and Reason are true and
reaction force as per Newton’s third law. Reason is the correct explanation of
31. (d) A vehicle moving with constant speed on Assertion.
a straight road is an inertial frame. Newton’s If roads of the mountain were to go straight
laws of motion is applicable only in inertial up, the slope (q) would have been large,
frame. the frictional force (mmg cos q) would be
32. (c) Ball bearing are used to convert sliding small. Due to small friction, wheels of
friction to rolling friction. Sliding friction is vehicle would slip. Also for going up a large
less than rolling friction. slope, a greater power shall be required.
33. (b) The maximum value of static friction up to 36. (d) The man can exert force on block by pulling
which body does not move is called limiting the rope. The tension in rope will make the
friction. man move. Hence Assertion is incorrect.
5 Work, Energy and Power

TYPE A : MULTIPLE CHOICE QUESTIONS 7. Two bodies of masses m and 4m are moving
with equal kinetic energy. Then the ratio of their
1. If a spring extends by x on loading, then energy
linear momentum will be : [1999]
stored by the spring is (if T is the tension in
(a) 1 : 1 (b) 2 : 1
spring and k is spring constant) [1997]
(c) 4 : 1 (d) 1 : 2
2T 2 8. A particle of mass m moving with velocity v
(a) (b)
k collides with a stationary particle of mass 2m.
T2 T2 Then the speed of the system after collision is :
(c) (d) v
2k k (a) 2 v (b) [1999]
2. A bullet is fired form a riffle. If the riffle recoils 2
v
freely, then the kinetic energy of the rifle will be : (c) 3 v (d)
3
(a) same as that of bullet [1998]
9. Which one of the following is true ? [2000]
(b) more than that of bullet
(a) momentum is conserved in all collisions but
(c) less than that of bullet
kinetic energy is conserved in elastic
(d) none of these
collisions
3. A spring 40 mm long is stretched by applying a
(b) momentum is conserved in all collisions but
force. If 10 N force is required to stretch the
not kinetic energy
spring through one mm, then work done in
(c) both momentum and kinetic energy are
stretching the spring through 40 mm is : [1998]
conserved in all collisions
(a) 24 J (b) 8 J
(d) neither momentum nor kinetic energy is
(c) 56 J (d) 64 J
conserved in elastic collisions
4. If the kinetic energy of the body becomes four
10. A ball of mass 10 kg is moving with a velocity of
times of its initial value, then the new momentum
10 m/s. It strikes another ball of mass 5 kg, which
will : [1998]
is moving in the same direction with a velocity
(a) become twice its initial value
of 4 m/s. If the collision is elastic their velocities
(b) remain constant
after collision will be respectively : [2000]
(c) become four times its initial value
(a) 12 m/s, 6 m/s (b) 12 m/s, 25 m/s
(d) become three times its initial value
5. If the water falls from a dam into a turbine wheel (c) 6 m/s, 12 m/s (d) 8 m/s, 20 m/s
19.6 m below (which have both KE + PE), then 11. A bullet of mass 10g leaves a riffle at an initial
the velocity of water at the turbine is : (take g = velocity of 1000 m/sec and strikes the earth at
9.8 m/sec2) [1998] the same level with a velocity of 500 m/sec. The
work in overcoming the resistance of air will be:
(a) 19.6 m/s (b) 39.0 m/s
(c) 98.8 m/s (d) 9.8 m/s (a) 500 J (b) 5000 J [2000]
6. If the force applied is F and the velocity gained (c) 3750 J (d) 475 J
is v, then the power developed is : [1998] 12. A body of mass 5 kg has momentum of 10 kg m/
v sec. When a force of 0.2 N is applied on it for 10
(a) (b) Fv sec, the change in its kinetic energy is [2000]
F
F (a) 4.4 J (b) 3.3 J
(c) Fv 2 (d) (c) 5.5 J (d) 1.1 J
v
EBD_7100
P-30 Topicwise AIIMS Solved Papers – PHYSICS
13. A metal ball of mass 2 kg moving with speed of 20. A particle of mass 10 g is kept on the surface of
36 km/hr is having a collision with a stationary a uniform sphere of mass 100 kg and radius
ball of mass 3 kg. If after collision, both the balls 10 cm. Find the work to be done against the
move together, the loss in kinetic energy due to gravitational force between them to take the
collision is : [2001] particle far away from the sphere (you may take
G = 6.67 × 10–11 Nm2 / kg2) [2008]
(a) 80 J (b) 40 J –10
(a) 3.33 × 10 J
(c) 60 J (d) 160 J
(b) 13.34 × 10–10 J
14. A force (3iˆ + 4j)
ˆ newton acts on a body and
(c) 6.67 × 10–10 J
displaced it by (3iˆ + 4j)
ˆ metre. The work done
(d) 6.67 × 10–9 J
by the force is : [2001] 21. A vertical spring with force constant k is fixed
(a) 5 J (b) 25 J on a table. A ball of mass m at a height h above
(c) 10 J (d) 30 J the free upper end of the spring falls vertically
15. The kinetic energy of a body becomes four times on the spring so that the spring is compressed
its initial value. The new linear momentum will be by a distance d. The net work done in the
(a) eight times of the initial value [2002] process is [2008]
(b) four times of the initial value 1
(a) mg(h + d) - kd 2
(c) twice of the initial value 2
(d) remain as the initial value 1
(b) mg(h - d) - kd 2
16. A block of mass 10 kg is moving in x-direction 2
with a constant speed of 10 m/sec. It is subjected 1
to a retarding force F = – 0.1x joule/metre during (c) mg(h - d) + kd 2
2
its travel from x = 20 metre to x = 30 metre. Its
1
final kinetic energy will be [2005] (d) mg(h + d) + kd2
2
(a) 475 joule (b) 450 joule
22. A shell of mass m moving with velocity v
(c) 275 joule (d) 250 joule suddenly breaks into 2 pieces. The part having
17. When a ball is thrown up vertically with velocity mass m/3 remains stationary. The velocity of
v0 it reaches a maximum height of h. If one wishes other part will be [2009]
to triple the maximum height then the ball should 2 7
be thrown with velocity : [2005] (a) v (b) v
3 5
(a) 3 v0 (b) 3v0 3
(c) v (d) none of these
(c) 9v0 (d) 3/2v0 2
23. A mass of 1 kg is hanging from a spring of spring
18. For inelastic collision between two spherical
constant 1 N/m. If Saroj pulls the mass down by
rigid bodies: [2006] 2m. The work done by Saroj is [2009]
(a) the total kinetic energy is conserved (a) 1 J (b) 2 J
(b) the total potential energy is conserved (c) 3 J (d) 4 J
(c) the linear momentum is not conserved 24. A ball loses 15.0% of its kinetic energy when it
(d) the linear momentum is conserved bounces back from a concrete wall. With what
speed you must throw it vertically down from a
19. If the water falls from a dam into a turbine wheel
height of 12.4 m to have it bounce back to the
19.6 m below, then the velocity of water at the
same height (ignore air resistance)? [2010]
turbines, is (take g = 9.8 m/s2) [2007]
(a) 6.55 m/s (b) 12.0 m/s
(a) 9.8 m/s (b) 19.6 m/s
(c) 8.6 m/s (d) 4.55 m/s
(c) 39.2 m/s (d) 98.0 m/s
Work, Energy and Power P-31

25. Two bodies of masses 0.1 kg and 0.4 kg move (a) 250 J (b) 200 J
towards each other with the velocities 1 m/s and (c) 150 J (d) 10 J
0.1 m/s respectively. After collision they stick 31. Figure shows three forces applied to a trunk that
together. In 10 sec the combined mass travels moves leftward by 3 m over a smooth floor. The
(a) 120 m (b) 0.12 m [2010] force magnitudes are F1 = 5N, F2 = 9N, and F3 =
(c) 12 m (d) 1.2 m 3N. The net work done on the trunk by the three
26. The potential energy of a certain particle is given forces [2017]
1 2
by U = (x - z 2 ). The force on it is : [2011]
2
(a) - xiˆ + zkˆ (b) xiˆ + zkˆ
1 ˆ ˆ 1 ˆ ˆ
(c) (xi + zk) (d) (xi - zk)
2 2
27. From a building two balls A and B are thrown (a) 1.50 J (b) 2.40 J
such that A is thrown upwards and B downwards (c) 3.00 J (d) 6.00 J
(both vertically). If vA and vB are their respective
velocities on reaching the ground, then[2012] TYPE B : ASSERTION REASON QUESTIONS
(a) vA > vB Directions for (Qs. 32-34) : These questions consist
(b) vA = vB of two statements, each printed as Assertion and
(c) vA < vB Reason. While answering these questions, you are
(d) their velocities depend on their masses. required to choose any one of the following five
28. A motor drives a body along a straight line with responses.
a constant force. The power P developed by the (a) If both Assertion and Reason are correct and
motor must vary with time t according to [2012] the Reason is a correct explanation of the
(a) (b) P Assertion.
P (b) If both Assertion and Reason are correct but
Reason is not a correct explanation of the
t t Assertion.
(c) If the Assertion is correct but Reason is
P P incorrect.
(c) (d) (d) If both the Assertion and Reason are incorrect.
t t (e) If the Assertion is incorrect but the Reason is
29. If the linear momentum is increased by 5%, the correct.
kinetic energy will increase by [2013, 2014] 32. Assertion : In an elastic collision of two billiard
(a) 50% (b) 100% balls, the total kinetic energy is conserved during
(c) 125% (d) 10% the short time of oscillation of the balls (i.e.,
30. Figure here shows the frictional force versus when they are in contact).
displacement for a particle in motion. The loss Reason : Energy spent against friction does not
of kinetic energy in travelling over s = 0 to 20 m follow the law of conservation of energy. [2002]
will be [2015] 33. Assertion : Frictional forces are conservative
f(N) forces.
15
Reason : Potential energy can be associated with
frictional forces. [2005]
10 34. Assertion : A quick collision between two
bodies is more violent than a slow collision; even
5 when the initial and final velocities are identical.
Reason : The momentum is greater in first case.
0 x(m)
0 5 10 20 [2008]
EBD_7100
P-32 Topicwise AIIMS Solved Papers – PHYSICS
Directions for (Qs. 35-40) : Each of these questions 37. Assertion : If collision occurs between two
contains an Assertion followed by Reason. Read them elastic bodies their kinetic energy decreases
carefully and answer the question on the basis of during the time of collision.
following options. You have to select the one that Reason : During collision intermolecular space
best describes the two statements. decreases and hence elastic potential energy
(a) If both Assertion and Reason are correct and increases. [2011]
38. Assertion : The total translational kinetic energy
Reason is the correct explanation of Assertion.
of all the molecules of a given mass of an ideal
(b) If both Assertion and Reason are correct, but gas is 1.5 times the product of its pressure and
Reason is not the correct explanation of its volume.
Assertion. Reason : The molecules of a gas collide with
(c) If Assertion is correct but Reason is incorrect. each other and the velocities of the molecules
(d) If both the Assertion and Reason are incorrect. change due to the collision. [2015]
35. Assertion : n small balls each of mass m colliding 39. Assertion : If collision occurs between two
elastically each second on surface with velocity elastic bodies their kinetic energy decreases
during the time of collision.
u. The force experienced by the surface is 2mnu.
Reason : During collision intermolecular space
Reason : On elastic collision, the ball rebounds decreases and hence elastic potential energy
with the same velocity. [2010] increases. [2015]
36. Assertion : A helicopter must necessarily have 40. Assertion : Graph between potential energy of
two propellers. a spring versus the extension or compression
Reason : Two propellers are provided in of the spring is a straight line.
helicopter in order to conserve linear momentum. Reason : Potential energy of a stretched or
[2010] compressed spring, proportional to square of
extension or compression. [2017]
Work, Energy and Power P-33

Type A : Multiple Choice Questions p12 2 ´ 4 m E


´ = =1
1 2 2m p 22 E
1. (c) Energy of spring = kx
2 p 1 p 1
12
T =Þ 1 = Þ p1 : p 2 = 1 : 2
T = kx Þ x = p 22 4 p2 2
k 8. (d) Applying conservation of momentum,
2
1 æ Tö 1 T2 mv + 0 = (2m + m)v ' = 3mv'
E = kç ÷ =
2 è kø 2 k
mv v
2. (c) For recoil of riffle, momentum will be v' = =
conserved 3m 3
9. (a) Kinetic energy is not conserved in inelastic
MV = mv collision as some energy is stored as
1 deformation at the point of collision in the
MV 2 form of potential energy. Since no
K.E of riffle 2
= deformation occurs in case of elastic
K.E of bullet 1
mv2 collision so, kinetic energy is conserved.
2
2
But momentum is conserved in both elastic
M æmö m and inelastic collisions as in both the cases,
=
´ç ÷ =
m èMø M no external force is applied on them so, no
As m < M, kinetic energy of riffle < kinetic change in momentum.
energy of bullet. 10. (c) Let their velocities after the collision be v1
3. (b) Force constant, and v2. As we know for elastic collision.
Relative velocity of approach
F 10
k= = = 104 N / m = relative velocity of separation
x 0.001
2
10 – 4 = v2 – v1 Þ 6 = v2 – v1
1 2 1 æ 40 ö Þ v1 = v2 – 6
Work done = kx = ´ 104 ´ ç
2 2 è 1000 ÷ø
Applying conservation of momentum,
1 16 10 ´ 10 + 5 ´ 4 = 10v1 + 5v2
= ´ 104 ´ 4 = 8 joule
2 10 120 = 10v1 + 5v2
p2 120 = 10(v 2 - 6) + 5v2 = 15v 2 - 60
4. (a) E= ; E µ p2
2m
15v2 = 180 Þ v2 = 12 cm/sec,
2
E1 p12 1 æ p1 ö p 1 v1 = 6 cm/sec
= Þ =ç ÷ Þ 1 =
E 2 p2 2 4 è p2 ø p2 2 11. (c) Loss of kinetic energy of bullet
ratio of momentum = 1 : 2 = The work done in over coming air
resistance.
5. (a) v 2 = 2gh = 2 ´ 9.8 ´ 19.6
1 10
= 19.6 m / sec = ´ (10002 - 5002 )
2 1000
Work F.s S 1 1
6. (b) Power = = = F. = F.v. = ´ ´ 1500 ´ 500 = 3750J
Time t t 2 100
7. (d) Relation between momentum and energy is 12. (a) Change in momentum,
p2 p2 p 2 Dp = F.t = 0.2 ´ 10 = 2
E= ; E= 1 ; E= 2 ;
2m 2m 2 ´ 4m
EBD_7100
P-34 Topicwise AIIMS Solved Papers – PHYSICS
10 Now, initial kinetic energy
Initial value of velocity = = 2 m / sec
5 1
= ´10 ´100 = 500 joule
1 2
Initial energy = ´ 5 ´ 2 ´ 2 = 10 J
2 Final kinetic energy = 500 – 25 = 475 joule
Total final momentum = 10 + 2 [Direction of force is opposite to direction
= 12 kg m/sec of motion]
12 1
Final velocity = m/sec 17. (a) mv 0 2 = mgh ................ (i)
5 2
1 12 12 1
Final energy = ´ 5 ´ ´ mv 2 = mg ´ 3h ................ (ii)
2 5 5 2
72 2
= = 14.4J Dividing v = 3; v 2 = 3v 0 2
5 v0
Change in energy = 14.4 – 10 = 4.4 joule
13. (c) Let v be the common velocity. Þ v = 3v 0
Applying conservation of momentum 18. (d) In inelastic collision, linear momentum is
conserved.
2 ´ 10 + 3 ´ 0 = (2 + 3)v 19. (b) v2 – u2 = 2as Þ v2 – 02 = 2 × 9.8 × 19.6
10 ´ 2 (Initial velocity = 0 as water falls from rest)
v= = 4 m / sec
5
Þ v= 2 ´ 9.8 ´ 2 ´ 9.8
[36 km/hour = 10m/sec]
= 2 × 9.8 = 19.6 m/s
1
Initial Energy = ´ 2 ´ (10) 2 + 0 = 100 J GMm
2 20. (c) W=
1 R
Final Energy = ´ 5 ´ 4 ´ 4 = 40 J 6.67 ´10 -11 ´ 100 10
2
W = ´
Loss of energy = 100 – 40 = 60 joule 0.1 1000
uur ur = 6.67 × 10–10 J
14. (b) Work done = F . s
21. (a) Gravitational potential energy of ball gets
= (3iˆ + 4j).(3i
ˆ ˆ + 4j)
ˆ = 9 + 16 = 25 joule
converted into elastic potential energy of
15. (c) The relation between kinetic energy and the spring.
linear momentum is 1 2
2 mg(h + d) = kd
p 2
E= so E µ p 2
2m 1 2
Net work done = mg(h + d) - kd = 0
If energy becomes four times th en 2
momentum will become twice as
pµ E h
16. (a) We know that,
d
Change in kinetic energy
= work done on the object by force
30
Here, work done = ò Fdx = ò 0.1x dx
20 22. (c) m v m/3 m – m/3 = 2m/3
30 v =0 v' = ?
é x2 ù 0.1 2
= 0.1 ê ú = [30 - 202 ] According to momentum conservation.
2
ëê ûú 20 2
2m
= 0.05[900 – 400] = 0.05 × 500 = 25 joule mv = m / 3 ´ 0 + v'
3
Work, Energy and Power P-35

3 \ 0.1 × 1 + 0.4 × (–0.1) = (0.1 + 0.4) v


2m
mv = v' v' = v or 01. – 0.04 = 0.5 v,
3 2
0.06
2 v= = 0.12 m/s
2 2
1 ù 0.5
23. (b) Workdone = ò F.dx = ò kx dx = kx 2 ú Hence, distance covered = 0.12 × 10 = 1.2 m.
2 úû
0 0 0
-dU -d æ x 2 - z2 ö
1 26. (a) Fx = =x ç ÷ = -x
= .1.(4 - 0) = 2 J dx dx è 2 ø
2
24. (a) Given: h = 12.4, v = ?
\ v2 = u2 + 2gh -dU -d æ x 2 - z 2 ö
FZ = = ç ÷ =z
i.e., v2 = u2 + 2×9.8×12.4 dt dz è 2 ø
= u2 + 243.04 r
Kinetic energy of the ball when it just hits
\ F = - xiˆ + zkˆ
the wall 27. (b) As the ball moves down from height ‘h’ to
ground the P.E. at height ‘h’ is converted
1 2 1 to K.E. at the ground (Applying Law of
= mv = m (u 2 + 243.04)
2 2 conservation of Energy).
The K.E. of ball after the impact 1
Hence, m v 2 = m Agh A or v A =
(100 - 15) 1 2 A A
= ´ m (u 2 + 243.04)
100 2 2gh ;
85 1
= ´ m (u 2 + 243.04) Similarly, vB = 2 gh or vA = vB
100 2 28. (d) P = F´ v Þ P = F a t
Let v2 be the upward velocity just after the
\ Pµt
collision with the ground.
1 2 85 1
p2
´ m (u 2 + 243.04) 29. (d) As E =
So, mv2 = 2m
2 100 2
85 2 dE æ dp ö
v22 = (u + 243.04) \ = 2 ç ÷ = 2 ´ 5% = 10%
100 E è pø
Now, taking upward motion 30. (a) Loss in K.E = Area under the curve
v = 0, u = v2 r
31. (a) F = –5iˆ + 9cos 60°iˆ + 9sin 60° ˆj - 3 ˆj
\ v2= u2 –2gh 9 9 3ˆ
85 2 = –5iˆ + iˆ + j - 3 ˆj
0= (u + 243.04) - 2 ´ 9.8 ´ 12.4 2 2
100 ˆ
i æ9 3 ö
= - 2 + çè - 3÷ ˆj
85 2 2 ø
u = 36.46 r
100 s = –3i .
ˆ
36.46 ´ 100 é iˆ æ9 3 ö ù
u2 = = 42.89 r
85 W = F .sr = ê - 2 + çè - 3÷ ˆj ú .( -3iˆ)
ø û
ë 2
u = 6.55 m/s = 1.5 J.
25. (d) According to conservation of momentum
m1v1 + m2v2= (m1 + m2) v, Type B : Assertion Reason Questions
where v is common velocity of the two 32. (d) In an elastic collision, no conversion of
bodies. energy, so K.E. remains constant during
m1 = 0.1 kg m2 = 0.4 kg the time of collision. There is no friction
v1 = 1 m/s, v2 = – 0.1 m/s, acting in this case. In case of friction too
EBD_7100
P-36 Topicwise AIIMS Solved Papers – PHYSICS
conservation of energy is followed 36. (c) If there were only one propeller in the
provided we take into account all the helicopter, the helicopter itself, would have
transformations there. turned in opposite direction of the direction
33. (d) Frictional force is non-conservative as of propeller due to conservation of angular
work done against frictional force can not momentum. Thus two propeller provides
be stored as potential energy. helicopter a steady movement.
34. (a) In a quick collision, time t is small. 37. (a)
As F × t = constant, therfore, force involved 38. (b) Total translational kinetic energy
is large. I.e., collision is more violent in 3 3
comparison to slow collision . = nRT = PV
2 2
Momentum, p = mv or p µ v In an ideal gas all molecules moving
i.e., momentum is directly proportional to randomly in all direction collide and their
its velocity, so the momentum is greater in velocity changes after collision.
a quicker collision. 39. (a)
35. (a) In elastic collision, kinetic energy remains 1 2
conserved therefore the ball rebounds with 40. (d) Potential energy U = kx i.e. U µ x 2
2
the same velocity. According to Newton’s This is a equation of parabola, so graph
second law between U and x is a parabola not a
F× t = change in linear momentum. straight line.
\ F × 1 = m× n (u + u) Þ F = 2mnu.
System of Particles and Rotational Motion P-37

6
System of Particles and
Rotational Motion
TYPE A : MULTIPLE CHOICE QUESTIONS 7. In an orbital motion, the angular momentum
vector is : [2004]
1. If there is change of angular momentum from 1J (a) along the radius vector
to 5 J in 5 second. Then the torque is : [1997] (b) parallel to the linear momentum
3J 4J (c) in the orbital plane
(a) (b) (d) perpendicular to the orbital plane
5 5
8. A horizontal platform is rotating with uniform
5J angular velocity around the vertical axis passing
(c) (d) none of these
4 through its centre. At some instant of time a
2. If the equation for the displacement of a particle viscous fluid of mass m is dropped at the centre
moving on a circular path is given as and is allowed to spread out and finally fall. The
q = 2 t 3 + 0.5 , angular velocity during this period : [2005]
where q is in radians and t is in second. Then (a) decreases continuously
the angular velocity of the particle after two (b) decreases initially and increases again
second will be : [1998] (c) remains unaltered
(a) 36 rad/sec (b) 8 rad/sec (d) increases continuously
(c) 48 rad/sec (d) 24 rad/sec 9. A solid sphere is rolling on a frictionless surface,
3. The moment of inertia of a regular circular disc shown in figure with a translational velocity v
of mass 0.4 kg and radius 100 cm about an axis m/s. If it is to climb the inclined surface then v
perpendicular to the plane of the disc and should be : [2005]
passing through its centre is: [1999]
v h
(a) 0.2 kg m2 (b) 0.02 kg m2
(c) 0.002 kg m2 (d) 2 kg m2 10
(a) ³ gh (b) ³ 2gh
4. A constant torque of 31.4 Nm is exerted on a 7
pivoted wheel. If the angular acceleration of the 10
(c) 2gh (d) gh
wheel is 4 p rad/sec2, then the moment of inertia 7
will be : [2001] 10. A ladder is leaned against a smooth wall and it
(a) 5.8 kg-m2 (b) 4.5 kg-m2 is allowed to slip on a frictionless floor. Which
(c) 5.6 kg-m2 (d) 2.5 kg-m2 figure represents the track of its centre of mass?
5. The motion of planets in the solar system is an [2005]
example of the conservation of : [2003] (a) (b)
(a) mass
(b) linear momentum
(c) angular momentum
(d) energy
time time
6. The direction of the angular velocity vector
along : (c) (d)
(a) the tangent to the circular path [2004]
(b) the inward radius
(c) the outward radius
(d) the axis of rotation
time time
EBD_7100
P-38 Topicwise AIIMS Solved Papers – PHYSICS

11. The moment of inertia of a rod about an axis 14. In the diagram shown below all three rods are of
through its centre and perpendicular to it is equal length L and equal mass M. The system is
1 rotated such that rod B is the axis. What is the
ML2 (where M is the mass and L, the length moment of inertia of the system? [2007]
12
of the rod). The rod is bent in the middle so that A
the two halts make an angle of 60°. The moment B
of inertia of the bent rod about the same axis
would be : [2006] C
2 4
ML
1 1 (a) (b) ML2
(a) ML2 (b) ML2 6 3
48 12
ML2 2
1 ML2 (c) (d) ML2
(c) ML2 (d) 3 3
24 8 3 15. The direction of the angular velocity vector is
12. If a street light of mass M is suspended from the along [2007]
end of a uniform rod of length L in different (a) the tangent to the circular path
possible patterns as shown in figure, then : (b) the inward radius
[2006] (c) the outward radius
Cable (d) the axis of rotation
Cable 16. Four point masses, each of value m, are placed
at the corners of a square ABCD of side l. The
moment of inertia of this system about an axis
passing through A and parallel to BD is
3L
4 (a) 2ml2 (b) 3ml 2 [2008]
(c) 3ml 2 (d) ml2
A B 17. For the given uniform square lamina ABCD,
whose centre is O, [2008]
Cable D F C
(a) IAC < 2 IEF

1 (b) 2IAC < IEF O


2 (c) IAD < 3IEF

C (d) IAC < IEF A E


B

(a) Pattern A is more sturdy 18. A wheel has angular acceleration of 3.0 rad/s2
(b) Pattern B is more sturdy and an initial angular speed of 2.00 rad/s. In a
time of 2 s it has rotated through an angle (in
(c) Pattern C is more sturdy
radian) of [2008]
(d) All will have same sturdiness (a) 6 (b) 10
13. If a solid sphere of mass 1 kg and radius 0.1 m (c) 12 (d) 4
rolls without slipping at a uniform velocity of 1 19. The angular speed of a body changes from w1
m/s along a straight line on a horizontal floor, to w2 without applying a torque but due to
the kinetic energy is [2007] changes in moment of inertia. The ratio of radii
7 2 of gyration in two cases is [2009]
(a) J (b) J
5 5 (a) w2 : w1 (b) w2 : w1
7
(c) J (d) 1 J (c) w22 : w12 (d) w32 : w13
10
System of Particles and Rotational Motion P-39

20. A disc is rolling without slipping on a straight


surface. The ratio of its translational kinetic Q
energy to its total kinetic energy is [2009] C
P
2 1
(a) (b)
3 3
(a) VQ > VC > VP
2 3 (b) VQ < VC < VP
(c) (d)
5 5 1
(c) VQ = VP , VC = VP
21. Two particles of masses m1 and m2 (m1 > m2), 2
initially at rest, move towards each other under (d) VQ = VC = VP
an inverse square law force of attraction. Pick 26. For the given uniform square lamina ABCD,
out the correct statement about the centre of whose centre is O, [2012]
mass (CM) of the system [2009]
D F C
(a) The CM moves towards m1
(b) The CM moves towards m2
(c) The CM remains at rest
O
(d) The motion of CM is accelerated
22. A wire of mass m and length l is bent in the form
of a circular ring, the moment of inertia of the A B
E
ring about its axis is [2010]
(a) IAC < 2 IEF (b) 2IAC < IEF
æ 1 ö 2 æ 1 ö
(a) ç 8p2 ÷ ml (b) ç 2 ÷ ml 2 (c) IAD < 3IEF (d) IAC < IEF
è ø è 2p ø
27. One quarter sector is cut from a uniform circular
disc of radius R. This sector has mass M. It is
æ 1 ö 2
(c) ç 4p2 ÷ ml (d) ml 2 made to rotate about a line perpendicular to its
è ø plane and passing through the centre of the
23. Which of the following is true about the angular original disc. Its moment of inertia about the axis
momentum of a cylinder down a slope without of rotation is [2013]
slipping : [2011] 1 1
(a) its magnitude changes but the direction (a) mR 2 (b) mR 2
2 4
remains same
1
(b) both magnitude and direction change (c) mR 2 (d) 2mR 2
8
(c) only the direction change
28. A particle is confined to rotate in a circular path
(d) neither change
decreasing linear speed, then which of the
24. A circular disc rotating with frequency f0 = 1.3 following is correct? [2013]
rev/sec comes to a stop in 30 seconds. The r
(a) L (angular momentum) is conserved about
approximate angular acceleration is : [2011]
the centre.
(a) + 0.27 rad/sec2 (b) – 0.27 rad/sec2 r
(b) Only direction of angular momentum L is
(c) + 0.54 rad/sec2 (d) + 0.27 rad/sec2 conserved.
25. A disc is rolling (without slipping) on a (c) It spirals towards the centre.
horizontal surface.C is its centre and Q and P are (d) Its acceleration is towards the centre.
two points equidistant from C. Let Vp, Vq and Vc 29. A flywheel rotates about an axis. Due to friction
be the magnitude of velocities of points P, Q at the axis, it experiences an angular retardation
and C respectively, then [2012] proportional to its angular velocity. If its angular
EBD_7100
P-40 Topicwise AIIMS Solved Papers – PHYSICS
velocity falls to half while it makes n rotations, TYPE B : ASSERTION REASON QUESTIONS
how many more rotations will it make before
coming to rest? [2013] Directions for (Qs. 34-39) : These questions consist
(a) 2n (b) n of two statements, each printed as Assertion and
(c) n/2 (d) n/3 Reason. While answering these questions, you are
30. A wheel is rolling straight on ground without required to choose any one of the following five
slipping. If the axis of the wheel has speed v, the responses.
instantenous velocity of a point P on the rim, (a) If both Assertion and Reason are correct and
defined by angle q, relative to the ground will the Reason is a correct explanation of the
be [2014] Assertion.
(b) If both Assertion and Reason are correct but
æ1 ö Reason is not a correct explanation of the
(a) v cos ç q ÷ P
è2 ø q Assertion.
(c) If the Assertion is correct but Reason is incorrect.
æ1 ö
(b) 2 v cos ç q ÷ (d) If both the Assertion and Reason are incorrect.
è2 ø (e) If the Assertion is incorrect but the Reason is
(c) v(1 + sin q) correct.
(d) v(1 + cos q) 34. Assertion : Moment of inertia depends on the
31. Consider a thin uniform square sheet made of a axis of rotation and the nature of distribution of
rigid material. If its side is ‘a’ mass m and moment the mass of the body.
of inertia I about one of its diagonals, then Reason : Moment of inertia is the rotational
[2016] inertia of the body. [1997]
35. Assertion : The earth is slowing down and as a
ma 2 ma 2 ma 2 result the moon is coming nearer to it.
(a) I> (b) <I<
12 24 12 Reason : The angular momentum of the earth
2 2 moon system is not conserved. [2003]
ma ma
(c) I = (d) I = 36. Assertion : There are very small sporadic
24 12
32. A small particle of mass m is projected at an angle changes in the speed of rotation of the earth
q with the x-axis with an initial velocity n0 in the Reason : Shifting of large air masses in the
x-y plane as shown in the figure. At a time earth's atmosphere produce a change in the
moment of inertia of the earth causing its speed
n0 sin q of rotation to change. [2004]
t< , the angular momentum of the
g 37. Assertion : For a system of particles under
particle is [2016] central force field, the total angular momentum
2
(a) - mg n 0 t cos q ˆj y is conserved.
ˆ Reason : The torque acting on such a system is
(b) mg n 0t cos q k v0
zero. [2005]
1
(c) - mg n 0 t cos q kˆ
2
38. Assertion : A judo fighter in order to throw his
2 opponent on to the mat tries to initially bend his
1 q opponent and then rotate him around his hip.
(d) mgn0t 2 cos q iˆ
2 x Reason : As the mass of the opponent is brought
33. A solid sphere rolls down two different inclined closer to the fighter’s hip, the force required to
planes of same height, but of different throw the opponent is reduced. [2006]
inclinations. In both cases [2017] 39. Assertion : The velocity of a body at the bottom
(a) speed and time of descent will be same of an inclined plane of given height is more when
(b) speed will be same, but time of descent it slides down the plane, compared to, when it
will be different rolling down the same plane.
(c) speed will be different, but time of descent Reason : In rolling down a body acquires both,
will be same kinetic energy of translation and rotation.
(d) speed and time of descent both are [2008]
different
System of Particles and Rotational Motion P-41

Directions for (Qs. 40-45) : Each of these questions Reason : If net external force is zero, then
contains an Assertion followed by Reason. Read them the linear momentum of the system changes.
carefully and answer the question on the basis of [2011]
following options. You have to select the one that 43. Assertion: A rigid disc rolls without slipping on
best describes the two statements. a fixed rough horizontal surface with uniform
angular velocity. Then the acceleration of lowest
(a) If both Assertion and Reason are correct and point on the disc is zero.
Reason is the correct explanation of Assertion. Reason : For a rigid disc rolling without slipping
(b) If both Assertion and Reason are correct, but on a fixed rough horizontal surface, the velocity
Reason is not the correct explanation of of the lowest point on the disc is always zero.
Assertion. [2013]
(c) If Assertion is correct but Reason is incorrect. 44. Assertion : For the planets orbiting around the
(d) If both the Assertion and Reason are incorrect. sun, angular speed, linear speed and K.E.
40. Assertion : The position of centre of mass of a changes with time, but angular momentum
body depends upon shape and size of the body. remains constant.
Reason : Centre of mass of a body lies always at Reason : No torque is acting on the rotating
the centre of the body. [2009] planet. So its angular momentum is constant.
41. Assertion : If polar ice melts, days will be shorter. [2013]
Reason : Moment of inertia decreases and thus 45. Assertion : Radius of gyration of body is a
angular velocity increases. [2010] constant quantity.
Reason : The radius of gyration of a body
42. Assertion : If no external force acts on a system
about an axis of rotation may be defined as the
of particles, then the centre of mass will not move root mean square distance of the particle from
in any direction. the axis of rotation. [2017]
EBD_7100
P-42 Topicwise AIIMS Solved Papers – PHYSICS

Type A : Multiple Choice Questions 8. (b) When the fluid spreads out, the moment of
inertia of the system is increased. If we apply
dL DL conservation of angular momentum.
1. (b) Torque = = ,
dt Dt Iw = I1w1
Here DL = 5J - J = 4J As I increases due to water spreading out,
4 the angular velocity decrease.
Dt = 5 sec torque = J When water level falls, I decreases resulting
5
in increased angular velocity.
2. (d) q = 2 t 3 + 0. 5 9. (a) Applying law of conservation of energy
dq for rotating body,
w= = 6t 2 = 6 ´ 22
dt 1 1
= 6 ´ 4 = 24 radian / sec mv 2 + Iw2 = mgh
2 2
3. (a) Moment of inertia of a circular disc
1 1 2 2 v2
1 2 1 2 mv 2 + mr ´ 2 = mgh
= MR = ´ 0.4 ´ 1 ´ 1 = 0.2 kg - m 2 25 r
2 2
4. (d) Torque = Ia v 2 2v 2
+ = gh
[I is moment of inertia and a is angular 2 10
acceleration] 5v 2 + 2v 2 10
= gh Þ v 2 = gh
31.4 = I ´ 4p 10 7
31.4
I= = 2.50 kg-m 2 v³
10
gh
4p 7
5. (c) For any circular motion the angular
10. (a) (0, 2 y)
momentum is conserved as no torque is
acting on it because centripetal force acts
through the point of axis.
6. (d)
w

(0, 0) (2x, 0)
w Let l be the length of ladder and (x, y) be its
Angular velocity is a vector whose centre of mass which is middle point of the
direction is perpendicular to the plane of ladder. From the figure it is clear that,
circular path or axis of rotation. Its direction
l2
has been shown in the figure. (2x)2 + (2y)2 = l 2 Þ x 2 + y 2 =
7. (d) Angular momentum is a vector quantity 4
whose direction is perpendicular to plane of l
.
So, locus of (x, y) is a circle with radius
revolution. It has been shown in the figure. 2
As centre of mass will always go
L downwards. So option (a) is correct.
11. (b)

L
L/2
System of Particles and Rotational Motion P-43

We know that for a body, moment of inertia 2ο π


15. (d) w = < in magnitude and direction is
M.I = åM r 2 T t
axis of rotation (direction in which q
Now, bending of rod does not alter the
changes with t)
distribution of individual particle, the body n
16. (c)
is made of, so the value of å M r 2 will D C

not change. Hence the changed moment of


1 B
inertia of the body will be ML2 . A
12
12. (a) Torque created due to weight of street light n'
remains same in all the three cases. It is 2
balanced by torque created by tension in æ l ö
I nn ' = 2 ´ mçç ÷÷ + m( 2l) 2
the string. So if t be the torque created by è 2ø
weight of lamp and T be tension in the
= ml 2 + 2 ml 2 = 3ml 2
string and d be perpendicular distance of 17. (d) By the theorem of perpendicular axes,
cable from the axis then, Iz = Ix + Iy or, Iz = 2 Iy
t = T.d (\ Ix = Iy by symmetry of the figure)
Tension will be least for largest d. This is in Iz Iy
pattern A. So Pattern A is more sturdy. D F C
13. (c) When a body rolls over a smooth surface,
it has linear K.E. and rotational K.E.
1 1 Ix
\E= mv 2 ∗ Iϖ 2
2 2
v 2 2
where w = and I = mr for solid sphere. A E B
r 5
1 1 æ 2 2 ö v2 Iz
\ K.E. = mv ∗ ççç mr ÷÷÷ .
2 \ IEF = ... (i)
2 2 è5 ø r2 2
Again, by the same theorem,
1 1 7 7 Iz = IAC + IBD = 2 IAC
= mv 2 ∗ mv 2 = mv 2 = ´1´12
2 5 10 10 (\ IAC = IBD by symmetry of the figure)
7 I
= J \ IAC = z ... (ii)
10 2
14. (a) Moment of inertia of system From (i) and (ii), we get IEF = IAC.
= M.I of A + M.I. of B + M.I of C 18. (b) Given : initial angular speed,
M.I of A = M. T through centre and w0 = 2 rad/s, angular acceleration,
1 a = 3 rad/s2, time, t = 2s
perpendicular to length = ML2
12 From the equation of the angular
1 displacement,
M.I of C = M.I of A = ML2
12 1 1
M.I of B = 0 q = w 0 t + at 2 = 2 ´ 2 + ´ 3 ´ (2) 2
2 2
(moment of mass about an axis passing
through its own position is zero) = 4 + 6 = 10 radians
1 19. (a) I1w1 = I2w2
1 2 1 2 2
\ Total M.I = ML + ML = ML
12 12 6 MK12 w1 = MK 22 w2
EBD_7100
P-44 Topicwise AIIMS Solved Papers – PHYSICS

2 Now, form Fig (II), it is clear that the


æ K1 ö w2 K
ç ÷ = Þ 1 = w2 : w1 distance, OP < OC < OQ ÞVP < VC < VQ or
è K2 ø w1 K2 VQ > VC > VP.
1 26. (d) By the theorem of perpendicular axes,
20. (a) TKE = mv 2 Iz = I x + I y or, Iz = 2 Iy
2
1 (Q Ix = Iy by symmetry of the figure)
RKE = Iw 2
2 Z
Y
w = v/R
D F C
TKE 2
Þ =
TKE + RKE 3
21. (c) When no external force acts on the system X
and initially velocity of centre of mass is O
zero and so the centre mass remain at rest.
22. (c) Length of the wire = l.
A E B
Let it is bent in the form of a circular ring of
radius r.
Iz
Thus, radius of the ring \ IEF = ...(i)
2
l
r= Again, by the same theorem
2p
Iz = IAC + IBD = 2 IAC
The moment of inertia of the ring about its
axis, (\ IAC = IBD by symmetry of the figure)

æ l ö
2
l2 Iz
\ IAC = ...(ii)
I = mr2 = m ç ÷ = m ´ 2 2
è 2p ø 4p
From (i) and (ii), we get
æ 1 ö IEF = IAC.
= ç 2 ÷ ml 2.
è 4p ø 27. (a) For complete disc with mass '4M', M.I.
23. (a) As axis of rotation is along the length of about given axis = (4M)(R2/2) = 2 MR2
the cylinder are remain same, but speed Hence, by symmetry, for the given quarter
increases continuously. of the disc
24. (b) Given, w0 = 2pf = 2p × 13. = 2.6p rad/s 1 2
M.I. = 2 MR2 /4 = MR
Using I equation of motion 2
w = w0 + a t 28. (b) Since v is changing (decreasing), L is not
0 = 2.6p + a × 30 conserved in magnitude. Since it is given
that a particle is confined to rotate in a
2.6 p
Þ a= = - 0.27 rad / s2 circular path, it can not have spiral path.
30 Since the particle has two accelerations ac
25. (a) VQ w and at therefore the net acceleration is not
v VCQ towards the centre.
C C
w P v
r r
L ac at
O O
(I) (II)
From Fig. (I), we have OC = r (radius)
Therefore, v = rw The direction of L remains same even
Since, w = constant, therefore v µ r when the speed decreases.
System of Particles and Rotational Motion P-45

29. (b) a is propotional to w 33. (b) In pure rolling, mechanical energy remains
Let a = kw (Q k is a constant) conserved. Therefore, when heights of
dω dθ dθ inclines are equal, speed of sphere will be
= kω [also = ω Þ dt = ]
dt dt ω same in both the case. But as acceleration
ωdω down the plane, a µ sin q. Therefore,
\ = kω Þ dw = kdq acceleration and time of descent will be

ω/ 2 different.

Now ò dω = kò dθ Type B : Assertion Reason Questions


ω
34. (b) We know that
0 θ
ω ω
ò dω = k ò dθ Þ - = kθ Þ - = kθ1
2 2 I=
m1r12 + m 2 r2 2 + m 3r3 3 + ..... + m n rn 2
ω/2 0 m1 + m 2 + m 3 + .....
(Q q1 = 2pn) where r1, r2, r3 are distances of mass m1,
\ θ = θ1 or 2pn1 = 2pn m2, m3 etc. from the axis. From the relation
n1 = n it is clear that I depends upon distribution
30. (b) v of the masses and position of axis.
qv So, Assertion is correct.
q We know that,
angular momentum = Iw
Torque = Ia
If we compare these equations with
equations like linear momentum = mv, force
v R = v 2 + v 2 + 2 v 2 cos q = 2 v 2 (1 + cos q) = ma, we find that I represents mass in
angular motion. As mass represents inertia
q
= 2 v cos in linear motion, I represents inertia in
2
angular motion.
31. (d) For a thin uniform square sheet
But assertion and reason are mutually
ma 2 exclusive. So (b) is the answer.
I1 = I2 = I3 = 35. (c) The angular momentum of earth-moon
12
system will be conserved because no
torque is acting on it.
dL
So, t =
I1 dt
dL
If t = 0, = 0 Þ L is constant.
I2 dt
I3 or angular momentum is constant. So
r r r Reason is incorrect.
32. (c) L = m( r ´ v ) So, I1w1 = I2w2
r
L = m éêv0 cos qt iˆ + (v0 sin qt - gt 2 ) ˆj ùú
1 where I1 & I2 are momentia of inertia of
ë 2 û earth and moon & w1 & w2 are their angular
velocities. If earth slows down w, will be
´ éë v0 cos q iˆ + (v0 sin q - gt ) ˆj ùû
decreased. So, I2 will be decreased if we
é 1
ˆ ù take w2 to remain constant.
= mv0 cos qt ê - gt ú k I2= m2r22 where m2 is mass of moon & r2 is
ë 2 û
radius of moon's orbit, r 2 will be reduced to
1
= - mgv0 t cos qkˆ
2
reduce I2. Hence moon will come near to
2 the earth. Hence Assertion is correct.
EBD_7100
P-46 Topicwise AIIMS Solved Papers – PHYSICS
36. (a) Along with earth, particles of atmosphere Many objects have a point, a line or a plane
also revolves around the axis of rotation. of symmetry. The centre of mass of such
Now due to change in the constitution of an object then lies at that point, on that
atmosphere there is small change in the total line or in that plane. Also the centre of mass
moment of inertia of the whole system. of an object need not lie within the object,
Applying conservation of an gular like no iron at the centre of mass of a horse
momentum shoe.
Iw = I'w' 41. (a)
If I' changes, there is corresponding change 42. (d)
in the angular velocity of the system. 43. (d) For a disc rolling without slipping on a
37. (a) Under central force field, force acts along horizontal rough surface with uniform
the line joining the bodies so it does not angular velocity, the acceleration of lowest
have rotatory effect i.e., torque is zero.
point of disc is directed vertically upwards
Hence angular momentum is conservative.
and is not zero (Due to translation part of
38. (a) When mass of the opponent is brought
rolling, acceleration of lowest point is zero.
nearby by his moment of inertia gets
Due to rotational part of rolling, the
reduced which makes the operation of
tangential acceleration of lowest point is
rotating him around the hip an easier
exercise. zero and centripetal acceleration is non-zero
39. (b) In sliding down, the entire potential energy and upwards). Hence Assertion is incorrect.
is converted into kinetic energy. While in 44. (a)
rolling down, some part of the potential 45. (d) Radius of gyration of body is not a
energy is converted into kinetic energy of constant quantity. Its value changes with
rotation. Therefore linear velocity acquired the change in location of the axis of
is less. rotation. Radius of gyration of a body
40. (c) The position of centre of mass of a body about a given axis is given as
depends on shape, size and distribution of
r12 + r22 + .....rn2
mass of the body. The centre of mass does K=
not lie necessarily at the centre of the body. n
Gravitation P-47

7 Gravitation

TYPE A : MULTIPLE CHOICE QUESTIONS at an angle of 50° from the horizontal, then escape
velocity is: [1999]
1. If a mass of a body is M on the earth surface, the (a) 12.8 km/sec (b) 16.2 km/sec
mass of the same body on moon surface will be (c) 11.2 km/sec (d) 11.8 km/sec
M 7. Knowing that the mass of the moon is 1/81 times
(a) M (b) [1997]
6 that of earth and its radius is 1/4 the radius of
(c) zero (d) none of these earth. If the escape velocity at the surface of the
2. The earth rotates about the sun in an elliptical earth is 11.2 km/sec, then the value of escape
orbit as shown in figure. At which point its velocity at the surface of the moon is [2000]
velocity will be maximum? [1997] (a) 2.5 km/sec (b) 0.14 km/sec
B (c) 5 km/sec (d) 8 km/sec
M
8. If the mass of moon is , where M is the mass
A S C 81
of earth, find the distance of the point from the
moon, where gravitational field due to earth and
D moon cancel each other. Given that distance
(a) at C (b) at A
between earth and moon is 60R where R is the
(c) at D (d) at B radius of earth [2000]
3. The ratio of the radii of two planets r1 and r 2 is k. (a) 4 R (b) 8 R
The ratio of acceleration due to gravity on them (c) 2 R (d) 6 R
is r. Then the ratio of the escape velocities from 9. Potential energy of a satellite having mass m
them, will be : [1997] and rotating at a height of 6.4 × 106 m from the
r k earth centre is [2000]
(a) (b) (a) – 0.2 mg Re (b) – 2 mg Re
k r
(c) –0.5 mg Re (d) – mg Re
(c) kr (d) kr
10. The escape velocity from the earth is 11.2 km/
4. The value of acceleration due to gravity, at earth
sec. The escape velocity from a planet having
surface is g. Its value at the centre of the earth,
twice the radius and the same mean density as
which we assume as a sphere of radius R and of
the earth, is : [2001]
uniform mass density, will be : [1997]
(a) 11.2 km/sec (b) 22.4 km/sec
(a) 10 R m/s2 (b) zero
(c) 5 R m/s (d) 20 R m/s2 (c) 15.00 km/sec (d) 5.8 km/sec
11. If v0 be the orbital velocity of a satellite in a
5. Gravitational mass is proportional to the
circular orbit close to the earth’s surface and ve
gravitational : [1998]
is the escape velocity from the earth, then
(a) intensity (b) field
relation between the two is [2002]
(c) force (d) none of these
6. Escape velocity of a body when projected from (a) ve = 2v0 (b) v e = 3v 0
the earth’s surface is 11.2 km/sec. If it is projected (c) ve = v0 2 (c) v0 = ve
EBD_7100
P-48 Topicwise AIIMS Solved Papers – PHYSICS
12. Hubble’s law is related with [2002] 20. A long straight wire of radius a carries a steady
(a) planetary motion current i. The current is uniformly distributed
(b) speed of galaxy across its cross section. The ratio of the magnetic
(c) black hole field at a/2 and 2a is [2008]
(d) comet (a) 1/2 (b) 1/4
13. The radius of earth is about 6400 km and that of (c) 4 (d) 1
mass is about 3200 km. The mass of the earth is 21. Two bodies of masses m1 and m2 are initially at
about 10 times of the mass. The object weighs rest at infinite distance apart. They are then
200 N on earth surface, then its weight on the allowed to move towards each other under
surface of mars will be [2002] mutual gravitational attraction. Their relative
(a) 80 N (b) 40 N velocity of approach at a separation distance r
(c) 20 N (d) 8 N between them is [2008]
1/ 2
14. A satellite is launched into a circular orbit of é (m1 - m 2 ) ù
radius R around the earth. While a second (a) ê 2G r ú
ë û
satellite launched into an orbit of radius 1.01R. 1/ 2
é 2G ù
The period of the second satellite is longer than (b) êë r (m1 + m )
2 ú
the first one by approximately : [2002] û
1/ 2
(a) 3.0% (b) 1.5% é r ù
(c) 0.7% (d) 1.0% (c) ê ú
ë 2G(m1m 2 ) û
15. The velocity with which a projectile must be fired
1/ 2
so that it escapes earth's gravitation does not é 2G ù
(d) ê m1m2 ú
depend on : [2003] ë r û
(a) mass of the earth 22. A satellite S is moving in an elliptical orbit around
(b) mass of the projectile the earth. The mass of the satellite is very small
(c) radius of the projectile's orbit compared to the mass of the earth. Then
(d) gravitational constant (a) the acceleration of S is always directed
16. The difference in the length of a mean solar day towards the centre of the earth
and a sidereal day is about : [2003] (b) the angular momentum of S about the centre
(a) 1 minute (b) 4 minute of the earth changes in direction, but its
(c) 15 minute (d) 56 minute magnitude remains constant
(c) the total mechanical energy of S varies
17. The condition for a uniform spherical mass m of
periodically with time
radius r to be a black hole is : [G = gravitational
(d) the linear momentum of S remains constant
constant and g = acceleration due to gravity] in magnitude [2010]
[2005] 23. If the earth were to cease rotating about its own
1/ 2 1/ 2 axis. The increase in the value of g in C.G.S.
æ 2Gm ö æ 2gm ö
(a) ç ÷ £c (b) ç ÷ =c system at a place of latitude of 45° will be
è r ø è r ø (a) 2.68 (b) 1.68 [2010]
1/ 2 1/ 2 (c) 3.36 (d) 0.34
æ 2Gm ö æ gm ö 24. A geostationary satellite is orbiting the earth at
(c) ç ÷ ³c (d) ç ÷ ³c
è r ø è r ø a height of 6 R from the earth’s surface (R is the
18. Height of geostationary satellite is [2007] earth’s radius ). What is the period of rotation of
(a) 16000 km (b) 22000 km another satellite at a height of 2.5 R from the
(c) 28000 km (d) 36000 km earth’s surface [2011]
19. The force of gravitation is [2007] (a) 6 2 hours (b) 10 hours
(a) repulsive (b) conservative 5 5
(c) electrostatic (d) non-conservative (c) hours (d) none of the above
3
Gravitation P-49

25. The angular speed of earth in rad/s, so that (c) g1 : g2 = R1r2 : R2r1
bodies on equator may appear weightless is : (d) g1 : g2 = R1r1 : R2r2
[Use g = 10 m/s2 and the radius of earth 30. A body starts from rest from a point distance R0
= 6.4 × 103 km] [2011] from the centre of the earth. The velocity
(a) 1.25 × 10–3 (b) 1.56 × 10–3 acquired by the body when it reaches the surface
(c) 1.25 ×10–1 (d) 1.56 of the earth will be (R represents radius of the
26. Suppose, the acceleration due to gravity at the earth). [2014]
Earth's surface is 10 m s–2 and at the surface of
æ1 1 ö
Mars it is 4.0 m s–2. A 60 kg pasenger goes from (a) 2 G M çç - ÷
÷
è R R 0ø
the Earth to the Mars in a spaceship moving
with a constant velocity. Neglect all other objects æ 1 1ö
in the sky. Which part of figure best represents (b) 2 G M çç - ÷÷
R
è 0 R ø
the weight (net gravitational force) of the
passenger as a function of time? ]2012] æ1 1 ö
(c) G M çç - ÷
÷
Weight è R R 0ø

600 N A æ1 1 ö
(d) 2 G M çç - ÷
÷
è R R0 ø
B 31. The potential energy of a satellite of mass m and
200 N revolving at a height Re above the surface of
C earth where Re = radius of earth, is [2014]
time -m g R e
D t0 (a) – m g Re (b)
2
(a) A (b) B -m g R e -m g R e
(c) (d)
(c) C (d) D 3 4
27. Two masses m1 and m2 (m1 < m2) are released 32. Four similar particles of mass m are orbiting in a
from rest from a finite distance. They start under circle of radius r in the same angular direction
their mutual gravitational attraction – [2012] because of their mutual gravitational attractive
(a) acceleration of m1 is more than that of m2. force. Velocity of a particle is given by [2015]
(b) acceleration of m2 is more than that of m1. m
(c) centre of mass of system will remain at rest
in all the reference frame
(d) total energy of system does not remain m r
m
constant
28. The escape velocity for a body projected
vertically upwards from the surface of earth is
11 km/s. If the body is projected at an angle of
45º with the vertical, the escape velocity will be m
1/ 2
(a) 22 km/s (b) 11 km/s [2013] é GM æ 1 + 2 2 ö ù
(a) ê ç ÷ú
11 êë r è 4 ø úû
(c) km/s (d) 11 2 km/s
2
29. The radii of two planets are respectively R1 and 3
GM
(b)
R2 and their densities are respectively r1 and r
r2. The ratio of the accelerations due to gravity
at their surfaces is [2013] (c)
GM
r
(
1+ 2 2 )
r1 r2 1/ 2
(a) g1 : g 2 = : é 1 GM æ 1 + 2 ö ù
R12 R 22 (d) ê ç ÷ú
(b) g1 : g2 = R1R2 : r1r2 ëê 2 r è 2 ø úû
EBD_7100
P-50 Topicwise AIIMS Solved Papers – PHYSICS
33. Taking the gravitational potential at a point infinte (d) If both the Assertion and Reason are incorrect.
distance away as zero, the gravitational potential (e) If the Assertion is incorrect but the Reason is
at a point A is –5 unit. If the gravitational potential correct.
at point infinite distance away is taken as + 10 37. Assertion : A balloon filled with hydrogen will
units, the potential at point A is [2015] g
(a) – 5 unit (b) + 5 unit fall with acceleration of the moon.
6
(c) + 10 unit (d) + 15 unit
Reason : Moon has no atmosphere. [2000]
34. A particle of mass M is situated at the centre of
38. Assertion : The length of the day is slowly
a spherical shell of same mass and radius a. The
increasing.
a Reason : The dominant effect causing a
gravitational potential at a point situated at
2 slowdown in the rotation of the earth is the
distance from the centre, will be [2016] gravitational pull of other planets in the solar
3GM 2 GM system. [2003]
(a) - (b) - 39. Assertion : An astronaut experience
a a
GM 4 GM weightlessness in a space satellite.
(c) - (d) - Reason : When a body falls freely it does not
a a
experience gravity. [2007]
35. The distance of neptune and saturn from the
Directions for (Qs. 40-43) : Each of these questions
sun is nearly 1013 and 1012 meter respectively.
contains an Assertion followed by Reason. Read them
Assuming that they move in circular orbits, their
carefully and answer the question on the basis of
periodic times will be in the ratio [2016]
following options. You have to select the one that
(a) 10 (b) 100
best describes the two statements.
(c) 10 10 (d) 1000 (a) If both Assertion and Reason are correct and
36. The change in the value of ‘g’ at a height ‘h’ Reason is the correct explanation of Assertion.
above the surface of the earth is the same as at a (b) If both Assertion and Reason are correct, but
depth ‘d’ below the surface of earth. When both Reason is not the correct explanation of
‘d’ and ‘h’ are much smaller than the radius of Assertion.
earth, then which one of the following is correct? (c) If Assertion is correct but Reason is incorrect.
[2017] (d) If both the Assertion and Reason are incorrect.
3h h 40. Assertion : In a free fall, weight of a body
(a) d = (b) d = becomes effectively zero.
2 2
Reason : Acceleration due to gravity acting on
(c) d = h (d) d =2 h a body having free fall is zero. [2011]
TYPE B : ASSERTION REASON QUESTIONS 41. Assertion : The escape speed does not depend
on the direction in which the projectile is fired.
Directions for (Qs. 37-39) : These questions consist Reason : Attaining the escape speed is easier if
of two statements, each printed as Assertion and a projectile is fired in the direction the launch
Reason. While answering these questions, you are site is moving as the earth rotates about its axis.
required to choose any one of the following five 42. Assertion : The earth without atmosphere would
responses.
be inhospitably cold.
(a) If both Assertion and Reason are correct and
the Reason is a correct explanation of the Reason : All heat would escape in the absence
Assertion. of atmosphere. [2016]
(b) If both Assertion and Reason are correct but 43. Assertion : Space rocket are usually launched
Reason is not a correct explanation of the in the equatorial line from west to east
Assertion. Reason : The acceleration due to gravity is
(c) If the Assertion is correct but Reason is incorrect. minimum at the equator. [2017]
Gravitation P-51

Type A : Multiple Choice Questions x = 540R - 9x Þ 10x = 540R


x = 54 R;
1. (a) The mass of a body does not change unless
we withdraw or add some mass to it. So Distance of the point from moon = 60R – 54R = 6R
mass of a body on the surface of moon will -GMm
9. (d) Potential energy of a satellite =
remain unchanged. R
2. (b) When earth rotates around the Sun, the - GM ´ R e m
net torque acting on the body is zero. So, = = -mgR e
R e2
angular momentum will be conserved at all
points. 10. (b) For escape velocity the formula is,
I1w1 = I 2w2 4
2G pR 3r
2GM 3
Þ mv1r1 = mv 2 r2 ve = =
R R
v1r1 = v 2 r2
[r is density of the planet, R is radius.]
At 'A' radius is minimum so, velocity of the
earth will be maximum. 8
= GpR 2r
3. (d) ve = 2gr 3
8
v'e 2g1r1 ve = R Gpr
= = k.r 3
v''e 2g 2 r2 ve µ R
4. (b) The value of acceleration due to gravity at If radius becomes twice, ve will also become
the centre of earth is zero. twice. So new escape velocity
5. (c) Inertial mass is free from gravitational force. = 2 × 11.2 = 22.4 km/sec
It depends upon only mass. Gravitational 11. (c) We know that,
mass is dependent on gravitational force.
v0 = gr & ve = 2gr
6. (c) Escape velocity does not depend on the
direction of throw of object. This is because So, ve = 2v0
gravitational field is a conservative field. 12. (b) Hubble’s law states that speed of a star is
2GM directly proportional to distance from the
7. (a) For escape velocity, ve =
R star i.e.
2GM / 81 4 ´ 2GM 2 GM v µ r Þ v = Hr
v'e = = =
R/4 81R 9 R where H is Hubble’s constant.
2 GM
v'e = ´ 11.2 = 2.5 km / sec 13. (a) We know that g =
9 R2
8. (d) Let at a distance x from the earth, field
G.M /10
equalises each other. So, applying Value of g for Mass =
Newton’s law of gravitation (R / 2) 2
[Radius of mass is 1/2 that of earth]
GM GM / 81
2
= GM 4 2 GM
x (60R - x) 2 = ´ 2 =
10 R 5 R2
1 1
= 2
x2 81(60 R - x ) 2 So, g ' = g
5
1 1 200 ´ 2
Þ = If mg = 200 N, mg ' = = 80 N.
x 9(60R - x ) 5
EBD_7100
P-52 Topicwise AIIMS Solved Papers – PHYSICS
14. (b) We know that relation between radius of 19. (b) The work done by force of gravitation does
orbit and time period of revolution is not depend on path taken hence force of
T2 µ R3 gravitation is conservative.
3/2 3/ 2 20. (d) Here, current is uniformly
T ' æ 1.01 ö æ 1 ö distributed across the
=ç ÷ = ç1 + ÷
T è 1 ø è 100 ø cross-section of the wire,
therefore, current enclosed
3 1 3 a/2
= 1+ ´ = 1+ in the amperean path
2 100 200 formed at a distance P1 P2
T' 3 æ aö
-1 = 1 + -1 r1 ç = ÷
T 200 è 2ø
T '- T 3 DT 3 a
= Þ = æ p r2 ö
T 200 T 200 = ç 12 ÷ ´ I , where I is total current
DT 3 ´ 100 è pa ø
´ 100 =
T 200 \ Magnetic field at
% Change in T = 1.5% m0 ´ current enclosed
15. (b) The value of escape velocity for a planet is P1 (B1 ) =
Path
ve = 2gR
æ p r2 ö
It does not depend upon the mass of the m 0 ´ ç 12 ÷ ´ I
body. è pa ø m ´ I r1
= = 0
16. (b) Solar day is the time taken by earth to 2p r1 2p a 2
complete one rotation about its axis with Now, magnetic field at point P2,
respect to sun. Sidereal day is the time taken
m0 I m I
by earth to complete one rotation about its (B2) = . = 0 .
axis with respect to distant star. 2p (2a) 4pa
There is a difference of 4 minutes between B m Ir 4pa
\ Required Ratio = 1 = 0 1 ´
solar day and sidereal day. Solar day is B 2 2 pa 2 m 0 I
longer as earth has to rotate greater angle
a
2 r1 2 ´ 2
to attain same position with respect to sun
due to its own motion (revolution) round = = = 1.
the sun. Its position with respect to distant a a
star remains almost fixed. 21. (b) By applyin g law of conservation of
17. (c) A black hole does not allow light to escape momentum,
its surface. In other words for a black hole m1 v1 - m 2 v 2 = 0 Þ m1v1 = m 2 v 2 … (i)
body escape velocity becomes > velocity Where v1 and v2 are the velocities of masses
of light m1 and m2 at a distance r from each other.
Now for a body of mass m, By conservation of energy,
1/ 2 Change in P.E = change in K.E.
æ 2Gm ö
Escape velocity = ç ÷ . Gm1m 2 1 1
è r ø = m1v12 + m 2 v 22 … (ii)
æ 2Gm ö
1/ 2 r 2 2
So, ç ÷ ³c Solving eqn. (i) and (ii) we get
è r ø
18. (d) The height of geostationary satellites is 2Gm 22 2Gm12
v1 = and v 2 =
æ T 2 R 2g ö÷1/ 3 r(m1 + m 2 ) r(m1 + m 2 )
ç
given by h = çç ÷÷ , R
çè 4ο2 ø÷÷ Relative velocity of approach, vR

T = 24 hr, R = 6.4 × 106m, g = 9.8 m/s2 and 2G


=| v1 | + | v 2 |= (m1 + m 2 )
comes out to be 35930 km. r
Gravitation P-53

22. (a) Force on satellite is always directed towards 1


earth, So, acceleration of satellite S is 26. (c) gµ so we will not get a straight line.
R2
always directed towards centre of earth. Net
torque of this gravitational force F about Also F = 0 at a point where Force due to
centre of earth is zero. Therefore , angular Earth = Force due to Mars
momentum (both in magnitude and 27. (a) Same force acts on both masses
direction) of S about centre of earth is
constant throughout. 1
Hence a µ (F = ma)
Since, the force F is conservative in nature, m
therefore, mechanical energy of satellite In absence of external force (remember
remains constant. Speed of S is maximum mutual gravitational force is an internal
when it is nearest to earth and minimum force for the system) total energy remains
when it is farthest. constant.
S (
28. (b) Since escape velocity v e = 2g R e is )
independent of angle of projection, so it
E F will not change.
29. (d) g µ rR
23. (b) l = 45°; R = 6400 × 103 m R GMm é1 1 ù
30. (b) P.E. = ò dr = -GMm ê - ú
2p R0 r 2
ë R R0 û
w=
24 ´ 60 ´ 60 The K.E. acquired by the body at the
The value of acceleration due to gravity 1 2
with latitude l due to rotation of earth is, surface = m v
2
g' = g – Rw2 cos2 l
g – g' = Rw2 cos2 l 1 2 é1 1 ù
\ mv = -GMm ê - ú
2 2 ë R R0 û
6400 ´103 æ 2 ´ 3.14 ö
= ´ç ÷
2 è 24 ´ 60 ´ 60 ø æ 1 1ö
v = 2G M ç - ÷
6400 ´ 103 ´ 4 ´ 3.14 ´ 3.14 è R0 R ø
= 31. (b) At a height h above the surface of earth
2 ´ 24 ´ 60 ´ 60 ´ 24 ´ 60 ´ 60
the gravitational potential energy of the
= 16.89 × 10–3 m/sec2
particle of mass m is
= 16.89 × 10–1 cm/sec2 GM e m
Uh = -
= 1.68 cm/sec2 Re + h
Where Me & Re are the mass & radius of
r3 earth respectively.
24. (a) T = 2p
GM In this question, since h = Re
2 3
æT ö ær ö æ 6R + R ö
3 GM e m - mgRe
\ ç 1÷ =ç 1÷ =ç =8 So U h = Re = - =
è T2 ø è r2 ø è 2.5R + r ÷ø 2 Re 2
T 24 32. (a) Centripetal force = net gravitational force
T2 = 1 = = 6 2hr
8 8 mv20
= 2Fcos 45° + F1
281 r
25. (a) w' = g/R = 2GM 2 1 Gm 2
6.4 ´ 106 = +
= 1.25 × 10–3 rad/s ( 2r) 2 2 4r 2
EBD_7100
P-54 Topicwise AIIMS Solved Papers – PHYSICS

45° 2r T12 T1
V0 \ = (10)3 or = 10 10
2 T2
F T2
F1 F 36. (d) Variation of g with altitude is,
r é 2h ù
g h = g ê1 - ú ;
ë Rû
variation of g with depth is,
é dù
g d = g ê1 - ú
ë Rû
Equating gh and gd, we get d = 2h
mv20 Gm2
= [2 2 + 1] ` Type B : Assertion Reason Questions
r 4r 2
1/2 37. (a) A balloon will not experience any buoyant
æ GM(2 2 + 1ö
Þç ÷ force on the moon because it has no
è 4r ø atmosphere, so it will have free fall under
33. (b) The gravitational potential V at a point gravitational pull of the moon with
distant ‘r’ from a body of mass m is equal
to the amount of work done in moving a acceleration equal to g .
6
unit mass from infinity to that point. 38. (c) The length of the day is slowly increasing
r ur r not due to gravitational pull of other planets
Vr - Vµ = - ò E.dr = -GM (1/ r - 1/ µ ) in the solar system but due to viscous force
µ between the earth and the atmosphere
-GM æ r -dV ö around it. So Assertion is correct but
= çè As E = ÷ Reason is incorrect.
r dr ø
39. (a) When a body falls freely its accelerating
(i) In the first case force is g thus apparent weight of body
- GM = M(g – g) is zero. Hence astronaut falling
when Vµ = 0, Vr = = -5 unit freely in space experiences weightlessness
r
(ii) In the second case Vµ = + 10 unit as its gravitational force is counter balanced
by centripetal force of satellite.
Vr – 10 = – 5 40. (c)
or Vr = + 5 unit 41. (b)
34. (a) Potential at the given point = Potential at 42. (b) In the absence of atmosphere, all the heat
the point due to the shell + Potential due to will escape from earth’s surface which will
the particle make earth in hospitably cold.
GM 2GM 3GM 43. (b) Space rocket are usually launched from
=- - =- west to east to take the advantage of rotation
a a a
of earth.
35. (c) T2 µ R3 (According to Kepler’s law) Also g' = g – w2R cos2 l, at equator l = 0,
T12 µ (1013)3 and T22 µ (1012)3 and so cos l = 1, and g' is least.
Mechanical Properties
8 of Solids
TYPE A : MULTIPLE CHOICE QUESTIONS 8. The adjacent graph shows the extension (Dl) of
a wire of length 1 m suspended from the top of a
1. If S is stress and Y is Young’s modulus of a roof at one end and with a load W connected to
material of wire, then energy stored in the wire the other end. If the cross-sectional area of the
per unit volume is : [1997] wire is 10–6 m2, calculate the Young’s modulus
S of the material of the wire. [2008]
(a) 2S 2 Y (b)
2Y
Dl (x10 -4 m)
2Y 2
S
(c) 2 (d)
S 2Y 4
2. Longitudinal strain is possible in : [1998] 3
(a) Liquid (b) Gases 2
(c) Solid (d) All of these 1
3. Which one of the following affects the elasticity W (N)
of a substance ? [1999] 20 40 60 80
(a) Change in temperature
(a) 2 × 1011 N/m2 (b) 2 × 10–11 N/m2
(b) Hammering and annealing
(c) 3 × 10–12 N/m2 (d) 2 × 10–13 N/m2
(c) Impurity in substance
9. There are two wire of same material and same
(d) All of these
length while the diameter of second wire is two
4. If in a wire of Young’s modulus Y, longitudinal
times the diameter of first wire, then the ratio of
strain X is produced then the potential energy
extension produced in the wires by applying
stored in its unit volume will be : [2001]
same load will be [2013]
(a) 0.5 YX 2 (b) 0.5 Y 2 X (a) 1 : 1 (b) 2 : 1
(c) 1 : 2 (d) 4 : 1
(c) 2 YX 2 (d) YX 2 10. Stress vs strain curve for the elastic tissue of
5. According to Hook’s law of elasticity, if the aorta, the large tube (vessel) carrying blood
stress is increased, then the ratio of stress to from the heart, will be : [stress is proportional to
strain : [2001] square of the strain for the elastic tissue of the
(a) becomes zero (b) remains constant aorta] [2015]
(c) decreases (d) increases
6. The bulk modulus of a metal is 1010 N/m2 and
Poisson's ratio 0.20. If average distance between
the molecules is 3Å then the interatomic force (a) (b)
constant : [2002]
(a) 5.4 N/m (b) 7.5 N/m
(c) 7.5 N/m (d) 30 N/m
7. Shear modulus is zero for [2007]
(a) solids (b) liquids (c) (d)
(c) gases (d) liquids and gases
EBD_7100
P-56 Topicwise AIIMS Solved Papers – PHYSICS
11. An iron rod of length 2m and cross-sectional Directions for (Qs. 13-15) : Each of these questions
area of 50 mm2 stretched by 0.5 mm, when a contains an Assertion followed by Reason. Read them
mass of 250 kg is hung from its lower end. Young’s carefully and answer the question on the basis of
modulus of iron rod is [2017] following options. You have to select the one that
(a) 19.6 × 1020 N/m2 (b) 19.6 × 1018 N/m2 best describes the two statements.
(c) 19.6 × 1010 N/m2 (d) 19.6 × 1015 N/m2 (a) If both Assertion and Reason are correct and
Reason is the correct explanation of Assertion.
TYPE B : ASSERTION REASON QUESTIONS
(b) If both Assertion and Reason are correct, but
Directions for (Qs. 12) : These questions consist of Reason is not the correct explanation of
two statements, each printed as Assertion and Reason. Assertion.
While answering these questions, you are required to (c) If Assertion is correct but Reason is incorrect.
choose any one of the following five responses. (d) If both the Assertion and Reason are incorrect.
(a) If both Assertion and Reason are correct and
13. Assertion: Strain causes the stress in an elastic body.
the Reason is a correct explanation of the
Reason: An elastic rubber is more plastic in
Assertion.
nature. [2014]
(b) If both Assertion and Reason are correct but
14. Assertion: Hollow shaft is found to be stronger
Reason is not a correct explanation of the
than a solid shaft made of same equal material.
Assertion. Reason: Torque required to produce a given
(c) If the Assertion is correct but Reason is twist in hollow cylinder is greater than that
incorrect. required to twist a solid cylinder of same length
(d) If both the Assertion and Reason are incorrect. and material. [2016]
(e) If the Assertion is incorrect but the Reason is 15. Assertion: Solids are least compressible and
correct. gases are most compressible.
12. Assertion : Stress is the internal force per unit Reason: solids have definite shape and volume
area of a body. but gases do not have either definite shape or
Reason : Rubber is more elastic than steel. definite volume. [2017]
[2002]
Mechanical Properties of Solids P-57

Type A : Multiple Choice Questions 9. (d) Q Both wires are same materials so both
1. (d) Energy stored in stretched wire per unit will have same Young’s modulus, and let it
stress F
be Y. Y = = ,
volume =
1
´ stress ´ strain strain A.(ΔL/L)
2 F = applied force
1 S 1 S2 A = area of cross-section of wire
= ´S´ =
2 Y 2 Y Now,
FL FL
2. (c) Longitudinal strain is possible only in Y1 = Y2 Þ ( )( =
solids because only solids can have length A1 DL1 ) (A 2 )(DL 2 )
Since load and length are same for both
which can be stretched by applying force.
3. (d) The elasticity of a material depends upon Þ r12 DL1 = r2 2 DL 2 ,
the temperature of the material. Hammering
& annealing reduces elastic property of a æ DL1 ö æ r2 ö
2
substance. çç ÷÷ = çç ÷÷ = 4 DL1 : DL2 = 4 :1
4. (a) Potential energy stored per unit volume of è DL 2 ø è r1 ø
a wire 10. (a) As stress µ strain 2 hence graph (a)
1 correctly dipicts.
= ´ Stress ´ Strain
2 250 ´ 9.8
-6
1
= ´Y´X´X [Stress = Y × X] 11. (c) Y = F / A = 50 ´ 10
2 Dl / l 0.5 ´ 10-3
= 0.5YX 2 2
5. (b) The Ratio of stress to strain is always 250 ´ 9.8 2
= ´ Þ 19.6 ´ 1010 N / m 2
constant. If stress is increased, strain will 50 ´ 10 - 6 0.5 ´ 10 -3
also increase so that their ratio remains
constant. Type B : Assertion Reason Questions
6. (a) Young's modulus, 12. (c) Stress is internal force (restoring force)
Y = 3K(1 - 2s) = 3 × 1010(1 – 2 × 0.2) developed within the body of the object.
= 1.8 × 1010 N/m2 Since it is easier to stretch rubber so it is
\ Interatomic force constant is less stressful and therefore less elastic.
K = Yr = 1.8 × 1010 × 3 × 10-10 = 5.4 N/m 13. (a)
7. (d) Shear modulus is applicable to solids where 14. (a) Torque required to produce a given twist
deforming force causes change in shape in hollow cylinder is greater than solid
of body. For fluids it is not possible since cylinder thus both are correct.
they have no fixed shape. 15. (b) The incompressibility of solids is primarily
F Dl 20 ´1 due to the tight coupling between the
8. (a) Y= = -6 neighbouring atoms. Molecules in gases
A l 10 ´ 10-4
are very poorly coupled to their
= 2 ´ 1011 N / m 2 neighbours.
EBD_7100
P-58 Topicwise AIIMS Solved Papers – PHYSICS

Mechanical Properties
9 of Fluids
TYPE A : MULTIPLE CHOICE QUESTIONS
(a) 200 m / s (b) 400 m / s
1. A big drop of radius R is formed by 729 small
drops of water of radius r, then the radius of (c) 500 m / s (d) 800 m / s
each small drop will be : [1997]
7. A spherical drop of water has 1 mm radius. If the
R R
(a) (b) surface tension of water is 70 ´ 10 -3 N / m .
9 900
Then the difference of pressures between inside
R R and outside of the spherical drop is : [2001]
(c) (d)
1800 9000
(a) 140 N / m 2 (b) 140 N/m
2. The work done in splitting a drop of water of 1
mm radius into 106 droplets is (surface tension (c) 35 Nm 2 (d) none of these
of water 72 ×10–3 N/m) : [1998]
8. Bernoulli’s principle is based on the law of
(a) 5.98 ´ 10 -5
J (b) 10.98 ´10 -5 J conservation of : [2001, 2013]
(a) mass
(c) 16.95 ´10 -5 J (d) 8.95 ´10 -5 J
(b) energy
3. The excess pressure inside the first soap bubble
(c) angular momentum
is three times that inside the second bubble then,
the ratio of volume of the first to the second (d) linear momentum
bubble will be : [1998] 9. Scent sprayer is based on [2002]
(a) 1 : 27 (b) 3 : 1 (a) Bernoulli’s theorem
(c) 1 : 3 (d) 1 : 9 (b) Archimedes principle
4. The rain drops are in spherical shape due to (c) Charle’s law
[1998] (d) Boyle’s law
(a) surface tension (b) viscosity
10. A soap bubble in vacuum has a radius 3 cm and
(c) residual pressure (d) thrust on drop another soap bubble in vacuum has radius 4 cm.
5. If work done in increasing the size of a soap film If two bubbles coalesce under isothermal
from 10 cm × 6 cm to 60 cm × 11 cm is 2 ´ 10 - 4 J. condition. Then the radius of the new bubble
What is the surface tension ? [2000] will be :
(a) 7 cm (b) 5 cm [2002]
(a) 2 ´10 -8 Nm -1 (b) 2 ´10- 2 Nm -1
(c) 4.5 cm (d) 2.3 cm
(c) 2 ´10- 4 Nm -1 (d) none of these 11. Two small drops of mercury, each of radius R
coalesce to form a single large drop. The radio
6. A hole is made at the bottom of the tank filled
of the total surface energies before and after the
with water (density 1000 kg/m3). If the total
change is : [2003]
pressure at the bottom of the tank is 3 1/3 1/3
atmosphere (1 atmosphere = 105 N/m2), then the (a) 1 : 2 (b) 2 : 1
velocity of efflux is [2000] (c) 2 : 1 (d) 1 : 2
Mechanical Properties of Fluids P-59

12. A lead shot of 1 mm diameter falls through a 17. A candle of diameter d is floating on a liquid in a
long column of glycerine. The variation of its cylindrical container of diameter D (D >> d) as
velocity v with distance covered is represented shown in figure. If it is burning at the rate of 2
by : [2003] cm/hour then the top of the candle will : [2005]

v v L
(a) (b)
Distance Distance
covered covered L
d
v v
D
(c) (d)
Distance Distance (a) remain at the same height
covered covered (b) fall at the rate of 1 cm/hour
13. In old age arteries carrying blood in the human
(c) fall at the rate of 2 cm/hour
body become narrow resulting in an increase in
(d) go up at the rate of 1 cm/hour
the blood pressure. This follows from : [2004]
18. A given shaped glass tube having uniform
(a) Pascal' law
cross-section is filled with water and is mounted
(b) Stoke's law
(c) Bernoulli's principle on a rotatable shaft as shown in figure. If the
(d) Archimedes principle tube is rotated with a constant angular velocity
14. A sphere of mass M and radius R is falling in a w
viscous fluid. The terminal velocity attained by then [2005]
the falling object will be proportional to :[2004]
(a) R2 (b) R
(c) 1/R (d) 1/R2 A B
15. For a constant hydraulic stress on an object,
the fractional change in the object’s volume
(DV / V) and its bulk modulus (B) are related
as : [2005]
DV DV 1
(a) µB (b) µ
V V B L 2L
DV
(c) µ B2 (d) DV µ B -2
V V (a) water levels in both sections A and B go
16. The apparent depth of water in cylindrical water up
tank of diameter 2R cm is reducing at the rate of (b) water level in section A goes up and that
x cm/minute when water is being drained out at in B comes down
a constant rate. The amount of water drained in (c) water level in section A comes down and
that in B it goes up
c.c. per minute is : ( n1 = refractive index of air,
(d) water levels remain same in both sections
n 2 = refractive index of water) [2005] 19. By sucking through a straw, a student can reduce
the pressure in his lungs to 750 mm of Hg (density
xpR 2 n1 xpR 2 n 2 = 13.6 gm/cm3). Using the straw, he can drink
(a) (b)
n2 n1 water from a glass upto a maximum depth of :
[2006]
2pRn1 (a) 10 cm (b) 75 cm
(c) (d) pR 2 x
n2 (c) 13.6 cm (d) 1.36 cm
EBD_7100
P-60 Topicwise AIIMS Solved Papers – PHYSICS
20. Work of 3.0 × 10–4 joule is required to be done 27. A spherical solid ball of volume V is made of a
in increasing the size of a soap film from 10 cm × material of density r1. It is falling through a liquid
6 cm to 10 cm × 11 cm. The surface tension of the of density r1 (r2< r1). Assume that the liquid
film is [2007] applies a viscous force on the ball that is
(a) 5 × 10–2 N/m (b) 3 × 10–2 N/m proportional to the square of its speed v, i.e.,
(c) 1.5 × 10–2 N/m (d) 1.2 × 10–2 N/m Fviscous = –kv2 (k > 0). The terminal speed of the
21. If the terminal speed of a sphere of gold (density ball is [2013]
= 19.5 kg/m3) is 0.2 m/s in a viscous liquid
(density = 1.5 kg/m3), find the terminal speed of Vg(r1 – r2 ) Vgr1
(a) (b)
a sphere of silver (density = 10.5 kg/m3) of the k k
same size in the same liquid [2008]
(a) 0.4 m/s (b) 0.133 m/s Vgr1 Vg(r1 – r2 )
(c) (d)
(c) 0.1 m/s (d) 0.2 m/s k k
22. Water is filled in a container upto height of 28. A ring is cut from a platinum tube 8.5 cm internal
3m. A small hole of area ‘A 0 ’ is punched in and 8.7 cm external diameter. It is supported
the wall of the container at a height 52.5 cm horizontally from the pan of a balance, so that it
from the bottom. The cross sectional area of comes in contact with the water in a glass vessel.
the container is A. If A0 /A = 0.1 then v2 is If an extra 3.97. If is required to pull it away from
(where v is the velocity of water coming out water, the surface tension of water is [2013]
of the hole) [2008] (a) 72 dyne cm–1 (b) 70.80 dyne cm–1
(a) 50 m 2 /s2 (b) 50 .5 m 2 /s2 (c) 63.35 dyne cm –1 (d) 60 dyne cm–1
(c) 51 m 2 /s2 (d) 52 m 2 /s2 29. A water tank of height 10m, completely filled
23. A boy has 60 kg weight. He wants to swim in a with water is placed on a level ground. It has
river with the help of a wooden log. If relative two holes one at 3 m and the other at 7 m from its
density of wood is 0.6, what is the minimum base. The water ejecting from [2014]
volume of wooden log? [2010] (a) both the holes will fall at the same spot
(density of river water is 1000 kg/m3) (b) upper hole will fall farther than that from
(a) 0.66 m3 (b) 150 m3 the lower hole
(c) upper hole will fall closer than that from
3 3 3 3
(c) m (d) m the lower hole
1 20 (d) more information is required
24. The work done in blowing a soap bubble of 30. Which of the following relation is true ?[2014]
radius 0.2m, given that the surface tension of
soap solution is 60 × 10–3 N/M is : [2011] 9hY
(a) 3Y = K (1 - s) (b) K=
(a) 24p ×10–4J (b) 24p ×10–4J Y +h
(c) 96p × 10–4J (d) 1.92p ×10–4J 05.Y - h
25. Water rises to a height of 10 cm in capillary tube (c) s = (6K + h)Y (d) s =
h
and mercury falls to a depth of 3.1 cm in the 31. A wooden block, with a coin placed on its top,
same capillary tube. If the density of mercury is floats in water as shown in fig. the distance l
13.6 and the angle of contact for mercury is 135°, and h are shown there. After some time the coin
the approximate ratio of surface tensions of water falls into the water. Then [2014]
and mercury is [2012] Coin
(a) 1 : 0.15 (b) 1 : 3
(c) 1 : 6 (d) 1.5 : 1
26. The lift of an air plane is based on [2012] l
(a) Torricelli's theorem h
(b) Bernoulli's theorem
(c) Law of gravitation
(d) Conservation of linear momentum.
Mechanical Properties of Fluids P-61

(a) l decreases and h increases


(a) | 2 P0 Rh + pR 2rgh - 2 RT |
(b) l increases and h decreases
(c) both l and h increases (b) | 2 P0 Rh + Rrgh2 - 2 RT |
(d) both l and h decreases
32. 1 m3 water is brought inside the lake upto 200 (c) | P0 pR2 + Rrgh2 - 2 RT |
metres depth from the surface of the lake. What
(d) | P0 pR 2 + Rrgh2 + 2 RT |
will be change in the volume when the bulk
modulus of elastically of water is 22000 36. A vessel contains oil (density = 0.8 gm/cm3) over
atmosphere? [2015] mercury (denisty = 13.6 gm/cm 3 ). A
3 3
(density of water is 1 × 10 kg/m atmosphere homogeneous sphere floats with half of its
pressure = 105 N/m2 and g = 10 m/s2) volume immersed in mercury and the other half
in oil. The density of the material of the sphere
(a) 8.9 × 10–3 m3 (b) 7.8 × 10–3 m3
in gm/cm3 is [2016]
(c) 9.1 × 10–4 m3 (d) 8.7 × 10–4 m3
(a) 3.3 (b) 6.4
33. The excess of pressure inside a soap bubble is
(c) 7.2 (d) 12.8
twice the excess pressure inside a second soap
bubble. The volume of the first bubble is n times 37. A uniform cylinder of length L and mass M
the volume of the second where n is [2015] having cross-sectional area A is suspended,
with its length vertical, from a fixed point by a
(a) 0.125 (b) 0.250
massless spring such that it is half submerged
(c) 1 (d) 2
in a liquid of density s at equilibrium position.
34. A steel wire is suspended vertically from a rigid The extension x0 of the spring when it is in
support. When loaded with a weight in air, it equilibrium is: [2016]
extends by la and when the weight is immersed
completely in water, the extension is reduced to Mg Mg æ LAs ö
lw. Then the relative density of material of the
(a) (b) ç1 – ÷
k k è M ø
weight is [2016]
Mg æ LAs ö Mg æ LAs ö
la (c) ç1 – ÷ (d) ç1 + ÷
(a) la / lw (b) k è 2M ø k è M ø
la -lw
38. If two glass plates have water between them
(c) l w /(l a - l w ) (d) l w / la and are separated by very small distance (see
35. Water is filled up to a height h in a beaker of figure), it is very difficult to pull them apart. It is
radius R as shown in the figure. The density of because the water in between forms cylindrical
water is r, the surface tension of water is T and surface on the side that gives rise to lower
the atmospheric pressure is P0. Consider a pressure in the water in comparison to
vertical section ABCD of the water column atmosphere. If the radius of the cylindrical
through a diameter of the beaker. The force on surface is R and surface tension of water is T
water on one side of this section by water on then the pressure in water between the plates is
the other side of this section has magnitude lower by [2017]

2R [2016] Cylindrical surface


of water
B
A 2T 4T
(a) (b)
h R R
C
T T
(c) (d)
D 4R R
EBD_7100
P-62 Topicwise AIIMS Solved Papers – PHYSICS

TYPE B : ASSERTION REASON QUESTIONS Directions for (Qs. 45-49) : Each of these questions
contains an Assertion followed by Reason. Read them
Directions for (Qs. 39-44) : These questions consist
carefully and answer the question on the basis of
of two statements, each printed as Assertion and
following options. You have to select the one that
Reason. While answering these questions, you are
best describes the two statements.
required to choose any one of the following five
responses. (a) If both Assertion and Reason are correct and
(a) If both Assertion and Reason are correct and Reason is the correct explanation of Assertion.
the Reason is a correct explanation of the (b) If both Assertion and Reason are correct, but
Assertion. Reason is not the correct explanation of
(b) If both Assertion and Reason are correct but Assertion.
Reason is not a correct explanation of the (c) If Assertion is correct but Reason is incorrect.
Assertion. (d) If both the Assertion and Reason are incorrect.
(c) If the Assertion is correct but Reason is 45. Assertion : A large force is required to draw
incorrect. apart normally two glass plates enclosing a thin
(d) If both the Assertion and Reason are incorrect. water film.
(e) If the Assertion is incorrect but the Reason is Reason : Water works as glue and sticks two
correct. glass plates. [2010]
39. Assertion : In a pressure cooker the water is 46. Assertion : Falling raindrops acquire a terminal
brought to boil. The cooker is then remove from velocity.
the stove. Now on removing the lid of the
Reason : A constant force in the direction of
pressure cooker, the water starts boiling against.
motion and a velocity dependent force opposite
Reason : The impurities in water bring down its
to the direction of motion, always result in the
boiling point [2004]
acquisition of terminal velocity. [2011]
40. Assertion : Smaller drops of liquid resist
deforming forces better than the larger drops 47. Assertion : The velocity of flow of a liquid is
Reason : Excess pressure inside a drop is directly smaller when pressure is larger and vice-versa.
proportional to its surface area. [2004] Reason : According to Bernoulli’s theorem, for
41. Assertion : For Reynold’s number Re > 2000, the stream line flow of an ideal liquid, the total
the flow of fluid is turbulent. energy per unit mass remains constant.
Reason : Inertial forces are dominant compared [2013, 14]
to the viscous forces at such high Reynold’s 48. Assertion : The buoyant force on a submerged
numbers. [2005] rigid object can be considered to be acting at
42. Assertion : A thin stainless steel needle can lay the centre of mass of the object.
floating on a still water surface. Reason : For a rigid body a force field distributed
Reason : Any object floats when the buoyancy uniformly through its volume can be considered
force balances the weight of the object [2006] to be acting at the centre of mass of the body.
43. Assertion : Machine parts are jammed in winter. [2015]
Reason : The viscosity of lubricant used
49. Assertion: The pressure of water reduces when
in machine parts increase at low temperatures.
it flows from a narrow pipe to a wider pipe.
[2007]
44. Assertion : A bubble comes from the bottom of Reason: Since for wider pipe area is large, so
a lake to the top. flow of speed is small and pressure also reduces
Reason : Its radius increases. [2008] proportionately. [2017]
Mechanical Properties of Fluids P-63

Type A : Multiple Choice Questions So, ratio of their volumes is,


1. (a) Equating volume in both cases, 4 3
pr
3 1 = v1 Þ v1 = 1
4 4
pR 3 = 729 ´ pr 3 4 3 v2
pr
v 2 27
3 3 3 2
R3 4. (a) The surface of water tends to contract
3
Þ r = which is known as surface tension. In case
729 of water droplets, the surface tension
R attains minimum value when its shape is
Þ r= spherical. So water droplets are spherical
9 in shape.
2. (d) Radius of new droplet if be r then, 5. (d) Work done
4 4 = Increase in surface area × Surface
106 ´ pr 3 = p ´ (0.001)3 tension
3 3
2(60 ´ 11 - 10 ´ 6) ´ T
2 ´ 10 -4 =
r 3 = 10 -15 Þ r = 10 -5 100 ´ 100
Increase in surface area 2 ´ 10 -4 1
T= = ´ 10 - 2 Nm -1
-5 2 -3 2
= éë 4p ´ (10 ) ´ 10 ùû - éë4p ´ (10 ) ùû
6 2 ´ 6 ´ 10 -2 6
6. (b) We know that velocity of efflux, v = 2gh
= ëé4p ´ 10-4 ûù - ëé4p ´ 10 -6 ûù = 4p 10 -6 [100 - 1] At the bottom of tank pressure is 3
-6 -6
atmosphere. So, total pressure due to water
= 4p ´10 ´ 99 = 4p ´ 10 ´ 99 column
Work done = hrg = 2 ´ 105 ( two atmosphere )
= surface tension × increase in surface area
2 ´ 105 2 ´ 105
-6 -3
= 72 ´ 4 p ´ 99 ´ 10 ´ 10 = 8.95 ´ 10 -5
J Þ gh = = = 2 ´ 10 2
r 10 3
3. (a) Excess pressure in first soap bubble,
4T Þ v = 2 ´ 2 ´ 102 = 400 m / sec
p1 =
r1
.
7. (a)
\ excess pressure inside second bubble,
4T
p2 =
r2
Excess pressure, Dp = 2T
On dividing these, we get r
p1 r2
= 2 ´ 70 ´ 10 -3
= = 140 newton / m 2
p 2 r1 1´ 10 -3

8. (b) Bernoulli’s principle is based on the law of


r1 1
but p1 = 3p2 Þ = conservation of energy. We equate total
r2 3 energy (pressure energy, potential energy
and kinetic energy) of a flowing liquid at
3
ær ö 1 different points flowing under constant
Þ ç 1÷ = pressure difference.
è r2 ø 27
EBD_7100
P-64 Topicwise AIIMS Solved Papers – PHYSICS
9. (a) Bernoulli’s theorem states that when there 14. (a) For a falling body in viscous fluid the
is greater speed in liquid, pressure is terminal velocity is related to radius as
reduced. When air is pumped inside the follows.
pipe, the velocity of air inside increases
2 2
which creates low pressure there. The liquid VT = R (r - s)g Þ vT µ R 2
in the basic is then travelled in upward 9h
direction. This is theory of Scent Sprayer. Stress Stress
10. (b) If r1, r2, r be radius of soap bubbles before 15. (b) B= =
Volume strain DV / V
and after the coalesce & p1, p2 and p the
pressure then, applying gas laws equation DV Stress
=
p1V1 + p2 V2 = pV V B
4T 4 3 4T 4 3 4T 4 3 DV 1
´ pr1 + ´ pr2 = ´ pr As stress is constant. So, µ
r1 3 r2 3 r 3 V B
16. (b) If apparent depth = dA and real depth = dR
r12 + r22 = r 2
dR n2 n
= Þ dA = 1 dR
32 + 4 2 = r 2 Þ r = 25 = 5 cm. d A n1 n2
11. (b) Let r be radius of common drop n
Dd A = 1 Dd R .............. (i)
4 3 4 n2
pr = 2 ´ pR 3
3 3 Now , V = pR 2 d R
1
r = (2) 3 R DV = pR 2 Dd R
Surface energy before the coalesce DV
Dd R =
= 2 ´ 4pR 2 T pR 2
Putting it in equation (i),
Surface energy after the coalesce = 4pr 2 T
n 1 DV
Dd A = Þ DV = n 2 pR 2 .x.
2 ´ 4pR 2 T 2R 2 n 2 pR 2 n1
Ratio = =
4pr 2 T 22 / 3 R 2 17. (b) The candle floats on the water with half its
1 2 1 length above and below water level. Let its
length be 10 cm. with 5 cm. below the
2 3 .2 3 23
= = surface and 5 cm. above it. If its length is
2 1 reduced to 8 cm. It will have 4 cm. above
23 water surface. So we see tip going down
12. (a) When a body falls through a viscous liquid, by 1 cm. So rate of fall of tip = 1 cm/hour.
its velocity increases due to gravity but 18. (a) Water level in both A and B will go up. The
after some time its velocity becomes pressure difference thus created will
uniform because of viscous force becoming provide the necessary centripetal force for
equal to the gravitational force. Viscous the water body to rotate around the vertical
force itself is a variable force which axis.
increases as velocity increases, so curve 19. (c) Pressure difference created = 10 mm of Hg
(a) represents the correct alternative.
This must be equal to the pressure of water
13. (c) In old age arteries carrying blood when column being created in the straw. If height
there in narrow arteries pressure is of water column be h
increased. Actually due to narrowness and
other obstruction the velocity of the flow 10
h rg = ´ 13.6 ´ g
of blood gets decreased. This results in 10
increased pressure inside the blood vessel, h × 1 = 13.6 Þ h = 13.6 cm.
according to Bernoulli’s principle.
Mechanical Properties of Fluids P-65

20. (b) Area increased = (10 × 11) – (10 × 6) cm2 24. (d) W = TDA = T × 2[4pR2]
= 110 – 60 = 50 cm2 = 60 × 10–3 × 8p × (0.2)2
Since film has 2 sides = 1.92 p × 10–4J
\ total increased area = 50 × 2 = 100 cm 2 2s cos q hr
work done = surface tension × increase in 25. (c) h= Þsµ
rrg cos q
surface area
Þ Surface tension s w h w r w cos q m
Þ = ´
Work done s m cos q w h m r m
=
increasein surface area 10 ´ 1 cos 135°
= ´
cos 0° - 3.1 ´ 13.6
3 ´ 10-4 3 ´ 10-4
= = 10 ´ ( -0.707 ) 1
100 cm 2 100 ´ 10 -4 m 2 = »
- 3.1 ´ 13.6 6
= 0.03 N/m = 3 × 10–2 N/m
26. (b) Apply Bernoulli's theorem.
21. (c) 2r 2 (d1 - d 2 )g 27. (a) The condition for terminal speed (vt) is
VT = Weight = Buoyant force + Viscous force
9h
Fv B=Vr2 g
VT2 (10.5 - 1.5) 9
= Þ VT2 = 0.2 ´
0.2 (19.5 - 1.5) 18

\ VT2 = 0.1 m / s
22. (a) The square of the velocity of flux

A
W=Vr 1 g
2.475 m =h
3m
\ Vr1g = Vr2g + kv t2
0.525 m
Vg (r1 - r2 )
\ vt =
2gh k
v2 = 2 28. (a) (2pr1 + 2pr2 )s = mg
æA ö
1- ç 0 ÷
è Aø é 8.7 8. 5 ù
ê2 p ´ 2 + 2p ´ 2 ús = 3.97 ´ 980
ë û
2 ´ 10 ´ 2.475
= = 50 m 2 / s 2
1 - (0.1) 2 Þ s = 72 dyne cm-1
23. (d) Archimedes principle states that 29. (a) Velocity of water from hole
weight of body displaced by liquid A = v1 = 2gh
= upthrust. Velocity of water from hole B
60 × g + V × 0.6 × 103 g = V × 1000 g
= v 2 = 2g(H 0 - h)
60 + 600 V = 1000V
60 = 400 V Time of reaching the ground from hole B

60 3 3 = t1 = 2( H 0 - h ) / g
V= = m Time of reaching the ground from hole A
400 20
where, V is the volume of wooden log. = t 2 = 2h / g
EBD_7100
P-66 Topicwise AIIMS Solved Papers – PHYSICS

0.5Y - h 36. (c)


30. (d) Y = 2h(1 + s) Þ s =
h
31. (d) As the block moves up with the fall of coil,
l decreases, similarly h will also decrease Oil
because when the coin is in water, it
displaces water equal to its own volume Mercury
only.
P PV
32. (c) K = \ DV = As the sphere floats in the liquid. Therefore
DV / V K
its weight will be equal to the upthrust force
P = hrg = 200 × 103 × 10 N/m2 on it
K = 22000 atm = 22000 × 105 N/m2
4 3
V = 1m3 Weight of sphere = p R rg … (i)
3
200 ´ 103 ´ 10 ´ 1
DV = = 9.1 ´ 10 - 4 m3 Upthrust due to oil and mercury
5
22000 ´ 10
2 3 2
= pR ´ s oil g + pR3 s Hg g … (ii)
4T 4T 3 3
33. (a) Given, = 2´ or r2 = 2r1
r1 r2 Equating (i) and (ii)
4 3 4 4 4 3 2 2
pr1 = n ´ pr23 = n ´ p(2 r1 )3 pR rg = pR 3 0.8 g + pR 3 + 13.6 g
3 3 3 3 3 3
1 Þ 2r = 0.8 + 13.6 = 14.4 Þ r = 7.2
or n = = 0.125
8
37. (c) From figure, kx 0 + FB = Mg
34. (b) Let V be the volume of the load and r its
relative density
FL VrgL KX0
So, Y = = .....(1)
A la Ala FB
When the load is immersed in the liquid,
then Mg
F¢ L (V r g - V ´1´ g) L ....(2)
Y= = L
Alw Alw kx 0 + s Ag = Mg
2
(Q Now net weight = weight – upthrust) [Q mass = density × volume]
From eqs. (1) and (2), we get
L
r (r - 1) la Þ kx 0 = Mg - s Ag
= or r = 2
la lw (l a - l w )
35. (b) Net force = Average pressure × Area sLAg
Mg -
– T × 2R Þ x0 = 2 = Mg æ1 - LAs ö
ç ÷
k k è 2M ø
æ hö
çè P0 + rg ÷ø (2 Rh) - T 2 R Hence, extension of the spring when it is in
2
Mg æ LAs ö
Þ | 2 P0 Rh + Rrgh 2 - 2 RT | equilibrium is, x 0 = ç1 - ÷
k è 2M ø
Mechanical Properties of Fluids P-67

T T dv
38. (d) Here excess pressure, Pexcess = r + r F = -hA
1 2 dx

T ær = R ö Þ force required to move the body or


Pexcess = Qç1 ÷
R è r2 = O ø machine increases hence, machines are
fammed.
Type B : Assertion Reason Questions 44. (b) The pressure will be greater at the bottom
than at the top. So the air bubble moves
39. (c) The water starts boiling a second time from the bottom to the top i.e., from higher
because when pressure cooker cools down to lower pressure. Further in coming from
pressure inside gets reduced. Reduced bottom to top, the pressure decreases and
pressure brings down the B.P. of water. The hence volume increases (By Boyle's law,
reduced B.P. makes the water boil a second PV = constant), thus radius also increases.
time.
45. (c) In this case, atmospheric pressure does not
40. (b) Smaller drops have larger excess pressure
comes into it because it acts in all direction.
inside. The excess pressure is related to
The force which is effective in case of
radius as follow
water between two pieces of glass is
4T adhesive force. As, adhesive forces are
p=
r considered that between two different
That is why smaller droplets resist bodies; cohesive forces are internal forces
deforming forces. of a body, resulting from attraction between
41. (a) Reynold number the molecules of it. The attractive force
between water and glass (the glass contain
Inertial force per unit area silicium atoms, negatively charged and
=
Viscous force perunit area water is a polar molecule so that the positive
So for higher value of Reynold’s number, side of water is attached and causes part
inertial force is dominant. of the bound) keep them firmly tovether.
42. (b) Assertion and Reason are correct. But Due to the big surface of the glass slide,
Reason does not explain Assertion. the resultant force is also big. So we have
Explanation of Assertion is that it is the to apply a large force in order to separate
surface tension of the water surface which two glass plates enclosed with water film.
is balancing the weight of the steel needle. 46. (c) 47. (d) 48. (c)
43. (a) Viscosity of a liquid decreases with 49. (d) Pressure of water reduces when it comes
increase in temperature and vice versa i.e, from wide pipe to narrow pipe. According
to equation of continuity, av = constant.
1
h= As the water flows from wider tube to
T narrow tube, its velocity increases.
\ at low temperatures viscosity increases According to Bernouli prinicple, where
Þ Viscous drag increases velocity is large pressure is less.
EBD_7100
P-68 Topicwise AIIMS Solved Papers – PHYSICS

Thermal Properties
10 of Matter
TYPE A : MULTIPLE CHOICE QUESTIONS 7. On a cold morning, a metal surface on touching
is felt colder than a wooden surface, because
1. If the temperature of a black body increases from the metal has : [1998]
7°C to 287°C, then the rate of emission of (a) low thermal conductivity
radiation energy is: [1997] (b) high thermal conductivity
(a) 8 times (b) 16 times (c) high specific heat
(c) 2 times (d) 4 times (d) low specific heat
2. The radiation from the sun, incident normally at 8. The relative humidity on a day, when partial
pressure of water vapour is 0.012 ×105 pa at 12°C
the surface of the earth is 20 kcal / m 2 min. is (take vapour pressure of water at this
What would have the radiant energy, incident temperature as 0.016 ×105 pa) : [1998]
normally on the earth if the sun had a (a) 70 % (b) 40 %
temperature, twice of the present one ? [1997] (c) 75 % (d) 25 %
9. The absolute zero is the temperature at which :
(a) 80 kcal / m 2 min (b) 320 kcal / m 2 min (a) all substances exist in solid state [1998]
(b) molecular motion ceases
(c) 160 kcal / m 2 min (d) 40 kcal / m 2 min (c) water freezes
3. The thermal conductivity of a rod is 2. What is (d) none of these
10. A quantity of heat required to change the unit
its thermal resistivity ? [1997]
mass of a solid substance to its liquid state, while
(a) 0.5 (b) 1 the temperature remains constant, is known as
(c) 0.25 (d) 2 (a) latent heat of vaporation [1998]
4. The instrument used to measure the temperature (b) latent heat of fusion
of the source from its thermal radiation is : [1997] (c) heat of condensation
(a) hydrometer (b) barometer (d) specific heat
(c) thermopile (d) pyrometer 11. Woolen clothes keep the body warm because
the wool : [1998]
5. The surface temperature of a body is 727°C and
(a) decreases the temperature of the body
that of another body is 327°C. The ratio of total
(b) is a good conductor of heat
energies radiated by them is : [1997] (c) increases the temperature of the body
(a) 625 : 81 (b) 125 : 27 (d) is a bad conductor of heat
(c) 8 : 27 (d) 9 : 25 12. Heat travels through vacuum by : [1998]
6. A Centigrade and Fahrenheit thermometers are (a) convection (b) radiation
dipped in boiling water. The water temperature (c) conduction (d) all of these
is lowered until the Fahrenheit thermometer 13. A black body has maximum wavelength lm at
registers a temperature of 140°C. 2000 K. Its corresponding wavelength at 3000 K
is: [1999]
The fall of the temperature as registered by the
16 3
centigrade thermometer is : [1998] (a) lm (b) lm
81 2
(a) 40° (b) 80°
81 2
(c) 50° (d) 90° (c) lm (d) lm
16 3
Thermal Properties of Matter P-69

14. When a solid is converted into a gas, directly


by heating then this process is known as: [1999]
I T2 I
(a) Sublimation (b) Vaporization (c) (d) T1
(c) Condensation (d) Boiling T1 T2
15. The sun emits a light with maximum wave length l l
510 nm while another star emits a light with 23. Suppose the sun expands so that its radius
maximum wavelength of 350 nm. The ratio of becomes 100 times its present radius and its
surface temperature of sun and the star will be : surface temperature becomes half of its present
(a) 0.68 (b) 2.1 [2000] value. The total energy emitted by it then will
(c) 1.45 (c) 0.46 increase by a factor of : [2004]
16. The real coefficient of volume expansion of (a) 104 (b) 625
glycerine is 0.000597 per°C and linear coefficient (c) 16 (d) 16
of expansion of glass is 0.000009 per °C. Then 24. Three objects colored black, gray and white can
the apparent volume coefficient of expansion of withstand hostile conditions upto 2800°C. These
glycerine is [2000] objects are thrown into a furnace where each of
(a) 0.000558 per °C (b) 0.00057 per °C them attains a temperature of 2000°C. Which
(c) 0.00027 per °C (d) 0.00066 per °C object will glow brightest? [2006]
17. The colour of a star indicates its : [2001] (a) the white object
(a) velocity (b) temperature (b) the black object
(c) size (d) length (c) all glow with equal brightness
18. A black body is heated from 27°C to 127°C. The (d) gray object
ratio of their energies of radiation emitted will 25. A bimetallic strip consists of metals X and Y. It is
be: [2001] mounted rigidly at the base as shown. The metal
(a) 9 : 16 (b) 27 : 64 X has a higher coefficient of expansion
(c) 81 : 256 (d) 3 : 4 compared to that for metal Y. When the bimetallic
19. A black body is at a temperature 300 K. It emits strip is placed in a cold bath: [2006]
energy at a rate, which is proportional to[2002]
(a) (300)4 (b) (300)3 X Y
(c) (300) 2 (d) 300
20. The density of a substance at 0°C is 10 g/cc and
at 100°C, its density is 9.7 g/cc. The coefficient
of linear expansion of the substance is [2002]
(a) 10–2 (b) 10–2 (a) It will bend towards the right
(c) 10 –3 (d) 10–4 (b) It will bend towards the left
21. A black body, at a temperature of 227°C, radiates (c) It will not bend but shrink
heat at a rate of 20 cal m –2 s–1 . When its (d) It will neither bend nor shrink
temperature is raised to 727°C, the heat radiated 26. If the temperature of a black body increases from
by it in cal m–2s–1 will be closest to : [2003] 7°C to 287°C then the rate of energy radiation
(a) 40 (b) 160 increases by [2007]
(c) 320 (d) 640 4
22. Shown below are the black body radiation curves æ 287 ö
(a)çè ÷ (b) 16
at temperatures T1 and T2 (T2 > T1). Which of 7 ø
the following plots is correct? [2003] (c) 4 (d) 2
27. The wavelength of maximum energy released
during an atomic explosion was 2.93 × 10–10 m.
I T2 I The maximum temperature attained must be,
(a) (b) T2
T1 T1 (Weins constant = 2.93 × 10–3 mK) [2010]
l l (a) 5.86 × 107 K (b) 10–13 K
(c) 10–7 K (d) 107 K
EBD_7100
P-70 Topicwise AIIMS Solved Papers – PHYSICS
28. If the temperature of the sun were to increase 33. A glass flask of volume 1 litre is fully filled with
from T to 2T and its radius from R to 2R, then the mercury at 0ºC. Both the flask and mercury are
ratio of the radiant energy received on earth to now heated to 100ºC. If the coefficient of volume
what it was previously will be [2014] expansion of mercury is 1.82 × 10 –4 /ºC,
(a) 32 (b) 16 volume coefficient of linear expansion of glass
(c) 4 (d) 64 is 10 × 10–6/ºC, the amount of mercury which is
29. A crystal has a coefficient of expansion 13×10– spilted out is [2015]
7 in one direction and 231 × 10–7 in every
(a) 15.2 ml (b) 17.2 ml
direction at right angles to it. Then the cubical
(c) 19.2 ml (d) 21.2 ml
coefficient of expansion is [2014]
34. Steam is passed into 22 g of water at 20°C . The
(a) 462 × 10–7 (b) 244 × 10–7
(c) 475 × 10–7 (d) 257 × 10–7 mass of water that will be present when the water
30. Two identical rods of copper and iron are coated acquires a temperature of 90°C is (Latent heat of
with wax uniformly. When one end of each is steam is 540 cal/gm) [2016]
kept at temperature of boiling water, the length (a) 24.8 gm (b) 24 gm
upto which wax melts are 8.4 cm amd 4.2 cm, (c) 36.6 gm (d) 30 gm
respectively. If thermal conductivity of copper 35. There rods of the same dimensions have thermal
is 0.92, then thermal conductivity of iron is conductivities 3K, 2K and K. They are arranged
(a) 0.23 (b) 0.46 [2015] as shown in fig. with their ends at 100°C, 50°C
(c) 0.115 (d) 0.69 and 20°C. The temperature of their junction is
31. The rate of heat flow through the cross-section
of the rod shown in figure is (T2 > T1 and thermal 50°C [2017]
conductivity of the material of the rod is K) [2015] 2K
100°C 3K q
r1 r2
L
K
T1 T2
Kpr1r2 (T2 - T1 ) 20°C
(a)
L (a) 60° (b) 70°
(c) 50° (d) 35°
Kp(r1 + r2 )2 (T2 - T1 )
(b) 36. A beaker is filled with water at 4°C. At one time
4L the temperature is increased by few degrees
Kp(r1 + r1 ) 2 (T2 - T1 ) above 4°C and at another time it is decreased by
(c) a few degrees below 4°C. One shall observe that:
L
[2017]
Kp(r1 + r1 ) 2 (T2 - T1 ) (a) the level remains constant in each case
(d)
2L (b) in first case water flows while in second
32. The diagram below shows the change in the case its level comes down
length X of a thin uniform wire caused by the (c) in second case water over flows while in
application of str ess F at two different first case its comes down
temperatures T1 and T2. The variation shown (d) water overflows in both the cases
suggests that [2015]
(a) T1 > T2
T2 TYPE B : ASSERTION REASON QUESTIONS
F
T1 Directions for (Qs. 37-50) : These questions consist
(b) T1 < T2
of two statements, each printed as Assertion and
(c) T2 > T1 Reason. While answering these questions, you are
X required to choose any one of the following five
(d) T1 ³ T2
responses.
Thermal Properties of Matter P-71

(a) If both Assertion and Reason are correct and 45. Assertion : A body that is good radiator is also
the Reason is a correct explanation of the a good absorber of radiation at a given
Assertion. wavelength.
(b) If both Assertion and Reason are correct but Reason : According to Kirchhoff’s law the
Reason is not a correct explanation of the absorptivity of a body is equal to its emissivity
Assertion. at a given wavelength. [2005]
(c) If the Assertion is correct but Reason is 46. Assertion : In pressure-temperature (P-T) phase
incorrect. diagram of water, the slope of the melting curve
(d) If both the Assertion and Reason are incorrect. is found to be negative.
(e) If the Assertion is incorrect but the Reason is Reason : Ice contracts on melting to water.
correct. [2005]
37. Assertion : The equivalent thermal conductivity 47. Assertion : For higher temperature the peak
of two plates of same thickness in contact is emission wavelength of a blackbody shifts to
less than the smaller value of thermal lower wavelengths.
conductivity. Reason : Peak emission wavelengths of a black
Reason : For two plates of equal thickness in body is proportional to the fourth-power of
contact the equivalent thermal conductivity is temperature. [2005]
given by : [1997] 48. Assertion : Perspiration from human body helps
1 1 1 in cooling the body.
= + Reason : A thin layer of water on the skin
K K1 K 2
enhances its emissivity. [2006]
38. Assertion : Melting of solid causes no change
49. Assertion : A hollow metallic closed container
in internal energy.
maintained at a uniform temperature can act as a
Reason : Latent heat is the heat required to melt
source of black body radiation.
a unit mass of solid. [1998] Reason : All metals act as black bodies. [2007]
39. Assertion: Fahrenheit is the smallest unit 50. Assertion : A brass tumbler feels much colder
measuring temperature. than a wooden tray on a chilly day.
Reason: Fahrenheit was the first temperature Reason : The thermal conductivity of brass is
scale used for measuring temperature. [1999] more than the thermal conductivity of wood.
40. Assertion: Bodies radiate heat at all temperature. [2008]
Reason: Rate of radiation of heat is proportional Directions for (Qs. 51) : Each of these questions
to the fourth power of absolute temperature. contains an Assertion followed by Reason. Read them
[1999] carefully and answer the question on the basis of
41. Assertion : Woolen clothes keep the body warm following options. You have to select the one that
in winter best describes the two statements.
Reason : Air is a bad conductor of heat. [2002] (a) If both Assertion and Reason are correct and
42. Assertion : Bodies radiate heat at all Reason is the correct explanation of Assertion.
temperatures. (b) If both Assertion and Reason are correct, but
Reason : Rate of radiation of heat is proportional Reason is not the correct explanation of
to the fourth power of absolute temperature. Assertion.
[2002]
(c) If Assertion is correct but Reason is incorrect.
43. Assertion : A tube light emits white light.
Reason : Emission of light in a tube takes place (d) If both the Assertion and Reason are incorrect.
at a very high temperature. [2003] 51. Assertion : Two thin blankets put together are
44. Assertion : It is hotter over the top of a fire than warmer than a single blanket of double the
at the same distance of the sides. thickness.
Reason : Air surrounding the fire conducts more Reason : Thickness increases because of air
heat upwards. [2003] layer enclosed between the two blankets. [2010]
EBD_7100
P-72 Topicwise AIIMS Solved Papers – PHYSICS

Type A : Multiple Choice Questions 6. (a) From the formula,


1. (b) For black body radiation C F - 32
=
E = sT4 5 9
[E is energy radiated per unit time per unit C 140 - 32 108
area, T is temperature of the body] Þ = =
5 9 9
4 4
E 2 æ T2 ö E 2 æ 273 + 287 ö So, fall of temperature in °C is
=ç ÷ Þ =ç ÷
E1 çè T1 ÷ø E1 è 273 + 7 ø 100 – 60 = 40°C
7. (b) Metal appears cool on touching because
4
æ 560 ö 16 heat flows from body (at higher temperature)
=ç ÷ = Þ E2 = 16E1
è 280 ø 1 to iron (at lower temperature). This can
2. (b) We know that, for a source, emitting energy happen only when metal conducts heat.
E at temperature T Wooden surface does not feel cool as it is
E = s T 4 Þ E1 = s(2T)4 non-conductor of heat.
8. (c) Relative Humidity
E sT 4 1
= = Partial pressure of water vapour
E1 16sT 4 16 =
Vapour pressure of water
Now radiation falling on the earth will be
proportional to radiation being emitted so. 0.012 ´ 10 5 ´ 100 12
E 20 = = ´ 100 = 75%
= 5 16
0.016 ´ 10
E1 X
9. (b) Absolute zero is the temperature at which
Here, X is the radiation falling in the earth molecular motion ceases when vibrational
in the latter case. energy exists.
20 1 10. (b) During fusion of solid into liquid some
=
X 16 energy is used up to transform the state of
Þ X = 20 ´ 16 = 320 kcal / m 2 min matter from solid to liquid. Since it does
not increase the kinetic energy of particles,
1
3. (a) Conductivity = there is no increase in the temperature of
Resistivity the material. So, phase transformation takes
Thermal conductivity = 2 place at constant temperature.
1 11. (d) Wool is a bad conductor of heat. It does
Thermal resistivity = = 0.5 not allow heat to pass on from body to
2
4. (c) Thermopile is a combination of surrounding. So, body is kept warm.
thermocouple which generates electrical 12. (b) The process of convection and conduction
energy when one end is kept at higher requires some medium made of material
temperature with respect to the other end. particle for transmission of heat. In vacuum
It is helpful in measuring the temperature there is no material. So, heat travels in
of a hot and radiating body. vacuum by radiation.
5. (a) We know that 13. (d) Applying Wein’s displacement law,
E = sT4
4
l m T = constant
4
E1 æ T1 ö 727 + 273 ö
=ç ÷ = æç ÷ l1m T1 = l 2m T2
E 2 çè T2 ÷
ø è 327 + 273 ø
4 l m ´ 2000 = l ´ 3000
æ 1000 ö 625
=ç ÷ = 2
è 600 ø 81 l = lm
3
Thermal Properties of Matter P-73

14. (a) Sublimation is conversion of a solid directly Coefficient of linear exapansion


into gas by heat.
g 3 ´ 10-4
15. (a) Applying Wein’s displacement law, a= = = 10-4
l m T = constant 3 3
21. (c) We know that
l'm T2 510 T2
= Þ = E1 T14
l''m T1 350 T1 E = sT 4 Þ =
E 2 T2 4
T2 51 T 35
= Þ 1 = = 0.68 E (727 + 273)4 (1000) 4
T1 35 T2 51 Þ = =
16. (b) Coefficient of volume expan sion of 20 (227 + 273)4 (500)4
glycerine 4
E1 æ 2 ö 16
= 0.000597 per°C =ç ÷ =
Coefficient of volume expansion of glass 20 è 1 ø 1
= 3 × 0.000009 = 0.000027per°C Þ E1 = 20 ´ 16 = 320 cal m–2 s–1
Apparent coefficient of volume expansion 22. (a) From Wein’s displacement law,
= 0.000597 - 0.000027 lmT = constant i.e.
= 0.00057per°C at greater temperature lm will be small lm is
the wavelength of the radiaton having
17. (b) The colour of the star in dicates highest intensity. In figure (a) curve
its temperature. Higher the wavelength representing T2 has lm smaller than that
emitted lower will be its temperature. for T1 so, (a) is the right answer.
This is from Wein’s displacement Law,
lmT = constant. 23. (b) E = sT4 , here, E is energy radiated per
A blue coloured star will have high
temperature than red coloured star. unit area. Total energy emitted = sT 4 ´ A
18. (c) We know that Total energy emitted by sun after
expansion
E = sT4 4
Where E is rate of emission of radiation of æTö
= s ´ ç ÷ ´ 100 ´ 100 A
a body at temperature T. è2ø
E1 = s(27 + 273) 2 [When radius becomes 100 times,
area becomes 1002 times]
E 2 = s(127 + 273)2 1 4
= AsT 4 ´ ´ 100 ´ 100 = 625 ´ sT A
E1 (300) 4 81 16
= = So, total energy emitted is 625 times.
E 2 (400) 4 256
24. (b) Black has greatest emissivity and greatest
19. (a) For black body radiation absorbtivity as compared with other
colours. At 2000°C it will have greatest
E = sT 4 or E µ T 4
emissivity so it will glow brightest.
Rate of emission of energy µ (300) 4 25. (b) As coefficient of thermal expansion of X is
20. (a) The coefficient of volume expansion, more. On cooling, it will shrink more. So
the strip will bend towards the left.
Dr 26. (b) By Stefan's law, energy radiated per sec by
g=
r1 ´ Dt a black body is given by E = AsT4
where A = area of black body, s = Stefan's
r1 - r2 10 - 9.7 constant. For a black body at temperature
= = = 3 ´ 10-4
r1 (T2 - T1 ) 10(100 - 0) T1, E1 = AsT14 , at T2, E2 = AsT4
(Since A,s all same)
EBD_7100
P-74 Topicwise AIIMS Solved Papers – PHYSICS

4
E 2 T2 100°C
\ <
E1 T14 100°C Water
Steam (H1= m × 540)
æ T2 ÷ö4
ç
Þ E2 = çç ÷÷ E1
çè T ÷ø
1
T2 = 287°C = 287 + 273 = 560 K, [H2= m ×1× (100× 90)]
T1 = 7°C = 7 + 273 = 280 K,
æ 560 ö÷4
\ E2 = ççç ÷ E = 24E1 = 16 E1
è 280 ø÷ 1
90°C
Water
\ Rate of energy radiated increases by
16 times. Heat gained by water (20°C) to raise it's
27. (d) lm × T = b (Wein's displacement Law) temperature upto 90°C = 22 × 1× (90 – 20)
Hence , in equilibrium, heat lost = Heat gain
2.93 ´ 10-3
T= = 107 K Þ m × 540 + m × 1 × (100 – 90)
-10
2.93 ´ 10 = 22 × 1 × (90 – 20)
28. (d) E = σAT 4 Þ m = 2.8 gm
A α R 2 \ Eα R 2 T 4 The net mass of the water present in the
mixture = 22 +2.8 =24.8 gm.
E R 2 T4 dQ DT
\ 2 = 2 2 35. (b) = KA
E1 R12 T14 dt L
put R 2 = 2R, R1 = R æ dQ ö 3KA
For the first rod, ç ÷ = (100 – q)
T2 = 2T, T1 = T è dt ø1 L
æ dQ ö A
E 2 (2R) 2 (2T) 4 Similarly, ç ÷ = 2K (q – 50)
Þ = = 64 è dt ø 2 L
E1 R 2T 4 æ dQ ö A
29. (a) g = a1 + a 2 + a 3 ç dt ÷ = K L (q – 20)
è ø3
= 13 ´ 10 - 7 + 231 ´ 10 - 7 + 231 ´ 10 - 7 æ dQ ö æ dQ ö æ dQ ö
= 475 ´ 10 - 7
Now, ç ÷ =ç ÷ +ç ÷
è dt ø1 è dt ø2 è dt ø3
K1 l12 Þ 3 (100 – q) = 2 (q – 50) + (q – 20)
30. (a) Use K = 2 Þ q = 70°
2 l2 36. (d) water expands on both sides of 4 °C.
31. (a) reff = r1r2 Type B : Assertion Reason Questions
dQ KA(T2 - T 1 ) Kpr1r2 (T2 - T1 ) 37. (a) For equivalent thermal conductivity, the
= =
dt L L relation is
32. (a) When same stress is applied at two 1 1 1
different temperatures, the increase in = + ; If K1 = K 2 = K
K R K1 K 2
length is more at higher temperature. Thus
1 1 1 2 K
T1 > T2. = + = Þ KR =
33. (a) DV = V0 ( g m - g g )DT KR K K K 2
Which is less than K.
= 1[1 .82 ´ 10 -4 - 3 ´ (10 ´ 10 -6 )] 100 If K1 > K 2 suppose K1 = K 2 + x
= 1[1 . 82 ´ 10 - 4 - 0 .3 ´ 10 - 4 )] 100 1 1 1 K + K1
= 15.2 ml = + = 2
K K1 K 2 K1K 2
34. (a) Let m g of steam get condensed into water
1 K2 + K2 + x 2
(By heat loss). This happens in following Þ = Þ K = K 2 + K 2x
two steps. K (K 2 + x)K 2 2K 2 + x
Thermal Properties of Matter P-75

K 22 + K 2 x 45. (c) According to Kirchoff’s law


Now, K 2 - K = K 2 - el
2K 2 + x = El .
al
2K 2 2 + K 2 x - K 2 2 - K 2 x
= Here, E l is emissivity of black body which
( 2K 2 + x )
is constant, so, el µ a l . It means good
K 22
= = positive emitter are good absorber of radiation.
2K 2 + x
46. (a) The slope of melting curve in phase
So, K2 > K, so the value of K is smaller than
diagram is negative for water as due to
K2 and K1.
38. (e) Melting of solid causes change in its increase in temperature vapour, pressure
internal energy. of ice decreases. The Reason is that ice
Latent heat is the heat required to melt one contracts on melting.
unit mass of solid. Option (e) is correct. 47. (c) According to Wein’s displacement law,
39. (e) Here, Assertion is incorrect & Reason is lmT = constant
correct. The temperature difference Naturally, when T increases lm decreases.
between boiling point of water and freezing lm is peak emission wavelength
point of water has been divided into 100 and E = sT4
parts in ºC scale, 180 parts in Fahrenheit Here, E is energy being radiated per unit
scale, 80 parts in Reaumer scale and 212 area per unit time.
parts in Rankine scale. So, Rankine scale is 48. (c) Perspiration envolves exchange of heat
the smallest unit. Assertion is incorrect. from body to surrounding. Water takes heat
Fahrenheit was the first temperature scale from the body and gets converted into
used for measuring temperature. So, the vapour. Hence, body cools down.
Reason is correct. A thin layer of water on the skin will reduce
40. (a) Bodies radiate heat at all temperatures. It is
rather than increase its emissivity. So,
true.
Assertion is correct but Reason is incorrect.
We also know that, E µ T4.
So, Reason is also correct and its explains 49. (d) A perfect black body is one which absorbs
Assertion. all heat radiations (whatever be the
41. (a) Woolen clothes keep the body warm. The wavelength) incident on it. No natural
air trapped in clothes are bad conductor of object is a perfect black body. Best
heat. approximations are lamp black and
42. (e) Bodies radiate heat at all temperature. The platinum black. Fery's black body is a
rate of radiation of heat is proportional to double walled metallic sphere coated with
the fourth power of absolute temperature lamp black on the inside and nickel on
is outside. It has a narrow opening opposite
E = sT4 which is Stefan’s Boltzmann’s law a conical projection inside.
43. (c) In tube light, the gas contains vapour of \ Both Assertion and Reason are incorrect.
metals. In metallic atoms, electronic 50. (a) Brass is a metal and good conductor. On a
transition occurs due to which light of a cold day, when brass tumbler is touched
particular wavelength is emitted. So heat transfers from our body to brass, since
emission of white light is due to electronic our body looses heat so the tumbler feels
transition and not due to vibration of atoms cold. On the other hand, transfer of heat
as in hot substances. So, Assertion is from our body to wood is slow and less,
correct but Reason is incorrect. hence wooden tray appears warm.
44. (c) It is hotter over the top of a fire. It is because 51. (c) Two thin blankets put together are more
of convection current established over the warm because an insulating layer of air (as
fire. As air warms up, its density decreases air is good insulator of heat) is enclosed
as a result of which it goes up and makes
between two blankets due to which it gives
upper layer of air hot.
more warmness.
The Reason is incorrect.
EBD_7100
P-76 Topicwise AIIMS Solved Papers – PHYSICS

11 Thermodynamics

TYPE A : MULTIPLE CHOICE QUESTIONS 2P0 B C


1. A sample of gas expands from volume V1 to V2.
The amount of work done by the gas is greatest,
when the expansion is : [1998] P
A D
(a) isothermal (b) adiabatic T0
(c) isobaric (d) all of these
2. In an adiabatic process the quantity which
remains constant is: [1999] V0 V 2V0
(a) total heat of system (a) 15% (b) 50% [2004]
(b) temperature (c) 20% (d) 25%
(c) volume 8. When you make ice cubes, the entropy of water
(d) pressure (a) does not change [2006]
3. During the adiabatic expansion of two moles of (b) increases
a gas the internal energy of a gas is found to (c) decreases
decrease by 2 joule. The work done on gas during (d) may either increase or decrease depending
on the process if used
the process will be equal to [2000]
9. In an adiabatic change, the pressure and
(a) – 2 J (b) 3 J temperature of a monoatomic gas are related as
(c) 1 J (d) 2 J P µ TC, where C equals [2007]
4. Which one of the following is not a 2 5
thermodynamical co-ordinate ? [2001] (a) (b)
5 2
(a) V (b) R 3 5
(c) T (d) P (c) (d)
5 3
5. The latent heat of vaporization of water is 2240 10. Two rigid boxes containing different ideal gases
J. If the work done in the process of vaporization are placed on a table. Box A contains one mole
of 1g is 168 J, then increase in internal energy is of nitrogen at temperature T 0, while box B
(a) 1940 J (b) 2072 J [2002] contains one mole of helium at temperature
(c) 2240 J (d) 2408 J æ7ö
ç ÷T0 . The boxes are then put into thermal
6. The volume of a gas is reduced adiabatically to è3ø
(1/4) of its volume at 27ºC. If g = 1.4. The new contact with each other, and heat flows between
temperature will be : [2002] them until the gases reach a common final
0.4 0.4 temperature (ignore the heat capacity of boxes).
(a) 300 × (4) K (b) 150 × (4) K
Then, the final temperature of the gases, T f in
(c) 250 × (4)0.4 K (d) none of these
terms of T0 is [2008]
7. N moles of a monoatomic gas is carried round 3 7
the reversible rectangular cycle ABCDA as (a) Tf = T0 (b) Tf = T0
7 3
shown in the diagram. The temperature at A is
3 5
T0. The thermodynamic efficiency of the cycle (c) Tf = T0 (d) Tf = T0
2 2
is :
Thermodynamics P-77

11. The temperature-entropy diagram of a reversible Pf


engine cycle is given in the figure. Its efficiency (d) W = -nRT log
Pi
is [2008] 15. Two moles of a monoatomic ideal gas is confined
T in a container and is heated such that its
temperature increases by 10°C. The approximate
2T0
change in its internal energy is [2011]
(R = 8.31 J/mole-K)
T0 (a) + 250 joules (b) + 350 joules
S (c) – 250 joules (d) + 450 joules
S0 2S0
16. If DQ and DW represent the heat supplied to the
1 1 system and the work done on the system
(a) (b)
4 2 respectively, then the first law of
2 1 thermodynamics can be written as [2013]
(c) (d)
3 3 (a) DQ = DU + DW
12. When a system is taken from a state i to f along (b) DQ = DU – DW
the path iaf (as shown in the figure). Q = 50 cal (c) DQ = DW – DU
and W = 20 cal; along path ibf, Q = 36 cal.[2009] (d) DQ = –DW – DU
a 17. A system goes from A to B via two processes I
f
and II as shown in figure. If DU1 and DU2 are the
P changes in internal energies in the processes I
and II respectively, then [2013, 2014]
p II
i b

V A B
(i) What is W along path ibf ? I
(ii) If W = 13 cal for path fi, what is Q for the
v
path fi ?
(iii) Take Eint, i = 10 cal then what is Eint,f ? (a) relation between DU1 and DU2 can not be
determined
(a) 30, 20, 40 cal (b) 6, – 43, 40 cal
(b) DU1 = DU2
(c) 10, – 20, 30, cal (d) 15, 35, 25 cal
(c) DU1 < DU2
13. The change in the entropy of a 1 mole of an ideal (d) DU1 > DU2
gas which went through an isothermal process 18. Figure below shows two paths that may be taken
from an initial state (P1, V1,T) to the final state
by a gas to go from a state A to a state C.
(P2, V2,T) is equal to [2010]
(a) zero (b) R lnT
P B C
V V 6×104 Pa
(c) R ln 1 (d) R ln 2
V2 V1
14. An ideal gas is subjected to an isothermal
expansion such that its volume changes from Vi 2×104 Pa
A
to Vf and pressure from Pi to Pf. The work done
on the gas is : [2011]

2 × 10 3 m3 4 × 10–3 m3
Vf
(a) W = + nRT log V
Vi In process AB, 400 J of heat is added to the
V system and in process BC, 100 J of heat is added
(b) W = - nRT log f
Vi to the system. The heat absorbed by the system
P in the process AC will be [2016]
(c) W = nRT log f (a) 500 J (b) 460 J
Pi
(c) 300 J (d) 380 J
EBD_7100
P-78 Topicwise AIIMS Solved Papers – PHYSICS
19. The internal energy change in a system that 26. Assertion : Reversible systems are difficult to
has absorbed 2 kcals of heat and done 500 J of find in real world. [2005]
work is [2017] Reason : Most processes are dissipative in
(a) 6400 J (b) 5400 J nature.
(c) 7900 J (d) 8900 J 27. Assertion : Air quickly leaking out of a balloon
20. In a heat engine, the temperature of the source becomes cooler.
and sink are 500 K and 375 K. If the engine Reason : The leaking air undergoes adiabatic
consumes 25 × 105 J per cycle, the work done expansion. [2005]
per cycle is [2017] 28. Assertion : In an isolated system the entropy
(a) 6.25 × 105 J (b) 3 × 105 J increases.
(c) 2.19 × 105J (d) 4 × 104 J Reason : The processes in an isolated system
are adiabatic. [2006]
TYPE B : ASSERTION REASON QUESTIONS 29. Assertion : The Carnot cycle is useful in
Directions for (Qs. 29-32) : These questions consist understanding the performance of heat engines.
of two statements, each printed as Assertion and Reason : The Carnot cycle provides a way of
Reason. While answering these questions, you are determining the maximum possible efficiency
required to choose any one of the following five achievable with reservoirs of given temperatures.
responses. [2006]
(a) If both Assertion and Reason are correct and 30. Assertion : When a glass of hot milk is placed in
the Reason is a correct explanation of the a room and allowed to cool, its entropy decreases.
Assertion. Reason : Allowing hot object to cool does not
(b) If both Assertion and Reason are correct but violate the second law of thermodynamics.
Reason is not a correct explanation of the [2006]
Assertion. 31. Assertion : In free expansion of an ideal gas, the
(c) If the Assertion is correct but Reason is incorrect. entropy increases.
(d) If both the Assertion and Reason are incorrect. Reason : Entropy increases in all natural
(e) If the Assertion is incorrect but the Reason is processes. [2007]
correct. 32. Assertion : The isothermal curves intersect each
21. Assertion : In isothermal process whole of the other at a certain point.
heat supplied to the body is converted into Reason : The isothermal changes takes place
internal energy. rapidly, so the isothermal curves have very little
Reason : According to the first law of slope. [2008]
thermodynamics : [1997] Directions for (Qs. 33-34) : Each of these questions
DQ = DU + pDV contains an Assertion followed by Reason. Read them
22. Assertion : In adiabatic compression, the carefully and answer the question on the basis of
internal energy and temperature of the system following options. You have to select the one that
get decreased. best describes the two statements.
Reason : The adiabatic compression is a slow (a) If both Assertion and Reason are correct and
process. [2001] Reason is the correct explanation of Assertion.
23. Assertion : The isothermal curves intersect each (b) If both Assertion and Reason are correct, but
other at a certain point. Reason is not the correct explanation of
Reason : The isothermal change takes place Assertion.
slowly, so, the isothermal curves have very little (c) If Assertion is correct but Reason is incorrect.
slope. [2001] (d) If both the Assertion and Reason are incorrect.
24. Assertion : When a bottle of cold carbonated 33. Assertion : Adiabatic expansion is always
drink is opened, a slight fog forms around the accompanied by fall in temperature.
opening. Reason : In adiabatic process, volume is
Reason : Adiabatic expansion of the gas causes inversely proportional to temperature.
lowering of temperature and condensation of [2011, 2013, 2014]
water vapours. [2003] 34. Assertion : The heat supplied to a system is
25. Assertion : Thermodynamic process in nature always equal to the increase in its internal energy.
are irreversible. Reason : When a system changes from one
Reason : Dissipative effects cannot be thermal equilibrium to another, some heat is
eliminated. [2004] absorbed by it. [2017]
Thermodynamics P-79

Type A : Multiple Choice Questions = 2P0V0 – P0V0 [Net work done in a cyclic
process is area of the loop]
1. (a) We know that = P0V0
Q = DE + work done by gas
P V
Work done by gas = Q – DE So, efficiency = 0 0 ´ 100 = 50%
Work done by gas is maximum when DE is 2P0 V0
minimum. For Isothermal Change, DE = 0. dQ
8. (c) dS =
So, for isothermal expansion work done is T
maximum. In freezing process dQ is negative so
2. (a) In adiabatic process there is no exchange entropy decreases.
of heat with the surroundings, so total heat 5
of the system remains constant. 9. (b) In adiabatic process, Pg– 1µ Tg where g =
3
3. (a) Gas is expanding at the cost of internal for monoatomic gas \ P µ Tg/(g–1)
energy of the gas. Work done by the
gas is 2 joule. So, work done on the gas g 5/3 5/3 5
\ C= = = =
= – 2 joule. g -1 5 / 3 -1 2 / 3 2
4. (b) R is a constant term. To define a thermo 10. (c) Heat lost by He = Heat gained by N2
dynamic state of a gas we use any two of
n1Cv1 DT1 = n 2 Cv2 DT2
three physical quantities P, V & T. Following
ratio is always constant 3 é7 ù 5
R ê T0 - Tf ú = R [Tf - T0 ]
PV 2 ë3 û 2
= R (constant)
T 7T0 - 3Tf = 5Tf - 5T0
So,if we change P & V, T will automatically
change itself to make the ratio constant. 12
Þ 12T0 = 8Tf Þ Tf = T0
5. (b) We know that for first law of 8
thermodynamics, equation is 3
Q = DE + DW Þ Tf = T0
2
Here, Q = 2240; DE = ? DW =168 11. (d) T
DE = Q - DW = 2240 - 168 = 2072 J
6. (a) For adiabatic change, the equation is
2T0
TV g-1 = constant Q1
Q3
T1V1g-1 = T2 V2 g-1 T0
g-1 Q2
æV ö
(27 + 273)V1g-1 = T2 ç 1 ÷ S
è 4 ø S0 2S0
T2 ´ V1g-1 1 3
300 ´ V1 g-1 = Þ T2 = 300 ´ 4 g-1 Q1 = T0S0 + T0S0 = T0S0
4 g-1 2 2
T2 = 300 ´ 41.4 -1 = 300 ´ 4 0.4 K Q2 = T0(2S0 – S0) = T0S0 and Q 3 = 0
7. (b) Heat absorbed = Work done by gas at W Q1 - Q 2
constant pressure h= =
Q1 Q1
= 2P0 (2V0 - V0 ) = 2P0 V0
Q2 TS 1
Net work done by the gas = 1- = 1- 0 0 =
Q1 3
= Workdone by the gas T0S0 3
– Workdone on the gas 2
EBD_7100
P-80 Topicwise AIIMS Solved Papers – PHYSICS
12. (b) For path iaf 17. (b) Change in internal energy do not depend
Q = 50 cal. upon the path followed by the process. It
W = 20 cal. only depends on initial and final states i.e.,
According ot I law of thermodynamics,
dQ = dU + dW DU1 = DU2
or dU = dQ – dW = 50 – 20 = 30 cal. 18. (b) In cyclic process ABCA
(i) For path iaf Qcycle = Wcycle
Q = 36 cal. QAB + QBC + QCA = ar. of DABC
W=?
1
dU = 30 cal (since internal energy depends + 400 + 100 + QC®A = (2 × 10–3) (4 × 104)
only on the initial and final positions of the 2
system). Þ QC ® A = – 460 J
\ W = Q – dU = 36 – 30 = 6 cal. Þ QA ® C = + 460 J
(ii) W = –13 cal. 19. (c) According to first law of thermodynamics
dU = –30 cal.
Q = DU + W
Q= ?
\ Q = dU + W = – 43 cal. DU = Q – W
(iii) Eint, f = Eint, i + DU = 10 cal + 30 cal. = 40 = 2 × 4.2 × 1000 – 500
cal. = 8400 –500
13. (d) Workdone by n moles of a gas when its = 7900 J
volume changes from V1 to V2 is, 20. (a) Here, T1 = 500 K, T2 = 375 K
V Q1 = 25 × 105 J
W = nRT loge 2
V1 T2 375
For, n = 1 \ h = 1- =1- = 0.25
T1 500
V
W = RT 1n 2 W = hQ = 0.25 × 25 × 105 = 6.25 × 105 J
V1
For an isothermal process, DU = 0 Type B : Assertion Reason Questions
\ DQ = DU + W = 0 + RT1n V2 21. (a) DQ = DU + PDV [PV = nRT
V1 DQ = DU + nRDT PDV = nRDT
\ Entropy = D Q V 2 For isothermal change DT = 0
= R1n .
T V1 \ DQ = DU
14. (d) The work done in expansion of gas
V P In other words whole of heat supplied is
W = nRT ln f = nRT ln i converted into internal energy.
Vi Pf
22. (d) Adiabatic compression is a fast process.
The work done on the gas
There is rise in temperature and also
æP ö
= - nRT ln ç f ÷ increase in internal energy.
è Pi ø So, both are false.
3
15. (a) DE = n R D T 23. (e) Isothermal curves have slope which is equal
2
3 P
to . It can be calculated as follows
= 2 × ´ 8.31 ´ 10 = 250 J V
2
16. (b) From FLOT DQ = DU + DW PV = RT
Q Heat supplied to the system so Differentiating,
DQ ® Positive PdV + VdP = 0
and work is done on the system so dP P
DW® Negative - =
dV V
Hence +DQ = DU - DW
Thermodynamics P-81

Now, if they cut each other at certain point, 29. (a) Carnot cycle represents process of an ideal
they will have different slope at the same heat engine which has maximum efficiency
point (for same value of P & V). So, they of conver sion of heat en ergy into
can not cut each other at some point. mechanical energy. So, A is right. The
P efficiency of a Carnot cycle depends only
Reason is true, slope is .
V on the temperature of heat reservoirs is
P source and sink. So, reason is right and
For adiabatic curve slope is g times .
V reason explains assertion.
24. (a) In cold carbonated drink, gas is dissolved 30. (b) A body cools, its entropy decreases as
under pressure, when pressure is released
expansion of gas occurs due to which gas dQ
dS = and dQ is –ve, dS is also –ve.
cools down and temperature falls. T
Condensation of water vapour occurs. R is also true. Second law states that
25. (a) Most of the phenomenon in nature is entropy of the universe increases. Universe
irreversible. A process becomes irreversible includes both system and surroundings. R
in case some energy is converted into heat does not explain A.
energy. This is known as dissipative effect. 31. (c) In free expansion of an ideal gas, work done
When there is dissipative effect, process comes from internal energy of the gas and
becomes irreversible. since randomness increases or Gibbs free
26. (a) In any process some energy is found to be energy increases so we can say entropy
converted into heat (dissipative in nature) increases.
due to which process becomes irreversible.
32. (d) As isothermal processes are very slow and
27. (a) Air cools down due to adiabatic expansion
as air has to do work against external so the different isothermal curves have
pressure at the cost of its internal energy. different slopes, they cannot intersect each
28. (b) In an unisolated system, heat may enter into other.
or escape from the system due to which 33. (c)
entropy may increase or decrease but for 34. (d) According to first law of thermodynamics,
isolated system we do not consider exchange DQ = DU + DW = DU +PDV. If heat is
of heat, so, in this case entropy will always supplied in such a manner that volume does
increase as the process is spontaneous. not change DV = 0,i.e., isochoric process,
An adiabatic process involves no exchange then whole of the heat energy supplied to
of heat. We also define isolated system as the system will increase internal energy
having no exchange of heat with the only. But, in any other process it is not
surrounding so it process in an isolated possible.
system are adibatic.
Also heat may be adsorbed or evolved
The two statements are independently
when state of thermal equilibrium changes.
correct but not co-related.
EBD_7100
P-82 Topicwise AIIMS Solved Papers – PHYSICS

12 Kinetic Theory

TYPE A : MULTIPLE CHOICE QUESTIONS 6. N1 molecules of a gas at temperature T1 are


mixed with N2 molecules at temperature T2. The
1. The average kinetic energy of a gas molecule at
resulting temperature of the mixture gas is
27°C is 6.21 × 10–21 J, then its average kinetic
(a) ( 1
energy at 227°C is : [1999] T - T2 )
[2010]
(a) 10.35 × 10–21 J (b) 11.35 × 10–21 J 2
(c) 52.2 × 10–21 J (d) 5.22 × 10–21 J
(b)
( N1T1 - N 2T2 )
2. An ideal gas at 27°C is compressed adiabatically
( N1 + N 2 )
8
to its original volume [TVg–1 = constant] (c)
( N1T1 + N 2T2 )
27
5
( N1 + N 2 )
and g = , then the rise in temperature will be: ì ( N + N 2 ) ïü ì T1 + T2 ü
3 (d) ïí 1 ýí ý
(a) 480°C (b) 450°C [1999] 2
îï þï î 2 þ
(c) 375°C (d) 225°C 7. At what temperature the molecules of nitrogen
3. vrma, vav and vmp are root mean square, average will have the same rms velocity as the molecules
and most probable speeds of molecules of a gas of oxygen at 127° C [2011]
obeying Maxwellian velocity distribution. Which
(a) 457° C (b) 273° C
of the following statements is correct ? [2005]
(c) 350° C (d) 77° C
(a) vrms < vav < vmp (b) vrms > vav > vmp
(c) vmp < vrms < vav (d) vmp > vrms < vav 8. If liquefied oxygen at 1 atmospheric pressure is
heated from 50 K to 300 K by supplying heat at
4. Let v , vrms and vp respectively denote the mean constant rate. The graph of temperature vs time
speed, root mean square speed and most will be [2012]
probable speed of the molecules in an ideal
monoatomic gas at absolute temperature T. The
mass of the molecule is m. Then [2010] T
(a) no molecule can have a speed greater than
(a) (b) T
( 2vrms )
(b) no molecule can have a speed less than t t
vp
( 2)
(c) v < vp < vrms (c) T (d) T
(d) the average kinetic energy of the molecules
3 2 t t
is (mv p ) 9. The figure shows the volume V versus
4
5. Two identical containers A and B with frictionless temperature T graphs for a certain mass of a
pistons contain the ideal gas at the same perfect gas at two constant pressures of P1 and
temperature and the same volume V. The mass P2 . What inference can you draw from the
of the gas in A is mA and in B is mB. The gas in graphs? V P2 [2014]
each cylinder is now allowed to expand
isothermally to the same final volume 2V. The (a) P1 > P2 P1
changes in pressure in A and B are found to be (b) P1 < P2 q2
Dp and 1.5DP respectively. Then [2010] (c) P1 = P2 q1 T
(a) 4mA = 9mB (b) 2mA = 3mB (d) No in ference can be drawn due to
(c) 3mA = 2mB (d) 9mA = 4mB insufficient information.
Kinetic Theory P-83

10. A gas mixture consists of molecules of type 1, 2 14. Assertion : The root mean square and most
and 3, with molar masses m1 > m2 > m3. vrms and probable speeds of the molecules in a gas are
K are the r.m.s. speed and average kinetic energy the same.
of the gases. Which of the following is true?[2015] Reason : The Maxwell distribution for the speed
(a) (v rms ) 1 < (v rms ) 2 < (v rms ) 3 and of molecules in a gas in symmetrical. [2006]
( K )1 = ( K ) 2 = ( K )3 Directions for (Qs. 15-20) : Each of these questions
(b) (v rms ) 1 = (v rms ) 2 = (v rms ) 3 and contains an Assertion followed by Reason. Read them
carefully and answer the question on the basis of
( K )1 = ( K ) 2 > ( K )3 following options. You have to select the one that
(c) (v rms ) 1 > (v rms ) 2 > (v rms ) 3 and best describes the two statements.
( K )1 < ( K ) 2 > ( K ) 3 (a) If both Assertion and Reason are correct and
(d) (v rms ) 1 > (v rms ) 2 > (v rms ) 3 and Reason is the correct explanation of Assertion.
( K )1 < ( K ) 2 < ( K )3 (b) If both Assertion and Reason are correct, but
11. A thermally insulated vessel contains an ideal Reason is not the correct explanation of
gas of molecular mass M and ratio of specific Assertion.
heats g. It is moving with speed v and its suddenly (c) If Assertion is correct but Reason is incorrect.
brought to rest. Assuming no heat is lost to the (d) If both the Assertion and Reason are incorrect.
surroundings, its temperature` increases by Cp
( g –1) gMv 2 15. Assertion : The ratio of for an ideal diatomic
(a) Mv 2 K (b) K [2016] Cv
2gR 2R gas is less than that for an ideal monoatomic gas
( g –1) ( g –1) 2 (where Cp and Cv have usual meaning).
(c) Mv 2 K (d) 2( 1) Mv K Reason : The atoms of a monoatomic gas have
2R g+ R
less degrees of freedom as compared to
12. If the root mean square velocity of the molecules molecules of the diatomic gas. [2009]
of hydrogen at NTP is 1.84 km/s. Calculate the 16. Assertion : The total translational kinetic energy
root mean square velocity of oxygen molecule of all the molecules of a given mass of an ideal
at NTP, molecular weight of hydrogen and gas is 1.5 times the product of its pressure and
oxygen are 2 and 32 respectively [2017] its volume.
(a) 1.47 km/sec (b) 0.94 km/s Reason : The molecules of a gas collide with
(c) 1.84 km/s (d) 0.47 km/sec each other and the velocities of the molecules
change due to the collision. [2013]
TYPE B : ASSERTION REASON QUESTIONS
17. Assertion : Mean free path of a gas molecules
Directions for (Qs. 13-14) : These questions consist varies inversely as density of the gas.
of two statements, each printed as Assertion and Reason : Mean free path varies inversely as
Reason. While answering these questions, you are pressure of the gas. [ 2014]
required to choose any one of the following five 18. Assertion : At a given temperature the specific
responses. heat of a gas at constant volume is always greater
(a) If both Assertion and Reason are correct and than its specific heat at constant pressure.
the Reason is a correct explanation of the Reason : When a gas is heated at constant
Assertion. volume some extra heat is needed compared to
(b) If both Assertion and Reason are correct but that at constant pressure for doing work in
Reason is not a correct explanation of the expansion. [2015]
Assertion. 19. Assertion : One mole of any substance at any
(c) If the Assertion is correct but Reason is incorrect. temperature or volume always contains 6.02 × 1023
(d) If both the Assertion and Reason are incorrect. molecules.
(e) If the Assertion is incorrect but the Reason is Reason : One mole of a substance always refers
correct. to S.T.P. conditions. [2016]
13. Assertion : For a gas atom the number of 20. Assertion : Air pressure in a car tyre increases
degrees of freedom is 3. during driving.
Cp Reason : Absolute zero temperature is not zero
Reason : =g [2000] energy temperature. [2017]
Cv
EBD_7100
P-84 Topicwise AIIMS Solved Papers – PHYSICS

Type A : Multiple Choice Questions and average kinetic energy of a gas


molecule
1. (a) Average kinetic energy of gas molecules
µ Temperature (Absolute) 1 2
Ek = mvrms
2
K.E.(at 227°C) 273 + 227 500 5
= = = 2
K.E.(at 27°C) 273 + 27 300 3 1 æ 3 ö 1 3 3
Ek = m ç v y ÷ = m ´ v 2p = mv 2p
5 ç
2 è 2 ø ÷ 2 2 4
K.E. (227º) =´ 6.21 ´10 - 21 J
3 5. (c) The initial pressure in the two containers
= 10.35 × 10–21 J will be
2. (c) Applying the formula nA RT æ RT ö
PA = = mA ç ÷
T1V1g-1 = T2 V2 g-1
V è MV ø
g -1
T1 æ V2 ö n RT æ RT ö
=ç ÷ PB = B = mB ç ÷
T2 çè V1 ÷ø V è MV ø
5/ 3 - 1
After isothermal expansion, pressure will
æ 27 + 273 ö æ 8 ö g-1 æ 8 ö be
Þ = =ç ÷
çè T2 ÷ø çè 27 ÷ø è 27 ø nA RT æ RT ö
PA' = = mA ç
2 2V è 2 MV ÷ø
´3
300 æ 2ö3 4
or, =ç ÷ = nB RT æ RT ö
T2 è 3 ø 9 PB' = = mB ç ÷
2V è 2MV ø
9 ´ 300
T2 = = 675°K æ RT ö
\ -D PA = PA – PA' = m A ç
4 è 2MV ÷ø
t °C = 675 - 273 = 402°C
Rise in temperature = 402 – 27 = 375°C ' æ RT ö
\ -DPB = PB - PB = mB ç ÷
3kT 2kT è 2 MV ø
3. (b) v rms = ; v mp = But DPA = DP and - DPB = 1.5DP
m m
8kT -DPA 1 -DPB
vav = So, = Þ -DPA =
pm -DPB 1.5 1.5
So, v rms > vav > vmp æ RT ö mB æ RT ö
mA ç ÷= ç ÷
3RT è 2MV ø 1.5 è 2MV ø
4. (d) vrms = 10mB
m or mA =
15
8 RT 2.5RT \ 3mA = 2mB
v= =
pm m
æ3 ö æ3 ö
2 RT 6. (c) çè kT1÷ø ´ N1 + çè kT2 ÷ø ´ N 2
2 2
and v p =
m 3
From these expressions, we can see that = ( N1 + N 2 ) ´ kT
2
v p < v < vrms T1N1+T2N2 = (N1+N2)T
3 N1T1 + N 2T2
Again, vrms = v p \ T=
2 N1 + N 2
Kinetic Theory P-85

7. (d) Rms velocity of gas is 11. (c) As no heat is lost,


1 Loss of kinetic energy = gain of internal
æ RT ö 2 energy of gas
vrms = 1.73 ç ;
è M ÷ø
1 2
M = molecular mass mv = nCV DT
For oxygen, M = 16 × 2, 2
T = 127°C = 127 + 273 = 400 K 1 2 m R
For nitrogen, M = 17 × 2, T = ? Þ mv = × DT
1 1 2 M g –1
Þ 1.73 æç
RT ö 2 æ RT ö 2
÷ = 1.73 ç mv 2 (g –1)
è M øO è M ÷ø N Þ DT = K
2 2
2R
1 1

Þ æç T ÷ = æç T ÷
ö2 ö2 12. (d) (crms )H 2 =1.84 km/s, (crms )O2 = ?
è MøO è MøN
2 2 M H2 = 2, M O2 = 32
M N2 28 Þ Rms velocity,,
Þ TN2 = TO2 = ´ 400
M O2 32 3RT
crms =
M
7
= ´ 400 cH 2 M O2
8 \ =
7 cO 2 M H2
Þ TN2 = ´ 400 = 350K
8 1.84 32
TN2 = 350K – 273K = 77°C Þ = =4
CO2 2
8. (c) Q = mcDT 1.84
Q = mc ( T – T0) ......(i) Þ CO2 = = 0.46 km/s
4
Q = Kt whereas K is heating rate
\ from 50 to boiling temperature, T Type B : Assertion Reason Questions
increases linearly. 13. (b) For a gas atom no. of degree of freedom is
At vaporization, equation is Q = mL 3 because it can have translatory motion in
so, temperature remains constant till three directions, along X-axis, Y-axis, and
vaporisation is complete Z-axis.
After that, again Eqn (i) is followed and Cp
temperature increases linearly = g is also correct but it is not the
Cv
9. (b) Q q1 < q2 Þ tan q1 < tan q2
Reason for Assertion given.
æV ö æV ö 14. (d) Both Assertion and Reason are incorrect.
Þç ÷ <ç ÷ 15. (a) If f be the degree of freedom, then the ratio
è T ø1 è T ø 2
Cp 2
V 1 ( = g ) is given by g = 1 + .
from PV = mRT ; µ Cv f
T P For monoatomic gas, f = 3;
æ 1ö æ 1ö
Hence çè P ÷ø < çè P ÷ø Þ P1 > P2 . \ g = 1+
2 5
= = 1.67
1 2 3 3
1 For diatomic gas , f = 5
10. (a) vrms µ Þ (vrms)1 < (vrms)2 < (vrms)3
M 2 7
also in mixture temperature of each gas will \ g = 1+ = = 1.4.
be same, hence kinetic energy also remains 5 5
same. \ gdiatomic < gmonoatomic
EBD_7100
P-86 Topicwise AIIMS Solved Papers – PHYSICS
16. (b) Total translational kinetic energy Hence, mean free path varies inversely as
3 3 density of the gas. It can easily proved that
= nRT = PV the mean free path varies directly as the
2 2
temperature and inversely as the pressure
In an ideal gas all molecules moving
of the gas.
randomly in all direction collide and their
18. (a)
velocity changes after collision.
19. (c) The number 6.02 × 1023 is Avogadro’s
17. (a) The mean free path of a gas molecule is the
number and one mole of a substance
averge distance between two successive contains Avogadro’s number of molecules.
collisions. It is represented by l. 20. (b) When a person is driving a car then the
1 kT m temperature of air inside the tyre is
l= and l =
2
2 ps P 2 × ps2 d increased because of motion. From the Gay
Lussac’s law,
Here, s = 0 diameter of molecule and PµT
k = Boltzmann’s constant. Hence, when temperature increases the
Þ l µ 1 / d, l µ T and l µ 1 / P. pressure also increase.
Oscillations P-87

13 Oscillations

TYPE A : MULTIPLE CHOICE QUESTIONS 6. Which one of the following statement is not
correct for a particle executing S.H.M.? [1999]
1. Two identical springs of spring constant k are (a) Acceleration of the particle is minimum at
connected in series and parallel as shown in the mean position
figure. A mass M is suspended from them. (b) Restoring force is always directed towards
a fixed point
(c) Total energy of the particle always remains
the same
(d) Restoring force is maximum at the extreme
position
7. A particle execute simple harmonic motion with
an angular velocity of 3.5 rad/sec and maximum
acceleration 7.5 m/s2 .The amplitude of oscillation
will be: [1999]
(a) 0.53 cm (b) 0.28 m
(c) 0.61 m (d) 0.36 m
M M 8. In arrangement given in figure if the block of
The ratio of their frequencies of vertical mass m is displaced, the frequency is given by:
oscillation will be : [1997] [1999]
(a) 1 : 2 (b) 2 : 1 A B
(c) 4 : 1 (d) 1 : 4
2. If the metal bob of a simple pendulum is replaced k1 k2
by a wodden bob, then its time period will be :
(a) decreased (b) the same [1998] 1 æ k1 + k 2 ö
(c) increased (d) first (c) then (a) (a) n= ç ÷
2p è m ø
3. If the time period of oscillation of mass m
suspended from a spring is 2 sec, the time period
1 æ m ö
of mass 4 m will be : [1998] (b) n= ç ÷
2p çk +k ÷
(a) 2 sec (b) 3 sec è 1 2ø
(c) 4 sec (d) T sec
4. If a simple pendulum oscillates with an amplitude 1 æ m ö
(c) n= ç ÷
of 50 mm and time period of 2 sec then its 2p çk -k ÷
è 1 2ø
maximum velocity is [1998]
(a) 0.6 m/s (b) 0.16 m/s 1 æ k1 - k 2 ö
(c) 0.8 m/s (d) 0.32 m/s (d) n= ç ÷
2p è m ø
5. A horizontal platform with an object placed on it
is executing simple harmonic motion in the 9. A spring is vibrating with frequency under same
mass. If it is cut into two equal pieces and same
vertical direction. The amplitude of oscillation is
mass is suspended then the new frequency will
3.92 × 10–3 m. What should be the least period of
be: [1999]
these oscillations, so that the object is not n
detached from the platform ? [1999] (a) n 2 (b)
(a) 0.145 sec (b) 0.1556 sec 2
n
(c) 0.1256 sec (d) 0.1356 sec (c) (d) n
2
EBD_7100
P-88 Topicwise AIIMS Solved Papers – PHYSICS
10. Simple pendulum is executing simple harmonic 14. Which of the following functions represents a
motion with time period T. If the length of the simple harmonic oscillation ? [2005]
pendulum is increased by 21 %, then the increase
(a) sin wt - cos wt (b) sin 2 wt
in the time period of the pendulum of the
increased length is : [2001] (c) sin wt + sin 2wt (d) sin wt - sin 2wt
(a) 22 % (b) 13 % 15. A large horizontal surface moves up and down
(c) 50 % (d) 10 % in S.H.M. with an amplitude of 1 cm. If a mass of
11. The frequency of oscillator of the springs as 10 kg (which is placed on the surface0 is to remain
shown in figure will be : [2001] continuously in contact with it, the maximum
frequency of S.H.M. will be [2007]
1 ( k1 + k 2 ) m (a) 5 Hz (b) 0.5 Hz
(a) (c) 1.5 Hz (d) 10 Hz
2p k 1k 2
k 16. A coin is placed on a horizontal platform which
1 k 1k 2 1 undergoes vertical simple harmonic motion of
(b) angular frequency w. The amplitude of oscillation
2p ( k1 + k 2 ) m
is gradually increased. The coin will leave
1 k k contact with the platform for the first time [2008]
(c) 2 (a) at the mean position of the platform
2p m
g
(b) for an amplitude of 2
k w
(d) 2p
m g2
(c) for an amplitude of
12. Two springs of force constant k and 2k are w2
connected to a mass as shown below : The (d) at the highest position of the platform
frequency of oscillation of the mass is : [2003]
17. The function sin 2 (wt ) represents [2008]
k 2k (a) a periodic, but not simple harmonic motion
m p
with a period
w
(b) a periodic, but not simple harmonic motion
(a) (1 / 2 p) ( k / m) (b) (1 / 2p) ( 2 k / m) 2p
with a period
w
(c) (1 / 2p) (3k / m ) (d) (1 / 2 p) ( m / k ) p
(c) a simple harmonic motion with a period
13. Two springs are connected to a block of mass w
M placed on a frictionless surface as shown 2p
below. If both the springs have a spring constant (d) a simple harmonic motion with a period
w
k, the frequency of oscillation of block is : [2004]
18. A particle of mass is executing oscillations about
the origin on the x-axis. Its potential energy is
V(x) = k | x |3, where k is a positive constant. If the
amplitude of oscillation is a, then its time period
k k T is [2008]
M
1
(a) proportional to
a
(b) proportional to a
1 k 1 k
(a) (b) 3
2p M 2p 2M
(c) independent a 2
1 2k 1 M (d) none of these
(c) (d)
2p M 2p k
Oscillations P-89

19. The average speed of the bob of a simple 25. The circular motion of a particle with constant
pendulum oscillating with a small amplitude A speed is [2012]
and time period T is [2009] (a) periodic but not simple harmonic
(b) simple harmonic but not periodic
4A 2 pA (c) periodic and simple harmonic
(a) (b)
T T (d) neither periodic nor simple harmonic
26. A child swinging on a swing in sitting position,
4 pA 2A
(c) (d) stands up, then the time period of the swing will
T T (a) increase [2012]
20. A pendulum is swinging in an elevator. Its period (b) decrease
will be greatest when the elevator is [2010] (c) remains same
(a) moving upwards at constant speed (d) increases of the child is long and decreases
(b) moving downwards if the child is short
(c) moving downwards at constant speed 27. A point particle of mass 0.1 kg is executing
(d) accelerating downwards S.H.M. of amplitude of 0.1 m. When the particle
21. If A is the area of cross-section of a spring L is passes through the mean position, its kinetic
its length E is the Young's modulus of the material energy is 8 × 10–3 Joule. Obtain the equation of
of the spring then time period and force constant motion of this particle if this initial phase of
of the spring will be respectively: [2010] oscillation is 45º. [2013]
æ pö
EA L (a) y = 0.1sin çè ±4t + ÷ø
(a) T = 2p ,k = 4
ML EA æ pö
(b) y = 0.2sin çè ±4t + ÷ø
1 EA A 4
(b) T= ,k = æ pö
2p ML EL (c) y = 0.1sin çè ±2t + ÷ø
4
(c) 1 EL EA æ pö
T= ,k = (d) y = 0.2 sin çè ±2t + ÷ø
2p MA L 4
28. The bob of a simple pendulum is a spherical
(d) ML EA hollow ball filled with water. A plugged hole near
T = 2p ,k = the bottom of the oscillating bob gets suddenly
EA L
unplugged. During observation, till water is
22. The time period of a seconds pendulum is 2 sec. coming out, the time period of oscillation would
The spherical bob which is empty from inside [2013]
has a mass 50 gram, this now is replaced by (a) first decrease and then increase to the
another solid of same radius but have different original value
mass of 100 gram. The new time period will be (b) first increase and then decrease to the
(a) 2 sec (b) 8 sec [2010] original value
(c) 4 sec (d) 1 sec (c) increase towards a saturation value
23. A particle executes SHM of amplitude 25 cm and (d) remain unchanged
time period 3 s. What is the minimum time é pt ù
required for the particle to move between two 29. y = 2 (cm) sin ê + fú what is the maximum
points 12.5 cm on either side of the mean ë2 û
position? [2010] acceleration of the particle doing the S.H.M.
[2014]
(a) 0.5 s (b) 1.0 s 2
(c) 1.5 s (d) 2.0 s p p
(a) cm/s2 (b) cm/s2
24. The displacement of a particle undergoing SHM 2 2
of time period T is given by x(t) = xmcos (wt + f). p2 p
The particle is at x = – xm at time t = 0. The (c) cm/s2 (d) cm/s2
4 4
particle is at x = + xm when : [2011] 30. Resonance is an example of [2014]
(a) t = 0.25 T (b) t = 0.50 T (a) tuning fork (b) forced vibration
(c) t = 0.75 T (d) t = 1.00 T (c) free vibration (d) damped vibration
EBD_7100
P-90 Topicwise AIIMS Solved Papers – PHYSICS
31. Two particles are executing S.H.M. of same 36. Assertion : In simple harmonic motion, the
amplitude and frequency along the same motion is to and fro and periodic
straight line path. They pass each other when Reason : Velocity of the particle
going in opposite directions, each time their
displacement is half of their amplitude. What is ( v) = w k 2 - x 2 (where x is the displacement).
the phase difference between them ? [2015] [2002]
(a) 5 p/6 (b) 2 p/3 37. Assertion : The time-period of pendulum, on a
(c) p/3 (d) p/6 satellite orbiting the earth is infinity.
32. The period of oscillation of a mass M suspended Reason : Time-period of a pendulum is inversely
from a spring of negligible mass is T. If along proportional to g . [2002]
with it another mass M is also suspended, the 38. Assertion : The amplitude of an oscillating
period of oscillation will now be [2016] pendulum decreases gradually with time
(a) T (b) T / 2 Reason : The frequency of the pendulum
(c) 2T (d) 2T decreases with time. [2003]
33. Two, spring P and Q of force constants kp and Directions for (Qs. 39-42) : Each of these questions
æ kp ö contains an Assertion followed by Reason. Read them
kQ ç kQ = ÷ are stretched by applying forces carefully and answer the question on the basis of
è 2ø
following options. You have to select the one that
of equal magnitude. If the energy stored in Q is
best describes the two statements.
E, then the energy stored in P is [2016]
(a) If both Assertion and Reason are correct and
(a) E (b) 2 E
(c) E/2 (d) E/4 Reason is the correct explanation of Assertion.
34. A particle moves with simple harmonic motion (b) If both Assertion and Reason are correct, but
in a straight line. In first t s, after starting from Reason is not the correct explanation of
rest, it travels a distance a, and in next t s, it Assertion.
travels 2a in same direction then [2016] (c) If Assertion is correct but Reason is incorrect.
(a) amplitude of motion is 3a (d) If both the Assertion and Reason are incorrect.
(b) time period of oscillations is 8t. 39. Assertion : In SHM, acceleration is always
(c) amplitude of motion is 4a. directed towards the mean position.
(d) time period of oscillations is 6t. Reason : In SHM, the body has to stop
35. The amplitude of a damped oscillator decreases momentary at the extreme position and move
to 0.9 times its original magnitude in 5s. In back to mean position. [2009]
another 10s it will decrease to a times its original 40. Assertion : For a particle performing SHM, its
magnitude, where a equals [2017] speed decreases as it goes away from the mean
(a) 0.7 (b) 0.81 position.
(c) 0.729 (d) 0.6 Reason : In SHM, the acceleration is always
opposite to the velocity of the particle. [2009]
TYPE B : ASSERTION REASON QUESTIONS 41. Assertion : Resonance is a special case of
Directions for (Qs. 36-38) : These questions consist forced vibration in which the natural frequency
of two statements, each printed as Assertion and of vibration of the body is the same as the
Reason. While answering these questions, you are impressed frequency of external periodic force
required to choose any one of the following five and the amplitude of forced vibration is
responses. maximum.
(a) If both Assertion and Reason are correct and Reason : The amplitude of forced vibrations of
the Reason is a correct explanation of the a body increases with an increase in the
Assertion. frequency of the externally impressed periodic
(b) If both Assertion and Reason are correct but force. [2010]
Reason is not a correct explanation of the 42. Assertion : In simple harmonic motion, the
Assertion. velocity is maximum when the acceleration is
(c) If the Assertion is correct but Reason is minimum.
incorrect. Reason : Displacement and velocity of S.H.M.
(d) If both the Assertion and Reason are incorrect. p
(e) If the Assertion is incorrect but the Reason is differ in phase by . [2014]
correct. 2
Oscillations P-91

Type A : Multiple Choice Questions SHM = w2a


2
1. (a) We know that
w 2 a = g Þ æç 2p ö÷ ´ 3.92 ´ 10 -3 = 10
m 1 K è T ø
T = 2p Þ n=
K 2p m
3.92 ´ 10 -3
For a spring mass system. T = 2p = 2 p 3.92 ´ 10 -4
10
In case I if K is the resultant spring constant,
then = 0.1256 secs.
6. (a) For a particle under SHM.
1 1 1 2 k
= + = Þ K= acceleration = w2x
K k k k 2 If x = 0 (at mean position), acceleration = 0
In case II, K = k + k = 2k So, acceleration at the mean position is
If n1 & n2 be frequencies in two cases, then zero.
1 k 1 2k 7. (c) w = 3.5 radian/sec
n1 = ; n2 = ; maximum acceleration of a particle under
2p 2m 2p m
SHM = w2 a where a is amplitude of
n1 1 n 1 oscillation.
Þ = Þ 1 =
n2 4 n2 2 w2a = 7.5 Þ (3.5) 2 a = 7.5
2. (b) The expression of time period
7.5 30
Þ a= Þ = = 0.61 m
l 3.5 ´ 3.5 49
T = 2p
g 8. (a) This is case of spring in series, so
K = k1 + k2
This formula co ntains nothing which
depends upon the nature of material from m
which bob is made of. So time period will T = 2p
(k1 + k 2 )
remain the same.
m m 1 k1 + k 2
3. (c) T = 2p Þ 2 = 2p Þ n(frequency) =
K K 2p m

4m m 1 k
T ' = 2p Þ T ' = 2 ´ 2p 9. (a) For a vibrating spring n =
K K 2p m
T ' = 2 ´ 2 = 4sec Now spring is cut into two pieces, so new
spring constant K' = 2k
4. (b) v = w a 2 - u 2 ; when u = 0, v = vmax.
So, vmax = wa 1 2k
n' = =n 2
[where w is angular velocity and a is amplitude] 2p m
2p 2p 50 l l
vmax.= ´a = ´ = 0.16 m / sec 10. (d) T = 2p Þ T 2 = 4p 2
T 2 1000 g g
5. (c) The possibility of the object to be
Taking log on both sides,
detached from the platform is at the highest
point when the platform starts going down. 2 log T = log 4p2 + log l - log g ,
If it is less than g then object will not get Differentiating on both sides,
away from the platform. So, maximum
acceleration of platform under SHM is g. 2 1
dT = 0 + dl - 0
From formula, maximum acceleration under T l
EBD_7100
P-92 Topicwise AIIMS Solved Papers – PHYSICS

dT 1 dl dT 1 dl é 1 1 ù
= . Þ ´ 100 = . ´ 100 14. (a) sin wt - cos wt = 2 ê sin wt - cos wt ú
T 2 l T 2 l ë 2 2 û
Percent increase in time period
é p p ù
1 = 2 êcos . sin wt - sin cos wt ú
= % increase in length ë 4 4 û
2
= 2 sin(wt - p / 4)
1
= ´ 21 = 10.5% which represents simple harmonic motion.
2
15. (a) Frequency of SHM,
Percent increase in time period @ 10%
1 acceleration 1 a
1 k n= =
11. (b) n= 2p displacement 2p x
2p m
As springs are in parallel, total spring 1 k
constant k of system of spring Þn=
2p m
1 1 1 k + k2
= + = 1 k a k m 1 m
k k1 k 2 k1 k 2 Þ = Þ = Þn=
m x a x 2p x
k1 k 2
k= 1 10 103 10 10
( k1 + k 2 ) n= = =
2p 10-2 2p 2p
1 k1k 2
n= 10 ´ 3.16
2p (k1 + k2 )m = = 5 Hz
2 ´ 3.14
12. (c) For any spring-mass system time period of 16. (b) For block A to move in SHM.
oscillation,
N
m 1 k A
T = 2p Þ n=
k 2p m
In the present case total spring constt. x
mg mean
= k + 2k = 3k. position

1 3k mg – N = mw2x
n=
2p m where x is the distance from mean position
For block to leave contact N = 0
1 k
13. (b) For spring block system, n = g
2p m Þ mg = mw2 x Þ x =
w2
Here two springs are joined in series so,
17. (a) Clearly sin2 wt is a periodic function as
k1k 2 sin wt is periodic with period p / w
total spring constant, k R =
k1 + k 2

k.k k
kR = =
2k 2

1 k 0 p/w 2p /w 3p /w
So, n =
2p 2m
Oscillations P-93

d2y FL
For SHM µ -y E=
dt 2 A.DL
where DL is the extension in the spring.
dy
= 2w sin wt cos wt = w sin 2wt EA DL
dt F= ...(1)
L
d2y Now, according to Hooke's law
= 2w 2 cos 2 wt which is not proporti- F=kDL ...(2)
2
dt where k is the spring constant
onal to –y. Hence it is not in SHM. By comparing (1) and (2)
18. (a) V(x) = k | x |3
EA DL
dV(x) kDL=
since, F = - = -3k | x |2 …(1) L
dx EA
x = a sin (wt) k=
L
This equation always fits to the differential
equation M
Time period, T = 2p
k
d2 x 2
= -w2 x or m d x = -mw 2 x ML
dt 2 dt T = 2p
EA
Þ F = – mw2x …(2)
Equation (1) and (2) give L
22. (a) T = 2p
2
-3k | x | = - mw x 2 g
i.e., time period of a simple pendulum
3kx 3ka depen ds upon effective length and
Þ w= = [sin(wt)]1/ 2
m m acceleration due to gravity, not on mass.
So, T = 2 sec.
1 23. (a) y = r sin w t
Þ wµ a Þ Tµ
a 2p æ 2p ö
12.5 = 25 sin
3
´t çèQ w = ÷ø
æ 2p ö T
19. (a) x = A sin ç t ÷ p 2p
è T ø = t
6 3
T 1
Þ distance covered in time t = =A t = sec = 0.25 sec
4 4
A 4A t' = 2t
Þ average speed = = = 2 × 0.25 = 0.5 sec
T/4 T
for either side
20. (d) Time period of a simple pendulum is given 24. (b) The time taken by particle from left extreme
by to right extreme = 0.5 T.
l l 25. (a) In circular motion of a particle with constant
T = 2p or T µ speed, particle repeats its motion after a
g g
regular interval of time but does not
when the elevator is accelerating oscillate about a fixed point. So, motion of
downwards, the net gravitational particle is periodic but not simple harmonic.
acceleration is (g – a), so, the time period
when elevation is accelerating downwards,
is greatest.
21. (d) According to the formula of Young's
Modulus
EBD_7100
P-94 Topicwise AIIMS Solved Papers – PHYSICS

26. (b) T = 2p leff / g ; l eff decreases when the d2y p 2 æ pt ö


acceleration =- sinç + f ÷
child stands up. dt 2 è2 ø
27. (a) The displacement of a particle in S.H.M. is
p2
given by Thus a max =
y = a sin (wt + f) 2
30. (b)
dy
velocity = = wa cos (wt + f) 31. (b) y = a sin (w t + f) ; when y = a/2,
dt a
The velocity is maximum when the particle then = a sin ( w t + f )
2
passes through the mean position i.e.,
1 p 5p
æ dy ö or sin ( w t + f) = = sin or sin
çè ÷ø 2 6 6
dt max = w a So phase of two particles is p/6 and 5 p/6
The kinetic energy at this instant is given radians
by Hence phase difference = (5 p/6) – p/6 = 2 p/3
2
1 æ dy ö 1 m T1 M1
m çè ÷ø = mw2 a2 = 8 × 10–3 joule 32. (d) T = 2 p \ T = M2
2 dt max 2 K 2

1 M2
or × (0.1) w2 × (0.1)2 = 2M
2 T2 = T1 M1 = T 1 M
8 × 10–3
Solving we get w= ±4 T2 = T1 2 = 2 T (where T1 =T)
Substituting the values of a, w and f in the
kp
equation of S.H.M., we get 33. (c) Here, kQ =
y = 0.1 sin (± 4t + p /4) metre. 2
28. (b) Centre of mass of combination of liquid and According to Hooke's law
hollow portion (at position l), first goes \ Fp = –kp xp
down ( to l + D l) and when total water is Fp k p x p
drained out, centre of mass regain its FQ = –kQxQ Þ F = k x
Q Q Q
original position (to l), Fp = FQ [Given]
l xp kQ
T = 2p =
g \ xQ kp ...(i)
\ ‘T’ first increases and then decreases 1 2
to original value. Energy stored in a spring is U = kx
2
Up k p x 2p kp kQ2 1 é kp ù
\ = 2
= ´ = êQ kQ = ú
UQ kQ xQ kQ k 2p 2 ë 2û
c UQ E
Þ Up = = [\ U Q = E ]
2 2
34. (d) As it starts from rest, we have,
æ pt ö x = A cos wt. At t = 0, x = A
29. (b) y = 2 sin ç + f ÷
è 2 ø When t = t, x = A – a and
when t = 2t, x = A – 3a
dy p æ pt ö
velocity of particle = 2 ´ cos ç + f ÷ Þ A – a = A ´ cos ´ wt and ....... (i)
dt 2 è 2 ø A – 3a = A ´ cos ´ 2wt ....... (ii)
Oscillations P-95

As, cos2wt = 2cos2wt – 1, 39. (b) SHM is basically to and fro motion about
2 the mean position. So when the body goes
A - 3a æ A-a ö away from mean position an acceleration
Þ = 2ç ÷ -1
A è A ø always try to return the body towards mean
A - 3a 2 A2 + 2a 2 - 4 Aa - A2 position. As the acceleration in SHM is
\ = always in opposite phase to that of
A A2 displacement. The displacement of the
2 2 2
\ A – 3aA = A + 2a – 4Aa particle in SHM at an instant is directed
\ a2 = 2aA Þ A = 2a away from the mean position then
Now, A – a = A ´ cos ´ wt ..... [From (i)] acceleration at that instant is directed
1 towards the mean position.
Þ cos ´ wt =
2
40. (c) Speed = w A2 - x 2
2p p
\ t = Þ T = 6t as | x | increases Þ speed decreases
T 3
Acceleration is in direction of speed as it
bt
-
2m
comes towards mean position.
35. (c) Q A = A0 e (where, A0 = maximum 41. (c) The amplitude become large when the
amplitude) frequency of the driving force (w) is near
According to the questions, after 5 the natural frequency of oscillation or when
seconds, w ; w0 . This frequency is known as
-
b(5) resonance frequency. Amplitude of
0.9A 0 = A 0 e 2m … (i) oscillation for a forced, damped oscillator
After 10 more seconds, is
b(15) F0 / m
-
…(ii) A=
A = A0 e 2m
(w - w02 ) + (bw / m)2
2

From equations (i) and (ii)


where b is constant related to the strength
A = 0.729 A0
\ a = 0.729 of the resistive force, w0 = k / m is
Type B : Assertion Reason Questions natural frequency of undamped oscillator
( b = 0).
36. (b) SHM is to and fro motion of an object and 42. (b) At the middle point velocity of the particle
it is periodic. under SHM is maximum but acceleration is
v = w k2 - x2
zero since displacement is zero. So Assertion
If x = 0, v has maximum value. At x = k, v is true.
has minimum velocity. Similarly, when x = We know that x = a sin wt ...(1)
– k, v has zero value, all these indicate to & Where x is displacement and a is amplitude.
fro movement. dx
Velocity = = aw cos wt
37. (a) Time period of pendulum in a satellite is dt
infinity. It means it may not oscillate as
æp ö
apparent value of g is zero. So, time period = aw cos( - wt ) = aw sin ç - (- wt )÷
è 2 ø
1 æ pö
being µ µ ¥
g = aw sinç wt + ÷ ...(2)
è 2ø
38. (c) The amplitude of an oscillating pendulum From equation (i) and (ii) it is clear that
decreases with time due to friction of air. In p
absence of air, its frequency and amplitude Velocity is ahead of displacement (x) by
2
will remain constant.
angle.
EBD_7100
P-96 Topicwise AIIMS Solved Papers – PHYSICS

14 Waves

TYPE A : MULTIPLE CHOICE QUESTIONS 7. The equation of a travelling wave is


y = 60 cos(1800t - 6x )
1. At which temperature velocity of sound (at 27°C)
doubles? [1997, 2002] where y is in microns, t in second and x in metre.
The ratio of maximum particle velocity to the
(a) 327°C (b) 927°C
velocity of wave propagation is : [1998]
(c) 54°C (d) –123 °C –4 - 6
(a) 3.6 ´ 10 (b) 3.6 ´ 10
2. The wave equation is y = 0.30 sin (314t – 1.57x)
where t, x and y are in second, metre and (c) 3.6 ´ 10 -8 (d) none of these
centimetre respectively. The speed of the wave 8. The waves in which the particles of the medium
is [1997] vibrate in a direction perpendicular to the
(a) 400 m/s (b) 100 m/s direction of wave motion is known as : [1998]
(c) 200 m/s (d) 50 m/s (a) longitudinal waves (b) propagated waves
3. An object producing a pitch of 1200 Hz is moving (c) transverse wave (d) none of these
9. Energy is not carried by which of the following
with a velocity of 50 m/s towards a stationary
wave? [1999]
person. The velocity of sound is 350 m/s. The
(a) Progressive (b) Electromagnetic
frequency of sound heard by the stationary
(c) Transverse (d) Stationary
person is : [1997]
10. If the vibrations of a string are to be increased
(a) 1250 Hz (b) 1050 Hz by a factor of two, then tension in the string
(c) 700 Hz (d) 1400 Hz should be made : [1999]
4. The air column in a pipe which is closed at one (a) Twice (b) Four times
end will be in resonance with a vibrating tuning (c) Eight times (d) Half
fork at a frequency 260 Hz. The length of the air 11. A resonance in air column of length 20 cm
column is : [1997] resonates with a tuning fork of frequency 450
(a) 12.5 cm (b) 35.75 cm Hz. Ignoring end correction, the velocity of
(c) 31.92 cm (d) 62.5 cm sound in air is: [1999]
5. Standing waves are produced in 10 m long (a) 1020 m/s (b) 720 m/s
(c) 620 m/s (d) 820 m/s
stretched string. If the string vibrates in 5
12. A transverse wave passes through a string with
segments and wave velocity is 20 m/s, then its
frequency will be : [1998] the equation : y = 10 sin p (0.02x - 2.00t )
(a) 5 Hz (b) 2 Hz where x is in metre and t in second. The maximum
(c) 10 Hz (d) 2 Hz velocity of the particle in wave motion is :[2000]
6. Newton’s formula for the velocity of sound in (a) 100 m/s (b) 63 m/s
(c) 120 m/s (d) 161 m/s
gas is : [1998]
13. A wave is represented by the equation
P 2 P y = a sin(0.01x - 2t )
(a) v= (b) v=
r 3 r where a and x are in cm and t in second. Velocity
of propagation of the wave is : [2000]
r v=
2P (a) 200 cm/sec (b) 10 cm/sec
(c) v= (d)
P r (c) 25 cm/sec (d) 50 cm/sec
Waves P-97

14. Two cars A and B approach a stationary observer 21. A string in a musical instrument is 50 cm long
from opposite sides as shown in fig. Observer and its fundamental frequency is 800 Hz. If a
hears no beats. If the frequency of the horn of frequency of 1000 Hz is to be produced, then
the car B is 504 Hz, the frequency of horn of car required length of string is [2002]
A will be : (a) 37.5 cm (b) 40 cm
15 m/sec 30 m/sec (c) 50 cm (d) 62.5 cm
22. An earthquake generates both transverse (S)
and longitudinal (P) sound waves in the earth.
The speed of S waves is about 4.5 km/s and that
of P waves about 8.0 km/s. A seismograph
A B
records P and S waves from an earthquake. The
(a) 529.2 Hz (b) 295.2 Hz [2000]
first P wave arrives 4.0 min. before the first S
(c) 440.5 Hz (d) none of these
wave. The epicenter of the earthquake is located
15. The tension in a piano wire is 10 N. The tension
at a distance of about : [2003]
in a piano wire to produce a node of double
frequency is : [2001] (a) 25 km (b) 250 km
(a) 20 N (b) 40 N (c) 2500 km (d) 5000 km
(c) 10 N (d) 120 N 23. An organ pipe closed at one end has
16. Two sound waves have phase difference of 60°, fundamental frequency of 1500 Hz. The maximum
then they will have the path difference of : number of overtones generated by this pipe
l which a normal person can hear is [2004]
(a) 3l (b) [2001] (a) 4 (b) 13
3
(c) 6 (d) 9
l 24. The wave produced by a motor boat sailing in
(c) (d) l
6 water are [2004]
17. A sings with a frequency (n) and B sings with (a) transverse
a frequency 1/8 that of A. If the energy remains (b) longitudinal
the same and the amplitude of A is a, then
(c) longitudinal and transverse
amplitude of B will be : [2001]
(a) 2a (b) 8a (d) stationary
(c) 4a (d) a 25. A boat at anchor is rocked by waves whose
18. If equation of sound wave is y = 0.0015 crests are 100 m apart and velocity is 25 m/sec.
sin (62.4 x + 316t), then its wavelength will The boat bounces up once in every: [2006]
be [2002] (a) 2500 s (b) 75 s
(a) 2 unit (b) 0.3 unit (c) 4s (d) 0.25 s
(c) 0.1 unit (d) 0.2 unit 26. A stone thrown into still water, creates a circular
19. A siren emitting sound of frequency 800 Hz is wave pattern moving radially outwards. If r is
gong away from a static listener with a speed of the distance measured from the centre of the
30 m/s. Frequency of the sound to be heard by
pattern. the amplitude of the wave aries as :
the listener is (Take velocity of sound = 300 m/s)
(a) 286.5 Hz (b) 481.2 Hz [2002] (a) r–1/2 (b) r–1 [2006]
(c) r –2 (d) r –3/2
(c) 733.3 Hz (d) 644.8 Hz
20. The velocities of sound at the same temperature 27. When a guitar string is sounded with a 440 Hz
in two monoatomic gases of densities r1 and r2 tuning fork, a beat frequency of 5 Hz is heard. If
are v1 and v2 respectively. If r1/r2 = 4, then the the experiment is repeated with a tuning fork of
value of v1/v2 is [2002] 437Hz, the beat frequency is 8 Hz. The string
(a) 4 (b) 2 frequency (Hz) is : [2006]
1 1 (a) 445 (b) 435
(c) (d) (c) 429 (d) 448
2 4
EBD_7100
P-98 Topicwise AIIMS Solved Papers – PHYSICS
28. For a wave propagating in a medium, identity 34. The expression y = a sin bx sin w t represents a
the property that is independent of the others: stationary wave. The distance between the
(a) velocity [2006] consecutive nodes is equal to : [2011]
(b) wavelength (a) p/b (b) 2p/b
(c) p/2b (d) 1/b
(c) frequency
35. An open and closed organ pipe have the same
(d) all these depend on each other length. The ratio of pth mode of frequency of
29. A string is stretched between fixed points vibration of two pipes is [2012]
separated by 75.0 cm. It is observed to have (a) 1 (b) p
resonant frequencies of 420 Hz and 315 Hz. There
are no other resonant frequencies between these 2p
(c) p (2p + 1) (d)
two. Then, the lowest resonant frequency for ( 2 p - 1)
this string is [2008] 36. When two tuning forks (fork 1 and fork 2) are
(a) 105 Hz (b) 1.05 Hz sounded simultaneously, 4 beats per second are
(c) 1050 Hz (d) 10.5 Hz heard. Now, some tape is attached on the prong
30. A person speaking normally produces a sound of the fork 2. When the tuning forks are sounded
intensity of 40 dB at a distance of 1 m. If the again, 6 beats per second are heard. If the
threshold intensity for reasonable audibility is frequency of fork 1 is 200 Hz, then what was the
original frequency of fork 2? [2012]
20 dB, the maximum distance at which he can be
heard clearly is [2008] (a) 202 Hz (b) 200 Hz
(c) 204 Hz (d) 196 Hz
(a) 4 m (b) 5 m
37. A sound absorber attenuates the sound level
(c) 10 m (d) 20 m
by 20 dB. The intensity decreases by a factor of
31. A wave on a string is travelling and the (a) 100 (b) 1000 [2012]
displacement of particles on it is given by (c) 10000 (d) 10
x = A sin (2t – 0.1 x). Then the wavelength of the
38. A fork of frequency 256 Hz resonates with a
wave is [2009]
closed organ pipe of length 25.4 cm. If the length
(a) 10p (b) 20p of pipe be increased by 2 mm, the number of
(c) (d) 20 beats/sec. will be [2014]
40p
(a) 4 (b) 1
32. A tuning fork of frequency 340 Hz is vibrated
(c) 2 (d) 3
just above the tube of 120 cm height. Water is
poured slowly in the tube. What is the minimum 39. The equation of a progressive wave is
height of water necessary for the resonance? é t x ù
y = 0.02 sin 2p ê -
(speed of sound in air = 340 m/s) [2009] ë 0. 01 0 .30 úû
(a) 45 cm (b) 30 cm
Here x and y are in metre and t is in second. The
(c) 40 cm (d) 25 cm velocity of propagation of the wave is [2014]
33. The velocity of sound in a gas at pressure P and (a) 300 m s–1 (b) 30 m s–1
density d is [2009] (c) 400 m s –1 (d) 40 m s–1
40. Two waves of wavelengths 99 cm and 100 cm
gP P both travelling with velocity 396 m/s are made
(a) v= (b) v=
d gd to interfere. The number of beats produced by
them per second is [2015]
g P 2P (a) 1 (b) 2
(c) v= (d) v=
d d (c) 4 (d) 8
Waves P-99

41. A massless rod of length L is suspended by two 44. A train moving at a speed of 220 ms–1 towards a
identical strings AB and CD of equal length. A stationary object, emits a sound of frequency 1000
block of mass m is suspended from point O such Hz. Some of the sound reaching the object gets
that BO is equal to ‘x’. Further it is observed reflected back to the train as echo. The frequency
that the frequency of 1st harmonic in AB is equal of the echo as detected by the driver of the train
to 2nd harmonic frequency in CD. ‘x’ is is (speed of sound in air is 330 ms–1) [2017]
(a) 3500 Hz (b) 4000 Hz
A C [2016] (c) 5000 Hz (d) 3000 Hz
TYPE B : ASSERTION REASON QUESTIONS
Directions for (Qs. 45-49) : These questions consist
of two statements, each printed as Assertion and
O Reason. While answering these questions, you are
B D
x L required to choose any one of the following five
m responses.
(a) If both Assertion and Reason are correct and
the Reason is a correct explanation of the
L 4L Assertion.
(a) (b) (b) If both Assertion and Reason are correct but
5 5
Reason is not a correct explanation of the
3L L Assertion.
(c) (d)
4 4 (c) If the Assertion is correct but Reason is
42. Two similar open organ pipe of length 50 cm and incorrect.
50.5 cm produce 3 beats per second when (d) If both the Assertion and Reason are incorrect.
sounded together. The velocity of sound in air (e) If the Assertion is incorrect but the Reason is
is [2016] correct.
45. Assertion : Sound travels faster in solids than
(a) 303 m/s (b) 330 m/s
gases.
(c) 151.5 m/s (d) 603 m/s Reason : Solids possess greater density than
43. A whistle S of frequency f revolves in a circle of gases. [2000]
radius R at a constant speed v. What is the ratio
of largest and smallest frequency detected by a wavelength
46. Assertion : Speed of wave =
detector D at rest at a distance 2R from the centre time period
of circle as shown in figure ? Reason : Wavelength is the distance between
(take c as speed of sound) [2016] two nearest particles in phase. [2002]
47. Assertion : When a beetle moves along the sand
within a few tens of centimeters of a sand
scorpion, the scorpion immediately turns
D S towards the beetle and dashes towards it
R
Reason : When a beetle disturbs the sand, it
sends pulses along the sand's surface. One set
2R of pulses is longitudinal while the other set is
transverse. [2003]
æ c + vö æ c + vö 48. Assertion : Sound waves cannot travel in
çè ÷ 2ç
(a)
c - vø
(b) è c - v ÷ø vacuum but light can travel in vacuum.
Reason : Sound waves are longitudinal waves
(c + v) and they can not be polar ised but
(c) 2 (d) electromagentic waves are transverse and they
c 2
can be polarised. [2007]
EBD_7100
P-100 Topicwise AIIMS Solved Papers – PHYSICS
49. Assertion : The change in air pressure affects 52. Assertion : A transverse waves are produced
the speed of sound. in a very long string fixed at one end. Only
Reason : The speed of sound in gases is progressive wave is observed near the free end.
proportional to the square of pressure. [2008] Reason : Energy of reflected wave does not
reach the free end. [2013]
Directions for (Qs. 50-56) : Each of these questions 53. Assertion : Doppler formula for sound wave is
contains an Assertion followed by Reason. Read them symmetric with respect to the speed of source
carefully and answer the question on the basis of and speed of observer.
following options. You have to select the one that
Reason : Motion of source with respect to
best describes the two statements.
stationary observer is not equivalent to the
(a) If both Assertion and Reason are correct and motion of an observer with respect to stationary
Reason is the correct explanation of Assertion. source. [2014]
(b) If both Assertion and Reason are correct, but 54. Assertion : Two waves moving in a uniform
Reason is not the correct explanation of string having uniform tension cannot have
Assertion. different velocities.
(c) If Assertion is correct but Reason is incorrect. Reason : Elastic and inertial properties of string
are same for all waves in same string. Moreover
(d) If both the Assertion and Reason are incorrect. speed of wave in a string depends on its elastic
50. Assertion : The pitch of wind instruments rises and inertial properties only. [2015]
and that of string instruments falls as an 55. Assertion : The base of Laplace correction was
orchestra warms up. that exchange of heat between the region of
Reason : When temperature rises, speed of compression and rarefaction in air is negligible.
sound increases but speed of wave in a string Reason : Air is bad conductor of heat and
fixed at both ends decreases. [2009] velocity of sound in air is quite large. [2016]
56. Assertion : The fundemental frequency of an
51. Assertion : For the formation of stationary
open organ pipe increases as the temperature is
waves the medium must be bounded having
increased.
definite boundaries.
Reason : As the temperature increses, the
Reason : In the stationary wave, some particles velocity of sound increases more rapidly than
of the medium remain permanently at rest. [2010] length of the pipe. [2017]
Waves P-101

Type A : Multiple Choice Questions Now, l = 4 m; v = 20 m/sec


From formula
1. (b) Velocity of sound µ T (where T is
v = nl Þ 20 = n ´ 4 Þ n = 5Hz
temperature of body in absolute scale). 6. (a) Newton’s formula for velocity of sound in
v1 T 2v t + 273 gas
= 1 Þ =
v2 T2 v 27 + 273 P
v= where P is pressure & r is
r
t + 273
4= Þ t + 273 = 1200 density of gas.
300
Þ t = 927° C 7. (a) y = 60 cos(1800t - 6x ) , y is in microns.
2. (c) Given equation y = 0.30 sin(314t - 1.57 x) y = 60 cos(1800t - 6 x) ´10 -6
Comparing it with standard equation of dy
wave, v= = particle velocity
dt
y = a sin(wt - kx)
w = 314; k = 1.57 = 60 ´10 - 6 ´1800 sin (1800t - 6x )

w 314 v max = 6 ´ 18 ´10-3


v= Þ v= = 200 m / sec
k 1.57 w
v wave = ; w = 1800; K = 6
3. (d) If n a be the apparent frequency, then k
1800
vs 1200 ´ 350 v wave = = 300
na = n ´ = 6
(vs - v0 ) (350 - 50)
vmax 6 ´18 ´10 -3
1200 ´ 350 =
= = 1400 Hz v wave 300
300
4. (c) For fundamental frequency = 36 ´ 10 - 5 = 3 .6 ´ 10 - 4
8. (c) Transverse waves h ave particles
v l oscillating perpendicular to the direction
n= & = l Þ l = 4l
l 4 of motion of wave. Ripple in the surface of
v 330 water is transverse in nature.
n= Þ 260 =
Direction of oscillation

4l 4l
of particles

330
Þ l= = 0.3173 m
260 ´ 4 P
l = 31.73 cm ~ 31.92 cm (given in option)
5. (a)
Direction of motion of wave
9. (d) Stationary waves do not carry energy with
it as it is stationary or does not change
position.
l
10. (b) We know that, n = 1 T Þ n µ T
There are 5 loops in 10 metre. 2 m
Length of one loop = 2 m If tension is increased four times, the
Length of two loops = 2 × 2 = 4 m frequency will become twice.
EBD_7100
P-102 Topicwise AIIMS Solved Papers – PHYSICS
11. (b) Let the air column be closed one.
17. (b) Energy of sound wave = 2p2 n2 a 2 vr
In closed organ pipe for fundamental node
n is frequency and v is velocity of sound;
l r is density of air or any other medium.
of vibration , = 0.2 Þ l = 0.8m,
4
v = nl Now, E1 = 2p2n 2a 2 vr
If we take the frequency of air column to be ænö
2
equal to tuning fork then, E 2 = 2p2 ç ÷ A 2 vr E1 = E 2
è8ø
v = 450 ´ 0.8 = 360 m / sec
n 2 2
If we take the frequency of air column to be 2p 2 n 2 a 2 vr = 2p 2 A vr
twice that of tuning fork then, 64
A2 = 64a2 A = 8a
v = 900 ´ 0.8 = 720 m / sec
18. (c) y = 0.0015 sin (62.8x + 314t)
This matches with alternative (b). Comparing it with the equation
12. (b) The equation of wave y = a sin ( w t + kx)
y = 10 sin p(0.02x - 2t) w = 314, k = 62.8
Particle velocity, 2p 2p
k= = 62.8 Þ l = = 0.1 unit.
dy l 62.8
= 10cos p(0.02x - 2t) ´ p ´ 0.2
dt 19. (c) Since, the source is going away from
= 0.63cos p(0.02x - 2t) v 330
listener n ' = n ´ = 800 ´
Maximum value of velocity = 63 m/sec. v+u (330 + 30)
13. (a) In the wave of the form 330
n ' = 800 ´ = 733.33 Hz
y = sin(wt - kx) 360
20. (c) For velocity of sound in gas
w
Velocity v =
k gP
v=
Here w = 2 ; k = 0.01 r
w 2 [P is pressure and r is density of gas, g is
v= = = 200 cm / sec Cp/Cv]
k 0.01
14. (a) Since, the person hears no beats therefore gP gP
their apparent frequency are the same. Here, v1 = and v 2 =
r1 r2
330 330
So, n ´ = 504 ´ v1 r2 1 1
330 - 15 330 - 30 = = =
v2 r1 4 2
504 ´ 315
n= = 529.2 Hz 21. (b) We know that for frequency of sound in
300
15. (b) For frequency of oscillation of wire. 1 T 1
string the formula is, n = or n =
2l m l
n µ T , Here T is tension in the wire.
In order to increase frequency twice, n1 l 2 800 l
= Þ = 2
tension needs to be made 4 times. So, new n 2 l1 1000 50
tension must be 4 ´10 = 40 N 800
l 2 = 50 ´ = 40 cm
l 1000
16. (c) Path diff. = ´ phase difference
2p 22. (c) Let d be the distance of epicenter.
l p l d
= ´ = Time taken by S-wave =
2p 3 6 4.5
Waves P-103

d Energy of crest (P.E) = (2pr.dr ´ h ´ r) ´ g ´ h


Time taken by P-wave = Now, as crest spread, this energy E remains
8
constant. So,
d d
Now - = 4 ´ 60 2prdrh 2 rg = E
4.5 8
E
é 10 1 ù
d ê - ú = 4 ´ 60 Þ h= or h µ r -1/ 2
ë 45 8 û 2prdrrg
27. (a) If we decrease the frequency of tuning fork
4 ´ 60 ´ 8 ´ 45
Þ d= no. of beat is increased i.e., difference of
35 frequency is increased. It means unknown
240 ´ 72 17280 frequency is more than 440 Hz.
= = = 2468.5 » 2500 km. So it is 440 + 5 = 445 Hz.
7 7 28. (c) In a medium velocity and wavelength are
23. (c) In an organ pipe only odd harmonics are dependent on refractive index of the
found. So, frequency equal to odd multiple medium but frequency remains unchanged.
of 1500 Hz may be found in this case. v
Maximum audible frequency is 20,000 Hz. 29. (a) Given nv = 315 and ( n + 1) = 420
2l 2l
So, possible frequency are 1500 Hz,
4500 Hz, 7500 Hz, 10500 Hz, 13500 Hz, 16500 n + 1 420
Þ = Þn =3
Hz, 19500 Hz. 19500 Hz will be sixth n 315
overtone as 4500 will be first overtone. v v
24. (c) The waves produced by a motorboat Hence 3 ´ = 315 Þ = 105 Hz
2l 2l
sailing in water are of both transverse and
The lowest resonant frequency is when
longitudinal type. Transverse waves are
n=1
produced on the surface and longitudinal Therefore lowest resonant frequency
waves are produced deep inside the water. = 105 Hz.
25. (c) Wavelength is distance between two crest.
l = 100 m; v = 25 æ Iö
30. (c) We have , b = 10log10 ç I ÷
è ø 0
25
n= Where I0 = threshold intensity of sound
100
1 100 = 10 -12 W / m 2
T= = = 4 sec
n 25 æI ö
i.e., 40 = 10 log10 1 … (i)
çè I ÷ø
26. (a) 0

æI ö
dv and 20 = 10 log10 ç 2 ÷ … (ii)
r è I0 ø

(i) 40 æI ö
Þ = log10 ç 1 ÷
(ii) 20 è I2 ø

æI ö I1 2
2 = log10 ç 1 ÷ or I = 10
è I2 ø 2

h r2 2
\ = 102 (since I µ 1 )
r12 r2
dr
r2 2 = 102 r12 or r2 = 10r1 = 10 ´ 1 = 10m
EBD_7100
P-104 Topicwise AIIMS Solved Papers – PHYSICS

31. (b) x = A sin(2t - 0.1x) v


35. (d) For open pipe, n = p
2l
æ 2pt 2p ö v
also x = A sin ç - x÷ For closed pipe n ¢ = ( 2p - 1)
è T l ø 4l
2p n 2p
Þ = 0.1 Þ l = 20p \ =
l n ¢ (2 p - 1)
32. (a) We have v = nl . 36. (d) Frequency of fork 1 = 200 Hz = n 0
v 340m / s No. of beats heard when fork 2 is sounded
or l = = = 1m with fork 1 = Dn = 4
n 340Hz
First resonating length, Now we know that if on loading (attaching
tape) an unknown fork, the beat frequency
l 1 increases (from 4 to 6 in this case) then the
l1 = = m = 25cm
4 4 frequency of the unknown fork 2 is given
Second resonating length, by, n = n 0 - Dn = 200 – 4 = 196 Hz
3l 3 ´1m æI ö
l2 = = = 75cm. 37. (a) We have, L1 = 10log ç 1 ÷
4 4
Third resonating length, è I0 ø
æI ö
5l 5 ´ 1m L2 = 10 log ç 2 ÷
l3 = = = 125cm.
4 4 è I0 ø
So third resonance is not possible since æI ö æI ö
the length of the tube is 120 cm. \ L1 – L2 = 10 log ç 1 ÷ - 10 log ç 2 ÷
\ Minimum height of water necessary for è I0 ø è I0 ø
resonance = 120 – 75 = 45 cm. æI I ö
or, DL = 10 log ç 1 ´ 0 ÷
gRT è I0 I 2 ø
33. (a) v =
M
PV = RT æI ö
or, DL = 10 log ç 1 ÷
M è I2 ø
P = RT
d æI ö
or, 20 = 10 log ç 1 ÷
P RT è I2 ø
=
d M
æI ö
gP or, 2 = log ç 1 ÷
v= è I2 ø
d
34. (a) y = a sin bx sin wt I1
on comparing with standard equation of or, = 102
I2
stationary wave
I1
y = R sin 2px . sin wt, we get or, I2 = .
l 100
Þ Intensity decreases by a factor 100.
2px
= bx , v v
l 38. (c) n 1 = 256 = =
2p 4 l 1 4 ´ 25 .4
\ l= \ v = 256 × 101.6 cm/s
b
The distance between constructive nodes v 256 ´ 101.6
n2 = = = 254 Hz
l 2p / b p 4l2 4 ´ 25.6
= = =
2 2 b No. of beats/sec = n 1 – n2 = 256 – 254 = 2
Waves P-105

Substituting these values in (ii) we get


2p 2p
39. (b) w= and k = 4mg mg
0.01 0.30 ´x = (L - x)
5 5
w 2p 0.30
v= = ´ = 30 m s -1 L
k 0.01 2p Þ 4x = L - x Þ x =
5
40. (c) Velocity of wave v = nl 42. (a) L1 = 50 cm, L2 = 50.5 cm
v as L2 > L1, so n2 < n1
where n = frequency of wave Þ n = For open pipe,
l
v
v 396 n=
n2 = 2 = = 396 Hz 2L
l 2 100 ´ 10-2 n1 – n2 = 3 beats/s
no. of beats = n1 –n2 = 4
41. (a) Frequency of 1st harmonic of AB væ 1 1 ö
\ ç - ÷ =3
2 è L1 L2 ø
1 TAB
= v æ 1 1 ö
2l m \ -
-2 çè 50 50.5 ÷ø = 6
Frequency of 2nd harmonic of CD 10
6 ´ 50 ´ 50.5 ´ 10 -2
1 TCD \ v= = 303 m/s
= 0.5
l m
c
S
A C v
43. (a) D D

l AB l CD = l c S v
=l
2 TAB TCD Largest frequency (f1) Lowest frequency (f2)

O Largest frequency will be detected when


B D the source approaches detector along the
x L–x
line joining and the smallest frequency will
be detected when the source recedes the
m detector along the line joining them
L
æ c ö
Given that the two frequencies are equal. f
f1 çè c - v ÷ø c+v
= =
1 TAB 1 TCD f2 æ c ö c- v
\ = çè ÷f
2l m l m c + vø
44. (c) Frequency of the echo detected by the
TAB
Þ = TCD Þ TAB = 4TCD ....(i) driver of the train is
4 (According to Doppler effect in sound)
For rotational equilibrium of massless rod,
taking torque about point O, æ v + uö
f '=ç f
è v - u ÷ø
TAB ´ x = TCD (L - x ) ....(ii)
where f = original frequency of source of
For translational equilibrium,
sound
TAB + TCD = mg ....(iii) f ¢ = Apparent frequency of source because
mg of the relative motion between source and
On solving, (i) & (iii) we get, TCD = ; observer.
5
4mg æ 330 + 220 ö
f '=ç 1000 = 5000 Hz
\ TAB = è 330 - 220 ÷ø
5
EBD_7100
P-106 Topicwise AIIMS Solved Papers – PHYSICS
Type B : Assertion Reason Questions P
remains constant. Hence there is no
45. (b) Sound travels faster in solids than gases. r
It is because the elasticity of solid is more effect of the pressure change on the speed
than that of gases. Solids posses greater of sound.
density than gases. Though density has v
50. (a) Pitch is related to frequency and f =
effect on the velocity of sound in the l
medium as follows 51. (b) For the formation of stationary waves, it is
necessary that the medium should not be
1
vµ unlimited but it should have a boundary.
r The wave propagating in such a medium
In case of solid, its elasticity far exceeds will reflect at the boundary and produce a
that of gas so its effect far exceeds the effect wave of the same kind travelling in the
of density. opposite direction. The position of two
46. (a) Since wavelength is distance between two waves will give rise to a stationary wave.
nearest particles in phase and time period At free end, transverse wave is reflected
is time required by a wave to cover this without change of phase. Hence as
distance. essential requirement for the formation of
stationary wave is that the medium must
wavelength
So speed of wave = be bounded having definite boundaries. In
time period stationary waves, there are certain points
47. (a) When beettle moves along the sand it of the medium, which are permanently at
sends two sets of pulses, one longitudinal rest i.e., their displacement is zero
and the other transverse. Scorpion has the throughout. These points are called nodes.
capacity to intercept the waves. By getting Similarly, there are some other points which
a sense of time interval between receipt of vibrate about their mean position with
these two waves, it can determine the largest amplitude. These points are called
distance of bettle also. antinodes.
48. (b) Longitudinal waves travel or propagate by 52. (a)
compression and rarefaction of the medium 53. (d) Reason is correct, Assertion is incorrect.
particles hence in absence of a medium In doppler for sound wave effect due to
they cannot propagate. observer and source motion are different.
Light waves are made of perpendicular 54. (d) Two waves moving in uniform string with
electric and magnetic field vectors normal uniform tension shall have same speed and
to direction of motion. Hence, they are may be moving in opposite directions.
transverse waves and if plane of vibration Hence both waves may have velocities in
is same for a wave then it is polarised but opposite direction. Hence Assertion is
this is not so for sound waves. incorrect.
49. (d) Speed of sound in cases is independent of 55. (c) Laplace assumed adiabatic process during
sound propagation.
gP v
pressure because v = 56. (a) As f = ; and so with increase in
p 2l
At constant temperature, if P changes then temperature v increases more than l.
r also changes in such a way that the ratio gRT
Also v=
M
15 Electric Charges and Fields

TYPE A : MULTIPLE CHOICE QUESTIONS 7. How many electrons make up a charge of 20 µC.
[2002]
1. A body can be negatively charged by : [1998] (a) 1.25 × 1014 (b) 2.23 × 1014
(a) removing some neutrons from it (c) 3.25 × 1014 (d) 5.25 × 1014
(b) giving excess electrons to it 8. A conducting sphere of radius 10 cm is charged
(c) removing some protons from it with 10 µC. Another uncharged sphere of radius
(d) removing some electrons from it 20 cm is allowed to touch it for some time. After
2. The number of electrons for one coulomb of that if the spheres are separated, then surface
charge are: [1999] density of charges on the spheres will be in the
(a) 6.25 × 10 23 (b) 6.25 × 10 21 ratio of [2002]
(c) 6.25 × 1018 (d) 6.25 × 1019 (a) 1 : 1 (b) 2 : 1
3. Let Ea be the electric field due to a dipole in its (c) 1 : 3 (d) 4 : 1
axial plane distant l and Eq be the field in the 9. An electric dipole placed in a non-uniform electric
equatorial plane distant l', then the relation field experiences : [2003]
between Ea and Eq will be : [2000] (a) both, a torque and a net force
(b) only a force but no torque
(a) Ea = 4Eq (b) Eq = 2Ea
(c) only a torque but no net force
(c) Ea = 2Eq (d) Eq = 3Ea
(d) no torque and no net force
4. A particle of mass 2g and charge 1mC is held at 10. Three charges are placed at the vertices of an
a distance of 1m from a fixed charge 1mC. If the equilateral triangle of side 'a' as shown in the
particle is released it will be repelled. The speed following figure. The force experienced by the
of particle when it is at a distance of 10 metre charge placed at the vertex A in a direction normal
from the fixed charge is [2000] to BC is : [2003]
(a) 90 m/s (b) 100 m/s A
(c) 45 m/s (d) 55 m/s +Q
5. What is the electric flux associated with one of
faces of a cube, when a charge (q) is enclosed in
the cube ? [2001]
6q q –Q +Q
(a) (b)
e0 6e 0 B C
q 3q (a) Q2/(4pe0a2) (b) –Q2/(4pe0a2)
(c) 3e (d) e (c) zero (d) Q2/(2pe0a2)
0 0
6. The point charges Q and –2Q are placed at some 11. Shown below is a distribution of charges. The
distance apart. If the electric field at the location flux of electric field due to these charges through
of Q is E. The electric field at the location of Q is the surfaces S is : [2003]
E. The electric field at the location of –2Q will be
(a) 3q/e0 +q –q
3E
(a) - (b) – E [2001] (b) 2q/e0
2
E
(c) q/e0
(c) - (d) -2E (d) zero +q
2
EBD_7100
P-108 Topicwise AIIMS Solved Papers – PHYSICS
12. The electric field due to a uniformly charged non- charged to QA and – QB (| QB | > | QA |). The
conducting sphere of radius R as a function of electrical field along a line, (passing through the
the distance from its centre is represented centre) is: [2005]
graphically by [2004]

V
E E (a)
(a) (b)
0 rA rB x
R r R r

E E V
(b)
(c) (d)

R r R r 0 rA rB x
13. In the basic CsCl crystal structure, Cs+ and Cl–
ions are arranged in a bcc configuration as shown
in the figure. The net electrostatic force exerted (c)
V
by the eight Cs+ ions on the Cl– ion is : [2004]
+ +
Cs Cs
0 rA rB x
+
Cs

Cl a
V
(d)
+ +
Cs Cs 0 rA rB x
a
16. A particle having charge q and mass m is projected
+ a +
Cs Cs with velocity v = 2 î - 3 ĵ in a uniform electric
2 2 uur
1 4e 1 16e field E = E 0 . ĵ. Change in momentum | D p |
(a) (b)
4pe 0 3a 2 4pe 0 3a 2 during any time interval t is given by : [2005]
(a) qE0 t (b) qE 0 t
1 32e 2
(c)
4pe 0 3a 2
(d) zero (c) qE 0 t (d) zero
m
14. Two infinitely long parallel conducting plates 17. Two parallel large thin metal sheets have equal
having surface charge densities +s and –s surface charge densities (s = 26.4 × 10–12 c/m2)
respectively, are separated by a small distance. of opposite signs. The electric field between
The medium between the plates is vacuum. If e0 these sheets is : [2006]
is the dielectric permittivity of vacuum then the
electric field in the region between the plates is : (a) 1.5 N/C (b) 1.5 ´ 10 -10 N / C
[2005] (c) 3 N/C (d) 3 ´ 10 -10 N / C
(a) 0 volt/m (b) s / 2e 0 volt/m
18. The spatial distribution of the electric field due
(c) s / e0 volt/m (d) 2s / e 0 volt/m to two charges (A, B) is shown in figure. Which
15. Two concentric conducting thin spherical shells one of the following statements is correct ?
A and B having radii rA and rB (rB > rA) are
[2006]
Electric Charges and Fields P-109

22. The electric field at a distance r from the centre


A in the space between two concentric metallic
B spherical shells of radii r1 and r 2 carrying charge
Q1 and Q2 is (r1 < r < r2) [2009]
Q1 + Q 2 Q1 + Q 2
(a) A is +ve and B –ve; |A| > |B| (a) 2 (b)
(b) A is –ve and B +ve; |A| = |B| : 4p Î0 (r1 + r2 ) 4p Î0 r 2
(c) Both are +ve but A > B Q1 Q2
(d) Both are –ve but A > B (c) 2 (d)
4p Î0 r 4p Î0 r 2
19. Three point charges +q, –2q and +q are placed
23. The potential at a point P due to an electric dipole
at point (x = 0, y = a, z = 0), (x = 0, y = 0, z = 0) and
is 1.8 × 105V. If P is at a distance of 50 cm apart
(x = a, y = 0, z = 0) respectively. The magnitude
from the centre O of the dipole and if CP makes
and direction of the electric dipole moment
an angle 60° with the positive side of the axial
vector of this charge assembly are [2008]
line of the dipole, what is the moment of the
(a) 2 qa along + y direction dipole? [2010]
(a) 10 C–m (b) 10–3 C –m
(b) 2qa along the line joining points (c) 10–4 C–m (d) 10–5 C–m
(x = 0, y = 0, z = 0) and (x = a, y = a, z = 0) 24. The figure shows two situations in which a
(c) qa along the line joining points Gaussian cube sits in an electric field. The arrows
(x = 0, y = 0, z = 0) and (x = a, y = a, z = 0) and values indicate the directions and
magnitudes (in N–m2 /C) of the electric fields.
(d) 2qa along + x direction What is the net charge (in the two situations)
20. A hollow cylinder has charge q C within it. If f is inside the cube? [2011]
the electric flux in unit of voltmeter associated
with the curved surface B, the flux linked with
the plane surface A in unit of voltmeter will be
[2008]

C A
(a) (1) negative (2) positive
(b) (1) negative (2) zero
1æ q ö q (c) (1) positive (2) positive
(a) ç - f÷ (b)
2 è e0 ø 2e 0 (d) (1) positive (2) zero
25. There exists a non-uniform electric field along
q q x-axis as shown in the figure below. The field
(c) (d) -f increases at a uniform rate along +ve x-axis. A
e0 e0
dipole is placed inside the field as shown.
21. Charge q is uniformly distributed over a thin half Which one of the following is correct for the
ring of radius R. The electric field at the centre of dipole? [2012]
the ring is [2008]
q q
(a) 2 2 (b) –q
2p e 0 R 4p e 0 R 2
2
a
q q +q x-axis
(c) 2 (d) 2
4 pe 0 R 2 pe 0 R
EBD_7100
P-110 Topicwise AIIMS Solved Papers – PHYSICS
(a) Dipole moves along positive x-axis and 30. The electrostatic potential inside a charged
undergoes a clockwise rotation spherical ball is given by f = ar2 + b where r is
(b) Dipole moves along negative x-axis and the distance from the centre a, b are constants.
undergoes a clockwise rotation Then the charge density inside the ball is [2015]
(c) Dipole moves along positive x-axis and (a) –6ae0r (b) –24pae0
undergoes a anticlockwise rotation (c) –6ae0 (d) –24pae0r
(d) Dipole moves along negative x-axis and 31. An infinitely long solid cylinder of radius R has
undergoes a anticlockwise rotation a uniform volume charge density r. It has a
26. Two point charges +q and –q are held fixed at spherical cavity of radius R/2 with its centre on
(–d, 0) and (d, 0) respectively of a x – y the axis of the cylinder, as shown in the figure.
coordinate system. Then [2013] The magnitude of the electric field at the point P,
(a) the electric field E at all points on the axis which is at a distance 2R from the axis of the
has the same direction 23rR
(b) work has to be done in bringing a test cylinder, is given by the expression . The
16Ke0
charge from ¥ to the orgin value of k is [2016]
(c) electric field at all points on y-axis is along
x-axis
(d) the dipole moment is 2qd along the x-axis
27. A charged particle q is placed at the centre O of
cube of length L (A B C D E F G H). Another
same charge q is placed at a distance L from O.
Then the electric flux through ABCD is [2013]
E F
D
C
O
q q
H
G (a) 6 (b) 5
A (c) 7 (d) 4
B
L
(a) q /4pÎ0L (b) zero 32. An electric dipole of moment P is placed in a
(c) q/2 pÎ0L (d) q/3pÎ0L uniform electric field E such that P points
28. In a medium
r of dielectric constant K, the electric along E . If the dipole is slightly rotated about
field is E . If Î0 is permittivity of the free space, an axis perpendicular to the plane containing
the electric displacement vector is [2014]
r r E and P and passing through the centre of
KE E the dipole, the dipole executes simple harmonic
(a) (b)
Î0 K Î0 motion. Consider I to be the moment of inertia of
r
Î0 E r the dipole about the axis of rotation. What is the
(c) (d) K Î0 E time period of such oscillation? [2016]
K
29. Three charge q, Q and 4q are placed in a straight (a) ( pE / I) (b) 2p ( I / pE)
1
line of length l at points distant 0, and l (c) 2p ( I / 2pE) (d) None of these
2
respectively from one end. In order to make the 33. A hollow insulated conduction sphere is given
net froce on q zero, the charge Q must be equal a positive charge of 10 mC. What will be the
to [2015] electric field at the centre of the sphere if its
(a) –q (b) – 2q radius is 2 m? [2017]
(a) Zero (b) 5 mCm–2
-q
(c) (d) q (c) 20 mCm–2 (d) 8 mCm–2
2
Electric Charges and Fields P-111

TYPE B : ASSERTION REASON QUESTIONS 39. Assertion : Four point charges q1, q2, q3 and q4
are as shown in figure. The flux over the shown
Directions for (Qs. 34-38) : These questions consist Gaussian surface depends only on charges q1
of two statements, each printed as Assertion and and q2.
Reason. While answering these questions, you are q4
required to choose any one of the following five
responses.
q1
(a) If both Assertion and Reason are correct and
the Reason is a correct explanation of the
q2
Assertion. Gaussian
(b) If both Assertion and Reason are correct but surface
Reason is not a correct explanation of the q3
Assertion.
(c) If the Assertion is correct but Reason is incorrect. Reason : Electric field at all points on Gaussian
(d) If both the Assertion and Reason are incorrect. surface depends only on charges q1 and q2.
(e) If the Assertion is incorrect but the Reason is [2012]
correct. 40. Assertion : Consider two identical charges
34. Assertion: Electron move away from a region of placed distance 2d apart, along x-axis.
higher potential to a region of lower potential.
Reason: An electron has a negative charge.
[1999]
35. Assertion : A metallic shield in form of a hollow
shell may be built to block an electric field.
Reason : In a hollow spherical shield, the electric
field inside it is zero at every point. [2001] The equilibrium of a positive test charge placed
36. Assertion : Electric lines of force never cross at the point O midway between them is stable
each other. for displacements along the x-axis.
Reason : Electric field at a point superimpose to Reason: Force on test charge is zero. [2013]
give one resultant electric field. [2002]
37. Assertion : The Coulomb force is the dominating 41. Assertion : A deuteron and an a-particle are
force in the universe. placed in an electric field. If F1 and F2 be the
Reason : The Coulomb force is weaker than the forces acting on them and a1 and a2 be their
gravitational force. [2003] accelerations respectively then, a1 = a2.
38. Assertion : In a cavity within a conductor, the Reason : Forces will be same in electric field.
electric field is zero. [2015]
Reason : Charges in a conductor reside only at 42. Assertion : In the absence of an external electric
its surface. [2007] field, the dipole moment per unit volume of a
Directions for (Qs. 39-43) : Each of these questions
polar dielectric is zero.
contains an Assertion followed by Reason. Read them
Reason : The dipoles of a polar dielectric are
carefully and answer the question on the basis of
randomly oriented. [2016]
following options. You have to select the one that
best describes the two statements. 43. Assertion : The positive charge particle is placed
(a) If both Assertion and Reason are correct and in front of a spherical uncharged conductor. The
Reason is the correct explanation of Assertion. number of lines of forces terminating on the
(b) If both Assertion and Reason are correct, but sphere will be more than those emerging from it.
Reason is not the correct explanation of Reason : The surface charge density at a point
Assertion. on the sphere nearest to the point charge will be
(c) If Assertion is correct but Reason is incorrect. negative and maximum in magnitude compared to
(d) If both the Assertion and Reason are incorrect. other points on the sphere. [2017]
EBD_7100
P-112 Topicwise AIIMS Solved Papers – PHYSICS

Type A : Multiple Choice Questions Charge q is lying inside the cube. Flux
q
1. (b) A body can be negatively charged by coming out of it = . It is now distributed
Î0
adding some excess electron to it. evenly over six faces. So, flux passing
2. (c) 1 electron carries a charge of through one face
1.6 × 10–19 coulomb. q 1 q
No. of electron in one coulomb = ´ =
Î0 6 6 Î0
1 6. (c) Field at Q is E. So, force on Q = QE
=
1.6 ´10 -19 This force will be applied on –2Q. Also
according to Coulomb’s law. So, field
1019 10
= = ´1018 = 6.25 ´1018. QE E
1.6 1.6 at –2Q is = .
3. (c) We know that for short dipole, –2Q 2
2p 7. (a) Charge on an electron
field at axial point, E a = = 1.6 × 10–19 coulomb
d3
and field at equatorial point, E q =
p 20 ´ 10 -6
No. of electrons required =
d3 1.6 ´ 10 -19
So, Ea = 2Eq
20
4. (a) Potential at 1 m from the charge = ´ 1013 = 1.25 ´ 1014
1. 6
K.10 -6 8. (b) Let the common potential after the touch is
VA = = K ´10 -6
1 V. So, applying conservation of charge
Potential at 10 m from the charge 10 × 10–6 = V × C1 + V × C2
K.10 -6 10 ´ 10 -6
VB = = K ´ 10 -7 V=
10 (C1 + C 2 )
Potential diff.= VA – VB = K(10–6 – 10–7) Charge on first sphere
Its velocity at 10 m is V, then
10 ´ 10-6
1 = C1V = ´ C1
´ mv2 = (VA - VB ) ´ q (C1 + C 2 )
2
Charge on second sphere
1 æ 1ö
´ 2 ´10-3 ´ v 2 = K ´10-6 ç1 - ÷ ´10-3 10 ´ 10-6
2 è 10 ø = C 2V = ´ C2
(C1 + C 2 )
K ´10-9 ´ 9 9
v2 = = K´ ´10-6 Charge densities are,
-3 10
10 ´10
10 ´10-6 ´ C1 10 ´ 10-6 ´ C2
9 = &
= 9 ´ 10 ´ ´10 -6 = 81 × 100
9
(C1 + C2 )4pr12 (C1 + C2 )4pr2 2
10
v = 90 m/sec C1 r22
5. (b) and their ratio = C ´ 2
2 r1

4p Î0 r1 r22
= ´
4p Î0 r2 r12
r 20
= 2 = = 2 :1
r1 10
[Capacity of spherical capacitor = 4pÎ0R]
Electric Charges and Fields P-113

9. (a) An electric dipole placed in a non-uniform s


electric field experiences a torque and a net Field at P due to plate, A =
2e 0
force. In a uniform field it experiences only
s
torque. Field at P due to plate, B =
2e 0
10. (c) +Q
A Both are acting in the same direction so,
2s s
Total field = =
2e 0 e 0
15. (c) Electric field inside sphere A is zero.
Potential inside is uniform. If we move out
B C of the sphere starting from centre, we find
-Q a +Q
potential dropping to –ve value at the
From the figure it is clear that force on Q surface of B. After that it becomes zero at
due to charges on B and C will be as shown infinity as we take potential at infinity to be
in the figure. Their resultant will be parallel zero.
to BC. So, their component perpendicular 16. (b) Impulse = mv – mu = Change in momentum
to BC will be zero.
= force × time qE0 × t = qE0t
11. (d) Net charge inside the surface is zero. So,
flux through the surface is zero. = Change in momentum
12. (b) Inside a uniformly charged non-conducting 17. (c) Field between two parallel sheet
sphere, charge is uniformly distributed. So, s 26.4 ´ 10 -12
field is there. In such cases electric field is = = =3 N/C
Î0 8.85 ´ 10 -12
directly proportional to the distance from
18. (a) A is positive as electric lines are coming
the centre. Outside the sphere, field is
out of it. B is negative as electric lines are
inversely proportional to (distance)2. So,
entering into it.
graph is as follows 19. (b) The given charge assembly can be
represented using the three co-ordinate axes
r0 x, y and z as shown in figure.
y

E 1
r

Eµ (0, a,0) q A(a, a,0)


r2
a
O r = r0
–2q
13. (d) All Cs ions are symmetrically distributed (0, 0,0) O x
around Cl– so, resultant of all the forces a q(a, 0,0)
acting on Cl– will be zero. p
14. (c) s -s
+ - z
The charge –2q is placed at the origin O.
+ -
One +q charge is place at (a, 0, 0) and the
+ P - other +q charge is placed at (0, a, 0). Thus
+ - the system has two dipoles along x-axis and
A B y-axis respectively.
EBD_7100
P-114 Topicwise AIIMS Solved Papers – PHYSICS
As the electric dipole moment is directed p/2
from the negative to the positive charge \ Total field at centre = 2 ò dE cos q
hence the resultant dipole moment will be 0
uuur
along OA where co-ordinates of point A p/2
klR cos q 2kl
p/2
are (a, a, 0). The magnitude of each =2 ò R2
dq =
R ò cos qdq
dipolemoment, 0 0
p = qa q q
So, the magnitude of resultant dipole = (since l = )
2
2p Î0 R 2 pR
moment is
Q1 Q1
PR = p 2 + p 2 = (qa) 2 + (qa) 2 22. (c) ò E.dx = Î0 Þ E = 4pr 2 Î
0
= 2 qa
20. (a) Let electric flux linked with surfaces A, B,
and C are fA, fB and fC respectively. v
Thus ftotal = fA + fB + fC
Q fA = fC
q
and f total = Î (From Gauss's Law) 1 p cos q
0 23. (d) V=
4pe0 r 2
q
\ Î = 2f A + f B But fB = f (given) Here, V = 1.8 × 105V, q = 60°,
0
r = 50 × 10–2 = 0.5 m
q
Hence, Î = 2f A + f p cos 60°
0 \ 1.8 ´ 105 = 9 ´ 109 ´
q 1æ q ö (0.5) 2
or Î - f = 2f A or fA = 2 ç Î - f÷
0 è 0 ø 1.8 ´ 105 ´ 0.25 ´ 2
21. (a) or p = = 10–5C–m
+ + + 9 ´ 109
+ + 24. (a) The field lines in 1 are :
+ + (6 + 7)out – (2 + 7+ 15 + 8)in = 19 (in)
+ +
It represents negative charge.
+ +
dq The field lines in 2 are :
+ q + (9 + 5 + 6) out – (7+ 3 + 2) in = 8 (out)
+ C + It represents positive charge.
R 25. (d) The dipole is placed in a non-uniform field,
q therefore a force as well as a couple acts on
it. The force on the negative charge is more
dE
(F µ E) and is directed along negative
dE cos q
x-axis. Thus the dipole moves along
From figure, dl = Rdq
negative x-axis and rotates in an
Charge on dl = lRdq , anticlockwise direction.
where l = linear charge density .
E 1q –q
Electric field at centre due to dl
lRdq
dE = k. 2 a
E
R
We need to consider only the component
dE cos q, as the component dE sin q will +q E2q
cancel out.
Electric Charges and Fields P-115

26. (c) If we take a point M on the X-axis as shown rR


in the figure, then the net electric field is in E1 = directed towards the +Y
4e 0
X-direction. direction
\ Option (a) is incorrect.
(ii) The electric field created due to the
If we take a point N on Y-axis, we find net
electric field along +X direction. The same spherical negative charge density
will be true for any point on Y-axis. (c) is a rR
correct option. E2 = directed towards th e –Y
96e 0
Wµ0 = q (Vµ - V0 ) = q(0 - 0) = 0 direction.
\ (b) is incorrect. The direction of dipole \ The net electric field is
moment is from –ve to +ve. Therefore (d) is é 23rR ù
1
incorrect. E = E1 - E 2 = ê ú
27. (b) The flux for both the charges exactly cancels ë 16e0 û
6
the effect of each other. 32. (b) The dipole experiences a torque pE sin q
tending to bring itself back in the direction
28. (d) Electric displacement vector, D = eE of field.
Therefore, on being released (i.e. rotated)
As, e = e 0 K \ D = e0K E the dipole oscillates about an axis through
(Fnet )q = 0 its centre of mass and perpendicular to the
29. (a)
field. If I is the moment of inertia of the
Qq 4q 2 dipole about the axis of rotation, then the
Þ k +k =0
2
l2 equation of motion is
ælö
ç ÷ I.d2q/dt2 = – pE sin q
è2ø For small amplitude sin q » q
l/2 l/2
Thus d 2 q / dt 2 = -(pE / I).q = -w 2 q
q Q 4q
1 where w = ( pE / I) .
where k =
4pe0 This is a S.H.M., whose period of oscillation
Þ 4Qq + 4q2 = 0 is T = 2p / w = 2p ( I / pE) .
Þ Q=–q 33. (a) Charge resides on the outer surface of a
df conducting hollow sphere of radius R. We
30. (c) Electric field, E = - = -2ar
dt consider a spherical surface of radius r < R.
q By Gauss theorem
By Gauss's theorem E (4pr 2 ) = ++ +++
e0 ++ +
+ +
+
Þ q = -8pe 0 ar 3 +
+ R
+
+
+ O +
dq dq dr + S +
r= = ´ + r +
dV dr dV + E +
+ +
+ +
æ 1 ö ++ + + +
= (–24pe 0 ar 2 ) ç +
è 4pr 2 ÷ø = –6e 0 a rr 1
31. (a) We suppose that the cavity is filled up by a òsE.ds = e0 ´ char ge enclosed or
positive as well as negative volume charge
1
of r. So the electric field now produced at P E ´ 4 pr 2 = ´0 ÞE =0
e0
is the superposition of two electric fields.
(i) The electric field created due to the i.e., electric field inside a hollow sphere is
zero.
infinitely long solid cylinder is
EBD_7100
P-116 Topicwise AIIMS Solved Papers – PHYSICS

Type B : Assertion Reason Questions applied and induced electric fields are equal
and opposite thus the net charge resides
34. (e) Direction of electric field is from region of
on surface only. If any cavity is there inside
high potential to low potential & electron
the conductor, electric field will be zero
or any -ve charged particle will move against
in it.
the field or lower potential to higher
39. (d) Electric field at any point depends on
potential.
presence of all charges.
35. (a) A metallic shield may be used to block an
40. (b) If +ve charge is displaced along x-axis, then
electric field because field inside a metallic
net force will always act in a direction
shield is zero.
opposite to that of displacement and the
36. (b) Electric lines of force never cross each
test charge will always come back to its
other. Electric field at a point add up
original position.
vectorally to give one resultant electric field.
41. (c) qd = e, md = 2mp = 2m
So, they do not have independent existence
at the point of superposition so, electric qa = 2e, ma = 4mp = 4m
lines of force do not cross each other F1 = Fa = eE, F2 = Fa = 2eE ¹ F1
(crossing of electric lines of force at a point
means at a point two fields are having F1 eE
independent existence). Further, a1 = =
2m 2m
37. (d) Gravitational force is the dominating force
in the universe so Assertion is incorrect. F2 2eE eE
and a 2 = = = = a1
Gravitational force is weaker than 2m 4m 2m
Coulombic force so, Reason is incorrect. 42. (a)
38. (a) Net field inside the conductor is zero 43. (d) No. of lines entering the surface = No. of
because by virtue of induced charges, lines leaving the surface.
Electrostatic Potential
16 and Capacitance
TYPE A : MULTIPLE CHOICE QUESTIONS 6. The capacitors C1, C3, C4, C5 have a capacitance
4µF each and C2 has capacitance 10µF. The
1. A condenser of capacitor 50mF is charged to 10 effective capacitance between P and Q will be :
volt, the energy stored is : [1997]
(a) 5 ´ 10 -3 J (b) 2.5 ´ 10 -3 J C4

(c) 3.75 ´ 10 -3 J (d) 1.25 ´ 10 -3 J b d


P Q
2. If the sep aration between the plates of a capacitor a c
C1 C2 C3
is 5 mm, then the area of the plate of a 3F parallel
plate capacitor is : [1998] C5
(a) 4.259 ´ 109 m2 (b) 1.964 ´ 109 m2
(c) 12.81 ´ 109 m2 (d) 1.964 ´ 109 m2 (a) 8 µF (b) 6 µF [2002]
3. Minimum numbers of 8mF and 250 V capacitors (c) 4 µF (d) 2 µF
are used to make a combination of 16mF and 7. A 40 µF capacitor in a defibrillator is charged to
1000V are: [2000] 3000 V. The energy stored in the capacitor is set
(a) 4 (b) 32 through the patient during a pulse of duration 2
(c) 8 (d) 3 ms. The power delivered to the patient is :[2004]
4. An insulated charged sphere of radius 5 cm has (a) 45 kW (b) 90 kW
a potential of 10 V at the surface. The potential (c) 180 kW (d) 360 kW
at the centre will be [2000] 8. Equipotential surfaces associated with an
(a) same as that at 5 cm from the surface electric field which is increasing in magnitude
(b) same as that at 25 cm from the surface along the x-direction are : [2004]
(c) 10 V (a) planes parallel to yz-plane
(d) zero (b) planes parallel to xy-plane
5. Two materials having the dielectric constants (c) planes parallel to xz -plane
K1 and K2 are filled between two parallel plates (d) coaxial cylinders of increasing radii around
of a capacitor, which is shown in figure. The the x-axis
capacity of the capacitor is : [2001] 9. Five capacitors, each of capacitance value C are
connected as shown in the figure. The ratio of
capacitance between P & R, and the capacitance
between P & Q, is : [2006]
K1 K2 P
C C

Ae 0 (K1 ´ K 2 ) Ae 0 ( K 1 - K 2 )
(a) A (b)
d (K1 + K 2 ) d C C

Ae 0 K1K 2 Ae 0 ( K 1 + K 2 ) R S
(c) (d) C
(K1 + K 2 ) d (a) 3 : 1 (b) 5 : 2
(c) 2 : 3 (4) 1: 1
EBD_7100
P-118 Topicwise AIIMS Solved Papers – PHYSICS
10. A circle of radius R is drawn with charge + q at 15. Two charges of magnitude + q and – 3q are
the centre. A charge q0 is brought from point B placed 100 cm apart. The distance from + q
to C, then work done is B [2009] between the charges where the electrostatic
q0 potential is zero is : [2011]
(a) positive
(a) 25 cm (b) 50 cm
(b) negative +q (c) 75 cm (d) 80 cm
(c) zero C r
(d) infinite 16. If a dipole of dipole moment p is placed in a
r
11. A parallel plate air capacitor has a capacitance uniform electric field E , then torque acting on
C. When it is half filled with a dielectric of it is given by [2012]
dielectric constant 5, the percentage increase in r rr r r r
the capacitance will be [2009] (a) t = p.E (b) t = p ´ E
(a) 400% (b) 66.6% r r r r r r
(c) t = p + E (d) t = p - E
(c) 33.3% (d) 200%
12. A capacitor is charged by using a battery which 17. The four capacitors, each of 25 m F are connected
is then disconnected. A dielectric slab is as shown in Fig. The dc voltmeter reads 200 V.
introduced between the plates which results in The charge on each plate of capacitor is [2012]
(a) increase in the potential difference across V
the plates and reduction in stored energy – + – +
but no change in the charge on the plates
(b) decrease in the potential difference across
– + – +
the plates and reduction in the stored
energy but no change in the charge on the
plates
(c) reduction of charge on the plates and
(a) ± 2 ´ 10 -3 C (b) ± 5 ´ 10 -3 C
increase of potential difference across the (c) ± 2 ´ 10 -2 C (d) ± 5 ´ 10 -2 C
plates 18. A network of four capacitors of capacity equal to
(d) increase in stored energy but no change in C1 = C, C2 = 2C, C3 = 3C and C4 = 4C are conducted
potential difference across the plates to a battery as shown in the figure. The ratio of
[2010] the charges on C2 and C4 is [2012]
13. The maximum electric field that can be held in air
without producing ionisation of air is 107 V/m. (a) 4/7
The maximum potential therefore, to which a
conducting sphere of radius 0.10 m can be (b) 3/22
charged in air is [2010]
(a) 109 V (b) 108 V (c) 7/4
(c) 107 V (d) 106 V
14. Three capacitors C1, C2 and C3 are connected as (d) 22/3
shown in the figure to a battery of V volt. If the 19. In a hollow spherical shell, potential (V) changes
capacitor C3 breaks down electrically the change with respect to distance (s) from centre as [2013]
in total charge on the combination of capacitors is (a) (b)
C2 [2010]
V V
C1
V
C3 S S
(c) (d)
(a) (C1 + C2) V [1– C3/(C1 + C2 + C3)]
(b) (C1 + C2) V [1–(C1 + C2)/(C1 + C2 + C3)] V V
(c) (C1 + C2) V [1+ C3/(C1 + C2 + C3)]
(d) (C1 + C2) V [1– C2/(C1 + C2 + C3)] S
S
Electrostatic Potential and Capacitance P-119

20. What is the effective capacitance between points TYPE B : ASSERTION REASON QUESTIONS
X and Y? C1 = 6m F [2014]
(a) 24 mF Directions for (Qs. 24-26) : These questions consist
C3 = 6m F C5 = 20m F C2 = 6m F of two statements, each printed as Assertion and
(b) 18 mF X Y Reason. While answering these questions, you are
A C B D
(c) 12 mF C4 = 6m F required to choose any one of the following five
(d) 6 mF responses.
21. The four capacitors, each of 25 m F are (a) If both Assertion and Reason are correct and
connected as shown in fig. The dc voltmeter the Reason is a correct explanation of the
reads 200 V. The charge on each plate of Assertion.
capacitor is [2015] (b) If both Assertion and Reason are correct but
Reason is not a correct explanation of the
V
Assertion.
– + – + (c) If the Assertion is correct but Reason is
incorrect.
– + – + (d) If both the Assertion and Reason are incorrect.
(e) If the Assertion is incorrect but the Reason is
correct.
(a) ± 2 ´ 10 -3 C (b) ± 5 ´ 10 -3 C 24. Assertion : If the distance between parallel
(c) ± 2 ´ 10 -2 C (d) ± 5 ´ 10 -2 C plates of a capacitor is halved and dielectric
constant is three times, then the capacitance
22. A charge +q is fixed at each of the points x = x0,
becomes 6 times.
x = 3x0, x = 5x0, .... upto ¥ on X-axis and charge
Reason : Capacity of the capacitor does not
–q is fixed on each of the points x = 2x0, x = 4x0,
depend upon the nature of the material. [1997]
x = 6x0, .... upto ¥ . Here x0 is a positive constant.
25. Assertion : If three capacitors of capacitances
Take the potential at a point due to a charge Q at
C1 < C2 < C3 are connected in parallel then their
Q equivalent capacitance CP > CS.
a distance r from it to be . Then the
4pe 0 r 1 1 1 1
potential at the origin due to above system of
Reason : = + + [2002]
C p C1 C 2 C3
charges will be [2016]
26. Assertion : A parallel plate capacitor is
q connected across battery through a key. A
(a) zero (b)
8pe 0 x 0 log e 2 dielectric slab of dielectric constant K is
introduced between the plates. The energy which
q log e 2 is stored becomes K times. [2008]
(c) infinity (d)
4pe 0 x 0 Reason : The surface density of charge onthe
plate remains constant or unchanged.
23. Two spherical conductors A and B of radii a and
Directions for (Qs. 27-33) : Each of these questions
b (b>a) are placed concentrically in air. The two
contains an Assertion followed by Reason. Read them
are connected by a copper wire as shown in
carefully and answer the question on the basis of
figure. Then the equivalent capacitance of the
following options. You have to select the one that
system is [2017]
best describes the two statements.
ab B
(a) 4pe 0 b (a) If both Assertion and Reason are correct and
b-a A
Reason is the correct explanation of Assertion.
(b) 4pe 0 (a + b) a (b) If both Assertion and Reason are correct, but
Reason is not the correct explanation of
Assertion.
(c) 4pe 0 b
(c) If Assertion is correct but Reason is incorrect.
(d) 4pe 0a (d) If both the Assertion and Reason are incorrect.
EBD_7100
P-120 Topicwise AIIMS Solved Papers – PHYSICS

27. Assertion : The total charge stored in a 31. Assertion : Charges are given to plates of two
capacitor is zero. plane parallel plate capacitors C1 and C2 (such
Reason : The field just outside the capacitor is that C2 = 2C1) as shown in figure. Then the key
s K is pressed to complete the circuit. Finally the
. (s is the charge density). [2009] net charge on upper plate and net charge the
e0
circuit. Finally the net charge on upper plate and
28. Assertion : The electrostatic force between the net charge on lower plate of capacitor C1 is
plates of a charged isolated capacitor decreases positive.
when dielectric fills whole space between plates. K
Reason : The electric field between the plates of
a charged isolated capacitance increases when
dielectric fills whole space between plates. 2m C 4m C
[2009] C1 C2
29. Assertion : Two concentric charged shells are 4m C 8m C
given. The potential difference between the
shells depends on charge of inner shell.
Reason : In a parallel plate capacitor both plates
always carry equal and opposite charge. [2012]
32. Assertion : For a non-uniformly charged thin
circular ring with net charge is zero, the electric
field at any point on axis of the ring is zero.
Reason : For a non-uniformly charged thin
Reason : Potential due to charge of outer shell circular ring with net charge zero, the electric
remains same at every point inside the sphere. potential at each point on axis of the ring is zero.
[2010] [2015]
30. Assertion : Two equipotential surfaces cannot 33. Assertion : Electric potential and electric
potential energy are different quantities.
cut each other.
Reason : For a system of positive test charge
Reason : Two equipotential surfaces are parallel
and point charge electric potential energy =
to each other. [2011]
electric potential. [2017]
Electrostatic Potential and Capacitance P-121

Type A : Multiple Choice Questions So, central capacitor, that is C2 becomes


ineffective. Now equivalent circuit will
become as follows.
1 Q2 1
1. (b) Energy of condenser = = ´ CV 2
2 C 2 C C

1 -3
= ´ 50 ´10 - 6 ´10 ´10 = 2.5 ´ 10 joule
2 P Q
A Î0 A ´ 8.85 ´10 -12 C C
2. (d) C= Þ 3=
d 0.005 Now, two C are in series, their equivalent
capacity
0.015 15 ´ 10 -3 ´ 1012
A= =
8.85 ´ 10 -12 8.85 C2 C C
= = and then two are in parallel
9 2 2C 2 2
= 1.694 ´10 m so their equivalent capacity will be
3. (b) 1000 V C C
= + =C
2 2
So, total capacity = C = 4µF
7. (b) Energy given during this time period
1 1
= CV 2 = ´ 40 ´ 10-6 ´ 3000 ´ 3000
To create 1000 V, we need to combine 4 2 2
capacitors in series. Total capacity = 2 ´ 9 ´ 10 joule
8µF Work 90 ´ 2
becomes = = 2µF . In order to obtain Power = = = 90 ´ 103 W
4 Time 2 ´ 10-3
capacity of 16µF, 8 rows of this combination = 90 kW
will be needed in parallel.
8. (a) Equipotential surface is always
Total capacity = 2µF ´ 8 = 16µF perpendicular to the direction of electric
Total number of capacitor = 4 ´ 8 = 32 field. As the field is along x-direction,
4. (c) We know that in case of hollow sphere equipotential surface must be parallel to
potential at the surface is equal to potential yz-plane.
at the centre. So potential at the centre will 9. (c) In the first case, 3C and 2C are parallel.
be 10 V. Equivalent capacitance is,
Î AK C C 5C
5. (d) Capacity of Ist capacitor, C1 = 0 1 + =
d 3 2 6
Î0 AK 2 In the second case 1C & 4C are parallel.
Capacity of 2nd capacitor, C 2 = C C 5C
d Equivalent capacitance = + =
Both of these capacitors are parallel so, total 1 4 4
Î0 A 5C 4
capacity, C = C1 + C 2 = (K1 + K 2 ) Ratio = ´ = 2:3
d 6 5C
6. (c) The combination forms Wheatstone bridge 10. (c) Circle represents equipotential surface
\ work done around it = 0
C1 C5
as = Î A
C4 C3 11. (b) Initial capacitance = 0
d
EBD_7100
P-122 Topicwise AIIMS Solved Papers – PHYSICS
When it is half filled by a dielectric of 14. (a) Equivalent capacitance of circuit,
dielectric constant K, then 1 1 1
K e0 A e A = +
C1 = = 2K 0 Ceq C3 C1 + C2
d /2 d
e 0 A 2e 0 A (Since C1 and C2 are in parallel and which
and C2 = = is in series with C3).
d /2 d
C1 C2 1 C + C2 + C3
ie, = 1
Ceq C3 (C1 + C2 )

C3 (C1 + C2 )
\ Ceq =
C1 + C2 + C3
Since V is the voltage of battery,
charge, q = CeqV
C3 (C1 + C 2 )V
d =
1 1 1 d æ1 ö C1 + C2 + C3
\ C ¢ = C + C = 2e A çè K + 1 ÷ø If the capacitor C3 breaks down, then
1 2 0
effective capacitance,
d æ1 ö 6 d
= ç + 1÷ = C'eq = C1 + C 2
2e0 A è 5 ø 10 e0 A
\ New charge q' =
5e0 A '
C¢ = C eq V = (C1 + C2)V
3d
Hence, % increase in capacitance Change in total charge = q' – q
æ 5 e0 A e0 A ö C (C + C 2 )V
- = (C1 + C 2 )V - 3 1
ç ÷ C1 + C2 + C3
=ç 3 d d ÷ × 100
ç e0 A ÷ é C3 ù
ç ÷ = (C1 + C 2 )V ê1 - ú
è d ø ë C1 + C 2 + C3 û
æ5 ö 2 15. (a) q – 3q
= ç -1÷ ´100 = ´100 = 66.6%
è3 ø 3
12. (b) If a dielectric slab of dielectric constant K 1m
is filled in between the plates of a capacitor Suppose x is the required distance from q,
after charging the capacitor (i.e., after then
removing the connection of battery with 1 é 9 (-3q) ù
the plates of capacitor) the potential + =0
difference between the plates reduces to 4pe0 êë x 1 - x úû
1 1
times and the potential energy of After solving for x, we get x = m
K 4
r
1 16. (b) Given : Dipole moment of the dipole = p
capacitor reduces to times but there is
K ®
no change in the charge on the plates. and uniform electric field = E . We know
13. (d) E = 107 V/m that dipole moment (p) = q.a (where q is the
r = 0.10 m charge and a is dipole length). And when a
®
dipole of dipole moment p is placed in
V
E= ®
r uniform electric field E , then Torque (t) =
Either force × perpendicular distance
10 between the two forces = qaEsinq or
V = Er = 107 ×
100 r r r
t = pE sin q or t = p ´ E (vector form)
V = 106 V
Electrostatic Potential and Capacitance P-123

17. (b) Charge on each plate of each capacitor 20. (d)


C1 = 6mF
Q = ±CV = ±25 ´ 106 ´ 200
= ±5 ´ 10 -3 C C3 C5 6mF
X A Y
6mF 20mF C2
18. (b)
C4
C2 C3 C2 C1
6mF
C3 C1 Equivalent circuit
C4
C4
6mF C2
C1 6mF
m
6F
V V 20mF
X C5 m
6F Y
Equivalent capacitance for three capacitors C3 6mF
(C1, C2 & C3) in series is given by 6mF C4
1 1 1 1 C 2C 3 + C 3C1 + C1C 2
= + + =
C eq. C1 C 2 C3 C1C 2C 3
C1 C2
Þ As C = C
3 4
C1C 2 C 3 Hence no charge will flow through 20mF
Ceq. =
C1C2 + C2 C 3 + C 3C1
C1 C2
C(2C)( 3C) 6
Þ Ceq. = = C
C(2C) + (2C)(3C) + (3C) C 11 X Y
Þ Charge on capacitors (C1, C2 & C3)
6C C3 C4
in series = Ceq V = V
11 C'
Charge on capacitor C4 = C4V = 4C V
6C ÞX Y
Charge on C2 V
6 1 3
= 11 = ´ =
Charge on C4 4CV 11 4 22 C''
19. (b) In shell, q charge is uniformly distributed C1 and C2 are in series, also C3 and C4 are in
over its surface, it behaves as a conductor. series.
+ Hence C' = 3 mF, C'' = 3 mF
+ C' and C'' are in parallel hence net
+ q capacitance
R + = C' + C'' = 3 + 3 = 6 mF
+
+ 21. (b) Charge on each plate of each capacitor
+
+ Q = ±CV = ±25 ´ 106 ´ 200
q
V= potential at surface = and = ±5 ´ 10 -3 C
4pe 0 R 22. (d) Potential at origin
q = (V1 + V3 + V5 + .....) – (V2 + V4 + V6 + .....)
inside V =
4pe 0 R
q é 1 1 1 ù
Because of this it behaves as an Þ ê - + .....¥ú
4pe 0 ë x 0 2x 0 3x 0 û
equipotential surface.
EBD_7100
P-124 Topicwise AIIMS Solved Papers – PHYSICS

A B
q é 1 1 1 ù
Þ ê1 - 2 + 3 - 4 .....¥ ú + –
4 pe 0 x 0 ë û + –
+ –
q q + –
Þ log e (1 + 1) Þ log e 2 + –
4pe 0 x 0 4pe 0 x 0 E E
+ –
23. (c) All the charge given to inner sphere will + –
pass on to the outer one. So capacitance + –
+ –
that of outer one is 4p Î0 b . + –
D C
Type B : Assertion Reason Questions
Draw a Gaussian surface ABCD as shown.
24. (c) We know that capacity of capacitor is ur
The field E is uniform on faces AD and
directly proportional to dielectric constant BC
and inversely proportional to distance. So, ur ur
the net effect of making distance halved & Ñò
Þ E .ds = 0 yields E = 0 .
making dielectric constant three times will 28. (d) For isolated capacitor Q = constant,
be capacity becoming six times. F = constant.
As nature of the material (dielectric s
But E = , hence E decreases.
constant) is a factor influencing the K Î0
capacity, therefore, Reason is incorrect. \ Assertion is incorrect and Reason is
25. (c) If three capacitors are joined in parallel then correct.
their equivalent capacitor will be less than 29. (a) A
the least value of capacitor so B
Cp > Cs Q Q2
R1 1
1 1 1 1 R2
= + + is incorrect.
C p C1 C 2 C3 1 Q1 + Q2
VA =
26. (c) In the given cases, V = V0 (remains 4 pe 0 R2
constant). 1 æ Q1 Q2 ö
VB = +
1 4 pe 0 çè R1 R2 ÷ø
2
Energy stored inthe capacitor U = CV 1 æ 1 1 ö
2 VB – VA = Q1 ç - ÷
When a dielectric slab of dielectric constant 4pe 0 è R1 R2 ø
K is introduced between the plates of the 30. (c) Two equipotential surfaces are not
necessarily parallel to each other.
condenser, then C ¾¾ ® KC 31. (d) Charge distribution on each surface makes
So energy stored will become K times . both capacitor of same potential difference
Since Q = CV , So Q will become K times hence charge will not flow.
\ Surface charge density 32. (d) For a non-uniformly charged thin circular
ring with net zero charge, electric potential
KQ at each point on its axis is zero. Hence
s' = = Ks 0
A electric field at each point on its axis must
27. (c) Charge stored on the two plates are + Q be perpendicular to the axis. Therefore
and –Q Þ Q + (– Q) = 0 and hence Assertion Assertion is incorrect and Reason is
is correct. The field however, outside the correct.
plates is zero. 33. (c) Potential and potential energy are different
quantities and cannot be equated.
Current Electricity P-125

17 Current Electricity

TYPE A : MULTIPLE CHOICE QUESTIONS R 2 = 4W

1. A wire of radius r has resistance R. If it is R1 = 2W R 4 = 2W


r A B
stretched to a radius , its resistance will be :
2
[1997]
R 3 = 4W
(a) 16 R (b) 2 R
(c) 4 R (d) zero (a) 4 W (b) 2 W
(c) 8 W (d) 6 W
2. A battery of e.m.f. 20 V and internal resistance
6. Two filaments of same length are connected first
6W is connected to a resistor as shown in figure.
in series then in parallel. For the same amount of
If the current in the circuit is 1 amp, the resistance
main current flowing, the ratio of the heat
of the resistor will be : [1997]
produced is: [1999]
20V (a) 1 : 2 (b) 4 : 1
(c) 1 : 4 (d) 2 : 1
7. Given a current carrying wire of non-uniform
cross-section. Which one of the following is
constant throughout the length of wire ?[2000]
(a) current only
(b) current and drift speed
(c) drift speed only
(a) 14W (b) 16W (d) current, electric field and drift speed
(c) 28W (d) 7W 8. The current in the given circuit is : [2000]
3. The value of current i in the circuit as shown in 10 W 5V
figure is : [1998]
R2= 5W A B

R1= 5W R3= 5W 2V 20 W
3V
(a) 0.3 amp (b) 0.4 amp
(c) 0.1 amp (d) 0.2 amp
R4= 5W 9. Same length of two identical wires are first
connected is series and then in parallel, then the
(a) 1.8 A (b) 0.8 A
amount of heat produced in both the conditions
(c) 0.2 A (d) 1.6 A
are in the ratio : [2000]
4. Electroplating is not provided to be used for
(a) shinning appearance [1998] (a) 1 : 4 (b) 4 : 1
(b) protection of metal against corrosion (c) 3 : 1 (d) 1 : 2
(c) fine finishing to the surface 10. The temperature of the cold junction of a
(d) hardening the metals thermocouple is 0°C and the temperature of the
5. In the given figure, the equivalent resistance hot junction is T°C. The relation for the thermo
between two points A and B will be: [1999] 1
emf is given by; E = AT - BT 2
2
EBD_7100
P-126 Topicwise AIIMS Solved Papers – PHYSICS
(when A = 16 and B = 0.08). The temperature of (a) 1.5 A (b) 3.0 A
inversion will be : [2001]
(c) 15 A (d) 300 A
(a) 500°C (b) 460°C
(c) 600°C (d) 400°C 16. The temperature (T) dependence of resistivity
11. The cell has an emf of 2V and the internal (r) of a semi-conductor is represented by :[2004]
resistance of this cell is 0.1W, it is connected to
a resistance of 3.9W. The voltage across the cell
r r
will be : [2001]
(a) (b)
(a) 1.95 V (b) 1.5 V
(c) 2 V (d) 1.8 V
12. The internal resistance of a cell is the resistance of : T T
(a) electrolyte used in the cell [2001]
(b) electrodes of the cell
(c) vessel of the cell r r
(d) none of these (c) (d)
13. An electric bulb marked 40 W and 200V, is used
in a circuit of supply voltage 100V. Now its power
is [2002] T T
(a) 10 W (b) 20 W 17. For ensuring dissipation of same energy in all
(c) 40 W (d) 100 W three resistors (R1, R2, R3) connected as shown
14. A wire of length L is drawn such that its diameter in figure, their values must be related as
is reduced to half of its original diameter. If the [2005]
initial resistance of the wire were 10 W, its new
resistance would be : [2003] R1
(a) 40 W (b) 80 W
(c) 120 W (d) 160 W
15. Eels are able to generate current with biological v R2 R3
cells called electroplaques. The electroplaques
is an eel are arranged in 100 rows, each row
stretching horizontally along the body of the
fish containing 5000 electroplaques. The (a) R1 = R2 = R3
arrangement is suggestively shown below. Each (b) R2 = R3 and R1 = 4R2
electroplaque has an emf of 0.15 V and internal (c) R2 = R3 and R1 = (1/4)R2
resistance of 0.25 W. The water surrounding the (d) R1 = R2 + R3
eel completes a circuit between the head and its
tail. If the water surrounding it has a resistance 18. The voltage of clouds is 4 × 106 volt with respect
of 500 W, the current an eel can produce in water to round. In a lighteing strike lasting 100 m sec,
is about : [2004] a charge of 4 coulombs is delivered to the ground.
0.15V 0.25W The power of lightening strike is : [2006]
+ – + –
(a) 160 MW (b) 80 MW
(c) 20 MW (d) 500 KW
+ – + –
19. Faraday law of electrolysis indirectly shows
100
5000 electroplaques per row rows (a) quantisation of charge [2007]
(b) quantisation of angular momentum
(c) quantisation of current
(d) quantisation of viscosity
+ – + –
20. Two sources of equal emf are connected to an
external resistance R. The internal resistance of
the two sources are R1 and R2 (R2 > R1). If the
500 W
Current Electricity P-127

potential difference across the source having (a) 9W (b) 12W


internal resistance R2 is zero, then [2008] (c) 15W (d) 8W
(a) R = R2 – R1 24. The resistance between points A and B is [2009]
(b) R = R2 × (R1 + R2)/(R2 – R1) R R R R
A
(c) R = R1R2/(R2 – R1)
R R R ¥
(d) R = R1R2/(R1 – R2) B
21. In the figure shown, the capacity of the R R R R
condenser C is 2mF. The current in 2W resistance
is [2008]
(a) ( )
3 +1 R (b) ( )
3 –1 R
2W (c) 4R (d) ( 3 + 2) R
25. A potentiometer is connected between A and B
and the balance point is obtained at 203. 6 cm.
3W
2µF When the end of the potentiometer connected
4W
to B is shifted to C, then the balance point is
6V obtained at 24.6 cm. If now the potentiometer be
+ – 2.8W connected between B and C, the balance point
(a) 9A (b) 0.9 A will be at
1 1
(c) A (d) A A B C
9 0.9 (a) 179. 0 cm (b) 197.2 cm [2010]
22. When the key K is passed at t = 0, which of the (c) 212.0 cm (d) 228.0 cm
following statements about the current I in the
resistor AB of the given circuit is ture? [2008] 26. Four wires of the same diameter are connected
in turn between two points, maintained at a
A 1000 W B constant potential difference. Their resistivities
2V K are; r and L (wire 1)., 1.2r and 1.2 L (wire 2), 0.9
r and 0.9L (wire 3) and r and 1.5 L (wire 4). Rank
1 µF C 1000 W
the wires according to the rates at which energy
is dissipated as heat, greatest first [2010]
(a) 4 > 3 > 1 > 2 (b) 4 > 2 > 1 > 3
(a) I = 2 mA at all t (c) 1 > 2 > 3 > 4 (d) 3 > 1 > 2 > 4
(b) I oscillates between 1 m A and 2 mA
27. The resistance of a galvanometer is 50 W and
(c) I = 1 mA at all t
current required to give full scale deflection is
(d) At t = 0, I = 2mA and with time it goes to 1
100 µA in order to convert it into an ammeter for
mA
reading upto 10 A. It is necessary to put an
23. What is the equivalent resistance across A and
resistance of [2010]
B in the figure shown, if R = 3W? [2009]
(a) 3.5 × 10–4 W (b) 10 × 10–4 W
A B (c) 2.5 × 10–4 W (d) 5 × 10–4 W
28. Two resistances equal at 0° C with temperature
R R coefficient of resistance a1 and a2 joined in series
act as a single resistance in a circuit. The
R temperature coefficient of their single resistance
will be : [2011]
a1a 2
R (a) a1+ a2 (b)
R a1 + a 2
a1 - a 2 a1 + a 2
(c) (d)
2 2
EBD_7100
P-128 Topicwise AIIMS Solved Papers – PHYSICS
29. When the power delivered by a 100 volt battery 2V
is 40 watts the equivalent resistance of the circuit + – – +
A
is : [2011]
(a) 100 ohms (b) 250 ohms
(c) 300 ohms (d) 350 ohms
2W
30. The electro-chemical equivalent of a substance
is numerically equal to the mass of the substance
deposited if a current I flows through the V
+ –
electrolyte for 0.25 seconds. The value of I is : (II)
(a) 1 A (b) 2 A [2011]
(a) 2 V in circuit I and 0 V in circuit II
(c) 3 A (d) 4 A
(b) 0 V in both circuits
31. Two wires of same metal have the same length
but their cross sections are in the ratio 3 : 1. (c) 2 V in both circuits
They are joined in series. The resistance of the (d) 0 V in circuit I and 2 V in circuit II
thicker wire is 10 W. The total resistance of the 35. Three copper wires of lengths and cross sectional
combination is [2012] areas are (l, A), (2 l, A/2) and (l/2, 2A). Resistance
(a) 5/2 W (b) 40/3 W is minimum in [2013, 2017]
(c) 40 W (d) 100 W (a) wire of cross-sectional area A/2
32. A constant voltage is applied between the two (b) wire of cross-sectional area A
ends of a uniform metallic wire. Some heat is (c) wire of cross-sectional area 2A
developed in it. The heat developed is double if (d) same in all the three cases
[2012] 36. The effective resistance between points P and
(a) both the length and radius of wire are Q of the electrical circuit shown in the figure is
halved [2013]
(b) both length and radius of wire are doubled
2Rr
(c) the radius of wire is doubled (a)
(d) the length of the wire is doubled R+r 2R 2R
33. In the circuit shown in figure, the 5W resistance 8R ( R + r )
develops 20.00 cal/s due to the current flowing (b) 2R
3R + r
through it. The heat developed in 2W resistance P r
Q
(c) 2r + 4R r
(in cal/s) is [2012] 2R 2R
2R
6W 9W 5R
(d) + 2r
2
l 2W l 37. Find out the value of current through 2W
A C 5W D B resistance for the given circuit [2014]

(a) 23.8 (b) 14.2


(c) 11.9 (d) 7.1 5W 10 W
10 V 20 V
34. For the circuits shown in figures I and II, the
2W
voltmeter reading would be [2012]
2V – + (a) zero (b) 2 A
+ – A (c) 5 A (d) 4 A
38. The cold junction of a thermocouple is
maintained at 10ºC. No thermo e.m.f. is
2W
developed when the hot junction is maintained
at 530ºC. The neutral temperature is [2014]
+
V
– (a) 260ºC (b) 265ºC
(I) (c) 270ºC (d) 520ºC
Current Electricity P-129

39. The equivalent resistance between A and B is (a) 1W (b) 2W


A [2015] (c) 3W (d) 4W
R R 44. In the circuit shown, the current in the 1W
resistor is : [2017]
B R 6V P 2W

R R 1W 9V
8R 5R
(a) (b)
5 8 3W W 3W
(a) 0.13 A, from Q to P
3R 7R
(c) (d) (b) 0.13 A, from P to Q
8 8
40. A potentiometer wire, 10 m long, has a resistance (c) 1.3A from P to Q
of 40W. It is connected in series with a resistance (d) 0A
box and a 2 V storage cell. If the potential gradient
TYPE B : ASSERTION REASON QUESTIONS
along the wire is 0.1 m V/cm, the resistance
unplugged in the box is [2015] Directions for (Qs. 45-50) : These questions consist
(a) 260 W (b) 760 W of two statements, each printed as Assertion and
(c) 960 W (d) 1060 W Reason. While answering these questions, you are
41. A current source drives a current in a coil of required to choose any one of the following five
resistance R1 for a time t. The same source drives responses.
current in another coil of resistance R2 for same (a) If both Assertion and Reason are correct and
time. If heat generated is same, find internal the Reason is a correct explanation of the
resistance of source. [2016] Assertion.
R1R 2 (b) If both Assertion and Reason are correct but
(a) (b) R1 + R 2 Reason is not a correct explanation of the
R1 + R 2
Assertion.
(c) zero (d) R 1R 2 (c) If the Assertion is correct but Reason is incorrect.
42. Two long conductors, separated by a distance d (d) If both the Assertion and Reason are incorrect.
carry current I1 and I2 in the same direction. They (e) If the Assertion is incorrect but the Reason is
exert a force F on each other. Now the current in correct.
one of them is increased to two times and its 45. Assertion : In a simple battery circuit, the point
direction is reversed. The distance is also of the lowest potential is positive terminal of the
increased to 3d. The new value of the force battery.
between them is [2016] Reason : The current flows towards the point of
2F F the higher potential, as it does in such a circuit
(a) - (b) from the negative to the positive terminal. [2002]
3 3
46. Assertion : A larger dry cell has higher emf.
F
(c) –2 F (d) - Reason : The emf of a dry cell is proportional to
3 its size. [2004]
43. Twelve resistors each of resistance 16 W are 47. Assertion : A current continues to flow in
connected in the circuit as shown. The net superconducting coil even after switch is off.
resistance between AB is [2016] Reason : Superconducting coils show Meissner
effect. [2007]
48. Assertion : Voltmeter is connected in parallel
with the circuit.
Reason : Resistance of a voltmeter is very
large. [2007]
EBD_7100
P-130 Topicwise AIIMS Solved Papers – PHYSICS
49. Assertion : Ohm's law is applicable for all
conducting elements.
Reason : Ohm's law is a fundamental law. [2007]
50. Assertion : An electric bulb becomes dim, when
the electric heater in parallel circuit is switched
on.
Reason : Dimness decreases after sometime.
[2008]
Directions for (Qs. 51-59) : Each of these questions
contains an Assertion followed by Reason. Read them
carefully and answer the question on the basis of Reason : An ideal voltmeter draws almost no
following options. You have to select the one that current due to very large resistance, and hence
best describes the two statements. (V) and (a) will read zero. [2012]
(a) If both Assertion and Reason are correct and 55. Assertion : Long distance power transmission
Reason is the correct explanation of Assertion. is done at high voltage.
(b) If both Assertion and Reason are correct, but
Reason : At high voltage supply power losses
Reason is not the correct explanation of
Assertion. are less. [2014]
(c) If Assertion is correct but Reason is incorrect. 56. Assertion : When current through a bulb decreases
(d) If both the Assertion and Reason are incorrect. by 0.5%, the glow of bulb decreases by 1%.
51. Assertion : The electric bulbs glows immediately Reason : Glow (Power) which is directly
when switch is on. proportional to square of current. [2015]
Reason : The drift velocity of electrons in a r
metallic wire is very high. [2009] 57. Assertion : The current density J at any point
52. Assertion : In a simple battery circuit, the point in
r ohmic resistor is in direction of electric field
of the lowest potential is negative terminal of E at that point.
the battery. Reason : A point charge when released from
Reason : The current flows towards the point of rest in a region having only electrostatic field
the higher potential, as it does in such a circuit always moves along electric lines of force.
from the negative to the positive terminal.
[2016]
[2010]
53. Assertion : Free electrons always keep on 58. Assertion : Bending a wire does not effect
moving in a conductor even then no magnetic electrical resistance.
force act on them in magnetic field unless a Reason : Resistance of wire is proportional ot
current is passed through it. resistivity of material. [2016]
Reason : The average velocity of free electron is
59. Assertion : Kirchoff¢s junction rule follows from
zero. [2011]
conservation of charge.
54. Assertion : All electric devices shown in the
circuit are ideal. The reading of each of ammeter Reason : Kirchoff¢s loop rule follows from
(a) and voltmeter (V) is zero. conservation of momentum. [2017]
Current Electricity P-131

Type A : Multiple Choice Questions


2I2 Rt
Ratio = = 4 :1
1. (a) Let length & cross sectional area of wire be I 2 Rt
l & a respectively. r be the specific
resistance, then 2
7. (a) Current is uniform throughout the wire even
l if it is of non-uniform thickness. This is
R=r
a because charge passing through every
1 cross sectional area per unit time remains
If radius becomes half, area becomes th
4 the same throughout the wire. So, current
or cross sectional area after the stretch remains the same.
a 8. (c) Two batteries are joined with opposite
= . Let its length increases to l'. Since polarity so, total e.m.f. = 5 – 2 = 3V
4
volume remains the same in the process, Total resistance = 10 + 20 = 30W
a 3
la = l'´ Þ l ' = 4l Current = = 0.1A
4 30
Let R' be the resistance of streched wire,
9. (b) P P
4l l
R' = r = 16 ´ r = 16 R
a/4 a V
20
2. (a) = 1 Þ 20 = R + 6 Þ R = 14 W
R +6
3. (b) Three 5W resistors are in series. Their total
resistance = 15W. Now it is in parallel with
5W resistor, so total resistance, V
1 1 1 3+1 4 Let the resistances be r for each wire.
= + = =
R 5 15 15 15 Heat generated = I2(2r) = 2I2r,
15 where I is current through both of them.
R= When they are connected in parallel each
4
I
V 3 3´ 4 will have current = .
\ I= = = = 0.8A 2
R 15 / 4 15 2
4. (d) Electroplating is a process which uses the æIö I 2 rt
=
Heat generated ç ÷ rt ´ 2 =
process of electrolysis to form a thin layer è 2ø 2
of a metal on any other metal. It is not used Ratio of heat generated in two cases
for hardening purpose.
2I 2 rt 4 ´ I 2 rt
6. (d) Two 4W resistors are in parallel so, their = = = 4 :1
total resistance = 2W. Now three 2W I 2 rt I 2 rt
resistors are in series. Their total resistance 2
will be 6 W. 1
10. (d) Since, E = AT - BT 2
7. (b) Let main current be I & let the resistance of 2
each wire be R. 1
E = 16T - 0.08T 2 = 16T - 0.04T2
In the first case heat produced 2
= I2 Rt + I2Rt = 2I2Rt At temperature of inversion, E = 0
In the second case heat produced So, 16T – 0.04T2 = 0 Þ T = 0
2 or 16 - 0.04T = 0
æ Iö I 2 Rt
= ç ÷ Rt ´ 2 = 16
è 2ø 2 T= = 400°C
0.04
EBD_7100
P-132 Topicwise AIIMS Solved Papers – PHYSICS

2 2 So, slope of r - T curve is negative & it is


11. (a) Current = = = 0.5 dependent upon T i.e. it is not constant.
3.9 + 0.1 4
Voltage across the cell So, alternative (c) is right choice.
= 2 - 0.5 ´ 0.1 = 2 - 0.05 = 1.95 V 17. (c) Let i1, i2 & i3 be current in R1, R2 & R3.
12. (a) Internal resistance of a cell is the resistance Now, power consumed are same so,
of electrolyte used in the cell. That is why
i12 R1 = i 2 2 R 2 = i32 R 3 .....(i)
when it is heated its conductivity is
increased due to mobility of ions. i1R2 = i3R3 [volt over them are same]
13. (a) Resistance of bulb .....(ii)
Dividing (i) with (ii), i2 = i3 Þ R2 = R3
V 2 200 ´ 200
= = = 1000W i1 = 2i2 [i1 is equally being divided]
P 40
i12
V 2
100 ´ 100 i12 R1 = i 2 2 R 2 = R2
New power = = = 10W . 4
R 1000 R2 = 4R1
14. (d) Let the new length be l1, keeping volume
constant, 18. (a) Energy delivered to the ground = V.Q.
2 6 6
ærö = 4 ´ 10 ´ 4 = 16 ´ 10 joule
pr 2 L = p ç ÷ l1
è2ø 16 ´ 106 16 ´ 106
Þ l 1 = 4L Power = = = 16 ´ 107
time -3
100 ´ 10
rL rl1 = 160 MW
Now, 10 = ; R=
pr 2 2 19. (a) Faraday's law says that amount of
ærö
pç ÷ substance liberated or deposited at an
è 2ø electrode is proportional to quantity of
R r l1 ´ 4 pr 2 4L ´ 4 ´ r 2 16 charge passed through it. Thus, mass µ
= ´ = = charge. Since mass is quantised hence
10 pr 2 rL r2 ´ L 1
R = 160 indirectly charge is quantised.
15. (a) It is an example of mixed arrangement. 2e
20. (a) I =
Total potential = 0.15 × 5000 = 750 V. R + R1 + R 2
Resistance per row = 0.25 × 5000 = 1250 W
R1 R2
There are 100 rows, so I
1250
Total resistance = = 12.5 W
100
It is connected with external resistance of
500W, so
Total resistance = 500 + 12.5 = 512.5 W
750 R
Current = = 1.5 A Pot. difference across second cell
512 .5
16. (c) In a semiconductor when temperature = V = e - IR 2 = 0
increases conductivity increases so 2e
resistivity decreases i.e. e = .R2 = 0
R + R1 + R 2
1
Tµ R + R1 + R 2 - 2R 2 = 0
r
1 R + R1 - R 2 = 0 \ R = R 2 - R1
r T = constant Þ r = K
T 21. (b) At the steady state, the branch containing
1 dr K capacitor will be in effective as no current
dr = -K 2 dT Þ =- 2
T dT T will be flowing through it
Current Electricity P-133

2W \ Rf = R'' + R5 + R4 = R + R + 2R/3 = 8R/3,


Since R = 3W
3W
3
I I \ R f = 8 ´ = 8W .
3
6V 2.8 W 24. (a) Let resistance between A and B be r.
+ –
R´r
Since 2W and 3W resistors are in parallel, \ r = 2R +
equivalent resistance, r+R
2´3 R
R'= = 1.2W A
2+3 R r
Total current throgh the battery, R
B
6
I= = 1.5A Þ r = ( 3 + 1)R
1.2 + 2.8
\ Current flowing through 2W resistor 25. (a) e1 = 203.6 ....(i)
e1 – e2 = 24.6 ...(ii)
3
I(2W) = ´ 1.5 = 0.9A (i) - (ii) gives e2 = 179.0 cm
5
22. (d) At time t = 0 i.e., when the capacitor is rL
26. (d) R1= ,
charging, then current through the circuit, A
2 (1.2r)(1.2L) 1.44rL
I= = 2mA R2 = = ,
1000 A A
When capacitor is fully charged, no current (0.9r)(0.9L) 0.81rL
will pass through it. R3 = = ,
A A
Hence current through the circuit
r(1.5L) 1.5rL
2 R4 = =
I= = 1mA A A
2000 \ R3 < R1 < R2 < R4
23. (d) R1 = R2 = R3 = R4 = R5 = R
1
For constant potential, P µ ,
R
R5 R4 \ P3 < P1 < P2 < P4
R3 27. (d) G = 50 W, IG = 100µA, I = 10 A
æ I ö
Shunt, S = ç G ÷ G
R1 R2
è I - IG ø
æ 100 ´ 10 -6 ö
Þ S= ç ÷ ´ 50
è 10 - 100 ´ 10 -6 ø
R ' & R2 are in series
Their equivalent resistance, 10-4
R ' = R1 + R2 = 2R = ´ 50 = 5 × 10–4 W
10
Now R' & R3 in are parallel 28. (d) R1 = R0(1 + a1t) + R0(1 + a2t)
Their equivalent resitance R" is given by
æ a + a2 ö
1
=
1
+
1
=
1
+
1 = 2R 0 ç1 + 1 t÷
è 2 ø
R '' R ' R 3 2R R
1 1+ 2 æ a1 + a 2 ö
= = R '0 çè1 + t÷
ø
R '' 2R 2
Comparing with R = R0 (1+ at)
2R
R '' = a + a2
3 a= 1
R'', R5 & R4 are in series 2
EBD_7100
P-134 Topicwise AIIMS Solved Papers – PHYSICS

V2 V2 1 1 1 1 1 1 R+r
29. (b) P= Þ R= \ = + + = + =
R P R PQ 4R 4R 2r 2R 2r 2Rr
30. (d) We have, m = Zq
m 2Rr
Þ R PQ =
or Z= R+r
q
2R A 2R
For q = 1C, Z = m
So, for 1 coulomb of charge, q = i t 2R
r O r
or 1 = i × 0.25 or i = 4 A P Q
31. (c) Resistance of a wire = rl/A. 2R
For the same length and same material, 2R B 2R
R 2 A1 3 If a potential difference is applied across P
= = or, R 2 = 3R1
R1 A 2 1 and Q, there will be no currents in arms AO
The resistance of thick wire, R1 = 10 W and OB. So these resistance will be
The resistance of thin wire = 3R1 = 3 × 10 ineffective.
= 30 W. 2R 2R
Total resistance = 10 + 30 = 40W.
rl r
32. (b) R = 2 P r Q
pr
l r 2R 2R
When l is and radius is , 37. (a) The current in 2W resistor will be zero
2 2
rl4 2rl because it is not a part of any closed loop.
\ R' = =
p2r 2 pr 2
So, R' = 2R. So, heat is doubled according 5W 10 W
to H = I2RT. 10 V 20 V
33. (b) Let I1 be the current throug 5 W resistance, 2W
I2 through (6 + 9) W resistance. Then as per 38. (c) Neutral temperature,
question,
q + q0 530 + 10
I12 ´ 5 = 20 or, I1 = 2A. qn = i = = 270º C.
2 2
Potential difference across C and D = 2 × 5 39. (b) The equivalent circuit can be redrawn as
= 10V R R
10 2 R R
Current I 2 = = A. A· · B º ·
6+9 3 R R
R R 2R
Heat produced per second in 2W R R
2
æ8ö º º º
= I2 R ç ÷ ´ 2 = 14.2cal / s. 2R / 3
è 3ø R 5R / 3
34. (d) In the circuit I, the key is open so current in 5R / 8
the circuit is zero. Therefore the voltmeter 40. (b) Potential gradient along wire
gives 0V reading. In the circuit II, the reading
potential difference along wire
of the voltmeter is 2V. =
length of wire
l
35. (c) R µ ; I ´ 40
A or, 0.1 ´10 -3 = V / cm
So, the resistance of the wire will be 1000
minimum when the area of cross-section is 1
or, Current in wire, I= A
maximum and length is minimum. 400
36. (a) The circuit is symmetrical about axis POQ. 2 1
or, = or R = 800 - 40 = 760 W
Therefore the equivalent circuit is drawn 40 + R 400
Current Electricity P-135

41. (d) Let internal resistance of source = R 44. (a) From KVL
Current in coil of resistance – 6 + 3I1 + 1 (Ii – I2) = 0
V
R1 = I1 =
R + R1 6V P 2W
Current in coil of resistance
V
R 2 = I2 = 9V
R + R2
Further, as heat generated is same, so 1W
I12R1t = I22R2t
2 2 3W q 4W
æ V ö æ V ö
or çç ÷÷ R1 = çç ÷÷ R 2 6 = 3 I 1 + I1 – I2
è R + R1 ø è R + R2 ø 4I1 – I2 = 6 ...(1)
Þ R1(R + R2)2 = R2(R + R1)2
– 9 + 2I2 – (I1 – I2) + 3I2 = 0
Þ R2R1 + R1R22 + 2RR1R2
– I1 + 6I2 = 9 ...(2)
= R2R2 + R12R2 + 2RR1R2?
On solving (1) and (2)
Þ R2(R1 – R2) = R1R2(R1 – R2)
I1 = 0.13A
Þ R= R 1R 2 Direction Q to P, since I1 > I2.
æm I ö Alternatively
42. (a) F = ç 0 1 ÷ lI2 When I, = – 2I, and r =3r,,
è 2 pr ø p
3
m 0 - 2I1lI2 V
6V 9V 8
then F' = – 1W 1W
2p.3r
æ m 0 I1I2 l ö æ 2 ö 2 3W 5W 15 W I
=ç ÷ ç - ÷ =- F 8
è 2 pr ø è 3 ø 3
43. (d) d
R
E1 E 2 6 9
+ -
r1 r2 3 5= 3
R Eq = 1 1 =
1 1 8V
+ +
r1 r2 3 5
R
R R
3
R R R
R 8 = 3
R \ I = 15 23 = 0.13A
+1
8
R Considering potential at P as 0V and at Q as
A B x volt, then
R/3 2W
6V O
P
1W
R/3 R/3 9V

A R/3 B
3W 3W Q
3R R
´ 2 x-6 x-0 x+9
3 3 = R = 4W + + =0
Rnet between AB =
3R R 4R 3 1 5
+
3 3
EBD_7100
P-136 Topicwise AIIMS Solved Papers – PHYSICS

2 it draws a little lesser current. Consequently,


\ x= the current through the electric bulb
23 recovers.
x-0 2 51. (c) In a conductor there are large number of
\ i= = =0.13A
1 23 free electrons. When we close the circuit,
the electric field is established instantly
From Q to P
with the speed of electromagnetic wave
Type B : Assertion Reason Questions which cause electron drift at every portion
of the circuit. Due to which the current is
45. (d) Positive terminal of a battery is point of set up in the entire circuit instantly. The
highest potential and current flows from current which is set up does not wait for
highest to lowest potential i.e. from +ve to the electrons flow from one end of the
–ve potential. conductor to the another end. It is due to
46. (d) The e.m.f. of a dry cell is dependent upon this r eason, the electric bulb glows
the electrode potential of cathode and immediatly when switch is on.
anode which in turn is dependent upon 52. (c) Positive terminal of a battery is point of
the reaction involved as well as highest potential and current flows from
concentration of the electrolyte. It has highest to lowest potential i.e. from +ve to
nothing to do with size of the cell. –ve potential.
So, both assertion & reason are wrong. 53. (a) In the absence of the electric current, the
47. (b) Current continues to flow in a super free electrons in a conductor are in a state
conducting coil even after switch off of random motion, like molecule in a gas.
because at critical temperature, its Their average velocity is zero. i.e. they do
resistance is zero so there is no hinderance not have any net velocity in a direction. As
to current flow. a result, there is no net magnetic force on
the free electrons in the magnetic field. On
Meissner effect says th at at critical
passing the current, the free electrons ac-
temperature magnetic field inside the
quire drift velocity in a definite direction,
conductor is zero i.e, B = 0 but this does hence magnetic force acts on them, unless
not explain assertion. the field has no perpendicular component.
48. (b) Voltmeter is a galvanometer with high 54. (a)
resistance. It measures potential drop 2
across any part of an electrical circuit. It is æ Pö
55. (a) Power loss = i2R = ç ÷ R
connected in parallel so that it does not è Vø
draw any current itself (due to high [P = Transmitted power]
resistance) an d does not affect net 56. (a) Glow = Power (P) = I2R
resistance of the circuit. dP æ dI ö
49. (c) A conducting device obeys ohm's law when \ = 2 ç ÷ = 2 ´ 0.5 = 1%
P è I ør
the resistance of device is independent of r
the magnitude and polarity of the applied 57. (c) From relation J = s E , the current density
r
potential difference which happens in J at any point in ohmic r resistor is in
metallic conductors. Reason is false as direction of electric field E at that point. In
ohm's law is not true for non-ohmic space having non-uniform electric field,
conductors such as junction diodes etc. charges released from rest may not move
50. (b) The electric power of a heater is more than along ELOF. Hence Assertion is correct
1 while Reason is incorrect.
that of a bulb. As P µ , the resistance of
R I
heater is less than that of the electric bulb. 58. (a) Resistance wire R = r , where r is
A
When a heater connected in parallel to the resistivity of material which does not
bulb is switched on, it draws more current depend on the geometry of wire. Since when
due to its lesser resistance, consequently, wire is bent resistivity, length and area of
the current through the bulb decreases and cross-section do not change, therefore
so it becomes dim. resistance of wire also remain same.
When the heater coil becomes sufficient 59. (c) Kir ch off 's loop rule follows from
hot, its resistance becomes more and hence conservation of energy.
Moving Charges
18 and Magnetism
TYPE A : MULTIPLE CHOICE QUESTIONS 8. A rectangular loop carrying a current i1 , is
situated near a long straight wire carrying a
1. An electron moving with kinetic energy steady current i2. The wire is parallel to one of
6.6 ´10-14 J enters in a magnetic field of the sides of the loop and is in the plane of the
4 ´ 10 -3 T at right angles to it. The radius of its loop as shown in the figure. Then the current
circular path will be nearest to : [1997] loop will : [2003]
(a) 100 cm (b) 75 cm
(c) 25 cm (d) 50 cm i1
2. A charged particle enters a magnetic field H with
its initial velocity making an angle of 45° with H. i2
Then the path of the particle will be: [1999]
(a) circle (b) helical
(c) a straight line (d) a circle (a) move away from the wire
3. What should be amount of current through the (b) move towards the wire
ring of radius of 5 cm so that field at the centre (c) remain stationary
equal to the magnetic field of 7 × 10–5 Wb/m2, is (d) rotate about an axis parallel to the wire
(a) 0.28 amp (b) 5.57 amp [2000] 9. The cyclotron frequency of an electrons
(c) 2.8 amp (d) none of these gyrating in a magnetic field of 1 T is
4. A closely wound flat circular coil of 25 turns of approximately :
wire has diameter of 10 cm which carries current (a) 28 MHz (b) 280 MHz [2003]
of 4 amperes, the magnetic field at the centre of (c) 2.8 MHz (d) 28 GHz
a coil will be : [2001] 10. The magnetic moment of current (I) carrying
(a) 1.256 ´ 10 -3 tesla circular coil of radius (r) and number of turns (n)
varies as : [2004]
(b) 1.679 ´ 10 -5 tesla (a) 1/r2 (b) 1/r
(c) 1.512 ´ 10 -5 tesla (c) r (d) r2
(d) 2.28 ´ 10 - 4 tesla 11. A circular coil of radius R carries an electric
5. Cyclotron is used to accelerate : [2001] current. The magnetic field due to the coil at a
(a) positive ion (b) negative ion point on the axis of the coil located at a distance
(c) electron (d) none of these r from the centre of the coil, such that r > > R,
6. The magnetic field at a given point is varies as [2004]
0.5 × 10–5 Wb m–2. This field is to be annulled by (a) 1/r (b) 1/r3/2
magnetic induction at the centre of a circular (c) 1/r2 (d) 1/r3
conducting loop of radius 5.0 cm . The current 12. The magnetic field due to a straight conductor
required to be flown in the loop is nearly[2002] of uniform cross-section of radius a and carrying
(a) 0.2 A (b) 0.4 A a steady current is represented by : [2004]
(c) 4A (d) 40 A
7. An electron is travelling along the x-direction. It
encounters a magnetic field in the y-direction.
B
Its subsequent motion will be : [2002]
(a) straight line along the x-direction (a)
(b) a circle in the xz-plane
(c) a circle in the yz-plane O a r
(d) a circle in the xy-plane
EBD_7100
P-138 Topicwise AIIMS Solved Papers – PHYSICS

Ie R IcR
(a) (b)
Ic p Iep
B
pI c Iep
(b) (c) (d)
Ie R IcR
O a 16. What is the magnetic field at a distance R from a
r
coil of radius r carrying current I? [2007]
m0 IR 2 m 0 Ir 2
(a) (b)
B 3 3
2(R 2 + r 2 ) 2 2(R 2 + r 2 ) 2
(c) m0 I m0 I
(c) (d)
O a 2r 2R
r
17. A long straight wire of radius a carries a steady
current i. The current is uniformly distributed
across its cross section. The ratio of the
B magnetic field at a/2 and 2a is [2007]
(a) 1/2 (b) 1/4
(d) (c) 4 (d) 1
O 18. In a mass spectrometer used for measuring the
a r
masses of ions, the ions are initially accelerated
13. Two parallel beams of positrons moving in the by an electric potential V and then made to
same direction will : [2004] describe semicircular paths of radius R using a
(a) repel each other magnetic field B. If V and B are kept constant,
(b) will not interact with each other
(c) attract each other æ charge on the ion ö
the ratio çè ÷ will be
(d) be deflected normal to the plane containing mass of the ion ø
the two beams proportional to [2008]
14. A proton and an a-particle, moving with the same
velocity, enter a uniform magnetic field, acting 1 1
(a) (b)
normal to the plane of their motion. The ratio of R R2
the radii of the circular paths described by the (c) R 2 (d) R
proton and a-particle is : [2004]
19. Two concentric coils each of radius equal to
(a) 1 : 2 (b) 1 : 4
2 p cm are placed at right angles to each other. 3
(c) 4 : 1 (d) 1 : 16 ampere and 4 ampere are the currents flowing in
15. Circular loop of a wire and a long straight wire each coil respectively. The magnetic induction
carry currents Ic and Ie, respectively as shown
in figure. Assuming that these are placed in the in Weber / m 2 at the centre of the coils will be
same plane, the magnetic fields will be zero at
the centre of the loop when the separation H
(m 0 = 4p ´10 - 7 Wb / A.m ) [2008]
is : [2006] (a) 10 -5 (b) 12 ´ 10 -5
wire (c) 7 ´ 10 -5 (d) 5 ´ 10 -5
R Ic 20. The magnetic field due to a square loop of side
a carrying a current I at its centre is [2009]
µ0i µ0i
H (a) (b)
2a 2pa
Ie µ0 i µ0i
(c) (d) 2
Straight 2 pa pa
Moving Charges and Magnetism P-139

21. Electron of mass m and charge q is travelling m0 I 2m 0 I


with a speed along a circular path of radius r at (a) (b)
right angles to a uniform magnetic field of ap ap
intensity B. If the speed of the electron is m0I 2 2 m0 I
doubled and the magnetic field is halved the (c) 2 ap
(d)
ap
resulting path would have a radius [2009]
27. A charged particle moves through a magnetic
(a) 2 r (b) 4 r
field in a direction perpendicular to it. Then the
r r (a) velocity remains unchanged [2013]
(c) (d)
4 2 (b) speed of the particle remains unchanged
22. Electron moves at right angles to a magnetic (c) direction of the particle remains unchanged
field of 1.5 × 10–2 tesla with speed of 6 × 107 m/s. (d) acceleration remains unchanged
If the specific charge of the electron is 1.7 × 1011 28. Wires 1 and 2 carrying currents i 1 and i 2
C/kg. The radius of circular path will be [2010] respectively are inclined at an angle q to each
(a) 3.31 cm (b) 4.31cm other. What is the force on a small element dl of
(c) 1.31 cm (d) 2.35 cm wire 2 at a distance of r from wire 1 (as shown in
23. An electron beam passes through a magnetic figure) due to the magnetic field of wire 1?
field of 2 × 10–3 Wb/m2 and an electric field of 1 2 [2013]
1.0 × 104 V/m both acting simultaneously. The
path of electron remains undeviated. The speed i1 r i2
of electron if the electric field is removed, and
the radius of electron path will be respectively q dl
(a) 10 × 106 m/s, 2.43 cm [2011]
(b) 2.5 × 106 m/s, 0.43 cm m0 m0
(c) 5 × 106 m/s, 1.43 cm (a) i1i 2 dl tan q (b) i1i 2 dl sin q
2 pr 2pr
(d) none of these
m0 m0
24. A charged particle is released from rest in a region (c) i1i 2 dl cos q (d) i1i 2 dl sin q
of uniform electric and magnetic fields which are 2 pr 4pr
parallel to each other. The particle will move on a: 29. If we double the radius of a coil keeping the
(a) straight line (b) circle [2011] current through it unchanged, then the magnetic
(c) helix (d) cycloid field at any point at a large distance from the
25. Four wires, each of length 2.0 m, are bent into centre becomes approximately [2014]
four loops P, Q, R and S and then suspended in (a) double (b) three times
a uniform magnetic field. If the same current is (c) four times (d) one-fourth
passed in each, then the torque will be maximum 30. A portion of a conductive wire is bent in the
on the loop [2012] form of a semicircle of radius r as shown below
in fig. At the centre of semicircle, the magnetic
induction will be [2015]
Q S
P R i
r
i
(a) P (b) Q O
(c) R (d) S (a) zero (b) infinite
26. A square coil of side a carries a current I. The μ0 π i μ0 πi
magnetic field at the centre of the coil is[2012] (c) . gauss (d) . tesla
4π r 4π r
31. A coil of circular cross-section having 1000 turns
and 4 cm2 face area is placed with its axis parallel
a to a magnetic field which decreases by 10–2 Wb
O
m–2 in 0.01 s. The e.m.f. induced in the coil is:
(a) 400 mV (b) 200 mV [2017]
(c) 4 mV (d) 0.4 mV
EBD_7100
P-140 Topicwise AIIMS Solved Papers – PHYSICS
TYPE B : ASSERTION REASON QUESTIONS 36. Assertion : A charge, whether stationary or in
motion produces a magnetic field around it.
Directions for (Qs. 32-35) : These questions consist Reason : Moving charges produce only electric
of two statements, each printed as Assertion and field in the surrounding space. [2009]
Reason. While answering these questions, you are 37. Assertion : A proton and an alpha particle having
required to choose any one of the following five the same kinetic energy are moving in circular
responses. paths in a uniform magnetic field. The radii of
(a) If both Assertion and Reason are correct and their circular paths will be equal.
the Reason is a correct explanation of the Reason : Any two charged particles having
Assertion. equal kinetic energies and entering a region of
(b) If both Assertion and Reason are correct but ur
Reason is not a correct explanation of the uniform magnetic field B in a direction
ur
Assertion. perpendicular to B , will describe circular
(c) If the Assertion is correct but Reason is trajectories of equal radii. [2009]
incorrect. 38. Assertion : If the current in a solenoid is
(d) If both the Assertion and Reason are incorrect. reversed in direction while keeping the same
(e) If the Assertion is incorrect but the Reason is magnitude, the magnetic field energy stored in
correct. the solenoid remains unchanged.
32. Assertion : Cyclotron is a device which is used Reason : Magnetic field energy density is
to accelerate the positive ion. proportional to the magnetic field.[2010, 2017]
Reason : Cyclotron frequency depends upon 39. Assertion : The magnetic field at the centre of
the velocity. [1997] the circular coil in the following figure due to
33. Assertion : Cyclotron does not accelerate the currents I1 and I2 is zero.
electron.
Reason : Mass of the electrons is very small. I1
I I
[2000]
34. Assertion : In electric circuits, wires carrying q
currents in opposite directions are often twisted
together
Reason : If the wires are not twisted together,
the combination of the wires forms a current
loop, the magnetic field generated by the loop I2
might affect adjacent circuits or components. Reason : I1 = I2 implies that the fields due to the
[2008] current I1 and I2 will be balanced. [2013]
35. Assertion : The magnetic field produced by a 40. Assertion : If the current in a solenoid is reversed
current carrying solenoid is independent of its in direction while keeping the same magnitude,
length and cross-sectional area. the magnetic field energy stored in the solenoid
Reason : The magnetic field inside the solenoid decreases.
is uniform. [2008] Reason : Magnetic field energy density is
Directions for (Qs. 36-42) : Each of these questions proportional to square of current. [2015]
contains an Assertion followed by Reason. Read them 41. Assertion : Free electrons always keep on
carefully and answer the question on the basis of moving in a conductor even then no magnetic
following options. You have to select the one that force act on them in magnetic field unless a
best describes the two statements. current is passed through it.
(a) If both Assertion and Reason are correct and Reason : The average velocity of free electron
Reason is the correct explanation of Assertion. is zero. [2016]
(b) If both Assertion and Reason are correct, but 42. Assertion: To convert a galvanometer into an
Reason is not the correct explanation of ammeter a small resistance is connected in
Assertion. parallel with it.
(c) If Assertion is correct but Reason is incorrect. Reason: The small resistance increases the
(d) If both the Assertion and Reason are incorrect. combined resistance of the combination. [2016]
Moving Charges and Magnetism P-141

Type A : Multiple Choice Questions move forward with velocity u cos 45°. Under
both these motions, it will have helical path
1. (d) When a charged particle enters a magnetic
as shown in the figure.
field, its path becomes circular whose
radius can be found out from the relation. 3. (b)
i
mv 2 mv
= Bqv Þ r =
r Bq

1
E= mv2 = 6.6 ´10 -14
2 Magnetic field at the centre of a coil
carrying current.
1 m 2v 2
= 6.6 ´10 -14 µ 0i
2 m B=
2r
Þ m 2 v 2 = 2m ´ 6.6 ´ 10 -14
4p ´ 10 -7 i
mv = 2m ´ 6.6 ´ 10 -14 7 ´ 10 -5 =
2 ´ 5 ´ 10 - 2
= 2 ´ 9.1´ 10 -31 ´ 6.6 ´ 10 -14 70 ´10 -7
i= = 5.57 amp
-31 -14 4p ´ 10 -7
2 ´ 9.1 ´10 ´ 6.6 ´10
r= m0i
4 ´ 10 ´ 1.6 ´10 -19
-3
4. (a) Flux density, B = ´n
2r
= 50 cm. approx.
2. (b) 4p ´ 10 -7 ´ 4 ´ 25 4p ´ 10 -7 ´ (100) 2
= =
u sin 45° × × × × × × 2 ´ 5 / 100 10
u× × × × × ×
= 1.256 ´ 10 -3 tesla.
× × × × × H
A 45° 5. (a) Cyclotron is used to accelerate positive
u cos 45° ions. Electron can not be used as its velocity
increases appreciably resulting into its
mass becoming very large. It creates
problem in synchronisation.
6. (b) Fields created at the centre
= 0.5 × 10–5 Wb/m2
Helical Path Due to current in circular loop, field created
At point A, charge is entering in a magnetic m0i
field in which direction of field is shown in in the centre =
2r
the figure. The velocity of particle is u
making an angle of 45° with field. We 4p ´ 10-7 ´ i
resolve it in two directions, one along the = -2
= 0.5 ´ 10-5
2 ´ 5 ´10
field and other perpendicular to it. Since u
sin45° is perpendicular to H, it will create a 0.5 ´ 10 -5 ´ 10 ´ 10 -2 5 35
rotatory effect on the charge. So charge i= = =
-7 4p 88
particle will start rotating with axis along 4p ´ 10
the direction of H. At the same time it will = 0.3980 = 0.4 A
EBD_7100
P-142 Topicwise AIIMS Solved Papers – PHYSICS
7. (b) When a charged particle enters a magnetic 11. (d) Magnetic field on the axis of the coil is
field, it experiences a force which is always
directed perpendicular to its direction of 1 pa 2 nix
B=
motion. In that case the path of the charged 4p Î0 (x 2 + a 2 )3/ 2
particle becomes circular and the plane of If x >> a
the circle is perpendicular to the plane
containing the magnetic field and velocity 1 pa 2 ni 1
B= or B µ
vector of the charged particle. 4p Î0 x 3
x3
8. (b) 12. (a) Inside the wire magnetic field will be directly
B C proportional to the distance from the axis.
Outside the wire, field is inversely
proportional to distance.

A D

B 1
Current in A B is same as that current in Bµ r Bµ
r
straight wire so it will be attracted. CD
portion will be repelled. Net force in the
loop will be attractive. Force on BC and O
AD will be opposite to each other so energy r = R0
will cancel out each other.
9. (d) For an electron gyrating in a magnetic field
13. (c) Two parallel beams of positron moving in
2 same direction is equivalent to two current
mv
= Bqv carrying conductor, carrying current in
r
same direction. They will attract each other.
mv v Bq 14. (a) When a charged particle enters a field, it
r= Þ =
Bq r m takes a circular path. The radius of circular
path is given by the relation
2 pr 1 v Bq
T= ; n= = = mv
v T 2pr 2 pm r=
Bq
1 ´ 1.6 ´ 10-19
n= mv
2 ´ p ´ 9.1 ´ 10-31 For proton, r1 =
Bq
1.6 ´ 1012
= = 28 GHz 4m ´ v 2mv
2p ´ 9.1 For a -particle, r2 = =
B ´ 2q Bq
10. (d) Magnetic moment of a coil of radius r
carrying current i is equal to Þ r1 : r2 : :1: 2
M = iAn [A is area of the coil, x is no. of
m 0 Ie
turns] 15. (a) Magnetic field due to straight wire =
2 pH
= nipr 2
m 0 Ic
So, M µ r 2 Magnetic field due to circular wire =
2R
Moving Charges and Magnetism P-143

m0 Ie m 0 I c
18. (b) The centripetal force is provided by the
Now = magnetic force.
2 pH 2R
mv2
R Ie i.e., = qvB … (1)
Þ H= R
p Ic
where m = mass of the ion, v = velocity, q =
charge of ion, B = flux density of the
m 0 Ir 2
16. (b) The magnetic field is B = magnetic field.
2(R 2 ∗ r 2 )3 / 2 we have, v = Rw
(see application of Biot-Savart law to v qB
magnetic field at a point along axis of coil) or w = = (From (1))
R m
17. (d) Here, current is uniformly distributed
across the cross-section of the wire, Energy of ion is given by,
therefore, current enclosed in the amperean 1 2 2
1 1 2 q B
E= mv 2 = m(Rw ) 2 = mR
æ aö 2 2 2 m2
path formed at a distance r1 ç = ÷
è 2ø
1 R 2 B2 q 2
or E = … (2)
2 m
If ions are accelerated by electric potential
a/2 V, the energy attained by ions,
E = qV … (3)
P1 P2
From eqns (2) and (3)
1 R 2 B2q 2 æ qö 2V
qV = or çè ÷ø = 2 2
a 2 m m R B

æ p r2 ö æ qö 1
= ç 12 ÷ ´ I , where I is total current i.e., çè ÷ø µ 2 ( If V and B are const.)
m R
è pa ø
19. (d) (1)
\ Magnetic field at
m0 ´ current enclosed
P1 (B1 ) =
Path
(2)
æ p r2 ö
m 0 ´ ç 12 ÷ ´ I
è pa ø m 0 ´ I r1
= =
2p r1 2p a 2 m 0 i1 m ´3
Now, magnetic field at point P2, B1 = = 0
2 ( 2 p) 4p
m0 I m I
(B2) = . = 0 . m 0i 2 m ´4
2p (2a) 4pa B2 = = 0
2 ( 2 p) 4p
B1 m 0 Ir1 4pa
\ Required Ratio = = ´ m0
B 2 2 pa 2 m 0 I B= B12 + B 22 = .5
4p
a
2 r1 2 ´ 2 Þ B = 10 - 7 ´ 5 ´ 10 2
= = = 1.
a a Þ B = 5 ´ 10 - 5 Wb / m 2
EBD_7100
P-144 Topicwise AIIMS Solved Papers – PHYSICS
25. (d) For a given perimeter the area of circle is
éµ i ù
20. (d) B = ê 0 (cos 45° - cos135°) ú ´ 4 maximum. So magnetic moment of (S) is
ë 4 pa û greatest.
26. (d) Btotal = 4Bside
µ0i 2
= 45° 45°
pa 2 m0 I é p pù
Btotal = 4 ê sin + sin ú
æa ö 4 4û
µ0 i 2p ç ÷ ë
= 2 è2ø
pa
mv 2 2m 0 I
21. (b) Radius of path is given by r = Bq Btotal =
ap
Here, m and q remain unchanged 27. (b) Magnetic force acts perpendicular to the
r v B v B/2 1 velocity. Hence speed remains constant.
So, 1 = 1 × 2 = × =
r2 v 2 B1 2v B 4 28. (c)
Þ r2 = 4r æ m 0 NI ö 2
22. (d) B = 1.5 × 10–2 T, 29. (c) Baxis = çç ÷÷R
è 2x 3 ø
q = 90°, sin q = 1, v = 6 × 107 m/s,
e B µ R2
= 1.7 × 1011 C/kg So, when radius is doubled, magnetic field
m
becomes four times.
mv 6 ´ 107 30. (d) The straight part will not contribute magnetic
r= = field at the centre of the semicircle because
Be 1.5 ´ 10-2 ´ 1.7 ´ 1011
every element of the straight part will be 0º
= 2.35 × 10–2m = 2.35 cm
or 180º with the line joining the centre and
23. (c) B = 2 × 10–3Wb/m2,
the element
E = 1× 104 V/m2
Due to circular portion, the field is
Since the path of electron remains
undeviated, qvB = qE or 1 m0i m 0 i
=
E 1 ´ 10+4 2 2r 4r
v= = = 0.5 × 107
B 2 ´ 10-3 m0 i
= 5 × 106 m/s Hence total field at O = tesla
4r
If the electricfield is removed, the path of
the charged particle is circular and 31. (a) Given: No. of turns N = 1000
magnetic field provides the necessary Face area, A = 4 cm2 = 4 × 10–4 m2
centripetal force. i.e., Change in magnetic field,
DB = 10–2 wbm–2
mv 2 mv
= Bev Þ r= Time taken, t = 0.01s = 10–2 sec
r Be Emf induced in the coil e = ?
9.1 ´ 10 -31 ´ 5 ´ 10 6 Applying formula,
=
2 ´ 10-3 ´ 1.6 ´ 10 -19 -d f
Induced emf, e =
= 14.3 × 10–3m = 1.43 cm dt
DB ö
24. (a) E = N æç ÷ A cos q
B è Dt ø
The magnetic force on charged particle is 1000 ´ 10-2 ´ 4 ´ 10 -4
zero. = = 400 mV
10-2
Moving Charges and Magnetism P-145

Type B : Assertion Reason Questions field, it also produces a magnetic field,


because moving charges produces
32. (c) Cyclotron is used to accelerate charged
magnetic field in the surrounding space.
particles. So Assertion is correct.
37. (c) The radius of the circular path is given by
We know that time period of a particle
2pr mv 2 mv 2 mK 1
Bqr r= = ; where K = mv 2
T= ; 0
= Bqv 0 Þ v 0 = qB qB 2
v0 r m
Since K and B are the same for the two
2pr 2p
T= ´m = m m
Bqr Bq particles, r µ . Now, the charge of an
q
1 Bq alpha particle is twice that of a proton and
Frequency = =
T 2 pm its mass is four times the mass of a proton,
This does not dpeend upon velocity so m q will be the same for both particles.
Reason is incorrect. Hence, r will be the same for both particles.
33. (c) Cyclotron does not accelerate electron 38. (c)
because mass of electron is very small. It
gets accelerated very appreciably as a I1 2p - q
39. (d) = Þ I1q = I2 (2p - q) ........ (1)
result of which its mass increases. It result I2 q
is mismatch between frequency of a.c. used
and frequency of rotation of electron in the q m 0 I1 2p - q m 0 I 2
Dee’s. So cyclotron stops accelerating B1 = . and B2 = .
2p 2R 2p 2R
electrons after some time.
Using (1), we get B1 = B2.
34. (a) If the wires are twisted together, they can
be formed as a single wire carrying currents
40. (d) Reversing the direction of the current
in opposite directions. In this pattern, in
reverses the direction of the magnetic field.
wires no magnetic field is induced which
However, it has no effect on the magnetic-
does not affect adjacent circuits.
field energy density, which is proportional
35. (b) Magnetic field inside the solenoid at point
to the square of the magnitude of the
P is given by,
magnetic field.
m0 41. (b) In the absence of the electric current, the
B= (2pni)[sin a + sin b]
4p free electrons in a conductor are in a state
where n = no. of turns per unit length of random motion, like molecule in a gas.
Their average velocity is zero. i.e. they do
=N/l
not have any net velocity in a direction. As
Thus it is clear that mangetic field is a result, there is no net magnetic force on
independent of length and cross sectional the free electrons in the magnetic field. On
area. passing the current, the free electrons
Also the magnetic field with in the solenoid acquire drift velocity in a definite direction,
is uniform and parallel to the axis of the hence magnetic force acts on them, unless
solenoid. the field has no perpendicular component.
36. (d) A charge, whether stationary or in motion, 42. (c) An ammeter should have a low resistance
produces an electric field around it. If it is which we get when we connect low
in motion, then in addition to the electric resistance in parallel with galvanometer.
EBD_7100
P-146 Topicwise AIIMS Solved Papers – PHYSICS

19 Magnetism and Matter

TYPE A : MULTIPLE CHOICE QUESTIONS (c) The magnetic lines form a closed loop
(d) Magnetic lines of force do not cut each
1. Domain formation is the necessary feature of : other
[1998] 8. Which one of the following are used to express
(a) ferromagnetism (b) diamagnetism intensity of magnetic field in vacuum ? [2000]
(c) paramagnetism (d) all of these (a) oersted (b) tesla
2. The best material for the core of a transformer is (c) gauss (d) none of these
(a) mild steel (b) stainless steel 9. A frog can be levitated in a magnetic field
(c) soft iron (d) hard steel [1998] produced by a current in a vertical solenoid
3. The north pole of a magnet is brought near a placed below the frog. This is possible because
metallic ring. Then the direction of the induced the body of the frog behaves as : [2003]
current in the ring will be: [1999] (a) paramagnetic (b) diamagnetic
(a) Towards north (b) Towards south (c) ferromagnetic (d) antiferromagnetic
(c) Anticlockwise (d) Clockwise 10. Liquid oxygen remains suspended between two
pole forces of a magnet because it is : [2004]
4. Angle of dip is 90° at: [1999]
(a) diamagnetic (b) paramagnetic
(a) Equator (b) Middle point
(c) ferromagnetic (d) antiferromagnetic
(c) Poles (d) None of these
11. The magnetic susceptibility of an ideal
5. What happens, when a magnetic substance is diamagnetic substance is [2007]
heated ? [1999]
(a) –1 (b) 0
(a) It loses its magnetism
(c) +1 (d) ¥
(b) It becomes a strong magnet
12. A magnet makes 40 oscillation per minute at a
(c) Does not effect the magnetism place having magnetic intensity of 0.1 × 10–5
(d) Either (b) and (c) tesla. At another place it takes 2.5 sec to
6. A magnet 10 cm long and having a pole strength complete one oscillation. The value of earth's
2 amp m is deflected through 30° from the horizontal field at that place is [2011]
magnetic meridian. The horizontal component (a) 0.76 × 10–6 tesla (b) 0.18 × 10–6 tesla
of earth’s induction is 0.32 ´10-4 tesla then the (c) 0.09 × 10–6 tesla (d) 0.36 × 10–6 tesla
value of deflecting couple is: [1999] 13. Curie temperature is the temperature above
which [2012]
(a) 32 ´ 10 -7 Nm (b) 16 ´ 10 -7 Nm
(a) a ferromagnetic material becomes
(c) 64 ´ 10 -7 Nm (d) 48 ´ 10 -7 Nm paramagnetic
7. Which one of the following statement is not (b) a paramagnetic material becomes
correct about the magnetic field ? [2000] diamagnetic
(c) a ferromagnetic material becomes
(a) Inside the magnet the lines go from north
diamagnetic
pole to south pole of the magnet
(d) a paramagnetic material becomes
(b) Tangents to the magnetic lines give the
ferromagnetic
direction of the magnetic field
Magnetism and Matter P-147

14. Of the following Fig., the lines of magnetic 19. Imagine rolling a sheet of paper into a cylinder
induction due to a magnet SN, are given by and placing a bar magnet near its end as shown
N N in figure. What can you say about the sign of
r r r
B.dA for every area dA on the surface? [2017]

(a) Positive
(b) Negative
S S (c) No sign
(1) (2) (d) Can be positive or negative
N N
TYPE B : ASSERTION REASON QUESTIONS
Directions for (Qs. 20-27) : These questions consist
of two statements, each printed as Assertion and
Reason. While answering these questions, you are
required to choose any one of the following five
S S responses.
(3) (4) (a) If both Assertion and Reason are correct and
(a) 1 (b) 2 [2012] the Reason is a correct explanation of the
(c) 3 (d) 4 Assertion.
15. The materials suitable for making electromagnets (b) If both Assertion and Reason are correct but
should have [2013] Reason is not a correct explanation of the
(a) high retentivity and low coercivity Assertion.
(b) low retentivity and low coercivity (c) If the Assertion is correct but Reason is
(c) high retentivity and high coercivity incorrect.
(d) low retentivity and high coercivity (d) If both the Assertion and Reason are incorrect.
16. Magnetic lines of force due to a bar magnet do (e) If the Assertion is incorrect but the Reason is
not intersect because [2014] correct.
(a) a point always has a single net magnetic 20. Assertion : We cannot think of a magnetic field
field configuration with three poles
(b) the lines have similar charges and so repel Reason : A bar magnet does exert a torque on
each other itself due to its own field. [2002]
(c) the lines always diverge from a single 21. Assertion : In high latitudes one sees colourful
force curtains of light hanging down from high
(d) the lines need magnetic lenses to be made altitudes
to interest Reason : The high energy charged particles from
17. At a temperatur of 30°C, the susceptibility of a the sun are deflected to polar regions by the
ferromagnetic material is found to be c . Its magnetic field of the earth. [2003]
susceptibility at 333°C is [2015] 22. Assertion : The true geographic north direction
(a) c (b) 0.5 c is found by using a compass needle.
(c) 2 c (d) 11.1 c Reason : The magnetic meridian of the earth is
18. Two points A and B are situated at a distance x along the axis of rotation of the earth. [2004]
and 2x respectively from the nearer pole of a 23. Assertion : A disc-shaped magnet is deviated
magnet 2 cm long. The ratio of magnetic field at above a superconducting material that has been
A and B is [2016] cooled by liquid nitrogen.
(a) 4 : 1 exactly (b) 4 : 1 approximately Reason : Superconductors repel a magnet.
(c) 8 : 1 approximately (d) 1 : 1 approximately [2005]
EBD_7100
P-148 Topicwise AIIMS Solved Papers – PHYSICS
24. Assertion : Magnetic Resonance Imaging (MRI) 28. Assertion : The ferromagnetic substance do
is a useful diagnostic tool for producing not obey Curie’s law.
images of various parts of human body. Reason : At Curie point a ferromagnetic
Reason : Protons of various tissues of the substance start behaving as a paramagnetic
human body play a role in MRI. [2006] substance. [2011]
25. Assertion : Diamagnetic materials can exhibit 29. Assertion : The ferromagnetic substance do not
magnetism. obey Curie’s law.
Reason : Diamagnetic materials have permanent Reason : At Curie point a ferromagnetic
magnetic dipole moment. [2006] substance start behaving as a paramagnetic
26. Assertion : Ferro-magnetic substances become substance. [2014]
paramegnetic above Curie temp.
30. Assertion : A paramagnetic sample display
Reason : Domains are destroyed at high greater magnetisation (for the same magnetic
temperature. [2007] field) when cooled.
27. Assertion : If a compass needle be kept at
Reason : The magnetisation does not depend
magnetic north pole of the earth the compass
on temperature. [2015]
needle may stay in any direction.
31. Assertion : Electromagnetic are made of soft
Reason : Dip needle will stay vertical at the north
iron.
pole of earth [2008]
Directions for (Qs. 28-33) : Each of these questions Reason : Coercivity of soft iron is small.[2016]
contains an Assertion followed by Reason. Read them 32. Assertion : The sensitivity of a moving coil
carefully and answer the question on the basis of galvanometer is increased by placing a suitable
following options. You have to select the one that magnetic material as a core inside the coil.
best describes the two statements. Reason : Soft iron has high magnetic
(a) If both Assertion and Reason are correct and permeability and cannot be easily magnetized
Reason is the correct explanation of Assertion. or demagnetized. [2016]
(b) If both Assertion and Reason are correct, but 33. Assertion : The poles of magnet can not be
Reason is not the correct explanation of separated by breaking into two pieces.
Assertion. Reason : The magnetic moment will be reduced
(c) If Assertion is correct but Reason is incorrect. to half when a magnet is broken into two equal
(d) If both the Assertion and Reason are incorrect. pieces. [2017]
Magnetism and Matter P-149

Type A : Multiple Choice Questions 8. (a) Intensity of magnetic field in vacuum is


expressed in oersted in c.g.s. system and
1. (a) In ferromagnetic material atomic magnets
of a substance form domain in which Am -1 in S.I. system. In the formula,
magnetic moment of all the tiny magnets B = µH
are oriented in the same direction. H is known as intensity of magnetic field.
9. (c) The frog will levitate in the magnetic field
due to repulsion. It suggests that the body
Different of frog is acting as ferromagnetic material.
Domains 10. (b) Oxygen is paramagnetic in nature. So if it
will be attracted both by North pole or
South pole. When it is placed exactly
2. (c) Soft iorn provides the best material for the between two magnetic poles, the forces
core of a transformer as its permeability (µ) acting on it due to magnetic poles will be
is very high. Its hysteresis curve is of small equal & opposite. Hence it will remain
area and its coercivity is very low. suspended between them.
3. (c) 11. (a) Magnetic susceptibility c is related to
permeability µ by µ = 1+ 4pcm . For
diamagnetic substances cm is negative and
µ<1 (negative). Ideal diamagnet should
expel all magnetic field lines inside it i.e. cm
< 0 or cm = – 1
When we bring a north pole near a metallic 12. (d) Time period of vibration of a magnet is
ring applying Lenz’s law the ring will behave
I
as north pole so that repulsion occurs. So T = 2p
direction of current induced in it will be MH
anti-clockwise as depicted above. For the same magnet, I and M are constant
4. (c) At poles angle of dip will be 90° because where M = magnetic moment,
earth’s magnetic field will be almost vertical
there. I = moment of inertia of magnet.
5. (a) When a magnetic substance is heated it
loses its magnetic property. It is because 1
Þ Tµ .
all the atomic magnet becomes randomly H
oriented due to heat. First case
10 60 3
6. (a) M = 2´ = 0.2
100 T1 = = = 1.5 sec.
40 2
Value of restoring couple = MH sin f H1 = 0.1 × 10–5T, T2 = 2.5 sec, H2 = ?

= 0.2 ´ 0.32 ´10-4 sin 30° 1.5 H2


Þ =
-4 1 2.5 H1
= 0.2 ´ 0.32 ´ 10 ´ = 32 ´ 10 -7 Nm
2
7. (a) Inside the magnet, lines go from south pole 15 H2
Þ =
to north pole. So, option (a) is correct. 25 10 -6
EBD_7100
P-150 Topicwise AIIMS Solved Papers – PHYSICS
2 22. (d) The true geographic north-south direction
æ 3ö -6
Þ H2 = ç ÷ ´ 10 is inclined at an angle with magnetic north-
è 5ø south direction. The angle between them
9
Þ H2 = ´ 10-6 = 0.36 × 10–6T is known as declination. Compass needle
25 gives us direction of north -south
13. (a) (magnetic) direction. Magnetic meridian to
14. (a) As lines of magnetic induction B are pass through magnetic north-south
continuous curves, they run continuously direction. Axis of rotation of the earth gives
through the bar and outside, as shown in us geographic north-south direction. So,
Fig. (1). both Assertion and Reason are incorrect.
15. (b) Electro magnet should be amenable to 23. (a) Superconductors are actually diamagnetic
magnetisation and demagnetization in nature so when it is placed above a
\ retentivity should be low and coercivity magnet it will be repelled by magnet or it
should be low. will tend to move from higher field to lower
16. (a) field. This is the principle behind leviation
of a superconducting material above
m0C magnet.
17. (b) According to Curie's law, cm =
T 24. (a) MRI is useful diagnostic tool for producing
where C is Curie constant, T = temperature images of various parts of human body
1 because it makes use of magnetic property
\ cma of spinning proton inside the nucleus.
T
25. (c) Diamagnetic material exhibits magnetism in
c m1 T 273 + 333 606 reverse direction. R is a wrong statement.
= 2 = = =2
c m2 T1 273 + 30 303 Because due to absence of unpaired
electron in diamagnetic material it does not
\ c m2 = c m1 / 2 = 0.5c m = 0.5 c exhibit permanent magnet dipole moment.
1
26. (a) Susceptibility of ferro magnets decreases
(Q cm 1
=c ) with increase of temperature. At a transition
18. (c) Taking distances from the centre of the temperature called Curie temperature they
magnet, pass over to paramagnets. When
temperature is increased, dipoles acquire
3 3
B1 æ x 2 ö æ 2 x + 1 ö kinetic energy and are disoriented, hence
=ç ÷ =ç ÷ = 8 : 1, approximately.
B2 çè x1 ÷ø è x + 1 ø domain internal interaction called exchange
coupling disappears.
19. (b) The field is entering into the surface so 27. (b) The earth has only vertical component of
flux is negative. its magnetic field at the magnetic poles.
Type B : Assertion Reason Questions Since compass needle is only free to rotate
in horizontal plane where H = 0, hence the
20. (d) Magnetic field may be formed with the help compass needle may stay in any direction.
of three poles. A bar magnet does not exert The dip needle rotates in a vertical plane
a torque on itself due to its own field. and angle of dip at poles is 90°. So it will
21. (a) In polar region like north pole and south stand vertical.
pole one sees colourful curtains of light 28. (b) The susceptibility of ferromagnetic
hanging down from light altitude. It results substance decreases with the rise of
from high energy charged particles from temperature in a complicated manner. After
the sun being attracted by the poles of the Curie point in the susceptibility of
earth. In northern hemisphere it is known ferromagnetic substance varies inversely
Aure Boreali’s and in southern hemisphere with its absolute tempearture.
it is known as Aura Australi’s. Ferromagnetic substance obey’s Curie’s
law only above its Curie point.
Magnetism and Matter P-151

29. (c) The susceptibility of ferromagnetic As the material in electromagnets is


substance decreases with the rise of subjected to cyclic changes (magnification
temperature in a complicated manner. After and demangetisation), the hysteresis loss
Curies point in the susceptibility of of the material must be small. The material
ferromagnetic substance varies inversely should attain high value of I and B with
with its absolute tempearture. low value of magnetising field intensity H.
Ferromagnetic substance obey’s Curie’s As soft iron has small coercivity, so it is a
law only above its Curie point. best choice for this purpose.
32. (c) Sensitivity of galvanometer,
30. (d) A paramagnetic sample display greater
magnetisation when cooled, this is because q tan q = m0 N .
s= ;
at lower temperature, the tendency to i i 2 RBH
disrupt the alignment of dipoles (due to If a magnetic material is placed inside coil
magnetising field) decreases on account of galvanometer, then
of reduced random thermal motion. mr m0 N
s¢ = .
31. (b) Electromagnets are magnets, which can be 2 RBH
turnd on and off by switching the current 33. (b) When a magnet is cut into pieces, each
piece becomes new magnet. M ¢ = ml = M .
on and off.
2 2
EBD_7100
P-152 Topicwise AIIMS Solved Papers – PHYSICS

20 Electromagnetic Induction

TYPE A : MULTIPLE CHOICE QUESTIONS 7. A conducting ring of radius 1 metre is placed in


an uniform magnetic field B of 0.01 tesla
1. A coil of copper having 1000 turns is placed in a oscillating with frequency 100 Hz with its plane
magnetic field (B = 4 × 10–5) perpendicular to its at right angle to B. What will be the induced
axis. The cross sectional area of the coil is 0.05 electric field ? [2005]
m2. If it turns through 180° in 0.01 second, then
(a) p volt / m (b) 2 volt / m
the e.m.f. induced in the coil will be ; [1997]
(a) 4 V (b) 0.04 V (c) 10 volt/m (d) 62 volt/m
(c) 0.4 V (d) 0.2 V 8. A magnet is made to oscillate with a particular
2. In a coil of self inductance of 5 henry, the rate of frequency, passing through a coil as shown in
change of current is 2 ampere per second, the figure. The time variation of the magnitude
e.m.f. induced in the coil is : [1997] of emf generated across the coil during on cycle
(a) 5 V (b) – 5V is [2005]
(c) – 10 V (d) 10 V S
3. According to Lenz’s law there is a conversion of: N
(a) momentum (b) collision [1997]
(c) voltage (d) energy
4. A 50 turn circular coil has a radius of 3 cm, it is
kept in a magnetic field acting normal to the area
of the coil. The magnetic field B is increased
from 0.10 T to 0.35 T in 2 milli second, the average
induced emf will be: [1999] V
(a) 177 V (b) 1.77 V
(c) 0.177 V (d) 17.7 V
(a) emf
5. A solenoid is 1.5 m long and its inner diameter is
t
4.0 cm. It has 3 layers of windings of 1000 turns
each and carries a current of 2.0 amperes. The
magnetic flux for a cross-section of the solenoid
is nearly [2000] (b) emf
(a) 4.1´ 10 -5 weber (b) 5.2 ´ 10 -5 weber t

(c) 6.31´10 -3 weber (d) 2.5 ´10 -7 weber


6. The current flows from A to B as shown in figure,
then the direction of the induced current in the (c) emf
t
loop will be : [2001]

(d) emf
A B t
(a) straight line (b) anti-clockwise
(c) clockwise (d) none of these
Electromagnetic Induction P-153

9. A metallic ring is dropped down, keeping its plane 13. Fig represents an area A = 0.5 m2 situated in a
perpendicular to a constant and horizontal uniform magnetic field [2012]
magnetic field. The ring enters the region of
magnetic field at t = 0 and completely emerges
out at t = T sec. The current in the ring varies as B

60
I
(a) [2006]
0 t T
B = 2.0 weber/m2 and making an angle of 60º with
respect to magnetic field. The value of the
I magnetic flux through the area would be equal to
(b)
(a) 2.0 weber (b) 3 weber
0 t T
(c) 3 / 2 weber (d) 0.5 weber
14. A charged particle moves through a magnetic
I field in a direction perpendicular to it. Then the
(c) (a) velocity remains unchanged [2013]
0 t (b) speed of the particle remains unchanged
T
(c) direction of the particle remains unchanged
(d) acceleration remains unchanged
15. Wires 1 and 2 carrying currents i 1 and i 2
I
(d) respectively are inclined at an angle q to each
0 other. What is the force on a small element dl of
t T wire 2 at a distance of r from wire 1 (as shown in
10. Which of the following conclusion can be drawn figure) due to the magnetic field of wire 1?
from the result [2010] 2
1
ur ur
Ñò
B.dA = 0
i1 r i2
(a) magnetic field is zero everywhere [2013]
(b) magnetic monopole cannot exist q dl
(c) magnetic lines of force do not intersect each
other m0 m0
(d) a current produces magnetic field (a) i1i 2 dl tan q (b) i1i 2 dl sin q
2 pr 2pr
11. A wheel with ten metallic spokes each 0.50m long
is rotated with a speed of 120 rev/min in a plane m0 m0
(c) i1i 2 dl cos q (d) i1i 2 dl sin q
normal to the earth’ s magnetic field at the place. 2 pr 4pr
If the magnitude of the field is 0.40 G, the induced 16. The flux linked with a coil at any instant 't' is
emf between the axle and the rim of the wheel is given by
equal to [2010] f = 10t2 – 50t + 250. The induced emf at t = 3s is
(a) 1.256 × 10 – 3 V (b) 6.28 × 10 – 4 V (a) –190 V (b) –10 V [2014]
(c) 1.256 × 10 – 4 V (d) 6.28 × 10 – 5 V (c) 10 V (d) 190 V
12. The magnetic flux through a circuit carrying a 17. In an AC generator, a coil with N turns, all of the
current of 2.0 A is 0.8 weber. If the current reduces same area A and total resistance R, rotates with
to 1.5 A in 0.1 s, the induced emf be : [2011] frequency w in a magnetic field B. The maximum
(a) 2.0 V (b) 4.0 V value of emf generated in the coil is [2015]
(c) 8.0 V (d) none of the above (a) N.A.B.R.w (b) N.A.B.
(c) N.A.B.R. (d) N.A.B.w
EBD_7100
P-154 Topicwise AIIMS Solved Papers – PHYSICS
r
18. In an inductor of self-inductance L = 2 mH, 20. Assertion : An emf E is induced in a closed
current changes with time according to relation loop where magnetic flux is varied. The induced
r
i = t2e–t. At what time emf is zero? (2016) E is not a conservative field.
(a) 4s (b) 3s r r
(c) 2s (d) 1s Reason : The line integral E.d l around the
19. A conducting rod AB moves parallel to X-axis in closed loop is nonzero. [2006]
a uniform magnetic field, pointing in the positive 21. Assertion : An electric motor will have maximum
X-direction. The end A of the rod gets (2017) efficiency when back emf becomes equal to half
Y of applied emf.
Reason : Efficiency of electric motor depends
B only on magnitude of back emf. [2008]
Directions for (Qs. 22-24) : Each of these questions
contains an Assertion followed by Reason. Read them
V carefully and answer the question on the basis of
following options. You have to select the one that
A
X
best describes the two statements.
O (a) If both Assertion and Reason are correct and
(a) positively charged
Reason is the correct explanation of Assertion.
(b) negatively charged
(b) If both Assertion and Reason are correct, but
(c) neutral
Reason is not the correct explanation of
(d) first positively charged and then negatively
Assertion.
charged
(c) If Assertion is correct but Reason is incorrect.
TYPE B : ASSERTION REASON QUESTIONS (d) If both the Assertion and Reason are incorrect.
Directions for (Qs. 20-21) : These questions consist 22. Assertion : An induced current has a direction
of two statements, each printed as Assertion and such that the magnetic field due to the current
Reason. While answering these questions, you are opposes the change in the magnetic flux that
required to choose any one of the following five induces the current.
responses. Reason : Above statement is in accordance with
(a) If both Assertion and Reason are correct and conservation of energy. [2011]
the Reason is a correct explanation of the 23. Assertion : Lenz's law violates the principle of
Assertion. conservation of energy.
(b) If both Assertion and Reason are correct but Reason : Induced emf always opposes the
Reason is not a correct explanation of the change in magnetic flux responsible for its
Assertion. production. [2014]
(c) If the Assertion is correct but Reason is 24. Assertion : Faraday¢s laws are consequence of
incorrect. conservation of energy.
(d) If both the Assertion and Reason are incorrect. Reason : In a purely resistive ac circuit, the
(e) If the Assertion is incorrect but the Reason is current legs behind the emf in phase. [2017]
correct.
Electromagnetic Induction P-155

Type A : Multiple Choice Questions 6. (c)


1. (c) Flux passing through the coil,
-5 -3
= 1000 ´ 4 ´ 10 = 2 ´ 10 Weber
Change in flux when the coil turns by 180°. A B
-3 -3
Direction of magnetic field due to indicated
= 2 ´10 - (-2 ´10 ) current will be in upward direction passing
through the coil. So, a current in coil will be
Df = 4 ´ 10 -3 induced so that it decreases the flux in the
coil. So, current in the clockwise direction
df 4 ´10 -3 will be induced.
e.m.f. = = = 0.4V
dt 0.01 7. (b) A changing magnetic field gives rise to
electric field as shown in the figure.
di
2. (c) e.m.f. = –L = -5 ´ 2 = -10V
dt
3. (d) Lenz’s law deals with conversion of × × × E
mechanical energy into electromagnetic × × × ×
× × × × ×
energy in case of electromagnetic × × ×
induction. × × × × ×
4. (b) Average flux in the beginning = pr2nB1 × × ×
Average flux in the final = pr2nB2
B
Increase in flux df = pr 2 n(B2 - B1 ) The relation between electric field and
= 3.14 × (0.03)2 × 50 (0.35 – 0.10) changing magnetic field is
df dBA
Ñò
= 314 × 45 × 25 × 10–7 Edl = =
Given time dt = 2 × 10–3 dt dt

df 314 ´ 45 ´ 25 ´ 10-7 pr 2 dB r dB
e=- = 2prE = Þ E=
dt 2 dt
dt 2 ´ 10-3
Here dB = 0.01 – ( – 0 .01) = 0.02
= 1.77 V (approx.)
5. (c) Magnetic flux (f) = nBA T 1
dt = =
where n is number of turns, B is magnetic 2 2 ´ 100
field and A is area
1 0.02 2 ´ 100 ´ 0.02
Given, n = 1000, i = 2 amp r = 0.02 E= ´ = = 2 volt
2 1 2
and l = 1.5 m
2 ´ 100
Magnetic field of solenoid of length l is
8. (a) We know that electromagnetic e.m.f.
m 0i induced,
B=
l df
e= -
m0 niA dt
\f = T
l Initially, df will be positive (during first
4
3 ´ 10-7 ´ 4p ´ (1000)2 ´ 2 ´ p (2 ´ 10 -2 ) 2 time period) then it becomes negative during
f=
1.5 T
= 6.31 × 10–3 Wb the period from to T .
4 2
EBD_7100
P-156 Topicwise AIIMS Solved Papers – PHYSICS

T 3T ur ur
During the period to it is again df d(NB.A)
2 4 17. (d) e=- =-
dt dt
T
positive and in the last time it is
4 d
negative. Accordingly sign of emf = -N (BA cos wt ) = NBAw sin wt
dt
produced will be changed. Figure (a) fits
exactly in this change pattern. So this figure Þ e max = NBAw
represents the answer. 18. (c) L = 2mH, i = t2e–t
9. (b) When ring enters the field an emf is
induced due to change in flux. Once with di
E= -L = - L[ - t 2 e - t + 2 te - t ]
the magnetic field, there is no change in dt
flux in the ring so there is no emf. When it when E = 0
emerges out of the field, once again there
–e–t t2 + 2te–t = 0
is a flux change which creates emf in reverse
2t e–t = e–t t2
direction. Graph (b) shows this result.
10. (b) Flux of certain closed surface is zero and t = 2 sec.
so it tells that net magnetic charge is equal 19. (a) According to right hand palm rule, the
to zero. This is possible when there are two Lorentz force on free electrons in the
equal and opposite poles. conductor will be directed towards end B.
Hence, the end A gets positively charged.
Bwl 2
11. (d) e=
2 Type B : Assertion Reason Questions
æ 120 ö 20. (a) Assertion and Reason is correct and
(0.4 ´ 104 ) ´ ç 2 p ´ ÷ ´ (0.5)
2
Reason explains Assertion. According to
è 60 ø
= Faraday’s law of electromagnetic induction
2
= 0.628 × 10 V – 4 r r df
12. (a) flux corresponds to 2A is = 0.8 weber ò E .d l = - dt
flux corresponds to 1.5 A is = 0.6 weber So, (E) is non-conservative field as in
Df 0.2 conservative field line integral over a closed
| e |= = = 2.0V
Dt 0.1 loop is zero.
13. (d) f = BA cos q = 2.0 ´ 0.5 ´ cos 60º 21. (c) Efficiency of an electric motor is maximum
when the back emf setup in the armature is
2.0 ´ 0.5 half the value of the applied battery emf
= = 0.5 Weber .
2
Pout e Back emf
14. (b) Magnetic force acts perpendicular to the Efficiency, h = = =
Pin E Supply voltage
velocity. Hence speed remains constant.
15. (c) 22. (a)
23. (a) Lenz's law (that the direction of induced
16. (b) f = 10t 2 - 50t + 250
emf is always such as to oppose the change
df that cause it) is direct consequence of the
e=- = - (20 t - 50) law of conservation of energy.
dt
24. (c) In purely resistive circuit, the current and
e t =3 = -10 V emf are in the same phase.
21 Alternating Current

TYPE A : MULTIPLE CHOICE QUESTIONS 7. The coefficient of mutual inductance, when


magnetic flux changes by 2 × 10–2 Wb and
1. An A.C. circuit containing only capacitance, the current changes by 0.01 A is [2002]
current : [1997] (a) 8 henry (b) 4 henry
(a) lags the voltage by 90° (c) 3 henry (d) 2 henry
(b) leads the voltage by 90° 8. In an ideal parallel LC circuit, the capacitor is
(c) remains in phase with voltage charged by connecting it to a D.C. source which
(d) leads the voltage in 180° is then disconnected. The current in the circuit
2. A choke coil has: [1999] (a) becomes zero instantaneously [2003]
(a) Low inductance and low resistance (b) grows monotonically
(b) High inductance and high resistance (c) decays monotonically
(c) Low inductance and high resistance (d) oscillates instantaneously
(d) High inductance and low resistance 9. A capacitor of capacitance 2µF is connected in
3. Turn ratio in a step up transformer is 1 : 2 if a the tank circuit of an oscillator oscillating with a
Lechlanche cell of 1.5 V is connected across the frequency of 1 kHz. If the current flowing in the
input, then the voltage across the output will be circuit is 2 m A, the voltage across the capacitor
(a) 0.1 V (b) 1.5 V [2000] will be : [2003]
(c) 0.75 V (d) zero (a) 0.16 V (b) 0.32 V
4. In the circuit shown below what will be the (c) 79.5 V (d) 159 V
reading of the voltmeter and ammeter ? [2000] 10. A 50 Hz a.c. source of 20 volt is connected across
(Total impedance of circuit Z = 100W) R and C as shown in figure. The voltage across
R is 12 volt. The voltage across C is : [2004]
R C

A
300 V 300 V V
50 Hz

(a) 200 V, 1A
220 V
(b) 800 V, 2A
~
(c) 100 V, 2A (d) 220 V, 2.2 A (a) 8 V
5. In a circuit the coil of a choke : [2001] (b) 16 V
(a) decreases the current (c) 10 V
(b) increases the current (d) not possible to determine unless values of
(c) has high resistance to D. C. circuit R and C are given
(d) no effect with the current 11. In an AC circuit the potential differences across
6. In a circuit, the current lags behind the voltage an inductance and resistance joined in series
by a phase difference of p /2, the circuit will are respectively 16 V and 20 V. The total potential
contain which of the following : [2001] difference of the source is [2007]
(a) only R (b) only C (a) 20.0 V (b) 25.6 V
(c) R and C (d) only L (c) 31.9 V (d) 53.5 V
EBD_7100
P-158 Topicwise AIIMS Solved Papers – PHYSICS
12. With the decrease of current in the primary coil 17. In a AC circuit the voltage and current are
from 2 amperes to zero value in 0.01s the emf described
p
generated in the secondary coil is 1000 volts. by V = 200sin æç 319t - ö÷ volts [2010]
The mutual inductance of the two coils is è 6ø
æ pö
(a) 1.25 H (b) 2.50 H [2007] and i = 50sin ç 314t + ÷ mA
(c) 5.00 H (d) 10.00 H è 6ø
13. An AC source of angular frequency w is fed respectively. The average power dissipated in
across a resistor R and a capacitor C in series. the circuit is : [2011]
The current registered is I. If now the (a) 2.5 watts (b) 5.0 watts
frequency of source is changed to w/3 (but (c) 10.0 watts (d) 50.0 watts
maintaining the same voltage), the current in 18. If we decrease the frequency of the applied A.C.
the circuit is found to be halved. Calculate with a purely capacitive load, do (1) the amplitude
the ratio of reactance to resistance at the of Vc and (2) amplitude of Ic increase, decrease
original frequency w [2008] of remain the same. [2011]
(a) (1) increase (2) same
3 2
(a) (b) (b) (1) same (2) increase
5 5 (c) (1) same (2) decrease
(d) (1) decrease (2) same
1 4 19. An inductor coil of inductance L is cut into two
(c) (d)
5 5 equal parts and both the parts are connected in
14. If an AC main supply is given to be 220 V. The parallel. The net inductance is : [2011]
average emf during a positive half cycle will be (a) L (b) L/2
(a) 198 V (b) 220 V [2009] (c) L/4 (d) 2 L.
(c) 240 V (d) 220 2 V 20. The current in resistance R at resonance is
15. A coil has an inductance of 0.7 henry and is R
joined in series with a resistance of 220 W. When
the alternating emf of 220 V at 50 Hz is applied to
it then the phase through which current lags
behind the applied emf and the wattless C
L
component of current in the circuit will be
respectively [2010] ~
(a) 30°, 1 A (b) 45°, 0.5 A V = V0 sin w t
(c) 60°, 1.5 A (d) none of these
(a) zero [2012]
16. An inductor and a resistor in series are
(b) minimum but finite
connected to an A.C. supply of variable
(c) maximum but finite
frequency. As the frequency of the source is
(d) infinite
increased, the phase angle between current and
21. An inductance L having a resistance R is
the potential difference across L will: [2010]
connected to an alternating source of angular
L frequency w. The Quality factor Q of inductance
is [2012]
(a) R/ wL (b) (wL/R)2
~ R (c) (R /wL)1/2 (d) wL/R
22. In an A.C. circuit, the current flowing in
inductance is I = 5 sin (100 t – p/2) amperes and
(a) first increase and then decrease the potential difference is V = 200 sin (100 t)
(b) first decrease and then increase volts. The power consumption is equal to [2013]
(c) go on decreasing (a) 1000 watt (b) 40 watt
(d) go on increasing (c) 20 watt (d) zero
Alternating Current P-159

23. For the circuit shown in the fig., the current a potential difference of 0.1 mV. The values of
through the inductor is 0.9 A while the current the capacitor to produce resonance is (Take
through the condenser is 0.4 A. Then (2013) p2 = 10) [2015]
C (a) 2.5 pF (b) 5.0 pF
(c) 25 pF (d) 50 pF
29. A coil has resistance 30 ohm and inductive
L reactance 20 ohm at 50 Hz frequency. If an ac
source, of 200 volt, 100 Hz, is connected across
~ the coil, the current in the coil will be [2016]
V = V0 sin wt (a) 4.0 A (b) 8.0 A
(a) current drawn from generator I = 1.13 A 20
(c) A (d) 2.0 A
(b) w = 1/(1.5 L C) 13
(c) I = 0.5 A 30. An ideal coil of 10H is connected in series with a
(d) I = 0.6 A resistance of 5W and a battery of 5V. After 2 sec
24. L, C, R represent physical quantities inductance, the connection is made, the current flowing in
capacitance and resistance respectively. The ampere in the circuit is [2016]
combinations which have the dimensions of (a) (1 – e )–1 (b) (1 – e)
frequency are [2013] (c) e (d) e–1
(a) 1/RC (b) R/L 31. An inductor, a resistor and a capacitor are joined
(c) 1 / LC (d) C/L in series with an AC source. As the frequency of
25. An inductance L having a resistance R is the source is slightly increased from a very low
connected to an alternating source of angular value, the reactance of the [2017]
frequency w. The Quality factor Q of inductance (a) inductor increases
is [2014] (b) resistor increases
(c) capacitor increases
(a) R/ wL (b) (wL/R)2
(d) circuit increases
(c) (R /wL)½ (d) wL/R
26. A step down transformer is connected to 2400 TYPE B : ASSERTION REASON QUESTIONS
volts line and 80 amperes of current is found to Directions for (Qs. 32-34) : These questions consist
flow in output load. The ratio of the turns in of two statements, each printed as Assertion and
primary and secondary coil is 20 : 1. If transformer Reason. While answering these questions, you are
efficiency is 100%, then the current flowing in required to choose any one of the following five
the primary coil will be [2015] responses.
(a) 1600 amp (b) 20 amp (a) If both Assertion and Reason are correct and
(c) 4 amp (d) 1.5 amp the Reason is a correct explanation of the
27. The mutual inductance of a pair of coils, each of Assertion.
N turns, is M henry. If a current of I ampere in (b) If both Assertion and Reason are correct but
one of the coils is brought to zero in t second, Reason is not a correct explanation of the
the emf induced per turn in the other coil, in Assertion.
volt, will be [2015] (c) If the Assertion is correct but Reason is incorrect.
MI NMI (d) If both the Assertion and Reason are incorrect.
(a) (b) (e) If the Assertion is incorrect but the Reason is
t t
correct.
MN MI
(c) (d) 32. Assertion : In series LCR circuit resonance can
It Nt take place.
28. The tuning circuit of a radio receiver has a Reason : Resonance takes place if inductance
resistance of 50W, an inductor of 10 mH and a and capacitive reactances are equal and
variable capacitor. A 1 MHz radio wave produces opposite. [1998]
EBD_7100
P-160 Topicwise AIIMS Solved Papers – PHYSICS
33. Assertion : Faraday’s laws are consequences Reason : At high voltage supply power losses
of conservation of energy. are less. [2011, 2013]
Reason : In a purely resistive A.C. circuit, the 38. Assertion : A capacitor blocks direct current in
current lags behind the e.m.f. in phase. [2002] the steady state.
34. Assertion : No power loss associated with pure Reason : The capacitive reactance of the
capacitor in ac circuit. capacitor is inversely proportional to frequency
Reason : No current is flowing in this circuit. f of the source of emf. [2011]
[2007] 39. Assertion : In the purely resistive element of a
Directions for (Qs. 35-41) : Each of these questions series LCR, AC circuit the maximum value of rms
contains an Assertion followed by Reason. Read them current increases with increase in the angular
carefully and answer the question on the basis of frequency of the applied e.m.f.
following options. You have to select the one that Reason :
best describes the two statements.
2
(a) If both Assertion and Reason are correct and e max æ 1 ö
Imax = , z = R 2 + ç wL - ÷ ,
Reason is the correct explanation of Assertion. z è wC ø
(b) If both Assertion and Reason are correct, but where Imax is the peak current in a cycle.
Reason is not the correct explanation of
[2012]
Assertion.
(c) If Assertion is correct but Reason is incorrect. 40. Assertion : In the purely resistive element of a
(d) If both the Assertion and Reason are incorrect. series LCR, AC circuit the maximum value of rms
35. Assertion : Ohm’s law cannot be applied to a.c current increases with increase in the angular
circuit. frequency of the applied emf.
Reason : Resistance offered by capacitor for a.c Reason :
source depends upon the frequency of the e max æ 1 ö
2
source. [2009] Imax = , z = R 2 + ç wL - ÷ ,
z è wC ø
36. Assertion : The resistance offered by an
where Imax is the peak current in a cycle.
inductor in a d.c circuit is always constant.
[2016]
Reason : The resistance of inductor in steady 41. Assertion : A laminated core is used in
state is non-zero. [2010] transformers to increase eddy currents.
37. Assertion : Long distance power transmission Reason : The efficiency of a transformer
is done at high voltage. increases with increase in eddy currents. [2017]
Alternating Current P-161

Type A : Multiple Choice Questions 8. (d) In an LC circuit current oscillates between


maximum and minimum value. So, LC circuit
1. (b) In an a.c. circuit containing resistance only needs oscillations (electrical). It occurs due
voltage & current remain in the same phase. to discharging and charging of capacitor
If circuit contains inductance only, voltage and magnetisation and demagnetisation
remains ahead of curr ent by phase of inductor.
difference of 90°. 9. (a) Here, oscillating frequency
If circuit contains capacitance only, current = 1 kHz = 103 Hz
remains ahead of voltage by a phase 1
difference of 90°. Reactance of capacitor =
wC
2. (d) A choke coil has high inductance and low 1 10 3
resistance so, it is capable of producing = 3 -6
=
2p ´ 10 ´ 2 ´ 10 4p
very high induced e.m.f. which produces
Potential over capacitance
discharge in the tube.
3. (d) A transformer can not step up a d.c. input 1 103 2 7
´i = ´ 2 ´ 10 -3 = = = 0.16 V
so output potential here will be zero. No wC 4p 4p 44
potential will be induced in the secondary 2
æ 1 ö
10. (b) E 2 = VR 2 + ç = VR 2 + VC 2
coil. è wC ÷ø
4. (d) Total impedence of the circuit = 100W [Phase difference between VR & VC is 90°]
220 (20)2 = (12)2 + VC2
Current = = 2.2 A
100 Þ VC2 = 400 – 144 = 256
Potential drop over inductance and
Þ V = 16
capacitance is 300 V. As they are in opposite C
phase they will sum up to zero. So whole of 11. (b) In any ac (LR) circuit, total potential is
220 volt (external source) will come over given by V = VR2 ∗ V 2 where VR and VL
the resistance. L
are potential across resistance and
(VL - VC ) 2 + VR 2 = (220) 2 inductance respectively.
As VL = VC so VR = 220 volt
Hence V = (16) 2 ∗ (20) 2 < 256 ∗ 400
5. (a) In a circuit with a.c. source, choke coil which = 25.6 V
is essentially an inductor with h igh 12. (c) The emf induced in secondary is given by
reactance, is used to decrease the current
,MdI æ 2 , 0 ö÷
without loss of energy. No heat is generated e= Þ 1000 = M ççç ÷
so no loss of energy. When we use dt è 0.01 ø÷
resistance to reduce current, there is loss (since current is reduced dI = – ve)
of electrical energy in the form of heat 1000´ 0.01
ÞM= = 5.00 H.
generated. 2
6. (d) If a circuit contains L, voltage leads current 13. (a) At angular frequency w, current through
p RC circuit is given by
by a phase angle of .
2 Vrms Vrms
7. (d) We know that I rms = =
2 2 …(1)
R + XC R + (1/ wC) 2
2
f = Mi
df = Mdi w
When angular frequency is changed to ,
3
df 2 ´ 10 -2 then the current becomes,
M= = = 2 henry
di 1 ´ 10-2
EBD_7100
P-162 Topicwise AIIMS Solved Papers – PHYSICS

I rms Vrms Vrms .. . (2) 1 Vc


= = 18. (c) Xc = and ic =
2
æ 1 ö
2
æ 3 ö
2 wc Xc
R2 + R2 + ç
ç æ wö ÷ è wC ø÷ With decrease in frequency, Xc increases
ç èç ø÷ C ÷
è 3 ø and hence ic decreases.
Dividing (i) by (ii) 19. (c) The inductance is proportional to the length
of the coil. So each part will have inductance
R 2 + (3 / wC) 2 L
2= . In parallel their equivalent becomes
R 2 + (1/ wC) 2 2
L1L 2 L
é 1 ö ù
2 2 = .
or 4 ê R 2 + æç ú = R 2 æ 3 ö
+ L1 + L 2 4
è wC ÷ø ú çè ÷
wC ø
ëê û 20. (c) At resonance XL = XC
5 1/ wC 3 Þ R & current is maximum but finite, which
3R 2 = Þ =
w 2 C2 R 5 E
, where E is applied voltage.
is Imax =
XC 3 R
Þ = 21. (d)
R 5
14. (a) Erms = 220 V Potential drop across capacitor or inductor
Q=
E0 Potential drop across R.
\ Erms = Þ E0 = 2 Erms wL
2 =
Average e.m.f over half cycle R
22. (d) Power, P = Ir.m.s. × Vr.m.s. × cos f
2
= E 0 = 0.637 × 1.41 × 220 = 198.15 V In the given problem, the phase difference
p
between voltage and current is p/2. Hence
15. (b) L = 0.7H, R = 220W, E0 = 220V, v = 50 Hz.
This is an L – R circuit P = Ir.m.s. × Vr.m.s. × cos(p/2) = 0.
Phase difference, 23. (c) The current drawn by inductor and
capacitor will be in opposite phase. Hence
X wL 2pvL
tan f = L = = net current drawn from generator
R R R = IL – IC = 0.9 – 0.4 = 0.5 amp.
22 1 1
[XL= 2pvL = 2 × × 50 × 0.7 = 220W] 24. (c) =
7 LC (ML2 T -2 A -2 ) ´ (M -1L-2 T 4 A 2 )
220
= = 1 or, f = 45° 1
220
= = T -1
Wattless component of current T 2
I 1 E0 25. (d)
= I0 sin f = 0 = .
2 2 Z Potential drop across capacitor or inductor
Q=
1 220 1 220 Potential drop across R.
= = .
2 X L2 + R 2 2 2202 + 220 2 wL
=
R
1
= = 0.5 A Ιs n p 80 20
2 26. (c) = ; = or Ι p = 4 amp.
Ιp ns Ιp 1
X
16. (d) tan q = L NMI
R d dI
27. (a) E = ( NMI) Þ E = NM Þ E=
dt dt t
V0i 0 cos f 200 ´ 50 ´ 103 p
17. (a) P = = cos E MI
2 2 3 emf induced per unit turn = =
N t
Alternating Current P-163

28. (a) L = 10 mHz = 10–2 Hz 33. (c) Faraday’s laws of electromagnetic induction
f = 1MHz = 106 Hz are consequences of conservation of
1 energy. It involves only transformation of
f= energy into electrical energy.
2p LC
In purely resistive circuit, current and
1
f2 = 2 voltage are in the same phase.
4p LC 34. (c) For a pure capacitor circuit average power
1 1 is given by Pav = EvIv cos (–p/2),
ÞC = =
4p f L 4 ´10 ´10-2 ´1012
2 2
(as f = –p/2)
10 -12 \ EvIv (0) = 0 Þ Pav = 0
= = 2.5 pF thus no power loss occurs. A pure capacitor
4 acts as a block of direct current (d.c) and
29. (a) If w = 50 × 2p then wL = 20W
easy path to a.c since reactance
If w¢ = 100 × 2p then w¢L = 40W
Current flowing in the coil is 1
XC = = ¥ for d.c (f = 0 for d.c). Hence
200 200 200 2pf c
I= = =
Z we can say no d.c. flows but a.c part is
R 2 + (w¢L)2 (30)2 + (40) 2
I = 4A. there hence reason is false.

( )
R 35. (d) Assertion is false and Reason is false.
30. (a) We have, I = - t
Io 1 - e L 36. (d) Resistance offered by an inductor in a d.c.
(When current is in growth in LR circuit) circuit at t = 0 is infinity, which decreases
æ to zero at steady state.
- ´2 ö
æ 5
- tö
R
E 5
= ç1 - e L ÷ = ç1 - e 10 ÷ æ Pö
2
R çè ÷ø 5 çè ÷ø 37. (a) Power loss = I2R = ç ÷ R
è Vø
= (1 – e–1) [P = Transmitted power]
31. (c) The phase angle is given by
38. (a)
w L 2 p ´ 50 ´ 0.21
tan f = = = 5.5 39. (c)
R 12
40. (c)
f = tan -1 5.5 = 80º
41. (d) Large eddy currents are produced in non-
Type B : Assertion Reason Questions laminated iron core of the transformer by
the induced emf, as the resistance of bulk
32. (a) In series resonance circuit, current becomes
iron core is very small. By using thin iron
maximum because total impedence becomes
sheets as core the resistance is increased.
zero. In case of LC circuit,
Laminating the core substantially reduces
1 the eddy currents. Eddy current heats up
Total impedence = wL - =0
wC the core of the transformer. More the eddy
1 1 currents greater is the loss of energy and
Þ wL = Þ w2 =
wC LC the efficiency goes down.
EBD_7100
P-164 Topicwise AIIMS Solved Papers – PHYSICS

22 Electromagnetic Waves

TYPE A : MULTIPLE CHOICE QUESTIONS 8. Electromagnetic wave consists of periodically


oscillating electric and magnetic vectors
1. According to Maxwell’s hypothesis, changing (a) in mutually perpendicular planes but
of electric field give rise to : [1998] vibrating with a phase difference of p
(a) magnetic field (b) pressure gradient (b) in mutually perpendicular planes but
(c) charge (d) voltage p
2. Frequency of infrared wave is approximately: vibrating with a phase difference of
2
(a) 1018 Hz (b) 1014 Hz [1999] (c) in randomly oriented planes but vibrating
(c) 109 Hz (d) 1016 Hz in phase
3. Which wavelength of sun is used finally as (d) in mutually perpendicular planes but
electric energy? [2007] vibrating in phase [2017]
(a) Radio waves (b) Infra red waves
(c) Visible light (d) Micro waves TYPE B : ASSERTION REASON QUESTIONS
4. If the magnetic field of a light wave oscillates Directions for (Qs. 9-10) : These questions consist
parallel to y-axis and is given by By = Bm sin (kz
of two statements, each printed as Assertion and
– wt), the direction of wave travel and the axis
Reason. While answering these questions, you are
along which the electric vector oscillates is : required to choose any one of the following five
(a) + ve y – axis, z– axis [2011]
responses.
(b) – ve z – axis, x– axis
(a) If both Assertion and Reason are correct and
(c) – ve x – axis, y– axis the Reason is a correct explanation of the
(d) – ve x – axis, z– axis
Assertion.
5. The energy of electromagnetic wave in vacuum
(b) If both Assertion and Reason are correct but
is given by the relation (2013) Reason is not a correct explanation of the
E2 B2 1 1 Assertion.
(a) + (b) e 0 E 2 + µ0 B 2 (c) If the Assertion is correct but Reason is
2e 0 2µ 0 2 2
incorrect.
(d) If both the Assertion and Reason are incorrect.
E 2 + B2 1 B2
(c) (d) e0E 2 + (e) If the Assertion is incorrect but the Reason is
c 2 2µ0 correct.
6. Which of the following are not electromagnetic 9. Assertion : X-ray travel with the speed of light.
waves? [2014] Reason : X-rays are electromagnetic rays. [2001]
(a) cosmic rays (b) g-rays 10. Assertion : Dipole oscillations produce
(c) b-rays (d) X-rays. electromagentic waves.
7. Light wave is travelling
r along y-direction. If the Reason : Accelerated charge produces
corresponding E vector atr any time is along electromagnetic waves. [2007]
the x-axis, the direction of B vector at that time Directions for (Qs. 11-13) : Each of these questions
is along y [2015] contains an Assertion followed by Reason. Read them
carefully and answer the question on the basis of
(a) y-axis
following options. You have to select the one that
(b) x-axis x best describes the two statements.
(c) + z-axis (a) If both Assertion and Reason are correct and
(d) – z-axis z Reason is the correct explanation of Assertion.
Electromagnetic Waves P-165

(b) If both Assertion and Reason are correct, but Reason : Increase of ozone increases the amount
Reason is not the correct explanation of of ultraviolet radiation on earth. [2014]
Assertion. 12. Assertion : Radio waves can be polarised.
(c) If Assertion is correct but Reason is incorrect. Reason : Sound waves in air are longitudinal in
(d) If both the Assertion and Reason are incorrect. nature. [2016]
11. Assertion : Environmental damage has 13. Assertion : Microwaves are better carrier of
increased the amount of ozone in the signals than optical waves.
atmosphere. Reason : Microwaves move faster than optical
waves. [2017]

Type A : Multiple Choice Questions 7. (c) Light wave is an electromagnetic wave in


which E and B are at right angles to each
1. (a) Changing of electric field produces other as well as at right angles to the
displacement current which produces direction of wave propagation.
magnetic field. So, changing electric field
8. (d) Electromagnetic wave consists of
produces magnetic field.
periodically oscillating electric and
2. (b) We know that visible light has wavelength magnetic vectors in mutually perpendicular
in the region of 4000Å-7000Å. If we take planes but vibrating in phase.
infrared as approximately having
wavelength of 7000Å then applying nl = c Type B : Assertion Reason Questions
n ´ 7000 ´ 10 -10 = 3 ´ 108 9. (a) All electromagnetic waves have same
speed in vacuum. X-ray is a high energy
3 ´ 108 3 electromagnetic wave.
n= -7
= ´ 1015 Hz
7 ´ 10 7 10. (b) Hertz produced em waves by oscillating
charge between dipolar electric field.
= 4.3 ´ 1014 Hz
A charge moving with non-zero
3. (b) The heating property of Infra red waves is acceleration where both magnetic and
used in solar heater and solar cells. Hence electric field are varying emits em waves
option (b) is correct. but this does not explain assertion.
4. (b) Given By = Bm sin (kz – wt). According to 11. (a) Ozone layer in the stratosphere helps in
this equation the direction of propagation protecting life of organism form ultraviolet
is along z axis. The direction of electric radiation on earth. Ozone layer is depleted
vector must be perpendicular both y and due to of several factors like use of
z-axis. So it is along x-axis. chlorofluoro carbon (CFC) which is the
1 cause of environmental damages.
5. (d) e 0 E 02 is electric energy density.
2 12. (c) Radio waves can be polarised because they
B2 are transverse in nature. Sound waves in
is magnetic energy density. air are longitudinal in nature.
2m 0
1 2 B 20 13. (d) The optical waves used in optical fibre
So, total energy = e E +
2 0 0 2m 0 communication are better carrier of signals
6. (c) b -rays are the beam of fast moving than microwaves. The speed of microwave
and optical wave is the same in vacuum.
electrons.
EBD_7100
P-166 Topicwise AIIMS Solved Papers – PHYSICS

23 Ray Optics and Optical


Instruments
TYPE A : MULTIPLE CHOICE QUESTIONS 6. How can the chromatic aberration be corrected
[1997]
1. If two mirrors are kept at 45° to each other and a (a) By providing different suitable curvature
body is placed in the middle then total number to its two surface
of images formed is : [1997] (b) By combining it with another lens of
(a) 7 (b) 8 opposite nature
(c) 14 (d) 4 (c) By reducing its aperture
2. An astronomical telescope of ten fold angular (d) By providing proper polishing of its two
magnification has a length of 44 cm. The focal surfaces
length of the objective is : [1997] 7. When a beam of light from air enters into the
(a) 44 cm (b) 440 cm water, the characteristics of light will not be
(c) 40 cm (d) 4 cm changed, is : [1997]
3. The refractive index of diamond is 2.0, velocity (a) frequency (b) speed
of light in diamond in cm per second is (c) colour (d) amplitude
approximately : [1997] 8. Mirage is a phenomenon due to : [1998]
(a) 1.5 × 1010 (b) 2.0 × 1010 (a) refraction of light
(c) 6 × 1010 (d) 3 × 1010 (b) diffraction of light
4. A cylindrical vessel is filled with water (µ = 4/3) (c) total internal reflection of light
as shown in figure. A coin placed in water at the (d) none of these
bottom appears upto maximum distance of : 9. In an astronomical microscope, the focal length
[1997] of the objective is made : [1998]
(a) shorter than that of the eye piece
(b) greater than that of the eye piece
H
(c) half of the eye piece
(d) equal to that of the eye piece
10. Light appears to travel in a straight line, because
3H (a) its wavelength is very small [1998]
(a) from the surface (b) its velocity is large
4
(c) it is not absorbed by surroundings
H
(b) from the surface (d) it is reflected by surroundings
4
11. Sky appears to be blue in clear atmosphere due
(c) H from the surface
to which property of light : [1999]
H (a) Scattering (b) Polarization
(d) from the surface
2 (c) Diffraction (d) Dispersion
5. The critical angle for the material of a prism is 12. A doctor prescribes spectacles to a patient with
45°and its refracting angle is 30°. A
a combination of a convex lens of focal length
monochromatic ray goes out perpendicular to
the surface of emergence from the prism. Then 40 cm, and concave lens of focal length 25 cm
the angle of incidence on the prism will be : then the power of spectacles will be : [2000]
(a) 60° (b) 75° [1997] (a) – 6.5 D (b) 1.5 D
(c) 45° (d) 30° (c) – 1.5 D (d) – 8.5 D
Ray Optics and Optical Instruments P-167

13. Match the items in list-1 with items in list-II and


collect the correct answers from the codes given
v v
below the lists [2000]
(a) (b)
List-I List-II
u u
I. Myopia A. Bifocal lens
II. Hyper-metropia B. Cylindrical lens
III. Presbyopia C. Concave lens v v
IV. Astigmation D. Convex lens (c) (d)
(a) I-D, II-C, III-A, IV-B u u
(b) I-C, II-D, III-A, IV-B 20. An object is immersed in a fluid. In order that the
(c) I-B, II-D, III-A, IV-C object becomes invisible, it should : [2004]
(d) I-A, II-B, III-C, IV-D (a) behave as a perfect reflector
(b) absorb all light falling on it
14. When a ray of light enters a glass slab, then
(c) have refractive index one
[2000]
(d) have refractive index exactly matching with
(a) its frequency and wavelength changes that of the surrounding fluid
(b) its frequency does not change 21. Sodium lamps are used in foggy conditions
(c) only frequency changes because : [2004]
(d) its frequency and velocity changes (a) yellow light is scattered less by the fog
15. An equilateral prism is made of a material of particles
refractive index 3 . The angle of minimum (b) yellow light is scattered more by the fog
deviation for the prism is [2000] particles
(c) yellow light is unaffected during its passage
(a) 90° (b) 60°
through the fog
(c) 45° (d) 30° (d) Wavelength of yellow light is the mean of
16. A concave mirror having the focal length 15 cm, the visible part of the spectrum
forms an image having twice of the linear 22. An endoscope is employed by a physician to
dimensions of the object. If the image is virtual, view the internal parts of a body organ. It is
then the position of the object will be : [2001] based on the principle of : [2004]
(a) 7.5 cm (b) 22.5 cm (a) refraction
(c) 40 cm (d) 30 cm (b) reflection
17. Four lenses having the focal length of + 15 cm, (c) total internal reflection
20 cm, +150 cm, and +250 cm respectively are (d) dispersion
provided to make an astronomical telescope. The 23. A telescope has an objective lens of focal length
focal length of the eyepiece to produce the 200 cm and an eye piece with focal length 2 cm.
largest magnification, should be : [2001] If this telescope is used to see a 50 metre tall
(a) + 250 cm (b) + 155 cm building at a distance of 2 km, what is the height
(c) 25 cm (d) + 15 cm of the image of the building formed by the
18. The Cauchy’s dispersion formula is [2002] objective lens? [2005]
(a) 5 cm (b) 10 cm
(a) h = A + B l + Cl
2 4
(c) 1 cm (d) 2 cm
(b) µ = A + B l–2 + Cl4 24. In refraction, light waves are bent on passing
(c) µ = A + B l2 + Cl–4 from one medium to the second medium, because,
(d) µ = A + B l–2 + Cl–4 in the second medium: [2006]
19. In an experiment to find the focal length of a (a) the frequency is different
concave mirror, a graph is drawn between the (b) the coefficient of elasticity is different
magnitudes of u and v. The graph looks like : (c) the speed is different
[2003] (d) the amplitude is smaller
EBD_7100
P-168 Topicwise AIIMS Solved Papers – PHYSICS
25. 1 wire mesh consisting of very small squares is 36
viewed at a distance of S cm through a (a) (b) 36 7
7
magnifying converging lens of focal length 10
cm, kept close to the eye. The magnification (c) 4 5 (d) 36 5
produced by the lens is: [2006] 32. A fish in an aquarium, 30 cm deep in water can
(a) 5 (b) 8 see a light bulb kept 50 cm above the surface of
(c) 10 (d) 20 water. The fish can also see the image of this
26. A lens is made of flint glass (refractive index bulb in the reflecting bottom surface of the
= 1.5). When the lens is immersed in a liquid of aquarium. Total depth of water is 60 cm. Then
refractive index 1.25, the focal length: [2006] the apparent distance between the two images
seen by the fish is (mw = 4/3) [2009]
(a) increases by a factor of 1.25
760
(b) increases by a factor of 2.5 (a) 140 cm (b) cm
3
(c) increases by a factor of 1.2
280 380
(d) decreases by a factor of 1.2 (c) cm (d) cm
3 3
27. A leaf which contains only green pigments, is
illuminated by a laser light of wavelength 0.6328 33. The focal length of the objective and eye piece
mm. It wound appear to be: [2006] of a telescope are respectively 200 cm and 5 cm.
The maximum magnifying power of the telescope
(a) brown (b) black
will be [2010]
(c) red (d) green
(a) – 100 (b) – 60
28. The focal length of the objective and eye lenses (c) – 48 (d) – 40
of a microscope are 1.6 cm and 2.5 cm respectively.
34. A thin prism P1 with angle 6° and made from
The distance between the two lenses is 21.7 cm.
glass of refractive index 1.54 is combined with
If the final image is formed at infinity, what is the
another thin prism P2 of refractive index 1.72 to
linear magnification? [2007]
produce dispersion without deviation. The angle
(a) 11 (b) 110 of prism P2 will be [2011]
(c) 1.1 (d) 44 (a) 4° 30' (b) 8.5°
29. The camera lens has an aperture of f and the (c) 6.5° (d) none of these
exposure time is (1/60)s. What will be the new 35. The focal length of a converging lens are fV and
exposure time if the aperture become 1.4 f ? fR for violet and red light respectively. Then
1 1 [2012]
(a) (b) [2007]
42 56 (a) fV > fR (b) fV = fR
1 1 (c) fV < fR (d) any of the three is
(c) (d)
72 31 possible depending on the value of the average
30. A thin glass (refractive index 1.5) lens has optical refractive index m
power of – 5 D in air. Its optical power in a liquid 36. A plano-convex lens of focal length 30 cm has
medium with refractive index 1.6 will be [2008] its plane surface silvered. An object is placed 40
(a) – 1D (b) 1 D cm from the lens on the convex side. The
(c) – 25 D (d) 25 D distance of the image from the lens is [2013]
31. A fish looking up through the water sees the (a) 18 cm (b) 24 cm
outside world contained in a circular horizon. If (c) 30 cm (d) 40 cm
4 37. A ray of light passes through four transparent
the refractive index of water is and the fish is media with refractive indices µ1, µ2, µ3 and µ4 as
3
12 cm below the surface, the radius of this circle shown in the figure. The surfaces of all media are
in cm is [2008] parallel. If the emergent ray CD is parallel to the
incident ray AB, we must have [2013]
Ray Optics and Optical Instruments P-169

µ1 µ2 µ3 D 41. An achromatic convergent lens of focal length


20 cms is made of two lenses (in contact) of
µ4 materials having dispersive powers in the ratio
of 1 : 2 and having focal lengths f1 and f2. Which
C of the following is true ? [2015]
B (a) f1 = 10 cms, f2 = –20 cms
A (b) f1 = 20 cms, f2 = 10cms
(a) µ1 = µ2 (b) µ2 = µ3 (c) f1 = –10 cms, f2 = –20 cms
(c) µ3 = µ4 (d) µ4 = µ1 (d) f1 = 20 cms, f2 = –20 cms
38. An equilateral prism is placed on a horizontal
42. A glass prism of refractive index 1.5 is immersed
surface. A ray PQ is incident onto it. For minimum
in water (refractive index 4/3). A light beam
deviation [2014]
incident normally on the face AB is totally
R reflected to reach on the face BC if [2016]
Q S 8 A B
P (a) sin q ³
9 q
2 8
(a) PQ is horizontal (b) < sin q <
3 9
(b) QR is horizontal
(c) RS is horizontal 2
(c) sin q £
(d) Any one will be horizontal 3
39. A light ray is incident perpendicularly to one (d) None of these
face of a 90° prism and is totally internally 43. A ray of light is incident at an angle of incidence,
reflected at the glass-air interface. If the angle of i, on one face of prism of angle A (assumed to be
reflection is 45°, we conclude that the refractive small) and emerges normally from the opposite
index [2014] face. If the refractive index of the prism is m, the
angle of incidence i, is nearly equal to [2016]
mA
(a) mA (b)
2
A A
(c) (d)
m 2m
44. A green light is incident from the water to the air
- water interface at the critical angle (q). Select
the correct statement. [2017]
(a) The entire spectrum of visible light will
come out of the water at an angle of 90º to
1 the normal.
(a) n> (b) n> 2
2 (b) The spectrum of visible light whose
1 frequency is less than that of green light
(c) n< (d) n < 2
2 will come out to the air medium.
40. A man 160 cm high stands in front of a plane (c) The spectrum of visible light whose
mirror. His eyes are at a height of 150 cm from the frequency is more than that of green light
floor. Then the minimum length of the plane mirror will come out to the air medium.
for him to see his full length image is [2015] (d) The entire spectrum of visible light will
(a) 85 cm (b) 170 cm come out of the water at various angles to
(c) 80 cm (d) 340 cm the normal.
EBD_7100
P-170 Topicwise AIIMS Solved Papers – PHYSICS

TYPE B : ASSERTION REASON QUESTIONS 50. Assertion: A double convex lens (m = 1.5) has
focal length 10 cm. When the lens is immersed in
Directions for (Qs. 45-61) : These questions consist water (m = 4/3) its focal length becomes 40 cm.
of two statements, each printed as Assertion and
Reason. While answering these questions, you are 1 µ1 - µ m æ 1 1 ö
Reason: = = çç - ÷
÷ [1999]
required to choose any one of the following five f µm è R1 R 2 ø
responses. 51. Assertion : The refractive index of diamond is
(a) If both Assertion and Reason are correct and
the Reason is a correct explanation of the 6 and that of liquid is 3 . If the light travels
Assertion. from diamond to the liquid, it will totally reflected
(b) If both Assertion and Reason are correct but when the angle of incidence is 30°.
Reason is not a correct explanation of the 1
Assertion. Reason : m = , where m is the refractive
sin C
(c) If the Assertion is correct but Reason is incorrect. index of diamond with respect to liquid [2000]
(d) If both the Assertion and Reason are incorrect. 52. Assertion : The setting sun appears to be red.
(e) If the Assertion is incorrect but the Reason is Reason : Scattering of light is directly
correct. proportional to the wavelength. [2000]
45. Assertion : The colour of the green flower seen
53. Assertion : In a movie, ordinarily 24 frames are
through red glass appears to be dark.
projected per second from one end to the other
Reason : Red glass transmits only red light.
of the complete film.
[1997]
46. Assertion : Newton’s rings are formed in the Reason : The image formed on retina of eye is
reflected system when the space between the sustained upto 1/10 second after the removal of
lens and the glass plate is filled with a liquid of stimulus. [2001]
refracitve index greater than that of glass, the 54. Assertion : Blue colour of sky appears due to
central spot of the pattern is bright. scattering of blue colour.
Reason : This is because the reflection in these Reason : Blue colour has shortest wave length
cases will be from a denser to rarer medium and in visible spectrum. [2001]
the two interfering rays are reflected under similar 55. Assertion : The air bubble shines in water.
conditions. [1998] Reason : Air bubble in water shines due to
47. Assertion : Corpuscular theory fails in explaining refraction of light. [2002]
the velocities of light in air and water. 56. Assertion : The stars twinkle while the planets
Reason : According to corpuscular theory, light do not.
should travel faster in denser medium than in Reason : The stars are much bigger in size than
rarer medium. [1998] the planets. [2003]
48. Assertion : Different colours travel with different 57. Assertion : A red object appears dark in the
speed in vacuum. yellow light.
Reason : Wavelength of light depends on Reason : The red colour is scattered less.[2004]
refractive index of medium. [1998] 58. Assertion : By roughening the surface of a glass
49. Assertion : The maximum refractive index of liquid sheet its transparency can be reduced.
for total internal reflection of the ray passing Reason : Glass sheet with rough surface absorbs
through the prism as shown in figure must be 2 . more light. [2005]
59. Assertion : Diamond glitters brilliantly.
Reason : Diamond does not absorb sunlight.
[2005]
60. Assertion : In optical fibre, the diameter of the
core is kept small.
Reason : This smaller diameter of the core
ensures that the fibre should have incident angle
more than the critical angle required for total
Reason: Here, critical angle is 45° [1999] internal reflection. [2006]
Ray Optics and Optical Instruments P-171

61. Assertion : A concave mirror and convex lens ray, then this ray emerges out of first surface
both have the same focal length in air. When along the previous incident ray. This particle is
they are submerged in water, they will have same called principle of reversibility of light. [2011]
focal length. 66. Assertion : A point object is placed at a distance
Reason : The refractive index of water is smaller of 26 cm from a convex mirror of focal length 26
than the refractive index of air. [2008] cm. The image will not form at infinity.
Directions for (Qs. 62-70) : Each of these questions Reason : For above given system the equation
contains an Assertion followed by Reason. Read them 1 1 1
carefully and answer the question on the basis of + = gives v = ¥. [2012]
u v f
following options. You have to select the one that
best describes the two statements. 67. Assertion : If the angle between the two plane
(a) If both Assertion and Reason are correct and mirror is 72° and the object is asymmetrically
Reason is the correct explanation of Assertion. placed between the two mirrors, then 5 images
(b) If both Assertion and Reason are correct, but of the object will be formed. [2012]
Reason is not the correct explanation of
Assertion.
(c) If Assertion is correct but Reason is incorrect.
(d) If both the Assertion and Reason are incorrect.
62. Assertion : The image of a virtual object due to
a plane mirror is real.
Reason : If the rays seem to be converging at a
point behind a plane mirror, they are reflected
and they actually meet in front of the mirror. Reason : For given system of mirror the total
[2009] number of images formed due to successive
63. Assertion : The formula connecting u, v and f 360° 360°
reflection is equal to either or -1
for a spherical mirror is valid only for mirrors q q
whose sizes are very small compared to their 360°
accordingly as is odd or even respectively..
radii of curvature. q
Reason : Laws of reflection are strictly valid for 68. Assertion: Two convex lenses joined together
plane surfaces, but not for large spherical cannot produce an achromatic combination.
surface. [2009] Reason : The condition for achromatism is
64. Assertion : Position of image approaches focus w1 w2
+ = 0 where symbols have their usual
of a lens, only when object approaches infinity. f1 f 2
Reason : Paraxial rays incident parallel to meaning. [2013]
principal axis intersect at the focus after 69. Assertion: Critical angle is minimum for violet
refraction from lens. [2010] colour.
65. Assertion : There exists two angles of incidence æ1ö
for the same magnitude of deviation (except Reason : Because critical angle qc = sin -1 ç ÷
èmø
minimum deviation) by a prism kept in air. 1
Reason : In a prism kept in air, a ray is incident and m µ . [2014]
l
on first surface and emerges out of second 70. Assertion : Plane mirror may form real image.
surface. Now if another ray is incident on second Reason : Plane mirror forms virtual image, if object
surface (of prism) along the previous emergent is real. [2017]
EBD_7100
P-172 Topicwise AIIMS Solved Papers – PHYSICS

Type A : Multiple Choice Questions 7. (a) When electromagnetic wave enters into any
other medium, its electric component
360 360 induces dipole in the molecule of the
1. (a) No. of images = -1 = -1 = 7
q 45 medium. These dipoles have oscillating
2. (c) In case of telescope, in normal adjustment, dipole moment and its time period is the
fo same as inducing electric field of incoming
m= = 10 wave. These dipole in turn radiates
fe
electromagnetic wave having same time
and f o + f e = length of tube = 44 period. So, in this phenomenon, time period
or frequency of the wave remains unaltered.
f o = 10 f e 10f e + f e = 44 Þ f e = 4 8. (c) Mirage is a phenomenon in which we see
f o = 10 ´ 4 = 40cm an inverted image of plants and trees on
the surface and it gives the impression that
velocity of light in air
3. (a) m= there is water around the trees. It occurs
velocity of light in diamond due to total internal reflection of light.
9. (b) In astronomical telescope the focal length
3 ´ 1010 cm / sec
2= of objective is larger in comparision.
v 10. (a) Light appears to travel in a straight line
3 because diffraction (or deviation from the
Þ v = ´1010 cm / sec
2 path) is least in light. Diffraction is least
because of small wavelength of light. So
= 1.5 ´ 1010 cm / sec small wave length of light causes the light
4. (b) to travel almost in straight line.
11. (a) Sky appears blue due to scattering of light.
12. (c) For combination of lenses, power
30° 100 100
P = P1 + P2 = - = -1.5D
5. (c) 40 25
A 13. (b) In myopia, we use concave lens. In hyper
metropia we use convex lens. In presbyopia
we use bifocal lens and in astigmatism we
In this case use cylindrical lens.
r + r ' = A Þ r + 0 = 30° Þ r = 30° 14. (b) When an electromagnetic wave enters a
1 1 medium (new), it undergoes change in
m= Þ m= Þ µ= 2 velocity. v = nl
sin C sin 45°
This change in velocity is due to change in
sin A sin A wave-length. The frequency remains
Now, =µÞ = 2
sin r sin 30° constant.
15. (b) A = 60°
1 1
sin A = sin 30°. 2 = ´ 2 = A= 45° æ A + dm ö æ 60° + d m ö
sin ç sin ç
è 2 ÷ø ÷ø
2 2 è 2
6. (b) Chromatic aberration takes place when µ= Þ 3=
sin A sin 30°
white light is used as source. We get a series
2
of images overlapping each other and made
by a lens. By using combination of lenses æ 60° + dm ö 1 3
of opposite nature (convex & concave) we
sin ç
è 2 ÷ø = 3 ´ 2 = 2 = sin 60°
can reduce this aberration. 60° + dm = 120° Þ dm = 120° – 60° = 60°
Ray Optics and Optical Instruments P-173

16. (a) Since, image is virtual v is +ve 20. (d) If the refractive index of the body becomes
f = – 15 cm, u = ?, equal to surrounding liquid, there will not
v be any deviation in the direction of light
m = 2 = v = 2u neither will any light get reflected from its
u
Applying mirror formula surface. So, the object becomes invisible.
1 1 1 21. (a) The higher the wavelength the lesser the
+ = scattering. Yellow light has higher
v u f
wavelength so it is least likely to be
1 1 1 1- 2 1
- =- Þ =- scattered among all colours in the visible
2u u 15 2u 15 region (except red & orange). So this light
1 1 is visible from long distance ever when there
- =- Þ u = 7.5 cm
2u 15 is foggy weather.
or 22. (c) In endoscopy a fine thread of hollow glass
We know that virtual and erect image is tube is send into the body cavity. Light
formed when we place an object within focal from the interior enters the tube & through
length of the mirror. As focal length given = internal reflector comes out at desired angle.
15 cm. So, object distance must be less than These rays are picked up by microscopy to
15 cm out of four given option only one take a view of internal parts.
option is correct. 23. (a) u = – 2000 m, v = ?, f = 200 cm = 2 m
17. (d) In case of astronomical telescope, magni- 1 1 1 1 1 1
- = Þ + =
f v u f v 2000 2
fication = o
fe 1 1 1 1000 - 1 999
For maximum magnification, fe should be = - = =
v 2 2000 2000 2000
least. So, fe should be 15 cm.
18. (d) The Cauchy’s formula is based on the 2000 v 2000 1
v= ; =m= =
relation between wavelength of light and 999 u 999 ´ 2000 999
the refractive index. It is as follows 1
Size of image = ´ 50 ´ 100 cm » 5 cm
B C 999
m =A+ 2
+
l l4 24. (c) Since the speed of light changes in the
second medium, its direction also changes.
1 1 1
19. (c) + = is the formula which relates 1 1 1
v u f 25. (a) Using - =
u & v. v u f
1 1 1 1 1 1
= - + =
u f v v 8 10
Slope of this curve can be found by
1 1 1 4-5 1
1 dv = - = =-
differentiation - du = 0 + v 10 8 40 40
u2 v2 v = –40 cm
2
dv v v 40
Þ =- m= = = 5.
du u2 u 8
dv
is the slope which is negative so either 1 æ1 1 ö
du 26. (b) = (m - 1) ç - ÷
f ç r1 r 2 ÷
curve (c) or curve (a) is right. Now the slope è ø
depends upon the value of u & v i.e. it
keeps changing at every point as per the 1 æ1 1 ö
equation above. So figure (c) is the answer. = (1.5 - 1) ç - ÷
fa ç r1 r 2 ÷
è ø
EBD_7100
P-174 Topicwise AIIMS Solved Papers – PHYSICS

1 æ1 1 ö æ ö
= (mg - 1) ç - ÷
ç r1 r 2 ÷ f m ç 1.5 - 1 ÷
fl è ø Dividing (i) by (ii), =ç ÷ =–8
f a ç 1.5 ÷
mg 1.5 6 ç -1 ÷
l
mg = = = è 1.6 ø
ml 1.25 5 m 1
Pa = - 5 = = 1
Þ fa = -
1 æ 6 öæ 1 1 ö æ ö fa fa 5
= ç - 1÷ ç - ÷ = 1ç 1 - 1 ÷
f l è 5 ø çè r1 r 2 ÷ 5ç r ÷ 1 8
ø è 1 r2 ø Þ f m = - 8 ´ f a = -8 ´ - =
5 5
1/ fa 0.5
= m 1.6
1/ fl 1/ 5 Pm = = ´ 5 = 1D
fm 8
fl
Þ = 0.5 ´ 5 = 2.5 1 3
fa 31. (a) sin qc = =
m 4
fl = 2.5 ´ fa
3 3 R
27. (b) 0.6328 µm = 6328Å or tan q c = = =
16 - 9 7 12
This is wave length of Red colour. So this
light will be absorbed by green pigments.
R
The leaf will appear black.
28. (b) In normal adjustment, object lies close to
focus of objective and image is formed at qc qc
12 cm
Ld
infinity, magnification, m = .
fe f0
Where f0 = 1.6 cm, fe = 2.5 cm, d = 21.7 cm,
L = d – f0 = 21.7 – 1.6 = 20.1 cm (approx.) 36
Þ R=cm
21.7 ´ 20.1 436.17 7
Þ m= = = 109.1 ; 110
1.6 ´ 2.5 4 32. (b) Apparent distance of the bulb from the fish
1 d1 = 50µ + 30
29. (d) aperture = f, exposure time sec.
60
1 50 cm
area of aperture = f2, area µ
exposure time
New aperture = 1.4f area = 1.96 f2 30 cm
Ratio of areas = 1.96. Since area of aperture 60 cm
has increased thus exposure time will
decrease in same proportion, i.e., apparent distance of the image
1 1 d2 = 50m + 60 + 30
new time = < sec. \ d1 + d2 = 100m + 120
(60 /1.96) 31
1 æ 1. 5 ö æ 1 1 ö 400 760
30. (b) =ç - 1÷ çç - ÷÷ ...(i) = + 120 = cm
fa è 1 ø è R1 R 2 ø 3 3
= 253.3 cm
1 æ mg öæ 1 1 ö
= çç - 1÷÷ çç - ÷
÷ f0 æ fe ö
fm è mm ø è R1 R 2 ø 33. (c) Magnifying power M = 1+ ÷
f e çè dø
1 æ 1.5 ö æ 1 1 ö ...(ii) Least distance of distinct vision, d = 25cm
=ç - 1÷ ç - ÷
f m è 1.6 ø çè R1 R 2 ÷ø
Ray Optics and Optical Instruments P-175

l
200 æ 5 ö æ 1ö
M= - ç1 + ÷ = -40 ç1 + ÷ B
q
A
5 è 25 ø è 5ø q
R P
æ6ö
= -40 ç ÷ = -48cm.
è5ø
C
34. (a) For dispersion without deviation, the
necessary condition is a
w gh 1. 5
A2 µ -1 (1.54 - 1) 0.54 Now, gm =
w
= = 1.125
=- 1 =- =- gm
4/3
A1 µ2 - 1 (1.72 - 1) 0.72
1 8
0.54 Putting in (i), sin q = =
or, A2 = ´ 6° = 4.5° » 4°30' 1.125 9
0.72 \ sin q should be greater than or equal to
35. (c) Since l R > l V m R < m V 8
æ 1ö æ 1 ö 9
çèQ m µ ÷ø Þ f V < fR çèQ µ (m - 1)÷ø 43. (a) For normally emerge e = 0
l f
Therefore r2 = 0 and r1 = A
36. (b) Snell’s Law for Incident ray’s
37. (d)
1sin i = m sin r1 = msin A
38. (b) For minimum deviation, incident angle is For small angle
equal to emerging angle. i = mA
\ QR is horizontal. 44. (b) For critical angle qc,
39. (b) The incident angle is 45° incident angle >
1
critical angle, i > i c sin qc =
m
\ sin i > sin i c or sin 45 > sin i c For greater wavelength or lesser frequency
1 m is less. So, critical angle would be more.
sin i c = So, they will not suffer reflection and come
n
out at angles less then 90°.
1 1 1
\ sin 45° > or > Þn> 2
n 2 n Type B : Assertion Reason Questions
40. (c) The minimum length of the mirror is half the 45. (a) A green flower absorbs all the light except
length of the man. This can be proved from green coloured light. So when red glass
the fact that Ði = Ðr.
transmits only red light and falls on green
f1 w 1 flower, it absorbs all the light. Therefore,
41. (a) =- 1 =- \ f 2 = -2 f1
f2 w2 2 colour of the green flower becomes dark.
1 1 1 46. (a) Newton’s rings are formed in reflected
As = + system and if the refractive index of the
F f1 f 2
first medium is more than the second
1 1 1 1 medium, there is no reversal of phase in
\ = - = \ f1 = 10 cm
20 f1 2 f1 2 f1 reflected ray so, central fringe remains
f2 = –20 cm bright.
42. (a) The phenomenon of total internal reflection 47. (a) Corpuscular theory fails to explain the
takes place during reflection at P. velocityof light in air and water because it
1 predicted light to have more velocity in
sin q = … (i) denser medium where as the fact is just the
w
gm opposite.
When q is the angle of incidence at P So option (a) is correct.
EBD_7100
P-176 Topicwise AIIMS Solved Papers – PHYSICS
48. (e) In vacuum all the colours have same 52. (c) Setting sun appears to be red because red
velocity but their velocity changes when light which has greatest wavelength is least
they enter in a medium. In the medium their scattered and reaches our eyes the most.
velocity changes due to change in their Other wavelength are scattered to the
wavelength. longest extent. So, reason given is wrong.
v = nl 53. (c) The image formed on retina of eye is
If l changes, v also changes for a particular 1
sustained upto th second after removal
colour therefore, different colours have 16
different wavelength (l). of stimulus. So, number of frames to be
49. (a) We know that in case of total internal projected on the screen must be at most 16
reflection, the minimum refractive index is per second.
given by the relation. 54. (a) Blue colour of sky is due to scattering of
1 1 1 blue colour to the maximum extent by dust
µ= = = = 2 particles. Blue colour appears to be coming
sin c sin 45° 1 / 2
from the sky. Blue colour has the least wave-
µ= 2 length.
50. (a) Using the given relation,
1 æ 1.5 - 1 ö æ 1 1 ö
=ç ÷ç - ÷÷
10 è 1 ø çè R 1 R 2 ø
Blue colour
1 æ 1 1 ö B
= 0.5 ç - ÷ ..........(i)
10 è R1 R 2 ø
4
In the second case, m = ;
3 55. (c) Air bubble shines in water due to total
internal reflection from the surface of the
æ 4ö
1. 5 - ÷ æ
1 çç
bubble.
= 3 ÷ ç 1 - 1 ö÷ So, Assertion is correct and Reason is
f ç 4 / 3 ÷ çè R1 R 2 ÷ø .........(ii) incorrect.
ç ÷
è ø 56. (b) Stars twinkle because of changing refractive
Dividing (ii) from (i), index of atmosphere. As the apparent size
f 0.5 4 ´ 0.5 of stars are small, the effect of this change
= = on the direction of rays coming from star is
10 0.5 / 4 0.5 more pronounced.
f = 4 ×10 = 40 cm 57. (b) A red object appears dark in the yellow light
a a
µl = 3, because red object absorbs all the light
51. (e) µd = 6 ,
falling on it except red. The Reason which
l a d l is a fact can not be assigned to the
µ d = ? ; µd ´ µ l ´ µ a = 1
Assertion.
1
6 ´ d µl ´ =1 58. (c) The transparency of rough glass is reduced
3 due to scattering of light.
3 1 59. (b) Diamond glitters brilliantly because of high
d
µl = = ; lµd = 2 refractive index and less critical angle.
6 2 60. (a) Assertion and Reason are correct and
If C be the critical angle, then Reason explains Assertion.
1 1
sin C = =
µ 2
A L
C = 45°. As angle of incidence < 45°, it will
not be internally reflected. So Assertion is
incorrect Reason is correct.
Ray Optics and Optical Instruments P-177

Small diameter core, angle A is larger Hence, refractive index of water is greater
than that of air.
62. (a)
B

Q P

Large diameter core, angle B is smaller. In


the former case ÐA is large so the The image of a real object is virtual while
possibility of this angle becoming greater that of a virtual object (as shown) is real.
than critical angle is more. Hence, the 63. (c) Assertion is correct. Laws of reflection can
chance of internal reflection is more. be applied to any type of surface.
61. (d) If a mirror is placed in a medium other than 64. (b) Assertion is correct, Reason is correct.
air, its focal length does not change as 65. (a) Reason is correct explanation of Assertion.
f = R/2. But for the lens, 66. (c)
1 æ 1 1 ö 67. (a)
= ( a n g - 1) ç - ÷ 68. (a)
fg è R1 R 2 ø
69. (b)
1 æ 1 1 ö
and = ( w n g - 1) ç - ÷ 70. (b) Plane mirror may form real image, if object
fw è R1 R 2 ø is virtual.
As wng < ang, hence focal length of lens in
water increase.
The refractive index of water is 4/3 and that O (virtual)
(Real) I
of air is 1
EBD_7100
P-178 Topicwise AIIMS Solved Papers – PHYSICS

24 Wave Optics

TYPE A : MULTIPLE CHOICE QUESTIONS 6. A light of intensity I0 passes through a material


of thickness d, then the intensity will be
1. Two waves of intensities I and 4I superimposes. (a) I = I0e–dl (b) I = I0edl [2000]
Then the maximum and minimum intensities are: –ld
(c) I = I0(1 – e ) (d) none of these
(a) 9I and I (b) 3I and I [1997] 7. Light of wavelength 589.3 nm is incident
(c) 9I and 3I (d) 6I and I normally on a slit of width 0.01 mm. The angular
2. On a rainy day, if there is an oil drop on tar road, width of the central diffraction maximum at a
coloured rings are seen around this drop. This distance of 1 m from the slit, is : [2000]
is because of : [1997] (a) 0.68° (b) 0.34°
(a) total internal reflection of light (c) 2.05° (d) none of these
(b) polarisation 8. In an electron microscope the accelerating
(c) diffraction pattern voltage is increased from 20 kV to 80 kV, the
(d) interference pattern produced due to thin resolving power of the microscope will become
films
3. Interference occurs in which of the following R
(a) 2R (b) [2000]
waves? [1999] 2
(a) Transverse (b) Electromagnetic (c) 4R (d) 3R
(c) Longitudinal (d) All of these 9. How does the red shift confirm that the universe
4. In young’s experiment the monochromatic light is expanding ? [2001]
is used to illuminate two slits A and B as shown (a) wavelength of light emitted by galaxies
in figure. Interference fringes are observed on a appears to decrease
screen placed in front of the slits. Now a thin (b) wavelength of light emitted by galaxies
glass plate is placed normally in the path of beam appears to be the same
coming from the slit A, then : [1999, 2004] (c) wavelength of light emitted by galaxies
appears to increase
(d) none of these
A 10. What change occurs, if the monochromatic light
used in Young’s double slit experiment is
replaced by white light ? [2001]
(a) only the central fringe is white and all other
B fringes are observed coloured.
(b) no fringes are observed.
(a) There will be no change in fringe width (c) all the bright fringes become white.
(b) Fringe width will decrease (d) all the bright fringes are coloured between
(c) Fringe width will increase violet and red.
(d) Fringes will disappear 11. Light of wavelength 6000Å is reflected at nearly
5. The ratio of intensities of two waves is 9 : 1. If normal incidence from a soap films of refractive
they superimpose, the ratio of maximum to index 1.4. The least thickness of the film that will
minimum intensity will be : [2000] appear black is : [2002]
(a) 3 : 1 (b) 4 : 9 (a) infinity (b) 200 Å
(c) 4 : 1 (d) 1 : 9 (c) 2000 Å (d) 1000 Å
Wave Optics P-179

12. A ray of light is incident on the surface of plate


of glass of refractive index 1.5 at the polarising
angle. The angle of refraction of the ray will be : (a) |E |
(a) 33.7º (b) 43.7º [2002]
(c) 23.7º (d) 53.7º t
13. When a beam of light is used to determine the
position of an object, the maximum accuracy is
achieved if the light is : [2003]
(a) polarised (b) |E |
(b) of longer wavelength
(c) of shorter wavelength t
(d) of high intensity
14. A double slit experiment is performed with light
of wavelength 500 nm. A this film of thickness 2 (c) |E |
µm and refractive index 1.5 is introduced in the
path of the upper beam. The location of the t
central maximum will : [2003]
(a) remain unshifted
(b) shift downward by nearly two fringes
(d) |E |
(c) shift upward by nearly two fringes
(d) shift downward by ten fringes
15. An astronaut is looking down on earth's surface t
from a space shuttle an altitude of 400 km. 20. Two point white dots are 1 mm apart on a black
paper. They are viewed by eye of pupil diameter
Assuming that the astronaut's pupil diameter is
3 mm. Approximately, what is the maximum
5 mm and the wavelength of visible light is 500
distance at which these dots can be resolved by
nm, the astronaut will be able to resolve linear the eye? [Take wavelength of light = 500 nm]
objects of the size of about : [2003] (a) 1 m (b) 5 m [2008]
(a) 0.5 m (b) 5 m (c) 3 m (d) 6 m
(c) 50 m (d) 500 m 21. What happens to fringe width in Young’s double
16. When a compact disc is illuminated by a source slit experiment if it is performed in glycerine
of white light, coloured lines are observed. This instead of air [2009]
is due to : [2004] (a) shrinks (b) disappears
(a) dispersion (b) diffraction (c) unchanged (d) enlarged
(c) interference (d) refraction 22. If a polaroid is kept in the path of an uniformly
17. In case of linearly polarised light, the magnitude unpolarised light, the intensity of the transmitted
of the electric field vector : [2005] light to the intensity of the light when the
(a) does not change with time polaroid was not kept in its path is [2009]
(b) varies periodically with time 1
(a) 1 (b)
(c) increases and decreases linearly with time 2
(d) is parallel to the direction of propagation 1 1
(c) (d)
18. When exposed to sunlight, thin films of oil on 2 2 2
water often exhibit brilliant colours due to the 23. In a single slit diffraction experiment, the width
phenomenon of : [2005] of the slit is made double its original width. Then
(a) interference (b) diffraction the central maximum of the diffraction pattern
(c) dispersion (d) polarisation will become [2009]
19. Which of the following diagrams represents the (a) narrower and fainter
variation of electric field vector with time for a (b) narrower and brighter
(c) broader and fainter
circulatory polarized light? [2006]
(d) broader and brighter
EBD_7100
P-180 Topicwise AIIMS Solved Papers – PHYSICS
24. If the source of light used in a Young’s double 29. Two sources of light of wavelengths 2500 Å
slit experiment is changed from red to violet: and 3500 Å are used in Young’s double slit expt.
(a) the fringes will become brighter [2011] simultaneously. Which orders of fringes of two
(b) consecutive fringes will comes closer wavelength patterns coincide? [2013]
(c) the intensity of minima will increase (a) 3rd order of 1st source and 5th of the 2nd
(d) the central fringe- will became a dark fringe (b) 7th order of 1st and 5th order of 2nd
25. Wavelength of light used in an optical instrument (c) 5th order of 1st and 3rd order of 2nd
o o (d) 5th order of 1st and 7th order of 2nd
are l1 = 4000 A and l 2 = 5000 A , then 30. In Young’s double slit experiment, we get 10
ratio of their respective resolving powers fringes in the field of view of monochromatic
(corresponding to l1and l2) is [2012] light of wavelen gth 4000Å. If we use
(a) 16 : 25 (b) 9 : 1 monochromatic light of wavelength 5000Å, then
(c) 4 : 5 (d) 5 : 4 the number of fringes obtained in the same field
26. The correct formula for fringe visibility is of view is [2014]
(a) 8 (b) 10
I max - I min
(a) V= [2012] (c) 40 (d) 50
I max + I min 31. The condition for obtaining secondary maxima
in the diffraction pattern due to single slit is
I max + I min
(b) V= (a) a sin q = nl [2014]
I max - I min
l
(b) a sin q = ( 2n - 1)
I max 2
(c) V= (c) a sin q = ( 2n - 1) l
I min
nl
(d) a sin q =
I min 2
(d) V= 32. The Fraunhoffer ‘diffraction’ pattern of a single
I max slit is formed in the focal plane of a lens of focal
27. In Young’s expt., the distance between two slits length 1 m. The width of slit is 0.3 mm. If third
d minimum is formed at a distance of 5 mm from
is and the distance between the screen and central maximum, then wavelength of light will
3
be [2015]
1 (a) 5000 Å (b) 2500 Å
the slits is 3 D. The number of fringes in m on
3 (c) 7500 Å (d) 8500 Å
the screen, formed by monochromatic light of 33. A parallel beam of monochromatic unpolarised
wavelength 3l, will be [2012] light is incident on a transparent dielectric plate
d d 1
of refractive index . The reflected beam is
(a) (b) 3
9Dl 27 D l
completely polarised. Then the angle of
d d incidence is [2015]
(c) (d) (a) 30º (b) 60º
81 D l Dl
(c) 45º (d) 75º
28. To demonstrate the phenomenon of
34. A single slit Fraunhoffer diffraction pattern is
interference, we require two sources which emit
formed with white light. For what wavelength
radiation of
of light the third secondary maximum in the
(a) nearly the same frequency [2013] diffraction pattern coincides with the second
(b) the same frequency secondary maximum in the pattern for red light
(c) different wavelengths of wavelength 6500 Å? [2016]
(d) the same frequency and having a definite (a) 4400 Å (b) 4100 Å
phase relationship (c) 4642.8 Å (d) 9100 Å
Wave Optics P-181

35. A diffraction pattern is obtained by using beam Reason : It happens due to the interference of
of red. light what will happen, if red light is light reflected from the upper surface of the thin
replced by the blue light? [2017] film. [2002]
(a) Bands disappear. 41. Assertion : At the first glance, the top surface
(b) Bands become broader and farther apart. of the Morpho butterfly's wing appears a
(c) No change will take place. beautiful blue-green. If the wind moves the
(d) Diffraction bands become narrow and colour changes.
crowded together. Reason : Different pigments in the wing reflect
TYPE B : ASSERTION REASON QUESTIONS light at different angles. [2004]
42. Assertion : A famous painting was painted by
Directions for (Qs. 36-48) : These questions consist not using brush strokes in the usual manner,
of two statements, each printed as Assertion and but rather a myriad of small colour dots. In this
Reason. While answering these questions, you are painting the colour you see at any given place
required to choose any one of the following five on the painting changes as you move away.
responses.
Reason : The angular separation of adjacent dots
(a) If both Assertion and Reason are correct and
changes with the distance from the painting.
the Reason is a correct explanation of the
Assertion. [2004]
(b) If both Assertion and Reason are correct but 43. Assertion : The clouds in sky generally appear
Reason is not a correct explanation of the to be whitish.
Assertion. Reason : Diffraction due to clouds is efficient in
(c) If the Assertion is correct but Reason is equal measure at all wavelengths. [2005]
incorrect. 44. Assertion : The resolving power of a telescope
(d) If both the Assertion and Reason are incorrect. is more if the diameter of the objective lens is
(e) If the Assertion is incorrect but the Reason is more.
correct. Reason : Objective lens of large diameter collects
36. Assertion : The colour of the green flower seen more light. [2005]
through red glass appears to be dark.
45. Assertion : Standard optical diffraction gratings
Reason : Red glass transmits only red light.
can not be used for discriminating between X-
[1997]
37. Assertion : In Young’s experiment, the fringe ray wavelengths.
width for dark fringes is different from that for Reason : The grating spacing is not of the order
white fringes. of X-ray wavelengths. [2006]
Reason : In Young’s double slit experiment the 46. Assertion : Goggles have zero power.
fringes are performed with a source of white light, Reason : Radius of curvature of both sides of
then only black and bright fringes are observed. lens is same. [2007]
[2001] 47. Assertion : A white source of light during
38. Assertion : Coloured spectrum is seen when we interference forms only white and black fringes.
look through a muslin cloth. Reason : Width of fringe is inversely
Reason : It is due to the diffraction of white proportional to the wavelength of the light used.
light on passing through fine slits [2002] [2007]
39. Assertion : When tiny circular obstacle is placed
48. Assertion : In Young's double slit experiment
in the path of light from some distance, a bright
spot is seen at the centre of the shadow of the the two slits are at distance d apart. Interference
obstacle. pattern is observed on a screen at distance D
Reason : Destructive interference occurs at the from the slits. At a point onthe screen when it is
centre of the shadow. [2002] directly opposite to one of the slits, a dark fringe
40. Assertion : Thin films such a soap bubble or a is observed. Then, the wavelength of wave is
thin layer of oil on water show beautiful colours proportional to square of distance of two slits
when illuminated by white light. Reason : For a dark fringe intensity is zero. [2008]
EBD_7100
P-182 Topicwise AIIMS Solved Papers – PHYSICS
Directions for (Qs. 49-51) : Each of these questions Reason : The fringe width is inversely
contains an Assertion followed by Reason. Read them proportional to the distance between the two
carefully and answer the question on the basis of slits. [2009]
following options. You have to select the one that 50. Assertion : In Young’s double slit experiment if
best describes the two statements. wavelength of incident monochromatic light is
(a) If both Assertion and Reason are correct and just doubled, number of bright fringe on the
Reason is the correct explanation of Assertion. screen will increase.
(b) If both Assertion and Reason are correct, but Reason : Maximum number of bright fringe on
Reason is not the correct explanation of the screen is directly proportional to the
Assertion. wavelength of light used. [2015]
(c) If Assertion is correct but Reason is incorrect. 51. Assertion : Diffraction takes place for all types
(d) If both the Assertion and Reason are incorrect. of waves mechanical or non-mechanical,
transverse or longitudinal.
49. Assertion : No interference pattern is detected
Reason : Diffraction's effect are perceptible
when two coherent sources are infinitely close
only if wavelength of wave is comparable to
to each other.
dimensions of diffracting device. [2017]

Type A : Multiple Choice Questions maximum intensity 4


=
minimum intensity 1
4 2
1. (a) Ratio of amplitudes = = [Intensity µ (amplitude)2]
1 1 6. (a) When a light passes through a material its
maximum amplitude 2 + 1 3 intensity goes on decreasing. First
= = equation given represents exponentially
minimum amplitude 2 -1 1
2
decreasing intensity. Option (b) and (c)
maximum intensity æ 3 ö 9 represents increasing intensity.
=ç ÷ = 7. (a) In case of diffraction,
minimum intensity è1ø 1
2. (d) When a thin layer of oil floats on the water 2l
Angular width of central fringe =
level, interference takes place between ray a
of light reflected from upper interface and 2 ´ 589.3 ´ 10 -9
from lower interface. Due to interference = = 2 × 589.3 × 10– 4
coloured fringes are formed in oil film. 1´ 10 -5
3. (d) Since, interference occurs both in sound = 1178.6 × 10 –5 rad.
and light waves so it will occur in all the 180
= 1178.6 × 10 – 5 × = 67563 × 10–5
forms of waves given. 11
4. (a) When we put a glass plate in the path of = 0.68°
one of the beams interfering with each other 8. (a) We know that wavelength and accelerating
then there is change of place of fringes on voltage for an electron is related to each
the screen. All the fringes including central other as follows
fringe are shifted. It does not result in 1
change of fringe width. lµ [V is potential applied]
V
I1 9 A 9 3 1
5. (c) = Þ 1 = = and resolving power µ
I2 1 A2 1 1 l
[Here A1 & A2 are amplitudes]
Maximum amplitude = A1 + A2 So, resolving power µ V
Minimum amplitude = A1 – A2 Now, if potential used is increased 4 times,
resolving power will be increased 2 times.
A1 + A 2 4 2
= = So, if resolving power earlier is R. It
A1 - A 2 2 1 becomes 2R.
Wave Optics P-183

9. (c) Red shift means shifting of colours in the higher is the accuracy in vision.
spectrum of a moving star towards the red æ 1ö
end of the spectrum. It happens when the ç Re solving power µ ÷
radiation emitting source goes away from è l ø
the earth. It is due to Doppler’s effect as 14. (c) When we introduce a thin film of some
we observe in case of sound. The apparent thickness in the path of upper beam actually
frequency decreases or wavelength we are increasing the optical path of the
increases for a receding source. upper beam. Now for the same path
10. (a) When we use white light in place of difference lower beam will shift upwards
monochromatic light then only central so that it is elongated so that path
fringe looks white and other fringes of difference remains unchanged.
different colour are observed. It is because 15. (c) The resolving power of an instrument is
central fringe for all the colours are formed lD
given by the formula, 1.22 =
at central point so it becomes white after d
mixing up. As fringes of others colours fall Here, d is aperture of the instrument, D is
at different places we see fringes of all distance of satellite from the earth. Here
colours on the central bright fringe. eye is the optical instrument.
11. (c) A 1.22 ´ 500 ´10-9
R.P = ´ 400 ´1000
B 5 ´10-3
10-2
= 1.22 ´ ´ 4 = 1.22 ´ 40 = 50 m
10-3
t
16. (b) A compact disc contains many fine circular
Soap film
lines on it so it acts as diffraction grating.
When white light falls upon it, it is
Interference occurs between two reflected diffracted as a result of which different
rays A and B. A is reflected from upper colours are diffracted at different angles.
surface and B is reflected from lower We see different colours when we look at it
surface. through different angles.
The path difference between the two 17. (b) In any type of light whether polarised or
l unpolarised, the magnitude of electric field
becomes 2mt + vector always varies periodically with time.
2
Actually the change in electric field vector
3l gives rise to periodically changing magnetic
If it is equal to then destructive
2 field.
interference occurs and we see dark fringes 18. (a) We see spectrum of colours when thin films
from above. So, the equation of oil on water is exposed to sunlight due
l 3l to phenomenon of interference.
2mt + = gives the least value of t for 19. (a) In circularly polarised light the magnitude
2 2
of electric vector remains constant with
which we see dark fringes.
respect to time.
l 6000
2µt = l Þ t = = @ 2000Å 20. (b) y l
2µ 2 ´ 1.4 ³ 1.22
D d
12. (a) If ip be the polarising angle, then yd 10 - 3 ´ 3 ´ 10 - 3
tan ip = µ = 1.5 Þ D£ =
(1.22) l (1.22) ´ 5 ´ 10 - 7
ip = 56.3°
Angle of refraction, r = 90º – ip = 30 » 5m
= 90º – 56.3º = 33.7° 6.1
13. (c) The resolving power of an instrument \ D max = 5m
depends upon the wave length of light Dl
used. The lower the wavelength of light 21. (a) b =
d
EBD_7100
P-184 Topicwise AIIMS Solved Papers – PHYSICS
if it be performed in glycerine then a x 0.3 ´10 -3 ´ 5 ´10 -3
l' =l/m or l = =
3f 3 ´1
Dl -7
b' = Since m > 1 = 5 ´ 10 m = 5000 Å.
md 33. (a) When angle of incidence i is equal to angle
So, b' < b (shrinks) of polarisation i.e, then reflected light is
22. (b) The component of the Electric vector of all completely plane-polarised whose
the electric vectors of unpolarised light in vibration is perpendicular to plane of
E incidence.
one direction only = 0
2 ( 2n + 1) l D
34. (c) x =
I 2a
and I µ E 2 Þ I = 0
2 ( 4 + 1) D
For red light, x = ´ 6500Å
l 1 2a
23. (b) sin q = \qµ
d d ( 6 + 1) D ´ lÅ
24. (b) The distance of n th fringe is given by For other light, x =
2a
nD l x is same for each.
yn =
d \ 5 ´ 6500 = 7 ´ l
As l violet < l red, 5
\ fringes will come closer. Þ l = ´ 6500 = 4642.8 Å
25. (d) Resolving power a (1/l). 7
35. (d) When red light is replaced by blue light
(R.P)1 l 2 5 the diffraction bands become narrow and
Hence, = = .
(R.P) 2 l1 4 crowded.
26. (a) Fringe visibility (V) is given by Type B : Assertion Reason Questions
Ι - Ι min
V = max 36. (b)
Ι max + Ι min . 37. (d) In Young’s experiments, fringe width of dark
¢ ¢ and white fringes are equal. If white light is
27. (c) b = l D = 3 l 3 D = 27 l D . used as source, coloured fringes are
d¢ d/3 d
observed representing bright band of
1/ 3 d different colours.
No. of fringes = = .
b 81 l D 38. (a) Porous muslin cloth has holes comparable
28. (d) to the wavelength of light falling on it. So,
29. (b) Let nth fringe of 2500 Å coincide with it get diffracted there forming fringes of
(n – 2)th fringe of 3500Å. different colours. We see it as colour
\ 3500 (n – 2) = 2500 × n spectrum.
1000 n = 7000, n = 7 39. (c) A bright spot is found at the centre of
\ 7th order fringe of 1st source will circular fringe patterns formed due to
coincide with 5th order fringe of 2nd source. diffraction of light at the edge of circular
30. (a) As b µ l obstacles. This bright spot is due to
5 constructive interference there by
\ fringe width becomes times, secondary wavelets.
4
40. (c) Interference in between two rays, one is
4
No, of fringes = ´ 10 = 8 reflected from the upper surface & second
5 from the lower surface.
31. (b)
32. (a) a sin q = nl
ax
= 3l
f
(since q is very small so
sin q » tan q » q = x / f )
Wave Optics P-185

41. (c) The Assertion is correct. When wind where D is diameter of objective and l is
moves the colour of the wing changes. The wavelength of light used.
visible colour of the wing is different from The Reason of this question is incorrect.
its original colour due to interference of 45. (a) For diffraction purpose, target should have
light. Interference occurs between incident size of the order of wavelength of light used.
ray and reflected ray. Due to wind the So, X-ray cannot be used in standard
reflectivity of the upper surface of the wing diffraction grating as target which is
changes (as it is covered by tiny hairs) spacing between the lines in different
which changes the reflected wave. The grating is large as compared with very short
resultant colour due to interference also wavelength of X-rays.
changes. This is the principle behind the 46. (b) Goggles protect from harmful UV light of
change of colour of wings of Morpho sun rays and do not correct sight defects
Butterfly. \ have zero power..
So, (c) is the answer. Both lens of goggles are identical hence
42. (a) We see two closely situated very small dots have same curvature. The Reason does not
separate when their angular separation for explain Assertion.
the viewer is more than that required by 47. (d) A white source of light during interference
Rayleigh’s criterion. will form coloured fringes.
l lD
q R = 1.22 Fringe width is given by b = i.e., it is
d d
Here, d is diameter of eye and l is
wavelength of light seen. directly proportional to wavelength.
If distance between spot is D and L be the 48. (b) When dark fringe is obtained at the point
distance of observer from the painting then, opposite to one of the slits then
D l D S1 P
q= ; So, 1.22 =
L d L
Dd d O
L=
1.22l
If for red light having greater l, the value S2 D
of L will be smaller and for violet colour, S1P = D
this distance is larger. If we move away from 1/ 2
the painting, adjacent red dots become æ d2 ö
indistinguishable before adjacent blue dots and S2 P = D2 + d 2 = D ç1 + 2 ÷
è D ø
do. So, to make two dots distinguishable
we shall have to give distance of the æ d2 ö
observer (L) a minimum value. At a greater = D ç1 + ÷ ( By binomial theorem)
è 2D2 ø
distance colours of adjacent dots blend
together. In this way we can have different Path difference = S2 P - S1P
view of dots by looking at it from different æ d2 ö 2
distance (or changing L). = D ç1 + 2 ÷ - D = d = l
So, both assertion and reason are correct. è 2D ø 2D 2
43. (c) The clouds in the sky appear white as the d2
size of cloud particle is not small enough or l= Þ l µ d2
D
to permit diffraction. So all the wavelength Now, intensity of a dark fringe is zero.
gets reflected and it appears white. 49. (a) When d is negligibly small, fringe width b
44. (a) Th e resolvin g power of a telescope which is proportional to 1/d may become
increases as diameter of objective lens too large. Even a single fringe may occupy
increases. the whole screen. Hence the pattern cannot
D be detected.
Resolving Power = 50. (d) 51. (b)
1.22 l
EBD_7100
P-186 Topicwise AIIMS Solved Papers – PHYSICS

25 Dual Nature of
Radiation and Matter
TYPE A : MULTIPLE CHOICE QUESTIONS 7. Light of wavelength 4000 Åis incident on a metal
plate whose work function is 2eV. What is
1. X-ray will not show the phenomenon of :[1997] maximum kinetic energy of emitted
(a) interference photoelectron? [2002]
(b) deflection by electric field (a) 0.5 eV (b) 1.1 eV
(c) diffraction (c) 2.0 eV (d) 1.5 eV
(d) superposition 8. A laser beam is used for carrying out surgery
2. Which one of the following is not dependent on because it [2003]
the intensity of incident photon in a photo- (a) is highly monochromatic
electric experiment ? [1998] (b) is highly coherent
(a) work function of the surface (c) is highly directional
(b) kinetic energy of photo-electron (d) can be sharply focussed
(c) stopping potential 9. A proton is about 1840 times heavier than an
(d) amount of photo-electric current electron. When it is accelerated by a potential
3. The kinetic energy of an electron, which is difference of 1 kV, its kinetic energy will be
accelerated in the potential difference of 100 V, [2003]
is : [1998] (a) 1840 keV (b) 1/1840 keV
(a) 1.6 × 10–10 J (b) 1.6 × 108 J (c) 1keV (d) 920 V
(c) 1.6 × 10–17 J (d) 1.6 × 10–18 J 10. If an electron and a photon propagate in the
4. When cathode rays strike a metal target of high form of waves having the same wavelength, it
melting point with a very high velocity then which implies that they have the same [2003]
of the following are produced ? [1999] (a) energy
(a) g-waves (b) Ultrasonic (b) momentum
(c) X-rays (d) a-rays (c) velocity
5. Particle nature and wave nature of (d) angular momentum
electromagnetic waves and electrons can be 11. Characteristic X-rays are produced due to [2003]
represented by : [2000] (a) transfer of momentum in collision of
(a) photoelectricity and electron microscopy electrons with target atoms
(b) light is refracted and diffracted (b) transition of electrons from higher to lower
(c) X-rays is diffracted, reflected by thick metal electronic orbits of an atom
sheet (c) heating of the target
(d) electrons have small mass, deflected by the (d) transfer of energy in collision of electrons
metal sheet. with atoms in the target.
6. The surface of zone material is radiated in turn 12. A photon of energy 4 eV is incident on a metal
by waves of l = 350 nm and 540 nm respectively. surface whose work function is 2eV. The minimum
The ratio of the stopping potential in the two reverse potential to be applied for stopping the
cases is 2 : 1. The work function of the material is emission of electrons is [2004]
(a) 4.20 eV (b) 0.15 eV [2000] (a) 2V (b) 4V
(c) 2.10 eV (d) 1.05 eV (c) 6V (d) 8V
Dual Nature of Radiation and Matter P-187

13. Solid targets of different elements are bombarded 18. The force on a hemisphere of radius 1 cm if a
by highly energetic electron beams. The parallel beam of monochromatic light of
frequency (f) of the characteristic X-rays emitted wavelength 500 nm. falls on it with an intensity
from different targets varies with atomic number of 0.5 W/cm2, striking the curved surface in a
Z as [2005, 2017] direction which is perpendicular to the flat face
of the hemisphere is (assume the collisions to
(a) fµ Z (b) f µ Z2 be perfectly inelastic) [2009]
(c) f µ Z (d) f µ Z3/2 (a) 5.2 × 10–13 N (b) 5.2 × 10–12 N
14. Hard X-rays for the study of fractures in bones (c) 5.22 × 10–9 N (d) zero
should have a minimum wavelength of 1011 m. 19. The energy of a photon of light with wavelength
The accelerating voltage for electrons in X-ray 5000 Å is approximately 2.5 eV. This way the
machine should be [2006] energy of an X-ray photon with wavelength 1 Å
(a) < 124.2 kV would be [2010]
(b) > 124.2 kV 2.5
(a) eV (b) 2.5 × 5000 eV
(c) Between 60 kV and 70 kV (5000) 2
(d) = 100 kV 2.5 2.5
15. In photoelectric effect, the electrons are ejected (c) eV (d) eV
2 5000
(5000)
from metals if the incident light has a certain
20. A 15.0 eV photon collides with and ionizes a
minimum [2006]
hydrogen atom. If the atom was originally in the
(a) Wavelength ground state (ionization potential =13.6 eV), what
(b) Frequency . is the kinetic energy of the ejected electron?
(c) Amplitude [2014]
(d) Angle of incidence (a) 1.4 eV (b) 13.6 eV
16. In a photoemissive cell with executing (c) 15.0 eV (d) 28.6 eV
wavelength l, the fastest electron has speed v. 21. The anode voltage of a photocell is kept fixed.
3l The wavelength l of the light falling on the
If the exciting wavelength is changed to , cathode is gradually changed. The plate current
4
the speed of the fastest emitted electron will be I of the photocell varies as follows [2017]
(a) v (3/4)1/2 [2008] I I
1/ 2
æ 4ö
(b) vç ÷
è 3ø (a) (b)
1/ 2
æ 4ö O l O l
(c) less than v ç ÷
è 3ø I I
1/ 2
æ 4ö
(d) greater than v ç ÷
è 3ø (c) (d)
17. The stopping potential doubles when the
O l O l
frequency of the incident light changes from n
3n TYPE B : ASSERTION REASON QUESTIONS
to . Then the work function of the metal must
2 Directions for (Qs. 22-28) : These questions consist
be [2009]
of two statements, each printed as Assertion and
hn Reason. While answering these questions, you are
(a) (b) hn
2 required to choose any one of the following five
(c) 2hn (d) none of the above responses.
EBD_7100
P-188 Topicwise AIIMS Solved Papers – PHYSICS
(a) If both Assertion and Reason are correct and Directions for (Qs. 29-34) : Each of these questions
the Reason is a correct explanation of the contains an Assertion followed by Reason. Read them
Assertion. carefully and answer the question on the basis of
(b) If both Assertion and Reason are correct but following options. You have to select the one that
Reason is not a correct explanation of the best describes the two statements.
Assertion. (a) If both Assertion and Reason are correct and
(c) If the Assertion is correct but Reason is incorrect. Reason is the correct explanation of Assertion.
(d) If both the Assertion and Reason are incorrect. (b) If both Assertion and Reason are correct, but
(e) If the Assertion is incorrect but the Reason is Reason is not the correct explanation of
correct. Assertion.
22. Assertion : Photosensitivity of a metal is high if (c) If Assertion is correct but Reason is incorrect.
its work function is small. (d) If both the Assertion and Reason are incorrect.
Reason : Work function = hf0 where f0 is the 29. Assertion : Soft and hard X-rays differ in
threshold frequency. [1997] frequency as well as velocity.
Reason : The penetrating power of hard X–rays
23. Assertion: Kinetic energy of photo electrons is more than the penetrating power of soft
emitted by a photosensitive surface depends
X–rays. [2010]
upon the intensity of incident photon.
30. Assertion : In photoemissive cell inert gas is
Reason: The ejection of electrons from metallic used.
surface is possible with frequency of incident
Reason : Inert gas in the photoemissive cell gives
photon below the threshold frequency. [1999]
greater current. [2010]
24. Assertion : If the speed of charged particle
31. Assertion : When ultraviolet light is incident on
increases both the mass as well as charge
a photocell, its stopping potential is V0 and the
increases.
maximum kinetic energy of the photoelectrons
Reason : If m0 = rest mass and m be mass at is Kmax .When the ultraviolet light is replaced
velocity v then by X-rays, both V0 and Kmax increase.
m0 Reason : Photoelectrons are emitted with speeds
m=
ranging from zero to a maximum value because
v2
1- of the range of frequencies present in the incident
c2 light. [2013]
where c = speed of light. [2000] 32. Assertion : Photoelectric saturation current
25. Assertion : Mass of moving photon varies increases with the increase in frequency of
inversely as the wavelength. incident light.
Reason : Energy of the particle [2000] Reason : Energy of incident photons increases
= mass × (speed of light)2 with increase in frequency and as a result
26. Assertion : Photoelectric effect demonstrates photoelectric current increases. [2015]
the wave nature of light. 33. Assertion : In process of photoelectric emission,
Reason : The number of photoelectrons is all emitted electrons do not have same kinetic
proportional to the frequency of light. [2004] energy.
27. Assertion : The energy (E) and momentum (p) Reason : If radiation falling on photosensitive
of a photon are related by p = E/c. surface of a metal consists of differ ent
Reason : The photon behaves like a particle. wavelength then energy acquired by electrons
[2005] absorbing photons of different wavelengths
28. Assertion : The photoelectrons produced by a shall be different. [2015]
monochromatic light beam incident on a metal 34. Assertion : The phtoelectrons produced by a
surface, have a spread in their kinetic energies. monochromatic light beam incident on a metal
surface have a spread in their kinetic energies.
Reason : The work function of the metal varies Reason : The work function of the metal is its
as a function of depth from the surface. [2006] characteristics property. [2017]
Dual Nature of Radiation and Matter P-189

Type A : Multiple Choice Questions æ c ö


hç = f + 2e.V
1. (b) X-rays are electromagnetic wave so it will è 350 ´ 10 -9 ÷ø .......(i)
remain undeflected in electric field. æ c ö
2. (b) The intensity of incident photon hç = f + eV
è 540 ´ 10 -9 ÷ø
......(ii)
determines the no. of electrons being ejected
from the surface. The kinetic energy of the 540 f + 2ev
=
photoelectron is deter mined by the 350 f + eV
frequency of incident photon. Þ 540 f + 540 eV = 350 f + 700 eV
1.227 hc Þ 190 f = 160eV
3. (c) l = and E =
V l
16
h = 6.6 ´ 10–34JS, C = 3 ´ 108m/s f= eV
19
and V = 100 volt.
4. (c) When electrons strike a metal target of high 6.6 ´ 10 -34 ´ 3 ´ 108 19 ´ 2f 27
=f+ = f
-9 16 8
melting point with high velocity, it knocks 350 ´ 10
out inner electrons of the atoms of the
8 6.6 ´ 3 ´ 10 -18
target material. To fill up this vacancy, f= ´
electrons from higher energy level make 27 35
transition to lower level resulting in 8 6.6 ´ 3 ´10-18
emission of radiation. If target material has = ´ eV = 1.05 eV
27 35 ´1.6 ´ 10-19
very high atomic number then the emitted 7. (b) Applying Einstein’s equation,
radiation is X-ray.
1
hn = f + mv2 = f + K.E
2
3 ´ 108
6.6 ´ 10 -34 ´
4000 ´ 10 -10

X – rays = 2 ´ 1.6 ´ 10 -19 + K.E


6.6 ´ 3
´10 -19 = 3.2 ´ 10-19 + K.E
4
5. (a) In photo electricity, photon is acting as
4.95 ´10-19 = 3.2 ´10-19 + K.E
particles which knocks out electron from
the surface of the metal. In electron K.E = (4.95 - 3.2) ´10 -19
microscopy electron behaves as waves so
we can take image of objects with the help = 1.75 ´ 10 -19 J.
of ray of electrons. 1.75 ´ 10 -19
= eV = 1.1 eV
6. (d) Let the work function be f. 1.6 ´ 10 -19
Einstein’s equation in photo – electricity, 8. (d) A laser beam has great power to be
1 focussed so energy density of radiation is
hn = f + mv 2
2 greatly enhanced which may have cutting
c 1 effect.
h = f + mv 2 = f + eV 9. (c) When a charge q is accelerated by a
l 2
potential difference of V, the energy
1
[ eV = mv2 , V is stopping potential] imparted to it is qV. It does not depend upon
2
EBD_7100
P-190 Topicwise AIIMS Solved Papers – PHYSICS
the mass. In the given problem potential 2hc (l 0 - l)
diff = 1kV. Charge on the proton = charge v max =
on electron = 1.6 × 10–19 coulomb. The m ll 0
energy imparted = 1 keV. æ 3l ö
[1eV is equal to energy acquired by an If wavelength is changed to çè ÷ø
4
electron when it is accelerated under
2hc (l 0 - 3l / 4)
potential of 1eV] then v 'max =
10. (b) A photon and electron will have same m l0 (3l / 4)
momentum if their wavelength are equal. v 'max ( l 0 - 3l / 4) ll 0
h = ´
For electron, momentum = . v max 3 l0 - l
l ll
4 0
For photon, momentum
4 (l 0 - 3l / 4)
mc 2 E hn h =
= = = = 3 l0 - l
c c c l
11. (b) When we bombard electrons on a target 4 l 0 - 3l / 4
consisting of heavier atoms, the electron i.e., v 'max = 3
´v´
l0 - l
of inner orbitals of target atoms get knocked
out. So to fill their position, other electrons 4
(Q v max = v) i.e., v 'max > v
from the higher energy orbitals make 3
transition to lower energy orbitals. Its 3
results is emission of X-rays. 17. (a) hn = f + Vs and h n = f + 2Vs
2
12. (a) From Einstein’s equation of photoelectric hn
effect Þ f=
2
1 h
hn = f + mv 2 18. (c) p = of each photon
2 l
hn = f + Ve [V is stopping potential]
6.63 ´ 10 -34
4 eV = 2 eV + Ve = = 1.33 ´ 10-27 kg-m/s
-9
Ve = 2 e volt 500 ´ 10
and no. of photons
V × e = 2 × e × 1 volt
0.5 0.5l
V (Stopping potential) = 2 Volt = / cm 2 = / cm 2
13. (b) From Mosley law, hn hc
f = a (Z – b)2 0.5 ´ 500
= / cm 2
f µ Z2 1240 ´ 1.6 ´10-19
hc hc 6.6 ´ 10 -34 ´ 3 ´ 108 = 1.25 × 1018 photons /cm2
14. (a) = ev Þ v = =
l el 1.6 ´ 10 -19 ´ 10 -11 \ force = 1.25 × 108 × 1.33 × 10–27 × p × 12
33 ´ 3 = 5.22 × 10–9 N
= ´ 10 4 = 12 .375 ´ 10 4 19. (b) Energy of photon
8
= 124 × 103V = 124 kV hc
E=
For minimum wavelength energy is 124 kV l
\ The accelerating voltage should be hc
< 124 kV Þ 2.5 eV = (as l = 5000Å)
15. (b) The incident wave must have a certain 5000 ´ 10 -10
minimum frequency. This is known as Þ hc = 2.5 × 5 × 10–7 eV
threshold frequency. For X-ray photon, c = same, h = constant
16. (d) We have from Einsteins photo electric hc
Þ E=
equation. l x - ray
Dual Nature of Radiation and Matter P-191

27. (a) Photon when behaves as a particle carries


2.5 ´ 5 ´ 10-7
= eV E
1 ´ 10-10 momentum equal to .
c
= 2.5 × 5000 eV
28. (a) Electrons being emitted as photoelectrons
20. (a) Conservation of energy requires that the have different velocities. Actually all the
15.0 eV photon energy first provides the electrons do not occupy the same level of
ionization energy to unbind the electron, energy but they occupy continuous band
and then allows any excess energy to and levels. So, electrons being knocked off
become the electron’s kinetic energy. The from different levels come out with different
kinetic energy in this case is 15.0 eV – 13.6 energies.Work function is the energy
eV = 1.4 eV. required to pull the electron out of metal
21. (d) As l is increased, there will be a value of l surface. Naturally electrons on the surface
above which photoelectrons will be cease will require less energy to be pulled out
to come out so photocurrent will become hence will have lesser work function as
zero. Hence (d) is correct answer. compared with those deep inside the metal.
So, assertion and reason are correct and
Type B : Assertion Reason Questions
reason correctly explains the assertion.
22. (b) The photosensitivity of a metal is high 29. (d) Soft and h ard X–rays differ only in
when its work function is small. Work frequency. Soft X–rays have low frequency
function of a metal depends not on the as compared to hard X–rays. But both types
threshold frequency but on the nature of of X–ray travel with speed of light.
the metal. 30. (a) The photoemissive cell contain two
23. (d) Intensity of incident photon decides the electrodes are enclosed in a glass bulb
number of electrons ejected and not the which may be evacuated or contain an inert
gas at low pressure. An inert gas in the cell
kinetic energy.
gives greater current but causes a time lag
Ejection of electron from metallic surface is in the response of the cell to very rapid
possible only when frequency of incident changes of radiation which may make it
photon is more than threshold frequency. unsuitable for some purpose.
24. (e) When speed of particle increases, the mass 31. (c) We know that
increases as eV0 = Kmax = hn – f
where, f is the work function .
m0
m= charge does not alter Hence, as n increases (note that frequency
v2 of X-rays is greater than that of U.V. rays),
1-
c2 both V0 and Kmax increase. So assertion is
h correct.
25. (b) We know that for photon , mv = 32. (d) Photoelectric saturation current is
l
independent of frequency. It only depends
mass varies inversely as the wavelength.
on intensity of light.
For particle E = mc2, E is energy of particle
33. (a) Both statement I and II are true; but even it
if its mass is converted into energy. Both
radiation of single wavelength is incident
are uncorrelated.
on photosensitive surface, electrons of
26. (d) Photoelectric effect demonstrate the different KE will be emitted.
particle nature of light. 34. (b) The kinetic energy of emitted
The number of ph otoelectr ons is photoelectrons varies from zero to a
proportional to the intensity of light. maximum value. Work function depends on
So, (d) is the answer. metal used.
EBD_7100
P-192 Topicwise AIIMS Solved Papers – PHYSICS

26 Atoms

TYPE A : MULTIPLE CHOICE QUESTIONS 7. The ground state energy of hydrogen atom is
– 13.6 eV. What is the potential energy of the
1. If the electron in hydrogen orbit jumps from third electron in this state ? [2005]
orbit to second orbit, the wavelength of the (a) 0 eV (b) – 27.2 eV
emitted radiation is given by : [1997]
(c) 1 eV (d) 2 eV
R 5 8. In the following diagram, which particle has
(a) l= (b) l=
6 R highest e/m value? [2007]
36 5R A
(c) l= (d) l =
5R 36
2. The radius of hydrogen atom in the first excited B
level is : [1998] C
(a) twice (b) four times D
(c) same (d) half (a) A (b) B
3. In Bohr’s theory, relation between principal (c) C (d) D
quantum number n and radius of orbit r is:[1999] 9. What is the energy of He+ electron in first orbit?
1 (a) 40.8 eV (b) –27.2 eV [2007]
(a) rµ (b) rµn
n2 (c) –54.4 eV (d) –13.6 eV
10. What is the energy of photon whose wavelength
1
(c) rµ (d) r µ n 2 is 6840 Å? [2007]
n (a) 1.81 eV (b) 3.6 eV
4. For an electron in the second orbit of hydrogen, (c) –13.6 eV (d) 12.1 eV
the moment of momentum as per Bohr’s model
11. The ratio of the energy of the photon emitted by
is [2000]
the ka line to that of the kb line is [2009]
h 2h (a) greater than 1 (b) Less than 1
(a) (b)
p p (c) 1 (d) indeterminate
h 12. The angular momentum of an electron in the 2nd
(c) (d) 2ph excited state of a Helium ion (He+) is [2009]
2p
5. The speed of an electron having a wavelength h 2h
of 10–10m is [2002] (a) (b)
6 6
2p 2p
(a) 4.24 × 10 m/s (b) 5.25 × 10 m/s
(c) 6.25 × 106 m/s (d) 7.25 × 106 m/s 3h 4h
(c) (d)
6. We wish to seen inside an atom. Assuming the 2p 2p
atom to have a diameter of 100 pm, this means 13. What is the wavelength of the least energetic
that one must be able to resolve a width of say photon emitted in the Lyman series of the
10 pm. If an electron microscope is used, the hydrogen atom spectrum ? [2011]
minimum electron energy required is about (a) 150 nm (b) 122 nm
(a) 1.5 keV (b) 15 keV [2004] (c) 102 nm (d) 82 nm
(c) 150 keV (d) 1.5 MeV
Atoms P-193

14. The energy of electron in the nth orbit of 18. As an electron makes a transition from an excited
-13.6 state to the ground state of a hydrogen - like
hydrogen atom is expressed as E n = 2 eV.
n atom/ion [2017]
The shortest and longest wavelength of Lyman
(a) kinetic energy decreases, potential energy
series will be [2013]
increases but total energy remains same
(a) 910 Å, 1213 Å (b) 5463 Å, 7858 Å
(b) kinetic energy and total energy decrease
(c) 1315 Å, 1530 Å (d) None of these but potential energy increases
15. Which of the following statements are true
(c) its kinetic energy increases but potential
regarding Bohr’s model of hydrogen atom? [2015]
energy and total energy decrease
(I) Orbiting speed of electron decreases as it
(d) kinetic energy, potential energy and total
shifts to discrete orbits away from the
energy decrease
nucleus
(II) Radii of allowed orbits of electron are TYPE B : ASSERTION REASON QUESTIONS
proportional to the principal quantum
number Directions for (Qs. 19-21) : These questions consist
of two statements, each printed as Assertion and
(III) Frequency with which electrons orbit
Reason. While answering these questions, you are
around the nucleus in discrete orbits is
inversely proportional to the cube of required to choose any one of the following five
principal quantum number responses.
(IV) Binding force with which the electron is (a) If both Assertion and Reason are correct and
bound to the nucleus increases as it shifts the Reason is a correct explanation of the
to outer orbits Assertion.
Select correct answer using the codes given (b) If both Assertion and Reason are correct but
below. Reason is not a correct explanation of the
Codes : Assertion.
(a) I and II (b) II and IV (c) If the Assertion is correct but Reason is
incorrect.
(c) I, II and III (d) II, III and IV
(d) If both the Assertion and Reason are incorrect.
16. The wavelength of the first line of Lyman series
for hydrogen atom is equal to that of the second (e) If the Assertion is incorrect but the Reason is
line of Balmer series for a hydrogen like ion. The correct.
atomic number Z of hydrogen like ion is[2016] 19. Assertion: The specific charge of positive rays
(a) 3 (b) 4 is not constant.
(c) 1 (d) 2 Reason: The mass of ions varies with speed.
17. Which one did Rutherford consider to be [1999]
supported by the results of experiments in which 20. Assertion : Bohr had to postulate that the
a-particles were scattered by gold foil? [2017] electrons in stationary orbits around the nucleus
(a) The nucleus of an atom is held together by do not radiate.
forces which are much stronger than
Reason : According to classical physics all
electrical or gravitational forces.
moving electrons radiate. [2003]
(b) The force of repulsion between an atomic
21. Assertion : Balmer series lies inthe visible region
nucleus and an a-particle varies with
distance according to inverse square law. of the electromagnetic spectrum.
(c) a-particles are nuclei of Helium atoms. 1 é1 1 ù
Reason : = R ê 2 - 2 ú , where n = 3, 4, 5
(d) Atoms can exist with a series of discrete l ë2 n û
energy levels [2008]
EBD_7100
P-194 Topicwise AIIMS Solved Papers – PHYSICS
Directions for (Qs. 22-25) : Each of these questions 23. Assertion : Between any two given energy
contains an Assertion followed by Reason. Read them levels, the number of absorption transitions is
carefully and answer the question on the basis of always less than the number of emission
following options. You have to select the one that transitions.
best describes the two statements. Reason : Absorption transitions start from the
lowest energy level only and may end at any
(a) If both Assertion and Reason are correct and
higher energy level. But emission transitions may
Reason is the correct explanation of Assertion. start from any higher energy level and end at
(b) If both Assertion and Reason are correct, but any energy level below it. [2015]
Reason is not the correct explanation of 24. Assertion : In Lyman series, the ratio of minimum
Assertion. 3
and maximum wavelength is .
(c) If Assertion is correct but Reason is incorrect. 4
(d) If both the Assertion and Reason are incorrect. Reason : Lyman series constitute spectral lines
22. Assertion : In Lyman series, the ratio of minimum corresponding to transition from higher energy
to ground state of hydrogen atom. [2016]
3
and maximum wavelength is . 25. Assertion : Bohr had to postulate that the
4
electrons in stationary orbits around the nucleus
Reason : Lyman series constitute spectral lines do not radiate.
corresponding to transition from higher energy Reason : According to classical physics all
to ground state of hydrogen atom. [2011] moving electrons radiate. [2017]
Atoms P-195

Type A : Multiple Choice Questions 1


\ Energy of electron = mv2
1. (c) We know that 2

1 æ 1 1 ö
= Rç - ÷
1 9.1 ´ 10 -31 ´ (7.25 ´ 107 )2
l çn 2 n 2 ÷
è 1 2 ø = ´ = 15 keV
2 1.6 ´ 10-19
1 æ 1 1 ö 7. (b) In case of hydrogen atom
= Rç - ÷ Þ æ 1 1ö
ç Rç - ÷ If –E = ground state energy (Total)
l è2
2
32 ÷ø è 4 9ø E is kinetic energy then –2E is potential
energy.
1 æ9-4ö 5R 36 Here E = 13.6 eV Þ 2E = 27.2 eV
=ç ÷R = Þ l=
l è 36 ø 36 5R So, potential energy = – 27.2 eV
2. (b) Radius of H-atom µ n 2 8. (d) The deflection of a particle along y-axis is
So for excitation from n = 1 to n = 2, radius
Ee x 2
becomes 4 times. electricfield is given by y =
3. (d) Radius of Bohr’s orbit µ n2, where n is 2mv 2
principal quantum no. e
4. (a) The moment of momentum is also known Þ yµ for all other values to be
m
as angular momentum of electron.
We know from Bohr’s theory that in an orbit constant. Since y is maximum for D, hence
it has highest e/m ratio.
h
angular momentum = n. ,13.6Z2
2p 9. (c) E = eV, for He+, Z =2, n = 1 (first
For second orbit n = 2 n2
orbit)
h h ,13.6 ´ 2
So, angular momentum = 2 ´ = is the
2p p \E= = – 54.4 eV
12
answer.
10. (a) Energy of photon
5. (d) We know that De-Broglie wavelength is
h h hc 6.6 ´ 10 -34 ´ 3 ´ 108
mv = Þl= E= =
l mv l 6840 ´ 10 -10

6.6 ´ 10 -34 6.6 ´ 3´10,26


10 -10
= = J
9.1´ 10 -31 ´ v
; 6.84 ´10,7
6.6 ´ 3´10,26
6.6 ´ 10-34 = eV = 1.8 eV
v= -41
= 7.25 ´ 10 6 6.84´10,7 ´1.6 ´10,19
9.1 ´ 10
æ 1 1ö
6. (b) From the de-Broglie equation, 11. (b) DE = E 0 (Z - 1)2 ç - ÷
2 2
h h è n1 n 2 ø
l= =
p mv for k a , n1 = 1, n2 = 2
where l is wavelength, h is plank's constant for kb , n1 = 1, n2 = 3
and p is momentum and v is velocity
so energy of the photon corresponding to
h 6.6 ´ 10 -34
v= = = 7.25 ´ 107 m / s k a line is less than that of kb line.
ml 9.1 ´ 10 -31 ´ 10 -11
EBD_7100
P-196 Topicwise AIIMS Solved Papers – PHYSICS
12. (c) Angular momentum in the ground state 20. (b) According to classical physics all moving
h electrons around the nucleus will radiate
= because an accelerated electron in electric
2p
Angular momentum in the first excited state field will radiate e.m. wave. So Reason is
correct. But Reason does not explain the
2h Assertion.
=
2p Bohr's postulates that the electron in
Angular momentum in the second excited stationary orbits do not radiate. His
3h postulate was based not out of compulsion
state =
2p but on the quantum theory. So Assertion
13. (b) The least energy is corresponding to is also correct but Reason does not explain
longest wavelength, which is l = 1216Å. Assertion.
21. (b) Wavelength is Balmer series is,
1 é 1 1 ù 4
14. (a) =Rê 2 - 2ú
Þ l max = » 1213Å
l max ë (1) (2) û 3R 1 é1 1 ù 1 é1 1ù
=Rê 2 - 2ú ; =Rê 2 - 2ú
1 é 1 1ù 1
l ë2 n û l max ë2 3 û
and = R ê 2 - ú Þ l min = » 910Å.
l min ë (1) ¥ û R 36 36
or l max = 5R = = 6563Å
15. (a) Orbital speed varies inversely as the radius 5 ´ 1.097 ´ 107
of the orbit.
é1 1 ù
1 1/ l min = R ê 2 - 2 ú
vµ ë2 ¥ û
n
16. (d) For first line of Lyman series of hydrogen 4 4
or l min = = = 3646Å
R 1.097 ´ 10 7
hc æ1 1 ö
= Rhc ç - ÷ So it lies in the visible region.
l1 è12
22 ø 22. (b)
For second line of Balmer series of 23. (a) Absorption transition
hydrogen like ion
C
hc æ 1 1 ö
= Z2 Rhc ç - ÷
l2 è2 2
42 ø B
By question, l1 = l2
æ1 1 ö 2 æ1 1 ö A
Þ ç - ÷ = Z ç - ÷ or Z = 2
è1 2 ø è 4 16 ø
17. (b) Two possibilities in absorption transition.
ze 2 k ze 2
18. (c) U = –K ; T.E = –
r 2 r
2
k ze
K.E = . Here r decreases
2 r
Type B : Assertion Reason Questions
e Three possibilities in emission transition.
19. (a) The specific charge of positive ray is
m Therefore, absorption transition < emission.
not constant because for differen t 24. (b)
velocities the rest mass of particles change 25. (b) Bohr postulated that electrons in stationary
e orbits around the nucleus do not radiate.
i.e. mass of ions varies with speed. So This is the one of Bohr ’s postulate.
m
will differ. Hence, Reason and Assertion According to this the moving electrons
both are correct and Reason explains radiates only when they go from one orbit
Assertion. to the next lower orbit.
27 Nuclei

TYPE A : MULTIPLE CHOICE QUESTIONS 9. Consider the following nuclear reaction


X200 ® A110 + B90 + Energy
1. In the nuclear reaction, there is a conservation of:
If the binding energy per nucleon for X, A and B
(a) momentum (b) mass [1997]
are 7.4 MeV, and 8.2 MeV and 8.2 MeV
(c) energy (d) all of these
respectively, the energy released will be :[2000]
2. If the radioactive decay constant of radium is (a) 90 MeV (b) 110 MeV
1.07 × 10–4 per year. Then its half life period (c) 200 MeV (d) 160 MeV
approximately is equal to : [1998]
10. In each fission of 92U235 releases 200 MeV, how
(a) 5000 years (b) 6500 years many fissions must occur per second to produce
(c) 7000 years (d) 8900 years power of 1 kW ? [2000]
226
3. In 88Ra nucleus there are : [1998] (a) 1.25 × 1018 (b) 3.125 × 1013
(a) 226 protons and 88 electrons (c) 3.2 × 1018 (d) 1.25 × 1013
(b) 138 protons and 88 neutrons 11. The function of heavy water in a nuclear reactor to
(c) 226 neutrons and 138 electrons (a) slow down the neutrons [2001]
(b) increase the neutrons
(d) 138 neutrons and 88 protons
(c) stop the electrons
4. The activity of radioactive sample is measured
(d) none of these
as 9750 counts per minute at t = 0 and as 975
12. Which one of the following has the highest
counts per minute at t = 5 minutes, the decay
neutrons ratio ? [2001]
constant is approximately : [1998] 235 16
(a) 92U (b) 8O
(a) 0.922 per minute (b) 0.270 per minute
(c) 4 56
(c) 0.461 per minute (d) 0.39 per minute 2He (d) 26Fe
5. The activity of a radioactive sample is 1.6 curie 13. When radioactive substance emits an a-particle,
and its half life is 2.5 days. Then activity after 10 then its position in the periodic table is lowered by
days will be : [1999] [2001]
(a) 0.16 curie (b) 0.8 curie (a) two places (b) three places
(c) 0.1 curie (d) 0.4 curie (c) five places (c) one place
6. Which one of the following is used as a 14. In an atom bomb, the energy is released because
moderator in nuclear reaction ? [1999] of the : [2001]
(a) Uranium (b) Heavy water (a) chain reaction of neutrons and 92U238
(c) Cadmium (d) Plutonium
(b) chain reaction of neutrons and 92U235
7. The reaction responsible for the production of
light energy from the sun will be: [1999] (c) chain reaction of neutrons and 92U236
(a) fission (b) fusion (d) chain reaction of neutrons and 92U240
(c) nuclear (d) none of these 15. A radioactive substance decays to 1/16th of its
8. Half life of a substance is 20 minutes, then the initial activity in 40 days. The half-life of the
time between 33 % decay and 67 % decay will be radioactive substance expressed in days is
(a) 20 minute (b) 40 minute [2000] (a) 2.5 (b) 5 [2003]
(c) 50 minute (d) 10 minute (c) 10 (d) 20
EBD_7100
P-198 Topicwise AIIMS Solved Papers – PHYSICS
16. Nuclear fusion is possible [2003] 2v 4v
(a) only between light nuclei (a) (b)
A+4 A+4
(b) only between heavy nuclei 4v 2v
(c) between both light and heavy nuclei (c) (d)
A-4 A-4
(d) only between nuclei which are stable 22. When an electron-positron pair annihilates, the
against b-decay energy released is about [2004]
17. Radioactive nuclei that are injected into a potient (a) 0.8 × 10–13 J (b) 1.6 × 10–13 J
collected at certain sites within its body, (c) 3.2 × 10–13 J (d) 4.8 × 10–13 J
undergoing radioactive decay and emitting 23. A radioactive material has half-life of 10 days.
electromagnetic radiation. These radiations can What fraction of the material would remain after
then be recorded by a detector. This procedure 30 days ? [2005]
provides an important diagnostic tools called (a) 0.5 (b) 0.25
(a) Gamma camera [2003] (c) 0.125 (d) 0.33
(b) CAT can 24. The operation of a nuclear reactor is said to be
(c) Radiotracer technique critical, if the multiplication factor (K) has a value
(d) Gamma ray spectroscopy (a) 1 (b) 1.5 [2006]
18. In a material medium, when a positron meets an (c) 2.1 (d) 2.5
25. 238U has 92 protons and 238 nucleons. It decays
electron both the particles annihilate leading to
the emission of two gamma ray photons. This by emitting an Alpha particle and becomes
process forms the basis of an important (a) 234 (b) 234 [2006]
92 U 90Th
diagnostic procedure called [2003] 237
(c) 235 92 U
(d) 93 Np
(a) MRI (b) PET
26. The fossil bone has a 14C: 12C ratio, which is
(c) CAT (d) SPECT
æ 1ö
19. The dependence of binding energy per nucleon, ç ÷ of that in a living animal bone. If the half-
BN, on the mass number A, is represented by è 16 ø
life time of l4C is 5730 years, then the age of the
[2004] fossil bone is [2006]
(a) 11460 years (b) 17190 years
BN BN
(c) 22920 years (d) 45840 years
27. Which one of the following is a possible nuclear
(a) (b) reaction [2006]
10 4
A=56 A A=124 A (a) ¾®137 N +11 H
5 B+ 2 He ¾
23 1
(b) ¾® 20
11 Na +1 H ¾
4
10 Ne+ 2 He
239 -
BN BN (c) ¾® 239
11 Np ¾ 94 Pu + b +v
(c) (d) ¾®126 C + b - + v
(d) 117 N +11H ¾
A=96 A A=96 A 28. If Alpha, Beta and Gamma rays carry same
momentum, which has the longest wavelength
20. Carbon dating is best suited for determining the (a) Alpha rays [2006]
age of fossils if their age in years is of the order (b) Beta rays
of [2004] (c) Gamma rays
(a) 103 (b) 104 (d) None, all have some wavelength
(c) 105 (d) 106 29. What is the amount of energy released by
21. In nucleus of mass number A, originally at rest, deuterium and tritium fusion? [2007]
emits an a-particle with speed v. The daughter (a) 60.6 eV (b) 123.6 eV
nucleus recoils with a speed : [2004] (c) 17.6 eV (d) 28.3 eV
Nuclei P-199

30. Calculate power output of 92 235


U reactor, if it 37. Radioactive element decays to form a stable
nuclide, then the rate of decay of reactant is
takes 30 days to use up 2 kg of fuel, and if each
[2012]
fission gives 185 MeV of useable energy.
Avogadro's number = 6 × 1023 /mol? [2007] N N
(a) 56.3 MW (b) 60.3 MW
(c) 58.3 MW (d) 54.3 MW (a) (b)
66Cu, 7 t t
31. Starting with a sample of pure of it
8
decays into Zn in 15 minutes. The corresponding N N
half life is [2008]
(a) 15 minutes (b) 10 minutes (c) (d)
1 t t
(c) 7 minutes (d) 5 minutes
2 38. A nuclear reaction is given by [2012]
32. A radioactive material decays by simultaneous A A 0
Z X ® Z+1Y + -1 e + n , represents
emission of two particles with respective half- (a) fission (b) b-decay
lives 1620 and 810 years. The time, in years, after (c) s -decay (d) fusion
which one-fourth of the material reamins is 39. An archaeologist analyses the wood in a
(a) 1080 (b) 2430 [2008] prehistoric structure and finds that C14 (Half life
(c) 3240 (d) 4860 = 5700 years) to C12 is only one-fourth of that
33. If the binding energy per nucleon of a nuclide is found in the cells of buried plants. The age of
high then [2009] the wood is about [2013]
(a) It should be abundantly available in nature (a) 5700 years (b) 2850 years
(b) It will decay instantly (c) 11,400 years (d) 22,800 years
(c) It will have a large disintegration constant 40. A radioactive nuclide is produced at the constant
(d) It will have a small half-life. rate of n per second (say, by bombarding a target
34. Activity of a radioactive sample decreases to with neutrons). The expected number N of nuclei
(l/3)rd of its original value in 3 days. Then, in 9 in existence t seconds after the number is N0 is
days its activity will become [2009] given by [2014]
(a) (1/27) of the original value (a) N = N0e–lt
(b) (1/9) of the original value (b) N = n + N 0e - lt
(c) (1/18) of the original value l
(d) (1/3) of the original value
(c) N = n + æç N 0 - n ö÷ e - l t
35. The half life of a radioactive substance against l è lø
a -decay is 1.2 × 107 s. What is the decay rate
for 4.0 × 1015 atoms of the substance [2010] n æ nö
(d) N= + ç N 0 + ÷ e - lt
(a) 4.6 × 10 12 atoms/s l è lø
(b) 2.3 × 10 11 atoms/s Where l is the decay constant of the sample
(c) 4.6 × 10 10 atoms/s 41. The fossil bone has a 14C : 12C ratio, which is
(d) 2.3 × 10 8 atoms/s é1ù
36. Actinium 231, 231Ac89, emit in succession two ê 16 ú of that in a living animal bone. If the half-
ë û
b-particles, four alphas, one b and one alpha
life of 14C is 5730 years, then the age of the
plus several g rays. What is the resultant isotope:
fossil bone is [2015]
(a) 221 Au79 (b) 211 Au 79 [2011] (a) 11460 years (b) 17190 years
(c) 221 Pb 82 (d) 211 Pb 82 (c) 22920 years (d) 45840 years
EBD_7100
P-200 Topicwise AIIMS Solved Papers – PHYSICS
42. Binding energy per nucleon versus mass number 46. Assertion: Separation of isotope is possible
curve for nuclei is shown in the figure. W, X, Y because of the difference in electron numbers of
and Z are four nuclei indicated on the curve. isotope.
The process that would release energy is [2016] Reason: Isotope of an element can be separated
(a) Y ® 2Z by using a mass spectrometer. [1999]
A
47. Assertion: zX undergoes 2a-decays, 2b-decays
(b) W ®X+Z
and 2g-decays and the daughter product is
(c) A–8.
W ® 2Y Z–2X
(d) X®Y+Z Reason : In a-decays the mass number
decreases by 4 and atomic number decreases by
2. In 2b-decays the mass number remains
43. The activity of a radioactive sample is measured unchanged, but atomic number increases by 1
as N0 counts per minute at t = 0 and N0/e counts only. [2001]
per minute at t = 5 minutes. The time (in minutes) 48. Assertion : Radioactive nuclei emit b– particles.
at which the activity reduces to half its value is Reason : Electrons exist inside the nucleus.
[2017] [2003]
5 49. Assertion : Neutrons penetrate mater more
(a) log e 2 / 5 (b)
log e 2 readily as compared to protons.
(c) 5 log 102 (d) 5 loge 2 Reason : Neutrons are slightly more massive
than protons. [2003]
TYPE B : ASSERTION REASON QUESTIONS 90
50. Assertion : Sr from the radioactive fall out from
Directions for (Qs. 44-55) : These questions consist a nuclear bomb ends up in the bones of human
of two statements, each printed as Assertion and beings through the milk consumed by them. It
Reason. While answering these questions, you are causes impairment of the production of red blood
required to choose any one of the following five cells.
responses. Reason : The energetic b-particles emitted in the
(a) If both Assertion and Reason are correct and decay of 90Sr damage the bone marrow. [2004]
the Reason is a correct explanation of the 51. Assertion : Energy is released in nuclear fission.
Assertion. Reason : Total binding energy of the fission
(b) If both Assertion and Reason are correct but fragments is larger than the total binding energy
Reason is not a correct explanation of the of the parent nucleus. [2004]
Assertion. 35
52. Assertion : It is not possible to use Cl as the
(c) If the Assertion is correct but Reason is incorrect. fuel for fusion energy.
(d) If both the Assertion and Reason are incorrect. Reason : The binding energy of 35Cl is too
(e) If the Assertion is incorrect but the Reason is small. [2005]
correct. 53. Assertion : The binding energy per nucleon, for
44. Assertion : Isobars are the elements having nuclei with atomic mass number A > 100,
same mass number but different atomic number. decreases with A.
Reason : Neutrons and protons are present Reason : The nuclear forces are weak for heavier
inside nucleus. [1997] nuclei. [2006, 2013]
45. Assertion : If the half life of a radioactive 54. Assertion : Cobalt-60 is useful in cancer therapy.
substance is 40 days then 25% substance decay
Reason : Cobalt-60 is a source of g-radiations
in 20 days. [1998]
n
capable of killing cancerous cells. [2006]
Reason : N = N 0 æç ö÷
1 55. Assertion : Heavy water is a better moderator
è2ø than normal water.
time elapsed Reason : Heavy water absorbs neutrons more
where, n =
half life period efficiently than normal water. [2007]
Nuclei P-201

Directions for (Qs. 56-61) : Each of these questions Reason : The mass of b-particle is less than the
contains an Assertion followed by Reason. Read them mass of a-particle. [2014]
carefully and answer the question on the basis of 59. Assertion : Energy is released when heavy
following options. You have to select the one that nuclei undergo fission or light nuclei undergo
best describes the two statements. fusion and
(a) If both Assertion and Reason are correct and Reason : For heavy nuclei, binding energy per
Reason is the correct explanation of Assertion. nucleon increases with increasing Z while for light
(b) If both Assertion and Reason are correct, but nuclei it decreases with increasing Z. [2015]
Reason is not the correct explanation of 60. Assertion : Energy is released when heavy
Assertion. nuclei undergo fission or light nuclei undergo
(c) If Assertion is correct but Reason is incorrect. fusion and
(d) If both the Assertion and Reason are incorrect. Reason : For heavy nuclei, binding energy per
56. Assertion : In a decay process of a nucleus, the nucleon increases with increasing Z while for
mass of products is less than that of the parent. light nuclei it decreases with increasing Z.
Reason : The rest mass energy of the products [2017]
must be less than that of the parent. [2012] 61. Assertion : Between any two given energy
levels, the number of absorption transitions is
57. Assertion : Binding energy (or mass defect) of
always less than the number of emission
hydrogen nucleus is zero.
transitions.
Reason : Hydrogen nucleus contain only one Reason : Absorption transitions start from the
nucleon. [2012] lowest energy level only and may end at any
58. Assertion : The ionising power of b-particle is higher energy level. But emission transitions may
less compared to a-particles but their penetrating start from any higher energy level and end at
power is more. any energy level below it. [2017]
EBD_7100
P-202 Topicwise AIIMS Solved Papers – PHYSICS

Type A : Multiple Choice Questions 7. (b) Fusion is the reaction responsible for the
production of light energy from the Sun. In
1. (a) In a nuclear reaction, th ere may be
it two hydrogen molecules fuse to form
conversion of some mass into energy. So,
both mass and energy are not conserved. helium.
1 1 4
It is the momentum which is conserved. 1H + 1H —® 2He + Energy

2. (b) l = 1.07 × 10–4 8. (a) When the body is already 33% decayed to
be decayed to a further 33%, it will require a
0.693 period equal to its half life. So achieve level
T1 / 2 = = 6500 years
1.07 ´10 - 4 of decay from 33% to 67% it takes time equal
3. (d) 226 to half life or 20 minutes.
88Ra
Number of protons = 88 9. (d) Energy released = total binding energy of
Number of neutrons = 226 – 88 = 138 A and B less total binding energy of X.
4. (c) We know that = (110 × 8.2 + 90 × 8.2)
– (200 × 7.4) (902 + 738) – 1480
dN = 160 MeV
= lN
dt 10. (b) Let n be number of fission per second
dN0 dN t n × 200 × 106 eV is produced in one second
Now, = lN 0 ; = lN t
dt dt = n × 200 × 106 × 1.6 × 10–19 Joule per second
9750 = lN 0 ; 975 = lN t n ´ 200 ´ 10 6 ´ 1.6 ´ 10 -19
= = 1(given )
N 0 9750 10 10 3
= = Þ N0 = 10N t
Nt 975 1
10 3 1014
We know that n= =
-11 3.2
2 ´ 1.6 ´ 10
Nt = N0e–lt
10
Nt 1 = ´ 1013 = 3.125 ´1013
= e-lt Þ = e -l.5 3.2
N0 10
11. (a) In a nuclear reactor neutrons are needed
10–1 = e–5l for nuclear reactions. The product of
Taking log on both sides, nuclear reaction is also neutrons. But fast
1 neutrons can not induce nuclear reactions.
- 1 = -5l ´ It requires to be slowed down. Fast neutrons
2.303
cannot transfer its energy to the target atom
1
l = ´ 2.303 = 0.461 per minute. effectively due to its high velocity. So, its
5 velocity is reduced. For it we uses heavy
5. (c) After every 2.5 days its activity reduces to water.
half the value,
n
10 12. (a) Neutrons ratio that is determines the
10days = = 4 half lives p
2.5 stability of nucleus. (Here, n is number of
4
æ1ö neutrons and p is number of protons).
Reduced activity = 1.6 ´ ç ÷ Now, n = 235 – 92 = 143
è2ø
p = 92
1
= 1.6 ´ = 0.1 curie 143
16 For 92U235; Neutron ratio = = 1.55
6. (b) Heavy water (D2O) is used as a moderator 92
in nuclear reaction. Which is the highest no. So, 92U235 is most
unstable.
Nuclei P-203

13. (a) When radioactive substance emits an fossil older than this are destroyed due to
a-particle then the atomic number is time factor. The tracks that are built by the
reduced by 2. Naturally its position in rays are destroyed. For fossil earlier then
periodic table will be reduced by 2 places. this, the tracks are so small that it cannot
14. (b) The nuclear reaction taking place in the be measured with accuracy.
atom is as follows 21. (c) Applying law of conservation of momentum
235
+ 0 n1 ¾
¾® MV = mv
92 U
141 Let mass of each proton = m
36 Ba + 56 Kr 92 + 3 0 n1 + Q
Mass of daughter nucleus = (Am – 4m)
Q = 200 MeV.
Mass of a-particle = 4m
The three neutrons generated are capable
of reacting with three atoms individually So, (Am – 4m) V = 4m × v
which leads to chain reaction. 4mv 4v
V= =
15. (c) Let half life = T m(A - 4) A - 4
40 22. (b) When electron-positron pair annihilates the
40 days = Half life
T energy released is due to conversion of
40 mass into energy.
Ratio of substance left = æç 1 ö÷ T Total mass = 2 × 9 × 10–31 kg
è2ø Energy produced
40 = 2 × 9 × 10–31 × (3 × 108)2
4
So, æç 1 ö÷ T = æç 1 ö÷ = 162 ´ 10 -15 = 1.62 ´ 10 -13 J
è2ø è2ø 23. (c) 30 days = 3 halflife
40 40 Fraction of material remained
=4 Þ T= = 10 days 3
T 4 æ1ö 1
16. (a) Nuclear fusion is possible only between = ç ÷ = = 0.125
è2ø 8
light nuclei as they become more stable by 24. (a) The operation of a nuclear reactor is said to
acquiring greater atomic no. be critical if the multiplication factor is
17. (c) In radiotracer technique we trace the rate of production of neutrons
existence of an atom by detecting the k=
rate of loss of neutrons
radiation emitted by it if atom is a
If k > 1 explosion occurs
radioactive one. The case as stated in the
If k < 1 the chain reaction comes to half.
question confirms to it so it is an example
238 234
of radiotracer technique. 25. (b) 92 U ¾¾
® 90 Th + 2 He 4
18. (b) Positron emission tomography (PET) is a p
nuclear medicine medical imaging technique 26. (c) Let be the initial ratio of 14C : 12C. Let it
q
which produces a three dimensional image
are map of functional process in the body. 1 p
is in the fossil. As q remains constant,
The positron annihilates with an electron 16 q
producing a pair of annihilation photons p
p must have reduced to during the
(gama rays) moving in opposite direction 16
period.
19. (a) Binding energy per nucleon is maximum for
atomic number (A) = 56, so figure (a) is p 1 1
= =
correct alternative. q 16 ( 2) 4
20. (b) Carbon dating is best suited for determining No. of half lives required = 4
the age of fossils if their age in years is of Age of fossil = 4 × 5730 = 22920 years
the order of 10, 000 years. This is because 27. (c)
EBD_7100
P-204 Topicwise AIIMS Solved Papers – PHYSICS

28. (d) h 0.693 0.693


l= Here l1 = and l 2 =
mv 1620 810
If they have same momentum (mv), they N 1 1 æ 0.693 0.693ö
must have same wave length. = Þ log e = - ç + ÷t
N0 4 4 è 1620 810 ø
2 3 4
29. (c) 1 H ∗ 1 H ¾¾
↑ 2 H ∗ n +17.59 MeV Þ 2.303[ -2 ´ (.3010)]
(Controlled thermonuclear fission reaction)
æ 0.693 0.693ö
30. (c) No. of 235U atoms in 2 kg of fuel = -0.693 ç + ÷t
è 1620 810 ø
6.023≥1023
= ≥ 2000 2 ´ 1620 ´ 810
235 Þ = t = 1080 year.
2430
fission energy per atom = 185 MeV
33. (a) High binding energy per nucleon ensures
\ Energy for 2 kg of fuel very high life of the nuclide. Hence they
6.023≥1026 ≥ 2 should be abundant in nature.
= ≥185MeV 34. (a) R = R0e–lt
235
Energy released 1
Power = Þ = e–l × 3 = e–3l ...(1)
time 3
Let activity in 9 days be R¢. Then
6.023≥1026 ≥ 2≥185≥1.6≥10,13 J R¢
= = e–l × 9 = e–9l e–l × 3 = (e–3l)3
235≥ 30days R0
( Q 1 MeV = 1.6 × 10–13 J, 30 days æ1ö
3

= 30 × 24 × 60 × 60 sec) =ç ÷ , from (1)


è3ø
6.023≥1026 ≥ 2 ≥185≥1.6≥10,13 1 R
\ Power = = Þ R¢ = 0 .
235≥30≥ 24 ≥ 60≥ 60 27 27
dN
3552≥1013 35. (d) We have, = lN
= W dt
235≥3≥ 6≥ 6≥ 24 ≥103 0.693 0.693
l= =
3552≥1010 T 1.2 ´ 107
=
235≥3≥ 6≥ 6≥ 24 dN 0.693
= ´ 4 ´ 1015
3552≥10 4
dt 1.2 ´ 107
= MW = 58.3 MW
235≥18≥ 6≥ 24 = 2.3 × 108 atom/s
7 36. (d) Five alpha and three beta particles results
31. (d) days of Cu decays. 231-5 ´ 4 =211
8 231
AC89 ¾¾
® Pb
3 (89 -5 ´ 2 +3´1) =82
7 1 æ1ö
\ Cu undecayed, N = 1 – = = ç ÷ 37. (c) No. of nuclide at time t is given by
8 8 è2ø N = N0e–lt
\ No. of half lifes = 3
Where N0 = initial nuclide
t 15
n= or 3 = thus this equation is equivalent to y = ae–kx
T T Thus correct graph is
15
Þ half life period, T = = 5 minutes N
3
- dN N
32. (a) = l1N + l 2 N Þ log e = - (l1 + l 2 )t
dt N0
t
where N0 is initial no. of atom
Nuclei P-205

0 –
–1e is known as b particle &
38. (b) n is known Given W = 2Y
as antineutrino. Since in this reaction n is BE of reactants = 120 × 7.5 = 900 MeV
emitted with –1e0 (b– particle or electron), BE of products = 2 × (60 × 8.5) = 1020 MeV.
so it is known as b-decay. 43. (d) N = N0e–lt
t / 5700 Here, t = 5 minutes
C14 1 æ 1 ö
39. (c) = =ç ÷
C12 4 è 2 ø N0
= N 0 × e -5l
t e
Þ = 2 Þ t = 11400 years 1
5700 Þ 5l = 1 , or l= ,
dN 5
40. (c) = n - lN l n2
dt Now, T1/2 = = 5 l n2
l
dN = (n - lN)dt
N t N
Type B : Assertion Reason Questions
dN 1 - ldN
ò n - lN ò
= dt Þ -
l ò n - lN
=t 44. (b) By definition, isobars are elements having
same mass number but different atomic
N0 0 N0
number. Presence of neutron and proton
1
Þ- [log e (n - lN )]NN0 = t inside nucleus has nothing to do with this
l definition.
1é æ n - lN ö ù 45. (e) Half life of radioactive substance is 40 days.
Þ- êlog e ç ú=t
l ëê è n - lN0 ø÷ ûú It means 50% substance decays in 40 days.
During this period rate of decay is on
é æ n - lN 0 öù decrease. So, 25% decay must have taken
Þ lt = êlog e ç ÷ú
ë è n - lN øû place is less than 20 days.
n
n - lN 0 æ1ö time elapsed
e lt = N = N 0 ç ÷ , where n =
n - lN è2ø half life period
n - lN = (n - lN 0 ) e - lt 46. (e) Isotope of an element can be separated by
using a mass spectrometer because
n æn ö
- ç - N 0 ÷ e -lt = N isotopes have different atomic mass.
l èl ø Alternative (e) is correct.
14 1 N 47. (a) The reason given is true. If we test the
C =
41. (c) = authenticity of assertion,
12
C 16 N0
a-decay a- rays
N æ 1ö
n
ZX
A
¾¾¾¾® Z-2 X A- 4 ¾¾¾¾
®
Q N0 = ç ÷
è 2ø A -8 (2b-rays)
Z- 4 X ® Z-2 X A -8
¾¾¾¾¾
n 4 n
1 æ 1ö æ 1ö æ 1ö
Þ = ç ÷ Þ çè ÷ø = çè ÷ø 47. (c) Radioactive nuclei emit b-particles. This
16 è 2 ø 2 2 b-particle comes from the splitting of
or, n = 4 neutron into b-particle and proton.
t 48. (b) Neturon is penetrate more readily as
or =4
T compared to protons because neutrons do
or t = 4 × T = 4 × 5730 = 22920 years not carry any change so there is no
42. (c) Energy is released in a process when total repulsion between nucleus and neutrons.
binding energy (BE) of products is more than So assertion is true reason is also true as
the reactants. By calculations we can see that
mass of neutron is more than proton but
this happens in option (c).
reason does not explain assertion.
EBD_7100
P-206 Topicwise AIIMS Solved Papers – PHYSICS
49. (a) RBC of blood are produced in the bone 56. (a)
marrow. The radiation from the radioactive 57. (a)
substances destroys of bone marrow which
result in hampered production of RBC. 58. (b) b-particles, being emitted with very high
50. (a) Total binding energy of fragment nucleus speed compared to a-particles, pass for
is more than total binding energy of parent very little time near the atoms of the medium.
nucleus. Since, binding energy results in So the probability of the atoms being
decrease of total energy. Hence there is great ionised is comparatively less. But due to
decrease in energy fragment nucleus this reason, their loss of energy is very slow
because energy is released in nuclear and they can penetrate the medium through
fission. a sufficient depth.
A ¾
¾® B + E 59. (c) We know that energy is released when
( Parent ) ( Fragment ) ( Energy ) heavy nuclei undergo fission or light nuclei
Energy of B is decrease but the binding undergo fusion. Therefore Assertion is
energy of B is increased due to release of correct.
energy from it. So, reason supports the The Reason is incorrect because for heavy
assertion. nuclei the binding energy per nucleon
51. (a) decreases with increasing Z and for light
52. (c) Since, 35Cl is stable so binding energy is nuclei, B.E/nucleon increases with
high. So it is not capable of disintegration. increasing Z.
53. (a) Binding energy per nucleon decreases with 60. (d) We know that energy is released when
A for nuclei with atomic mass number heavy nuclei undergo fission or light nuclei
A > 100 due to weak nuclear forces. It can undergo fusion. Therefore Assertion is
be explain as : correct.
At short distances, the nuclear force is The Reason is incorrect because for heavy
stronger than the Coulomb force; it can nuclei the binding energy per nucleon
overcome the Coulomb repulsion of decreases with increasing Z and for light
protons inside the nucleus. At typical nuclei, B.E/nucleon increases with
nucleon separation (1.3 fm) it is a very increasing Z.
strong attractive force (104 newtons). 61. (a) Absorption transition
Beyond about 1.3 fm separation, the force
exponentially dies off to zero. However, the C
Coulomb force between protons has a much
larger range and becomes the only B
significant force between protons when
their separation exceeds about (2.5 fm, A
A > 100).
54. (a) Cobalt 60 is radioactive isotope of cobalt. Two possibilities in absorption transition.
g-radiation emitted by it is used in radiation
therapy is cancer as it destroys cancerous
cells.
So, assertion and reason is true and reason
explains assertion.
55. (c) Heavy water has better ability to slow down
neutrons by elastic collision between their
protons and neutrons hence they are better Three possibilities in emission transition.
moderators. Heavy water does not absorb Therefore, absorption transition < emission.
neutrons.
28
Semi-Conductor, Electronics :
Materials, Devices and Simple Circuits
TYPE A : MULTIPLE CHOICE QUESTIONS 9. In n-type semiconductor, majority charge carriers
are [1999]
1. In p-type semiconductor major current carriers (a) electrons (b) neutrons
are : [1997] (c) holes (d) protons
(a) negative ions (b) holes 10. In a full wave rectifier circuit operating from 50
(c) electrons (d) all of these Hz mains frequency, what is the fundamental
2. In a diode, when there is a saturation current, frequency in the ripple? [2000]
the plate resistance will be [1997] (a) 50 Hz (b) 100 Hz
(a) data insufficient (c) 70 Hz (d) 25 Hz
(b) zero
(c) some finite quantity 11. A B Q
(d) infinite quantity 0 0 0
3. When the two semiconductors p- and n-type are 0 1 1
brought into contact they form a p-n junction, 1 0 1
which acts like a/an : [1997] 1 1 1
(a) rectifier (b) amplifier
(c) conductor (d) oscillator The truth table given above for which of the
4. The transfer ratio b of a transistor is 50. The following gates is correct [2000]
input resistance of the transistor when used in (a) NAND gate (b) OR gate
the common emitter configuration is 1kW. The (c) AND gate (d) NOT gate
peak value of the collector A.C. current for an 12. The variation of anode current in a triode valve
A.C. input voltage of 0.01 V, is [1998] corresponding to a change in grid potential at
(a) 500 µA (b) 0.25 µA three different values of the plate potential is
(c) 0.01 µA (d) 100 µA shown in th e given figure. The mutual
5. When n-p-n transistor is used as an amplifier, conductance of triode is [2000]
then [1999] I P mA
(a) electrons move from base to collector VP = 100 V
(b) holes move from emitter to base 20 mA
VP = 80 V
(c) electrons move from collector to base 15 mA
(d) holes move from base to emitter VP = 60 V
6. Boolean algebra is essentially based on:[1999] 10 mA
(a) Numbers (b) Symbol
5 mA
(c) Logic (d) Truth
7. A triode valve has an amplification factor of 20 -8 -6 -4 -2
and its plate is given a potential of 300 V. The
- Vg volt Vg volt
grid voltage to reduce the plate current to zero,
is [1999] (a) 5 × 10–3 mho (b) 2.5 × 10–3 mho
–3
(c) 7.5 × 10 mho (d) 9.5 × 10–3 mho
(a) 25 V (b) 15 V
(c) 12 V (d) 10 V 13. Which one of the following is true about the
8. Diode is used as a/an [1999] p-type and n-type semi-conductor ? [2001]
(a) modulator (b) rectifier (a) n-type semi-conductor have holes in
(c) oscillator (d) amplifier majority.
EBD_7100
P-208 Topicwise AIIMS Solved Papers – PHYSICS
(b) the concentration of electrons and holes electron-hole pairs is ~1019/m3, the concentration
are equal in both n-type and p -type semi- of electrons in the specimen is [2004]
conductors. (a) 1017/m3 (b) 1015/m3
(c) n-type semi-conductors have free electrons (c) 104/m3 (d) 102 /m3
in majority. 20. Which logic gate is represented by the following
(d) p-type semi-conductor has excess negative combination of logic gates ? [2004]
charge.
14. When added an impurity into the silicon which A
one of the following produces n-type of semi- Y
conductors : [2001] B
(a) iron (b) magnesium (a) OR (b) NAND
(c) aluminium (d) phosphorous (c) AND (d) NOR
15. At 0 K temperature, a p-type semiconductor 21. The voltage gain of the following amplifier is
[2002] [2005]
(a) has equal number of holes and free 100 kW
electrons
(b) has few holes but no free electrons 1 kW

(c) has few holes and few free electrons +
(d) does not have any charge carriers Vi – Vo
16. The value of current gain a of a transistor is 10 kW
0.98. The value of b will be [2002]
(a) 490 (b) 4.9
(a) 10 (b) 100
(c) 59 (d) 49
(c) 1000 (d) 9.9
17. To a germanium sample, traces of gallium are
22. Which of the following logic gates is an universal
added as an impurity. The resultant sample would
behave like : [2003] gate ? [2005]
(a) a conductor (a) OR (b) NOT
(b) a p-type semi-conductor (c) AND (d) NAND
(c) an n-type sem-iconductor 23. Consider an n-p-n transistor amplifier in common
(d) an insulator emitter configuration. The current gain of the
18. In the following common emitter configuration transistor is 100. If the collector current changes
an npn transistor with current gain b = 100 is by 1mA, what will be the change in emitter current
used. The output voltage of the amplifier will (a) 1.1 mA (b) 1.01 mA [2005]
be : [2003] (c) 0.01 mA (d) 10 mA
24. In a semi-conducting material the mobilities of
electrons and holes are µe and µh respectively.
Which of the following is true ? [2005]
10kW (a) me > mh (b) me < mh
Vout (c) me = mh (d) me = 0; mh > 0
1kW
1mV 25. When a p-n diode is reverse biased, then [2006]
(a) no current flows
(b) the depletion region is increased
(c) the depletion region is reduced
(a) 10mV (b) 0.1 V (d) the height of the potential barrier is reduced
(c) 1.0 V (d) 10 V 26. An amplifier has a voltage gain Av = 1000. The
19. A Ge specimen is doped with Al. The voltage gain in dB is: [2006]
concentration of acceptor atoms is ~1021 atoms/ (a) 30 dB (b) 60 dB
m3. Given that the intrinsic concentration of (c) 3 dB (d) 20 dB
Semi-Conductor, Electronics : Materials, Devices and Simple Circuits P-209

27. If the highest modulating frequency of the wave (a) 3.12 S/m (b) 2.12 S/m
is 5 kHz, the number of stations that can be (c) 1.12 S/m (d) 4.12 S/m
accomdated in a 150 kHz bandwidth are[2007] 35. An LED (Light Emitting Diode) is constructed
(a) 15 (b) 10 from a p–n junction based on a certain Ga–As–
(c) 5 (d) none of these P semi–conducting material whose energy gap
28. Zener diode acts as a/an [2007] is 1.9 eV. What is the wavelength of the emitted
(a) oscillator (b) regulator light? [2010]
(c) rectifier (d) filter (a) 650 nm (b) 65Å
29. In the half wave rectifier circuit operating from
(c) 800 nm (d) 8000 Å
50 Hz mains frequency, the fundamental
frequency in the ripple would be [2007] 36. In n-p-n transistor circuit the collector current is
(a) 25 Hz (b) 50 Hz 10 mA. If 90% of the electron reach the collector,
(c) 70.7 Hz (d) 100 Hz then emitter current will be [2010]
30. A transistor is a/an [2007] (a) 1 mA (b) 0.1 mA
(a) chip (b) insulator (c) 2 mA (d) nearly 11 mA
(c) semiconductor (d) metal 37. The cause of the potential barrier in a p-n junction
31. If collector current is 120 mA and base current is diode is [2011]
2 mA and resistance gain is 3, what is power (a) depletion of positive charges near the
gain? junction
(a) 180 (b) 10800 [2007] (b) concentration of positive charges near the
(c) 1.8 (d) 18 junction
32. If in a p-n junction diode, a square input signal (c) depletion of negative charges near the
of 10 V is applied as shown [2008] junction
(d) concentration of positive and negative
5V
charges near the junction
RL 38. Identify the logic operation of the following logic
circuit : [2011]
-5V
Then the output signal across RL will be
10 V
+5V
(a) NAND (b) AND
(a) ` (b)
(c) NOR (d) OR
39. Which represents NAND gate ? [2012]

(c) (d)
-5V (a)
-10 V
33. The input resistance of a common emitter
transistor amplifier, if the output resistance is
500 kW, the current gain a = 0.98 and power gain (b)
is 6.0625 × 106, is [2008]
(a) 198 W (b) 300 W
(c) 100 W (d) 400 W (c)
34. Mobilities of electrons and holes in a sample of
intrinsic Ge at room temperature are 0.35m2/V–s
and 0.18m2/V–s respectively. If the electron and
hole densities are each equal to 2.5×1019/m3, the (d)
Ge conductivity will be [2010]
EBD_7100
P-210 Topicwise AIIMS Solved Papers – PHYSICS
40. The real time variation of input signals A and B 45. Truth table for system of four NAND gates as
are as shown below. If the inputs are fed into shown in figure is [2017]
NAND gate, then select the output signal from A
the following. [2012]

A Y
A
B Y
B
t (s) B
A B Y A B Y
Y Y 0 0 0 0 0 0
(a) (b) (a) 0 1 1 (b) 0 1 0
t (s) t (s)
0 2 4 6 8 0 2 4 6 8 1 0 1 1 0 1
1 1 0 1 1 1
A B Y A B Y
(c) Y (d) Y 0 0 1 0 0 1
t (s) t (s)
0 2 4 6 8 0 2 4 6 8 (c) 0 1 1 (d) 0 1 0
41. An oscillator is nothing but an amplifier with 1 0 0 1 0 1
(a) positive feedback [2013, 2014] 1 1 0 1 1 1
(b) large gain TYPE B : ASSERTION REASON QUESTIONS
(c) no feedback
Directions for (Qs. 46-52) : These questions consist
(d) negative feedback
of two statements, each printed as Assertion and
42. Following diagram performs the logic function
Reason. While answering these questions, you are
of [2006, 2014]
required to choose any one of the following five
A responses.
Y
B (a) If both Assertion and Reason are correct and
the Reason is a correct explanation of the
(a) XOR gate (b) AND gate
Assertion.
(c) NAND gate (d) OR gate
(b) If both Assertion and Reason are correct but
43. Carbon, Silicon and Germanium atoms have four
Reason is not a correct explanation of the
valence electrons each. Their valence and
Assertion.
conduction bands are separated by energy band
(c) If the Assertion is correct but Reason is incorrect.
gaps represented by (E g)C, (E g)Si and (Eg)Ge (d) If both the Assertion and Reason are incorrect.
respectively. Which one of the following (e) If the Assertion is incorrect but the Reason is
relationship is true in their case? [2015] correct.
(a) (Eg)C > (Eg)Si (b) (Eg)C < (Eg)Si 46. Assertion : The resistivity of a semi-conductor
(c) (Eg)C = (Eg)Si (d) (Eg)C < (Eg)Ge increases with temperature.
44. Two ideal diodes are connected to a battery as Reason : The atoms of semi-conductor vibrate
shown in the circuit. The current supplied by with larger amplitude at higher temperatures
the battery is [2017] thereby increasing its resistivity. [2003]
D1 10W 47. Assertion : In a transition the base is made thin.
A B Reason : A thin base makes the transistor stable.
20W
[2004]
D2
C D 48. Assertion : The logic gate NOT can be built
using diode.
5V Reason : The output voltage and the input
E F
(a) 0.75 A (b) 0.5 A voltage of the diode have 180° phase difference.
(c) 0.25 A (d) zero [2005]
Semi-Conductor, Electronics : Materials, Devices and Simple Circuits P-211

49. Assertion : The number of electrons in a p-type (b) If both Assertion and Reason are correct, but
silicon semiconductor is less than the number Reason is not the correct explanation of
of electrons in a pure silicon semiconductor at Assertion.
room temperature. (c) If Assertion is correct but Reason is incorrect.
Reason : It is due to law of mass action. [2005] (d) If both the Assertion and Reason are incorrect.
50. Assertion : In a common emitter transmitter 53. Assertion : NAND or NOR gates are called
amplifier the input current is much less than the digital building blocks.
out put current. Reason : The repeated use of NAND (or NOR)
Reason : The common emitter transistor amplifier gates can produce all the basis or complicated
has very high input impedance. [2005] gates. [2011]
51. Assertion : In common base configuration. the 54. Assertion : A p-n junction with reverse bias can
current gain of the transistor is less than unity.. be used as a photo-diode to measure light
Reason : The collector terminal is reverse biased intensity.
for amplification. [2006] Reason: In a reverse bias condition the current
52. Assertion : The value of current through p-n is small but is more sensitive to changes in
junction in the given figure will be 10 mA. incident light intensity. [2012]
55. Assertion : If the temperature of a semiconductor
+ 5V 300 W +2V
is increased then it’s resistance decreases.
Reason : The energy gap between conduction
Reason : In the above figure, p-side is at higher band and valence band is very small.
potential than n-side. [2008] [2015]
Directions for (Qs. 53-57) : Each of these questions 56. Assertion : A transistor amplifier in common
contains an Assertion followed by Reason. Read them emitter configuration has a low input impedence.
carefully and answer the question on the basis of Reason : The base to emitter region is forward
following options. You have to select the one that biased. [2016]
best describes the two statements. 57. Assertion : Diode lasers are used as optical
(a) If both Assertion and Reason are correct and sources in optical communication.
Reason is the correct explanation of Assertion. Reason : Diode lasers consume less energy.
[2017]

Type A : Multiple Choice Questions Ic


or 50 = Þ I c = 50 ´10 -5
1. (b) In p-type semiconductor, holes 10 -5
predominates, so, holes are major current = 500 × 10–6
carriers. Ic = 500 mA
DV 5. (a)
Collector
2. (d) Dynamic plate resistance = N P N
DI Ie
Emitter
In case of saturation current, DI = 0 Ic
Plate resistance = ¥ Ib
3. (a) In case of diode (formed by p-n junction) it
acts as rectifier. It means it conducts only The connection in case of n-p-n transistor
in case of forward biasing & not in reverse is shown in the figure. Current is flowing
biasing. from collector to base so electrons will flow
Ic 0.01 in reverse direction i.e., from base to
4. (a) b= Þ Ib = = 10-5 collector.
Ib 1´103 6. (c) Boolean algebra is based on logic.
EBD_7100
P-212 Topicwise AIIMS Solved Papers – PHYSICS

Plate potential 17. (b) Gallium is a trivalent atom so, the resultant
7. (b) Amplification factor = 20 = sample will behave as p-type semi-
Grid potential
conductor.
300 300 I
20 = ÞV= = 15V 18. (c) b = c
V 20 Ib
8. (b) In diode, current can flow only in one
direction. In reverse bias it does not work. V V
Here, I c = =
So, it is used as rectifier. 10 ´ 10 3
10 4
9. (a) In n-type semiconductor we add 1´ 10 -3
pentavalent impurities so there is excess of Ib = = 10 - 6 ; b = 100
electrons inside the material. So, majority 10 ´ 10 3
I V
charge carriers are electrons. Now, 100 = c = ´ 10- 6
I b 10 4
10. (b)
V = 100 ´104 ´10-6 = 1 volt .
Frequency = 1 Frequency = 2 19. (a) nenh = ni2
Unrectified wave Rectified wave n e is concentration of electron, n h is
In case of full wave rectification, frequency concentration of holes and n i is
becomes twice. concentration of electron hole pairs in
So, new frequency = 50 ´ 2 = 100Hz intrinsic semi-conductor.
11. (b) Given table represents 'OR' gate. Here nh = 1021, ne = ?, ni = 1019
1021 × ne = 1019 × 1019
12. (b) Mutual conductance
1038
DI p 5 ´ 10-3 ne = = 1017 m -3
= = = 2.5 ´ 10-3 mho 10 21
DVg 2
20. (c) This is a case of AND gate. Input & output
13. (c) n-type semiconductors have free electrons are shown below
in majority p-type semiconductors have A
holes in majority. A A+B
14. (d) Phosphorus which is pentavalent produces y
n-type semiconductor. Here only four B
electrons of phosphorus are used in making B
bonds with silicon crystal. The fifth electron
\ y = A + B = A . B = AB (since
remains free.
15. (d) At 0 K an electron hole pair is formed as all A + B = A.B )
the electrons remain in the valence band. output voltage (Vo )
So we can say that there is no charge carrier 21. (b) Voltage gain =
input voltage(Vi )
at this temperature. From Ohm's law V = iR
I I R 100kW
16. (d) a = c and b = c \ Voltage gain, A v = = = 100
Ie Ib Ri 1kW
[Ic, Ib and Ie are collector, base and emitter 22. (d) NAND gate is considered universal gate.
current] As other gates can be formed from this gate.
DI 1
Ie I 23. (b) 100 = c Þ DI b = mA = 0.01mA
Ie = Ic + Ib Þ =1 + b DI b 100
Ic Ic
DIe = DIc + DI b = 1 + 0.01 = 1.01mA
1 1 a 0.98 24. (a) Mobility of electron is more than that of
=1+ Þ b= =
a b 1 - a 1 - 0.98 holes.
0.98 25. (b) When p-n junction is reverse biased, the
= = 49 depletion region is increased i.e., the
0.02
potential barrier increased.
Semi-Conductor, Electronics : Materials, Devices and Simple Circuits P-213

26. (a) Voltage gain = log 1000 Bel Given 6.0625 ´ 106 = A V ´ b
log 103 = 3Bel = 30 dB [1 Bel = 10 dB]
27. (a) In case of complex audio signals, æ 24.5 ´ 106 ö
=ç ÷ ´ 49
bandwidths (B.W) = 2 × maximum frequency è Ri ø
of modulating signal = 2 × 5 kHz = 10 kHz.
This is B.W for one channel. 24.5 ´ 49
or Ri = = 198 W
Total B.W 6.0625
\ No. of stations =
B.W for each channel 34. (b) Conductivity of Ge
150 kHz s = e (ne me + nh mh )
= = 15 Here ne = nh = 2.5 ×1019 /m3
10 kHz
28. (b) Zener diode is used to supply constant e = 1.6 × 10–19C,
voltage in voltage regulator circuit hence me=0.35m2/V–s, mh =0.18m2/V–s
option (b) is correct. \ s = 1.6 × 10–19
29. (b) In half wave rectifier, negative half of an (2.5 ×1019×0.35+2.5 ×1019×0.18)
a.c. cycle is removed from the sinusoidal = 1.6×10–19×2.5×1019×0.53
pulse, but the time lag between two positive = 2.12 S/m
pulses is same, hence frequency remains 35. (a) The wavelength of emitted light
same as the input (mains) frequency i.e.,
hc
50 Hz. l=
30. (c) As transistor is made of two p-types with Eg
n-type semiconductor in between or two n- where Eg = energy gap of semiconductor
types with p-type in between. = 1.9eV
31. (b) Ic = 120 mA, Ib = 2mA Þ Ie = Ib + Ic = 1.9×1.6×10–19V
(emitter current)
6.6 ´ 10 -34 ´ 3 ´108
Þ Ie = 120 + 2 = 122 mA l= m
1.9 ´ 1.6 ´ 10 -19
Resistance gain = 3
= 6. 5 ×10–7 m
d.c power gain
= 650×10–9 m
= (current gain)2 × resistance gain
= 650 nm
æ I ö2 36. (d) Ic = 10 mA; Ie = Ic + Ib
= çç c ÷÷÷ × resistance gain
ççè I ÷ø If Ic = 90% of Ie,
b
90
æ120 ö÷2 then Ic = ´ Ie
= çç 100
çè 2 ø÷÷
×3
100
Þ Ie = Ic
= (60)2 × 3 = 3600 × 3 = 10800 90
32. (a) The current will flow through RL when the 37. (d) During the formation of a junction diode,
diode is forward biased. holes from p-region diffuse into n-region
33. (a) R0 = 500 kW; a = 0.98 and electrons from n-region diffuse into p-
Power gain = 6.0625 × 106 region. In both cases, when an electrons
R0 meets a hole, they cancel the effect at each
we have, voltage gain A V = b. other and as a result, a thin layer at the
Ri junction becomes free from any of charges
But current gain, carriers. This is called depletion layer. There
a 0.98 is a potential gradient in the depletion layer,
b= = = 49 negative on the p-side, and positive on the
1 - a 1 - 0.98
n-side. The potential difference thus
500 ´ 103 24.5 ´ 106 developed across the junction is called
\ A V = 49 ´ =
Ri Ri potential barrier.
EBD_7100
P-214 Topicwise AIIMS Solved Papers – PHYSICS
38. (b) NAND + NAND 48. (d) The diode is unidirectional it allows current
Þ AND to pass through it in a particular direction.
39. (d) NAND is combination of AND gate and It does not change the phase of input signal.
NOT gate. 49. (a) We know that n enh = ni2
40. (b) From input signals, we have,
This formula is based on law of mass action.
A B Output NAND gate
In p-type semiconductor nh > ni
0 0 1
1 0 1 So, ne < ni
0 0 1 50. (c) In an amplifier output current is always more
1 1 0 than input current. Amplifier has low input
0 0 1 impedance.
The output signal is shown at B. 51. (b) For common base, Input is Ic and output is Ie.
41. (a) Ic Ic
42. (b) A Current gain = = which is less
Y
Ie Ic + I b
B X than unity.
Collector terminal is reversed baised to
X = AB
increase the collector current. Both
\ Y = X = AB Assertion and Reason are correct but they
Y = AB by Demorgan theorem are uncorrelated statements.
\ This diagram performs the function of 52. (b) The p-side of the junction diode is at a
AND gate. higher potential than the n-side. So p-n
junction is forward biased. Hence a current
43. (a) Due to strong electronegativity of carbon.
flows through it and is given by
44. (b) Here D1 is in forward bias and D2 is in
reverse bias so, D1 will conduct and D2 will V 5-2
I= = = 10-2 A = 10 mA
not conduct. Thus, no current will flow R 300
through DC. 53. (a) These gates are called digital building
V 5 1 blocks because using these gates only
I= = = A (either NAND or NOR) we can compile all
R 10 2
other gates also (like OR, AND, NOT, XOR).
45. (a)
A
54. (a)
55. (a) In semiconductors the energy gap between
Y2 = A.AB
conduction band and valence band is small
Y = A.AB B.AB (»1 eV). Due to temperature rise, electron in
Y1 = AB
the valence band gain thermal energy and
B may jumpy across the small energy gap, (to
Y3 = B.AB the conduction band). Thus conductivity
By expanding this Boolen expression increases and hence resistance decreases.
Y = A.B + B.A 56. (a) Input impedance of common emitter
configuration.
Type B : Assertion Reason Questions
DVBE
46. (d) The resistivity of semiconductor decreases =
DiB
with increase in temperature as more V
CE = constant
electrons jump into conduction band where DVBE = voltage across base and
increasing its conductivity. emitter (base emitter region is forward
47. (c) In a transistor the base is made thin so that
biased)
base current remains small and we can get
output or collector current. We know that DiB = base current which is order of few
Ic = I e – I b microampere.
Reason is incorrect. 57. (c)
29 Communication Systems

TYPE A : MULTIPLE CHOICE QUESTIONS 6. For 100% modulation (AM), the useful part of
the total power radiated is [2015]
1. In communication with help of antenna if height
1
is double then the range covered which was (a) of the total power
initially r would become [2007] 2
1
(a) 2r (b) 3r (b) of the total power
3
(c) 4r (d) 5r
1
2. Commun ication on ground is th rough (c) of the total power
4
electromagnetic waves of wavelength [2010]
2
(a) larger than 600 m (d) of the total power
(b) between 200 and 600 m 3
7. For transmission of e.m.wave of audible
(c) between 1 and 5 m
frequency, these waves are superimposed with
(d) between 10–3 and 0.1 waves of [2017]
3. 12 signals each band limited to 5 kHz are to be (a) frequency less than 20 Hz
transmitted by frequency-division multiplexer. If (b) frequency between 20 Hz and 10 KHz
AM-SSB modulation guard band of 1 kHz is used (c) frequency in the audible range
then the bandwidth of multiplexed signal is (d) radio-frequency
[2012]
(a) 101 kHz (b) 99 kHz TYPE B : ASSERTION REASON QUESTIONS
(c) 84 kHz (d) 71 kHz Directions for (Qs. 8) : These questions consist of
4. For 100% modulation (AM), the useful part of two statements, each printed as Assertion and Reason.
the total power radiated is [2013] While answering these questions, you are required to
1 choose any one of the following five responses.
(a) of the total power (a) If both Assertion and Reason are correct and
2
the Reason is a correct explanation of the
1 Assertion.
(b) of the total power
3 (b) If both Assertion and Reason are correct but
1 Reason is not a correct explanation of the
(c) of the total power Assertion.
4
(c) If the Assertion is correct but Reason is incorrect.
2
(d) of the total power (d) If both the Assertion and Reason are incorrect.
3 (e) If the Assertion is incorrect but the Reason is
5. Sky wave propagation is not possible for correct.
frequencies [2014] 8. Assertion : Optical fibres are used for
(a) equal to 30 MHz telecommunication.
(b) less than 30 MHz Reason : Optical fibres are based on the
(c) greater than 30 MHz phenomenon of total internal reflection. [2007]
(d) None of these
EBD_7100
P-216 Topicwise AIIMS Solved Papers – PHYSICS

Type A : Multiple Choice Questions 5. (c) Sky wave propagation is not possible for
frequency > 30 MHz because they are not
1. (a) Range of antenna = r 2hr , h = height of reflected by ionosphere.
antenna,
6. (b) 100% modulation Þ ma = 1
R = radius of earth
if h is doubled i.e., h' = 2h, then new range 2
useful power ma
=
r' = 2h ' r total power radiated 2 + m
a2
Þ r' = 2.2hr = 2 2hr = 2 r 1 1
= =
2. (d) The required wavelength should be from 2 +1 3
10–3m to 0.1m. Þ Useful power
100 1
Þ Ie = ´ 10 mA ; 11.1 mA = ( total power radiated )
90 3
3. (d) Total signal B.W = 12 × 5 = 60 kHz 7. (d) Since radio frequency waves can travel long
11 guard band are required between 12 distances because these waves are of wave
signal length of the order of 100 m and their energy
\ guard bandwidth = 11 × 1 kHz = 11 kHz content is quite large therefore e.m.wave of
\ total bandwidth = 60 + 11 = 71 kHz audible frequency are superimposed with
4. (b) 100% modulation Þ ma = 1 radio frequency waves.

useful power ma 2 Type B : Assertion Reason Questions


=
total power radiated 2 + m 8. (a) Both parts are true but optical fibres are
a2
used for telecommunication because they
1 1 produce the same signal with the same
= =
2 +1 3 intensity at the other end of the optical fibre
Þ Useful power even if fibre is several kilometre long i.e.,
total internal reflection. They have other
1
= ( total power radiated ) advantages like being light weight, flexible,
3 etc.
Some Basic Concepts
1 of Chemistry
TYPE A : MULTIPLE CHOICE QUESTIONS 8. 0.4 moles of HCl and 0.2 moles of CaCl 2 were
1. The weight of a single atom of oxygen is : [1997] dissolved in water to have 500 mL of solution,
the molarity of Cl– ion is: [2000]
(a) 1.057 × 1023 g (b) 3.556 × 1023 g
(a) 0.8 M (b) 1.6 M
(c) 2.656 × 10–23 g (d) 4.538 × 10–23 g
(c) 1.2 M (d) 10.0 M
2. Volume of a gas at NTP is 1.12 10 7
cc. The 9. 1021 molecules are removed from 200 mg of CO2.
number of molecule in it is : [1998] The moles of CO2 left are : [2001]
3 3
(a) 2.88 10 (b) 28.8 10
(a) 3.01 1012 (b) 3.01 1018
(c) 288 10 3 (d) 28.8 10 3
(b) 3.01 10 24 (d) 30
3.01 10 10. The weight of NaCl decomposed by 4.9g of
3. The oxide of an element contains 67.67% oxygen H2SO4, if 6 g of sodium hydrogen sulphate and
and the vapour density of its volatile chloride is 1.825 g of HCl, were produced in the reaction is:
79. Equivalent weight of the element is: [1998] (a) 6.921 g (b) 4.65 g [2001]
(a) 2.46 (b) 3.82 (c) 2.925 g (d) 1.4 g
11. Temperature does not affect : [1997, 2001]
(c) 4.36 (d) 4.96
(a) Molality (b) Formality
4. 60 gm of an organic compound on analysis is
(c) Molarity (d) Normality
found to have C = 24 g, H = 4 g and O = 32 g. The
empirical formula of compound is: [1998] 12. The molarity of H 2SO 4 solution, which has a
(a) CH2O (b) CHO density 1.84 g/cc. at 35° C and contains 98% by
(c) C2H2O (d) C2H2O2 weight, is : [2001]
5. The molar concentration of 20 g of NaOH (a) 1.84 M (b) 18.4 M
present in 5 litre of solution is : [1998] (c) 20.6 M (d) 24.5 M
(a) 0.1 moles/litre (b) 0.2 moles/litre 13. The normality of orthophosphoric acid having
purity of 70 % by weight and specific gravity
(c) 1.0 moles/litre (d) 2.0 moles/litre
1.54 is : [2001]
6. The normality of solution obtained by mixing 10 (a) 11 N (b) 22 N
ml of N/5 HCI and 30 ml of N/10 HCl is : [1999]
(c) 33 N (d) 44 N
N N 14. The weight of one molecule of a compound of
(a) (b)
15 5 molecular formula C60H122 is [2002]
N N –20 23
(c) (d) (a) 1.2 × 10 g (b) 5.025 × 10 g
7.5 8 –21
(c) 1.4 × 10 g (d) 6.023 × 10–20 g
7. The empirical formula of a compound is CH2O.
Its molecular weight is 180. The molecular 15. During electrolysis of water the volume of O2
formula of compound is : [1999] liberated is 2.24 dm3. The volume of hydrogen
liberated, under same conditions will be [2008]
(a) C 4 HO4 (b) C3H 6O 3 (a) 2.24 dm3 (b) 1.12 dm3
(c) C 6 H12O 6 (d) C 5 H10 O 5 (c) 4.48 dm 3 (d) 0.56 dm3
EBD_7100
C-2 Topicwise AIIMS Solved Papers – CHEMISTRY
16. Calculate the millimoles of SeO32– in solution on (a) CH4 (b) CH2
the basis of following data : [2009] (c) C2H (d) CH3
M 22. The vapour density of a mixture containing
70ml of solution of KBrO3 was added to
60 NO 2 and N 2 O 4 is 27.6. Mole fraction of NO 2
SeO3 2– solution. The bromine evolved was
in the mixture is [2012]
removed by boiling and excess of KBrO3 was
(a) 0.8 (b) 0.6
M (c) 0.4 (d) 0.2
back titrated with 12.5 mL of solution of
25 23. An aqueous solution of 6.3 g of oxalic acid
NaAsO2. dihydrate is made up to 250 ml. The volume of
The reactions are given below. 0.1 N NaOH required to completely neutralise
I. SeO32 BrO3 H SeO 42 Br2 H 2O 10 ml of this solution is [2013]
II. BrO3 AsO 2 H 2 O Br AsO34 H (a) 20 ml (b) 40 ml
(a) 1.6 × 10–3 (b) 1.25 (c) 10 ml (d) 4 ml
(c) 2.5 × 10–3 (d) None of these 24. KMnO4 reacts with oxalic acid according to the
17. The reaction of calcium with water is represented equation: [2013]
by the equation 2MnO 4– 5C 2 O 4 16 H
Ca + 2H2O Ca(OH)2 + H2
What volume of H2 at STP would be liberated Mn CO H O
when 8 gm of calcium completely reacts with Here 20 mL of 0.1 M KMnO4 is equivalent to:
water? [2010] (a) 20 mL of 0.5 M H2C2O4
(a) 0.2 cm3 (b) 0.4 cm3 (b) 50 mL of 0.5 M H2C2O4
(c) 2240 cm3 (d) 4480 cm3 (c) 50 mL of 0.1 M H2C2O4
18. A solution is prepared by dissolving 24.5 g of (d) 20 mL of 0.1 M H2C2O4
sodium hydroxide in distilled water to give 1 L
25. Calculate the normality of 10 volume H2O2 ?
solution. The molarity of NaOH in the solution
(a) 1.7 N (b) 12 N [2013]
is [2010]
(a) 0.2450 M (b) 0.6125 M (c) 30.3 N (d) 0.0303 N
(c) 0.9800 M (d) 1.6326 M 26. Which has the maximum number of molecules
(Given that molar mass of NaOH = 40.0 g mol–1) among the following ? [2014]
19. Which of the following pairs of solutions are (a) 44 g CO2 (b) 48 g O3
expected to be isotonic, temperature being the (c) 8 g H2 (d) 64 g SO2
same? [2011] 27. Sulphur forms the chlorides S2Cl2 and SCl2. The
(a) 0.1 M glucose and 0.1M C6H5N+H3Cl– equivalent mass of sulphur in SCl2 is [2015]
(b) 0.1 M NaCl and 0.05 M BaCl2 (a) 8 g/mol (b) 16 g/mol
(c) 0.1 M Na2SO4 and 0.1 M KNO3 (c) 64.8 g/mol (d) 32 g/mol
(d) 0.1 M BaCl2 and 0.075 M FeCl3 28. Arrange the following in the order of increasing
20. For preparing 0.1 N solution of a compound from mass (atomic mass: O = 16, Cu = 63, N = 14)
its impure sample of which the percentage purity I. one atom of oxygen
is known, the weight of the substance required II. one atom of nitrogen
will be [2012] III. 1 × 10–10 mole of oxygen
(a) less than the theoretical weight IV. 1 × 10–10 mole of copper [2016]
(b) more than the theoretical weight (a) II < I < III < IV (b) I < II < III < IV
(c) same as the theoretical weight (c) III < II < IV < I (d) IV < II < III < I
(d) none of these 29. Volume of water needed to mix with 10 mL 10N
21. In a hydrocarbon, mass ratio of hydrogen and HNO3 to get 0.1 N HNO3 is : [2017]
carbon is 1:3, the empirical formula of (a) 1000 mL (b) 990 mL
hydrocarbon is [2012] (c) 1010 mL (d) 10 mL
Some Basic Concepts of Chemistry C-3

TYPE B : ASSERTION REASON QUESTIONS 32. Assertion :One molal aqueous solution of
glucose contains 180g of glucose in 1 kg water.
Directions for (Qs. 30-32) : These questions consist
Reason : Solution containing one mole of solute
of two statements, each printed as Assertion and
Reason. While answering these questions, you are in 1000 g of solvent is called one molal solution.
required to choose any one of the following five [2008]
responses. Directions for (Qs.33-34) : Each of these questions
contains an Assertion followed by Reason. Read them
(a) If both Assertion and Reason are correct and
carefully and answer the question on the basis of
the Reason is a correct explanation of the
following options. You have to select the one that
Assertion.
best describes the two statements.
(b) If both Assertion and Reason are correct but
Reason is not a correct explanation of the (a) If both Assertion and Reason are correct and
Assertion. Reason is the correct explanation of Assertion.
(b) If both Assertion and Reason are correct, but
(c) If the Assertion is correct but Reason is incorrect.
Reason is not the correct explanation of
(d) If both the Assertion and Reason are incorrect.
Assertion.
(e) If the Assertion is incorrect but the Reason is (c) If Assertion is correct but Reason is incorrect.
correct. (d) If both the Assertion and Reason are incorrect.
30. Assertion : Atoms can neither be created nor 33. Assertion : The normality of 0.3 M aqueous
destroyed.
solution of H3PO3 is equal to 0.6 N.
Reason : Under similar condition of temperature
Reason : Equivalent weight of H3PO3
and pressure, equal volume of gases does not
Molecular weight of H 3PO3
contain equal number of atoms. [2002] [2011, 13]
31. Assertion :Equivalent weight of a base 3
34. Assertion : Equal moles of different substances
Molecular weight contain same number of constituent particles.
Acidity [2017]
Reason : Acidity is the number of replaceable Reason : Equal weights of different substances
hydrogen atoms in one molecule of the base. contain the same number of constituent
[2008] particles.
EBD_7100
C-4 Topicwise AIIMS Solved Papers – CHEMISTRY

Type A : Multiple Choice Questions 0.8


Molarity of Cl– = 1.6 M.
1. (c) Weight of single atom of oxygen 0.5
Wt. in g
16 23
9. (a) No. of moles =
= 2.656 10 g Mol. wt
23
6.02 10 200
No. of moles in 200 mg =
2. (a) No. of molecules 1000 44
6.02 1023 1.12 10 7 = 4.5 × 10–3 moles
21
No. of moles in 10 molecules
22400
6.02 10 7
1.12 1023 10 21
= 3.01 × 1012 = = 1.67 × 10–3 moles
2.24 10 4 6.02 1023
3. (b) Equivalent weight of an element is its weight No. of moles left = (4.5 – 1.67) × 10–3
which reacts with 8 gm of oxygen to form = 2.88 × 10–3
oxide. 10. (c) NaCl + H 2 SO 4 NaHSO 4 HCl
Thus eq. weight of the given element xg 4.9 g 6g 1.825 g

32.33 According to law of conservation of mass


8 3.82
67.67 "mass is neither created nor destroyed
24 4 32 during a chemical change"
4. (a) Ratio of no. of atoms = : :
12 1 16 Mass of the reactants = Mass of products
=2:4:2=1:2:1 x + 4.9 = 6 + 1.825
Empirical formula = CH2O.
or x = 2.925 g
Mole 20 / 40 11. (a) Temperature does not affect molality as it
5. (a) molar concentration =
Vol. in L 5 does not depend upon volume factor.
20 12. (b) 100 gm solution contains 98 gm H2SO4.
= 0.1 mole/litre.
5 40 100
6. (d) Normality of a mixture of two or more acids c.c contains 98 gm H2SO4.
is given by 1.84
N1V1 N 2 V2 1000 c.c solution contains
N 98
V1 V2 1.84 1000 gm H 2SO4
100
1 1 98 1.84 1000
10 30 5 1 N moles of H 2SO 4
5 10 or 100 98
10 30 40 8 8
= 18.4 M.
7. (c) Empirical formula weight
13. (c) 70% by weight means
= 12 + 2 + 16 = 30
Wt. of solute = 70 g
180
n= 6 Wt of solution = 100 g
30
mass 100
Molecular formula = (CH2O)6 = C6H12O6. Vcc of solution =
8. (b) HCl H Cl density 1.54
0.4moles 0.4moles We know that
2 W 1000
CaCl2 Ca 2Cl
0.2moles 2 0.2 0.4 moles N=
Eq.Wt × Vcc
Total Cl– moles = 0.4 + 0.4 = 0.8 moles
70 3 1000 1.54
Moles = 33 N
Molarity = 98 100
Vol.in L
Some Basic Concepts of Chemistry C-5

14. (c) M.W. = 60 × 12 + 122 = 842 18. (b) Given WNaOH = 24.5 g
842 24.5
gm No. of moles of NaOH = moles
Weight of one molecule = 23 40
6.02 10
= 0.6125 moles
= 140 × 10–23 gm = 1.4 × 10–21 gm
\ Molarity of solution
Electrolysis
15. (c) 2H 2 O 2H 2 O2 0.6125 moles
2 vol. 2 vol. 1 vol. = = 0.6125 M
1L
Thus, the volume of hydrogen liberated is
19. (d) Effective molarity of BaCl2 = 3 × 0.1 = 0.3 ;
twice that of the volume of oxygen liberated.
effective molarity of FeCl3 = 4 × 0.075 = 0.3
When 2.24 dm3 of oxygen is liberated the
volume of hydrogen liberated will be 2 × 2.24 20. (b) More than theoretical weight since impurity
dm3 or 4.48 dm3 will not contribute.
4 +5 +6 0 21. (a) Mass ratio of H : C = 1 : 12
16. (c) (I) SeO32 BrO3 H SeO42 Br2 H2O However, given mass ratio of H : C = 1 : 3
+5 +3 1 +5 Therefore, for every C atom, there are 4 H
(II) BrO3 AsO2 H2O Br AsO34 H atoms, hence empirical formula = CH4
In reaction (II)
22. (a) V.D max =
gm. eq. of BrO3– = gm. eq. of AsO2–
X NO2 (V.D) NO2 + X N 2O 4 ( V.D) N 2O4
n 6 n 2
BrO3 AsO2
27.6 = X × 23 + (1 – x) × 46
12.5 1 3 X NO2 = 0.8
2 10
1000 25
23. (b) Normality of oxalic acid
10 3 6.3 1000
n = 0. 4 N
BrO3 6 63 250
In reaction (I) N1V1 = N2V2 10 × 0.4 = V × 0.1 = 40 ml.
moles of BrO3– consumed 24. (c) Meq of A = Meq of B.
0.1 M KMnO4 = 0.5 N KMnO4
70 1 10 3
= 10 3 Meq of KMnO4 = 20 × 0.5 = 10 (n factor = 5)
1000 60 6 Meq of 50 ml of 0.1 M H2C2O4 = 50 × 0.2 = 10
gm eq. of SeO32– = gm. eq. of BrO3–
(0.1 M H2C2O4 = 0.2 N H2C2O4)
3
n 2 10 5 ; 25. (a) Normality of 10V of H2O2
SeO32
68 10 N = 1.78
n 2.5 10 3 17 N
SeO32 22.4
26. (c) No. of molecules
17. (d) Ca + 2H2O Ca(OH)2 + H2
44
According to the stoichiometry of reaction, Moles of CO2 = 1 NA
44
40 gm of Ca on complete reaction with water 48
liberates = 2 gm H2 Moles of O3 = 1 NA
48
8 gm of Ca, on complete reaction with 8
Moles of H2 = 4 4NA
2 2
water liberates = 8 gm H2 64
40 Moles of SO2 = 1 NA
= 0.40 gm H2 64
27. (b) The atomic weight of sulphur =32
0.40 In SCl2 valency of sulphur = 2
= 22400 cm3
2 32
= 4480 cm3 of H2 at S.T.P. So equivalent mass of sulphur = 16
2
EBD_7100
C-6 Topicwise AIIMS Solved Papers – CHEMISTRY
28. (a) Mass of 6.023 × 1023 atoms of oxygen = 16 g
Molecular weight
Mass of one atom of oxygen 31. (c) Yes, Eq. wt of a base =
Acidity
16
= 2.66 10 23 g Thus, assertion is correct.
6.023 10 23 Acidity is the number of replacable –OH
Mass of 6.023 × 1023 atoms of nitrogen = 14 g groups (not hydrogen atoms) present in a
Mass of one atom of nitrogen molecule . Thus, reason is false.
14 No.of moles of solute
23
2.32 10 23 g 32. (a) Molality =
6.023 10 Wt of solvent in kg.
Mass of 1 × 10–10 mole of oxygen = 16 × 10–10 Molecular mass
Mass of 1 mole of copper = 63 g Here, No. of moles =
Wt of solvent
Mass of 1 mole of oxygen = 16 g
180
Mass of 1 × 10–10 mole of copper =1
= 63 × 1 × 10–10 180
= 63 × 10–10 1
Molality = =1
So, the order of increasing mass is 1
II < I < III < IV. Hence assertion and reason, both are correct,
29. (b) Given N1 = 10N, V1 = 10 ml, N2 = 0.1N, and reason is the correct explanation of
V2 = ? assertion.
N1V1 N 2 V2 33. (c) Assertion is true, reason is false.
or 10 × 10 = 0.1 × V2
mol.wt
10 10 Eq. wt. of H3PO3 [ Basicity of
or V2 , V2 = 1000 ml 2
0.1
Volume of water to be added H3PO3 = 2]
= V2 – V1 = 1000 – 10 = 990 ml. 34. (c) Equal moles of different substances contain
same number of constituent particles but
Type B : Assertion Reason Questions equal weights of different substances do
30. (d) Atoms can be created and can be destroyed. not contain the same number of consituent
At N.T.P., number of molecules or atoms particles.
contained in same volume remains equal.
2 Structure of Atom

TYPE A : MULTIPLE CHOICE QUESTIONS (a) 54.47 10 12


erg atom 1

1. The configuration 1s2, 2s22p5, 3s1 shows : 12 1


(b) 5.447 10 erg atom
(a) excited state of O 2 [1997]
12 1
(c) 0.5447 10 erg atom
(b) excited state of neon atom
(c) excited state of fluorine atom 12 1
(d) 0.05447 10 erg atom
(d) ground state of fluorine atom
8. Deuterium nucleus contains: [2000]
2. The total number of orbitals in a shell with
(a) 1 proton, 1 electron
principal quantum number ‘n’ is : [1997]
(b) 1 proton, 1 neutron
(a) n2 (b) n + 1 (c) 2 protons, 1 electron
(b) 2n (d) 2n 2 (d) 1 proton, 2 electrons
3. Positron is : [1997] 9. The outer most configuration of most
electronegative element is: [2000]
(a) electron with positive charge
(b) a nucleus with one neutron and one proton (a) ns 2 np5 (b) ns 2 np 6
(c) a nucleus with two protons 2 3
(c) ns 2 np 4 (d) ns np
(d) a helium nucleus
10. Bohr's theory is not applicable to: [2000]
4. The wavelength of visible light is : [1998] (a) H (b) He +
(a) 2000 Å – 3700 Å (b) 7800 Å – 8900 Å (c) Li2+ (d) H+
(c) 3800 Å – 7600 Å (d) None of these 11. The de-Broglie wavelength of an electron in the
5. The wavelength of a 150 g rubber ball moving ground state of hydrogen atom is :
with a velocity of 50 ms–1 is : [1998]
33 cm 33
[K.E. = 13.6 eV ; 1eV = 1.602 10 19 J] [2000]
(a) 3.43 10 (b) 5.86 10 cm
(a) 33.28 nm (b) 3.328 nm
(c) 7.77 10 33 cm (d) 8.83 10 33 cm (c) 0.3328 nm (d) 0.0332 nm
6. If e = 1.60206 10 19 C, [1999] 12. The de-Broglie wavelength associated with a
particle of mass 10–6 kg moving with a velocity
e of 10 ms–1 is :
1.75875 1011 C kg 1 [2001]
m 7m 16
(a) 6.63 10 (b) 6.63 10 m
then the mass of electron is
21 29
(a) 7.5678 × 10–31 kg (b) 9.1091 × 10–31 kg (c) 6.63 10 m (d) 6.63 10 m
(c) 11.2531 × 10–31 kg (d) 13.0513 × 10–31 kg 13. The velocity of electron in second shell of
hydrogen atom is : [2001]
7. The energy of electron in first energy level is
21.79 10 12 erg per atom. The energy of
(a) 10.94 106 ms 1
(b) 18.88 10 6 ms 1

electron in second energy level is : [1999] (c) 1.888 10 6 ms 1 (d) 1.094 106 ms 1
EBD_7100
C-8 Topicwise AIIMS Solved Papers – CHEMISTRY
14. Which of the following element is represented 23. The uncertainties in the velocities of two
particles, A and B are 0.05 and 0.02 ms–1
by electronic configuration 1s2 2s 2 2 p1x 2 p1y 2 p1z ? respectively. The mass of B is five times to that
[2001] of the mass of A. What is the ratio of
(a) nitrogen (b) oxygen
uncertainties xA in their positions ? [2008]
(c) fluorine (d) sulphur xB
15. Quantum numbers of an atom can be defined on
(a) 2 (b) 0.25
the basis of [2002]
(c) 4 (d) 1
(a) Hund’s rule
24. Find the frequency of light that corresponds to
(b) Pauli’s exclusion principle photons of energy 5.0 × 10–5 erg [2010]
(c) Aufbau’s principle (a) 7.5 × 10–21 sec–1 (b) 7.5 × 10–21 sec
(d) Heisenberg’s uncertainity principle (c) 7.5 × 1021 sec–1 (d) 7.5 × 1021 sec
16. Spectrum of Li2+ is similar to that of [2002] 25. Ratio of energy of photon of wavelength 3000 Å
(a) H (b) Be and 6000Å is [2012]
(c) He (d) Ne (a) 3 : 1 (b) 2 : 1
17. Azimuthal quantum number defines [2002] (c) 1 : 2 (d) 1 : 3
(a) e/m ratio of electron 26. Which of the following combinations of quantum
(b) angular momentum of electron numbers is allowed? [2013]
(c) spin of electron n l m m s
(a) 3 2 1 0
(d) magnetic momentum of electron
18. The quantum number 'm' of a free gaseous atom 1
(b) 2 0 0
is associated with : [2003] 2
(a) the effective volume of the orbital 1
(b) the shape of the orbital (c) 3 –3 –2 +
2
(c) the spatial orientation of the orbital
1
(d) the energy of the orbital in the absence of a (d) 1 0 1 +
2
magnetic field
27. Among the following groupings which
19. For principle quantum number n = 4, the total represents the collection of isoelectronic
number of orbitals having l = 3 is : [2004] species? [2013]
(a) 3 (b) 7
(c) 5 (d) 9 (a) NO , C 22 , O 2 , CO
20. The isoelectronic pair is : [2005] (b) N 2 , C 22 , CO, NO
(a) Cl2O3 and ICl 2– (b) ICl 2– and CIO2
(c) IF2 and I3– (d) ClO 2 and ClF2 (c) CO, NO , CN , C 22
21. The most probable radius (in pm) for finding the
electron in He+ is [2005] (d) NO, CN , N 2 , O 2
(a) 0.0 (b) 52.9 28. The electrons, identified by quantum numbers n
(c) 26.5 (d) 105.8 and l (i) n = 4, l = 1 (ii) n = 4, l = 0 (iii) n = 3, l = 2
22. The de Broglie wavelength associated with a (iv) n = 3, l = 1 can be placed in order of increasing
ball of mass 1 kg having kinetic energy 0.5 J is : energy, from the lowest to highest, as [2014]
[2006] (a) (iv) < (ii) < (iii) < (i)
34 34
(b) (ii) < (iv) < (i) < (iii)
(a) 6.626 10 m (b) 13.20 10 m (c) (i) < (iii) < (ii) < (iv)
21 34 (d) (iii) < (i) < (iv) < (ii)
(c) 10.38 10 m (d) 6.626 10 Å
Structure of Atom C-9
29. In hydrogen atomic spectrum, a series limit is TYPE B : ASSERTION REASON QUESTIONS
found at 12186.3 cm–1. Then it belong to
[2014] Directions for (Qs. 34-42) : These questions consist
(a) Lyman series (b) Balmer series of two statements, each printed as Assertion and
Reason. While answering these questions, you are
(c) Paschen series (d) Brackett series
required to choose any one of the following five
Z2 responses.
30. Based on equation E = – 2.178 × 10-18 J , (a) If both Assertion and Reason are correct and
n2
the Reason is a correct explanation of the
certain conclusions are written. Which of them Assertion.
is not correct ? [2015] (b) If both Assertion and Reason are correct but
(a) Larger the value of n, the larger is the orbit Reason is not a correct explanation of the
radius. Assertion.
(b) Equation can be used to calculate the (c) If the Assertion is correct but Reason is incorrect.
change in energy when the electron (d) If both the Assertion and Reason are incorrect.
changes orbit. (e) If the Assertion is incorrect but the Reason is
(c) For n = 1, the electron has a more negative correct.
energy than it does for n = 6 which mean 34. Assertion : An atom is electrically neutral
that the electron is more loosely bound in Reason : Atom contains equal number of protons
the smallest allowed orbit. and neutrons. [1997]
(d) The negative sign in equation simply 35. Assertion : The sum of protons and neutrons is
means that the energy or electron bound to always different in isobars.
the nucleus is lower than it would be if the Reason : Isobars are atoms of different elements
electrons were at the infinite distance from having same mass number but different atomic
number. [1997]
the nucleus.
36. Assertion : All photons possess the same
31. Let mp be the mass of a proton, mn that of a
amount of energy.
neutron, M1 that of a 20 nucleus and M2 that
10 Ne Reason : Energy of photon does not depend
upon wavelength of light used. [1998]
of a 40 nucleus. Then [2015]
20 Ca 37. Assertion : Atoms are not electrically neutral.
(a) M2 = 2M1 (b) M1<10(mp + mn) Reason : Number of protons and electrons are
(c) M2 > 2M1 (d) M1 = M2 different [1999]
32. Which transition in the hydrogen atomic 38. Assertion : For Balmer series of hydrogen
spectrum will have the same wavelength as the spectrum, the value n1 = 2 and n2 = 3, 4, 5.
transition, n = 4 to n = 2 of He+ spectrum? Reason : The value of n for a line in Balmer
series of hydrogen spectrum having the highest
[2016]
wave length is 4 and 6. [2002]
(a) n = 4 to n = 3 (b) n = 3 to n = 2 39. Assertion : Absorption spectrum consists of
(c) n = 4 to n = 2 (d) n = 2 to n = 1 some bright lines separated by dark spaces.
33. In Bohr series of lines of hydrogen spectrum, the Reason : Emission spectrum consists of dark
third line from the red end corresponds to which lines. [2002]
one of the following inter-orbit jumps of the 40. Assertion : Nuclear binding energy per nucleon
electron for Bohr orbits in an atom of hydrogen is in the order 94 Be 7
Li 4
He .
3 2
[2017]
Reason : Binding energy per nucleon increases
(a) 5 2 (b) 4 1 linearly with difference in number of neutrons
(c) 2 5 (d) 3 2 and protons. [2004]
EBD_7100
C-10 Topicwise AIIMS Solved Papers – CHEMISTRY
41. Assertion :A spectral line will be observed for a 43. Assertion : Angular momentum of an electron
2px– 2py transition. in any orbit is given by angular momentum
Reason : The energy is released in the form of n.h
, where n is the principal quantum number..
wave of light when electron drops from 2px to 2
2py orbital. [2008] Reason : The principal quantum number, n, can
42. Assertion : An orbital designated by n = 3, have any integral value. [2012, 13]
l = 1 has double dumb-bell shape. 44. Assertion : Spin quantum number can have two
Reason : It belongs to p-subshell. [2011] 1 1
Directions for (Qs.43-46) : Each of these questions values, + and – .
2 2
contains an Assertion followed by Reason. Read them
Reason : + and – signs signify the positive and
carefully and answer the question on the basis of negative wave functions. [2014]
following options. You have to select the one that 45. Assertion : It is impossible to determine the exact
best describes the two statements. position and exact momentum of an electron
(a) If both Assertion and Reason are correct and simultaneously.
Reason is the correct explanation of Assertion. Reason : The path of an electron in an atom is
(b) If both Assertion and Reason are correct, but clearly defined. [2016]
Reason is not the correct explanation of 46. Assertion : The radius of the first orbit of
Assertion. hydrogen atom is 0.529Å. [2017]
(c) If Assertion is correct but Reason is incorrect. Reason : Radius of each circular orbit (rn) - 0.529Å
(d) If both the Assertion and Reason are incorrect. (n2/Z), where n = 1, 2, 3 and Z = atomic number.
Structure of Atom C-11

Type A : Multiple Choice Questions 11. (c) We know that


1. (b) Atomic number of the given element = 10 1 2
K .E. mv
Electronic configuration = 1s2, 2s22p6 2
1s22s22p6 is electronic configuration of Ne.
2 K .E.
1s22s22p53s1 is excited oxidation state. v
m
2. (a) The total no of orbital in a shell is n2.
3. (a) Positron is electron with positive charge, 19
0 2 13.6 1.602 10
+1e
4. (c) Wavelength of visible light is 9.1 10 31
3800Å – 7600Å. = 2.18824 × 106 m/s
5. (d) From the de-Broglie formula for wavelength h
No since,
h h 6.6 10 34 mv
33 cm.
8.83 10 34
p mv 150 50 6.626 10

e 9.1 10 31 2.18824 106


11
6. (b) 1.75875 10 = 0.3328 × 10–9 = 0.3328 nm
m
19 34
1.60206 10 h 6.6 10 29 m
1.75875 1011 12. (d) 6.63 10
mc 6
m 10 10

1.60206 10 19
1.60206 13. (d) According to Bohr, velocity (v) of an
30
m= 10 electron is given by relation.
1.75875 1011 1.75875
= 9.1091 × 10–31 kg. Z
v = 2.188 × 106 m/s
7. (b) For a particular element, n
Here n = 2, and Z (for H) = 1
13.6Z 2 1
En 2
erg atm 2.188 106 1
n v
2
1 =1.094 × 106 m/s
En
n2 14. (a) Electronic configuration corrosponds to
atomic number 7; hence the element is
E1 (n22 ) nitrogen.
E2 (n1 )2 15. (b) Quantum numbers of an atom can be
defined on the basis of Pauli’s exclusion
(1) 2 ( 21.79 10 12
) principle which states that no two
or E2 electrons can have the same value of all
(2) 2 the four quantum numbers.
= –5.447 × 10–12 erg atm–1 16. (a) Li2+ will have only one electron in its outer-
8. (b) Deuterium nucleus contains 1 proton and 1 most electron. Its spectrum will be very
neutron because it is an isotope of hydrogen. similar to that of hydrogen.
9. (a) Most electronegative element corresponds 17. (b) Azimuthal quantum no. 'm' defines angular
to ns2np5 configuration. momentum of electron.
10. (d) Bohr's Theory is not applicable to H+ as it 18. (c) Magnetic quantum no. ‘m’ represents the
has no electron. spatial orientation of the orbital.
EBD_7100
C-12 Topicwise AIIMS Solved Papers – CHEMISTRY
19. (b) For n = 4 and = 3, the orbital is 4 f. 26. (b)
No. of values of m = (2 + 1) = 7 27. (c) The species CO, NO+, CN– and C22– contain
14 electrons each.
20. (d) No. of electrons in ClO2 = 17 + 16 + 1 = 34
28. (a) (n + l) rule the higher the value of (n + l), the
No. of electrons in ClF2 =17 + 9 ×2 – 1 = 34 higher is the energy. When (n + l) value is the
same see value of n.
21. (c) Bohr's radius (r)
i ii iii iv
0.529 10 10 n2 52.9 n2 (n + l) (4 + 1) (4 + 0) (3 + 2) (3 + 1)
= m pm
Z Z 5 4 4 5
Here n (No of shell) = 1 iv < ii < iii < i
Z (At. No) = 2 29. (c) Series limit is the last line of the series, i.e.
2 n2 = .
52.9 1
r= 26.45 pm
2
h h 1 1 1 1 1 R
22. (a) R R
mv n12 n 22 n12 2
n12
2mE
09677.76
6.6 10 –34 12186.3
= 6.6 × 10–34 m n12
2 1 0.5
23. (a) Using the relation, 109677.76
n12 9 n1 3
12186.3
h The line belongs to Paschen series.
x. v
4 m 30. (c) Energy of an electron at infinite distance
[Heisenberg' uncertainty principle] from the nucleus is zero. As an electron
h approaches the nucleus, the electron
or x attraction increases and hence the energy
4 m. v
of electron decreases and thus becomes
h negative. Thus as the value of n decreases,
Thus, xA … (i)
4 0.05 m i.e. lower the orbit is, more negative is the
h energy of the electron in it.
xB … (ii) 31. 20
(a) 10
4 0.02 5m Ne contains 10 protons and 10 neutrons
Dividing (i) by (ii), we get M1 = 10 mp + 10mn
40
xA 0.02 5
10 20 Ca contains 20 protons and 20 neutrons
or 2
xB 50.05 M2 = 20 mp + 20 mn
24. (c) Using E = hv, we get M2 =2M1

E 5.0 10 5 erg 1 1 1
32. (d) For He+ ion, Z2 R
= = n12 n 22
h 6.63 10 34 Js
2 1 1 3R
2 R
5.0 10 5 erg 22 42 4
= 34
6.63 10 107 erg sec 1 1 1
For hydrogen atom, R
n12 n 22
[ 1J 107 erg]
= 7.54 × 1021 sec–1 3R 1 1 1 1 3
R or
hc E1 6000 4 n12 n 22 n12 n 22 4
25. (b) 2
E ; 2 :1
E2 1 3000 n1 = 1 and n2 = 2.
Structure of Atom C-13
33. (a) The lines falling in the visible region comprise nucleon, higher is stability of the nucleus.
Balmer series. Hence the third line would The order of binding energy is
4
be n =2, n = 5 i.e. 5 2.
2 He > 73 Li > 94 Be
Type B : Assertion Reason Questions 41. (d) In this case both assertion and reason are
false. Both 2px and 2py orbitals have equal
34. (c) Atom is electrically neutral. Atoms energy (2p orbitals are degenerate), there
necessarily contain equal number of is no possibility of electron transition and
protons and electrons, but not neutrons. hence no energy is released and thus no
35. (e) Isobars have the same atomic mass (sum of
spectral line will be observed.
protons and neutrons) but different atomic
42. (d) The orbital has dumb-bell shape and
numbers.
belongs to p-subshell.
c 43. (b) Both assertion and reason are correct.
36. (d) Energy of a photon = hv = h. .
Reason is not the correct explanation of
So, energy depends upon wavelength. assertion.
37. (d) Atoms are electrically neutral as number of
44. (c) Plus and minus signs of spin quantum
electrons and protons are same.
38. (c) The value of n for a line in Balmer series of numbers imply that spin angular momentum
hydrogen spectrum having the highest of the electron, a vector quantity, acts in
wave length will be n1 = 2 and n2 = 3 because the same or opposite directions of orbital
this transition will have lowest energy and angular momentum.
so highest wavelength. 45. (c)
39. (d) Absorption spectrum consists of dark lines 46. (a) Both assertion and reason are true and
separated by bright space and emission reason is the correct explanation of
spectrum consists of bright lines. assertion.
40. (d) Binding energy depends on the mass defect
n2 h2 n2
(mass lost when the constituent protons and Radius, rn 0.529Å.rn
2 Z
neutrons combine to form nucleus). Binding 4 e mZ
For first orbit of H-atom
energy is direct measurement of stability of
n=1
nucleus Higher the binding energy (means
high loss of energy during formation of (1)2
r1 0.529Å = 0.529Å
nucleus from protons and neutrons) per 1
EBD_7100
C-14 Topicwise AIIMS Solved Papers – CHEMISTRY

Classification of Elements and


3 Periodicity in Properties
TYPE A : MULTIPLE CHOICE QUESTIONS 8. Which of the following does not have valence
electron in 3d-subshell? [2002]
1. The first ionisation potential is maximum for: (a) Fe(III) (b) Cr(I)
(a) Lithium (b) Uranium [1998] (c) Mn(II) (d) P(0)
(c) Iron (d) Hydrogen 9. Which of the following statement is correct
2. Which element has high electron affinity? about ionization potential? [2010]
(a) Na (b) Cl [1998] (a) It is independent of atomic radii
(c) Be (d) Ca (b) It remains constant with change in atomic
3. Which group of periodic table contains no metal: radii
(a) IA (b) IIIA [1999] (c) It increases with an increase in atomic radii
(c) VIIA (d) VIII (d) It decreases with an increase in atomic radii
4. Which of the following is the atomic number of 10. Consider the following statements [2012]
metal? [2000] I. The radius of an anion is larger than that of
(a) 32 (b) 34 (c) 36 (d) 38 the parent atom.
5. The correct order of hydration energy of alkali is II. The ionization energy generally increases
(a) Li+ > Na+ > K+ > Rb+ [2000] with increasing atomic number in a period.
(b) Rb+ > K+ > Na+ > Li+ III. The electronegativity of an element is the
tendency of an isolated atom to attract an
(c) Na+ > K+ > Li+ > Rb+
electron.
(d) K+ > Rb+ > Na+ > Li+
Which of the above statements is/are correct?
6. Sodium forms Na+ ion but it does not form Na2+
(a) I alone (b) II alone
because of [2001]
(c) I and II (d) II and III
(a) very low value of 1st and IInd I.E. 11. Which of the order for ionization energy is
(b) very high value of Ist and IInd I.E. correct? [2013]
(c) high value of Ist I.E. and low value of IInd (a) Be < B < C < N < O
I.E. (b) B < Be < C < O < N
(d) low value of Ist I.E. and high value of IInd (c) Be > B > C > N > O
I.E. (d) B < Be < N < C < O
7. Which of the following has maximum energy? 12. The element with atomic number 117 has not
[2002] been discovered yet. In which family would you
3s 3p 3d place this element if discovered? [2014]
(a) (a) Alkali metals
(b) Alkaline earth metals
3s 3p 3d
(c) Halogens
(b) (d) Noble gases
3s 3p 3d 13. In which of the following arrangements, the order
(c) is NOT according to the property indicated
against it? [2015]
3s 3p 3d
(a) Li < Na < K < Rb :
(d) Increasing metallic radius
Classification of Elements and Periodicity in Properties C-15

(b) I < Br < F < Cl : (b) If both Assertion and Reason are correct, but
Increasing electron gain enthalpy Reason is not the correct explanation of
(with negative sign) Assertion.
(c) B < C < N < O (c) If Assertion is correct but Reason is incorrect.
Increasing first ionization enthalpy (d) If both the Assertion and Reason are incorrect.
16. Assertion : First ionization energy for nitrogen
(d) Al3 Mg 2 Na F
is lower than oxygen.
Increasing ionic size Reason : Across a period effective nuclear
14. The correct decreasing order of first ionisation charge decreases. [2005]
enthalpies of five elements of the second period 17. Assertion : Electron affinity refers to an isolated
is [2016] atom’s attraction for an additional electron while
(a) Be > B > C > N > F (b) N > F > C > B > Be electronegativity is the ability of an atom of an
(c) F > N > C > Be > B (d) N > F > B > C > Be element to attract electrons towards itself in a
15. The law of triads is applicable to a group of shared pair of electrons.
[2017] Reason : Electron affinity is a relative number
(a) Cl, Br, I (b) C, N, O and electronegativity is experimentally
(c) Na, K, Rb (d) H, O, N measurable. [2010]
18. Assertion : Element has a tendency to lose the
TYPE B : ASSERTION REASON QUESTIONS electron(s) to attain the stable configuration.
Directions for (Qs.16-19) : Each of these questions Reason : Ionization enthalpy is the energy
contains an Assertion followed by Reason. Read them released to remove an electron from an isolated
carefully and answer the question on the basis of gaseous atom in its ground state. [2011]
following options. You have to select the one that 19. Assertion : Both Be and Al can form complexes
best describes the two statements. such as BeF42– and AlF63– respectively, BeF63–
is not formed. [2015]
(a) If both Assertion and Reason are correct and Reason : In case of Be, no vacant d-orbitals are
Reason is the correct explanation of Assertion. present in its outermost shell.
EBD_7100
C-16 Topicwise AIIMS Solved Papers – CHEMISTRY

Type A : Multiple Choice Questions 11. (b)


12. (c)
1. (d) First ionisation potential is maximum for
13. (c) In a period the value of ionisation potential
hydrogen, as electron is withdrawn from
increases from left to right with breaks
the first orbital which is very near to
where the atoms have some what stable
nucleus.
configuration. In this case N has half filled
2. (b) Cl has high electron affinity.
stable orbital. Hence has highest ionisation
3. (c) Group IA and III A contain mostly metals.
energy. Thus the correct order is
Group VIII contains transition elements
B < C < O < N not B < C < N < O
which are metals. Group VII A contains
14. (c) As we move along the period, the atomic
mostly non-metals (F, Cl, Br).
size decreases due to increase in nuclear
4. (d) Elements having 1, 2 or 3 electrons in its
charge. Therefore, it is more difficult to
last shell act as metals.
remove electron from an atom. Hence the
32 = [Ar] 3 d10 4s2 p2
sequence of first ionization enthalpy in
34 = [Ar] 3 d10 4s2p4
decreasing order is
36 = [Ar] 3d10 4s2p6
F > N > C > Be > B
38 = [Ar] 3d10, 4s2p6, 5s2
But ionization enthalpy of boron is less as
1 compared to beryllium because first
5. (a) Hydration energy
Size electron in boron is to be removed from p-
In a group, size increases on going down orbital while in beryllium, it is to be removed
the group from top to bottom i.e., from s-orbital.
Rb + K+ Na + Li + As s-orbital is closer to nucleus in comparison
to p-orbital thus energy required to remove
Increasing order of hydration energy
an electron from s-orbital is greater.
Li + Na + K+ Rb + 15. (a) According to the law of triads the atomic
6. (d) Na (Z = 11; 1s , 2s 2p , 3s1 ) readily gives
2 2 6 wt of the middle element is arithmatic mean
one electron because the resulting Na+ of I and III.
(1s 2 2 s 2 2 p 6 ) has stable configuration . At wt of Br
At.wt of Cl At wt of I
Thus the first IE of Na is less. However, 2
removal of an electron from a stable (noble
Type B : Assertion Reason Questions
gas) configuration requires high energy
and thus IE2 of Na will be very high. 16. (d) The ionisation energy of N is more than
7. (c) The atomic no of this element is highest that of O because N has exactly half filled
which indicates that it will have highest valence p orbital.
2 2 3 2 2 4
energy. 7N 1s 2s p 8O 1s 2s p
8. (d) Electronic configuration of Cr (I) is The nuclear charge increases across a
1s2 2s2 2p6 3s2 3p6 3d5. period.
Electronic configuration of Fe(III) is 17. (c) Assertion is true but Reason is false.
1s2 2s2 2p6 3s2 3p6 3d5. Electron affinity is experimentally
Electronic configuration of Mn (II) is measurable while electronegativity is a
1s2 2s2 2p6 3s2 3p6 3d5. relative number.
So, all these have valence electrons in 3d- 18. (c) Ionization enthalpy is the energy required
subshell. So, option (d) is correct answer. to remove an electron from an isolated
9. (d) The ionisation potential decreases with gaseous atom in its ground state.
increase in atomic radii. 19. (a) Both assertion and reason are correct and
10. (c) reason is correct explanation of assertion.
Chemical Bonding and
4 Molecular Structure
TYPE A : MULTIPLE CHOICE QUESTIONS 9. Shape of O2F2 is similar to that of : [2004]
(a) C2F2 (b) H2O2
1. The strongest hydrogen bond is : [1997] (c) H2F2 (d) C2H2
(a) O – H...........S (b) S – H.............O 10. The ONO angle is maximum in : [2004]
(c) F – H............F (d) F – H.............O
(a) NO 3– (b) NO 2–
2. Shape of XeF4 molecule is: [1998]
(a) Pyramidal (b) Square planar (c) NO 2 (d) NO 2
(c) Triangular planar (d) Linear 11. Among the following molecules
3. The shape of NH3 molecule is : [2001]
(i) XeO 3 (ii) XeOF4 (iii) XeF6
(a) Tetrahedral (b) Trigonal planar
(c) Trigonal pyramidal (d) Linear Those having same number of lone pair of Xe
4. Which of the following molecule has highest are : [2005]
bond energy? [2002] (a) (i) and (ii) only (b) (i) and (iii) only
(a) F – F (b) N – N (c) (ii) and (ii) only (d) (i) , (ii) and (ii)
(c) C – C (d) O – O 12. Among the following, the species having square
5. Which of the following has the highest dipole planar geometry for central atom are
moment? [2002] (i) XeF4 (ii) SF4
(iii) [NiCl4]2– (iv) [PtCl4]2– [2006]
H CH3 H (a) (i) and (iv) (b) (i) and (ii) .
| | (c) (ii) and (iii) (d) (iii) and (iv)
(a) C=O (b) C C 13. In [Ag (CN2)]–, the number of bonds is :
| |
H CH 3 H (a) 2 (b) 3 [2006]
(c) 4 (d) 6
H CH3 CH3
14. Bond length order is [2007]
Cl
| | | | (a) O2 < O3 < O22– (b) O2 < O22– < O3
(c) C C (d) C C (c) O22– < O3 < O2 (d) O2 = O22– > O3.
| | | | 15. Sulphur reacts with chlorine in 1 : 2 ratio and
CH3 H CH3 Cl forms X. Hydrolysis of X gives a sulphur
6. The number of - and -bonds present in compound Y. What is the structure and
pent-4-en- 1-yne is [2002] hybridisation of anion of Y? [2008]
(a) 10, 3 (b) 4, 9 (a) tetrahedral, sp3 (b) linear, sp
(c) 3, 10 (d) 9, 4 (c) pyramidal, sp3 (d) trigonal planar, sp2
7. Which of the following are arranged in the 16. Which of the following molecule has highest
decreasing order of dipole moment? [2003] dipole moment? [2010]
(a) CH3Cl, CH3Br, CH3 F (a) B2H6 (b) NF3
(b) CH3Cl, CH3F, CH3Br (c) NH3 (d) BF3
(c) CH3Br, CH3Cl, CH3 F
17. The geometry of ClO3 according to valence
(d) CH3Br, CH3F, CH3Cl
8. The paramagnetic species is : [2003] shell electron pair repulsion (VSEPR) theory will
(a) KO2 (b) SiO2 be [2012]
(c) TiO2 (d) BaO2 (a) Planar triangle (b) Pyramidal
(c) Tetrahedral (d) Square planar
EBD_7100
C-18 Topicwise AIIMS Solved Papers – CHEMISTRY
18. N2 and O2 are converted into monocations, N2+ (c) If the Assertion is correct but Reason is incorrect.
and O2+ respectively. Which of the following (d) If both the Assertion and Reason are incorrect.
statements is wrong ? [2013] (e) If the Assertion is incorrect but the Reason is
(a) In N2+, the N—N bond weakens correct.
(b) In O2+, the O—O bond order increases
23. Assertion : Water is liquid but H2S is a gas.
(c) In O2+, paramagnetism decreases
Reason : Oxygen is paramagnetic. [1999]
(d) N2+ becomes diamagnetic
24. Assertion : Boiling and melting points of amides
19. Match List I and List II and pick out correct
are higher than corresponding acids.
matching codes from the given choices : [2015]
Reason : It is due to strong intermolecular
List I List II
hydrogen bonding in their molecules. [2002]
Compound Structure
25. Assertion : Sigma ( ) is a strong bond, while pi
A. ClF3 1. Square planar
( ) is a weak bond.
B: PCl5 2. Tetrahedral
Reason : Atoms rotate freely about pi ( ) bond.
C. IF5 3. Trigonal bipyramidal
[2002]
D. CCl4 4. Square pyramidal
26. Assertion : The O—O bond length in H2O2 is
E. XeF4 5. T-shaped
shorter than that of O2F2.
Codes
Reason : H2O2 is an ionic compound. [2003]
(a) A-5, B-4, C-3, D-2, E-1
27. Assertion : All F—S—F angle in SF4 are greater
(b) A-5, B-3, C-4, D-2, E-1
than 90° but less than 180°,
(c) A-5, B-3, C-4, D-1, E-2
Reason : The lone pair- bond pair repulsion is
(d) A-4, B-3, C-5, D-2, E-1
weaker than bond pair-bond pair repulsion.
20. XeO4 molecule is tetrahedral having : [2016]
[2004]
(a) Two p – d bonds
28. Assertion : SeCl4 does not have a tetrahedral
(b) One p – d bonds
structure.
(c) Four p – d bonds
Reason : Se in SeCl4 has two lone pairs. [2005]
(d) Three p – d bonds
29. Assertion : B2 molecule is diamagnetic.
21. Among the following species, identify the pair
Reason : The highest occupied molecular orbital
having same bond order CN–, O2– , NO+, CN+
is of type. [2005]
[2016]
30. Assertion : Ozone is powerful oxidising agent
(a) CN– and O2– (b) O2– and NO+
in comparison to O2.
(c) CN– and NO+ (d) CN– and CN+
Reason : Ozone is diamagnetic but O2 is
22. Which of the following substances has the least
paramagnetic. [2005]
covalent character ? [2017]
31. Assertion : Molecular nitrogen is less reactive
(a) Cl2O (b) NCl3
than molecular oxygen.
(c) PbCl2 (d) BaCl2
Reason : The bond length of N2 is shorter than
TYPE B : ASSERTION REASON QUESTIONS that of oxygen. [2007, 2006]
32. Assertion :Fluorine molecule has bond order
Directions for (Qs. 23-32) : These questions consist one.
of two statements, each printed as Assertion and Reason : The number of electrons in the
Reason. While answering these questions, you are antibonding molecular orbitals is two less than
required to choose any one of the following five that in bonding molecular orbitals. [2008]
responses. Directions for (Qs.33-38) : Each of these questions
(a) If both Assertion and Reason are correct and contains an Assertion followed by Reason. Read them
the Reason is a correct explanation of the carefully and answer the question on the basis of
Assertion. following options. You have to select the one that
(b) If both Assertion and Reason are correct but best describes the two statements.
Reason is not a correct explanation of the (a) If both Assertion and Reason are correct and
Assertion. Reason is the correct explanation of Assertion.
Chemical Bonding and Molecular Structure C-19

(b) If both Assertion and Reason are correct, but Reason : Electronegativity of the central atom
Reason is not the correct explanation of increases, bond angle decreases. [2013]
Assertion. 36. Assertion : LiCl is predominantly a covalent
(c) If Assertion is correct but Reason is incorrect. compound.
(d) If both the Assertion and Reason are incorrect. Reason : Electronegativity difference between
33. Assertion : Molecules of larger size have higher Li and Cl is too small. [2014]
polarizability. 37. Assertion : Cuprous ion (Cu+) has unpaired
Reason : Polarizability is observed only in those electrons while cupric ion (Cu++) does not.
molecules which has permanent dipole moment. Reason : Cuprous ion (Cu+ ) is colourless
[2010] whereas cupric ion (Cu++) is blue in the aqueous
34. Assertion : Bond angle of H2S is smaller than solution. [2014]
H2O. 38. Assertion : Lone pair-lone pair repulsive
Reason : Electronegativity of the central atom interactions are greater than lone pair-bond pair
increases, bond angle decreases. [2011] and bond pair-bond pair interactions.
35. Assertion : Bond angle of H2S is smaller than Reason : The space occupied by lone pair
H2O. electrons is more as compared to bond pair
electrons. [2016]
EBD_7100
C-20 Topicwise AIIMS Solved Papers – CHEMISTRY

Type A : Multiple Choice Questions Triple bond between C and C contains one
bond and two bonds. Double bond
1. (c) Higher the electronegativity of the other between C and C contains one bond and
atom, greater is the strength of hydrogen one bond.
bond. Strongest hydrogen bond is between
Total bonds = 10
H and F.
Total bonds = 3
F – H ------- F.
7. (b) Fluorine is most electronegative and Br is
2. (b) Shape of XeF4 molecule is square planar.
least electronegative. So CH3 F should have
It involves sp3d2 hybridisation.
highest dipole moment but as C – F bond
length is very small so inspite of greater
F F polarity in CH3F, it has less dipole moment
than CH3Cl.
Xe 8. (a) Species having unpaired electron or odd
F F number of electrons are paramagnetic while
species having paired or even number of
electrons are called diamagnetic.
3. (c) In NH 3 , N is sp3 hybridised ; N of NH3 has KO2 has 35 electrons — paramagnetic
a lone pair of electrons. The lone pair SiO2 has 30 electrons — diamagnetic
distorts the normal tetrahedral geometry TiO2 has 38 electrons — diamagnetic
due to lp–bp interaction to trigonal BaO2 has 72 electrons — diamagnetic
bipyramidal. 9. (b) Shape of O2F2 is similar to that of H2O2
4. (c) Greater the number of lone pairs present on because both of them are peroxides.
the bonded atoms, greater is the repulsive 10. (d) In NO2+ nitrogen is in sp hybridised state.
force between them and hence smaller the So, the molecule is linear. So ONO angle is
bond energy. 180° which is maximum among all other
molecules.
Molecule C – C N – N : O – O : : F – F :
11. (d) In all compounds, Xe will have same no. of
No. of lp 0 1 2 3 lone pair (one only)
5. (a) Greater the difference in electronegativity 12. (a) Square Planar Structure : XeF4 and [PtCl4]2–
between the two atoms, larger will be
13. (c) In one – C N, No. of bonds = 2
polarity and hence dipole moment. Thus
So in [Ag (CN2)]– , No. of bonds = 2 ×2 = 4
(a) has maximum dipole moment.
14. (a) Bond length is inversely proportional to
H
H3C H bond order. Bond orders of O2, O3 and O 22-
H–C=O C=C
H3C H are 2, 1.5 and 1 respectively. Hence the
(a) (b) correct sequence of bond lengths is
(C–O bond is (very less polar)
more polar) O2 < O3 < O22-
H CH3 Cl CH3 15. (c) S 2Cl 2 SCl4
C C x
SCl 4 4H 2 O S(OH) 4 4HCl
C C
H3C H H3C Cl S(OH)4 H 2SO3 H 2O
(c) (d) y
Symmetrical molecules (µ = 0) The anion of y is SO32– in which S is sp3
6. (a) HC C CH 2 CH CH 2 hybridised with one lone pair of electrons
Pent-4-en-1-yne on S, giving SO32– pyramidal shape.
Chemical Bonding and Molecular Structure C-21

16. (c) BF3 and B2H6 being non-polar molecules do One s and three p orbital undergo sp3
not show dipole moment. In NF3 , the hybridization. Four sp3 hybrid orbitals form
electronegative F pulls the electron toward four bonds with oxygen atoms. They are
itself due to which bond moments of the sp3 – p. Four p – d bonds are also formed
three N–F bonds is opposite to that of lone pair with oxygen atoms by th e unpair ed
while in NH3 bond moments of the N–H electrons.
bonds are in same direction to that of lone pair. 21. (c) M.O. electronic configuration of CN– is
.. .. 1s2 *1s2 s2 *2s2 2px2 2py2 2pz2

N 10 – 4
N B.O. 3
2
F F H
M.O. electronic configuration of O2– is
F H H
1s2 *1s2 2s2 *2s2 2pz2 2px2
NF3 NH3
2py2 *2px2 *2py1
Hence, bond moments add up in NH3
resulting in highest dipole moment among 10 – 7
B.O. 1.5
the given options. 2
17. (b) Hybridisation is sp3 and shape pyramidal M.O. electronic configuration of CN+
.. – .. – 1s2 *1s2 2s2 *2s2 2px2 2py2 2pz1
Cl Cl
: 9–4
:O: :O: or :O: O
: B.O. 2.5
:
:

2
:

:O: :O:
:

18. (d) N2+ is paramagnetic M.O. electronic configuration of NO+ is


1s2, *1s2, *2s2, 2p2x = 2p2y , 2p1z 1s2 *1s2 2s2 *2s2 2pz2 2px2 2py2
10 – 4
19. (b) List I List II B.O. 3
2
Compound Structure
CN– and NO+ have bond order equal to 3
(A) ClF3 T-shaped
22. (d) According to Fajan's rule :
(B) PCl5 Trigonal bipyramidal
(C) IF5 Square pyramidal 1
Covalent character
(D) CCl4 Tetrahedral size of cation
(E) XeF4 Square planar size of anion
20. (c) Xenon undergo sp3 hybridization. Among the given species order of size of
cations
N3+ < O2+ < Pb2+ < Ba2+
5s 5p
(ground order of size of anions O2– > Cl–.
state) Hence the order of covalent character is
5d
NCl3 Cl2O PbCl2 BaCl2
BaCl2 is least covalent in nature.
5s 5p
(third excited Type B : Assertion Reason Questions
state)
5d 23. (b) Water is liquid but H2 S is gas due to
In the fourth excited state xenon atom, has hydrogen bond in water. Oxygen is
8 unpaired electrons paramagnetic.
24. (a)
25. (c) Sigma ( ) bond is formed by axial overlap
5s 5p of atomic orbitals while pi ( ) bond is formed
by lateral overlap. Since axial overlapping
5d takes place to a greater extent than the
EBD_7100
C-22 Topicwise AIIMS Solved Papers – CHEMISTRY
lateral overlapping, former ( ) bond is orbitals. Thus, reason is true and it is also
stronger than pi bond. Atoms attached to the correct explanation of assertion
doubly bonded atom can't rotate freely because
around the double bond. Nb Na 10 8
26. (d) H2O2 is a covalent compound. In O2F2, Bond order = = 1
2 2
O – O bond is shorter than in H2O2 due to
higher electronegativity of F. 33. (c) Assertion is true but Reason is false.
27. (d) SF4 has see-saw type structure in which As the size of the atom increases the
influence of the electric dipole also
bond angles are different between different
increases.
S—F atoms. It has non-planar structure.
34. (c) Bond angle of H2S (92°) < H2O (104°31). As
101° F
F the electronegativity of the central atom
S 89° decreases, bond angle decreases. In the
F F present case, S is less electronegative than
177°
According to VSEPR theory p – p oxygen. Thus bond pairs in H2S are more
away from the central atom than in H2O and
repulsion > p –bp > bp –bp repulsion thus repulsive forces between bond pairs
28. (c) SeCl4 has distorted trigonal pyramidal are smaller producing smaller bond angle.
geometry; here Se has only one lone 35. (c) Bond angle of H2S (92°) < H2O (104°31).
pair of electrons hence it has sp 3 d As the electronegativity of the central atom
hybridisation. decreases, bond angle decreases. In the
29. (d) B2 molecule has no of electrons = 10 present case, S is less electronegative
than oxygen. Thus bond pairs in H2S are
Molecular orbital configuration more away from the central atom than in
(1s )2 *(1s )2 (2 s)2 *(2s )2 2 p1x 2 py1 H2O and thus repulsive forces between
bond pairs are smaller producing smaller
Due to unpaired electron, it is paramagnetic. bond angle.
The highest occupied MO is of -type. 36. (c) LiCl is a covalent compound. Due to the
30. (b) Ozone is a powerful oxidising agent large size of the anion (Cl–) its effective
because it is unstable and breaks into nuclear charge lessens and its valence shell
oxygen as it has higher energy content than is held less tightly towards its nucleus.
oxygen. Here, assertion is correct but reason is
O3 O2 + O incorrect.
It is also true that O3 is diamagnetic, while 37. (d)
O2 is paramagnetic.
31. (a) Nitrogen molecule has triple bond, whereas 38. (a) While the lone pairs are localised on the
oxygen has double bond. N-N Bond length central atom, each bonded pair is shared
of N2 is shorter. Hence, it is difficult to break between two atoms. As a result, the lone
the triple bond of N2. Hence N2 is less pair electrons in a molecule occupy more
reactive. Both A and R are true. space as compared to the bonding pairs of
32. (a) MO electronic configuration of F2 molecule. electrons. This results in greater repulsion
1s 2 , *1s 2 , 2s 2 , *2s 2 , 2p x 2 , between lone pairs of electrons as
2py2 2pz2., *2py2 *2pz2 compared to the lone pair -bond pair and
Thus there are 10 electrons in bonding bond pair - bond pair repulsions.
orbitals and 8 electrons in antibonding Hence (b) is the correct option.
5 States of Matter

TYPE A : MULTIPLE CHOICE QUESTIONS (a) 0.6 atm. (b) 1.2 atm
1. A gas occupies a volume of 300 cc at 27°C and (c) 2.4 atm (d) 3.6 atm.
620 mm pressure. The volume of gas at 47°C 8. Which equation shows correct form of Berthelot
and 640 mm pressure is : [1997] equation. [2000]
(a) 260 cc (b) 310 cc
a
(c) 390 cc (d) 450 cc (a) P (V b) RT
2. The compressibility factor of an ideal gas is : T (V C )2
(a) 0 (b) 2 [1997] a
(c) 1 (d) 4 (b) P (V b) RT
T (V C )2
3. A gas cylinder containing cooling gas can
withstand a pressure of 14.9 atmosphere. The a
(c) P (V b) RT
pressure gauge of cylinder indicates 12 TV 2
atmosphere at 27°C. Due to sudden fire in the
a
building the temperature starts rising. The (d) P (V b) RT
temperature at which cylinder explodes is : TV 2
(a) 87.5°C (b) 99.5°C [1997] 9. If P is pressure and is density of a gas, then P
(c) 115.5°C (d) 135.5°C and are related as [2002]
4. Van der Waal’s equation [1998] (a) P (b) P (1/ )
(c) P 2 (d) P (1/ 2)
a
P (V b ) nRT is applicable for : 10. Dominance of strong repulsive forces among the
V2 molecules of the gas (Z = compressibility factor):
(a) Ideal gas (b) Non-ideal gas [2006]
(c) Both (a) and (b) (d) None of these (a) Depends on Z and indicated by Z = 1
5. For the diffusion of a gas at pressure P, the rate (b) Depends on Z and indicated by Z > 1
of diffusion is expressed by: [1998] (c) Depends on Z and indicated by Z < 1
1 P (d) Is independent of Z.
(a) r (b) r
M M 11. In P versus V graph, the horizontal line is found
in which ______ exits. [2007]
M P
(c) r (d) r (a) Gas
P M
(b) Liquid
6. The transport of matter in the absence of bulk
(c) Equilibrium between gas and liquid
flow is known as: [1999]
(d) Super critical temperature.
(a) Diffusion (b) Transfusion
(c) Translation (d) Rotation 12. Critical temperatures for A, B, C and D gases are
25°C, 10°C, –80°C and 15°C respectively. Which
7. At 298 K, equal volumes of SO2, CH4 and O2 are
gas will be liquefied more easily? [2007]
mixed in empty container. The total pressure
(a) A (b) B
exerted is 2.1 atm. The partial pressure of CH4 in
mixture is : [2000] (c) C (d) D
EBD_7100
C-24 Topicwise AIIMS Solved Papers – CHEMISTRY
13. The volume-temperature graphs of a given mass 18. A bottle of dry ammonia and a bottle of dry
of an ideal gas at constant pressure are shown hydrogen chloride connected through a long
below. [2008] tube are opened simultaneously at both ends
p2 p
3 the white ammonium chloride ring first formed
V p1
will be [2014]
(a) at the centre of the tube.
(b) near the hydrogen chloride bottle.
(c) near the ammonia bottle.
O 273 T(K)
(d) throughout the length of the tube.
What is the correct order of pressures ?
(a) p1 > p3 > p2 (b) p1 > p2 > p3 19. The gas with the highest critical temperature is
(c) p2 > p3 > p1 (d) p2 > p1 > p3 [2014]
14. The inversion temperature Ti (K) of hydrogen is (a) H2 (b) He
(given van der Waal’s constants a and b are (c) N2 (d) CO2
0.244 atm L 2 mol –2 and 0.027 L mol –1 20. Cyclopropane and oxygen at partial pressures
respectively) 170 torr and 570 torr respectively are mixed in a
(a) 440 (b) 220 [2010] gas cylinder. What is the ratio of the number of
(c) 110 (d) 330 moles of cyclopropane to the number of moles
15. Amongst the following statements, the correct of oxygen (nC3H6/nO2)? [2015]
one is : [2011] 170 42
(a) The gas can not be compressed below the (a) 0.39
570 32
critical temperature.
(b) Below critical temperature, thermal motion 170 170 570
(b) 0.19
of the molecules is slow enough for the 42 42 32
intermolecular forces to come into play
leading to condensation of the gas. 170
(c) 0.23
(c) At critical temperature liquid and gaseous 740
phase can be distinguished.
170
(d) An ideal gas has a characteristic critical (d) 0.30
570
temperature.
21. When a sample of gas is compressed at constant
16. X ml of H2 gas effuse through a hole in a
temperature from 15 atm to 60 atm, its volume
container in 5 seconds. The time taken for the changes from 76 cm3 to 20.5 cm3. Which of the
effusion of the same volme of the gas specified following statements are possible exlanations of
below under identical conditions is this behaviour?
[2012] (1) The gas behaves non-ideally
(a) 10 seconds : He (b) 20 seconds : O2 (2) The gas dimerises
(3) The gas is adsorbed into the vessel walls
(c) 25 seconds : CO (d) 55 seconds : CO 2 [2016]
(a) 1, 2 and 3 (b) 1 and 2 only
17. The rate of diffusion of SO2, CO2 , PCl3 and SO3
(c) 2 and 3 only (d) 1 only
are in the following order [2013]
22. Pure hydrogen sulphide is stored in a tank of 100
(a) PCl3 > SO3 > SO2 >CO2 litre capacity at 20°C and 2 atm pressure. The
(b) CO2 > SO2 > PCl3 > SO3 mass of the gas will be [2017]
(c) SO2 > SO3 > PCl3 > CO2 (a) 34 g (b) 340 g
(d) CO > SO > SO > PCl (c) 282.68 g (d) 28.24 g
2 2 3 3
States of Matter C-25

TYPE B : ASSERTION REASON QUESTIONS Directions for (Qs.29-33) : Each of these questions
contains an Assertion followed by Reason. Read them
Directions for (Qs. 23-28) : These questions consist carefully and answer the question on the basis of
of two statements, each printed as Assertion and following options. You have to select the one that
Reason. While answering these questions, you are best describes the two statements.
required to choose any one of the following five (a) If both Assertion and Reason are correct and
responses. Reason is the correct explanation of Assertion.
(a) If both Assertion and Reason are correct and (b) If both Assertion and Reason are correct, but
the Reason is a correct explanation of the Reason is not the correct explanation of
Assertion. Assertion.
(b) If both Assertion and Reason are correct but (c) If Assertion is correct but Reason is incorrect.
Reason is not a correct explanation of the (d) If both the Assertion and Reason are incorrect.
Assertion. 29. Assertion : The molecules of the dissolved gas
(c) If the Assertion is correct but Reason is incorrect. present in a liquid gain kinetic energy as
(d) If both the Assertion and Reason are incorrect. temperature is raised.
(e) If the Assertion is incorrect but the Reason is Reason : Gases tends to be more soluble in
correct. liquids as the temperature is raised. [2009]
23. Assertion : Ice melts faster at high altitude. 30. Assertion : Greater the value of van der Waal's
Reason : At high altitude, atmospheric pressure constant 'a' greater is the liquefaction of gas.
is high. [1997] Reason : 'a' indirectly measures the magnitude
24. Assertion : Gases do not settle to the bottom of of attractive forces between the molecules.
container. [2014]
Reason : Gases have high kinetic energy 31. Assertion : Compressibility factor (Z) for
[1997] non-ideal gases can be greater than 1. [2015]
25. Assertion : Wet air is heavier than dry air. Reason : Non-ideal gases always exert higher
Reason : The density of dry air is more than pressure than expected.
density of water. [1999]
32. Assertion : Gases do not liquefy above their
26. Assertion : Use of pressure cooker reduces
critical temperature, even on applying high
cooking time.
pressure.
Reason : At higher pressure cooking occurs
faster. [2000] Reason : Above critical temperature, the
27. Assertion : All molecules in a gas have same molecular speed is high and intermolecular
speed. attractions cannot hold the molecules together
Reason : Gas contains molecules of different size because they escape because of high speed.
and shape. [2001] [2016]
28. Assertion : Compressibility factor for hydrogen 33. Assertion : At critical temperature liquid passes
varies with pressure with positive slope at all into gaseous state imperceptibly and
pressures. continuously.
Reason : Even at low pressure, repulsive forces Reason : The density of liquid and gaseous
dominate hydrogen gas. [2005] phase is equal to critical temperature.
EBD_7100
C-26 Topicwise AIIMS Solved Papers – CHEMISTRY

Type A : Multiple Choice Questions Total number of moles of the three gases
PV P2V2 x x x 7x
1 1
1. (b) From 64 16 32 64
T1 T2
Partial pressure exerted by a gas in the
V1 640 620 300 mixture of non-reacting gases (p) is given
(273 47) (273 27) by
620 300 320 moles of that gas
V1 = 310 cc p= Total pressure
640 300 total moles
2. (c) Compressibility factor (Z) is a convenient
method of showing deviation of real gases x 64
p CH 2.1 1.2 atm
from an ideal gas 4 16 7 x
PV a
Z 8. (c) P (V nb) RT is
nRT TV 2
For ideal gas, PV nRT ; Z=1 Berthelot equation. The volume correction
For real gases, PV nRT ; Z 1 is same as in van der Waal's equation, but
When, Z 1 , it refers positive deviation the pressure correction is different. He
i.e., gas is less compressible than ideal gas. a
introduced the pressure correction as
Z < 1, it refers negative deviation, i.e., gas
a TV 2
is more compressible than ideal gas. in place of 2
P P2 V
3. (b) From Charle's law, 1 1 m 1
T1 T2 9. (a) P and ;
V V V m
12 14.9
So, P i.e. Pressure
(273 27) T2 m
14.9 300 10. (b) Repulsive force will decrease the
T2 372.5 K compressibility factor i.e. so, value of
12
= 372.5 – 273 = 99.5°C Z > 1as
4. (b) Van der Waal’s equation is applicable for PV
Z
real (non-ideal) gases. RT
5. (a) According to Graham's law of diffusion or Due to repulsion value of PV will be greater
effusion "under similar conditions of than RT so Z > 1.
temperature and pressure, rate of diffusion 11. (c) Generally most of real gases show the same
is inversely proportional to square root of type of isotherm.
1
molecular weight" r .
M
6. (a) Diffusion is the process by which matter is d
transported in small quantities. T4
T3
7. (b) Let the wt. of each gas mixed = x g
x Tc
mole of SO2 =
64
x
mole of CH4 = c b T2
16 aT
1
x
mole of O2 = Volume V
32
States of Matter C-27
ab represents the gaseous state, line bc 1
which is horizontal line shows liquid 18. (b) Rate of diffusion
Molecular mass
and vapour equilibrium. Pressure
corresponding to the line bc is known as Molecular mass of HCl > Molecular mass
vapour pressure of liquid. Line cd of NH3
represents liquid state. HCl diffuses at slower rate and white
12. (a) Critical temperature of a gas is given by ammonium chloride is first formed near HCl
Þ a TC
8a bottle.
TC
27 Rb
19. (d) CO2 has highest critical temperature of
where a is measure of vander Waal's forces
304.2 K
of attraction. Higher the critical temperature
of a gas greater the intermolecular forces of 20. (d) By ideal gas equation
attraction between the molecules of gas and PV
1 n1RT
easily the gas can be liquefied. Hence gas
A whose critical temperature (25°C) is n1 P1 and n2 P2
highest among all the given options will be n1 P1 n1 170
liquified more easily. n2 P2 n2 570
13. (a) From the graph we can see the correct order
21. (d) Given, P1 = 15 atm, P2 = 60 atm
of pressures p1 > p3 > p2
14. (b) Gases become cooler during Joule V1 = 76 cm3, V2 = 20.5 cm3.
Thomson’s expansion only if they are If the gas is an ideal gas, then according to
below a certain temperature known as Boyle's law, it must follow the equation,
inversion temperature (Ti). The inversion P1V1 = P2V2
temperature is characteristic of each gas and P1 × V1 = 15 × 76 = 1140
is given by P2 × V2 = 60 × 20.5 = 1230
2a P1V1 P2 V2
Ti
bR
The gas behaves non-ideally.
where R is gas constant
Given a = 0.244 atm L2 mol–2 The given information is not sufficient to
b = 0.027 L mol–1 comment on other statements.
R = 0.0821 L atm deg–1 mol–1 PV m
22. (c) n
2 0.244 RT M
Ti 220 K
0.027 0.0821 MPV 34 2 100
15. (b) m 282.68gm
RT 0.082 293
16. (b) For effusion of same volume,
Type B : Assertion Reason Questions
t1 M1 t1 t2
t2 M2 M1 M2 23. (d) Ice does not melt faster at high altitude
This is clearly seen from the options that because melting is favoured at high
pressure, whereas atmospheric pressure
t
the ratio of is same for H2 and O2. decreases as we go higher. So, assertion
M
and reason both are false.
5 20 5 24. (a) Gases do not settle to the bottom because
of its kinetic energy. They are always in
2 32 2
motion. Because of small mass, the effect
1 of gravity on them is negative.
17. (d) Rate . The smaller the value of M
M 25. (e) Wet air is lighter than dry air because
the more is the rate of diffusion density of air is more than water.
EBD_7100
C-28 Topicwise AIIMS Solved Papers – CHEMISTRY
26. (a) Use of pressure cooker reduces cooking tend to be less soluble in liquids at higher
time because increase of pressure increases temperature.
b.p and so cooking occurs faster. 30. (a) Both assertion and reason are trueand
27. (d) All molecules of a gas are identical in shape reason is the correct explanation of assertion.
and size, but have different energies due to Considering the attractive force pressure
which they have different speeds. in ideal gas equation (PV = nRT) is corrected
28. (a) In case of hydrogen, Z increases with
pressure. At 273K , Z > 1. which shows an2
by introducing a factor of where a is
that it is difficult to compress the gas as V2
compared to ideal gas. In this case repulsive van der waal constant.
forces dominate. 31. (c) Z can be greater than 1 or less than 1. Non
29. (c) When the temperature is raised, the - ideal gases exert less pressure than
molecules of the dissolved gas present in a expected due to backward pull by other
liquid gain kinetic energy. Higher kinetic molecules.
energy of the gas molecules make them to 32. (a)
escape from its solution. That is why, gases 33. (a)
6 Thermodynamics

TYPE A : MULTIPLE CHOICE QUESTIONS 7. Internal energy does not include: [1999]
(a) Rotational energy
1. The enthalpy change of a reaction does not
(b) Nuclear energy
depend on : [1997]
(a) initial and final enthalpy change of reaction (c) Vibrational energy
(b) state of reactants and products (d) Energy due to gravitational pull
(c) different intermediate reactions 8. A gas expands isothermally against a constant
(d) nature of reactants and products external pressure of 1 atm from a volume of 10
2. S O 2 SO2 x kcal .....(1) dm3 to a volume of 20 dm 3. It absorbs 300 J of
1 thermal energy from its surrounding.
SO2 O2 SO3 y kcal .....(2) The U is:
2
The heat of formation of SO3 in the above (a) – 312 J (b) + 123 J [1999]
reaction is [1997] (c) – 213 J (d) + 231 J
(a) (x + y) (b) (x – y) 9. During isothermal expansion of one mole of an
(c) (2x – y) (d) (2x + y) ideal gas from 10 atm to 1atm at 273 K, the work
3. At a constant volume the specific heat of a gas done is [gas constant = 2] : [2000]
is 0.075 and its molecular weight is 40. The gas (a) – 895.8 cal (b) – 1172.6 cal
is: [1998]
(c) – 1257.43 cal (d) – 1499.6 cal
(a) Monoatomic (b) Diatomic
10. One mole of an ideal gas for which Cv = (3/2)R is
(c) Triatomic (d) None of the above
heated reversibly at a constant pressure of 1
4. The heat of combustion of yellow phosphorous
is – 9.91 kJ and of red phosphorous is – 8.78 kJ. atm from 25°C to 100°C. The H is : [2000]
The heat of transition of yellow phosphorus to (a) 3.775 cal (b) 37.256 cal
red phosphorus is : [1998] (c) 372.56 cal (d) 3725.6 cal
(a) – 9.91 kJ (b) – 8.78 kJ 11. Enthalpy of neutralisation of CH 3COOH by
(c) – 9.34 kJ (d) – 1.13 kJ
5. Combustion of glucose takes place according NaOH is –50.6 kJ/mol and the heat of
to the equation: neutralisation of a strong acid with NaOH is
–55.9 kJ/mol. The value of H for the ionisation
C6 H12O6 6O 2 6CO2 6H 2O,
of CH 3COOH is : [2000]
H 72 k cal
(a) 3.5 kJ/mol (b) 4.6 kJ/mol
The energy required for production of 1.6 g of
glucose is [molecular mass of glucose is 180 g] (c) 5.3 kJ/mol (d) 6.4 kJ/mol
(a) 0.064 k cal (b) 0.64 k cal [1998] 12. The internal energy of a substance : [2001]
(c) 6.4 k cal (d) 64 k cal (a) increases with increase in temperature
6. For a spontaneous process, entropy: [1998] (b) decreases with increase in temperature
(a) Increases (b) Decreases (c) remains unaffected with temperature
(c) Unchanged (d) Not clear (d) can be calculated by the reaction, E = mc2
EBD_7100
C-30 Topicwise AIIMS Solved Papers – CHEMISTRY

13. The heat of reaction for : (a) – 6.2 kJ (b) –37.4 kJ


C10H 8 (s) 12O 2 (g) 10CO 2 (g) + 4H2O(l) (c) – 35.5 kJ (d) – 20.0 kJ
21. H f (298K) of methanol is given by the
at constant volume is –1228.2 kcal at 25°C. The
heat of reaction at constant pressure and same chemical equation : [2005]
temperature is : [2001] 1
(a) CH 4 (g ) O 2( g ) CH 3 OH (g )
(a) –1228.2 k cal (b) –1229.3 k cal 2
(c) –1232.9 k cal (d) –1242.6 k cal 1
14. Two moles of an ideal gas are compressed (b) C(graphite) + O 2(g) +2H 2(g) CH3OH ( )
2
isothermally (100°C) and reversibly from a
pressure of 10 atm to 25 atm, then the free energy (c) C(diamond) 1 O 2(g) +2H 2(g) CH3OH (l )
change is : [2001] 2
(a) + 15.482 kJ (b) + 10.462 kJ (d) CO ( g) 2H 2( g) CH 3OH (l)
(c) + 5.684 kJ (d) + 3.364 kJ
22. For the reaction of one mole of zinc dust with
15. In the exothermic reaction, the enthalpy of
reaction is always : [2001] one mole of sulphuric acid in a bomb calorimeter,
(a) zero (b) positive U and w corresponds to [2005]
(c) negative (d) none of these (a) U < 0, w = 0 (b) U < 0, w < 0
16. The heat of neutralization of a strong base and a (c) U > 0, w = 0 (d) U > 0, w > 0
strong acid is 57 kJ. The heat released when 0.5 23. For a spontaneous process, the correct statement
moles of HNO3 solution is added to 0.20 moles is : [2006]
of NaOH solution, is [2002]
(a) Entropy of the system always increases
(a) 11.4 kJ (b) 34.7 kJ
(c) 23.5 kJ (d) 58.8 kJ (b) Free energy of the system always increases
17. One gram sample of NH4NO3 is decomposed in (c) Total entropy change is always negative
a bomb calorimeter, the temperature of the (d) Total entropy change is always positive
calorimeter increases by 6.12 K. The heat 24. For a phase change,
capacity of the system is 1.23 kJ/g/deg. What is H 2O( ) H 2O(s ) [2006]
the molar heat of decomposition for NH4NO3? 0 C,1bar
[2003] (a) G 0 (b) S 0
(a) –7.53 kJ/mol (b) –398.1 kJ/mol
(c) H 0 (d) U 0
(c) –16.1 kJ/mol (d) –602 kJ/mol
18. Which one of the following has S greater 25. The enthalpy change ( H) for the reaction,
than zero? [2003] N 2 (g) 3H 2 (g) 2NH 3 (g) is –92.38 kJ at
(a) CaO(s) + CO2(g) CaCO3(s)
298 K. The internal energy change U at 298 K
(b) NaCl(aq) NaCl(s) is [2006]
(c) NaNO3(s) Na+ (aq) + NO3–(aq) (a) – 92.38kJ (b) – 87.42 k.J
(d) N2(g) + 3H2(g) 2NH3 (g) (c) –97.34kJ (d) – 89.9kJ
19. Which of the following is arranged in the 26. Ssurr for an exothermic reaction is [2007]
increasing order of enthalpy of vaporisation?
(a) always positive
[2004]
(b) always negative
(a) NH3, PH3, AsH3 (b) AsH3, PH3, NH3
(c) zero
(c) NH3, AsH3, PH3 (d) PH3, AsH3, NH3
(d) may be positive or negative.
20. How much energy is released when 6 moles of 27. Calculate change in internal energy if [2007]
octane is burnt in air ? Given H of for CO2(g), H = – 92.2 kJ, P = 40 atm and H = –1L.
(a) – 42 kJ (b) – 88 kJ
H2O (g) and C8H18 (l) respectively are –490, (c) + 88 kJ (d) + 42 kJ.
–240 and +160 J/mol. [2004]
Thermodynamics C-31

28. H fusion of a substance is 'x' and H vap is 'y', (a) 100 K (b) 150 K
(c) 195 K (d) 255 K
then Hsublimation will be [2007] 37. Enthalpy of formation of HF and HCl are –161 kJ
(a) x + y (b) x – y and –92 kJ respectively. Which of the following
(c) x/y (d) y/x. statements is incorrect? [2010]
29. For a reaction to be spontaneous at all (a) HCl is more stable than HF
temperatures [2008]
(b) HF and HCl are exothermic compounds
(a) G – ve, H + ve and S +ve
(c) The affinity of fluorine to hydrogen is
(b) G + ve, H – ve and S +ve
(c) G – ve, H –ve and S –ve greater than the affinity of chlorine to
(d) G – ve, H –ve and S +ve hydrogen
30. What will be the heat of formation of methane, if (d) HF is more stable than HCl
the heat of combustion of carbon is '–x' kJ, heat 38. Which of the following processes takes place
of formation of water is '–y' kJ and heat of with decrease of entropy? [2011]
combustion of methane is 'z' kJ ? [2008] (a) Solid gas
(a) (–x – y + z) kJ (b) (–z – x + 2y) kJ (b) sugar + water solution
(c) (–x – 2y – z) kJ (d) (–x – 2y + z) kJ
(c) NH3(g) + HCl(g) NH4Cl(s)
31. When a solid melts reversibly [2009]
(a) H decreases (b) G increases (d) A(g) + B(g) mixture
(c) E decreases (d) S increases 39. Enthalpy of combustion of CH4, C2H6 and C3H8
32. 6 moles of an ideal gas expand isothermally and are – 210.8, – 368.4 and – 526.2 k cal mol–1
reversibly from a volume of 1 litre to a volume of respectively. Enthalpy of combustion of hexane
10 litres at 27ºC. What is the maximum work can be predicted as [2011]
done? [2009] (a) – 840 k cal mol–1 (b) – 684 k cal mol–1
(a) 47 kJ (b) 100 kJ (c) – 1000 k cal mol–1 (d) none of these
(c) 0 (d) 34.465 kJ 40. AB, A2 and B2 are diatomic molecules. If the
33. The standard enthalpy of combustion at 25°C of bond enthalpies of A2, AB and B2 are in the
hydrogen, cyclohexene (C6H10) and cyclohexane ratio 1:1 :0.5 and enthalpy of formation of AB
(C6H12) are –241, – 3800 and – 3920 kJ/mole
from A2 and B2 is –100 kJ mol–1 . What is the
respectively. Calculate the heat of hydro-
bond energy of A2 : [2012]
genation of cyclohexene. [2009]
(a) – 111 kJ/mol (b) – 121 kJ/mol (a) 200 kJ mol–1 (b) 100 kJ mol–1
(c) – 118 kJ/mol (d) – 128 kJ/mol (c) 300 kJ mol–1 (d) 400 kJ mol–1
34. One mole of an ideal gas at 300 K is expanded 41. Which of the following condition favours the
isothermally from an initial volume of 1 litre to reduction of a metal oxide to metal? [2012]
10 litres. The value of E for this process is (a) H = +ve, T S = + ve at low temperature
(R = 2 cal mol–1 K–1) [2010] (b) H = +ve, T S = – ve at any temperature
(a) 163.7 cal (b) zero
(c) H = –ve, T S = – ve at high temperature
(c) 138.1 cal (d) 9 litre atm
35. For the reaction, 2Cl(g) — Cl2(g), the signs of (d) H = –ve, T S = + ve at any temperature
H and S respectively, are: [2010] 42. The Hfo for CO2 (g), CO(g) and H2 O (g)
(a) +, – (b) +, + are –393.5, –110.5 and –241.8 kJ mol–1
(c) –, – (d) –, + respectively. The standard enthalpy change
36. One mole of an ideal gas is allowed to expand (in kJ) for the reaction [2013]
reversibly and adiabatically from a temperature CO2 (g) H 2 (g) CO (g) H 2O (g) is
of 27°C. If the work done during the process is
(a) 524.1 (b) 41.2
3 kJ, then final temperature of the gas is
(c) –262.5 (d) – 41.2
(Cv = 20 J/K) [2010]
EBD_7100
C-32 Topicwise AIIMS Solved Papers – CHEMISTRY

43. What is the enthalpy change for, 49. The H° for CO2(g) , CO(g) and H2O(g) are
f
2H 2O2 (l ) 2 H 2O(l ) O 2 ( g ) if heat of –393.5, –110.5 and –241.8 kJ/mol respectively,
formation of H2O2 (l) and H2O (l) are –188 and the standard enthalpy change (in kJ) for the
–286 kJ/mol respectively? [2014] reaction
(a) –196 kJ/mol (b) + 948 kJ/mol CO2(g) + H2(g) CO(g) + H2O(g) is : [2017]
(c) + 196 kJ/mol (d) –948 kJ/mol (a) 524.1 (b) 41.2
44. Following reaction occurrs in an automobile (c) – 262.5 (d) – 41.2
2C8 H18 g 25O 2 g 16CO 2 g 18H 2O g . TYPE B : ASSERTION REASON QUESTIONS
The sign of H, S and G would be [2015] Directions for (Qs. 50-60) : These questions consist
(a) +, –, + (b) –, +, – of two statements, each printed as Assertion and
(c) –, +, + (d) +, +, – Reason. While answering these questions, you are
required to choose any one of the following five
45. The enthalpy changes for the following
responses.
processes are listed below : [2015]
(a) If both Assertion and Reason are correct and
Cl2(g) 2Cl(g), 242.3 kJ mol –1
the Reason is a correct explanation of the
I2(g) 2I(g), 151.0 kJ mol–1 Assertion.
ICl(g) I(g) + Cl(g), 211.3 kJ mol–1 (b) If both Assertion and Reason are correct but
I2(s) I2(g), 62.76 kJ mol–1 Reason is not a correct explanation of the
Assertion.
Given that the standard states for iodine and
(c) If the Assertion is correct but Reason is incorrect.
chlorine are I2(s) and Cl2(g), the standard enthalpy
(d) If both the Assertion and Reason are incorrect.
of formation for ICl(g) is:
(e) If the Assertion is incorrect but the Reason is
(a) +16.8 kJ mol–1 (b) +244.8 kJ mol–1 correct.
(c) –14.6 kJ mol–1 (d) –16.8 kJ mol–1 50. Assertion : Ionic reactions are not instantaneous.
46. Choose the reaction in which H is not equal to Reason : Oppositely charged ions exert strong
U? [2016] forces. [1997]
(a) C(graphite) + O2(g) CO2(g) 51. Assertion : Entropy of ice is less than water.
Reason : Ice has cage like structure. [2000]
(b) C2H4(g) + H2(g) C2H6(g) 52. Assertion : During an adiabatic process, heat
(c) 2C(graphite) + H2(g) C2H2(g) energy is not exchanged between system and
its surroundings.
(d) H2(g) + I2(g) 2HI(g) Reason : The temperature of a gas increases
47. The standard enthalpies of combustion of when it undergoes an adiabatic expansion.
C6H6(l), C(graphite) and H2(g) are respectively [2002]
53. Assertion : Mass and volume are extensive
– 3270 kJ mol–1, – 394 kJ mol–1 and – 286 kJ mol–1.
properties.
What is the standard enthalpy of formation of Reason : Mass / volume is also an extensive
C6H6(l) in kJ mol–1 ? [2016] parameter. [2002]
(a) – 48 (b) + 48 54. Assertion : Absolute values of internal energy
(c) – 480 (d) + 480 of substances cannot be determined.
48. The molar heat capacity of water at constant Reason : It is impossible to determine exact
pressure is 75 JK–1 mol–1. When 1kJ of heat is values of constituent energies of the substances.
supplied to 100 g of water, which is free to [2002]
expand, the increase in temperature of water is 55. Assertion : The increase in internal energy ( E)
[2017] for the vaporization of one mole of water at 1 atm
(a) 6.6 K (b) 1.2 K and 373 K is zero.
(c) 2.4 K (d) 4.8 K Reason : For all isothermal processes, E = 0
[2003]
Thermodynamics C-33

56. Assertion : H and E are almost same for the (a) If both Assertion and Reason are correct and
reaction N 2 (g) O 2 (g) 2NO(g) Reason is the correct explanation of Assertion.
(b) If both Assertion and Reason are correct, but
Reason : All reactants and products are gases. Reason is not the correct explanation of
[2003] Assertion.
57. Assertion : Molar enthalpy of vaporisation of (c) If Assertion is correct but Reason is incorrect.
water is different from ethanol. (d) If both the Assertion and Reason are incorrect.
Reason : Water is more polar than ethanol. 61. Assertion : For an isothermal reversible process
[2004] Q = –W i.e. work done by the system equals the
58. Assertion : Water in liquid state is more stable heat absorbed by the system.
than ice at room temperature. Reason : Enthalpy change ( H) is zero for
Reason :Water in liquid form has higher entropy isothermal process. [2011]
than ice. [2006] 62. Assertion : Many endothermic reactions that
59. Assertion : When a salt such as NaCl dissolves, are not spontaneous at room temperature
the Na+ and Cl– ions leaving the crystal lattice become spontaneous at high temperature.
acquire far greater freedom. Reason : Entropy of the system increases with
Reason : In thermodyanamic terms, the formation increase in temperature. [2012]
of solution occurs with a favourable change in 63. Assertion : For an isothermal reversible process
free energy, i.e., H has a high positive value Q = –W i.e. work done by the system equals the
heat absorbed by the system. [2013]
and T S a low negative value. [2007]
60. Assertion :For a reaction Reason : Enthalpy change ( H) is zero for
isothermal process.
2NH 3 (g) N 2 (g) 3H 2 (g); H E 64. Assertion : For a reaction 2NH3(g)
Reason : Enthalpy change is always greater than N2(g) + 3H2 (g); H > E.
internal energy change. [2008] Reason : Enthalpy change is always greater than
Directions for (Qs.61-64) : Each of these questions internal energy change. [2014]
contains an Assertion followed by Reason. Read them
carefully and answer the question on the basis of
following options. You have to select the one that
best describes the two statements.
EBD_7100
C-34 Topicwise AIIMS Solved Papers – CHEMISTRY

Type A : Multiple Choice Questions 6CO2 6H 2O 72Cal C6 H12 O6 6O 2


1. (c) Enthalpy change of a reaction does not For production of 180 g of glucose, heat
depend upon different intermediate energy is 72 k cals.
reactions. So required heat for 1.6 g of glucose
H H p – Hr 72
= 1.6 0.64 k cal
2. (a) S + O2 SO2 + x ..... (i) 180
6. (a) For a spontaneous process, the entropy
1
SO2 + O2 SO3 + y ..... (ii) should increase.
2 7. (d) Internal Energy of a gas consists of (i)
Adding (i) and (ii), we have kinetic energy of gas molecules (ii) potential
1 energy of gas molecules due to
S O2 SO2 O2 SO2 SO3
2 intermolecular attractions. It does not
include any other form of energy. So,
1
S 1 O2 SO 3 potential energy due to gravitational pull is
2 not included in it.
So, heat of formation of SO3 will be (x + y).
3. (a) We know that, V2
Molar heat capacity at constant volume, 8. (c) W PdV P(V2 V1 )
Cv = Specific heat at constant volume × V1
Mol. wt. W = –(1 atm) (20 – 10) = –10 dm3 atm.
= 0.075 × 40 = 3.0 cal
Cp – Cv = R 8.314 J / K / mol
10 dm 3 1013 J
or Cp = R + Cv = 2 + 3 = 5 0.08206 dm 3 / K / mol
Cp 5 According to Ist law of thermodynamics,
Now, ; 1.66 U = q + W = 800J + (–1013 J) = –213 J
Cv 3
9. (c) Work done in expansion of gas
This value shows that the gas is monoatomic. P
= – 2.303 nRT log 1
5 P2
4. (d) P(red) + O2 P2 O5 ; H 8.78
2 10
= – 2.303 × 1 × 2 × 273 log
5 1
P(yellow) + O2 P2 O5 ; H 9.91 = –1257.43 cal.
2
10. (c) H E P V PV RT; P V R T
Subtracting E CV (T2 T1 )
5 5 3 5
P(Re d) O2 P (Yellow) O2 R 75 R 75 75
2 2 2 2
= 372.56 cal
P2 O 5 P2 O 5 11. (c) Enthalpy of neutralisation of CH3COOH by
P (Red) – P (Yellow) = 0; NaOH is –50.6 kJ/mole ; for strong acid and
H 8.78 9.91 = 1.113 base this value is 55.9 kJ/mole. Heat evolved
P (Red) P (Yellow) , H 1.13 in the first case is less as some heat is used
P (Yellow) P (Red) , H 1.13 up in ionisation of CH3COOH. So, H for
5. (b) Formation of glucose will be the given by ionisation of CH3COOH = 55.9 – 50.6 = 5.3
reaction taking place in reverse direction. kJ/mol.
Thermodynamics C-35

12. (a) The internal energy of a substance (iii) In (d), both reactants and products are
increases with increase in temperature due in gaseous state, but number of moles
to increase in rotational, translational and of products are decreasing.
vibrational energy of the molecule. (iv) In (c), products are in liquid state hence
high entropy than reactant which is
13. (b) E –1228.2 k cal = –1228.2 × 103 cal
present in solid state.
H = E + nRT 19. (d) The order of heat of vaporisation or boiling
= – 1228.2 × 103 + (–2)(2)(298) point of the hydrides of VA group depend
= –1229392 cal upon their molecular weight. Anomalous
= –1229.392 kcal behaviour of NH3 is due to H-bonding
14. (c) For isothermal reversible process, present in NH3. Thus the correct order is
NH3 > AsH3 > PH3
P1
Wrev = –2.303 nRT log 20. (c) C O2 CO2 H 490 kJ/mol (i)
P2
1
10 H2 O2 H 2 O H 240 kJ/mol (ii)
= –2.303 × 2 × 8.314 × 373 log 2
25 8C 9H 2 C8 H18 H 160 kJ/mol (iii)
= + 5684.1 J = 5.684 kJ 2C8 H18 25O 2 16CO 2 18H 2O
15. (c) Enthalpy of reaction ( H) = Hp – HR The required reaction can be obtained by
For exothermic reactions, Hp < HR 2 × (iii) – 16 (i) – 18(ii)
H is negative
16C 18H 2 16C 16O 2 18H 2 9O 2
16. (a) The chemical reaction between strong base
and strong acid is a neutralisation reaction = 2C8 H18 16CO 2 18H 2 O
between H+ ion and OH– ion.
or – 25O 2 2C8H18 16CO 2 18H 2 O
H+ + OH– H2O + Heat
One mole of H+ and one mole of OH– give 2C8 H18 25O 2 16CO 2 18H 2O
57 kJ. When 0.5 moles of H+ (from HNO3)
[ H = 2 × 160 – 16 × – 490 – 18 × – 240]
reacts with 0.2 mole of OH– (from NaOH),
0.2 mole of H+ is neutralised by 0.2 mole of H = – 11840 J = – 11.840 kJ for 2 moles of
OH– and 0.3 mole of H+ remains unreacted. octane. Energy released for 6 moles of
octane = –11.840 × 3 = –35.5 kJ
H OH H 2O 21. (b) Heat of formation is defined as the heat
0.2 mole 0.2 mole
exchange when one mole of a compound is
57 0.2 formed from its constituent elements at STP.
Heat evolved will be = 11.4 kJ
1 In case of allotropes, most stable allotrope
17. (d) Heat produced (Q) = mC T is taken into account. Among graphite and
= 1× 1.23 × 6.12 kJ. diamond, graphite is stable. Hence, heat of
Molecule weight of NH4NO3 = 80 g formation of CH3OH is represented by the
Heat produced per mole = 80 × 1.23 × 6.12 eqn.
= 602 kJ/mol 1
18. (c) Entropy ( S) of a reaction is positive if the C(graphite) O2(g) 2H 2(g) CH3 OH(l )
2
products are in more random state as
22. (a) Zn H 2SO4 ZnSO4 H 2
compared to reactants.
Order of randomness : Gas > Liquid > Solid In bomb calorimeter, there is no expansion
(i) In (b), product NaCl (solid) has lesser in volume, so, work done will be zero. This
entropy as compared to NaCl (aq) (Na+ reaction is exothermic. So, some heat will
and Cl–) in reactant. be evolved which will result in lowering of
(ii) In (a), product (solid) has lesser internal energy. Hence,
entropy as compared to reactants. U 0&w 0
EBD_7100
C-36 Topicwise AIIMS Solved Papers – CHEMISTRY

23. (d) Total entropy change (system and To get the required equation, operate
surrounding) is always positive. (i) + 2× (ii) – (iii)
24. (a) G = 0 at equilibrium Thus, we get
25. (b) E = H – nRT C 2H 2 CH 4 [( x ) ( 2 y ) ( z )]
–2 8.31 298 Thus, heat of formation of methane is
–92.38 –
1000 (–x – 2y + z) kJ
= – 92.38 + 4.95 = – 87.43 kJ 31. (d) When solid melts S increases because when
26. (d) Heat is released in an exothermic process solid changes into liquid randomness
which increases the entropy of increases.
surroundings. V2
When entropy of the system is positive, 32. (d) W = – 2.303 nRT log
V1
overall entropy change is also positive. In
some exothermic reactions, entropy of Given n = 6, T = 27°C = 273 + 27 = 300 K
system may decrease. V1 = 1 L, V2 = 10 L
27. (b) H = – 92.2 kJ, P = 40 atm, V = – 1 L 10
Using H = E + P V W = – 2.303 × 6 × 8.314 × 300 log
1
E = H– P V = 34.465 kJ
R 33. (b) The required reaction is
= (– 92.2 kJ) – (40 atm) (– 1 L) ×
R C6 H10 H2 C6 H12 , H1 ? ... (1)
= – 92.2 kJ + (40 atm L) × Cyclohexene Cyclohexane

8.314 (JK-1mol-1 )
Let us write the given facts

0.0821 (atm L K-1 mol-1 )


1
H2 + O2 H2O ;
2
= – 92200 J + 4050.66 J = – 88149.34 J H2 = – 241 kJ/mol ...(2)
= – 88.149 kJ » – 88 kJ 17
C6 H10 O2 6CO 2 5H 2 O,
28. (a) 2
H fusion H vap H 3 = – 3800 kJ/mol ... (3)
Solid Liquid Vapours
C6H12 + 9O2 6CO2 + 6H2O,
H Sub H 4 = – 3920 kJ/mol ... (4)
So using Hess law, we get The required reaction (1) can be obtained
Hsub. = Hfus. + Hvap. = x + y by adding equations (2) and (3), and
29. (d) Since G H T S subtracting (4) from the sum of (2) and (3).
If G is negative, H is negative and S C6H10 + H2 C6H12.
is positive, then the reaction will be H1 = ( H2 + H3) – H4
spontaneous at all temperatures. = [–241 + ( – 3800)] – (–3920)
30. (d) From given data, we have = (–241 – 3800) – ( – 3920)
C + O2 CO2 – x kJ … (i) = – 4041 + 3920 = – 121 kJ/mol
1 34. (b) Change in internal energy ( E) is zero during
H2 O2 H 2O y kJ …(ii)
2 isothermal expansion of a gas.
35. (c) 2Cl(g) — Cl2(g)
CH 4 2O 2 CO 2 2H 2 O z kJ …(iii)
Entropy is decreasing (–ve) in the reaction.
The required equation is
Further the reaction is exothermic since a
C 2H 2 CH 4 Q bond is being formed, i.e., H is also –ve.
Thermodynamics C-37

36. (b) Since the gas expands adiabatically (i.e., no 41. (d)
change in enthalpy) so the heat is totally 42. (b) H Hº f (products) H º f (reactants)
converted into work. = –110.5 + (–241.8) – (–393.5+0) = 41.2 kJ
For thegas, CV = 20 J/K. Thus, 20 J of heat is
required for 1° change in temperature of the 43. (a) 2H 2 O 2 (l ) ¾¾
® 2H 2O(l ) + O 2 ( g ) DH = ?
gas.
Heat change involved during the process DH = [2´DH f of H 2O(l ) + (DHf of O2 )
i.e., work done = 3 kJ = 3000 J.
-(2´DHf of H 2O2 (l ))]
3000
Change in temperature K = 150 K
20 [(2 286) (0) (2 188)]
Initial temperature = 300 K [ 572 376] 196 kJ / mol
Since, the gas expands so the temperature 44. (b) For combustion reaction, H is negative,
decreases and thus final temperature is
300 – 150 = 150 K n 16 18 – 25 2 7, so S is + ve,
37. (a) More the enthalpy of formation less will be reaction is spontaneous, hence G is
the stability of the compound. –ve.
38. (c) 45. (a) I 2 ( s ) Cl2 ( g ) 2ICl( g )

39. (c) H c (C2 H6 ) H c (CH 4 ) rH = [ H(I2(s)


I2(g)) + HI–I
+ HCl–Cl] – [ HI – Cl]
368.2 ( 210.8) 157.4 = 151.0 + 242.3 + 62.76 –2 × 211.3 = 33.46
33.46
H c (C3 H8 ) H c (C2 H 6 ) Df H°(ICl) = = 16.73 kJ / mol
2
526.2 ( 368.4) 157.8 46. (b) H is given by
Thus, average
H U n g RT
157.4 157.8
H c ( CH 2 )
2 When ng 0, H U
–1
157.6 kcal mol
When ng 0, H U
Then,
H c (C6 H14 ) H c (C3H8 ) 3 H c ( CH 2 ) For C2 H 4 (g) H 2 (g) C2 H 6 (g)
= 526.2 3( 157.6) 999 k cal mol 1 ng 1 2 1
40. (d) Let bond energy of A2 be x then bond
47. (b) We are given,
energy of AB is also x and bond energy of
B2 is x/2. 15
C6 H 6 (l ) O2 (g) 6CO2 (g) 3H 2 O(l )
Enthalpy of formation of AB is –100 kJ/mole: 2
A2 B2 2 AB; H 3270 kJ mol 1 … (i)
1 1 C(gr) O 2 (g) CO2 (g),
A2 B2 AB; H 100kJ
2 2
1 …(ii)
H 394 kJ mol
1x x
or 100 x 1
2 4 H 2 (g) O2 (g) H 2 O(l ),
2
2x x 4x 1 1
100 x 400 kJ mol H 286 kJ mol …(iii)
4
EBD_7100
C-38 Topicwise AIIMS Solved Papers – CHEMISTRY
Formation of C6H6 53. (c) The properties of a system which depend
6C(gr) 3H 2 (g) upon quantity of matter contained in
it are called extensive properties, e.g.,
C6 H 6 (l ); H ? …(iv) mass, volume, heat capacity etc. But
By multiplying eq. (ii) with 6 and eq. (iii) mass / volume = density is an intensive
with 3 and adding we get, property as density does not depend upon
3 quantity of matter in a system.
6C(gr) 6O 2 (g) 3 H 2 (g) O 2 (g) 54. (a) Absolute values of internal energy of
2
6CO 2 ( g ) 6 H 2 O(l ) substance can not be determined because
it is impossible to determine exact values of
H 6( 394) 3( 286) constituent energies of the substances.
( 2364) ( 858) 55. (a) For all isothermal process change in internal
3222 kJ/mol energy is zero.
Now, by substracting eq. (i) from (v) we get n
56. (c) H E (RT)
6C(gr) 3H 2 (g) C 6 H 6 (l )
If there is no change in the moles of
H 3222 ( 3270) 48 kJ / mol
reactants (gaseous) and products
48. –1
(c) Given Cp = 75 JK mol –1
(gaseous) n = 0
100 H= E
n mole , Q = 1000 J T=?
18 57. (b) Molar heat of vaporisation of water is more
than ethanol because of presence of
Q = nCp T 1000 18 stronger H-bonding in water as compared
T 2.4 K
100 75 to ethanol. It is also true that water is more
polar than ethanol.
49. (b) H H f products 58. (a) At room temperature water is more stable
because ice will tend to melt at room
H f reactants temperature. So, A is true. But water will
have larger entropy than ice because ice
H° [ H f (CO)(g) H f (H 2 O)(g)] –
absorbs heat to be converted into water.
[ H f (CO 2 )(g) H f (H 2 )(g)] H
S
= [– 110.5 + ( – 241.8)]– [– 393.5 + 0] T
= 41.2 kJ
We know that greater the entropy, greater
Type B : Assertion Reason Questions the stability. Hence, water is more stable
50. (e) Ionic reactions are spontaneous as than ice. Both A and R is true and R
oppositely charged ions exert stronger explains A.
forces and combine immediately. 59. (c) In NaCl crystal Na+ and Cl– are strongly
51. (b) Entropy is defined as the extent of bonded due to electrostatic attraction. As
randomness in a system. When a substance it is dissolved in solvent, Na + and Cl –
is heated, its randomness and hence acquire greater freedom. In thermodynamic
entropy increases. Thus entropy of ice is terms formation of solution occurs with a
less than water because of lesser molecular favourable change in G. T S is largely
motion in ice. It is also true that ice has an –ve which overcomes the small +ve value
open cage like structure. of H. Thus G is negative for dissolution
52. (c) The temperature of a gas decreases when it of salt. Hence assertion is true but reason
undergoes adiabatic expansion. is false.
Thermodynamics C-39

60. (c) For given reaction H E because 62. (b) The factor T S increases with increase in
{ H E nRT }. In this reaction temperature.
63. (b) In an isothermal process change in internal
n(g) is [(3 + 1) – 2 = + 2 ] so H E. energy ( E) is zero (as it is a function of
Thus, assertion is true. temperature).
In some cases, the enthalpy change may According to first law
be less than internal energy change (in of thermodynamics
these cases n(g) , is negative), so the Q + W = E. Hence Q = –W (if E = 0)
reason is false. If a system undergoes a change in which
61. (b) In an isothermal process change in internal internal energy of the system remains
energy ( E) is zero (as it is a function of constant (i.e. E = 0) then –W = Q. This
temperature). means that work done by the system equals
According to first law of thermodynamics the heat absorbed by the system.
Q + W = E. Hence Q = –W (if E = 0) 64. (c) Assertion is true but reason is false.
If a system undergoes a change in which H = E + ngRT
internal energy of the system remains ng = (1 + 3) – 2 H > E.
constant (i.e. E = 0) then –W = Q. This If the value of ng is less than one then
means that work done by the system equals H < E.
the heat absorbed by the system.
EBD_7100
C-40 Topicwise AIIMS Solved Papers – CHEMISTRY

7 Equilibrium

TYPE A : MULTIPLE CHOICE QUESTIONS 8. A + 2B 2C + D, initial concentration of


B was 1.5 times that of A, but the equilibrium
1. In the reaction I 2 I I3 , the Lewis base concentration of A and B are found to be equal.
is : [1997] The equilibrium constant for the reaction is:
(a) 4 (b) 8 [2000]
(a) I3 (b) I 2
(c) 12 (d) 16
(c) I – (d) none of the above 9. pKavalue of four acids are given below. The
2. The equilibrium constant for the following strongest acid is: [2000]
reaction will be : 3A 2B C [1997] (a) 4.0 (b) 3.5
(c) 2.5 (d) 2
[3A ][2B] [C] 10. Ostwald's dilution law is applicable on: [2000]
(a) (b)
[C] [3A ][2B] (a) Strong electrolytes
[C ] (b) Weak electrolytes
[C ]
(c) (d) (c) Both strong and weak electrolytes
2
[A] [B] 2 [A]3 [ B]2
(d) None of them
3. Which is Lewis acid ? [1998] 11. The pH value of N/10 NaOH is : [2001]
(a) C2H5OH (b) BF3 (a) 9 (b) 10
(c) Cl– (d) NH3 (c) 12 (d) 13
4. The pH value of ordinary water is: [1998] 12. A solution having hydrogen ion concentration
(a) 7 (b) 6.5 is 0.0005 g eqvt./litre, its pOH is : [2001]
(c) 5.3 (d) 7.8 (a) 8.2798 (b) 10.6990
(c) 12.7854 (d) 13.3344
5. The value of K p for the reaction: 13. The pH of solution containing 0.10 M sodium
2
acetate and 0.03 M acetic acid is
2H 2S(g) 2H 2(g) S 2(g) is 1.2 10 at (pKa for CH3COOH = 4.57) [2002]
(a) 4.09 (b) 6.09
1065°C. The value for KC is: [1999] (c) 5.09 (d) 7.09
(a) <1.2 10 2
(b) >1.2 10 2 14. The solubility of CuBr is 2 × 10–4 mol/L at 25ºC.
The Ksp value of CuBr is [2002]
(c) 1.2 10 2
(d) 0.12 10 2 (a) 4 × 10–8 mol2 L–2 (b) 4 × 10–4 mol2 L–2
(c) 4 × 10–11 mol2 L–2 (d) 4 × 10–15 mol2 L–2
6. One mole of CH 3COOH and one mole of
15. At 80ºC, distilled water (H3O+) has concentration
2 equal to 1 × 10–6 mole/litre. The value of Kw, at
C 2 H 5 OH r eacts to produce mole of this temperature will be [2002]
3
(a) 1 × 10–6 (b) 1 × 10–12
CH 3COOC 2 H 5 . The equilibrium constant is : (c) 1 × 10–9 (d) 1 × 10–15
(a) 2 (b) + 2 [1999] 16. Which one of the following is NOT a buffer
(c) – 4 (d) + 4 solution? [2003]
7. The pH of a solution having the H+ ion (a) 0.8 M H2 S + 0.8 M KHS
concentration of 1 × 10–4 ions/litre is [1999]
(b) 2MC6H5NH2 + 2MC6H5 N H 3 Br –
(a) 2 (b) 3 (c) 3MH2CO3 + 3MKHCO3
(c) 4 (d) 5 (d) 0.05 M KClO4 + 0.05 M HClO4
Equilibrium C-41

17. In which of the following acid-base titration, pH 23. 40 ml of 0.1 M ammonia solution is mixed with 20
is greater than 8 at equivalence point. [2003] ml of 0.1 M HCl. What is the pH of the mixture?
(a) Acetic acid versus ammonia (pKb of ammonia solution is 4.74). [2006]
(b) Acetic acid versus sodium hydroxide (a) 4.74 (b) 2.26
(c) Hydrochloric acid versus ammonia (c) 9.26 (d) 5.00
(d) Hydrochloric acid versus sodium hydroxide 24. During titration of acetic acid with aq. NaOH
solution, the neutralisation graph has a vertical
18. What is the pH of 0.01 m glycine solution ? For
line. This line indicates [2007]
3
glycine, K a1 4.5 10 and

10
K a2 1.7 10 at 298 K? [2004] pH
(a) 3.0 (b) 10.0
(c) 6.1 (d) 7.1 V
(a) alkaline nature of equivalence
19. Of the following which change will shift the (b) acidic nature of equivalence
reaction towards the product? (c) neutral nature of equivalence
I2(g) 2I (g), H r0 (298 K) = + 150 kJ (d) depends on experimental proceeding.
25. The pH of the solution obtained on
(a) Increase in concentration of I2 [2004] neutralisation of 40 mL 0.1 M NaOH with 40 mL
(b) Decrease in concentation of I2 0.1 M CH3COOH is [2007]
(c) Increase in temperature (a) 7 (b) 8
(d) Increase in total pressure (c) 6 (d) 3
20. For the equilibrium 26. What is the pH value of M H2SO4 ? [2008]
(a) 0 (b) – 0.213
H2O (l) H2O (g) at 1 atm and 298 K :
(c) – 2 (d) – 0.3010
[2004]
27. In which of the following reactions, the
(a) standard free enery change is equal to zero concentration of the product is higher than the
( G° = 0) concentration of reactant at equilibrium ?
(b) free energy change is less than zero (K = equilibrium constant) [2008]
( G < 0) (a) A B; K 0.001
(c) standard free energy change is less than (b) M N; K 10
zero ( G° < 0) (c) X Y; K 0.005
(d) standard free energy change is greater than (d) R P; K 0.01
zero ( G° > 0) 28. On adding 0.1 M solution each of [Ag+], [Ba2+],
[Ca 2+] in a Na 2 SO 4 solution, species first
21. When 10 mL of 0.1 M acetic acid (pKa = 5.0), is
precipitated is [2008]
titrated against 10 mL of 0.1 M ammonia solution
[K sp BaSO 4 = 10 –11 , K sp CaSO 4 = 10 –6 ,
(pKb = 5.0), the equivalence point occurs at pH : KspAg2SO4 = 10–5]
[2005] (a) Ag2SO4 (b) BaSO4
(a) 5.0 (b) 6.0 (c) CaSO4 (d) All of these
(c) 7.0 (d) 9.0 29. A weak acid, HA is found to be 10% ionized in
22. For reaction, 2NOCl(g) 2 NO(g) + Cl2(g); Kc 0.01 M aqueous solution. Calculate the pH of a
–6 –1
at 427°C is 3 × 10 L mol . The value of Kp is solution which is 0.1 M in HA and 0.05 M in
nearly : [2005] NaA. [2009]
(a) 5.365 (b) 6.355
(a) 7.50 × 10–5 (b) 2.50 × 10–5
(c) 3.653 (d) 6.593
(c) 2.50 × 10–4 (d) 1.75 × 10–4
EBD_7100
C-42 Topicwise AIIMS Solved Papers – CHEMISTRY
30. The solubility of PbF2 in water at 25ºC is ~ 10–3 The equilibrium constant of the reaction
M. What is its solubility in 0.05 M NaF solution? 5
Assume the latter to be fully ionised. [2009] 2NH 3 O2 2NO + 3H2O in terms of
2
(a) 1.6 × 10–6 M (b) 1.2 × 10–6 M
–5 K1, K2 and K3 is
(c) 1.2 × 10 M (d) 1.6 × 10–4 M
31. Air containing 79% of nitrogen and 21% of K1K 2 K1K 32
oxygen by volume is heated at 2200 K and 1 atm (a) (b)
K3 K2
until equilibrium is established according to the
reaction
K 2 K 33
N2(g) + O2(g) 2NO(g) (c) (d) K1K2K3
If the Kp of the reaction is 1.1 × 10–3, calculate K1
the amount of nitric oxide produced in terms of 36. The pH of blood does not appreciably change by
volume percent. [2009] a small addition of acid or a base because blood
(a) 1.33 (b) 1.12 (a) contains serum protein which acts as buffer
(c) 1.02 (d) 1.44
(b) contains iron as a part of the molecule
32. Solubility product of a salt AB is 1 × 10–8 in a
(c) can be easily coagulated
solution in which the concentration of A+ ions
is 10–3 M. The salt will precipitate when the (d) is body fluid [2012]
concentration of B– ions is kept [2010] 37. If K1 and K2 are respective equilibrium constants
(a) between 10–8 M to 10–7 M for the two reactions [2013]
(b) between 10–7 M to 10–8 M XeF6 (g) + H2O (g) XeOF4 (g) + 2HF (g)
(c) > 10–5 M XeO4 (g) + XeF6 (g)
(d) < 10–8 M XeOF4 (g) + XeO3F2 (g)
33. In the manufacture of NH3 in Haber’s continuous
the equilibrium constant for the reaction
flow process involving the reaction [2011]
[Fe 2O3 ] XeO4 (g) + 2HF (g) XeO3F2 (g) + H2O (g)
N 2(g) + 3H 2(g) 2NH 3(g) ,
will be
H = –22.08 kcal. The favourable conditions are:
(a) High pressure and low temperature due to K1
low activation energy (Ea). (a) (b) K1.K 2
K 22
(b) Low pressure and low temperature due to
low Ea K1 K2
(c) High pressure and elevated optimum (c) (d)
K2 K1
temperature due to high Ea.
(d) None of these 38. Which equilibrium can be described as an acid-
34. In which of the following cases, pH is greater base reaction using the Lewis acid-base
than 7? [2011] definition but not using the Bronsted-Lowry
(a) 50 ml of 0.1 M HCl + 50 ml of 0.1 M NaCl definition? [2013]
(b) 50 ml of 0.1 M H2SO4 + 50 ml of 0.2 M NaOH (a) 2NH3 + H2SO4 2NH4+ + SO42–
(c) 50 ml of 0.1 M CH3COOH + 50 ml of 0.1 M
KOH (b) NH3 + CH3COOH
(d) 50 ml of 0.1 M HNO3 + 50 ml of 0.1 M NH3 NH4+ + CH3COO–
35. The following equilibria are given : [2012] (c) H2O + CH3COOH
N2 + 3H2 2NH3 ; K1
H3O+ + CH3COO–
N2 + O2 2NO ; K2
(d) [Cu(H2O)4]2+ + 4 NH3
1 [Cu(NH3)4]2+ + 4H2O
H2 + O 2 H2O ; K3
2
Equilibrium C-43

39. Steam reacts with iron at high temperature to The most soluble and least soluble compounds
give hydrogen gas and Fe3O4 (s). The correct are respectively. [2016]
expression for the equilibrium constant is (a) AgCl and PbCrO4
PH2 (b) AgI and Ag2CO3
2 (PH 2 ) 4 (c) AgCl and Ag2CO3
(a) (b) [2013]
PH2 O ( PH O ) 4 (d) Ag2CO3 and AgI
2 2
(PH 2 ) 4 [Fe 3 O 4 ] [Fe 3O 4 ] 45. Two equilibria, AB A B and
(c) (d) AB B AB2 are simultaneously
(PH 2 O ) 4 [Fe] [Fe]
maintained in a solution with equilibrium
40. Why only As3+ gets precipitated as As2S3 and constants, K1 and K2 respectively. The ratio of
not Zn2+ as ZnS when H2S is passed through [A+] to [AB2–] in the solution is [2016]
an acidic solution containing As3+ and Zn 2+?
(a) directly proportional to [B–]
(a) Solubility product of As2S3 is less than that
(b) inversely proportional to [B–]
of ZnS [2013]
(c) directly proportional to the square of [B–]
(b) Enough As3+ are present in acidic medium
(d) inversely proportional to the square of [B–]
(c) Zinc salt does not ionise in acidic medium
46. For the following reaction in gaseous phase
(d) Solubility product changes in presence of
1
an acid CO( g ) O2 ( g ) CO2 ( g ), K p / K c is
2
41. Ksp of M(OH) 2 is 3.2 × 10–11. The pH of saturated [2017]
solution in water is [2014] (a) (RT)1/2 (b) (RT)–1/2
(a) 3.40 (b) 10.30 (c) (RT) (d) (RT)–1
(c) 10.60 (d) 3.70 47. At 25°C, the solubility product of Mg(OH)2 is
42. Which of the following equilibria will shift to 1.0 × 10–11. At which pH, will Mg2+ ions start
right side on increasing the temperature?[2014] precipitating in the form of Mg(OH)2 from a
(a) CO (g) + H2O (g) CO2 (g) + H2 (g) solution of 0.001 M Mg2+ ions? [2017]
(b) 2SO2(g) + O2 (g) 2SO3 (g) (a) 9 (b) 10
(c) 11 (d) 8
(c) H2O (g) H2 (g) + 1 2 O 2 (g )
(d) 4HCl (g) + O2 (g) 2H2O (g) + 2Cl2 (g) TYPE B : ASSERTION REASON QUESTIONS
43. The reaction 2A( g ) + B( g ) 3C( g ) + D( g ) Directions for (Qs. 48-53) : These questions consist
is began with the concentrations of A and B both of two statements, each printed as Assertion and
at an initial value of 1.00 M. When equilibrium is Reason. While answering these questions, you are
reached, the concentration of D is measured and required to choose any one of the following five
found to be 0.25 M. The value for the equilibrium responses.
constant for this reaction is given by the (a) If both Assertion and Reason are correct and
expression [2015] the Reason is a correct explanation of the
(a) [(0.75)3 (0.25)] [(0.75)2 (0.25)] Assertion.
(b) [(0.75)3 (0.25)] [(1.00)2 (1.00)] (b) If both Assertion and Reason are correct but
(c) [(0.75)3 (0.25)] [(0.50)2 (0.75)] Reason is not a correct explanation of the
(d) [(0.75)3 (0.25)] [(0.50)2 (0.25)] Assertion.
44. The solubility product (Ksp) of the following (c) If the Assertion is correct but Reason is incorrect.
compounds are given at 25°C. (d) If both the Assertion and Reason are incorrect.
Compound Ksp (e) If the Assertion is incorrect but the Reason is
AgCl 1.1 × 10–10 correct.
AgI 1.0 × 10–16 48. Assertion: The aqueous solution of FeCl3 is
PbCrO4 4.0 × 10–14 basic in nature.
Ag2CO3 8.0 × 10–12 Reason: FeCl3 hydrolyses in water. [1998]
EBD_7100
C-44 Topicwise AIIMS Solved Papers – CHEMISTRY
49. Assertion : Addition of silver ions to a mixture (b) If both Assertion and Reason are correct, but
of aqueous sodium chloride and sodium bromide Reason is not the correct explanation of
solution will first precipitate AgBr rather than Assertion.
AgCl (c) If Assertion is correct but Reason is incorrect.
Reason : Ksp of AgCl < Ksp of AgBr [2004] (d) If both the Assertion and Reason are incorrect.
50. Assertion : Addition of NH4OH to an aqueous 54. Assertion : In a titration of weak acid and NaOH,
solution of BaCl2 in the presence of NH4Cl the pH at half equivalence point is pKa.
(excess) precipitates Ba(OH)2. Reason : At half equivalence point, it forms an
Reason : Ba(OH)2 is insoluble in water [2005] acidic buffer and the buffer capacity is maximum
51. Assertion : Mixture of CH 3 COOH and where [acid] = [salt] [2009]
CH3COONH4 is an example of acidic buffer. 55. Assertion : [Al(H2O)6]3+ is a stronger acid than
Reason : Acidic buffer contains equimolar [Mg(H2O)6]2+.
mixture of a weak acid and its salt with weak Reason : Size of [Al(H2O)6]3+ is smaller than
base. [2007] [Mg(H2O)6 ]2+ and possesses more effective
52. Assertion : The equilibrium constant is fixed and nuclear charge. [2008, 2010]
a characteristic for any given chemical reaction 56. Assertion : Reaction quotient is defined in the
at a specified temperature. same way as equilbrium constant at any stage
Reason : The composition of the final equilibrium of the reaction.
mixture at a particular temperature depends upon Reason : If Qc (reaction quotient) < KC (equilibrium
the starting amount of reactants. [2007] constant) reaction moves in direction of reactants.
53. Assertion :For reaction [2011]
N 2 (g) 3H 2 (g) 2NH3 (g) 57. Assertion : KOH is more soluble in water than
NaOH.
Unit of KC = L2mol–2
Reason : NaOH is a stronger base than KOH.
Reason : For the reaction
[2011]
N 2 (g) 3H 2 (g) 2NH3 (g) 58. Assertion : Buffer system of carbonic acid and
sodium bicarbonate is used for the precipitation
[NH3 ]2 of hydroxides of third group elements. [2015]
Equilibrium constant, KC [2008]
[N 2 ][H 2 ]3 Reason : It maintains the pH to a constant value,
about 7.4.
Directions for (Qs.54-59) : Each of these questions
59. Assertion : Addition of silver ions to a mixture
contains an Assertion followed by Reason. Read them
of aqueous sodium chloride and sodium bromide
carefully and answer the question on the basis of
following options. You have to select the one that solution will first precipitate AgBr rather than
best describes the two statements. AgCl.
Reason : Ksp of AgCl > Ksp of AgBr. [2016]
(a) If both Assertion and Reason are correct and
Reason is the correct explanation of Assertion.
Equilibrium C-45

Type A : Multiple Choice Questions 7. (c) pH = –log [H+]


= – log 10–4
1. (c) Here I is Lewis base as it is giving electron = – (–4) log 10
to I2 molecule. =4
8. (a) A + 2B 2C + D
[C] Initial 1 1.5 0 0
2. (d) Equilibrium constant, K = .
[A]3 [B]2 Equb. 1–x 1.5 – 2x 2x x
3. (b) In BF3, B is electron deficient because it At equilibrium, [A] = [B]
has incomplete octet and thus can accept
1– x = 1.5 – 2x
electron pair (Lewis acid).
x = 0.5
4. (c) pH value of ordinary water is about 5.3
2
because some CO 2 from atmosphere [C] [D]
Kc
dissolves in pure water to form [A][B]2
H2CO3(carbonic acid), thus making water
slightly acidic. (2 x ) 2 ( x)
5. (a) For the given reaction, (1 x )(1.5 – 2 x) 2
2H 2S (g) 2H 2 ( g ) S2 ( g ) 1 1 0.5
4
0.5 0.5 0.5
n 3–2 1
2 9. (d) Strength of acid Ka
Kp 1.2 10
Kc 1
( RT ) n ( RT )1 or pK a
Kc K p or Kc 1.2 10 –2
Hence acid with losest pKa value (2.0) will
6. (d)
be strongest.
CH3COOH + C2H5OH CH3COOC2H5 + H2O
10. (b) Ostwald's dilution law is applicable for
At start 1 mole 1 mole 0 0
weak electrolytes because strong
2 2 2 2 electrolytes are 100% ionised at all
At equb. 1 – 1–
3 3 3 3 concentrations while ionisation of weak
Let the total volume = V L electrolytes increases with increase in
1 dilution.
[CH3OOH] = V mol/L
3 11. (d) 0.1 N NaOH Na+ + OH–
1 Normality of NaOH = Molarity of NaOH
[C2H5OH ] = V mol/L
3 M
2 10
[CH3COOC2H5] = V mol/L 1
3 OH 10
2
[H2O] = V mol/L 14
3 [H ][OH ] 10
14
2 2 10 13
V V [H ] 10
10 1
K= 3 3 4
1 1 pH = – log 10–13
V V
3 3 pH = 13
EBD_7100
C-46 Topicwise AIIMS Solved Papers – CHEMISTRY
12. (b) pH = – log [H+] = – log [5 × 10–4] H K.C
= 4 – log 5 = 4 – [log 10 – log 2]
–8
= 3 + log 2 = 3.3010 7.65 10–13 0.01 = 8.7 × 10
pOH = 14 – pH = 14 – 3.3 = 10.7.
Hence, pH – log H
13. (c) According to Henderson equation, pH of
acidic buffer is given as – log 8.7 10 –8
Salt
pH = pKa + log – log10 –8 log 8.7
Acid
8 – 0.93 7.07 7.1
0.10
= 4.57 + log 19. (c) The given reaction is endothermic. If we
0.03
increase the temp. forward reaction will be
= 4.57 + [log 10 – log 3] promoted so that heat is absorbed to
= 4.57 + log 3.33 decrease the temp.
= 4.57 + 0.52 = 5.09
20. (b) The process, H 2O( ) H 2O(g) , is an
14. (a) CuBr Cu Br
[CuBr will be fully dissociated] endothermic process, ( H = +ve) and
4 entropy increases during this change
[Cu ] 2 10
( S = +ve) . Hence th is process is
and [Br ] 2 10 4
spontaneous at all temperatures above 0°C
[Cu+] [Br–] = 2 ×10–4 × 2 × 10–4 ( T S > H , so G is negative,
= 4 × 10–8 mol2 L–2 G = H – T S ). Thus free energy
15. (b) [H3O ] = [H+] = 10–6
+
change ( G ) will be less than zero
Kw = [H+] [OH–] (negative) at 1 atm and 298 K.
In pure water, [H+] = [OH–] 21. (c) CH3COOH NH 4OH CH 3COONH 4 H2O
So, [OH–] = [H+] = [H3O+] = 10–6 For salt of weak base & weak acid
Kw = [10–6] [10–6] = 10–12.
1
Remember that Kw increases with increase pH [log Ka log K w log Kb ]
2
in temperature.
1
16. (d) Buffer solution contains weak base + salt pK a log K w pK b
of weak base with strong acid or weak acid 2
+ salt of weak acid with strong base. 1
[ 5 log10 14 5]
In option (d) the acid used is HClO4 which 2
is strong acid and KClO4 is salt of this acid 1
[ 5 14 5]
with strong base. So it is not an example of 2
buffer solution. =7
17. (b) Acetic acid is a weak acid and sodium n
hydroxide is strong base. Salt of the two 22. (d) Kp K c [RT]
will hydrolyse to give basic solution. So at 6
neutral point pH of the solution will be = 3 10 (0.082 700)1
greater than 8. 4
= 3 10 6 0.082 7 102 = 1.75 10
18. (d) For glycine solution ; 23. (c) NH3 + HCl NH4Cl
K Ka1 K a2
Salt
–13 –10 pOH = pKb + log
4.5 10 1.7 10 Base
= 7.65 × 10–13
Equilibrium C-47

0.1/ 60 in this solution is S M, then


pOH = 4.74 + log total [F–] = [2S + 0.05] M.
0.1/ 60
pOH = 4.74 + 0 = 4.74 S[2S + 0.05]2 = 4 × 10–9
pH = 14 – 4.74 = 9.26 Assuming 2S << 0.05,
24. (a) In the titration of weak acid (CH3COOH) S × 25 × 10–4 = 4 × 10–9
with a strong base (NaOH), there is a change S = 0.16 × 10–5 M 1.6 × 10–6 M
We observe that our approximation that 2S
in the pH value at the end point. But it is
<< 0.05 is justified.
not sharp due to weak ionisation of
31. (a) N2(g) + O2(g) 2NO(g)
CH3COOH. Due to the excess of free base
At equilibrium, we have [N2] = 0.79 (1 – );
beyond the equivalence point, there is
[O2] = 0.21 (1 – ) ; [NO] = 2
steep rise in pH which is indicated by
Total number of moles
vertical line. Hence, the vertical line in the = 0.79 (1 – ) + 0.21 (1 – ) + 2 = 1 +
graph indicates alkaline nature of equivalence.
25. (b) As NaOH is a strong base and CH3COOH pN2 = 0.79 (1 ) × 1;
is a weak acid. So on neutralisation of 1
40 mL 0.1 M NaOH with 40 mL 0.1 M pO2 = 0.21(1 2
× 1; pNO = ×1
CH3COOH, we obtain a basic solution of 1
pH > 7. Hence, option (b) is correct. p 2NO
Kp =
26. (d) H 2SO 4 2H SO 24 p N 2 .p O2
1M 2M 1M 2
4
[H ] 2 1.1 × 10–3 =
0.79 0.21(1 )2
pH = –log[H+] = – log 2
or = 0.0067
= –0.3010 [ log 2 = 0.3010]
vol % of NO = 2 × 100
[Product] = 2 × 0.0067 × 100 = 1.33 %
27. (b) K
[Reactant] 32. (c) AB A B
Hence, [Product] K [Reactant]
K sp [A ][B ]
[Product] > Reactant, when K > 1
28. (b) The species having minimum value of Ksp Salt will precipitate if ionic conc. > Ksp
will get precipitated first of all because ionic [A+][B–] > 1 × 10–8
–3 –
(1 × 10 )[B ] > 1 ×10 –8
product will exceed the solubility product of
such a species. 8
1 10
The Ksp value is minimum for BaSO4(10–11), [B ] or 1 × 10–5
so, BaSO4 will get precipitated first of all. 1 10 3
29. (c) = 0.1, 33. (c) The reaction is exothermic and takes place
with a decrease in number of molecules of
2
C (0.1)2 (0.01)
Ka = = = 1.11 × 10–4 gaseous species. Obviously high pressure
1 (1 0.1) and low temperature are the favourable
Now pKa = –log 1.11 × 10–4 =3.9542 conditions for the shift of equilibrium to
[salt] products side. However, in continuous flow
pH = pKa + log process, optimum elevated temperature is
[acid]
0.05 required to have more NH3 due to high
= 3.9542 + log = 3.653 activation energy of the reaction.
0.10
30. (a) Solubility of PbF2 10 M –3 0.1
34. (c) (a) [ H 3O ] 0.05 ;
Ksp = 4S3 = 4 × 10–9 2
In 0.05 M NaF we have 0.05 M of F– ion pH= – log 0.05 = 1.301
contributed by NaF. If the solubility of PbF2
EBD_7100
C-48 Topicwise AIIMS Solved Papers – CHEMISTRY
(b) Complete neutralization of strong 4
acid and strong base, pH = 7. 41. (c) Ksp =4 S3 3.2 16 11 S = 2 × 10 M
(c) Hydrolysis of the salt CH3COOK, 4
[ OH ] = 2 S = 2 × 2 × 10 M
pH > 7 (salt of weak acid with strong
pH = 14 – pOH
base)
4
(d) Hydrolysis of the salt NH4 NO 3 , = 14 log 4 10 10.60
pH < 7 (salt of strong acid with weak
42. (c) Reaction (c) is endothermic. Electrolysis or
base)
decomposition of H2O is endothermic in
35. (c) (I) N2 + 3H2 2NH3; nature.
[ NH 3 ]2 43. (c) 2A(g) + B(g) 3C(g) + D(g)
K1 =
[ N 2 ][H 2 ]3 Mole ratio 2 1 3 1
(II) N2 + O2 2NO; Molar conc. 1 1 0 0
at t = 0
[ NO]2 Equilibrium 1– 0.5 1– 0.25 0.75 0.25
K2 =
[ N 2 ][O 2 ] molar
Concentration = 0.50 = 0.75
(III) H2 + 1 2 O2 H2O;
[H 2 O] [C ]3[ D] (0.75)3 (0.25)
K3 = Kc
1
2 [ A]2 [ B ] (0.50) 2 (0.75)
[ H 2 ][O 2 ]
44. (d) The solubility equilibrium for AgI is
(IV) 2NH3 + 5 2 O2 2NO + 3H2O
AgI (aq) Ag aq + I aq ;
[ NO]2 [ H 2 O ]3 K 2 K 33
Kc =
5 K1 Ksp = [Ag+][I–]
[ NH 3 ]2 [O 2 ] 2
Let solubility of AgI be S moles per litre,
36. (a) Blood contains serum protein which acts
as buffer. [Ag+] = S, [I–] = S
37. (d) Reaction (II) and reverse of reaction (I) gives Ksp = [Ag+][I–]
the desired reaction hence 1 × 10–16 = (S) × (S) = S2
1
1 K2 16 2 8
K= K2 . S = 1 10 1 10
K1 K1
On calculating solubility of all given
38. (d) [Cu(H 2 O) 4 ] 2+ + 4NH 3 compounds
[Cu(NH3)4]2+ + 4H2O involves lose and
gain of electrons. H2O is coordinated to Cu Compound Solubility
by donating electrons (LHS). It is then 5
AgCl 1 10
removed by withdrawing electrons. 8
AgI 1 10
39. (b) 3Fe(s) + 4H2O (steam) 7
PbCrO 4 2 10
Fe3O4 (s) + 4 H2 (g) 4
Ag 2 CO3 1.26 10
(p H 2 ) 4
Kp = only gaseous products Ag2CO3 is most soluble and AgI is least
(p H 2 O ) 4 soluble.
and reactants.
45. (d) Given,
40. (a) Ksp of As2 S3 is less than ZnS. In acid
K1 1
medium ionisation of H2S is suppresed AB A B
(common ion effect) and Ksp of ZnS does [A ] [B ]
not exceed. K1
[AB]
Equilibrium C-49

AB B
K2
AB 2 minimum. Further, Ba(OH)2 is soluble in
water.
[AB 2 ]
K2 51. (d) Acidic buffer is the solution of a mixture of
[AB] [B ]
weak acid and a salt of this weak acid with
Dividing K1 and K2 we get a strong base.
K1 [A ] [B ]2 CH3COOH is weak acid and CH3COONH4
K
K2 [AB2 ] is a salt of CH3COOH with weak base. So
the mixture of CH3COOH and CH3COONH4
[A ] K
is not an example of acidic buffer. Hence,
[AB2 ] [B ]2
both assertion and reason are false.
46. (b) For a gaseous phase reaction Kp and Kc
are related as 52. (c) The equilibrium constant has a fixed value
for every reaction at a particular
ng
Kp K c ( RT ) temperature. However composition of final
For the given reaction, equilibrium mixture at a particular
1 temperature does not depend upon the
CO(g) + O2 ( g ) CO2(g) starting amount of reactant.
2
1 Hence, assertion is true but reason is false.
ng = 1– (1 + 0.5) = – 0.5 or
2 53. (a) For the reaction
1
Kp = Kc ( RT ) 2 [NH3 ]2
Kc
1 [N 2 ][H 2 ]3
Kp
or = ( RT ) 2
Kc mol 2 L L3
Kc = L2 mol–2
47. (b) L2 mol mol 3
Mg(OH) 2 Mg 2 OH
So, the units for Kc are L2 mol–2
Ksp = [Mg++][OH–]2
The assertion and reason, both are true and
1.0 × 10–11 = 10–3 × [OH–]2
reason is correct explanation of assertion.

10-11
54. (a) Both assertion and reason are correct and
[OH- ] = = 10-4 reason is the correct explanation of assertion.
10-3 55. (a) The size of [Al(H2O)6]3+ is smaller than
pOH = 4 [Mg(H2O)6]2+. Also, the former possesses
pH + pOH = 14 more effective nuclear charge and thus,
attracts electron pair from donor more
pH = 10 effectively. This gives rise to relatively
Type B : Assertion Reason Questions strong acidic nature of [Al(H2O)6]3+.

48. (e) FeCl3 is acidic is nature due to hydrolysis. [C ]c [ D ]d


56. (c) aA + bB c C + dD, Qc
FeCl3 3H 2 O Fe(OH)3 3HCl [ A]a [ B ]b
If Qc > Kc, reaction will proceed in the
49. (c) Ksp of AgCl > Ksp of AgBr
direction of reactants
50. (e) Presence of NH 4 Cl suppresses th e
If Qc < Kc, reaction will move in direction of
dissociation of NH 4 OH, hence, products.
concentration of OH – is reduced to
EBD_7100
C-50 Topicwise AIIMS Solved Papers – CHEMISTRY
If Qc = Kc, the reaction mixture is already at
equilibrium.
57. (c) NaOH is a weaker base than KOH.
58. (d) In biological systems buffer system of
carbonic acid and sodium bicarbonate is
found in our blood. It maintains the pH of
blood to a constant value of about 7.4.
59. (a) Ionic product of AgBr is greater than that
of AgCl in comparison with there solubility
product AgBr will precipitate first rather
than that of AgCl.
8 Redox Reactions

TYPE A : MULTIPLE CHOICE QUESTIONS 8. MnO42– (1 mole) in neutral aqueous medium


disproportionates to [2003]
1. Oxidation is : [1999] (a) 2/3 mole of MnO4– and 1/3 mole of MnO2
(a) Gain of electrons (b) 1/3 mole of MnO4– and 2/3 mole of MnO2
(b) Loss of neutrons (c) 1/3 mole of Mn2O7 and 1/3 mole of MnO2
(c) Loss of electrons (d) 2/3 mole of Mn2O7 and 1/3 mole of MnO2
(d) Decrease in positive valency
9. For decolourisation of 1 mole of KMnO4, the
2. Oxidation number of Os in OsO 4 : [1999] moles of H2O2 required is: [2004]
(a) + 2 (b) + 4 (a) 1 / 2 (b) 3 / 2
(c) + 8 (d) + 10 (c) 5 / 2 (d) 7 / 2
3. The oxidation number of sulphur in H 2S2 O 7 is: 10. In the balanced chemical reaction,
(a) +2 (b) +6 [2000] IO3– aI bH cH 2 O d I2
(c) +4 (d) +8
a, b, c and d respectively corresponds to :
4. In the following chemical reaction:
(a) 5, 6, 3, 3 (b) 5, 2, 6, 3 [2005]
Ag 2 O H 2 O 2e 2Ag 2OH [2000] (c) 3, 5, 3, 6 (d) 5, 6, 5, 5
11. The oxidation states of iodine in HIO4, H3IO5
(a) Hydrogen is reduced
and H5IO6 are respectively [2008]
(b) Electrons are reduced
(a) + 1, + 3, + 7 (b) + 7, + 7, + 3
(c) Water is oxidised
(c) + 7, + 7, + 7 (d) + 7, + 5, + 3
(d) Silver is oxidised
12. Which of the following molecules can act as an
5. Both oxidation and reduction takes place in :
oxidizing as well as a reducing agent ? [2008]
(a) NaBr HCl NaCl HBr [2001] (a) H2S (b) SO3
(b) HBr AgNO3 AgBr HNO 3 (c) H2O2 (d) F2
13. The oxidation states of sulphur in S8, S2F2, H2S
(c) H2 Br2 2HBr respectively, are [2010, 12]
(d) CaO H 2SO 4 CaSO 4 H2O (a) 0, +1 and – 2
(b) + 2, +1 and – 2
6. The oxidation number of Cr in K 2Cr2 O 7 is : (c) 0, +1 and + 2
(a) +3 (b) – 3 [1997, 2001] (d) – 2, +1 and – 2
(c) +6 (d) – 6 14. The strength of an aqueous solution of I2 can
7. Oxidation state of Fe in Fe3O4 is [2002] be determined by titrating the solution with
standard solution of: [2012]
3 5
(a) (b) (a) Oxalic acid
2 4 (b) Sodium thiosulphate
(c) Sodium hydroxide
4 8
(c) (d) (d) Mohr’s salt
5 3
EBD_7100
C-52 Topicwise AIIMS Solved Papers – CHEMISTRY
15. Which of the following is a redox reaction ? (a) + 3, + 6 and + 5 (b) + 5, + 3 and + 6
(a) H2SO4 with NaOH [2013] (c) – 3, + 6 and + 6 (d) + 5, + 6 and + 6
(b) In atmosphere, O3 from O2 by lightening 18. Which of the following species can function
(c) Nitrogen oxides from nitrogen and oxygen both as oxidizing as well as reducing agent ?
by lightening [2016]
(d) Evaporation of H2O (a) Cl– (b) ClO4–
16. Which of the following involves transfer of five (c) ClO– (d) MnO4 –
electrons ? [2014] 19. Consider the following reaction occuring in basic
medium [2017]
(a) MnO Mn
2MnO –4 (aq)+Br – (aq)
(b) CrO Cr
2MnO2 (s) + BrO3– (aq)
(c) MnO MnO
How the above reaction can be balanced
(d) Cr O Cr further?
(a) By adding 2 OH– ions on right side
17. Oxidation numbers of P in PO3– 2–
4 , of S in SO 4 (b) By adding one H2O molecule to left side
and that of Cr in Cr2 O 72– are respectively (c) By adding 2H+ ions on right side
[2015] (d) Both (a) and (b)
Redox Reactions C-53

Type A : Multiple Choice Questions +5 0


1. (c) Oxidation involves loss of electrons.
10. (a) I O3– I – H + H2O + I 2
2. (c) Oxidation No. of Os in OsO4 is +8. (A) Oxidation half cell
3. (b) Oxidation no. of sulphur in H2S2O7 can be I– I2
calculated as follows : 2 + 2x – 14 = 0 (i) Balancing the number of atoms
2x = 14 – 2 = 12
2I – I2
12
x= 6 (ii) Balancing charge
2
4. (c) In the given reaction, water is being oxidised 2I – I2 2e –
because it is accepting oxygen from Ag2O,
while Ag+ is reduced. (B) Reduction half reaction

Ag 2 O H 2 O 2e 2Ag 2OH IO3– H+ H 2 O + I2


5. (c) In the reaction : (i) Balancing number of atoms
H 2 Br2 2HBr
2IO 3– 12H + 6H 2 O + I 2
Oxidation number of H in H2 is zero whereas
its value is +1 in HBr ; similarly oxidation (ii) Balancing the charge
number of Br in Br 2 is zero wheras its value
2IO 3– 12H + 10e 6H 2 O + I 2
is –1 in HBr. So, here H is oxidised and Br
reduced. In all other reactions, there in no Multiplying the balanced oxidation half
change in the oxidation number of any reaction by 5 and adding it to balanced
element. reduction half reaction
6. (c) Oxidation no of Cr in K2Cr2O7
2I – I2 2e – ] 5
=2+2×x–7×2=0
14 2 2IO 3– 12H + 10e – 6H 2 O + I 2
x 6
2
7. (d) Oxidation state of Fe in Fe3O4 2IO 3– 10I – 12H + 6I 2 6H 2 O
8
3x – 8 = 0; x or IO3– 5I – 6 H+ 3I 2 3H 2 O
3
8. (a) 3MnO 4 2– 2H 2 O Hence a 5 , b 6 , c 3 and d 3
11. (c) Calculating the oxidation states of I, we get
MnO 2 2MnO 4– 4OH –
H5IO6 ; 5 + x – 12 = 0 or x = +7
2
or MnO2–4 H2O H3IO5; 3 + x – 10 = 0 or x = +7
3 HIO4 ; 1 + x – 8 = 0 or x = +7
1 2 4
MnO 2 MnO 4– OH – Thus, the correct answer is + 7, +7, +7
3 3 3 12. (c) H2O2 can act both as an oxidising as well as
9. (c)
a reducing agent.
2KMnO 4 3H 2SO 4 K 2SO4 2MnSO4 3H 2O + 5[O]
Oxidising agent
[H 2 O2 O H2 O + O2 ] 5
PbS + 4 H2O2 PbSO 4 4H 2 O
2KMnO4 5H 2 O 2 3H 2SO4 K 2SO4 2MnSO 4
white ppt.
8H 2 O + 5O2
Reducing agent
2 mol of KMnO4 oxidise 5 mol of H2O2
Ag 2O H 2 O2 2Ag H 2O O 2
5 (moist)
1 mol of KMnO4 will oxidise mol of H 2O2
2
EBD_7100
C-54 Topicwise AIIMS Solved Papers – CHEMISTRY
18. (c) Species O.N.
0
13. (a) Oxidation number of S in S8 = 0 Cl – –1
8
ClO4– +7
Let oxidation number of S in S2F2 be x
2x + 2(–1) = 0 ClO– +1
2x – 2 = 0 MnO4 – +7
NO3 – +5
2x = 2 ; x = + 1
Let oxidation number of S in H2S be x. In ClO– chlorine is in +1 oxidation state
2 (+1) + x = 0 which can be increased or decreased thus
2+x=0 it acts as an oxidising or reducing agent.
x=–2 In other given species the underlined
elements are either in their minimum or
14. (b) I2 2 Na 2S2 O3 Na 2S4 O 6 2NaI
maximum oxidation state.
15. (c) N2 + O2 2NO 19. (d) Since reaction is occuring in basic medium
O.N. of N changes from 0 to +2 (oxidation) therefore 2OH– are added on right side.
and O.N. of O changes from 0 to –2
(reduction). 2MnO 4 (aq) + Br– (aq)
2MnO2(s) + BrO–3 (aq) + 2OH–(aq)
16. (a) O.N. of Mn in MnO – is +7 and in Mn it
Now, hydrogen atoms can be balanced by
is +2. The difference is of 5 electrons.
adding one H2O molecule to the left side
17. (d) PO3–
4 = x + 4 (– 2) = – 3; x – 8 = – 3; x = + 5 2MnO 4 (aq) Br (aq) H 2 O( )

SO 2–
4 = x + 4 (– 2) = – 2; x – 8 = – 2; x = + 6
2MnO2(s) + BrO3 (aq) 2 OH (aq)

Cr2 O72– = 2x + 7 (– 2) = – 2; 2x – 14 = – 2;
2x =12; x = + 6
9 Hydrogen

TYPE A : MULTIPLE CHOICE QUESTIONS 7. The alkali metals form salt-like hydrides by the
direct synthesis at elevated temperature. The
1. Hydrogen is not more reactive at ordinary thermal stability of these hydrides decreases in
temperature, because at ordinary temperature it which of the following orders ? [2015]
is in its: [1997] (a) CsH > RbH > KH > NaH > LiH
(a) gaseous state (b) liquid state (b) KH > NaH > LiH > CsH > RbH
(c) molecular state (d) atomic state (c) NaH > LiH > KH > RbH > CsH
2. Hydrogen has high ionization energy than alkali (d) LiH > NaH > KH > RbH > CsH
metals because it has : [1999] 8. Which of the following statements is incorrect?
(a) Ionic bond (b) Covalent bond (a) Ionic hydrides are stoichiometric
(c) Large size (d) Small size compounds of dihydrogen formed with
3. Which of the following statement is not correct most of s-block elements [2016]
regarding hydrogen atom? [2000] (b) Ionic hydrides are crystalline, non-volatile
and non-conducting in solid state.
(a) It resembles with halogens in some properties
(c) Melts of ionic hydrides conduct electricity
(b) It resembles with alkali metals in some and liberate dihydrogen gas at cathode.
properties.
(d) Both (a) and (c)
(c) It cannot be placed in first group of periodic 9. When zeolite (hydrated sodium aluminium
table. silicate) is treated with hard water the sodium
(d) It is the lightest element ions are exchanged with [2017]
4. Which of the following reaction produces (a) H+ ions (b) Ca2+ ions
hydrogen? [2002]
(c) SO 4 2 ions (d) OH– ions
(a) Mg + H2O (b) H2S2O8 + H2O
(c) BaO2 + HCl (d) Na2O2 + 2HCl TYPE B : ASSERTION REASON QUESTIONS
5. Consider the following statements : [2012]
1. Atomic hydrogen is obtained by passing Directions for (Qs. 10-11) : Each of these questions
hydrogen through an electric arc. contains an Assertion followed by Reason. Read them
2. Hydrogen gas will not reduce heated carefully and answer the question on the basis of
aluminium oxide. following options. You have to select the one that
3. Finely divided palladium adsorbs large best describes the two statements.
volume of hydrogen gas (a) If both Assertion and Reason are correct and
4. Pure nascent hydrogen is best obtained by Reason is the correct explanation of Assertion.
reacting Na with C2H5OH (b) If both Assertion and Reason are correct, but
Which of the above statements is/are correct ? Reason is not the correct explanation of Assertion.
(a) 1 alone (b) 2 alone (c) If Assertion is correct but Reason is incorrect.
(c) 1, 2 and 3 (d) 2, 3 and 4 (d) If both the Assertion and Reason are incorrect.
10. Assertion : HOF bond angle in HFO is higher
6. Which of the following statement is not correct
than HOCl bond angle in HClO.
about Ellingham diagram? [2012]
Reason : Oxygen is more electronegative than
(a) G increases with an increase in temperature halogens. [2014]
(b) It consists of plots of fGº vs T for 11. Assertion : Hydrogen combines with other
formation of oxides elements by losing, gaining or sharing of
(c) a coupling reaction can be well expressed electrons. [2015]
by this diagram Reason : Hydrogen forms electrovalent and
(d) It express the kinetics of the reduction process covalent bonds with other elements.
EBD_7100
C-56 Topicwise AIIMS Solved Papers – CHEMISTRY

Type A : Multiple Choice Questions 5. (c) Pure hydrogen is evolved by reacting


absolute alcohol and Na
1. (c) Hydrogen is not reactive at ordinary C 2 H 5OH Na C 2 H5ONa ½H 2
temperature because it is in molecular state
other statements are correct.
i.e., as H2 with its complete duplet which
6. (d) Ellingham diagrams are based on thermo-
makes it stable and lesser reactive. At very
dynamic concepts. It does not tell anything
high temperature, hydrogen exists in atomic
about the kinetics of the reduction process.
form having one electron i.e., incomplete
duplet which makes it less stable and highly 7. (d) The stability of alkali metal hydrides
reactive. decreases from Li to Cs. It is due to the fact
2. (d) Hydrogen has high ionisation energy in that M–H bonds become weaker with
comparision with alkali metals because it increase in size of alkali metals as we move
has only one orbital i.e. smaller size. Due to down the group from Li to Cs. Thus the
smaller size, hold of nucleus on outermost order of stability of hydrides is
electron is greater in case of hydrogen. LiH > NaH > KH > RbH > CsH
3. (c) Actually hydrogen is in the first group of i.e. option (d) is correct answer.
periodic table due to its much resemblance 8. (c) Melts of ionic hydrides conduct electricity
with alkali metals and liberate dihydrogen gas at anode.
4. (a) Mg reacts with hot water to produce H2 9. (b) Na zeolite + CaCl 2 Ca zeolite 2NaCl
because it is more electropositive than
hydrogen. Type B : Assertion Reason Questions
Mg H 2 O MgO H 2 10. (d) Both assertion and reason are false.
All other reactions are used for HOF bond angle in HFOis lesser than that
manufacture of H2O2. of HOCl bond angle in HClO. Oxygen is
H 2S2 O8 2H 2 O 2H 2SO 4 H 2 O 2 more electronegative than all halogens
except fluorine.
BaO 2 2HCl BaCl2 H 2 O 2
11. (a)
Na 2 O 2 2HCl 2NaCl H 2 O 2
10 The s-Block Elements

TYPE A : MULTIPLE CHOICE QUESTIONS (a) 15.78 mL (b) 153.8 mL


(c) 198.4 mL (d) 295.5 mL
1. An inorganic compound which on heating first 9. On dissolving moderate amount of sodium metal
melts, then solidifies and liberates a gas, is in liquid NH3 at low temperature, which one of
(a) Al2O3 (b) KMnO4 [1997] the following does not occur? [2003]
(c) MnO2 (d) KClO3 (a) Blue coloured solution is obtained
2. Sodium carbonate is soluble in water because of (b) Na+ ions are formed in the solution
(a) high lattice energy [1997] (c) Liquid NH3 becomes good conductor of
(b) low lattice energy electricity
(c) low molecular weight (d) Liquid ammonia remains diamagnetic
(d) high molecular weight 10. The pair whose both species are used in ant-
3. Chemical A is used for softening of water to acid medicinal preparations is : [2006]
remove temporary hardness. A reacts with (a) NaHCO3 and Mg(OH)2
sodium carbonate to produce caustic soda. (b) Na2CO3 and Ca(HCO3)2
When CO2 is bubbled through ‘A’, it turns (c) Ca(HCO3)2 and Mg(OH)2
cloudy. Chemically ‘A’ is : [1999] (d) Ca(OH)2 and NaHCO3
(a) CaO (b) CaCO3 11. Largest difference in radii is found in case of the
(c) Ca(HCO3)2 (d) Ca(OH)2 pair [2007]
4. Sodium on heating with moist air produces (a) Li, Na (b) Na, K
(a) NaO (b) NaOH [1999] (c) K, Rb (d) Rb, Cs.
(c) Na 2O (d) Na 2CO 3 12. Among the following components of cement
5. Nitrolim is: [2000] which is present in highest amount? [2007]
(a) CaC2 and graphite (a) Ca2SiO4 (b) Ca3SiO5
(b) CaCN2 and graphite (c) Al2O3 (d) Ca3Al2O6.
(c) Ca(CN)2 and graphite 13. The commercial salt is purified by passing
(d) CaCN2 + N2 [2009]
6. The molecular formula of Plaster of paris is: (a) H2S gas through an alkaline solution of salt
(b) HCl gas through a saturated solution of salt
(a) 2CaSO 4 .H 2 O [2000]
(c) H2 gas through a solution of salt
(b) CaSO 4 .2H 2 O (d) Cl2 gas through a saturated solution of salt
(c) CaSO 4 .3H 2 O 14. Na2CO3 reacts with SO2 in alkaline medium to
(d) 2CaSO 4 .1 / 2 H 2 O form: [2010]
7. Which one of the following hydroxide is (a) Na2SO3 (b) NaHSO3
insoluble in water ? [2001] (c) Na2SO4 (d) NaHSO4
(a) Ca (OH ) 2 (b) Ba (OH ) 2 15. The correct sequence of increasing melting
points of BeCl2, MgCl2, CaCl2, SrCl2 and BaCl2
(c) Be(OH) 2 (d) Mg (OH) 2
is [2011]
8. 1 g equimolecular mixture of Na2 CO3 and (a) BaCl2 < SrCl2 < CaCl2 < MgCl2 < BaCl2
NaHCO3 is reacted with 0.1 N HCl. The mL of (b) BeCl2 < MgCl2 < CaCl2 < SrCl2 < BaCl2
0.1N HCl required to react completely with the (c) BeCl2 < CaCl2 < MgCl2 < SrCl2 < BaCl2
above mixture is : [2001] (d) MgCl2 < BeCl2 < SrCl2 < CaCl2 < BaCl2
EBD_7100
C-58 Topicwise AIIMS Solved Papers – CHEMISTRY
16. Which one of the alkaline earth metals shows 23. Which of the following is not correct ?
some anomalous behaviour and has same [2016]
electronegativity as aluminium? [2011] heat
(a) 2Li 2 O Li 2 O 2 2Li
(a) Sr (b) Ca 673K
(c) Ba (d) Be heat
17. Which property of sodium is being used in street (b) 2K 2O K 2O2 2K
673K
lights ? [2011] heat
(a) It shows photoelectric effect (c) 2Na 2 O Na 2O 2 2Na
673K
(b) It has low melting point
heat
(c) Sodium atom emits photons in the yellow (d) 2Rb 2O Rb 2 O 2 2Rb
673K
region of visible spectrum, due to
electrically stimulated transitions TYPE B : ASSERTION REASON QUESTIONS
(d) Sodium vapours show golden colour Directions for (Qs. 24-28) : These questions consist
18. A metal salt solution forms a yellow precipitate of two statements, each printed as Assertion and
with potassium chromate in acetic acid, a white Reason. While answering these questions, you are
precipitate with dil H2 SO 4 , but gives no required to choose any one of the following five
precipitate with NaCl. The metal salt solution responses.
will consist of [2011] (a) If both Assertion and Reason are correct and
(a) PbCO3 (b) BaCO3 the Reason is a correct explanation of the
(c) MgCO3 (d) CaCO3 Assertion.
19. The order of solubility of lithium halides in non (b) If both Assertion and Reason are correct but
polar solvents follows the order : [2012] Reason is not a correct explanation of the
(a) Assertion.
LiI LiBr LiCl LiF
(c) If the Assertion is correct but Reason is incorrect.
(b) LiF LiI LiBr LiCl
(d) If both the Assertion and Reason are incorrect.
(c) LiCl LiF LiI LiBr (e) If the Assertion is incorrect but the Reason is
(d) LiBr LiCl LiF LiI correct.
20. The following compounds have been arranged 24. Assertion : Magnesium continues to burn in
in order of their increasing thermal stabilities. nitric oxide.
Reason : During burning heat evolved does not
Identify the correct order. [2012]
decompose NO. [2001]
K2CO3 (I) MgCO3 (II) 25. Assertion : Potassium and caesium are used in
CaCO3 (III) BeCO3 (IV) photoelectric cells.
(a) I < II < III < IV (b) IV < II < III < I Reason : Potassium and caesium emit electrons
(c) IV < II < I < III (d) II < IV < III < I on exposure to light. [2002]
21. Chemical A is used for water softening to remove 26. Assertion : Barium is not required for normal
biological function in human
temporary hardness. A reacts with Na2CO3 to
Reason : Barium does not show variable
generate caustic soda. When CO2 is bubbled oxidation sate. [2003]
through A, it turns cloudy. What is the chemical 27. Assertion : BaCO3 is more soluble in HNO3 than
formula of A [2013] in plain water.
(a) CaCO3 (b) CaO Reason : Carbonate is a weak base and reacts
(c) Ca(OH)2 (d) Ca(HCO3)2 with the H+ from the strong acid, causing the
barium salt to dissociate. [2003]
22. Which of the following is commercially known
28. Assertion : Mg is not present in enamel of human
as oxone ? [2014]
teeth.
(a) Na2O2 + HCl (b) Na2O + HCl
Reason : Mg is an essential element for
(c) Na2O2 + Na2 (d) none of these biological functions of human. [2004]
The s-Block Elements C-59

Directions for (Qs.29-37) : Each of these questions 32. Assertion : In fused state, calcium chloride
contains an Assertion followed by Reason. Read them cannot be used to dry alcohol or NH3.
carefully and answer the question on the basis of Reason : CaCl2 is not a good dessicant. [2010]
following options. You have to select the one that 33. Assertion : LiCl is predominantly a covalent
best describes the two statements.
compound.
(a) If both Assertion and Reason are correct and Reason : Electronegativity difference between
Reason is the correct explanation of Assertion. Li and Cl is too small. [2012]
(b) If both Assertion and Reason are correct, but 34. Assertion : Of the various chlorides of alkaline
Reason is not the correct explanation of earth metals BeCl2 is covalent in nature, where
Assertion. as MgCl2 and CaCl2 are ionic compounds.
(c) If Assertion is correct but Reason is incorrect.
Reason : Be is the first member of group 2.
(d) If both the Assertion and Reason are incorrect.
[2012]
29. Assertion : We feel cold on touching the ice. 35. Assertion : Best diagonal relationship is shown
Reason : Ice is a solid form of water. [2009] between Be and Al.
30. Assertion : s-Block elements do not occur free Reason : Ionization energy of Be is almost the
in nature. same as that of Al. [2013]
Reason : s-Block elements are highly 36. Assertion : K, Rb and Cs (all belonging to
group 1) can also form superoxides.
electropositive in nature. [2009]
Reason : The ionic radii of K, Rb and Cs show
31. Assertion : Both Be and Al can form complexes the following trend Cs+ < Rb+ < K+. [2014]
such as BeF42– and AlF63– respectively, BeF63– 37. Assertion : Lithium carbonate is not so stable to
is not formed. heat.
Reason : Lithium being very small in size
Reason : In case of Be, no vacant d-orbitals are
present in its outermost shell. [2009] polarizes large CO32 ion leading to the
formation of more stable Li2O and CO2
[2017]
EBD_7100
C-60 Topicwise AIIMS Solved Papers – CHEMISTRY

Type A : Multiple Choice Questions 0.5


Mole of Na 2CO3 0.0047
106
heat 0.5
1. (d) 2KClO3 2KCl 3O 2 Mole of NaHCO3 0.0059
melts solid 84
solid
0.0047 mole of Na2CO3
2. (b) Sodium carbonate ionises in aqueous
0.0047×2 mole of HCl
solution. It is soluble in water because of
= 0.0094 mole of HCl
low lattice energy. Low lattice energy helps
0.0059 mole of NaHCO3
in solubility. For substances which doe not
0.0059 mole of HCl
ionise
Total mole required = 0.0153 moles of HCl
1 If V be the volumes in litre of HCl required
Solubility
Molecular mass then mole of HCl required =V × 0.1
3. (d) Ca(OH)2 is used to remove temporary 0.0153
hardness of water. V= 0.153 litre
0.1
Ca(OH)2+Ca(HCO3)2 2CaCO3 + 2H2O = 153 ml
Ca(OH)2 + Na2CO3 CaCO3 + 2NaOH 9. (d) Sodium dissolves in liquid ammonia to form
Ca(OH)2 + CO2 CaCO3 + H2O blue solution which is good conductor of
(Cloudy electricity. The blue colour of the solution
solution) is due to formation of ammoniated
4. (c) Sodium on heating with moist air produces electrons.
Na2O. Na Na+ + e–
2Na + H2O Na2O + H2 +
Na + xNH3 Na+ (NH3)x
5. (b) Nitrolim is CaCN2 + C. It is used as fertilizer –
e + yNH3 [e (NH3)y]–
since it reacts with H2O to form NH3. ammoniated electron

CaCN 2 3H 2 O CaCO 3 2NH 3


Due to presence of unpaired electron liquid
ammonia become paramagnetic.
1 10. (a) NaHCO3 is used in medicine to neutralise
6. (d) 2CaSO 4 . H 2 O . the acidicty in the stomach. Mg(OH)2 is
2
7. (c) In hydroxides of alkaline earth metals, basic in nature and dissolves in acids
hydration energy does not alter very much forming corresponding salts. So both are
used in antacid medicinal preparations.
while lattice energy decreases down the
11. (b) Radii of atoms increase as we go down the
group due to incrase in size.
group due to the addition of extra energy
The overall effects of the two factors is that
shell. Nuclear charge also increases on
H solution of the hydroxides
moving down the group but the influence
( H solution = Hlattice – Hhydration) of addition of energy shell predominates.
becomes more negative from Be to Ba and This predomination is larger in case when
hence solubility increases from top to we move from Na to K among all the options
bottom. given.
Be(OH)2 < Mg(OH)2 < Ca(OH)2 < Sr (OH)2 12. (b) Composition of cement depends upon its
< Ba(OH)2 type. Portland cement is most common kind
8. (b) Na 2 CO 3 2HCl 2NaCl H 2 O CO 2 and contains 51% tricalcium silicate, 24%
of dicalcium silicate and 6% of tricalcium
NaHCO3 HCl NaCl H 2 O CO 2 aluminate.
The s-Block Elements C-61

13. (b) The common table salt is prepared by 25. (a) Potassium and Cesium are electropositive
passing HCl gas through a saturated elements with very low ionisation potential
solution of salt. The precipitation is due to so they emit electrons on exposure to light.
common ion effect. 26. (b) Both the assertion and reason are correct.
NaCl H Cl NaCl The reason is not the explanation of the
Salt ppt assertion.
solution
Note that alkaline earth metals are bivalent,
14. (a) In alkaline medium Na2CO3 reacts with SO2
i.e. have an oxidation state of +2.
to form Na2SO3 (sodium sulphite). 27. (a) Barium carbonate is more soluble in HNO3
On reaction with caustic alkalies SO2 forms than in water because carbonate is a weak
two types of salts – bisulphites and base and reacts with the H+ ion of HNO3
sulphites. In alkaline medium, it forms causing the barium salt to dissociate.
Na2SO3.
BaCO3 2HNO3 Ba(NO3 ) 2 CO2 H 2O
15. (b) The ionic character and hence the m.pt. 28. (e) Calcium is present in the enamel but Mg is
increases as the size of cation increases. an essential element for biological functions
16. (d) Be shows diagonal relationship with Al of human.
17. (c) The property of sodium atom to emit 29. (b) It is correct that on touching the ice we feel
photons in yellow region of visible cold because ice absorbs heat from our
spectrum, due to electrically stimulated hand.
electron transitions is used in street lights. 30. (a) The s-block elements are highly
18. (b) BaCO3 forms a yellow ppt of barium electropositive in nature. So, these are very
chromate. BaCO3 forms a white precipitate reactive. That is why these elements do not
of BaSO4. BaCl2 is soluble in water. occur free in nature. Usually, the s-group
elements occur in nature as halides,
19. (a) As the size of the anion increases
carbonates and sulphates. These metals are
from, F– toI – the covalent character increase obtained from their molten halides by
and hence the solubility in non-polar electrolytic reduction.
solvent increases. 31. (a) Both assertion and reason are correct and
LiI > LiBr > LiCl > LiF reason is correct explanation of assertion.
32. (c) Assertion is true but Reason is false. CaCl 2
20. (b) forms addition products with alcohol and
21. (c) ammonia.
33. (c) LiCl is a covalent compound. Due to the large
Ca (HCO3 ) 2 Ca (OH) 2 2CaCO3 2H 2 O size of the anion (Cl–) its effective nuclear
temp. hardness
charge lessens and its valence shell is held
Ca(OH) 2 Na 2CO3 2NaOH CaCO3 less tightly towards its nucleus. Here,
A Caustic soda assertion is correct but reason is incorrect.
34. (b) Both assertion and reason are true but
Ca(OH)2 +CO 2 CaCO 3 +H 2O
A milkiness
reason is not the correct explanation of
22. (a) (Na2O2 + HCl) is commercially known as assertion. The ionic nature of MgCl2 and
oxone and is used for bleaching of delicate CaCl 2 is due to the large difference in
fibres. electronegativity values of Mg, Ca to that
of chlorine.
23. (a) Lithium does not form peroxide. 35. (a) Reason is the correct explanation of
Type B : Assertion Reason Questions Assertion.
36. (c) Assertion is correct. Reason is incorrect.
24. (c) Magnesium continues to burn in nitric oxide 37. (a) Lithium carbonate is unstable to heat;
because the heat evolved decomposes NO lithium being very small in size polarises
to give N2and O2 and Mg burns readily in
a large CO32 ion leading to the formation
the atmosphere of N2 and O2.
of more stable Li2O and CO2.
EBD_7100
C-62 Topicwise AIIMS Solved Papers – CHEMISTRY

11 The p-Block Elements

TYPE A : MULTIPLE CHOICE QUESTIONS 10. In diamond crystal, each carbon atom is linked
with carbon atoms? The number of carbon atoms
1. The approximate percentage of silica in cement linked is : [2001]
is: [1998] (a) 2 (b) 4
(a) 5 – 10% (b) 15 – 20% (c) 3 (d) 1
(c) 20 – 25% (d) 25 – 30% 11. The electrical conductivity of semiconductors :
2. The most abundant metal in the earth crust is: (a) decreases with temperature [2001]
(a) Pb (b) Cu [1998] (b) increases with temperature
(c) Al (d) Mg
(c) remains constant on heating
3. Bell metal is an alloy of: [1999]
(d) none of the above
(a) Sn and Pb (b) Cu and Pb
12. Which of the following is only acidic in nature?
(c) Sn and Cu (d) In, Sn and Cu
(a) Be(OH)2 (b) Mg(OH)2 [2004]
4. Al2O3 on heating with carbon in an atmosphere
of N2 at high temperature produces : [1999] (c) B(OH)3 (d) Al(OH)3
(a) Al + CO2 (b) Al + CO2 + NO 13. The liquefied metal expanding on solidification
(c) Al4C3 + CO2 (d) AlN + CO is : [2004]
5. Producer gas is a mixture of: [1998, 2000] (a) Ga (b) Al
(c) Zn (d) Cl
(a) CO H 2 (b) H2 CH 4
14. In diborane, the two H–B–H angles are nearly :
(c) CO N 2 (d) CO H 2O (a) 60° , 120° (b) 95° , 120° [2005]
6. Solid CO2 is known as dry ice: [2000] (c) 95° , 150° (d) 120° , 180°
(a) It melts at 0°C 15. The pair of amphoteric hydroxides is [2005]
(b) Its B.P. is more than 199°C (a) Al(OH)3, LiOH
(c) It evaporates at – 78°C without melting (b) Be(OH)2, Mg(OH)2
(d) None of the above (c) B(OH)3, Be(OH)2
7. Water glass is : [2001] (d) Zn(OH)2, Be(OH)2
(a) Na 2SiO 3 (b) Mg 2Si 16. Borax is used as cleansing agent because on
dissolving in water it gives [2006]
(c) SiCl 4 (d) Ca (H 2 PO 4 ) 2
(a) Alkaline solution
8. Water gas is produced by : [2001] (b) Acidic solution
(a) passing steam over red hot coke (c) Bleaching solution
(b) passing steam and air over red hot coke (d) Colloidal solution
(c) burning coke in excess of air 17. The energy gaps (Eg) between valence band and
(d) burning coke in limited supply of air conduction band, for diamond, silicon and
9. In lake test of Al3+ ion, there is formation of germanium are in the order: [2006]
coloured floating. It is due to : [2001]
(a) Eg (diamond) > Eg (silicon) > Eg (germanium)
(a) adsorption of litmus by H2O
(b) Eg (diamond) < Eg (silicon) < Eg (germanium)
(b) adsorption of litmus by Al(OH)3
(c) Eg (diamond) = Eg (silicon) = Eg (germanium)
(c) adsorption of litmus by Al(OH)4–
(d) Eg (diamond) > Eg (germanium) > Eg (silicon)
(d) none of these
The p-Block Elements C-63

18. The repeating unit in silicone is [2007] (c) Ionic radii of Pb2+ and Pb4+ are larger than
R those of Ge2+ and Ge4+
| (d) More pronounced inert pair effect in lead
(a) SiO2 (b) – Si O than in Ge
| 26. Carbon cannot be used to produce magnesium
R by chemical reduction of MgO because: [2014]
R (a) Carbon is not a powerful reducing agent
| | | (b) Magnesium reacts with carbon to form
(c) O Si O (d) Si O O R
| carbides
|
R R (c) Carbon does not react with magnesium
(d) Carbon is a non-metal
19. Select correct statement(s). [2008]
27. Carborundum is obtained when silica is heated
(a) Cyanamide ion (CN22–) is isoelectronic with
at high temperature with [2014]
CO2 and has the same linear structure
(a) carbon (b) carbon monoxide
(b) Mg2C3 reacts with water to form propyne
(c) carbon dioxide (d) calcium carbonate
(c) CaC2 has NaCl type lattice
28. An inorganic salt (A) is decomposed on heating
(d) All of the above
to give two products (B) and (C). Compound (C)
20. Non-oxide ceramics can be [2008]
is a liquid at room temperature and is neutral to
(a) B4C (b) SiC
litmus while the compound (B) is a colourless
(c) Si3N4 (d) All of these
21. The chemical formula of 'tear gas" is [2008] neutral gas. Compounds (A), (B) and (C) are [2014]
(a) COCl2 (b) CO2 (a) NH4NO3, N2O, H2O
(c) Cl2 (d) CCl3NO2 (b) NH4NO2, NO, H2O
22. Thermodynamically, the most stable form of (c) CaO, H2O, CaCl2
carbon is [2009] (d) Ba(NO3)2, H2O, NO2
(a) Diamond (b) Graphite 29. Which of the following oxides is strongly basic?
(c) Fullerenes (d) Coal (a) B2O3 (b) Al2O3 [2015]
23. Which of the following product is formed when (c) Ga2O3 (d) Tl2O3
SiF4 reacts with water? [2010] 30. Chemically borax is [2015]
(a) SiF3 (b) H4SiO4 (a) Sodium metaborate
(c) H2SO4 (d) H2SiF4 (b) Sodium orthoborate
24. BCl3 does not exist as dimer but BH3 exists as (c) Sodium tetraborate
dimer (B2H6) because [2012] (d) Sodium tetraborate decahydrate
(a) Chlorine is more electronegative than 31. Match List – I with List –II for the compositions
hydrogen of substances and select the correct answer
(b) there is p -p back bonding in BCl3 but using the code given below the lists : [2015]
BH3 does not contain such multiple List - I List - II
bonding Substances Composition
(c) large sized chlorine atoms do not fit in (A) Plaster of paris (i) CaSO4.2H2O
between the small boron atoms where as (B) Epsomite (ii) CaSO4.½ H2O
small sized hydrogen atoms get fitted in (C) Kieserite (iii) MaSO4.7 H2O
between boron atoms (D) Gypsum (iv) MgSO4. H2O
(d) none of the above (v) CaSO4
25. Ge(II)compounds are powerful reducing agents Code :
whereas Pb(IV)compounds are strong oxidants. (A) (B) (C) (D)
It can be due to [2012] (a) (iii) (iv) (i) (ii)
(a) Pb is more electropositive than Ge (b) (ii) (iii) (iv) (i)
(b) Ionization potential of lead is less than that (c) (i) (ii) (iii) (v)
of Ge (d) (iv) (iii) (ii) (i)
EBD_7100
C-64 Topicwise AIIMS Solved Papers – CHEMISTRY
32. The liquefied metal which expands on 40. Assertion : PbI4 is a stable compound.
solidification is : [2016] Reason : Iodide stabilizes higher oxidation state
(a) Ga (b) Al [2003]
(c) Zn (d) In
41. Assertion : SiF62 is known but SiCl26 – is not
33. A neutral molecule XF3 has a zero dipole moment.
The element X is most likely [2016] Reason : Size of fluorine is small and its lone pair
(a) chlorine (b) boron of electrons interacts with d-orbitals of Si
(c) nitrogen (d) carbon strongly. [2005]
34. The correct formula of borax is [2016] 42. Assertion : Silicones are hydrophobic in nature.
(a) Na2[B4O4(OH)3].9 H2O Reason : Si-O-Si linkages are moisture sensitive.
(b) Na2[B4O5(OH)4].8 H2O [2006]
(c) Na2[B4O6(OH)5].7 H2O 43. Assertion :PbCl2 is more stable than PbCl4.
(d) Na2[B4O7(OH)6].6 H2O Reason : PbCl4 is powerful oxidising agent.
[2008]
TYPE B : ASSERTION REASON QUESTIONS Directions for (Qs.44-47) : Each of these questions
Directions for (Qs. 35-43) : These questions consist contains an Assertion followed by Reason. Read them
of two statements, each printed as Assertion and carefully and answer the question on the basis of
Reason. While answering these questions, you are following options. You have to select the one that
required to choose any one of the following five best describes the two statements.
responses. (a) If both Assertion and Reason are correct and
(a) If both Assertion and Reason are correct and Reason is the correct explanation of Assertion.
the Reason is a correct explanation of the (b) If both Assertion and Reason are correct, but
Assertion. Reason is not the correct explanation of
(b) If both Assertion and Reason are correct but Assertion.
Reason is not a correct explanation of the (c) If Assertion is correct but Reason is incorrect.
Assertion. (d) If both the Assertion and Reason are incorrect.
44. Assertion : Coloured cations can be identified
(c) If the Assertion is correct but Reason is incorrect.
by borax bead test.
(d) If both the Assertion and Reason are incorrect. Reason : Transparent bead (NaBO2 + B2O3)
(e) If the Assertion is incorrect but the Reason is forms coloured bead with coloured cation. [2009]
correct. 45. Assertion : Pb4+ compounds are stronger
35. Assertion : Al(OH)3 is insoluble in NH 4OH oxidizing agents than Sn4+ compounds.
Reason : The higher oxidation states for the
but soluble in NaOH . group 14 elements are more stable for the heavier
Reason : NaOH is strong alkali. [1997] members of the group due to ‘inert pair effect’.
36. Assertion : Boron is metalloid. [2014]
Reason : Boron shows metallic nature. [1997] 46. Assertion : Pb4+ compounds are stronger
37. Assertion : NH3 is absorbed more readily over oxidizing agents than Sn4+ compounds.
activated charcoal than CO2. Reason : The higher oxidation states for the
Reason : NH3 is non-polar. [2000] group 14 elements are more stable for the heavier
38. Assertion : Stannous chloride gives grey members of the group due to ‘inert pair effect’.
precipitate with mercuric chloride, but stannic [2016]
chloride does not do so. 47. Assertion : Atomic radius of gallium is higher
Reason : Stannous chloride is a powerful than that of aluminium [2017]
oxidising agent which oxidises mercuric chloride Reason : The presence of additional d-electron
to metallic mercury. [2002] offer poor screening effect for the outer
39. Assertion : Diamond is a bad conductor. electrons from increased nuclear charge.
Reason : Graphite is a good conductor. [2002] [2017]
The p-Block Elements C-65

Type A : Multiple Choice Questions 13. (a) Gallium (Ga) is soft , silvery metal. Its melting
point is 30ºC. This metal expands by 3.1%
1. (c) The approximate composition of cement is : when it solidifies and hence, it should not
Lime (CaO) = 50 – 60% be stored in glass or metal containers.
Silica (SiO2) = 20 – 25 % H
Alumina (Al2O3) = 5 – 10 % H H
Magnesia (MgO) = 2 – 3% 14. (b) 120° B 95° B
Ferric oxide (Fe2O3) = 2 – 3% H H
2. (c) H
3. (d) Bell metal is an alloy of Cu and Sn. 15. (d) Be(OH)2 and Zn(OH)2 are amphoteric
hydoxides in nature.
4. (d) Al2 O3 N 2 3C 2Al N 3CO Be(OH)2 + 2HCl BeCl2 + 2 H2O
Be(OH)2 + 2 NaOH Na2BeO2 + H2O
5. (c) Producer gas is a mixture of CO and N2.
Zn(OH)2 + 2 HCl ZnCl2 + 2 H2O
6. (c) Solid CO2 is known as dry ice because it Zn(OH)2 + 2 NaOH Na2ZnO2 + 2H2O
evaporates at –78°C without melting. 16. (a) Borax is Na2B4O7.10H2O. It gives alkaline
7. (a) Sodium silicate (Na2SiO3) is known as water solution on dissolution in water as it is a
glass. It is soluble in water. Just as plants salt of strong base and weak acid.
grow in the soil, coloured crystals grow in Na2B4O7 + 7H2O 4H3BO3 + 2NaOH
the water glass. 17. (a) In diamond, it is highest (6 eV), for silicon,
8. (a) Water gas is produced by passing steam it is 1.1 eV and for germanium, it is 0.72 eV.
over hot coke. 18. (b) Polymeric organosilicon compounds
containing Si–O–Si bonds are called
H 2O C CO H 2 silicones. Silicones have general formula
water gas (R2SiO)n. Hence repeating unit of silicone
9. (b) In lake test of Al3+, a coloured floating lake is R2SiO –.
is formed as Al(OH)3 which being a solid 19. (d) In CO2 we have 22 (6 + 8 + 8 = 22) electrons.
surface (suspension), adsorbs litmus In (CN22–), we have 22 (6 + 7 + 7 + 2 =22)
colour, resulting in coloured floating lake electrons. Both CO2 and (CN22–) have linear
appearance. structures. Thus, statement (a) is correct.
10. (b) In diamond crystal, carbon atom is in sp3 Mg 2 C3 4H 2 O
hybridised state so each carbon is linked 2Mg(OH) 2 CH3C CH
Propyne
with four other carbons by bond.
11. (b) With the increase in temperature, more i.e., statement (b) is also correct .
The structure of CaC2 is of NaCl type
number of covalent bonds are broken,
i.e., statement (c) is also correct.
resulting large increase in current carrier
20. (d) Ceramics are inorganic , non-metallic, solid
concentration (i.e. electrons and holes). minerals. They come in a variety of forms,
Due to which the conductivity of including silicates (silica, SiO2 with metal
semiconductor increases. oxides), oxides (oxygen and metals),
12. (c) B(OH)3 is actually H3BO3 which is an acid carbides (carbon and metals), aluminates
(boric acid): It is a monobasic acid. It is not (alumina, Al2O3 with metal oxides) and
a proton donor (Bronsted acid), but nitrides.
behaves as Lewis acid. It can accept a lone The given ceramics are B4C (carbides), SiC
pair of electrons from OH– ion. (carbides), Si3N4 (nitrides) and thus, none
B(OH ) 3 NaOH NaBO 2 2H 2 O of these is an oxide . All of these are non-
oxide ceramics.
EBD_7100
C-66 Topicwise AIIMS Solved Papers – CHEMISTRY
21. (d) "Tear gas" is the name given to the 33. (b) BF3 has planar and symmetrical structure
compound, CCl 3 NO2 . It is also called thus as a result the resultant of two bond
chloropicrin. moments, being equal and opposite to the
CHCl3
HNO3
CCl3 NO 2 third, cancels out and hence molecule
possess zero dipole moment.
Cl2
or CH3 NO2 CCl3 NO2 F
NaOH
chloropicrin
22. (b) Graphite is thermodynamically, the most ; =0
F B
stable allotrope of carbon. That is why fHº
(graphite) is taken as zero.
–1
f Hº (diamond) = + 1.90 kJ mol F
–1 34. (b) Borax is sodium tetraborate decahydrate i.e.
f Hº (fullerene) = + 38.1 kJ mol
23. (b) In reaction with water, SiF4 (like SiCl4) gets Na2[B4O5(OH)4].8H2O
hydrolysed to form H4SiO4 (silicic acid).
Type B : Assertion Reason Questions
SiF4 4H 2 O
Si(OH) 4 or H 4SiO 4 4HF 35. (a) Al(OH)3 is soluble in strong alkali like NaOH
(Silicic acid) because of formation of meta-aluminate ion
24. (c) NaOH Al(OH)3 NaAlO 2 2H 2O
25. (d) Ge(II) tends to acquire Ge (IV) state by loss 36. (c) Boron is a metalloid. Thus, assertion is true.
of electrons. Hence it is reducing in nature. Metalloids possess metallic as well as non-
Pb (IV) tends to acquire Pb (II) O.S. by gain metallic nature. Hence, reason is false.
of electrons. Hence it is oxidising in nature. 37. (c) It is correct that NH3 is absorbed more
This is due to inert pair effect. readily over activated charcoal than CO2
26. (b) because of its polar nature.
27. (a) Silica on heating with carbon at elevated 38. (c) Sn4+ is more stable than Sn2+.
temperature, gives carborundum (silicon So, Sn2+ is oxidised to Sn4+ by losing 2
carbide) electrons when it reacts with mercuric
SiO 2 3C SiC 2CO chloride, i.e., SnCl2 is a reducing agent.
carborundum 2HgCl2 SnCl 2 Hg 2 Cl2 SnCl 4
Carborundum is a very hard substance.
Hg 2 Cl 2 SnCl 2 2Hg SnCl 4
28. (a) Reaction involved is : (grey ppt)
NH4 NO3 N2O 2H 2 O So assertion is correct but reason is wrong.
A B C
Hence option (a) is correct. Hence correct option is (c).
29. (d) The correct sequence of increasing basic 39. (b) Diamond is a bad conductor because of lack
of free electrons in its lattice. Graphite is a
strength is
good conductor of electricity because of
B2O3 < Al2O3 < Ga2O3 < In 2O3 < Tl2O3
free electron in its lattice.
30. (d) Chemically borax is sodium tetraborate So both assertion and reason are correct but
decahydrate and it has the formula reason is not correct explanation of assertion.
Na2B4O7.10H2O. 40. (d) As we move down the group IVA, +2
1 oxidation state becomes more stable. Thus
31. (b) (A) Plaster of paris = CaSO4. H 2 O
2 Pb4+ is not possible, i.e., PbI4 is highly
(B) Epsomite = MgSO4.7H2O unstable. I2 is a weak oxidising agent so it
(C) Kieserite = MgSO4.H2O cannot oxidise Pb to Pb4+ oxidation state.
(D) Gypsum = CaSO4.2H2O So assertion and reason both are wrong.
32. (a) Gallium (Ga) is soft , silvery metal. Its melting 41. (a) SiF62– is known because F has small size
point is 30ºC. This metal expands by 3.1% and thus the ion is quite stable unlike
when it solidifies and hence, it should not SiCl62– in which size of Cl atom is large
be stored in glass or metal containers. which destabilise it.
The p-Block Elements C-67

42. (c) Silicons are hydrophobic in nature i.e. it is qualitative analysis. This test is called borax
water repellant because most of the groups bead test.
which form bulky silicon molecule are 740°C
organic in nature so they are water repellant. Na 2 B4O7 2NaBO2 B 2 O3
sodium boric
Thus assertion is true. The Si–O–Si linkages metaborate anhydride
are stable, so these are moisture resistant.
glassy bead
Hence reason is false.
43. (b) Pb2+ is more stable than Pb4+ due to inert 45. (c) Assertion is true because lower oxidation
pair effect. Due to this reason, PbCl 4 state becomes more & more stable for
decomposer readily into PbCl 2 and Cl2 heavier elements in p-block due to inert pair
PbCl4 PbCl2 + Cl2 effect. Hence Reason is false.
Thus Pb4+ salts are better oxidising agents 46. (c) Assertion is true because lower oxidation
44. (a) Borax (Na2B4O7) when heated at about state becomes more & more stable for
740°C, forms a glassy bead which gives heavier elements in p-block due to inert pair
different colour beads with different effect. Hence Reason is false.
cations. 47. (c) Atomic radius of gallium is less than that
Hence, it is used to identify cations in of aluminium.
EBD_7100
C-68 Topicwise AIIMS Solved Papers – CHEMISTRY

Organic Chemistry–Some Basic


12 Principles and Techniques
TYPE A : MULTIPLE CHOICE QUESTIONS H H R R
| | | |
1.. The IUPAC name of following compound is (b) H __ C R __ C R __ C R __ C
| | | |
H H H R
CH 3 __ CH __ CH __ CH __ CH 2 __ CH 3
| | | R R H H
[1997] | | | |
CH 3 CH 3 CHO
(c) R __ C + > R __ C + > R __ C + > H __ C
(a) 2-ethyl –3, 4-dimethylpentanal | | | |
H R H H
(b) 2, 3-dimethyl –4-aldohexane
H R R H
(c) 3-Aldo –4, 5-dimethylhexane | | | |
(d) 1, 3, 4, 5-tetraethylbutanal (d) R __ C+ > R __ C+ > R __ C+ > H __ C
2. The number of enantiomers of the compound | | | |
CH 3 CHBr CHBr COOH is : [1997] H H R H
7. The number of sigma electrons in toluene are
(a) 2 (b) 3 (a) 6 (b) 9 [2001]
(c) 4 (d) 6 (c) 15 (d) 30
3. The process of decomposition of organic 8. The most suitable method for removing water
compound by the application of heat is :[1999] traces from ethanol is : [2001]
(a) pyrolysis (b) evaporation (a) distillation
(c) sublimation (d) condensation (b) passing dry HCI
4. Turpentine oil can be purified by: [1999] (c) reacting it with Mg
(d) heating with sodium metal
(a) Steam distillation
9. IUPAC name of
(b) Sublimation
(c) Vacuum distillation CH 3 C H CH 2 CH CH 3
| | [2002]
(d) Fractional distillation CH3 CN
5. Which pairs have same percentage of carbon?
(a) 2-cyano –3-methylhexane
(a) CH 3COOH and C 6 H12O 6 [2000] (b) 2, 4-dimethylcyanopentane
(b) CH 3COOH and C12 H 22O11 (c) 3-methyl –5-cyanohexane
(d) 2-cyano –3-methylhexane
(c) CH 3COOH and C 2 H 5OH
(d) C 6 H12O 6 and C12 H 22O11 CH3
6. The decreasing order of stability of alkyl
carbonium ion is in the order of : [2001] 10. IUPAC name of is [2003]

R R H H (a) 3-methylcyclohexene
| | | | (b) 1-methylcyclohex-2-ene
(a) R __ C R __ C R __ C H __ C (c) 6-methylcyclohexene
| | | |
R H H H (d) 1-methylcyclohex-5-ene
Organic Chemistry–Some Basic Principles and Techniques C-69

11. The most reactive nucleophile among the 18. Correct configuration of the following is [2005]
following is : [2003]
CH3
(a) CH3O– (b) C6H5O–
(c) (CH3)2 CHO– (d) (CH3)3CO– H OH
12. The absolute configuration of the following : CH3 OH
CH3 H
H Cl (a) 1S, 2S (b) 1S, 2R
Cl H is [2003] (c) 1R, 2R (d) 1R, 2S
C2H5 19. Among the following, L-serine is : [2006]
CO 2 H
(a) 2S, 3R (b) 2S, 3S
(c) 2R, 3S (d) 2R, 3R (a) H2N CH 2 OH
13. Which of the following compound possesses
the C—H bond with th e lowest bond H
dissociation energy? [2003] CO 2 H
(a) Toluene
(b) Benzene (b) HOH 2 C H
(c) n-pentane
(d) 2, 2-dimethylpropane NH 2
14. The dipole moment is the highest for : [2004] NH 2
(a) trans-2-butene
(b) 1, 3 -dimethylbenzene (c) H CO 2 H
(c) acetophenone
CH 2 OH
(d) ethanol
15. The geometrical isomerism is shown by [2004] CH 2 OH
CH2 (d) H2N H
CH2
(a) (b) CO 2 H
20. Methyl- - D-glucoside and methyl- -D-
CCl2 CHCl glucoside are: [2006]
(a) Epimers
(c) (d)
(b) Anomers
(c) Enantiomers
(d) Conformational diastereomers
16. Among the following, the strongest nucleophile
21. Chain transfer reagent is [2007]
is : [2005]
– (a) CCl4 (b) CH4
(a) C2H5SH (b) CH3COO
(c) O2 (d) H2.
(c) CH3NH2 (d) NCCH2–
22. The correct structure of
17. Among the following the most stable compound
4-bromo-3-methylbut-1-ene is [2008]
is : [2005]
(a) cis-1, 2-cyclohexanediol (a) Br–CH = C(CH3)2
(b) trans-1, 2-cyclohexanediol (b) CH2 =CH – CH(CH3)–CH2Br
(c) cis-1,3-cyclohexanediol (c) CH2 = C(CH3)CH2CH2Br
(d) trans-1, 3-cyclohexanediol (d) CH3– C(CH3) = CHCH2 – Br
EBD_7100
C-70 Topicwise AIIMS Solved Papers – CHEMISTRY
23. Spin isomerism is shown by [2008] 29. The incorrect IUPAC name is [2012]
(a) dichlorobenzene (b) hydrogen (a) CH 3 C C H CH 3
(c) dibasic acid (d) n-butane || |
24. The correct stability order of following species O CH3
is – [2009] 2-methyl-3-butanone
(b) CH3 – CH – CH – CH 3
(x) | |
(y)
CH 3 CH 2 CH 3
H
C 2, 3 -dimethylpentane
(z) (c) CH3– C CCH(CH3)2
4-methyl-2-pentyne

(w) (d) CH3 CH CH CH3


| |
(a) x > y > w > z (b) y > x > w > z Cl Br
(c) x > w > z > y (d) z > x > y > w
3-chloro-2-bromobutane
25. Which one of the following compounds is
30. Among the following compounds (I-III), the
resistant to nucleophilic attack by hydroxyl ions?
correct order of reactivity towards electrophilic
[2010]
substitution reaction is [2012]
(a) Methyl acetate (b) Acetonitrile
OCH3 NO2
(c) Diethyl ether (d) Acetamide
26. The correct IUPAC name for [2011]

I II III
(a) II > III > I (b) III < I < II
(c) I > II > III (d) I = II > III
(a) 5-methyl -4-(1'-2'-demethylpropyl) heptane 31. The most stable carbanion among the following
(b) 3-methyl-4-(1',2'-dimethylpropyl) heptane is [2012, 2013]
(c) 2,3,5-trimethyl-4-propylheptane CH2 – CH2– CH2–
(d) 4-propyl-2,3,5-trimethylpeptane
27. Which of the cations in more stable ? [2011] (a) (b)
H
|
(a) R ' C OR CH2– CH2–
H
|
(b) R ' C OR (c) (d)
(c) both equal
(d) both are unstable OCH3 NO2
28. The Lassaigne’s extract is boiled with dil. HNO3 32. Which one of the statements about
before testing for halogens because [2012] HOH2CCH(OH)CHO is not correct ? It [2013]
(a) Silver halides are soluble in HNO3 (a) is an isomer of 1, 3-dihydroxypropanone
(b) Na2S and NaCN are decomposed by HNO3 (b) contains a tertiary alcoholic group
(c) Ag2S is soluble in HNO3 (c) has the same empirical formula as glucose
(d) AgCN is soluble is HNO3 (d) can show optical isomerism
Organic Chemistry–Some Basic Principles and Techniques C-71

33. Which is the correct order of stability of the 37. Name of the compound given below is
following three carbonium ions ? [2014]
CH 3
+ + +
CH2 = CHC HCH 3 CH2 = C C H2 CH3CH = CH CH2
I II III
(a) I > II > III
(b) II > I > III (a) 3- methyl -4- ethyloctane [2013, 2015]
(c) I III > II (b) 2, 3- diethylheptane
(d) All are equally stable (c) 5- ethyl - 6- methyloctane
34. The molecular formula of diphenyl methane, (d) 4- ethyl- 3- methyloctane
38. Which of the following represents the given
CH2 , is C13H12. mode of hybridisation sp2–sp2 – sp – sp from
How many structural isomers are possible when left to right? [2015]
one of the hydrogens is replaced by a chlorine (a) H2C = CH – C N
atom? [2014] (b) CH C – C CH
(a) 6 (b) 4 (c) H2C = C = C = CH2
(c) 8 (d) 7 CH2
35. The compound CHCl == CHCHOHCOOH with
molecular formula C4H5O3Cl can exhibit[2014] (d) CH2
(a) geometric, optical position and functional 39. In Lassaigne's test for the detection of halogens,
isomerism
the sodium fusion extract is first boiled with
(b) geometric, optical and functional isomerism
concentrated nitric acid. This is [2016]
only
(a) to remove silver halides
(c) position and functional isomerism only
(d) geometric and optical isomerism only (b) to decompose Na2S and NaCN, if present
(c) to dissolve Ag2S
36. Tautomerism is exhibited by – [2015]
(d) to dissolve AgCN, if formed
40. Which one among the following cannot exhibit
(1) CH = CH – OH
enantiomerism? [2016]
(a) Diphenyl methanol
(b) 1-Bromo-2-chlorobutane
(2) O O
(c) 2-Butanol
(d) Tartaric acid
41. The IUPAC name of the following compound is
(3) O
(CH 3 ) 2 CH CH 2CH CH CH CH CHCH 3
|
O C2 H 5

[2016]
(4) O
(a) 1,1,7,7-tetramethyl-2,5-octadiene
(b) 2,8-dimethyl-3,6-decadiene
O (c) 1,5-di-iso-propyl-1,4-hexadiene
(a) (1), (3) and (4) (b) (2), (3), and (4) (d) 2,8-dimethyl-4,6-decadiene
(c) All of these (d) None of these
EBD_7100
C-72 Topicwise AIIMS Solved Papers – CHEMISTRY
42. Consider the following bromides : TYPE B : ASSERTION REASON QUESTIONS
Me Me
Me Br Me Directions for (Qs.46-48) : Each of these questions
Br Br contains an Assertion followed by Reason. Read them
(A) (B) carefully and answer the question on the basis of
The correct order of SN1 reactivity is following options. You have to select the one that
[2016] best describes the two statements.
(a) B > C > A (b) B > A > C (a) If both Assertion and Reason are correct and
(c) C > B > A (d) A > B > C Reason is the correct explanation of Assertion.
43. The strongest ortho - para and strongest meta - (b) If both Assertion and Reason are correct, but
directing groups respectively are [2017] Reason is not the correct explanation of
(a) –NO2 and –NH2 Assertion.
(b) –CONH2 and –NH2 (c) If Assertion is correct but Reason is incorrect.
(c) –NH2 and –CONH2 (d) If both the Assertion and Reason are incorrect.
(d) –NH2 and –NO2 46. Assertion : In the third group of qualitative
44. Hybridisation states of C in CH3+ and CH4 are analysis, NH4Cl is added to NH4OH medium.
[2017] Reason : This is to convert the ions of group
(a) sp2 & sp3 (b) sp3 & sp2 into their respective chlorides. [2014]
(c) sp2 & sp2 (d) sp3 & sp3 47. Assertion : Carbanions like ammonia have
45. The increasing order of stability of the following pyramidal shape. [2015]
free radicals is [2017] Reason : The carbon atom carrying negative
• • • charge has an octet of electrons.
(a) (C6H5)2 C H < (C6H5)3 C < (CH3)3 C <
• 48. Assertion : NF 3 is a weaker ligand than
(CH3)2 C H
• • •
N(CH3)3. [2017]
(b) (CH3)2 C H < (CH3)3 C < (C6H5)2 C H < Reason : NF3 ionizes to give F– ions in aqueous

(C6H5)3C solution.
• • •
(c) (CH3)2 C H < (CH3)3 C < (C6H5)2 C H <

(C6H5)3 C
• • •
(d) (C6H5)3C < (C6H5)2 C H < (CH3)3 C <

(CH3)2 C H
Organic Chemistry–Some Basic Principles and Techniques C-73

Type A : Multiple Choice Questions 11. (d) The most reactive nulceophile will be
(CH3)3 –CO– due to +I effect of three –CH3
5 4 3 2
1. (a) C H3 – CH – C H – CH – CH 2CH 3 groups.
| | |
CH3 CH3 CHO S
1
1
2. (c) No. of asymmetric carbon = 2 CH3
No. of enantiomers = 22 = 4. 12. (b) H 2
Cl
3. (a) Pyrolysis is the process of decomposition 3
of organic compound by the application of Cl H
heat. C2H5
4. (a) Turpentine oil is volatile and is insoluble in S
water. Therefore, it can be purified by steam
distillation. So compound is (2S, 3S)
5. (a) Molecular formula of CH3COOH = C2H4O2 13. (a) Among the carbonium ions formed by the
Both the compounds given have same cleavage of the C—H bond, the order of
empirical formula that is CH2 O. So, stability is :
percentage of carbon in both of them will C6 H5CH 2 (CH 3 )3 CCH 2
be same.
CH3 (CH 2 )3 CH 2 C6H5
6. (a) Tertiary carbonium ion has highest stability
followed by secondary and then primary As C6H5CH2+ is most stable so, in C6H5
carbonium ion. So, option (a) is correct. CH2 – H, C – H bond energy will be lowest.
14. (c) Dipole moment will be highest in case of
H
acetophenone as it has strong electron
CH3 C C H withdrawing group.
15. (d) Geometrical isomerism is obseved when
H H different groups are attached to each of the
7. (d) or
doubly bonded carbon atom.
H H
16. (a) Strongest nucleophile is C2H5SH.
H In this compound S is electron pair donor.
There are 15 bonds in toluene, so 30
Since S is least electronegative, hence, its
-electrons.
tendency to donate electron pair is highest.
8. (c) Magnesium reacts only with H2O to form
17. (d) Among 1, 2- and 1, 3- configurations, 1, 3-
insoluble Mg(OH)2 and not with alcohol.
is more stable due to less repulsion.
Mg 2H 2 O Mg(OH) 2 H2
Further among cis and trans isomers, trans
9. (b) 2, 4-Dimethylcyanopentane is more stable due to less crowding.
5 4 3 2
C H3 CH CH3
C H CH3 18. (a)
| | 19. (c)
5 CH 1
3 CN
10. (a) The IUPAC name is 3-methylcyclohexene. 20. (b) In -D glucoside –OCH3 group at C1 is
towards right while in -D glucose, it is
CH3 towards left at C1. Such pairs which differ
3
2
around only C1 in configuration are called
anomers.
1
EBD_7100
C-74 Topicwise AIIMS Solved Papers – CHEMISTRY
21. (a) Some molecules react with main growing to Na2S or white precipitate due to AgCN
chain to interrupt further growth of the will be formed and thus white precipitate of
original chain. This leads to lowering of AgCl will not be identified easily.
average molecular mass of the polymer.
Such reagents are called chain transfer Na 2S 2AgNO3 2 NaNO3 Ag 2S
Black
agents. CCl4 is the only chain transfer agent
among the given options. NaCN AgNO3 NaNO3 AgCN
White
H
|
22. (b) 1
CH 2 2
CH 3
C 4
CH 2 Br NaCl AgNO3 NaNO 3 AgCl
|
CH3 white
boil
23. (b) Spin isomerism is shown by hydrogen. Na 2S 2HNO 3 2 NaNO 3 H 2S
In ortho-hydrogen, the spin of nuclei of boil
two atoms of the molecule are in same NaCN HNO 3 NaNO 3 HCN
direction whereas in case of para hydrogen,
the spin of nuclei of two atoms are in O CH3
1 2 3 4
opposite direction. 29. (a) CH3 – C – CH – CH3
24. (c) x is a conjugated diene system, w is an
isolated diene system, z is a cumulated diene 3-Methyl-2-butanone
system, y is antiaromatic system. 30. (c) :O CH3
25. (c) Diethyl ether (CH3CH2)2O is resistant to
nucleophilic attack because it does not have
an electron deficient carbon. >
O
||
C 2 H5 O C 2 H5 CH3 C OCH3 Electron-releasing, No group
Ether –OCH3 group
Methyl acetate
hence activating
O NO2
||
CH3 C N CH 3 C NH 2
Acetonitrile Acetamide >
26. (c) In case two or more chains are of equal
length, then the chain with greater number Electron-withdrawing,
of side chains is selected as the principal NO2 group
chain. hence deactivating
31. (d) –NO 2 group, being strong electron-
withdrawing, disperses the negative
7
6
5 3
2 charge, hence stabilizes the concerned
4
1 carbanion.
2, 3, 5-Trimethyl-4-propylheptane 32. (b) It has 1º and 2º alcoholic group, but not 3º
27. (b) Although (b) has positive charge on O (an 33. (c) Let us first write the resonance hybrid of
electronegative element), it is more stable the three allyl carboniun ions.
because here every atom has octet of CH3
electrons. 1 2 3

28. (b) Na2S and NaCN, formed during fusion with CH2 CH CH – CH 3 CH 2 C CH2

metallic sodium, must be removed before


adding AgNO3, otherwise black ppt. due I II
Organic Chemistry–Some Basic Principles and Techniques C-75

3 2 1 O O
CH3 – CH CH CH2

CH C C OCH3 , CH3 C C C OH
III
We know that better the dispersal of + charge,
more will be the stability of the carbonium ion. Cl OH Cl OH
Functional isomers
Further, we know that C1 and C3 carry most of the
positive charge which is H
1 2 3 1 2 3
C H2 = C H — C H — CH3 C H2— C H = C H—CH3 H2C C C COOH, H2C C CH COOH
dispersed by the methyl group (+ I group) Cl OH Cl OH
present on I and III, thus these two are more and Position isomers
equally stable than the II in which methyl group 36. (a)
is present on C2 which carry little of the positive (a)
charge.
34. (b) In diphenylmethane monochlorination at O
following positions will produce structured
CH = CH – OH CH2 – CH
isomers
enol form keto form

CH2
(b) O O cannot tautomerise

1 (c)
2 3 4
O OH O, OH

Cl CH2 CH2
O keto form O OH OH
(2) enol form
(1) Cl
(d)
Cl
CH2 CH
O OH , OH
Cl (3) (4)
35. (a) H H O O enol form OH
C C keto form
Cl CHOHCOOH 37. (d)
cis

H CHOHCOOH 38. (a) H H


C C sp2
Cl H H N
trans
Geometric isomers sp2 sp sp
COOH 39. (b) If the organic compound also contains
nitrogen or sulphur, the Lassaigne's extract
C on boiling with dil. HNO3 decomposes
H CH CH
sodium cyanide or sodium sulphide formed
HO
Cl during fusion.
COOH NaCN HNO3 NaNO 3 HCN
Na 2S 2HNO3 2NaNO 3 H 2S
HClC HC C
H If cyanide and sulphide ions are not
OH decomposed, they will react with silver
Optical isomers nitrate and hence will interfere with the test.
EBD_7100
C-76 Topicwise AIIMS Solved Papers – CHEMISTRY

NaCN AgNO3 AgCN NaNO 3


determining step, more is the stability of
Silver carbocation higher will be the reactivity of
cyanide alkyl halides towards SN1 route. Now we
(White ppt.)
know that stability of carbocations follows
Na 2S 2AgNO3 Ag 2S 2NaNO3 the order : 3° > 2° > 1°, so SN1 reactivity
Silver
sulphide should also follow the same order.
(Black ppt.) 3° > 2° > 1° > Methyl (SN1 reactivity)
H
43. (d)
44. (a) Hybridisation of carbon in CH 3 is sp2 and
40. (a) C in CH4 its hybridisation is sp3
OH
45. (b) The order of stability of free radicals
(Diphenyl methanol) • • • •
(C 6 H 5 ) 3 C (C 6 H 5 ) 2 CH (CH 3 ) 3 C (CH 3 ) 2 C H
It does not contain any chiral carbon atom. The stabilisation of first two is due to
CH3 resonance and last two is due to inductive
1 2| 3 4 5 6 7 8
41. (d) CH3CHCH2CH CHCH CH CH — CH3
effect.
2,8-Dimethyl-4,6-decadiene |
9
CH2 — CH3
10
Type B : Assertion Reason Questions
46. (c) Assertion is true but reason is false.
42. (a) Me Br ionisation
+
+Br
– NH4Cl suppresses the ionisation of NH4OH
Me
due to common ion effect and so ions of
(A) third group get precipitated as their
Me
ionisation Me Me
hydroxides.

+ + Br 47. (b)
Br
(B) 48. (c) It is correct statement that NF3 is a weaker
Me
ligand than N(CH3)3, the reason is that
ionisation Me –
Me
Me + + Br fluorine is highly electronegative therefore,
Br
(C) it with draw electrons from nitrogen atom.
Hence, the lone pair of nitrogen atom cannot
Since SN1 reactions involve the formation of be ligated. While N(CH3)3 is a strong ligand
carbocation as intermediate in the rate because CH3 is electron releasing group.
13 Hydrocarbons
TYPE A : MULTIPLE CHOICE QUESTIONS 8. C6 H 6 CH3Cl
AlCl3
C6H5CH3 +HCl
Benzene Methyl chloride Toluene
1. Which is not aromatic compound ? [1997]
(a) Cyclohexane (b) Trinitrotoluene The above reaction is: [1998]
(c) Picric acid (d) Xylene (a) Wurtz Fittig reaction
2. 1-Butyne and cold alkaline KMnO4 react to (b) Grignard reaction
produce : [1997] (c) Friedel-Craft's reaction
(a) CH3 CH2COOH (d) Ullmann reaction
(b) CH3 CH2COOH + CO2 9. Geometrical isomerism is possible in case of :
(c) CH3 CH2COOH + HCOOH (a) tartaric acid (b) 1-butene [1999]
(d) CH3 CH2CH2COOH (c) 2-butene (d) propene
3. Which is used as antiknock in petrol ? [1997] 10. Alkynes usually show which type of reaction?
(a) Tetraethyl lead (b) Tetramethyl lead [1999]
(c) Tetrapropyl lead (d) Tetrabutyl lead (a) Substitution (b) Elimination
4. In the following reaction, Z is identified as (c) Addition (d) Replacement
11. The product obtained by treating benzene with
Z
CH CH CH 3 CHO [1997] chlorine in presence of ultraviolet light is:
(a) concentrated H2SO4 [1999]
(b) CH3COCl (a) CCl 4 (b) C6 H 5Cl
(c) 20 % H2SO4 + HgSO4
(d) CH3OH (c) C6 H 6Cl6 (d) C 6 Cl 6
5. The number of and bonds present in ethene is : 12. The natural gas mainly contains : [1999]
(a) methane (b) propane
[1997]
(c) butane (d) pentane
(a) 6 (b) 3
13. Which compound can be sulphonated easily ?
(c) 4 , 2 (d) 5 , 1
(a) benzene (b) toluene [1999]
6. Glycerol contains [1997] (c) nitrobenzene (d) chlorobenzene
(a) one primary and two secondary alcoholic 14. With ammonical cuprous chloride solution, a
groups reddish brown precipitate is obtained on treating
(b) two primary and one secondary alcoholic with :
groups (a) CH 4 (b) C2H 4 [2001]
(c) one primary, one secondary and one tertiary
alcoholic groups (c) C2H 2 (d) C3H 6
(d) one secondary and two tertiary alcoholic 15. The boiling points of four saturated
groups hydrocarbons are given below. Which boiling
point suggests maximum number of carbon
7. Prestone is a mixture of: [1998]
atoms in its molecule : [2001]
(a) Glycol + H2O (b) Glycerol + H2O (a) – 162° C (b) – 88.6° C
(c) Acetone + H2O (d) propanal + H2O (c) – 0.5° C (d) – 42.2° C
EBD_7100
C-78 Topicwise AIIMS Solved Papers – CHEMISTRY
16. The size of C – C bond in benzene is [2002] 25. Which of the following sequence of reactions
(a) 1.22 Å (b) 1.54 Å (reagents) can be used for the conversion of
(c) 1.39 Å (d) 1.56 Å C 6 H 5CH 2 CH 3 into C 6 H 5CH CH 2 ?[2006]
17. Thermite is a mixture of iron oxide and [2002]
(a) zinc powder (a) SOCl 2 : H 2 O (b) SO 2 Cl 2 : alc KOH
(b) potassium metal (c) Cl 2 / hv : H 2 O (d) SOCl 2 : alc KOH
(c) sodium shavings 26. Propene on hydroboration and oxidation produces
(d) aluminium powder (a) CH3CH2CH2OH [2007]
18. The treatment of benzene with isobutene in the (b) CH3CHOHCH3
presence of sulphuric acid gives : [2003] (c) CH3CHOHCH2OH
(a) iso-butylbenzene (b) tert-butylbenzene (d) CH3CH2CHO.
(c) n-butylbenzene (d) no reaction
19. The compund having only primary hydrogen CH 2CH = CH2
atoms is : [2004] 27. [2007]
(a) isobutene (b) 2, 3-dimethylbutene-1
on mercuration and demercuration produces
(c) cyclohexane (d) propane
20. Among the following, the aromatic compund is : CH 2CHOHCH3
(a)
(a) (b) [2004]
+ + CH2CH2CH2OH
(b)

(c) (d) CH 2CHOHCH 2OH


– (c)
21. Which of the following gives propyne on
hydrolysis? [2005] (d) none of these.
(a) Al4C3 (b) Mg2C3 28. Which of the following species participate in
(c) B4C (d) La4C3 sulphonation of benzene ring ? [2007]
22. The major product obtained on treatment of (a) H2SO4 (b) SO3
CH3CH2CH(F)CH3 with CH3O–/CH3OH is : (c) HSO3– (d) SO2–
(a) CH3CH2CH(OCH3)CH3 [2005] 29. The most important method of preparation of
(b) CH3CH = CHCH3 hydrocarbons of lower carbon number is [2009]
(c) CH3CH2CH = CH2 (a) Pyrolysis of higher carbon number of
(d) CH3CH2CH2CH2OCH3 hydrocarbons
23. 3-Phenylpropene on reaction with HBr gives (as (b) Electrolysis of salts of fatty acids
a major product) : [2005] (c) Sabatier and Senderen’s reaction
(a) C6H5CH2CH(Br)CH3 (d) Direct synthesis
(b) C6H5CH(Br)CH2CH2OCH3 30. The alkene R – CH = CH2 reacts readily with
(c) C6H5CH2CH2CH2Br B2H6 and formed the product B which on
(d) C6H5CH(Br)CH = CH2 oxidation with alkaline hydrogen peroxide
24. Below, some catalysts and corresponding produces [2010]
proceses/reactions are matched. The mismatch is : (a) R – CH2 – CHO
(b) R – CH2 – CH2 – OH
(a) [RhCl(PPh3 )2 ] : Hydrogenation [2006]
(c) R C O
(b) TiCl 4 Al(C 2 H 5 ) 3 : Polymerization |
CH3
(c) V2 O 5 : Haber-Bosch process R CH CH2
(d)
(d) Nickel-Hydrogenation | |
OH OH
Hydrocarbons C-79

31. 1-Butyne can be distinguished most easily from CH3 CH3


(a) C=C
2-butyne by [2011] H H
(a) bromine water (b) ozonolysis (b) CH 3C CCH 3
(c) Tollen’s reagent (d) KMnO4 solution (c) CH 3CH 2C CH
32. Compound X of molecular formula C4H6 takes (d) CH 2 CH C CH
up one equivalent of hydrogen in presence of Pt
to form anoth er compound Y which on 37. Predict the nature of P in the following reaction
NaNH 2 / inert solvent
ozonolysis gives only ethanoic acid. The CH 3 C CCH 3 P
compound X can be [2011] heat
[2014]
(a) CH 2 CH CH CH 2
(a) CH2=CHCH=CH2
(b) CH2 = C = CHCH3 (b) CH2=C=CH–CH3
(c) CH 3C CCH 3 (c) CH3CH2C CH
(d) All the three (d) No reaction
33. The chemical system that is non-aromatic is 38. Which of the followin g would not give
2-phenylbutane as the major product in a Friedel-
Crafts alkylation reaction ? [2014]
(a) [2012] (a) 1-butene + HF
(b) 2-butanol + H2SO4
(c) Butanoyl chloride + AlCl3 then Zn, HCl
(b) (d) Butyl chloride + AlCl3
39. Which is the most suitable reagent among the
following to distinguish compound (3) from rest
+
of the compounds ? [2015]
1. CH3– C C – CH3
(c)
2. CH 3 CH 2 CH 2 CH 3
3. CH 3 CH 2C CH
+ –
(d) 4. CH 3 CH CH 2.
(a) Bromine in carbon tetrachloride
34. Consider the following statements : A (b) Bromine in acetic acid
hydrocarbon of molecular formula C5H10 is a (c) Alk KMnO4
I. monosubstituted alkene (d) Ammonical silver nitrate.
II. disubstituted alkene 40. The alkene that will give the same product with
III. trisubstituted alkene HBr in the absence as well as in the presence of
Which of the following statement(s) is(are) peroxide is
correct? [2012] [2016]
(a) I, II and III (b) I and II (a) 2-butene (b) 1-butene
(c) II and III (d) I and III (c) propene (d) 1-hexene
41. The end product (C) in the following sequence of
35. Which one of the following cannot be prepared
reactions is [2017]
by Wurtz reaction ? [2012]
1% HgSO 4 CH 3MgX [O]
(a) CH4 (b) C2H6 HC CH A B (C)
20% H 2SO 4 H 2O
(c) C3H8 (d) C4H10
36. Which of the following has the lowest dipole (a) acetic acid (b) isopropyl alcohol
moment ? [2013] (c) acetone (d) ethanol
EBD_7100
C-80 Topicwise AIIMS Solved Papers – CHEMISTRY
42. In the given reaction 47. Assertion : Rates of nitration of benzene and
X hexadeuterobenzene are different.
CH3CH 2 CH CHCH3
Reason : C–H bond is stronger than C–D bond.
CH3CH 2COOH CH3COOH [2005]
The X is [2017] 48. Assertion : Cyclopentadienyl anion is much
more stable than allyl anion.
(a) C2 H5ONa Reason : Cyclopentadienyl anion is aromatic in
(b) Conc. HCl +Anhy.ZnCl2 character. [2005]
(c) Anh. AlCl3 49. Assertion : 1, 3-Butadiene is the monomer for
(d) KMnO4/OH– natural rubber.
TYPE B : ASSERTION REASON QUESTIONS Reason : Natural rubber is formed through
anionic addition polymerization. [2006]
Directions for (Qs. 43-53) : These questions consist 50. Assertion : Addition of HBr on 2-butene gives
of two statements, each printed as Assertion and two isomeric products.
Reason. While answering these questions, you are Reason : Addition of HBr on 2-butene follows
required to choose any one of the following five Markovnikov rule. [2006]
responses. 51. Assertion : trans-butene-2 on reaction with
(a) If both Assertion and Reason are correct and bromine forms racemic mixture.
the Reason is a correct explanation of the Reason : trans-Compound in trans addition
Assertion. forms two types of stereoisomers. [2007]
(b) If both Assertion and Reason are correct but 52. Assertion : Acetylene on reacting with
Reason is not a correct explanation of the sodamide gives sodium acetylide and ammonia.
Assertion. Reason : sp hybridised carbon atoms of
(c) If the Assertion is correct but Reason is incorrect. acetylene are considerably electronegative.
(d) If both the Assertion and Reason are incorrect. [2007]
(e) If the Assertion is incorrect but the Reason is 53. Assertion :Friedel-Craft's reaction is used to
correct. introduce an alkyl or acyl group in benzene
nucleus.
43. Assertion : CH4 does not react with Cl2 in dark.
Reason : Benzene is a solvent for the Friedel-
Reason : Chlorination of CH4 takes place in
Craft's alkylation of bromobenzene. [2008]
sunlight. [2001]
Directions for (Qs.54-61) : Each of these questions
44. Assertion : Alkylbenzene is not prepared by
contains an Assertion followed by Reason. Read them
Friedel-Craft alkylation of benzene.
carefully and answer the question on the basis of
Reason : Alkyl halides are less reactive than acyl
following options. You have to select the one that
halides. [2003]
best describes the two statements.
45. Assertion : trans-2-Butene on reaction with Br2
gives meso-2, 3-dibromobutane. (a) If both Assertion and Reason are correct and
Reason : The reaction involves syn-addition of Reason is the correct explanation of Assertion.
bromine. [2003] (b) If both Assertion and Reason are correct, but
Reason is not the correct explanation of
46. Assertion : 2-Bromobutane on reaction with
Assertion.
sodium ethoxide in ethanol gives 1-butene as a
(c) If Assertion is correct but Reason is incorrect.
major product.
(d) If both the Assertion and Reason are incorrect.
Reason : 1-Butene is more stable than 2-butene
54. Assertion : 1-Butene on reaction with HBr in
[2004]
the presence of a peroxide produces 1-bromo-
According to Saytzeff's rule, 2-butene should
be the product which is more branched or butane.
substituted compound and hence, more stable Reason : It involves the free radical mechanism.
than butene-1 [2009]
Hydrocarbons C-81

55. Assertion : Benzene removes a butter stain 60. Assertion : Tropylium cation is aromatic in
from a table cloth. nature
Reason : Butter has an affinity towards benzene.
[2010]
56. Assertion : Trans-2-butene on reaction with Br2
gives meso-2, 3-dibromobutane. Reason : The only property that determines its
Reason : The reaction involves syn-addition of aromatic behaviour is its planar structure.
bromine. [2009, 2014]
[2014, 2016]
57. Assertion : 1-Butene on reaction with HBr in the
61. Assertion : Benzene exhibit two different bond
presence of a peroxide produces 1-bromobutane.
lengths, due to C – C single and C = C double
[2015]
bonds. [2017]
Reason : It involves the formation of a primary
Reason : Actual structure of benzene is a
radical.
58. Assertion : Nitrating mixture used for carrying hybrid of following two structures.
out nitration of benzene consists of conc. HNO3
+ conc. H2SO4. [2015]
Reason : In presence of H2SO4, HNO3 acts as a
base and produces NO2+ ions.
59. Assertion : Energy of resonance hybrid is equal
to the average of energies of all canonical forms.
Reason : Resonance hybrid cannot be presented
by a single structure. [2016]
EBD_7100
C-82 Topicwise AIIMS Solved Papers – CHEMISTRY

Type A : Multiple Choice Questions 11. (c) In presence of sunlight, benzene reacts with
1. (a) Aromatic compounds are closed chain chlorine to form addition product.
planar compunds with (4n + 2) electrons H Cl
and show delocalization of electrons.
Cl H
Cyclohexane does not coincide with this
definition as it does not have benzene ring, H Cl
+ 3Cl2 Cl H
while other three have benzene ring.
H Cl
KMnO4
2. (b) CH3 CH 2 C CH + Cl H
CH3CH2COOH + CO2
Benzene hexachloride
3. (a) Tetraethyl lead (TEL) is used as antiknock
in petrol. 12. (a) Natural gas contains mainly methane
HgSO4 / H 2SO4 13. (b) Sulphonation is electrophilic substitution
4. (c) C2 H 2 CH3CHO
H 2O reaction of benzene. This reaction is
So, z is 20% H 2SO4 HgSO 4 facilitated by any group having + I effect
H H (inductive effect). As CH3 has + I effect,
toluene facilitates this reaction most.
5. (d) C C 14. (c) Acetylene forms brown copper acetylide
H H with ammonical cuprous chloride solution.
No. of bonds 5 and no. of bond = 1
6. (b) The structure of glycerol is Cu 2 Cl 2 C2 H 2 Cu 2 C 2 2HCl
CH 2 OH 2HCl 2NH 4 OH 2NH 4Cl 2H 2 O
|
CHOH 15. (c) The heavier the molecule, greater is the
| boiling point. So molecule with boiling point
CH 2 OH – 0.5°C will have maximum number of carbon
It contains two primary and one secondary atoms.
alcoholic groups. 16. (c) The size of C – C bond in benzene is 1.39 Å
7. (a) Prestone is a mixture of glycol & H2O. It
which lies between 1.34 Å (bond length of
has freezing point much below 0°C, hence
C – C) and 1.54 Å (bond length of C = C).
it is used as an antifreeze for automobile
radiators. 17. (d) Thermite is a mixture of iron oxide and Al
8. (c) The given reaction is known as Friedel- powder. Al reduces iron oxide to iron giving
Craft reaction. out enormouns heat.
9. (c) Geometrical isomerism is shown by Fe 2 O 3 2Al 2Fe Al 2 O 3 + Heat
molecules containing double bond having
unlike groups on each of the doubly 18. (b) CH 2 C CH3 H CH3 C CH3
| |
bonded carbon atom. CH3 CH3
10. (c) In most cases, alkynes show addition Highly stable
reactions as they contain two double
bonds. In some cases, it undergoes CH 3
|
substitution reaction. CH 3 C CH 3
C2H2 + AgNO3 + 2NH4OH
C2Ag2 + 2NH3 + 2H2O
Silver + (CH3)3 C
acetylide
This reaction occurs only in terminal
alkynes.
Hydrocarbons C-83

p p p (Hydroboration)
CH3 CH 3 CH 3 3CH3– CH = CH2 + BH3
19. (a) p | p p | | p
CH 3 — C CH 2 CH 3 — C Hs — C CH 2
H 2 O2 /OH
(CH3CH2CH2)3B (Oxidation)
p s p 3 CH3CH2CH2OH
CH3 — CH 2 — CH3
All H’s are secondary
20. (a) According to Huckel's rule, the cyclic planar CH2CH=CH2
27. (a) (CH3COO)2Hg
conjugated system having (4n +2) (mercuration)
electrons show aromaticity.
(NaBH4/NaOH
CH2CH – CH2–Hg
THF
(demercuration)
+ + – OCOCH3
2 electrons 4 electrons 4 electrons 4 electrons
21. (b) Mg2C3 gives propyne.
Mg 2 C3 4H 2 O 2Mg(OH) 2 CH3 C CH CH2–CH–CH3
22. (b) CH3CH 2 CH(F) CH3 is a secondary OH
halide. So, it will undergo dehydro-
halogenation to form alkene. 28. (b) SO 3 participates in sulphonation of
benzene.
CH3O –
CH3CH 2 CH(F) CH3
Pyrolysis
29. (a) C 6 H14 C2 H 4 C4 H10
CH3 CH CH CH3
(Saytzeff product) Hexane Ethene Butane
23. (a) CH2 – CH = CH2 B 2H 6
+
30. (b) 6R CH CH 2 2(RCH 2CH 2 ) 3 B
+ H from HBr Ether,0 C

H 2O 2
3-Phenylpropene
6RCH 2 CH 2 OH 2H 3 BO 3 .
NaOH

+ Br 31. (c) Tollen’s reagent is ammonical silver nitrate


CH2 CH CH3 | which reacts with 1-alkynes to form white
CH2 – CH – CH3 percipitate of silver alkynide.
Br
– CH3 CH 2 C CH AgNO3 NH 4 OH
more stable 2°carbocation
CH3 CH 2 C CAg NH 4 NO3 H2 O
24. (c) V2 O5 is used as a catalyst in contact white ppt.
process for the manufacture of SO3 and 32. (d) Formation of only CH3COOH by ozonolysis
hence H2SO4. In Haber-Bosch process for indicates that the compound Y should be
the manufacture of NH3, finely divided Fe CH3CH = CHCH3 which can be formed by
all of the three given compounds
+ molybdenum are used.
1H 2 /Pt
25. (b) SO2Cl2 causes free radical substitution at CH 2 CH CH CH 2
benzylic position. X
CH 3 CH CH CH 3
SO 2 Cl2 Y
C 6 H 5CH 2CH3 C 6H 5CHClCH 3 1H 2 /Pt
CH3C CCH3 CH3CH CHCH3
alc
C 6 H 5CH CH 2 X Y
KOH 1H 2 /Pt
26. (a) Propene on hydroboration and oxidation CH 2 C CHCH 3
X
produces propanol. O3
CH 3CH CHCH 3 2CH 3COOH
Y
EBD_7100
C-84 Topicwise AIIMS Solved Papers – CHEMISTRY
33. (c) 34. (a) undergoes alkylation to produce
35. (a) CH4 has only one carbon atom, hence it polyalkylated benzene. Further the reason
can’t be prepared by Wurtz reaction, which that "acyl halides are more reactive than
involves two molecules of alkyl halide. alkyl halides" although not correct
36. (b) CH 3C CCH 3 , being symmetrical, has the explanation of the assertion it is true
lowest dipole moment because acyl halides are more electron
deficient than alkyl halides
37. (c) When non-terminal alkynes are heated with
+ +
NaNH2 in an inert solvent, the triple bond CH 3 CH2
CH 3— C = O
migrates to the end carbon atom. more electron less electron
38. (c) The Friedal-crafts alkylation reaction will deficient deficient
give propyl phenyl ketone which further Br
on Clemmenson’s reduction will give butyl H CH 3 H CH 3
benzene
39. (d) Br2 in CCl4 (a), Br2 in CH3COOH (b) and C C
45. (c) || +Br2 |
alk. KMnO 4 (c) will react with all C C
unsaturated compounds, i.e., 1, 3 and 4 while
ammonical AgNO3 (d) reacts only with CH3 H CH3 H
terminal alkynes, i.e., 3 and hence 3 can be trans-2-Butene Br
distinguished from 1, 2 and 4 by. ammonical meso-2,3-Dibromobutane
AgNO3 (d). The reaction is trans addition of Br2 and
40. (a) The addition of HBr takes place according not syn addition.
to anti-Markovnikoff’s rule in presence of
Br
peroxide for unsymmetrical alkenes. |
The addition of HBr to symmetrical alkenes 46. (d) CH 3 CH 2 CH CH 3
is not affected by the presence or absence
of peroxide. CH 3 CH CH CH 3
41. (c) 2–Butene
47. (d) Rates of nitration of benzene and
1%HgSO4 CH3MgX
HC CH CH 3CHO hexadeuterobenzene are same because the
20%H 2SO4 [A ] H 2O rate determining step (formation of
[O] carbocation) is same in both the cases, i.e.,
CH 3CHOHCH 3 CH 3COCH 3 H NO2
[ B] Acetone [C]
+
42. (d) A doubly bonded carbon atom having an + NO2
+
alkyl group is oxidised to aldehyde which
is further oxidised to carboxylic acid. D NO2
D
(i) KMnO 4 ,OH D D D + D
CH3CH 2CH CH CH3 +
(ii) H + NO2
CH3CHO CH3CH 2CHO D D D D
D D
CH3COOH CH3CH 2COOH it does not involve the cleavage of C —H /
C— D bond which takes in the second step.
Type B : Assertion Reason Questions H –
:

43. (b) Chlorination of CH4 takes place either in


presence of light or at high temperature, 48. (a)
and not in dark because in darkness, Cl free
radicals are not produced.
Cyclopentadienyl anion contains 6
44. (b) Alkylbenzene is not prepared by Friedel-
electrons, so it is aromatic and stabilised
Craft alkylation because monoalkyl product
Hydrocarbons C-85

by reasonance. Thus it is more stable than alkylation of bromobenzene, benzene will


allyl anion which is not aromatic, although be alkylated in preference to bromobenzene
it is also stabilised by resonance. because benzene is more reactive for SE
49. (d) Natural rubber is polymer of isoprene. Thus than bromobenzene.
assertion is false. Further the reason is also 54. (a) This reaction takes place against
false because 1, 3- butadiene undergoes Markovnikoff ’s srule
free radical polymeration rather than anionic. H H H H
50. (c) CH3– CH = CH – CH3 on reaction with HBr | | | |
gives H C C C C + HBr Peroxide
H | | |
| H H H
CH3 – CH 2 – C– * CH 1-Butene
3
| H H H H
Br | | | |
which contains one chiral carbon. So, it will H C C C C Br
give two optical isomers. Hence A is correct. | | | |
Since 2-butene is symmetrical molecule so H H H H
1-Bromobutane
it will not follow Markownikov rule. Thus R
is false. In this reaction anti-Markovnikoff’s addition
51. (d) On anti addition of Br2 to trans-butene-2, is explained on the basis of the fact that in
we get meso compounds the presence of peroxide the addition takes
place via a free radical mechanism.
CH3 CH3 55. (b) Benzene is a non-polar solvent. Butter is
H – C – CH3 H Br Br H composed of organic compounds of low
+ Br2 +
CH3 –C – H H Br Br H polarity. So, it gets dissolved in benzene.
CH3 CH3 56. (c) The assertion that trans-2 butene reacts
with Br 2 to product meso-2,
While syn addition gives a racemic mixture. 3-dibromobutane is correct but it does not
Hence both assertion and reason are false. involve syn-addition of Br 2.
52. (a) Acetylene on reaction with sodamide gives 57. (c) Here assertion is correct but reasoning is
sodium acetylide and ammonia. incorect. In presence of peroxide, addition
of HBr on alkenes takes place via free
NaNH 2
CH CH HC C Na NH3 radicals; here two free radical are formed,
monosodium 2° free radical, being more stable, governs
acetylide the product.
Here formation of sodium acetylide can be • Br •
CH3CH 2CHBr CH 2 CH3CH 2 CH=CH 2
explained by sp-hybridisation of carbon 1° free radical
atom. As we know that an electron in (less stable)
s-orbital is more tightly held than in a Br• • •
p-orbital. In sp hybridisation, s-character is CH3CH 2 C HCH 2 Br CH 3CH 2 C HCH 2 Br
2° free radical 2 free radical
50% as compared to sp2 (33%) or sp3(25%). (more stable) (more stable)
So, due to large s-character the carbon atom
58. (a) HNO3 2H 2SO 4
is quite electronegative and hence Na of
NaNH2 can replace H+ of C–H bond. 2HSO4 NO2 H3O
Hence assertion and reason both are true and 59. (d)
reason is the correct explanation of assertion. 60. (c) (4n + 2) electrons and planar structure are
53. (c) Yes, we use Friedel-Crafts reaction for the essential conditions for aromaticity.
introducing an alkyl or acyl group in 61. (c) Benzene has a uniform C – C bond
benzene nucleus. Thus, assertion is true. distance of 139 pm, a value intermediate
However, the reason is not true because if between the C – C single. (154 pm) and C
benzene is used as a solvent, during = C double (134 pm) bonds.
EBD_7100
C-86 Topicwise AIIMS Solved Papers – CHEMISTRY

14 Environmental Chemistry

TYPE A : MULTIPLE CHOICE QUESTIONS (iii) Carbon dioxide


(iv) Mists, smoke and dust
1. Nitrolium is; [1999] (v) Smog [2016]
(a) CaCN 2 (b) Ca (CN) 2 (a) (i), (iv) and (v) (b) (iii) only
(c) (ii) only (d) (ii) and (v)
(c) CaCN 2 C (d) Ca ( NO 3 ) 2
TYPE B : ASSERTION REASON QUESTIONS
2. Which one of the following is not a green house
gas? [2001] Directions for (Qs.7-8) : Each of these questions
(a) CO 2 (b) H2O contains an Assertion followed by Reason. Read them
carefully and answer the question on the basis of
(c) N 2 (d) O 3 following options. You have to select the one that
3. Which of the following is not a green house best describes the two statements.
gas? [2003]
(a) If both Assertion and Reason are correct and
(a) Carbon dioxide (b) Water vapour
(c) Oxygen (d) Methane Reason is the correct explanation of Assertion.
4. Ozone in stratosphere is depleted by : [2004] (b) If both Assertion and Reason are correct, but
(a) CF2Cl2 (b) C7F16 Reason is not the correct explanation of
(c) C6H6Cl6 (d) C6F6 Assertion.
5. The secondary precursors of photochemical (c) If Assertion is correct but Reason is incorrect.
smog are [2014] (d) If both the Assertion and Reason are incorrect.
(a) SO2 and NO2 7. Assertion : Photochemical smog is produced
(b) SO2 and hydrocarbons by nitrogen oxides.
(c) NO2 and hydrocarbons Reason : Vehicular pollution is a major source of
(d) O3 and PAN nitrogen oxides. [2003]
6. The irritant red haze in the traffic and congested 8. Assertion : Suspended particulate matter (SPM)
places is due to presence of which of the is an important pollutant released by diesel
following ? vehicles. [2015]
(i) Oxides of sulphur Reason : Catalytic converters greatly reduce
(ii) Oxides of nitrogen pollution caused by automobiles.
Environmental Chemistry C-87

Type A : Multiple Choice Questions Type B : Assertion Reason Questions


7. (b) It is correct that photochemical smog
1. (c) Nitrolium is the name of a fertilizer which is is produced by oxides of nitrogen and
chemically a mixture of CaCN2 + C. It reacts it is also a fact that vehicular pollution
with water to form ammonia, vital for plants. is a major source of nitrogen oxides
CaCN2 + 3H2O CaCO3 + 2NH3 but it is not the correct explanation of
assertion.
2. (c) Most of the radiations of the sun falling on
8. (b) SPM (Suspended Particulate Matter) is
earth are radiated back in the space.
defined as particles floating in the air
However, certain atmospheric gases, like
with a diameter below 10 m. Studies
CO2 , O 3 , water vapour, chlorofluoro-
have shown that high SPM
carbons, oxides of nitrogen etc. trap these
concentrations in the air can have a
radiations. This is known as green house detrimental impact on respiratory organs.
effect. SPM is generated from natural sources
3. (c) Greenhouse gases (GHGs) are gaseous (e.g., volcanoes or dust storms) and
components of the atmosphere that human activities (vehicles, incinerators
contribute to the “greenhouse effect”. and industrial plants).
Although uncertainty exists about exactly
how earth’s climate responds to these SPM Other aerosols
gases, global temperatures are rising. Some Less than 10 m Less than 100 m
greenhouse gases occur naturally in the Tend to float longer in Tend to settle fairly
atmosphere, while others result from human Air due to small size due to comparative
activities. Naturally occurring greenhouse quickly heaviness
gases include water vapor, carbon dioxide,
methane, nitrous oxide, and ozone. Certain Catalytic converters is a device designed
human activities, however, add to the levels to reduce the amount of emissions f
of most of these naturally occurring gases. rom automobiles. The current (so-called
Note : Greenhouse gases act as blanket to three-way) systems use a heated metal
the earth and keep it at about 33° C; warmer catalyst to reduce the emissions of
than it would be without these gases in the carbon monoxide (CO), hydrocarbons,
atmosphere. and nitric oxide (NO), all of which
4. (a) Ozone is depleted by CF2Cl2 (dichloro- contribute to the formation of
difluoromethane or freon.) photochemical smog. In an automobile’s
5. (d) exhaust system, a catalytic converter
6. (c) The irritant red haze in the traffic and provides an environment for a chemical
congested places is due to the presence of reaction where unburned hydrocarbons
oxides of nitrogen. completely combust.
EBD_7100
C-88 Topicwise AIIMS Solved Papers – CHEMISTRY

15 The Solid State

TYPE A : MULTIPLE CHOICE QUESTIONS 9. The coordination number in hcp is [2007]


(a) 6 (b) 12
1. Schottky defect defines imperfection in the (c) 18 (d) 24
lattice structure of [2002] 10. The space lattice of graphite is [2009]
(a) solid (b) gas (a) Cubic (b) Tetragonal
(c) liquid (d) plasma (c) Rhombic (d) Hexagonal
2. An AB2 type structure is found in [2002] 11. Coordination numbers of Zn 2+ and S2– in the
(a) NaCl (b) CaF2 crystal structure of wurtzite are [2010]
(c) Al2O3 (d) N2O (a) 4, 4 (b) 6, 6
3. An element (atomic mass 100 g/mol) having bcc (c) 8, 4 (d) 8, 8
structure has unit cell edge 400 pm. The density 12. Gold has a face centred cubic lattice with an edge
of element is (No. of atoms in bcc, Z = 2). [2002] length of the unit cube of 407 pm. Assuming the
(a) 2.144 g/cm3 (b) 7.289 g/cm3 closest packing, the diameter of the gold atom is
(c) 5.188 g/cm3 (d) 10.376 g/cm3 (a) 576.6 pm (b) 287.8 pm [2011]
4. What is the coordination number of sodium in (c) 352.5 pm (d) 704.9 pm
Na2O? [2003] 13. Which is not correct about the Schottky defects?
(a) 6 (b) 4 [2011]
(c) 8 (d) 2
(a) Both cations and anions are missing from
5. The crystal system of a compound with unit cell
their lattice sites without affecting the
dimensions "a = 0.387, b = 0.387 and c = 0.504
nm and = = 90° and = 120°" is : [2004] stoichiometry of the compound
(a) cubic (b) Because of presence of holes the lattice
(b) hexagonal energy decreases.
(c) orthorhombic (c) The presence of holes causes the density
(d) rhombohedral of the crystal to decrease.
6. If z is the number of atoms in the unit cell that (d) The defect increases th e electrical
represents the closest packing sequence ..... ABC conductivity of the solid due to migration
ABC ......, the number of tetrahedral voids in the of the ions into the holes.
unit cell is equal to : [2005] 14. The existence of a substance in more than one
(a) z (b) 2z solid modifications is known as [2012]
(c) z/2 (d) z/4
7. The compound, found in nature in gas phase (a) isomorphism
but ionic in solid state is : [2006] (b) Polymorphism
(a) PCl5 (b) CCl4 (c) Amorphism
(c) PCl3 (d) POCl3
8. The Ca2+ and F– are located in CaF2 crystal, (d) Allotropy
respectively at face centred cubic lattice points 15. An element (atomic mass = 100 g / mol) having
and in [2006] bcc structure has unit cell edge 400 pm. Then,
(a) Tetrahedral voids density of the element is [2013]
(b) Half of tetrahedral voids (a) 10.376 g/cm 3 (b) 5.188 g/cm 3
(c) Octahedral voids
(d) Half of octahedral voids (c) 7.289 g/cm3 (d) 2.144 g/cm3
The Solid State C-89

16. If AgI crystallises in zinc blende structure TYPE B : ASSERTION REASON QUESTIONS

with I ions at lattice points. What fraction of
tetrahedral voids is occupied by Ag+ ions? Directions for (Qs.21-26) : Each of these questions
[2014] contains an Assertion followed by Reason. Read them
(a) 25% (b) 50% carefully and answer the question on the basis of
following options. You have to select the one that
(c) 100% (d) 75%
best describes the two statements.
17. Which set of following characteristics for ZnS
crystal is correct? [2015] (a) If both Assertion and Reason are correct and
Reason is the correct explanation of Assertion.
(a) Coordination number (4 : 4); ccp; Zn2+ ion
(b) If both Assertion and Reason are correct, but
in the alternate tetrahedral voids
Reason is not the correct explanation of
(b) Coordination number (6 : 6); hcp; Zn2+ ion Assertion.
in all tetrahedral voids. (c) If Assertion is correct but Reason is incorrect.
(c) Coordination number (6 : 4); hcp; Zn2+ ion (d) If both the Assertion and Reason are incorrect.
in all octahedral voids
21. Assertion : Graphite is an example of tetragonal
(d) Coordination number (4 : 4); ccp; Zn2+ ion
crystal system.
in all tetrahedral voids.
Reason : For a tetragonal system, a = b c,
18. Molecules/ions and their magnetic properties are
given below. [2016] = = 90°, = 120°. [2006]
Molecule/ion Magnetic property 22. Assertion :No compound has both Schottky
(i) C6H6 (1) Antiferromagnetic and Frenkel defects.
(ii) CrO2 (2) Ferrimagnetic Reason : Both defects change the density of the
(iii) MnO (3) Ferromagnetic solid. [2008]
(iv) Fe3O4 (4) Paramagnetic 23. Assertion : Stability of a crystal is reflected in
(v) Fe3+ (5) Diamagnetic the magnitude of its melting.
The correctly matched pairs in the above is Reason : The stability of a crystal depends upon
(a) i-5, ii-3, iii-2, iv-1, v-4 the strength of the interparticle attractive force.
(b) i-3, ii-5, iii-1, iv-4, v-2 [2009]
(c) i-5, ii-3, iii-1, iv-2, v-4 24. Assertion : Due to Frenkel defect, there is no
(d) i-5, ii-3, iii-1, iv-4, v-2 effect on the density of the crystalline solid.
19. Which one of the following statements about Reason : In Frenkel defect, no cation or anion
packing in solids is incorrect ? [2016] leaves the crystal. [2011]
(a) Coordination number in bcc mode of 25. Assertion : On heating ferromagnetic or
packing is 8. ferrimagnetic substances, they become
(b) Coordination number in hcp mode of paramagnetic.
packing is 12. Reason : The electrons change their spin on
heating. [2014]
(c) Void space in hcp mode of packing is 32%.
26. Assertion : In close packing of spheres, a
(d) Void space is ccp mode of packing is 26%.
tetrahedral void is surrounded by four spheres
20. Sodium metal crystallizes in a body centred cubic whereas an octahedral void is surrounded by
lattice with a unit cell edge of 4.29Å. The radius six spheres.
of sodium atom is approximately : [2017] Reason : A tetrahedral void has a tetrahedral
(a) 5.72Å (b) 0.93Å shape whereas an octahedral void has an
(c) 1.86Å (d) 3.22Å octahedral shape. [2015]
EBD_7100
C-90 Topicwise AIIMS Solved Papers – CHEMISTRY

Type A : Multiple Choice Questions 8. (a) F– are in tetrahedral voids. Four atoms of
Ca2+ and eight atoms of F– are in unit cell.
1. (a) Schottky defects are found in solids.
Each F– atom occupies 8 tetrahedral voids.
2. (b) AB2 A2 2B 9. (b) In this structure, each atom is surrounded
CaF2 Ca 2
2F by twelve nearest touching neighbours.
Hence coordination number is 12.
100
3. (c) Mass of two atoms = 2g 10. (d) In graphite, the carbon atoms are arranged
6.02 1023 in regular hexagons in flat parallel layers
2 10 21 and this fact is responsible for its softness
kg
6.02 103 and lubricating action.
10 3 30 11. (a) Wurtzite has face centred cubic structure in
Volume of cell = (4 10 ) 64 10
which each Zn2+ ion is attached to four S2–
mass 2 10 24 ions and each S2– ion remains in contact
Density =
volume 64 10 30 6.02 with four Zn 2+ ions. Hence coordination
2 number of each ion is 4.
Density 106 kg / m3 12. (b) For fcc lattice,
6.02 64
2 10 6 10 3 4 r = diagonal of the face = a 2
g / cc or 2 r = diameter
6.02 64 10 6
2 a 2 407× 2
10 3 g / cc = 5.188 g / cc = = =287.8 pm
6.02 64 2 2
13. (b)
4. (b) In Na2O there is antifluorite structure. Here
14. (b)
negative ions form the ccp arrangement so
that each positive ion is surrounded by 4 15. (b) z×M 2 100
=
negative ions and each negative ion is N A ×a 3 6.023 10 23
(400 10 10 ) 3
surrounded by 8 positive ion s. So
coordination no. of Na+ is 4. 5.188 g / cm 3
5. (b) a b c , 16. (b) For each central atom there are two
90 , 120
tetrahedral voids in AgI crystal. The number

These are the characteristics of a hexagonal of Ag+ ion is equal to number of I ion. It
system. means only 50% of the void will be occupied
6. (b) In cubic closed pack system (ccp system) by Ag+ ion.
.......ABC ABC....... type of arrangement of 17. (a) ZnS has cubic close packed (ccp) structure.
layers is found. In this system, there are The S2– ions are present at the corners of
atoms at the corners as well as centre of the the cube and at the centre of each face.
unit cell. Zinc ions occupy half of the tetrahedral
sites. Each zinc ion is surrounded by four
1
No. of atoms per unit cell 8 1 2 sulphide ions which are disposed towards
8 the corners of a regular tetrahedron.
Hence, the no. of tetrahedral voids in a unit Similarly, S2– ion is surrounded by four Zn2+
cell = 2z ions.
7. (a) PCl5 exists in gas phase, however it remains 18. (c) C6H6 is diamagnetic (i – 5)
in solid state in ionic form in the form of CrO2 is ferromagnetic (ii – 3)
[PCl4] + [PCl6]–. The cation is tetrahedral MnO is antiferromagnetic (iii – 1)
and anion is octahedral. Fe3O4 is ferrimagnetic (iv – 2)
The Solid State C-91

Fe3+ is paramagnetic with 5 unpaired missing, while Frenkel defects do not


electron (v – 4) involve any change in density because
19. (c) The hcp arrangement of atoms occupies number of cations and anions are same,
74% of the available space and thus has there is only change of position of ions.
26% vacant space. 23. (a) The stability of a crystal depends upon the
20. (c) In bcc the atoms touch along body diagonal strength of the interparticle attractive force.
2r 2r 3a The melting point of a solid depends on
the strength of the attractive force acting
3a 3 4.29 between the constituent particles.
r= 1.857Å
4 4 Therefore, the stability of a crystal gets
reflected in its melting point.
Type B : Assertion Reason Questions 24. (a) In a Frenkel defect an ion leaves its position
21. (d) Like quartz and ice, graphite is an example in the lattice and occupies normally vacant
of hexagonal system for which = = 90° interstitial position.
and = 120° and a b c. So, A is 25. (a) All magnetically ordered solids
incorrect and R is also incorrect as for (ferromagnetic, ferrimagnetic and
tetragonal system a = b c and = = = antiferromagnetic solids) transform to the
90°. paramagnetic state at high temperature due
22. (d) Certain ionic solids (for example, AgBr) have to the randomisation of spins.
both Schottky and Frenkel defects. Only 26. (c) Tetrahedral void is so called because it is
Schottky defects change the density of surrounded by four spheres tetrahedrally
solids because anions or cations are while octahedral void is so called because
it is surrounded by six spheres octahedrally.
EBD_7100
C-92 Topicwise AIIMS Solved Papers – CHEMISTRY

16 Solutions

TYPE A : MULTIPLE CHOICE QUESTIONS 7. 20 g of a substance were dissolved in 500 mL of


water and the osmotic pressure of the solution
1. The vapour pressure of benzene at 30°C is 121.8 was found to be 600 mm of mercury at 15°C. The
mm. By adding 15 g of non-volatile solute in 250 molecular weight of substance is : [2001]
g of benzene, its vapour pressure is decreased
(a) 998 (b) 1028
to 120.2 mm. The molecular weight of solute is :
(c) 1098 (d) 1198
(a) 156.6 g (b) 267.4 g [1997]
8. Which one of the statements given below
(c) 356.3 g (d) 467.4 g
concerning properties of solutions, describes a
2. Pure benzene freezes at 5.45°C. A 0.374 m
colligative effect? [2003]
solution of tetrachloroethane in benzene freezes
(a) Boiling point of pure water decreases by
at 3.55°C. The Kf for benzene is: [1998]
the addition of ethanol
(a) 0.508 (b) 5.08
(b) Vapour pressure of pure water decreases
(c) 50.8 (d) 508
by the addition of nitric acid
3. 0.450 g of urea (mol.wt.60) in 22.5 g of water show
0.170°C of elevation in boiling point. The molal (c) Vapour pressure of pure benzene decreases
elevation constant of water is: [1998] by the addition of naphthalene
(a) 0.051°C (b) 0.51°C (d) Boiling point of pure benzene increases by
(c) 5.1°C (d) 0.83°C the addition of toluene
4. Van't Hoff factor is: [2000] 9. The average osmotic pressure of human blood
(a) More than one in case of association is 7.8 bar at 37°C. What is the concentration of
(b) Less than one in case of dissociation an aqueous NaCl solution that could be used in
the blood stream? [2004]
normal molecular mass
(c) (a) 0.16 mol/L (b) 0.31 mol / L
observed molecular mass (c) 0.60 mol / L (d) 0.45 mol / L
observed molecular mass 10. A 5% solution (by mass) of cane sugar in water
(d)
normal molecular mass has freezing point of 271 K and freezing point of
5. On mixing 3 g of non - volatile solute in 200 mL pure water is 273.15 K. The freezing point of a
of water, its boiling point (100°C) becomes 5% solution (by mass) of glucose in water is
100.52°C. If Kb for water is 0.6 K/m then molecular (a) 271 K (b) 273.15K [2006]
wt. of solute is : [2000] (c) 269.07 K (d) 277.23 K
1 1 11. The vapour pressure of pure benzene at a certain
(a) 10.5 g mol (b) 12.6 g mol
temperature is 0.850 bar. A non-volatile, non-
1 1
(c) 15.7 g mol (d) 17.3 g mol electrolyte solid weighing 0.5 g is added to 39.0
6. The colligative property is not represented by : g of benzene (molar mass 78 g/mol). The vapour
(a) elevation in boiling point [2001] pressure of the solution then is 0.845 bar. What
(b) osmotic pressure is the molecular mass of the solid substance?
(c) optical activity (a) 58 (b) 180 [2007]
(d) relative lowering of vapour pressure (c) 170 (d) 145.
Solutions C-93

12. 1 mol each of the following compounds is 19. At a particular temperature, the vapour pressures
dissolved in 1L of solution. Which will have the of two liquids A and B are respectively 120 and
largest Tb value? [2007] 180 mm of mercury. If 2 moles of A and 3 moles
(a) HF (b) HCl of B are mixed to form an ideal solution, the
(c) HBr (d) HI. vapour pressure of the solution at the same
13. 0.01 M solution of KCl and BaCl2 are prepared in temperature will be (in mm of mercury) [2013]
water. The freezing point of KCl is found to be (a) 156 (b) 145
–2°C. What is the freezing point of BaCl 2 to be (c) 150 (d) 108
completely ionised ? [2008] 20. The freezing point of equimolal aqueous solution
(a) – 3°C (b) + 3°C will be highest for [2013]
(c) – 2°C (d) – 4°C (a) C6H5NH3+Cl– (b) Ca(NO3)2
14. An aqueous solution of hydrochloric acid (c) La(NO3)2 (d) C6H12O6
(a) Obeys Raoult’s law [2009] 21. Which of the following 0.10 m aqueous solutions
will have the lowest freezing point ? [2014]
(b) Shows negative deviation from Raoult’s law
(a) Al2(SO4)3 (b) C6H12O6
(c) Shows positive deviation from Raoult’s law
(c) KCl (d) C12 H22 O11
(d) Obeys Henry’s law at all compositions
15. At 300K, the vapour pressure of an ideal solution 22. A solution containing 10g per dm 3 of urea
containing 3 mole of A and 2 mole of B is 600 torr. (molecular mass = 60 gmol–1) is isotonic with a
At the same temperature, if 1.5 mole of A and 0.5 5% solution of a non volatile solute. The molecular
mole of C (non-volatile) are added to this solution mass of this non volatile solute is [2014]
the vapour pressure of solution increases by (a) 300 g mol–1 (b) 350 g mol–1
30 torr. What is the value of p0B ? (c) 200 g mol–1 (b) 250 g mol–1
23. The vapour pressure of a solvent decreases by
(a) 940 (b) 405 [2009]
10 mm of Hg when a non-volatile solute was
(c) 90 (d) None of these
added to the solvent. The mole fraction of the
16. Exactly 1 g of urea dissolved in 75 g of water
solute in the solution is 0.2. What should be the
gives a solution that boils at 100.114°C at
mole fraction of the solvent if the decrease in
760 torr. The molecular weight of urea is 60.1.
the vapour pressure is to be 20 mm of Hg ?
The boiling point elevation constant for water is
[2015]
(a) 1.02 (b) 0.51 [2010]
(a) 0.8 (b) 0.6
(c) 3.06 (d) 1.51
(c) 0.4 (d) 0.2
17. An aqueous solution of NaCl shows the
24. A solution of urea (mol. mass 56 g mol 1) boils
depression of freezing point of water equal to
at 100.18 C at the atmospheric pressure. If Kf
0.372 K. The boiling point of BaCl2 solution
and Kb for water are 1.86 and 0.512 K kg mol 1
of same molality will be [Kf (H2O)= 1.86 K kg
respectively, the above solution will freeze at
mol–1; Kb(H2O) = 0.52 K kg mol–1] [2011]
(a) 0.654 C (b) 0.654 C [2015]
(a) 100.52ºC (b) 100.104ºC
(c) 6.54 C (d) 6.54 C
(c) 101.56ºC (d) 100.156ºC
25. A solution containing 1.8 g of a compound
18. 12g of urea is dissolved in 1 litre of water and
(empirical formula CH2O) in 40 g of water is
68.4 g of sucrose is dissolved in 1 litre of water.
observed to freeze at –0.465° C. The molecular
The lowering of vapour pressure of first case is
formula of the compound is
(a) equal to second [2012]
(Kf of water = 1.86 kg K mol–1) [2016]
(b) greater than second
(a) C2H4O2 (b) C3H6O3
(c) less than second
(c) C4H8O4 (d) C6H12O6
(d) double that of second
EBD_7100
C-94 Topicwise AIIMS Solved Papers – CHEMISTRY
26. Which observation(s) reflect(s) colligative 29. Assertion : If red blood cells were removed from
properties? [2017] the body and placed in pure water, pressure
(i) A 0.5 m NaBr solution has a higher vapour inside the cells increases.
pressure than a 0.5 m BaCl2 solution at the Reason : The concentration of salt content in
same temperature the cells increases. [2006]
(ii) Pure water freezes at the higher temperature 30. Assertion : The water pouch of instant cold pack
than pure methanol for treating athletic injuries breaks when
(iii) a 0.1 m NaOH solution freezes at a lower squeezed and NH4NO3 dissolves lowering the
temperature than pure water temperature.
Reason : Addition of non-volatile solute into
Choose the correct answer from the codes given
solvent results into depression of freezing point
below
of the solvent. [2006]
(a) (i), (ii) and (iii) (b) (i) and (ii)
Directions for (Qs.31-33) : Each of these questions
(c) (ii) and (iii) (d) (i) and (iii) contains an Assertion followed by Reason. Read them
TYPE B : ASSERTION REASON QUESTIONS carefully and answer the question on the basis of
following options. You have to select the one that
Directions for (Qs. 27-30) : These questions consist best describes the two statements.
of two statements, each printed as Assertion and (a) If both Assertion and Reason are correct and
Reason. While answering these questions, you are Reason is the correct explanation of Assertion.
required to choose any one of the following five (b) If both Assertion and Reason are correct, but
responses. Reason is not the correct explanation of
(a) If both Assertion and Reason are correct and Assertion.
the Reason is a correct explanation of the (c) If Assertion is correct but Reason is incorrect.
Assertion. (d) If both the Assertion and Reason are incorrect.
(b) If both Assertion and Reason are correct but 31. Assertion : Lowering of vapour pressure is
Reason is not a correct explanation of the directly proportional to osmotic pressure of the
Assertion. solution. [2012]
(c) If the Assertion is correct but Reason is incorrect. Reason : Osmotic pressure is a colligative
(d) If both the Assertion and Reason are incorrect. property.
(e) If the Assertion is incorrect but the Reason is 32. Assertion : If a liquid solute more volatile than
correct. the solvent is added to the solvent, the vapour
27. Assertion : Molecular mass of benzoic acid when pressure of the solution may increase i.e., ps > po.
determined by colligative properties is found Reason : In the presence of a more volatile liquid
high solute, only the solute will form the vapours and
Reason : Dimerisation of benzoic acid. [1998] solvent will not.
28. Assertion : The molecular weight of acetic acid [2016]
determined by depression in freezing point 33. Assertion : If one component of a solution obeys
method in benzene and water was found to be Raoult’s law over a certain range of composition,
different. the other component will not obey Henry’s law
Reason : Water is polar and benzene is non- in that range. [2011, 2013, 2017]
polar. [2005] Reason : Raoult’s law is a special case of Henry’s
law.
Solutions C-95

Type A : Multiple Choice Questions 8. (c) The decrease in vapour pressure of


1. (c) According to Raoult's law, benzene by addition of naphthalene is an
example of colligative property.
p0 ps w M Change in vapour pressure of solvent or
ps m W change in boiling point of solvent may also
121.8 120.2 15 78 be due to formation of hydrogen bond or
interaction between solvent molecule and
121.8 m 250
solute molecule.
15 78 121.8
m 356.265 g 9. (b) cRT
250 1.6
7.8 = c × 0.082 × 310
2. (b) Tf = Kf × m
7.8
(5.45 – 3.55) = Kf × 0.374 c= = 0.31 mol /L
0.082 310
1.9 10. (c) From the relation
Kf = 5.08
0.374 w 1000
Tf Kf
w 1000 m W
3. (b) T Kb .
m W It is obvious that
0.450 1000 1
0.170 K b Tf
60 22.5 m
0.170 60 22.5
Kb 0.51 C. T f1
1000 0.450 m2
4. (c) Van’t Hoff factor T f2 m1
Normal Molecular Mass Cane sugar solution Glucose solution
=
Observed Molecular Mass T f1 273.15 271 T f2 ?
w 1000 m1 = 342 m2 = 180
5. (d) T Kb
m W 2.15 180
Hence
3 1000 T f2 342
0.52 = 0 .6
m 200
1.8 5 342 2.15
m= 17.3g mol 1 T f2 4.085 K
0.52 180
6. (c) Optical activity is not dependent upon So, freezing point of glucose solution
number of molecules of the compound, so = 273.15 – 4.085
it is not a colligative property. = 269.07 K
7. (d) V nRT
11. (c) p°benzene = 0.850 bar; Wbenzene = 39.0 g.
500 20 M.W. = 78 g/mol.
0.082 (273 15)
1000 m
°
psolution = 0.845 bar; w = 0.5 g.
500 600 20 Let n be the no. of moles of non volatile
0.082 288
1000 760 m solid; w and m be the weight of solid and
20 0.082 288 1000 760 molecular mass respectively.
m 1198
500 600
EBD_7100
C-96 Topicwise AIIMS Solved Papers – CHEMISTRY
phase will be smaller, then the partial vapour
Using, pbenzene - psolution = n solid
° °
pressure will be smaller than predicted by
°
pbenzene n benzene Raoult’s law and the system exhibits a
negative deviation.
0.5 15. (c) P p0A x A p 0B x B
0.850 - 0.845
= m 3 2
0.850 39 600 p0A p0B ;
78 3 2 2 3

3p0A 2p0B 3000


0.005 0.5 78
= ´ 4.5 2
0.850 m 39 630 p0A 0
pB
4.5 2 0.5 4.5 2 0.5
100 ´ 0.850
m 4.5 p0A 2 p 0B 4410
0.005
12. (d) The value of DTb depends upon two 1.5p0A 1410 ; p0A 940 and p 0B 90
factors 'i ' and 'm'. It is given that 1 mol of 16. (b) Weight of solute (w) = 1 g
each compound is dissolved in 1 L of Weight of solvent (W) = 75 g
solution. Hence molarity is same for all the Boiling point of solution = 100.114°C
compounds. Now the van't Hoff factor Boiling point of solvent = 100°C
depends on number of particle i.e. on degree
T = 100.114 – 100 = 0.114°C
of ionisation which further depends on the
Molecular weight of solute (m) = 60.1
bond dissociation energy which is in the
order Boiling point elevation constant (K) = ?
HI < HBr < HCl < HF 1000 K w
i.e., bond dissociation energy of HI is least. m=
T W
Lower the bond dissociation energy, higher
is the degree of ionisation and hence higher m T W
K
the number of particles, thus i will be 100 w
maximum for HI and hence DTb value will 60.1 0.114 75
=
be larger for HI. 1000 1
13. (a) T f i 513.8
= 0.513
1000
Tf for KCl i for KCl
Tf for BaCl 2 i for BaCl2 Tf i1 K f m 2 1.86
17. (d) ;
Tb i2 K b m 3 0.52
Tf for KCl 2
Tf for BaCl2 3 ( Tf for KCl = 2)
Tf 0.372 . Hence, Tb 0.156º C
3 2 i.e., Tb = 100.156°C
Tf for BaCl 2 3
2 12
Freezing point of BaCl2 = – 3°C 18. (a) Moles of urea = 0.2
60
14. (b) Shows negative deviation from Raoult’s
law. 68.4
If the attraction between different Moles of sucrose = 0. 2
342
molecules, for example between HCl and
H2O molecules, is stronger, the escaping Both are non electrolyte hence lowering of
tendency from the solution to the vapour V.P. will be same.
Solutions C-97

19. (a) Vapour pressure of solution = PA + PB Mole fraction of solute = 0.4


120 2 180 3 As mole fraction of solute + mole fraction
= poA x A poB x B 156 mm Hg
5 5 of solvent = 1
20. (d) Glucose is non electrolyte hence depression Hence, mole fraction of solvent = 1 – 0.4
in freezing point will be minimum, hence = 0.6
freezing point will be highest. 24. (b) As Tf = Kf. m
21. (a) Depression in freezing point No. of Tb = Kb. m
particles. Tf Tb
Hence, we have m
(when concentration of different solutions Kf Kb
is equal)
Kf
Al 2 (SO4 )3 provides five ions on or Tf Tb
ionisation Kb
[ Tb 100.18 100 0.18 C]
Al 2 (SO 4 ) 3 2 Al 3 3SO 24 –
while KCl provides two ions 1.86
= 0.18 × = 0.654°C
0.512
® K + + Cl-
KCl ¾¾ As the Freezing Point of pure water is
C6H12O6 and C12H22O11 are not ionised 0°C,
so they have single particle in solution. Tf = 0 –Tf
Hence, Al2(SO4)3 have maximum value of 0.654 = 0 – Tf
depression in freezing point or lowest Tf = – 0.654
freezing point. Thus the freezing point of solution will
be – 0.654°C.
10 25. (d) Tf = Kf × m
22. (a) Molar concentration of urea = per dm3
60
1000 K f w 2 solute
Molar concentration of volatile solute M
Tf w1 solvent
solution
5 50 1000 1.86 1.8
= per 100 ml. or, per dm3 M = 180
0.465 40
M M
For solution of same concentration Molecular formula = (empirical formula)n
or, isotonic solution at same temperature Molecular mass 180
n 6
Empirical formula mass 30
10 50
or, M = 300 g mol–1
60 M Molecular formula = (CH2O)6 = C6H12O6.
23. (b) According to Raoult's law 26. (d) Colligative properties depends upon the no.
of particles. Since methanol is non
p n
(mole fraction of solute) electrolyte hence cannot be considered.
p n N
10 Type B : Assertion Reason Questions
0.2 p° = 50 mm of Hg
p 27. (a) A molecular mass of benzoic acid is found
For other solution of same solvent high because of dimerisation of benzoic
20 n acid.
(Mole fraction of solute)
p n N 28. (a) In polar solvent, acetic acid will dissociate
leading to greater depression in freezing
20
Mole fraction of solute point.
50
EBD_7100
C-98 Topicwise AIIMS Solved Papers – CHEMISTRY
29. (c) A is true. Pressure inside the cell increases n M
due to osmosis. Water enters the cell or ....(iii)
N .R.T .
because concentration is higher inside the
From Raoult’s law,
cell. This results in lowering of
concentration of medium inside the cell. So, P° – P n
....(iv)
A is true but R is false. P N
30. (a) A is true and B is also true and B is correct P –P M
[From (iii) and (iv)]
explanation of A. P RT
31. (b) Both assertion and reason are correct but M
reason is not the correct explanation of or P –P P
RT
assertion.
The relationship between lowering of MP
The factor is constant at constant
vapour pressure and osmotic pressure can RT
be derived as follows: temperature.
Van’t Hoff equation for dilute solutions is
P –P
n
RT ....(i) or lowering of V.P. osmotic pressure
V Thus assertion is correct.
In case of a dilute solution, the volume of
Osmotic pressure is a colligative property
solution can be taken as equal to that of
is correct.
solvent. If N is the number of moles of
solvent of molecular weight M and density 32. (c) Both the solute and solvent will form the
, the volume V is given by vapours but vapour phase will become
richer in the more volatile component.
NM
V ....(ii) 33. (b)
17 Electrochemistry

TYPE A : MULTIPLE CHOICE QUESTIONS 8. Which of the following statements is true for
the electrochemical Daniel cell? [2004]
1. Effect of dilution on conductivity of solution:
(a) Electrons flow from copper electrode to zinc
(a) Increases (b) Decreases [1998]
electrode
(c) Unchanged (d) None of the above
(b) Current flows from zinc electrode to copper
2. Through a solution of CuSO4 a current of 3
electrode
amperes was passed for 2 hours. At cathode 3 g
(c) Cations move toward copper electrode
of Cu2+ions were discharged. The current
(d) Cations move toward zinc electrode
efficiency is [At. wt. of Cu = 63.5] [1998]
9. The chemical reaction,
(a) 33.3% (b) 42.2% 2AgCl(s) H 2(g) 2HCl(aq) 2Ag(s)
(c) 48.7% (d) 54.4% taking place in a galvanic cell is represented by
3. Which shows electrical conductance? [1999] the notation : [2005]
(a) Sodium (b) Diamond
(a) Pt(s) | H2(g) 1 bar | 1 M KCl(aq) | AgCl(s)|Ag(s)
(c) Potassium (d) Graphite
4. Which cannot displace hydrogen from its (b) Pt(s) | H2(g) 1 bar | 1 M HCl(aq) |
compound? [1999] 1MAg+(aq)|Ag(s)
(a) Al (b) Fe
(c) Pt(s) | H2(g) 1 bar | 1 M HCl(aq) | AgCl(s)|Ag(s)
(c) Hg (d) Pb
(d) Pt(s) | H2(g) 1 bar | 1 M HCl(aq) | Ag(s)|AgCl(s)
5. For reducing one mole of Cr2 O 27 to Cr 3 the
charge required is : [2000] 10. The charge required for the reduction of 1 mol of
(a) 3 × 96500 coulomb (b) 6 ×96500 coulomb MnO4 to MnO2 is : [2006]
(a) 1F (b) 3 F
(c) 0.3 Faradays (d) 0.6 Faradays
(c) 5F (d) 6F
6. Which of the following reactions is used to make
11. The products formed when an aqueous solution
a fuel cell? [2003]
of NaBr is electrolysed in a cell having inert
(a) Cd(s) + 2Ni(OH)3 (s) CdO(s) electrodes are: [2006]
+ 2Ni(OH)2(s) + H2O(l) (a) Na and Br2
(b) Pb(s) + PbO2(s) + 2H2SO4(aq) (b) Na and O2
2PbSO4(s) + 2H2O (l) (c) H2 ,Br2 and NaOH
(c) 2H2(g) + O2 (g) 2H2O (l) (d) H2 and O2
(d) 2Fe(s) + O2 (g) + 4 H+ (aq) 12. The emf of Daniell cell at 298 K is E1 [2008]
2Fe2+ (aq) + 2H2O (l) Zn | ZnSO4 (0.01 M) | | CuSO4 (1.0 M) | Cu
7. Time required to deposit one millimole of When the concentration of ZnSO4 is 1.0 M and
aluminium metal by the passage of 9.65 amperes that of CuSO4 is 0.01 M, the emf changed to E 2
through aqueous solution of aluminium ion is What is the relation between E1 and E2?
(a) 30 s (b) 10 s [2003] (a) E1 = E2 (b) E2 = 0 E2
(c) E1 > E2 (d) E1 < E2
(c) 30,000 s (d) 10,000 s
EBD_7100
C-100 Topicwise AIIMS Solved Papers – CHEMISTRY
13. Equivalent conductance of an electrolyte 2H+ (aq) + 2e H2 (g) ; 0.00 V
containing NaF at infinite dilution is 90.1 3
Fe (aq) e Fe 2 (aq) ; 0.770 V
Ohm–1cm2. If NaF is replaced by KF what is the
value of equivalent conductance? [2009] Which is the strongest reducing agent? [2013]
(a) 90.1 Ohm cm –1 2 (b) 111.2 Ohm cm2–1 (a) Zn (s) (b) Cr (s)
(c) 0 (d) 222.4 Ohm–1cm2 (c) H2(g) (d) Fe3+ (aq)
14. For a cell reaction involving two electron change, 20. A 1.0 M with respect to each of the metal halides
the standard EMF of the cell is 0.295 V at 2°C. AX 3 , BX 2 , CX 3 and DX 2 is electrolysed
The equilibrium constant of the reaction at 25°C using platinum electrodes. If
will be [2010]
E oA 3 / A 1.50 V, E oB2 / B 0.3 V,
(a) 29.5 × 10–2 (b) 10 o
(c) 1 × 1010 (d) 2.95 × 10–10 E oC3 / C 0.74 V, E D 2 / D 2.37 V.
15. A 0.5 M NaOH solution offers a resistance of The correct sequence in which the various
31.6 ohm in a conductivity cell at room metals are deposited at the cathode is [2014]
temperature. What shall be the approximate molar (a) A, B, C, D (d) A, B, C
conductance of this NaOH solution if cell (c) D, C, B, A (b) C, B, A
constant of the cell is 0.367 cm–1 . [2010] 21. A hydrogen electrode is immersed in a solution
(a) 234 S cm2 mole–1 (b) 23.2 S cm2 mole–1 with pH = 0 (HCl). By how much will the potential
(c) 4645 S cm2 mole–1 (d) 5464 S cm2 mole–1 (reduction) change if an equivalent amount of
16. Molar conductances of BaCl2, H2SO4 and HCl NaOH is added to the solution.
at infinite dilutions are x1, x2 and x3 respectively. (Take pH2 = 1 atm), T = 298 K. [2015]
Equivalent conductance of BaSO4 at infinite (a) increase by 0.41 V (b) increase by 59 mV
dilution will be : [2011] (c) decrease by 0.41 V (d) decrease by 59 mV
(a) (x1 + x2 – x3) /2 (b) x1 + x2 – 2x3 22. Given that the standard reduction potentials for
(c) (x1 – x2 – x3) /2 (d) (x1 + x2 – 2x3) /2 M+/M and N+/N electrodes at 298 K are 0.52 V
17. Given that E 2.93V ; and 0.25 V respectively. Which of the following
K /K
is correct in respect of the following
E 0.44 V ; E 0.76 V ; electrochemical cell ?
Fe2 / Fe Zn 2 / Zn
M/M+ | | N+/N [2016]
E 0.34V (a) The overall cell reaction is a spontaneous
Cu 2 / Cu
Based on this data, which of the following is the reaction.
strongest reducing agent ? [2011] (b) The standard EMF of the cell is – 0.27 V.
(a) Cu(s) (b) K (aq) (c) The standard EMF of the cell is 0.77 V.
(d) The standard EMF of the cell is – 0.77 V.
(c) Zn (2aq ) (d) Fe(s) 23. On the basis of the following E° values, the
18. The cell constant of a given cell is 0.47 cm –1. strongest oxidizing agent is : [2017]
The resistance of a solution placed in this cell is [Fe(CN)6]4– [Fe(CN)6]3– + e– ; E° = – 0.35 V
measured to be 31.6 ohm. The conductivity of Fe2+ Fe3+ + e–; E° = – 0.77 V
the solution (in S cm –1 where S has usual (a) [Fe(CN)6] 4– (b) Fe2+
meaning) is [2012] (c) Fe3+ (d) [Fe(CN)6]3–
(a) 0.15 (b) 1.5 24. Consider the following cell reaction: [2017]
(c) 0.015 (d) 150 2Fe( s) O2 ( g ) 4H ( aq)
19. The standard reduction potentials at 298K for the 2Fe2 (aq) 2H 2 O(l ); E 1.67V
following half reactions are given against each At [Fe2+] = 10–3 M, p(O2) = 0.1 atm and pH = 3,
Zn 2 (aq) 2e Zn(s) ; –0.762 V the cell potential at 25ºC is
(a) 1.47 V (b) 1.77 V
Cr3 (aq) 3e Cr (s); –0.740 V (c) 1.87 V (d) 1.57 V
Electrochemistry C-101

TYPE B : ASSERTION REASON QUESTIONS 32. Assertion :The cell potential of mercury cell is
1.35V, which remains constant.
Directions for (Qs. 25-32) : These questions consist Reason : In mercury cell, the electrolyte is a paste
of two statements, each printed as Assertion and of KOH and ZnO. [2008]
Reason. While answering these questions, you are Directions for (Qs.33-37) : Each of these questions
required to choose any one of the following five contains an Assertion followed by Reason. Read them
responses. carefully and answer the question on the basis of
(a) If both Assertion and Reason are correct and following options. You have to select the one that
the Reason is a correct explanation of the best describes the two statements.
Assertion.
(a) If both Assertion and Reason are correct and
(b) If both Assertion and Reason are correct but
Reason is the correct explanation of Assertion.
Reason is not a correct explanation of the
(b) If both Assertion and Reason are correct, but
Assertion.
Reason is not the correct explanation of
(c) If the Assertion is correct but Reason is incorrect. Assertion.
(d) If both the Assertion and Reason are incorrect. (c) If Assertion is correct but Reason is incorrect.
(e) If the Assertion is incorrect but the Reason is (d) If both the Assertion and Reason are incorrect.
correct.
33. Assertion : If º Na and º Cl are molar limiting
25. Assertion: Electrical conductivity of copper
increases with increase in temperature conductivity of sodium and chloride ions
Reason: The electrical conductivity of metals is respectively, then the limiting molar conductivity
due to motion of electrons. [1998] for sodium chloride is given by the equation :
26. Assertion : A small amount of acid or alkali is º NaCl º Na º Cl
added before electrolysis of water.
Reason : Pure water is weak electrolyte. [1999] Reason : This is according to Kohlrausch law
27. Assertion : Copper reacts with HCI and liberates of independent migration of ions. [2011]
hydrogen. 34. Assertion : On increasing dilution, the specific
Reason : Hydrogen is present above Cu in the conductance keep on increasing.
reactivity series. [2000] Reason : On increasing dilution, degree of
28. Assertion : Copper metal gets readily corroded ionisation of weak electrolyte increases and
in an acidic aqueous solution. molality of ions also increases. [2014]
Reason : Free energy change for this process is 35. Assertion : During electrolysis of CuSO4(aq)
positive. [2004] using copper electrodes, copper is dissolved at
29. Assertion : Galvanised iron does not rust. anode and deposited at cathode.
Reason : Zinc has a more negative electrode Reason : Oxidation takes place at anode and
potential than iron. [2005] reduction at cathode. [2014]
30. Assertion : E° for Mn3+ /Mn2+ is more positive 36. Assertion : On increasing dilution, the specific
than Cr3/Cr2+ conductance keep on increasing.
Reason : The third ionization energy of Mn is Reason : On increasing dilution, degree of
larger than that of Cr. [2006] ionisation of weak electrolyte increases and
31. Assertion : For the Daniel cell, Zn|Zn 2+ ||Cu2+ molality of ions also increases. [2015, 2016]
Cu with, Ecell = 1.1 V, the application of opposite 37. Assertion : Zinc can be used while copper
potential greater than 1.1 V results into flow of cannot be used in the recovery of Ag from the
electron from cathode to anode. complex [Ag(CN)2]–. [2017]
Reason : Zn is deposited at anode, and Cu is Reason : Zinc is a powerful reducing agent than
deposited at cathode. [2006] copper.
EBD_7100
C-102 Topicwise AIIMS Solved Papers – CHEMISTRY

Type A : Multiple Choice Questions 6. (c) Reaction used in fuel cell is


2H 2 (g) O 2 (g) 2H 2 O( )
1. (a) The conductivity of a weak electrolyte
increases with increase in dilution due to At anode : [H 2 2H 2e] 2
increase in ionisation. Remember that At cathode : O 2 2H 2 O 4e 4OH
conductivity of a strong electrolyte remains
constant at all dilution because strong 7. (a) Al3 3e Al
electrolytes are completely ionised under 1 mole requires = 3 × 96500 coulomb
all dilutions. 1 millimole (10–3 mole) requires
2. (b) According to law of electrolysis,
=3 × 96500 × 10–3 C
Mass deposited (m) = Z i t
we know that, Q = i t
m 96500
or i Q
t Z or t
i
Here, m = 3g, t = 2 × 60 × 60 = 7200 sec
3
Eq.wt At.wt 3 96500 10
z ; Eq. wt. = =
96500 Oxidation number 9.65

3 96500 2 3 9.65 10
i = 30 sec.
63.5 7200 9.65
= 1.266 A 8. (c) Daniel cell is an electrochemical cell in which
Efficiency of current Cu acts as cathode and Zn acts as an anode.
In Daniel cell,
Current used
= 100 (i) Electrons flow from Zn (anode) to Cu
Total current passed (cathode).
1.266 (ii) Current flows from Cu (cathode) to Zn
= 100 42.22% (anode).
3
3. (d) Though sodium and potassium are metals (iii) Cu2+ ions (cations) move towards Cu
and show electrical conductance but (cathode) and accumulated as Cu
graphite has more conductance due to metal.
presence of -electrons in its crystal lattice. 9. (b) In the given reaction
Sodium and potassium have only one 2AgCl (s) + H 2 ( g ) (1 Bar) 2HCl (aq)+ 2Ag (s)
electron in its outermost shell. So, inspite
of being metal, their conductivity is not so Silver is undergoing reduction (Ag + Ag),
good. hence it will act as cathode in the following cell.
4. (c) Hg is lower than hydrogen on the Pt (s) | H2 (g), 1 bar | 1 M HCl (aq) | | 1 M Ag+(aq) | Ag (s)
electrochemical series. So, mercury is (anode) (cathode)
incapable of displacing hydrogen from (i) Thus, since option (a) has KCl which is not
dilute acids. present in the cell, it is incorrect.
2
5. (b) Cr2O7 14H 6e– 2Cr 3 7H2O (ii) Since option (c) has AgCl (s) which does
not ionise, it is incorrect.
For reducing one mole of Cr2O72 , (iii) Since in option (d), at cathode, Ag is being
oxidised to Ag+ which is not possible , so it
charge required = 6 × 96500 coulomb.
is also incorrect.
Electrochemistry C-103

10. (b) MnO4– + 4H+ + 3e– MnO2 + 2H2O or Kc = 1 × 1010


So, 3 F of charge will be required to reduce 15. (b) Here, R = 31.6 ohm
1 mole of MnO4–. 1 1
C= ohm 1 = 0.0316 ohm–1
11. (c) + – R 31.6
NaBr Na Br
Specific conductance
2H 2 O + 2e H2 OH – = conductance × cell constant
At cathode
Na + OH – NaOH = 0.0316 ohm–1 × 0.367 cm–1
= 0.0116 ohm–1 cm–1
Br – Br + e – Now, molar concentration = 0.5 M (given)
Br + Br Br2 At anode = 0.5 × 10–3 mole cm–3
So the products are H2 and NaOH (at k
cathode) and Br2 (at anode) Molar conductance =
molar conc.
12. (c) Using the relation
0 0.0591 [anode] 0.0116
E cell E cell log
n [cathode] 0.5 10 3
0.0591 [Zn 2+ ] = 23.2 S cm2 mol–1
E 0cell log
n [Cu 2+ ] 16. (d) m (BaSO 4 ) Ba 2 SO24
Substituting the given values in two cases.
m (BaCl 2 ) m (H 2SO 4 ) 2 (HCl)
0.0591 0.01
E1 E 0 log º 1 º
2 1.0 x1 x2 2 x3 ; e m ( BaSO 4 )
0.0591 2
E0 log10 2
2 17. (d) K (aq ) and Zn (2aq ) can not lose
0.0591 electron(s). Fe has greater tendency to
E0 2 or (E0 0.0591)V
2 lose electrons than Cu.
0.0591 1
E2 E0 log [Eº 0.44V Eº 0.34V]
2 0.01 Fe / Fe2 Cu / Cu 2

0 0.0591
E – log102 k
1
Cell constant
0.47
0.01487
2 18. (c)
R 31.6
2 0.0591
E0 – or (E 0 – 0.0591)V 19. (a) Since oxidation potential of Zn is highest
2 hence strongest reducing agent.
Thus, E1 E 2 20. (b) The more the reduction potential, the more is
13. (a) Because at infinite dilution the equivalent the deposition of metals at cathode. Cation
conductance of strong electrolytes having E° value less than – 0.83 V (reduction
furnishing same number of ions is same. potential of H 2 O ) will not deposit from
14. (c) Using the relation, aqueous solution.
2.303 RT 0.0591 21. (c) pH changes from 0 to 7.
E°cell = log K c log K c [H+] changes from 1 to 10–7 M.
nF n
-0.059 1
0.0591
log K c Accordingly Ered. = log +
0.295 V = n [H ]
2
= 0.059 log 10–7
2 0.295
or log Kc = 10 i.e., 0.059 × (–7) = – 0.41 volt.
0.0591
EBD_7100
C-104 Topicwise AIIMS Solved Papers – CHEMISTRY
potential (–0.76 V) than iron (– 0.41 V) i.e.,
22. (b) Eocell o
Ecathode o
Eanode o
Eright o
Eleft Zn is less reactive than Fe.
30. (a) Reduction potential of Mn3+ is more than
E ocell 0.25 0.52 0.27V Cr 3+ . So, Mn 3+ is easily reduced in
23. (c) From the given data we find Fe3+ is comparision with Cr3+ or Cr2+ is easily
strongest oxidising agent. More the oxidised than Mn2+. Hence Mn2+ will lose
positive value of E°, more is the tendency electrons with difficulty or ionisation
to get oxidized. Thus correct option is (c). potential of Mn2+ is more than Cr2+. Both A
24. (d) Here n = 4, and [H+] = 10– 3 (as pH = 3) and R are true.
Applying Nernst equation 31. (a) In a Daniel cell,

2 2
Zn | Zn 2+ || Cu 2+ | Cu , E cell 1.1V
0.059 [Fe ] The oxidation half cell is
E = Eº – log
n [H ]4 (pO2 )
Zn Zn 2+ 2e
3 2
0.059 (10 ) The reduction half cell is
1.67 log
4 (10 3 ) 4 0.1 Cu 2+ 2e – Cu
2+
0.059 So, Zn + Cu Zn 2+ Cu
= 1.67 - log107
4 Thus here Zn is oxidised and deposited at
= 1.67 – 0.103 = 1.567 anode, and Cu is reduced and deposited at
cathode. If the opposite potential is greater
Type B : Assertion Reason Questions
than 1.1 V, then electrons flow from cathode
25. (e) Electrical conductivity of every metal to anode. So both A and R are correct.
decreases with increase in temperature due 32. (b) Zn(l) + HgO (s) Zn(s) + Hg (l)
to increase in molecular motion which The cell potential remains constant during
increases resistance. its life as the overall reaction does not
26. (a) Water is a covalent compund, hence pure involve any ion in solution whose
water is a weak electrolyte and feebly concentration changes during its life time.
ionised and thus bad conductor of electricity. 33. (a) According to Kohlrausch law, “limiting
However, addition of a small amount of acid molar conductivity of an electrolyte can be
or alkali increases ionisation of water represented as the sum of the individual
making at good conductor of electricity. contributions of the anion and cation of
27. (e) Copper does not liberate hydrogen from HCl the electrolyte.”
because copper is above hydrogen in 34. (d) The specific conductivity decreases while
reactivity series. equivalent and molar conductivities
28. (d) Copper lies below hydrogen in electro- increase with dilution.
chemical series so it cannot displace H2 from 35. (a) At cathode : Cu2+(aq) + 2e– Cu(s)
dilute acids. Hence it is not easily corroded (Reduction)
in acidic solutions. Further, corrosion is a At anode : Cu(s) Cu2+(aq) + 2e–
spontaneous process for which free energy (Oxidation)
change must be negative. 36. (d) The specific conductivity decreases while
29. (a) Galvanised iron does not rust easily equivalent and molar conductivities
because zinc (which is present in the form increase with dilution.
of coating) has more negative electrode 37. (a)
Chemical Kinetics C-105

18 Chemical Kinetics

TYPE A : MULTIPLE CHOICE QUESTIONS is 1.3 × 10–2s–1. Which equation given below
describes the change of [N2O5] with time ?
1. The rate constant of first order reaction is 3 × 10–6 [N2O5]0 and [N2O5]t corrospond to concentration
per second. The initial concentration is 0.10 M. of N2O5 initially and at time t.
The initial rate is: [1998] (a) [N2O5]t = [N2O5]0 + kt [2004]
(a) 3 × 10–7 mol/litre/sec (b) [N2O5]0 = [N2O5]t ekt
(b) 3 × 10–8 mol/litre/sec (c) log [N2O5]t = log [N2O5]0 + kt
(c) 3 × 10–5 mol/litre/sec [N 2 P5 ]0
(d) log [N P ] kt
(d) 3 × 10–8 mol/litre/sec 2 5 t
2. Which of the following statement is true for the 6. An endothermic reaction with high activation
reaction, H 2 Br2 2HBr . The rate law is energy for the forward reaction is given by the
dx diagram : [2005]
k [ H 2 ][ Br2 ]1/ 2 : [2000] (a)
dt
(a) order of reaction is 1.5
(b) molecularity of the reaction is 2 R
(c) by increasing the concentration of Br 2 four
P
times the rate of reaction is doubled
(d) all the above are correct. Reaction co-ordinate
3. For the reaction :
(b)
sunlight
H 2 Cl 2 2 HCl
the order of reaction is [2002]
(a) 0 (b) 2
(c) 1 (d) 3 R P
4. The potential energy diagram for a reaction
Reaction co-ordinate
R P is given below
(c)
Transition
Potential

P
energy

a state
R b c R
P
Reaction coordinate Reaction co-ordinate
H of the reaction corresponds to the energy : (d)
[2003]
(a) a (b) b P
(c) c (d) a + b R
5. The rate constant k, for the reaction
1
N 2 O 5 (g ) 2 NO 2 (g ) O 2 (g)
2 Reaction co-ordinate
EBD_7100
C-106 Topicwise AIIMS Solved Papers – CHEMISTRY

7. For reaction aA xP , when [A] = 2.2 mM, the 10. For a first order reaction, to obtain a positive
rate was found to be 2.4 mMs–1. On reducing slope, we need to plot {where [A] is the
concentration of A to half, the rate changes to concentration of reactant A} [2008]
0.6 mMs–1. The order of reaction with respect to (a) – log10[A] vs t (b) – loge[A] vs t
A is : [2005] (c) log10[A] vs log t (d) [A] vs t
(a) 1.5 (b) 2.0
11. T50 of first -order reaction is 10 min. Starting
(c) 2.5 (d) 3.0
8. Which of the following graphs represent relation with 10 mol L–1, rate after 20 min is [2008]
between initial concentration of reactants and (a) 0.0693 mol L–1 min–1
half-life for third order reaction? (b) 0.0693 × 2.5 mol L–1 min–1
(a) (c) 0.0693 × 5 mol L–2 min–1
(d) 0.0693 × 10 mol L–1 min–1
t1/2 12. The first order rate constant for a certain reaction
increases from 1.667 × 10–6 s–1 at 727ºC to
1.667 × 10–4 s–1 at 1571ºC. The rate constant at
1150ºC, assuming constancy of activation
a
energy over the given temperature range is
(b)
[Given : log 19.9 = 1.299] [2009]
(a) 3.911 × 10 s –5 –1 (b) 1.139 × 10 s–1
–5
t1/2 (c) 3.318 × 10 s –5 –1 (d) 1.193 × 10–5 s–1
13. In most cases, for a rise of 10K temperature the
rate constant is doubled to tripled. This is due
a to the reason that [2011]
(c) (a) collision frequency increases by a factor of
2 to 3.
t1/2 (b) fraction of molecules possessing threshold
energy increases by a factor of 2 to 3
(c) Activation energy is lowered by a factor of
1/a 2 to 3.
(d) none of these
(d)
14. The rate constant for the reaction,
t1/2 4 1
2N 2O5 4 NO 2 O 2 is 3.0 10 s . If
start made with 1.0 mol L–1 of N2O5, calculate
the rate of formation of NO2 at the moment of
1/a 2 the reaction when concentration of O2 is 0.1 mol
9. For the reaction, L–1.
2 N 2O 5 4 NO 2 O2 4
(a) 2.7 10 mol L 1s 1 [2011]
the rate of reaction is : [2006]
4
1 d d (b) 2.4 10 mol L 1s 1
(a) [N2 O5 ] (b) 2 [N 2 O5 ]
2 dt dt 4
(c) 4.8 10 mol L 1s 1

1 d d
(c) [NO2 ] (d) 4 [NO2 ] (d) 9.6 10 4
mol L 1s 1
4 dt dt
Chemical Kinetics C-107

15. A reaction which is of first order w.r.t. reactant 20. Which of the following relation represents correct
A, has a rate constant 6 min–1. If we start with relation between standard electrode potential and
[A] = 0.5 mol L–1, when would [A] reach the equilibrium constant? [2017]
value of 0.05 mol L–1 [2013] I.
nFE
log K
(a) 0.384 min (b) 0.15 min 2.303 RT
(c) 3 min (d) 3.84 min nFE

16. Half-lives of a first order and a zero order reaction II. K e RT


are same. Then the ratio of the initial rates of nFE
III. log K
first order reaction to that of the zero order 2.303 RT
reaction is [2014] nFE
IV. log K 0.4342
1 RT
(a) (b) 2 × 0.693 Choose the correct statement(s).
0.693
2 (a) I, II and III are correct
(c) 0.693 (d) (b) II and III are correct
0.693
17. Select the rate law that corresponds to the data (c) I, II and IV are correct
shown for the following reaction A B C (d) I and IV are correct
[2012, 2015]
TYPE B : ASSERTION REASON QUESTIONS
Expt. No. (A) (B) Initial Rate
1 0.012 0.035 0.10 Directions for (Qs. 21-26) : These questions consist
2 0.024 0.070 0.80 of two statements, each printed as Assertion and
3 0.024 0.035 0.10 Reason. While answering these questions, you are
4 0.012 0.070 0.80 required to choose any one of the following five
(a) Rate = k[B]3 (b) Rate = k [B]4 responses.
(c) Rate = k [A] [B]3 (d) Rate = k [A]2 [B]2 (a) If both Assertion and Reason are correct and
18. Collision theory is used to explain how chemical the Reason is a correct explanation of the
species undergo a reaction. Using this theory Assertion.
and the kinetic molecular model, which of the (b) If both Assertion and Reason are correct but
following does NOT influence the rate of a Reason is not a correct explanation of the
chemical reaction? Assertion.
[2016] (c) If the Assertion is correct but Reason is incorrect.
(a) The temperature of the system (d) If both the Assertion and Reason are incorrect.
(b) The geometry or orientation of the collision (e) If the Assertion is incorrect but the Reason is
(c) The velocity of the reactants at the point of correct.
collision 21. Assertion : According to transition state theory
(d) All of the above influence the rate for the formation of an activated complex, one of
19. For the following reaction: NO2(g) + CO(g) the vibrational degree of freedom is converted
NO(g) + CO2(g), the rate law is: Rate = k [NO2]2.
into a translational degree of freedom.
If 0.1 mole of gaseous carbon monoxide is added
Reason : Energy of the activated complex is
at constant temperature to the reaction mixture
higher than the energy of reactant molecules.
which of the following statements is true?
[2016] [2006]
(a) Both k and the reaction rate remain the same 22. Assertion :The order of a reaction can have
(b) Both k and the reaction rate increase fractional value.
(c) Both k and the reaction rate decrease Reason : The order of a reaction cannot be
(d) Only k increases, the reaction rate remain written from balanced equation of a reaction.
the same [2008]
EBD_7100
C-108 Topicwise AIIMS Solved Papers – CHEMISTRY
23. Assertion : In rate law, unlike in the expression 25. Assertion : If the activation energy of a reaction
for equilibrium constants, the exponents for is zero, temperature will have no effect on the
concentrations do not necessarily match the rate constant. [2015]
stoichiometric coefficients. Reason : Lower the activation energy, faster is
Reason : It is the mechanism and not the the reaction.
balanced chemical equation for the overall 26. Assertion : The kinetics of the reaction –
change that governs the reaction rate. [2009]
mA nB pC m' X n 'Y p 'Z
24. Assertion : The rate of the reaction is the rate of
change of concentration of a reactant or a dX
obey the rate expression as k[ A]m [ B ]n .
product. dt
Reason : Rate of reaction remains constant Reason : The rate of the reaction does not
during the course of reaction. [2010] depend upon the concentration of C.[2011, 17]
Chemical Kinetics C-109

Type A : Multiple Choice Questions It means, rate [ reactant]2


So, order of reaction = 2
dc
1. (a) k [c ] 3 10 6 0.1 8. (d) Graph (d) represents graph between t½ and
dt initial concentration for 3rd order reaction :
dc
3 10 7 mol litre 1 sec 1 (a) Zero order reaction
dt (b) 1st order reaction
2. (d) All the statements are correct. (c) 2nd order reaction.
1 9. (c) Rate of reaction
Order of reaction = 1 1.5
2 1 d N2O5 1 d NO2

Molecularity = 1 + 1 = 2 2 dt 4 dt
dx 10. (b) For a first order reaction the positive slope is
[Br2 ]1/ 2 obtained when we plot – loge [A] vs t.
dt
11. (b) Initial concentration = 10 mol L–1
So, reaction rate will be doubled if
Conc. after 20 min (two half lives) = 2.5 mol L–1
concentration of Br2 is increased by 4 times.
3. (a) The order of all photochemical reactions is 0.693 0.693
Now, k
zero as it does not depend upon the t1 / 2 10 min
concentration of reactants. or 0.0693 min–1
4. (c) Activation energy of forward reaction = a rate = k × [reactant]
Activation energy of backward reaction = b = 0.0693 × 2.5 mol L–1 min–1
H=b–a
k2 Ea T2 T1
Since b > a, therefore reaction is exothermic 12. (c) log
5. (d) As the unit of rate constant is sec–1, so the k1 2.303R T1T2
reaction is first order reaction. Hence k E a T2 T1
or 2.303log 2
1 a [N O ] k1 R T1T2
k = log or kt = log 2 5 0
t ( a x) [N 2 O5 ]t 4
1.667 10 Ea 1 1
6. (c) Reactions which involve absorption of heat 2.303 log =–
1.667 10 6 R 1844 1000
energy are called endothermic reactions. For
such reactions. Ea 844
HP > HR 2.303 × 2 = × ........(1)
R 1844 1000
In graphs (c) and (d), the heat of products
is more than heat of reactants and thus they Ea 4.606 1844 1000
represent endothermic reactions. But in (d) =
R 844
only small amount of energy is absorbed
(less difference between energy of reactants k3 E a 1423 1000
2.303 log = ×
an d products). Thus, (c) represents 1.667 10 6 R 1423 1000
maximum activation energy.
Note : Activation energy is the excess energy Ea 423
= × 1423 1000 ........(2)
that the reactant molecule must possess to R
cross energy barrier. Dividing equation (2) by equation (1)
7. (b) When the concentration of reactant is
reduced to half its initial value, the rate is k3
log 6
1.667 10
reduced by 2.4 4 times
0.6 2
EBD_7100
C-110 Topicwise AIIMS Solved Papers – CHEMISTRY

423 1844 1000 20. (c) G = –2.303 RT log K


= 1423 1000 × –nFE° = –2.303 RT log K
844
nFE
k3 log K
log 2.303 RT
6 nFE
1.667 10 0.4342 ........ (i)
423 1844 RT
= 2× = 1.299 nFE
1423 844 ln K
RT
On taking Antilog, k3 = 19.9 nFE
k3 = 19.9 × 1.667 × 10–6 = 3.318 × 10–5 s–1 ....... (ii)
K e RT
13. (b) For a 10 K rise in temperature, collision
Type B : Assertion Reason Questions
frequency increases merely by 1 to 2% but
the number of effective collisions increases 21. (b) The formation of an activated complex takes
by 100 to 200%. place when vibrational degree of freedom
1 convert into a translational degree of
14. (d) Mol L of N 2 O 5 reacted = 2 × 0.1 = 0.2 freedom. This statement is given by
[ N 2 O5 ] left = 1.0 – 0.2 = 0.8 mol L 1 transition statement theory. Also the energy
of activated complex is higher than the
Rate of reaction = k × [ N 2 O5 ]
energy of reactant molecule is true but it is
4
3.0 10 0.8 not the correct explanation of the assertion.
2.4 10 4
mol L 1s 1 22. (b) The order of a reaction can have fractional
value. Assertion is true.
Rate of formation of NO2 The order of a reaction can not be written
4 2.4 10 4 9.6 10 4 mol L–1s –1 from balanced equation of a reaction
2.303 a because its value changes with pressure,
15. (a) t log temperature and concentration. It can only
k a x
be determined experimentally. Thus the
2.303 0.5 reason is also correct, but the reason is not
log 0.384 min .
6 0.05 the correct explanation of assertion.
16. (b) For first order reaction, 23. (a) Rate law is always written according to the
slowest step and thus the exponents for
0.693
Rate kA'0 .A 0 concentrations do not necessarily match
t1 2 the stiochiometric coefficients.
A0 24. (c) Rate of reaction does not remain constant
For zero order reaction, Rate kA00 during the complete reaction because rate
2t1 2
depen ds upon th e concentration of
Ratio in rates = 2 × 0.693
reactants which decreases with time.
17. (a) Let the rate law be r =k [A]x[B]y
25. (b) According to Arrhenius equation, k = Ae–
0.10 [0.024]x [0.035] y Ea / RT
Divide (3) by (1)
0.10 [0.012]x [0.035] y When Ea = 0, k = A.
1 = [2]x, x = 0 dX
26. (a) Rate expression k[ A]m [ B ]n shows
x y dt
0.80 [0.024] [0.070]
Divide (2) by (3)
0.10 [0.024]x [0.035]y that the total order of reaction is m + n + 0
8 = (2)y , y = 3 = m+n
Hence rate equation, R = k[A]0[B]3 =k[B]3 As the rate of reaction is independent of
18. (d) concentration of C, i.e., the order with
19. (a) k remains constant at constant temperature respect to C is zero. This is the reason that
and CO does not effect the rate of reaction. C does not figure in the rate expression.
19 Surface Chemistry

TYPE A : MULTIPLE CHOICE QUESTIONS + – O


(b) Na O – +
1. The physical adsorption of gases on the solid O Na
surface is due to: [1998] O
(a) Covalent bond
(b) Hydrogen bond (c)
CH3
(c) Ionic bond
(d) Van der waal's forces
2. The electrical charge on a colloidal particle is (d) Cl
observed by: [1998] 10. A catalyst [2007]
(a) Ultramicroscope (a) changes the equilibrium constant
(b) Scattering (b) lowers the activation energy
(c) Brownian movement (c) increases the forward and backward
(d) Electrophoresis
reactions at different speeds.
3. Lyophilic colloids are stable due to: [1998]
(d) follows same mechanism for the reaction.
(a) Small size of the particle
(b) Large size of the particle 11. Which of the following is a lyophobic colloidal
(c) Layer of dispersion medium on the particle solution ? [2008]
(d) Charge on the particle (a) Aqueous starch solution
4. Purple of cassius is colloidal solution of : (b) Aqueous protein solution
(a) Silver (b) Lead [1999] (c) Gold solution
(c) Gold (d) Mercury (d) Polymer solvent in some organic solvents
5. Milk is colloid in which : [2000] 12. The density of gold is 19 g/cm3. If 1.9 × 10–4 g of
(a) Liquid is dispersed in liquid gold is dispersed in one litre of water to give a
(b) Gas is dispersed in liquid sol having spherical gold particles of radius
(c) Sugar is dispersed in water
10 nm, then the number of gold particles per mm3
(d) Solid is dispersed in liquid
6. The colloid is : [2001] of the sol will be [2010]
(a) urea (b) blood (a) 1.9 × 10 12 (b) 6.3 × 10 14
(c) cane sugar (d) NaCl (c) 6.3 × 1010 (d) 2.4 × 106
7. The movement of colloidal particles, under 13. Which of the following electrolyte will have
applied electric current is known as : [2001] maximum flocculation value for Fe(OH)3 sol?
(a) electrodialysis (b) dialysis (a) NaCl (b) Na2S [2010]
(c) electrophoresis (d) none of the above
8. The size of colloidal particle is [2002] (c) (NH4)3PO4 (d) K2SO4
(a) 10–3 to 10–9 m (b) 10–3 to 10–12 m 14. Which of the following ions will have the
(c) 10–6 to 10–9 m (d) 10–12 to 10–19 m minimum coagulating value for the sol obtained
9. Which of the following molecules is most by adding FeCl3 solution to slight excess of
suitable to disperse benzene in water ? [2005] NaOH [2011]
O (a) SO 24 (b) [Fe(CN) 6 ]3
(a) – +
O Na (c) Ba2+ (d) Al3+
EBD_7100
C-112 Topicwise AIIMS Solved Papers – CHEMISTRY
15. Preparation of Lyophobic sols by chemical (a) If both Assertion and Reason are correct and
method involves [2012] the Reason is a correct explanation of the
(a) double decomposition Assertion.
(b) oxidation & reduction (b) If both Assertion and Reason are correct but
(c) hydrolysis Reason is not a correct explanation of the
(d) all of these Assertion.
16. A colloidal solution is subjected to an electrical (c) If the Assertion is correct but Reason is incorrect.
field. The particles move towards anode. The (d) If both the Assertion and Reason are incorrect.
coagulation of same sol is studied using NaCl,
(e) If the Assertion is incorrect but the Reason is
BaCl2 and AlCl3 solutions. Their coagulating
correct.
power should be [2013]
21. Assertion: A catalyst is more effective in finely
(a) NaCl > BaCl2 > AlCl3
divided form.
(b) BaCl2 > AlCl3 > NaCl
Reason: Finely divided form has more surface
(c) AlCl3 > BaCl2 > NaCl
area. [1998]
(d) BaCl2 > NaCl > AlCl3
22. Assertion : Sky appears blue.
17. Under the influence of an electric field, the
Reason : Colloidal particles of dust scatter blue
particles in a sol migrate towards cathode. The
light. [2000]
coagulation of the same sol is studied using NaCl,
23. Assertion : Physical absorption of molecules
Na2SO4 and Na3PO4 solutions. Their coagulating
takes place on surface only.
values will be in the order [2014]
Reason : In this process, the bonds of the
(a) NaCl > Na2SO4 > Na3PO4
absorbed molecules are broken. [2002]
(b) Na2SO4 > Na3PO4 > NaCl
24. Assertion : The micelle formed by sodium stearate
(c) Na3PO4 > Na2SO4 > NaCl
in water has —COO– groups at the surface.
(d) Na2SO4 > NaCl > Na3PO4
Reason : Surface tension of water is reduced by
18. Gold numbers of protective colloids A, B, C and
the addition of stearate. [2003]
D are 0.50, 0.01, 0.10 and 0.005, respectively. The
25. Assertion : Aqueous gold colloidal solution is
correct order of their protective powers is [2015]
red in colour.
(a) D < A < C < B (b) C < B < D < A
Reason : The colour arises due to scattering of
(c) A < C < B < D (d) B < D < A < C
light by colloidal gold particles. [2004]
19. The disease kala azar is caused by [2016]
26. Assertion : Alcohols are dehydrated to
(a) colloidal antimony
hydrocarbons in the presence of acidic zeolites.
(b) milk of magnesia
Reason : Zeolites are porous catalysts. [2004]
(c) argyrols
27. Assertion :Colloidal solutions are stable but
(d) colloidal gold
colloidal particles do not settle down.
20. Which one of the following impurities present in
Reason : Brownian movement counters the force
colloidal solution cannot be removed by
of gravity actively on colloidal particles.[2008]
electrodialysis? [2017]
Directions for (Qs.28-31) : Each of these questions
(a) Sodium chloride
contains an Assertion followed by Reason. Read them
(b) Potassium sulphate
carefully and answer the question on the basis of
(c) Urea
following options. You have to select the one that
(d) Calcium chloride
best describes the two statements.
TYPE B : ASSERTION REASON QUESTIONS (a) If both Assertion and Reason are correct and
Directions for (Qs. 21-27) : These questions consist Reason is the correct explanation of Assertion.
of two statements, each printed as Assertion and (b) If both Assertion and Reason are correct, but
Reason. While answering these questions, you are Reason is not the correct explanation of
required to choose any one of the following five Assertion.
responses. (c) If Assertion is correct but Reason is incorrect.
(d) If both the Assertion and Reason are incorrect.
Surface Chemistry C-113

28. Assertion : In chemisorption, adsorption first the mass of gas adsorbed by m grams of
increases with temperature and then decreases. adsorbate, p is the equilibrium pressure, k and n
Reason : Heat keeps on providing more and more are constants for given system and temperature.
activation energy. [2009] Reason : When several substances have same
29. Assertion : A catalyst does not alter the
1
equilibrium constant of a reaction. value of , the lines by which their adsorption
Reason : The catalyst forms a complex with the n
reactants and provides an alternate path with isotherms can be represented will meet at a point.
lower energy of activation for the reaction; the [2013]
forward and the backward reactions are affected 31. Assertion : The enthalpy of physisorption is
to the same extent. [2010] greater than chemisorption. [2016, 17]
Reason : Molecules of adsorbate and adsorbent
x are held by van der Waal’s forces in
30. Assertion : The relation k . p1/ n is known
m physisorption and by chemical bonds in
as Freundlich adsorption isotherm, where x is chemisorption.
EBD_7100
C-114 Topicwise AIIMS Solved Papers – CHEMISTRY

Type A : Multiple Choice Questions Number of gold particles present


Volume of gold in solution
1. (d) Vander Waal's forces are responsible for
formation of bonds in case of physical Volume of each particle
adsorption.
0.1 10 4 cm 3
2. (d) The electrical change on a colloidal particle
88
is observed by electrophoresis. Under 10 18 cm3
electric field, charged particles move in a 21
particular direction. 21
3. (c) Lyophilic colloids are stable due to layer of 1013 particles
88
dispersion medium on the particle.
= 2.4 × 1012 particles
4. (c) Purple of cassius is colloidal solution of
2.4 × 1012 particles of gold are present in
gold.
1000 cm3 (1 litre).
5. (a) Milk is colloid in which liquid (liquid fat) is
Number of particles present per mm3
dispersed in liquid (water).
6. (b) Blood is a colloidal solution with plasma as 2.4 1012
dispersion medium and biomolecules as [1 L = 106 mm3]
106
colloidal particles. = 2.4 × 106
7. (c) The movement of colloidal particles under
applied electric potential in known as 1
13. (a) Flocculating value , where z = valency
electrophoresis. (z)
8. (c) Size of colloidal particle = 10–9 to 10–6 m. of coagulating ion.
9. (c) Benzene is non-polar and hence dissolves
Since, Fe(OH)3 is a positively charged sol
non-polar compounds (like dissolves like).
so it will be coagulated by negative ions
Among the given compounds, only (c) is
considering th e valencies of var ious
non-polar hence it dissolves in benzene.
10. (b) A catalyst lowers down the activation negative ions used i.e., Cl–, S2–, PO43– and
energy. Greater is decrease in activation SO42–, we find that the valency of Cl– is least
energy, higher will be the reaction rate. so it will have maximum flocculation value
11. (c) Of the given solution the gold sol is i.e., NaCl.
lyophobic colloidal solution. Gold particles 14. (d)
have very less affinity towards dispersion FeCl 3 3NaOH Fe (OH) 3(s) 3NaCl
medium, hence its sol can be easily
coagulated. Fe ( OH ) 3 OH (from excess of NaOH)
12. (d) Volume of gold present in solution
[Fe(OH)3 ]OH
Mass of gold 1.9 10 4 g The sol is negatively charged and would
Density of gold 19g / cm 3 be coagulated most effectively by Al3+.
= 0.1 × 10–4 cm3. 15. (d)
For spherical particles of gold with radius equal 16. (c) The movement towards anode shows that
to 10 nm sol is negative. For coagulation of negative
The volume of each particle sol. Cation with higher charge is more
4 3 4 22 effective.
r (10 10 7 cm)3
3 3 7 17. (a) Since the sol particles migrate towards
88 cathode, they are positively charged. Hence,
10 18 cm3
21 anions would be effective in coagulation.
Surface Chemistry C-115

Greater is the valence of effective ion, not the correct explanation. Micelle is
smaller will be its coagulating value. formed if molecules with polar and nonpolar
18. (c) For a protective colloid lesser the value of ends assemble in the bulk.
gold number more will be the protective 25. (a) The colour of the gold sol is due to
power. Thus the correct order of protective scattering of light by colloidal particles.
power of A, B, C and D is 26. (b) In petroleum industry zeolites are used to
(A) < (C) < (B) < (D) convert alcohol directly to hydrocarbon by
Gold number 0.50 0.10 0.01 0.005 dehydrating it. Zeolites are porous catalysts
but this is not the reason for its use to
Hence (c) is the correct answer
convert alcohol into petrol.
19. (a) Colloidal antimony is used in curing kala-
27. (a) The Brownian movement of particles is due
azar.
to bombardment on the particles by the
20. (c) Electrodialysis involves movement of ions molecules of the dispersion medium. Thus
towards oppositely charged electrodes. Brownian movement does not permit the
Urea being a covalent compound does not particles to settle down and induces
dissociate to give ions and hence it cannot stability of colloidal solution.
be removed by electrodialysis.However all 28. (c) In chemisorption, adsorption first increases
the other given compounds are ionic which
with temperature and then decreases.
can undergo dissociation to give oppositely
The heat initially supplied acts as activation
charged ions and thus can be separated.
energy for the chemical reaction occurring
Type B : Assertion Reason Questions between adsorbate and adsorbent.
21. (a) A calalyst is more effective in finely divided 29. (a) Catalyst has an equal effect over forward
form because of more surface area. and backward reactions and hence, it does
22. (a) Sky appears blue colour due to scattering not affect the equilibrium state.
of light by the colloidal particles (dust) of 30. (c) Assertion is true, reason is false.
air. Most scattered ray is blue. When several lines have the same value of
23. (d) The assertion that physical absorption of
molecules takes place on surface only is 1
, then the lines by which their adsorption
false. Actually absorption takes place on n
the whole body. Further it is a simple isotherms can be represented will be
physical phenomenon which involves no parallel and will not meet at a point.
breaking of bond. Hence, both are false. 31. (d) Assertion is false but Reason is true. The
24. (b) The assertion that the micelle formed by enthalpyof chemisorption is of the order of
sodium stearate in water has –COO– groups 40 - 400 kJmol–1 while for physical adsorption
at the surface is true as it is hydrophilic it is of the order of 20 - 40 kJmol–1.
end. The given reason is also true but is
EBD_7100
C-116 Topicwise AIIMS Solved Papers – CHEMISTRY

General Principles and Processes


20 of Isolation of Elements
TYPE A : MULTIPLE CHOICE QUESTIONS 10. Match list I with list II and select the correct
answer using the codes given below the lists:
1. The process does not involve a catalyst is : List I List II
[1997] I. Cyanide process A. Ultrapure Ge
(a) Haber process (b) Contact process II. Floatation process B. Pine oil
(c) Thermite process (d) Ostwald process III. Electrolytic reduction C. Extraction of Al
2. Chief ore of Al is : [1997] IV. Zone refining D. Extraction of Au
(a) cryolite (b) bauxite Codes: [2013]
(c) feldspar (d) kaolin (a) I-C, II-A, III-D, IV-B
3. Froth floatation is a process of: [1998] (b) I-D,II-B,III-C,IV-A
(c) I-C,II-B,III-D, IV-A
(a) Oxidation (b) Reduction
(d) I-D,II-A,III-C,IV-B
(c) Refining (d) Concentration
11. Sulfide ores are common for the metals [2015]
4. Flux is used to remove : [1999]
(a) Ag, Cu and Pb (c) Ag, Cu and Sn
(a) basic impurities (b) Ag, Mg and Pb (d) Al, Cu and Pb
(b) acidic impurities 12. Match List I with List II and select the correct
(c) all types of impurities answer using the codes given below the list
(d) acidic and basic both impurities [2015]
5. Mac Arthur process is used for the extraction List I List II
of: [2000] 1. Ti A. Bauxite
(a) Au (b) Pt (c) Cu (d) Zn 2. Si B. Cerussite
6. In the extraction of copper from its sulphide ore, 3. Al C. Van-Arkel method
the metal is formed by reduction of Cu 2O with : 4. Pb D. Zone refining
(a) FeS (b) C O [2003] (a) 1–B, 2–A, 3–C, 4–D
(c) Cu2S (d) SO2 (b) 1–B, 2–C, 3–A, 4–B
7. Which of the following is a carbonate ore? (c) 1–C, 2–A, 3–B, 4–D
(a) Pyrolusite (b) Malachite [2005] (d) 1–C, 2–D, 3–A, 4–B
(c) Diaspore (d) Cassiterite 13. The main reactions occurring in blast furnace
8. Carbon and CO gas are used to reduce which of during extraction of iron from haematite
the following pairs of metal oxides for extraction are________.
of metals? [2007] (i) Fe2O3 + 3CO — 2Fe + 3CO2
(a) FeO, SnO (b) SnO, ZnO (ii) FeO + SiO2 — FeSiO3
(iii) Fe2O3 + 3C — 2Fe + 3CO
(c) BaO, Na2O2 (d) FeO, ZnO
(iv) CaO + SiO2 — CaSiO3 [2016]
9. In metallurgical process of aluminium, cryolite
(a) (i) and (iii) (b) (ii) and (iv)
is mixed with alumina in its molten state, because
(c) (i) and (iv) (d) (i), (ii) and (iii)
it [2011]
14. 2CuFeS2 O 2 Cu 2 S 2FeS SO 2
(a) decreases the amount of alumina
Which process of metallurgy of copper is
(b) oxidises the alumina
represented by above equation? [2017]
(c) increases the melting point of alumina
(a) Concentration (b) Roasting
(d) decreases the melting point of alumina
(c) Reduction (d) Purification
General Principles and Processes of Isolation of Elements C-117

Directions for (Qs.16-18) : Each of these questions


TYPE B : ASSERTION REASON QUESTIONS
contains an Assertion followed by Reason. Read them
Directions for (Q. 15) : These questions consist of carefully and answer the question on the basis of
two statements, each printed as Assertion and Reason. following options. You have to select the one that
While answering these questions, you are required to best describes the two statements.
choose any one of the following five responses. (a) If both Assertion and Reason are correct and
(a) If both Assertion and Reason are correct and Reason is the correct explanation of Assertion.
the Reason is a correct explanation of the (b) If both Assertion and Reason are correct, but
Assertion. Reason is not the correct explanation of
Assertion.
(b) If both Assertion and Reason are correct but
(c) If Assertion is correct but Reason is incorrect.
Reason is not a correct explanation of the (d) If both the Assertion and Reason are incorrect.
Assertion. 16. Assertion : Copper obtained after
(c) If the Assertion is correct but Reason is incorrect. bassemerization is known as blister copper.
(d) If both the Assertion and Reason are incorrect. Reason : Blisters are produced on the surface of
(e) If the Assertion is incorrect but the Reason is the metal due to escaping of dissolved SO2.
correct. [2013]
15. Assertion : Extraction of iron metal from iron 17. Assertion : Coke and flux are used in smelting.
oxide ore is carried out by heating with coke. [2017]
Reason : The reaction, Reason : The phenomenon in which ore is mixed
with suitable flux and coke is heated to fusion is
Fe 2 O 3 (s ) Fe (s ) 3 / 2 O 2 (g ) known as smelting.
is a spontaneous process. [2005] 18. Assertion : Galvanised iron does not rust. [2017]
Reason : Zinc has a more negative electrode
potential than iron.
EBD_7100
C-118 Topicwise AIIMS Solved Papers – CHEMISTRY

Type A : Multiple Choice Questions 11. (a) Silver, copper and lead are commonly found
1. (c) In thermite process, no catalyst is required. in earth's crust as Ag2 S (silver glance),
Fe2O3 + 2Al Al2O3 + 2Fe + Heat CuFeS2 (copper pyrites) and PbS (galena).
This reaction evolves a lot of heat which 12. (d) Ti – van-Arkel method
provides energy to the reaction.
Si – Zone refining method
2. (b)
3. (d) Froth floatation is a process of concentration Al – Bauxite (Al2O3)
of sulphide ore. Pb – Cerussite (PbCO3)
4. (d) Flux is the material which is used in the 13. (c) 14. (b)
metallurgy of metals to remove acidic and
basic impurities. Type B : Assertion Reason Questions
Acidic flux like silica is used to remove basic 15. (c) Iron is obtained from Fe2O3 by heatig it
impurities present in ore like CaO. with a mixture of coke and CaCO3 in a blast
CaO + SiO2 CaSiO3. furnace in which CO formed reduces Fe2O3
Basic flux like limestone is used to remove to Fe.
silica like acidic impurities.
Fe 2 O3 CO Fe + CO 2
SiO2 + CaCO3 CaSiO3 + CO2.
However, the reason is false because the
5. (a) Mac Arthur process involves extraction of
gold from sulphide ore with the help of 3
KCN. reaction Fe2 O3 ( s ) Fe(s ) + O2 ( g )
2
6. (c) Cu2S + 2Cu2O 6 Cu + SO2 is not spontaneous as for this reaction both
7. (b) Malachite is carbonate ore. H and S are negative, so G will be
Cu(OH)2CuCO3 positive
8. (d) ZnO + C ¾¾ ® Zn + CO 16. (a) Both assertion and reason are correct and
FeO + C ¾¾ ® Fe + CO reason is the correct explanation of
In the process of smelting the oxide ore is assertion.
reduced by carbon and the metal may be 17. (b) Both assertion and reason are true but
obtained in the molten state or as vapours reason is not the correct explanation of
which are condensed. Metals like Zn, Fe, assertion. Non fusible mass present in ore
Pb or Sn are obtained by this process.
in mixing with suitable flux are fused which
9. (d)
are then reduced by coke to give free metal.
10. (b) Cyanide process is for gold (I-D); floatation
18. (a) Zinc metal which has a more negative
process - pine oil (II-B); Electrolytic
electrode potential than iron will provide
reduction - Al (III-C); Zone refining-Ge
electrons in preference of the iron, and
(IV-A).
therefore corrodes first. Only when all the
zinc has been oxidised, the iron start to rust.
The p-Block Elements
21 (Group 15, 16, 17 and 18)
TYPE A : MULTIPLE CHOICE QUESTIONS 9. Iodide of Millon's base is : [1999]
(a) HIO3 (b) K2HgI4
1. The ion or group detected by K2[HgI4] is :
(a) NO (b) Cl – [1997] (c) NH 2 HgO.HgI (d) Hg ( NH 2 )I
(c) NH2 – (d) NH4+ 10. Which of the following is not the characteristic
2. Which gas cannot be collected over water ? of interhalogen compounds? [1999]
(a) O2 (b) PH3 [1997] (a) they are covalent
(c) N2 (d) SO2 (b) more reactive than halogens
3. The correct order of increasing oxidising power is : (c) have low B.P. and high volatile
(a) Cl 2 Br2 F2 I2 [1997] (d) quite unstable but not explosive
11. Hypo on treatment with iodine produces :
(b) F2 Br2 Cl 2 I2 (a) H2S (b) Na2SO4 [2000]
(c) F2 Br2 Cl 2 I2 (c) Na2SO3 (d) Na2S4O6
12. The tribasic acid is : [2001]
(d) I2 Br2 Cl 2 F2 (a) H3PO4 (b) H3PO3
4. H2S does not produce metallic sulphide with : (c) H3PO2 (d) HPO3
(a) CuCl2 (b) COCl2 [1997] 13. The strongest oxidising agent is : [2001]
(c) CdCl2 (d) ZnCl2 (a) F2 (b) Cl2
5. The correct order of acid strength of oxyacids (c) I2 (d) Br2
is: [1998] 14. The correct order of solubility in water for He,
(a) HClO HClO 2 HClO 3 HClO 4 Ne, Ar, Kr, Xe is [2002]

(b) HClO2 > HClO3 > HClO > HClO4 (a) He > Ne > Ar > Kr > Xe
(b) Xe > Kr > Ar > Ne > He
(c) HClO 4 HClO 3 HClO 2 HClO
(c) Ne > Ar > Kr > He > Xe
(d) HClO3 HClO4 HClO HClO2
(d) Ar > Ne > He > Kr > Xe
6. Chlorine acts as a bleaching agent only in
15. Which of the following compound is a tribasic
presence of: [1998]
acid? [2002]
(a) Sunlight (b) Moisture
(a) H3PO2 (b) H3PO4
(c) Dry air (d) Pure oxygen
(c) H3PO3 (d) H4P2O7
7. The laughing gas is: [1998]
16. The true statement for the acids of phosphorus.
(a) NO (b) N2O
H3PO2, H3PO3, and H3PO4 is : [2003]
(c) N2O4 (d) N2O5
(a) the order of their acidity is H3 PO 4 <
8. The correct decreasing order of basic strength is:
H3PO3<, H3PO2
(a) AsH 3 SbH 3 PH 3 NH 3 [1999] (b) all of them are reducing in nature
(b) SbH 3 AsH 3 PH 3 NH 3 (c) all of them are tribasic acids
(c) NH 3 PH 3 AsH 3 SbH 3 (d) the geometry of phosphorus is tetrahedral
in all the three
(d) PH 3 AsH 3 SbH 3 NH 3
EBD_7100
C-120 Topicwise AIIMS Solved Papers – CHEMISTRY
17. The mixture of concentrated HCl and HNO3 made 26. Inert gases are mixed in iodine vapours. Then
in 3 : 1 ratio contains : [2003] there are _______ between them. [2007]
(a) ClO2 (b) NOCl (a) H-bonding
(c) NCl3 (d) N2O4
(b) vander Waal’s forces
18. The element which forms oxides in all oxidation
states varying from +1 to +5 is [2004] (c) Electrostatic forces
(a) N (b) P (d) Metallic bonds.
(c) As (d) Sb 27. The element which is the most abundant in the
19. The statement true for N3– is : [2004] earth crust is [2007]
(a) it has non-linear structure (a) O (b) S
(b) it is called pseudohalogen (c) Al (d) H.
(c) the formal oxidation state of nitrogen in this 28. Cyanogen gas is obtained in the reaction
anion is – 1 (a) CuSO4(aq) + KCN [2007]
(d) it is isoelectronic with NO2
(b) K4[Fe(CN)6] heat
20. For electron affinity of halogens which of the
following is correct? [2004] (c) CH3CN + H2O
(a) Br > F (b) F > Cl
(c) Br > Cl (d) F > I (d) CH3CONH2 + P2O5
21. The number of P–O–P bridges in the structure 29. Perdisulphuric acid has the following bond
of phosphorus pentoxide and phosphorus [2008]
trioxide are respectively : [2005] (a) O O O (b) O O
(a) 6, 6 (b) 5, 5 (c) (d) —O — O —
O O
(c) 5, 6 (d) 6, 5
30. S2– and SO32– can be distinguished by using
22. The incorrect statement among the following is:
(a) C 60 is an allotropic form of carbon [2006] [2008]
(b) O3 is an allotropic form of oxygen (a) (CH3COO)2Pb (b) Na2[Fe(CN)5NO]
(c) S8 is the only allotropic form of sulphur. (c) both (a) and (b) (d) None of these
(d) Red phosphorus is more stable in air than 31. Choose the correct statement – [2009]
white phosphorus (a) White or yellow phosphorus is stored
23. Tincture of iodine is : [2006] under water
(a) Aqueous solution of I2 (b) Ignition temperature of red phosphorus is low
(b) Solution of I2 in aqueous KI (c) Black phosphorus is non crystalline in
(c) Alcoholic solution of I2 nature
(d) Aqueous solution of KI (d) Phosphorus does not form hydrides.
24. Which two of the following salts are used for 32. An element (X) forms compounds of the formula
preparing iodized salt? [2006] XCl3, X2O5 and Ca3X2, but does not form XCl5.
(i) KIO3 (ii) KI Which of the following is the element [2009]
(iii) I2 (iv) HI (a) B (b) Al
(a) (i) and (ii) (b) (i) and (iii) (c) N (d) P
(c) (ii) and (iv) (d) (iii) and (iv) 33. H2S gas is not evolved when SO32– ion reacts
25. XeF6 on hydrolysis produces [2007] with following reagents. [2009]
(a) Zn + dil. H2SO4 (b) Al + Conc. NaOH
(a) XeOF4 (b) XeO2F2
(c) Al + dil. HCl (d) None of these
(c) XeO3 (d) XeO2
The p-Block Elements (Group 15, 16, 17 and 18 ) C-121

34. In the nitrogen family the H – M – H bond angle 43. What are the products formed in the reaction or
in the hydrides MH3 gradually becomes closer xenon hexafluoride with silicon dioxide ?
to 90° on going from N to Sb. This shows that (a) XeSiO4 + HF (b) XeF2 + SiF4 [2012]
gradually – [2009] (c) XeOF4 + SiF4 (d) XeO3 + SiF2
(a) The basic strength of hydrides increases 44. P4O10 is not used to dry NH3 gas because
(b) Almost pure p-orbitals are used for M – H
(a) P4O10 reacts with moisture in NH3 [2012]
bonding
(c) The bond energies of M – H bond increases (b) P4O10 is not a drying agent
(d) The bond pairs of electrons becomes nearer (c) P4O10 is acidic and NH3 is basic
to the central atom (d) P4O10 is basic and NH3 is acidic
35. Which of the following oxides is most acidic? 45. Oxygen is more electronegative than sulphur,
(a) Ag2O (b) V2O5 [2009] yet H2S is acidic while H2O is neutral. This is
(c) CO (d) N2O5 because
36. Iodine exists in which form in iodised salt (a) Water is a highly associated [2012]
(a) I2 (b) NaIO3 [2009] (b) H—S bond is weaker than H—O bond
(c) ICl (d) IO4– (c) H2S is a gas while H2O is a liquid
37. When chlorine water is added to a solution of
(d) The molecular weight of H2S is more than
KBr, the solution immediately turns orange red,
that of H2O
because [2009]
46. When chlorine reacts with cold and dilute
(a) Chlorine is reduced to chloride ion
solution of sodium hydroxide, the products
(b) Of the formation of BrCl obtained are [2013]
(c) Bromide ion is oxidized to bromine
(d) Of the formation of Br 3– (a) Cl– + OCl– (b) Cl– + ClO 2
38. Which of the following halides is the most stable? (c) Cl– + ClO3 (d) Cl– + ClO 4
(a) CCl4 (b) CBr4 [2010]
(c) CF4 (d) CI4 47. When Br2 is treated with aqueous solutions of
39. Noble gases are absorbed by [2010] NaF, NaCl and NaI separately [2014]
(a) anhydrous calcium chloride (a) F2, Cl2 and I2 are liberated
(b) ferric hydroxide (b) only F2 and Cl2 are liberated
(c) conc. H2SO4
(c) only I2 is liberated
(d) activated coconut charcoal
40. Which one of the following is the correct (d) only Cl2 is liberated
decreasing order of boiling point? [2010] 48. Which one of the following statements regarding
helium is incorrect ? [2015]
(a) H2Te > H2O > H2Se > H2S
(a) It is used to produce and sustain powerful
(b) H2O > H2S > H2Se > H2Te superconducting magnets.
(c) H2Te > H2Se > H2S > H2O (b) It is used as a cryogenic agent for carrying
(d) H2O > H2Te > H2Se > H2S out experiments at low temperatures.
41. Total number of lone pair of electrons in XeOF4 (c) It is used to fill gas balloons instead of
is [2011] hydrogen because it is lighter and non-
(a) 0 (b) 1 (c) 2 (d) 3 inflammable.
(d) It is used in gas-cooled nuclear reactors.
42. The reason for not storing XeF6 in a glass or a
49. Regarding F– and Cl– which of the following
quartz vessel is that [2011]
statements is/are correct? [2015]
(a) it forms an explosive having the formula
(i) Cl– can give up an electron more easily than F–
XeO2F2 (ii) Cl– is a better reducing agent than F–
(b) it forms an explosive having the formula (iii) Cl– is smaller in size than F–
XeOF4 (iv) F– can be oxidized more readily than Cl –
(c) it forms XeO2 which is explosive substance (a) (i) and (ii) (b) (i), (ii) and (iv)
(d) it forms XeO64– which is explosive in nature (c) (iii) and (iv) (d) Only (i)
EBD_7100
C-122 Topicwise AIIMS Solved Papers – CHEMISTRY

50. What is X and Y in the given reactions ? [2016] 57. Assertion : Fluorine exists only in –1 oxidation
2X2 (g) + 2H2O (l) 4H+ (aq) + 4X– (aq) + O2 (g) state.
Y2 (g) + H2O (l) HY(aq) + HOY(aq) Reason : Fluorine has 2s22p5 configuration.
(a) X = Cl , Y = F (b) X = Cl , Y = Br [2001]
(c) X = F , Y = Cl (d) X = I , Y = F 58. Assertion : The fluorine has lower reactivity.
51. Which of the following fluorides does not exist? Reason : F – F bond has low bond dissociation
[2017] energy. [2002]
(a) NF5 (b) PF5 59. Assertion : Dinegative anion of oxygen (O2–) is
(c) AsF 5 (d) SbF5 quite common but dinegative anion of sulphur
52. Which of the following are peroxoacids of (S2–) is less common.
sulphur? [2017] Reason : Covalency of oxygen is two. [2002]
(a) H2SO5 and H2S2O8 60. Assertion : The halogens absorb visible light.
(b) H2SO5 and H2S2O7 Reason : All halogens are coloured. [2002]
(c) H2S2O7 and H2S2O8 61. Assertion : HClO4 is a strong acid than HClO3.
(d) H2S2O6 and H2S2O7 Reason : Oxidation state of Cl in HClO4 is +7 and
TYPE B : ASSERTION REASON QUESTIONS in HClO3, it is + 5. [2004]
62. Assertion : Reaction of SO2 and H2S in the
Directions for (Qs. 53-64) : These questions consist
presence of Fe2 O3 catalyst gives elemental
of two statements, each printed as Assertion and
sulphur.
Reason. While answering these questions, you are
required to choose any one of the following five Reason : SO2 is a reducing agent [2005]
responses. 63. Assertion : Ozone is an allotrope of oxygen.
(a) If both Assertion and Reason are correct and Reason : Oxygen is bluish coloured liquid and
the Reason is a correct explanation of the in singlet state it is para-magentic. [2007]
Assertion. 64. Assertion : F is more electronegative than Cl.
(b) If both Assertion and Reason are correct but Reason : F has high electron affinity than Cl.
Reason is not a correct explanation of the [2007]
Assertion. Directions for (Qs.65-73) : Each of these questions
(c) If the Assertion is correct but Reason is incorrect. contains an Assertion followed by Reason. Read them
(d) If both the Assertion and Reason are incorrect. carefully and answer the question on the basis of
(e) If the Assertion is incorrect but the Reason is following options. You have to select the one that
correct. best describes the two statements.
53. Assertion: A mixture of He and O2 is used for (a) If both Assertion and Reason are correct and
respiration for deep sea divers. Reason is the correct explanation of Assertion.
(b) If both Assertion and Reason are correct, but
Reason: He is soluble in blood. [1998]
Reason is not the correct explanation of
54. Assertion : Iodine is more soluble in water than
Assertion.
in carbon tetrachloride. (c) If Assertion is correct but Reason is incorrect.
Reason : Iodine is a polar compound. [1999] (d) If both the Assertion and Reason are incorrect.
55. Assertion : Inert gases are monoatomic. 65. Assertion : The S–S–S bond angle in S8
Reason : Inert gases have stable configuration. molecule is 105°.
[1999] Reason : S8 has a V-shape. [2008, 2009]
56. Assertion : Anhydrous BaO 2 is used for 66. Assertion : Sulphuric acid is more viscous than
preparing H2O2. water.
Reason : Hydrated BaO2 is not available. Reason : Concentrated sulpuric acid has a great
affinity for water. [2009]
[2001]
The p-Block Elements (Group 15, 16, 17 and 18 ) C-123

70. Assertion : Silicones are hydrophobic in nature.


67. Assertion : Although PF5 , PCl5 and PBr5 are
[2015]
known, the pentahalides of nitrogen have not Reason : Si – O – Si linkages are moisture
been observed. sensitive.
Reason : Phosphorus has lower electronegativity 71. Assertion : SF6 cannot be hydrolysed but SF4
than nitrogen. [2010] can be.
68. Assertion : White phosphorus is more reactive Reason : Six F atoms in SF6 prevent the attack of
than red phosphorus. H2O on sulphur atom of SF6.
[2016]
Reason : Red phosphorus consists of P 4
72. Assertion : Both rhombic and monoclinic
tetrahedral units linked to one another to form
sulphur exist as S8 but oxygen exists as O2.
linear chains. [2012] [2017]
69. Assertion : When a metal is treated with Reason : Oxygen forms p – p multiple bond
conc. HNO3 it generally yields a nitrate, NO2 due to small size and small bond length but
and H2O. p – p bonding is not possible in sulphur.
Reason : Conc. HNO3 reacts with metal and first 73. Assertion : HClO4 is a stronger acid than HClO3.
produces a metal nitrate and nascent hydrogen. [2017]
The nascent hydrogen then further reduces Reason : Oxidation state of Cl in HClO4 is +VII
HNO3 to NO2. [2013] and in HClO3 +V.
EBD_7100
C-124 Topicwise AIIMS Solved Papers – CHEMISTRY

Type A : Multiple Choice Questions molecule becomes a little polar the boiling
point increases due to inter-molecular
1. (d) The ion or group detected by K2[HgI4] is
attraction. They are reactive and unstable
NH 4 . It is known as Nessler's reagent test.
because of low bond energy.
2K2Hg I4 + NH3 + 3KOH 11. (d) 2Na 2S2 O3 I2 Na 2S4O6 2NaI
H 2 N Hg O Hg I 7KI 2H 2O Sodium thiosulphate Sodium tetrathionate

Million base (brown ppt) 12. (a) Orthophosphoric acid (H3PO4) is tribasic
2. (d) SO2 cannot be collected over water because because it has three replacable H-atoms.
it is soluble in water. O O
3. (d) Metals are good reducing agents, while P OH P OH
non-metals are good oxidising agents. H
HO OH OH
Further among halogens, metallic character
increases as we move from top to bottom Orthophosphoric acid, Phosphorous acid, H3PO3
in a group, i.e., the oxidising nature is H3PO4 (tribasic) (dibasic)
O O
I2 Br2 Cl2 F2
4. (b) COCl2 is carbonyl chloride and does not P H P
have any metal atom. Further it is a covalent H OH HO O
compound. Metaphosphoric acid
Hypophosphorous acid
5. (c) The acid strength of oxyacids of the same HPO3 (monobasic)
H3PO2 (monobasic)
halogen increases with increase in oxidation
13. (a) An element which can readily accept an
state of the halogen electron is a good oxidising agent. Halogens
7 5 3 1 are good oxidising agents because they
HClO4 HClO3 HClO2 HClO have strong tendency to accept an electron
Thus Cl in HClO4 is in highest oxidation due to (i) small atomic size, (ii) high
state (+7) and hence strongest acid, while electronegativity and (iii) high electron
HClO (Cl is in +1 oxidation state) is weakest. affinity. Fluorine, being smallest, is best
6. (b) Chlorine acts as bleaching agent only in oxidising agent.
presence of moisture. 14. (b) As the molecular weight of noble gas atoms
Cl2 + H2O 2HCl + O. increases down the group its polarity
7. (b) Nitrous oxide (N2O) is known as laughing increases due to which van-der-waal’s force
gas . A mixture of N2O & O2 is used as between them increases. Due to increased
polarity of heavier inert gas, its solubility
anaesthesia in dental surgery.
in water also increases. So, most soluble
8. (c) Lone pair of electrons on N, P, As and Sb gas will be Xe and least soluble will be He.
are dispersed in a greater way as the size of So correct order is Xe > Kr > Ar > Ne > He
atom increases from N to Sb. Therefore, NH3 15. (b) H3PO4 is tribasic acid
is most basic and SbH3 is least basic.
9. (c) Iodide of Millon's Base is NH2.HgO.HgI
It is formed when Nessler's reagent is O O
reacted with NH3. P P
10. (c) Interhalogen compounds are covalent in OH
HO OH O OH
nature. More reactive than halogens
because of low bond en ergy due to Pyrophosphoric acid
dissimilar size of atoms in molecule. As the H 4P2O 7 (tetrabasic)
The p-Block Elements (Group 15, 16, 17 and 18 ) C-125

16. (d) (i) The geometry of phosphorus is O


tetrahedral in all the three. ••
P P
O O O O O O
O O O
O

••
P O P P P

••
P P P
O O
O O O O
H H H OH OH OH P P
OH ••
OH OH O
H 3 PO 2 H 3 PO 3 H 3 PO 4 (P4O10) (P4O6)
Hence, number of P—O—P bridges is 6 in both.
(ii) The acidity increases with increase in 22. (c) S8 is not the only allotropic form of sulphur.
oxidation number of central atom 23. (c) Tincture of iodine is 2% solution of iodine
in alcohol.
1 3 5
H3PO2 H3PO3 H3 PO4 24. (a) KIO3 and KI are used for preparing iodised
salt.
(iii) H3PO4 is not reducing. 25. (c) Complete hydrolysis of XeF6 yields XeO3.
XeF6 + 3H2O ¾¾ ® XeO3 + 6HF
(iv) H3PO3 is diabasic, while H3PO2 is 26. (b) All molecules have short range London
monobasic. dispersive forces which is type of vander
17. (b) The mixture of 1 part of conc. HNO3 and 3 Waals forces. When inert gases are mixed
in iodine vapours there exists short range
parts of conc. HCl is known as aqua regia.
London dispersive forces.
It contains NOCl.
27. (a) A little more than 47% of earth crust consists
HNO3 + 3 HCl NOCl + 2H2O +2 (Cl) of oxygen. The more common rock
18. (a) Nitrogen has oxidation states from + 1 to constituents of earth's crust are nearly all
+ 5 in its oxides oxides.
N2O, NO, N2O3, NO2, N2O5 28. (a) Among the given options, only CuSO4(aq.)
and KCN react to form an unstable copper
+1 +2 +3 +4 +5
(II) cyanide which rapidly decomposes to
19. (d) Azide ion, N 3– has a linear structure. It has give copper (I) cyanide and cyanogen.
2CuSO4(aq) + 4KCN ¾¾ ®
22 (21 + 1) electrons, thus it is iso-electronic
2 Cu(CN)2 + 2K2SO4
with N2O (14 + 8 = 22).
20. (d) Electron affinity decreases in a group from 2 Cu(CN)2 ¾¾
® 2CuCN + (CN) 2
Cyanogen
top to bottom due to increase in size, so the
expected order of electron affinity for 29. (d) The structure of peroxydisulphuric acid
halogens is F > Cl > Br > I (H2S2O8) is
However, it is not so. The electron affinity O O
of elements of III period is higher than that S S
O O O
of II period because elements of II period O OH OH
have small size and greater electron density
so the incoming electrons suffer a repulsive
30. (c) S2– (CH 3COO) 2 Pb
force. Thus the order of electron affinity is Pbs 2CH 3COO –
black
Cl > F > Br > I .
21. (a) Both have six P–O–P bonds as shown in S2– [Fe(CN)5 NO]2 [Fe(CN)5 NOS]4–
violet
the figure. Thus S2– and SO32– can be distinguished
by using both the given reagents.
EBD_7100
C-126 Topicwise AIIMS Solved Papers – CHEMISTRY
31. (a) Due to low ignition temperature of white form reddish yellow or orange red BrCl.
phosphorus, it undergoes oxidation in Br2 + Cl2 –––– 2BrCl
presence of air which slowly raises its 38. (c) Stability of carbon tetrahalides decreases
temperature and after a few moments it
with increase in the size of halogen atom.
catches fire spontaneously. Due to this
So, CF4 is the most stable halide among the
reason, it is stored under water. Ignition
given options.
temperature of red phosphorus is high.
39. (d) Activated charcoal is a very good absorbent
Black phosphorus is crystalline in nature.
Phosphorus forms a number of hydrides. of gas, whereas anhydrous CaCl2, Fe(OH)3
32. (c) N forms NCl3, N2O5 and Ca3N2. Nitrogen, and conc. H2SO4 are dehydrating agents.
due to absence of empty d-orbitals, cannot 40. (d) Among the hydrides of group 16 elements,
boiling point of H2O is higher than H2S
extend its covalency by more than three
(difference in boiling points of H2O and H2S
and hence do not form NCl5. Due to its is ar ound 200°C) because of strong
small size and high electronegativity it can intermolecular hydrogen bonding. After the
accept 3 electrons to form N3– ion (Ca3N2) decrease in boiling point from H2O to H2S,
33. (b) SO32 2Al 2OH 3H 2O from H2 S to H2 Te, it increases due to
increase in size of the atoms from S to Te
S2 2 [Al(OH) 4 ] which increases the magnitude of van der
H2S does not evolve as liberated H2S is Waal's forces among the molecules. So, the
neutralized by NaOH and Na2S is formed. correct order of boiling points is
34. (b) The hydrides have a pyramidal or H2O > H2Te > H2Se > H2S
tetrahedral shape with a lone pair of O
electrons in one of the orbitals. The H – M 41. (b)
– H bond angle is less than the original F F
109°28' tetrahedral bond angle (H – N – H Xe
in NH3 is 106°45') because of greater
repulsion between lone pair and a bond pair F F
..
than between two bond pairs of electrons.
Because electro-negativity of M decreases
from N to Bi, the bond pair lie farther away 42. (b) 2XeF6 SiO 2 2XeOF4 SiF4
from the central atom, and the lone pair (glass) (explosive nature)
causes greater distortion of bond angle.
Thus H – P – H bond in PH3 is 94°, while in 2XeOF4 SiO 2 2XeO 2 F2 SiF4
AsH3 and SbH3 it is about 91.8° and 91.3°
respectively (closer to 90°). This suggests
2XeO 2 F2 SiO2 2XeO3 SiF4
that orbitals used for bonding are closer to
(explosive)
pure p-orbitals.
35. (d) The oxide with the highest positive 43. (c) 2XeF6 SiO 2 SiF4 2XeOF4
oxidation state on the element other than O
should be most acidic. Oxidation state of V 44. (c) P4 O10 2H 2 O 4HPO3
in V2O5 and N in N2O5 are +5. But the Acidic meta phosphoric
electronegativity of N is higher, making acid
N2O5 the most acidic oxide. NH 3
36. (b) Iodised salt is table salt mixed with a minute NH 4 PO3
basic meta ammonium
amount of one of the four iodine containing phosphate
salts of hydroiodic acid or iodic acid : 45. (b) H - S bond is weaker than H—O bond due
potasium iodate or potasium iodide and to large size of S. Moreover due to high
sodium iodate or sodium iodide. electronegativity of oxygen it forms
37. (b) Br2, formed by adding chlorine water to a hydrogen bond in H2O.
solution of KBr, further reacts with Cl2 to
The p-Block Elements (Group 15, 16, 17 and 18 ) C-127

46. (a) 2NaOH Cl2 NaCl NaOCl H 2 O BaO 2 .8H 2 O H 2SO 4


hence Cl and OCl BaSO4 H2 O2 8H2 O.
47. (c) Br2 reacts with NaI only to get I2.
57. (a) Fluorine exists in only –1 oxidation state as
2NaI + Br2 2NaBr + I2 it does not have vacant d-orbital to receive
48. (c) Helium is twice as heavy as hydrogen it is more than one electron.
inflammable but not lighter than hydrogen.
58. (e) Fluorine is a highly reactive non-metal due
Helium has the lowest melting and boiling
point of any element which makes liquid to low bond dissociation energy.
helium an ideal coolant for many extremely 59. (c) Dinegative anion of oxygen is quite
low temperature application such as super common due to higher electron gain
conducting magnet and cryogenic research enthalpy of oxygen.
60. (a) All halogens are coloured as they absorb
where temperature close to absolute zero
light in the visible region.
are needed. He is used in gas cooled atomic
reactors as a heat transfer agent. 61. (a) HClO4 is a stronger acid than HClO3
49. (d) because in the former Cl is in higher oxidation
50. (c) 2F2 (g) + 2H2O (l) state (+7) than that in HClO3 (+5).
4H+ (aq) + 4F– (aq) + O2 (g) 62. (c) 2H 2S + SO 2 2H 2 O + 3S
Cl2 (g) + 2H2O (l) HCl (aq) + HOCl Here H2S is oxidised to sulphur, while SO2
51. (a) NF5 does not exist because N does not form is reduced to water. Thus SO2 is oxidising
agent and H2S is reducing agent.
pentahalides due to the absence of d-orbital
63. (c) Here the assertion given is true and reason
in its valence shell. While P, As and Sb form
is false. Ozone (O3) is a triatomic molecule
pentahalides of the general formula MX5
consisting of three oxygen atoms. It is an
(where, M = P, As and Sb) due to the
allotrope of oxygen that is much less stable
presence of vacant d-orbitals in their
than the diatomic O2. Liquid O2 is a clear
respective valence shell.
substance with a light sky blue colouration.
52. (a) Both have peroxy linkage In triplet form it is paramagnetic due to spin
magnetic moments of the unpaired
Type B : Assertion Reason Questions electrons in the molecule and the negative
53. (c) A mixture of He and O 2 is used for exchange energy between neighbouring O2
respiration by deep sea divers but He is molecules.
not soluble in blood. In singlet oxygen all the electron spins are
54. (d) Iodine is less soluble in water than in paired and hence it is not paramagnetic in
carbon tetrachloride. Iodine is a non polar nature.
compound. 64. (c) Here assertion is true but reason is false. F
Iodine, being a non-polar compound is more is more electronegative than chlorine, in fact
soluble in CCl4 (non-polar compound) than it is most electronegative element of the
in water because "like dissolves like". so
periodic table but its electron affinity, the
both the assertion and reason are false.
55. (a) Inert gases are monoatomic because they tendency to accommodate electrons, is less
have stable configuration as their octet is than the Cl due to non availability of
complete. d-electrons.
56. (d) H2O2 is prepared by the action of H2SO4 65. (c) S8 has puckered ring type structure and
on hydrated BaO2 and not on dehydrated S-S-S bond angle ranges from 102-108°. Thus,
BaO2. When dehydrated BaO2 is used, then assertion is correct but reason is incorrect.
66. (b) Both assertion and reason are true but
a layer of BaSO4 is formed around it which reason is not the correct explanation of
prevents reaction to proceed. assertion. Molecules of H 2 SO 4 are
EBD_7100
C-128 Topicwise AIIMS Solved Papers – CHEMISTRY
associated due to large n umber of 70. (b) 71.(a) 72. (a)
intermolecular H-bonds. 73. (b) Both Assertion and Reason are true but
67. (b) Nitrogen cannot form pentahalides because reason is not the correct explanation of
it cannot expand its octet due to non- assertion. Greater the number of negative
availability of d-orbitals. So, Reason is not atoms present in the oxy-acid make the acid
the correct explanation of Assertion . stronger. In general, the strengths of acids
68. (b) White phosphorus exists as P4 tetrahedral
that have general formula (HO)m ZOn can
molecule having P-P-P bond angle 60º.
Hence the molecule is under strain and more be related to the value of n. As the value of
reactive. On the other hand red phosphorus n increases, acidic character also increases.
exists as P4 tetrahedra which are joined The negative atoms draw electrons away
together through covalent bonds giving from the Z-atom and make it more positive.
polymeric structure. The Z-atom, therefore, becomes more
69. (a) Both assertion and reason are true and effective in withdrawing electron density
reason is the correct explanation of away from the oxygen atom that bonded to
assertion. hydrogen. In turn, the electrons of H – O
M HNO3 MNO3 H bond are drawn more strongly away from
(metal) (conc.) (metal nitrate) (nascent hydrogen) the H-atom. The net effect makes it easier
2HNO3 2H 2NO2 2H 2O from the proton release and increases the
(nascent hydrogen) acid a strength.
22 The d- and f-Block Elements

TYPE A : MULTIPLE CHOICE QUESTIONS 10. The composition of duralumin is : [2001]


(a) Al 94%, Mg 6%
1. Malachite is an ore of: [1998] (b) Cu 56%, Zn 24%, Ni 20%
(a) Silver (b) Mercury (c) Cu 95%, Al 5%
(c) Magnesium (d) Copper (d) Al 95%, Cu 4%, Mn 0.5%, Mg 0.5%
2. The chief ore of Hg is: [1999] 11. The number of water molecules in Mohr's salt is :
(a) Pyrolusite (b) Bauxite (a) 2 (b) 4 [2001]
(c) Galena (d) Cinnabar (c) 6 (d) 8
3. The transition element which shows the highest 12. Philosopher's wool on heating with BaO at
oxidation state is: [1999] 1100°C produces : [2001]
(a) Iron (b) Vanadium (a) Ba + ZnCl2 (b) BaCdO2
(c) Manganese (d) Chromium (c) BaZnO2 (d) BaO2 + Zn
4. Gun metal is: [1999] 13. Which of the following is not an ore of iron?
(a) Cu + Zn (b) Cu + Sn + Zn (a) limonite (b) casiterite [2002]
(c) Cu + Sn (d) Zn + Sn (c) magnetite (d) none of these
5. Transition elements form coloured ions due to : 14. The colourless species is : [2003]
(a) d-d transition [1999] (a) VCl3
(b) fully filled d-orbitals (b) VOSO4
(c) smaller atomic radii (c) Na3VO4
(d) availability of s-electrons (d) [V(H2O)6]SO4. H2O
6. CuSO4 and KCN react to produce : [1999] 15. Lanthanide for which +II and +III oxidation states
(a) CuCN2 (b) CuCN are common is : [2003]
(c) K 3[Cu (CN) 4 ] (d) K 4 [Cu (CN) 6 ] (a) La (b) Pr
7. Which of the following is greatest paramagnetic? (c) Ce (d) Eu
(a) Cu+ (b) Fe2+ [2000] 16. Among the following pairs of ions, the lower
(c) Fe3+ (d) Cu2+ oxidation state in aqueous solution is more
8. The Mohr’s salt is shown by: [2000] stable than the other, in : [2005]
(a) Ti+, Ti3+ (b) Cu+ , Cu2+
(a) FeSO 4 ( NH 4 ) 2 SO 4 .6H 2O
(c) Cr2+, Cr3+ (d) V2+, VO2+
(b) FeSO 4 ( NH 3 ) 2 SO 4 .6H 2O 17. F2 is formed by reacting K2MnF6 with : [2005]
(c) K 2SO 4 Al 2 (SO 4 )3 .24H 2 O (a) SbF5 (b) MnF3
(c) KSbF6 (d) MnF4
(d) FeSO3 (NH 2 )4SO 4 .6H 2O 18. The colour imparted by Co(Il) compounds to
9. German silver is an alloy of: [2000] glass is: [2006]
(a) Fe, Cr, Ni (b) Ag, Cu Au (a) Green (b) Deep blue
(c) Cu, Zn, Ni (d) Cu, Zn, Sn (c) Yellow (d) Red
EBD_7100
C-130 Topicwise AIIMS Solved Papers – CHEMISTRY
19. Which of the following radioisotopes is used as 27. White silver surface turns black when O3 is
anticancerous? [2007] passed over it. This is due to the formation of
(a) Na-24 (b) C-14 (a) silver hydroxide [2011]
(c) U-235 (d) Co-60 (b) freshly reduced silver which is black in
20. Which of the following compound is coloured? colour
(a) TiCl3 (b) FeCl3 [2008] (c) silver oxide
(c) CoCl2 (d) All of these (d) a complex compound of silver and ozone
21. The purple colour of KMnO4 is due to the 28. In which of the following cases, the stability of
transition [2009] two oxidation states is correctly represented
[2011]
(a) C.T. (L M) (b) C.T. (M L)
(a) Ti3+ > Ti4+ (b) Mn2+ > Mn3+
(c) d – d (d) p – d
(c) Fe2+ > Fe3+ (d) Cu+ > Cu2+
22. To obtain silver from silver amalgam, it is heated 29. Cuprous ion is colourless while cupric ion is
in vessel which is made of – [2009] coloured because [2012]
(a) Cu (b) Fe (a) Both have half filled p-and d-orbitals
(c) Ni (d) Zn (b) Cuprous ion has incomplete d-orbitaland
23. The compounds containing complex oxyanion cupric ion has a complete d-orbital
CrO24 are intensely yellow coloured because (c) Both have unpaired electrons in the
(a) Chromium ion in CrO24 is a transition d-orbitals
metal ion [2010] (d) Cuprous ion has complete d-orbital and
(b) Cr and O are -bonded and the allowed cupric ion has an incomplete d-orbital.
* transition occurs in the visible
region 30. A compound of a metal ion M x Z 24 has a
(c) Cr O charge transfer is responsible for spin only magnetic moment of 15 Bohr
yellow colour Magnetons. The number of unpaired electrons
(d) Compounds have strong absorption at in the compound are [2013]
~ 640 nm (a) 2 (b) 4
24. The only cations present in a slightly acidic (c) 5 (d) 3
solutions are Fe3+, Zn2+ and Cu2+. The reagent 31. Identify the incorrect statement among the
that when added in excess to this solution would following : [2015]
identify and separate Fe3+ in one step is (a) Lanthanoid contraction is the accumulation
(a) 2 M HCI (b) 6 M NH3 [2010] of successive shrinkages.
(c) 6 M NaOH (d) H2S gas (b) As a result of lanthanoid contraction, the
25. The colour of copper sulphide is [2010] properties of 4d series of the transition
(a) Blue (b) Black elements have no similarities with the 5d
(c) Red (d) Green series of elements.
26. Which one of the following statements (c) Shielding power of 4f electrons is quite
concerning lanthanide elements is false? [2010] weak.
(a) Lanthanides are separated from one another (d) There is a decrease in the radii of the atoms
by ion exchange method or ions as one proceeds from La to Lu.
32. Consider the following statements [2016]
(b) The ionic radii of trivalent lanthanides
steadily increase with increase in atomic (I) La(OH) 3 is the least basic among
number hydroxides of lanthanides.
(c) All lanthanides are highly dense metals (II) Zr4+ and Hf4+ posses almost the same ionic
radii.
(d) Most characteristic oxidation state of
lanthanides is +3 (III) Ce4+ can as an oxidizing agent.
The d- and f-Block Elements C-131

Which of the above is/are true ? 40. Assertion : SnI4 is an orange solid.
(a) (I) and (III) (b) (II) and (III) Reason : The colour arises due to charge
(c) (II) only (d) (I) and (II) transfer. [2007]
33. For d-block elements the first ionization potential 41. Assertion : Magnetic moment of Dy is highest
is of the order [2017] among lanthanoids.
(a) Zn > Fe > Cu > Cr (b) Sc = Ti < V = Cr Reason : Orbital motion contributes
(c) Zn < Cu < Ni < Co (d) V > Cr > Mn > Fe magnetic moment. [2007]
42. Assertion : C – O bond in metal carbonyls is
TYPE B : ASSERTION REASON QUESTIONS long.
Directions for (Qs. 34-42) : These questions consist Reason : There is delocalisation of
of two statements, each printed as Assertion and electrons from filled d orbitals into
Reason. While answering these questions, you are the empty orbitals on the CO
required to choose any one of the following five ligands. [2007]
responses. Directions for (Qs.43-47) : Each of these questions
(a) If both Assertion and Reason are correct and contains an Assertion followed by Reason. Read them
the Reason is a correct explanation of the carefully and answer the question on the basis of
Assertion. following options. You have to select the one that
(b) If both Assertion and Reason are correct but best describes the two statements.
Reason is not a correct explanation of the (a) If both Assertion and Reason are correct and
Assertion. Reason is the correct explanation of Assertion.
(c) If the Assertion is correct but Reason is incorrect. (b) If both Assertion and Reason are correct, but
(d) If both the Assertion and Reason are incorrect. Reason is not the correct explanation of
(e) If the Assertion is incorrect but the Reason is Assertion.
correct. (c) If Assertion is correct but Reason is incorrect.
(d) If both the Assertion and Reason are incorrect.
34. Assertion: Pure iron is not used for making tools
43. Assertion : Europium (II) is more stable than
and machines.
cerium (II).
Reason: Pure iron is hard. [1998] Reason : Cerium salts are used as catalyst in
35. Assertion : Iron is found in free state in nature. petroleum cracking. [2010]
Reason : Iron is highly reactive element.[2001] 44. Assertion : Lead, tin and bismuth are purified
36. Assertion : A solution of FeCl3 in water by liquation method.
produces brown precipitate on standing. Reason : Lead, tin and bismuth have low m.p. as
Reason : Hydrolysis of FeCl3 takes place in water. compared to impurities [2010]
45. Assertion : Transition metals are good catalysts.
[2001]
37. Assertion : Cuprous ion (Cu+) has unpaired [2015]
electrons while cupric ion (Cu2+) does not. Reason : V2O5 or Pt is used in the preparation of
Reason : Cuprous ion (Cu+) is colourless H2SO4 by contact process.
whereas cupric ion (Cu2+) is blue in the aqueous 46. Assertion : Magnetic moment values of
solution. [2002] actinides are lesser than the theoretically
38. Assertion : Solution of Na2CrO4 in water is predicted values.
intensely coloured. Reason : Actinide elements are strongly
Reason : Oxidation state of Cr in Na2CrO4 is +VI paramagnetic. [2013, 2016]
[2003] 47. Assertion : Transition metals show variable
39. Assertion : The free gaseous Cr atom has six valency.
unpaired electrons Reason : Transition metals have a large energy
Reason : Half-filled s orbital has greater stability. difference between the ns2 and (n – 1)d electrons.
[2004] [2017]
EBD_7100
C-132 Topicwise AIIMS Solved Papers – CHEMISTRY

Type A : Multiple Choice Questions 14. (c) Species having incompletely filled
d-orbitals also have unpaired electron in
1. (d) Malachite is an ore of copper d-orbital and hence will be coloured.
Cu(OH)2 . CuCO3 Species having completely filled or vacant
2. (d) The chief ore of mercury is HgS (cinnabar). d-orbitals does not have any unpaired
3. (c) Out of all the elements given, manganese electron in d orbital, hence it will be
has the highest oxidation state of + 7 in colourless.
KMnO4. Chromium has highest oxidation V3+ in VCl3 : [Ne] 3s2p6d2
2+
V in VSO4 : [Ne] 3s2 p6 d3
state of +6 in K2Cr2O7, Vanadium has + 5 in
5+
V in Na3VO4 : [Ne] 3s2p6 d0
V2O5 and iron has maximum of + 3 in Fe2O3.
4. (b) Gun metal is an alloy of Cu, Zn and Sn. V in [V(H2O)6] SO4 : [Ne] 3s2 p6 d3
2+

5. (a) Transition elements form coloured ions due 15. (d)


to d-d transitions. In the presence of 16. (a) As we move down in groups 13, 14 and 15,
ligands, there is splitting of energy levels inertness of s2 electrons of valence shell
of d-orbitals. They no longer remain increases (inert pair effect). Thus in metals
degenerated. So, electronic transition may present at the bottom of the groups 13, 14,
occur between two d-orbitals. and 15 lower oxidation state becomes more
6. (c) CuSO4 and KCN reacts to form complex. important , viz. Pb (+2 state rather +4), Bi
CuSO 4 2KCN Cu(CN) 2 KSO 4 (+3 state rather +5) and Ti (+1 rather than +3).
Cu(CN) 2 2Cu CN (CN) 2 17. (a) SbF5 , being the stronger Lewis acid,
2Cu CN 6KCN 2K 3 [Cu(CN) 4 ] . displaces the weaker one (MnF4) from its
salt. MnF4, being unstable, decomposes to
7. (c) The paramagnetic character is exhibited by
species having unpaired electrons. Further give MnF3 and fluorine.
paramagnetic character is directly K 2 MnF6 2SbF5 2SbF6 MnF4
proportional to number of unpaired Unstable
electrons.
Configuration Unpaired electron
1
MnF4 MnF3 F2
Cu+ [ Ne] 3s2 p6 d10 0 2
Cu2+ [ Ne] 3s2 p6 d9 1 18. (b) Deep Blue.
Fe2+ [ Ne] 3s2 p6 d6 4 19. (d) Co-60 is used as anticancerous among the
Fe3+ [Ne] 3s2 p6 d5 5 given radioactive isotopes. It emits -
8. (a) Mohr's salt is FeSO4 .(NH 4 ) 2 SO 4 .6H 2 O particles and energetic gamma rays, for
9. (c) German silver is alloy of Cu, Zn and Ni. Cu which reason it is used in radiation therapy.
is 60%, Zn is 20%, and Ni is 20%. 20. (d) The colour is due to presence of unpaired
10. (d) Composition of duralumin is 95% Al, 4% d-electrons.
Cu, 0.5% Mn, 0.5% Mg. Ti3+ = [Ar] 3d1 (one unpaired electron)
It is light tough and anti-corrosive alloy Fe3+ = [Ar] 3d5 (five unpaired electrons)
which is used in making aeroplanes etc. Co2+ = [Ar] 3d7 (three unpaired electrons)
11. (c) Mohr's salt is (NH 4 ) 2 SO4 . FeSO4 . 6H2 O. So all of them are coloured.
21. (a) The permanganate ion has an intense
12. (c) BaO + ZnO BaZnO2 purple colour. Mn (+ VII) has a d 0
Philosopher’s wool
configuration. So the colour arises from
13. (b) Casiterite is an ore of tin (Sn). charge transfer and not from d—d spectra.
The d- and f-Block Elements C-133

In MnO4 – an electron is momentarily 31. (b) As a result of lanthanoid contraction change


changing O– – to O– and reducing the in ionic radii on going from elements of 4d
oxidation state of the metal from Mn(VII) to to 5d transition series is very small. Thus
Mn (VI). Charge transfer requires that the chemical properties of 4d and 5d series of
energy levels on the two different atoms transition elements are similar.
are fairly close.
32. (b) As a result of lanthanide contraction Zr4+
O = (8) = 2, 6 ; Mn (25) = 2 , 8, 15 and Hf4+ possess almost the same ionic
K L K L M
Hence the charge transfer occurs from radii. Ce4+ is an oxidising agent. Ce4+ gains
L M. electron to acquire more stable Ce3+state.
22. (b) Fe and Pt do not form amalgam La(OH)3 is the most basic among lanthanide
Fe vessel
hydroxides.
Ag Hg Ag Hg 33. (a) The ionisation energies increase with
Silver amalgam
increase in atomic number. However, the
Vessel made of other metal will form trend is irregular among some d-block
amalgam with liberated mercury. elements. On the basis of electronic
configuration, the
23. (c) Chromium is in VI oxidation state in CrO24
Zn : 1s 2 2s 2 p 6 3s 2 p6 d 10 4s 2
ion and therefore, it has no unpaired electron
Fe : 1s 2 2s 2 p 6 3s 2 p 6 d 6 4s 2
in its d-orbital. However, due to charge
transfer Cr — O, this ion shows intense Cu : 1s 2 2s 2 p6 3s 2 p 6 d 10 4s1
yellow colour. Cr : 1s 2 2s 2 p6 3s 2 p 6 d 5 4s1
24. (b) Fe3+ does not form complex compound with IE1 follows the order : Zn > Fe > Cu > Cr
NH3 whereas Cu2+ and Zn2+ form complex Type B : Assertion Reason Questions
with NH3.
25. (b) Copper sulphide is black in colour. Cu2+ is 34. (c) Pure iron is not used for making tools and
placed in II group of inorganic qualitative machines as it is soft. However, an alloy of
analysis. It is precipitated in the form of iron with carbon is hard and less reactive
sulphide by passing H2S in presence of than pure iron.
dil. HCl. 35. (e) Iron is not found in free state in nature
Cu2+ + H2S — CuS + 2H+ because it is highly reactive.
36. (a) Solution of FeCl 3 produces brown
black
precipitate on standing due to hydrolysis
26. (b) The ionic radii of trivalent lanthanides in water
decreases progressively with increase in
FeCl3 3H 2 O Fe(OH)3 3HCl
atomic number. This decrease is known as +
lanthanide contraction. 37. (e) Cu (cuprous ion) does not have any
27. (b) 2Ag O3 unpaired electron while cupric (Cu2+) ion
Ag 2 O O 2 has one unpaired electron in 3d shell.
Ag 2O O3 2Ag 2O2 Cu+ = 3d10 4s0
(black)
Cu2+ = 3d9 4s0
28. (b) Mn2+ (3d5) is more stable than Mn 3+ (3d4).
Cuprous ion is colourless because it does
29. (d) In Cu Ar 3d10 there is no unpaired not have any unpaired electron but Cu2+
ion is blue in aqueous solution due to
electron, Cu 2 Ar 3d 9 contains one formation of complex with water molecules.
unpaired electron hence coloured. So assertion is wrong but reason is true.
38. (b) The colour of CrO24 is due to charge
30. (d) Magnetic moment n n 2 where
transfer spectra. There is no electron in the
n = number of unpaired electrons
d-orbital of Cr (VI). So, no electronic
15 nn 2 n=3 excitation is possible with their d-orbital.
EBD_7100
C-134 Topicwise AIIMS Solved Papers – CHEMISTRY
39. (c) The free gaseous Cr atom has six unpaired 42. (a) In metal carbonyls a pair of –bond arises
electrons, because one electron jumps from from overlap of filled d-orbitals on the metal
4s2 orbital to 3d orbital making latter exactly with a pair of –antibonding orbitals
half-filled, a comparatively stable entity. projecting from the carbon of the CO.
Cr24 [Ar] 3d5 4s1. The –bonding has the effect of weakening
40. (a) The orange colour of SnI4 is due to charge the C–O bond as compared with free CO.
transfer spectra. Blue light is absorbed by Hence C–O bond in metal carbonyls is long.
the compound which causes the transfer Hence assertion and reason both are true
of an e – from I to Sn. (Here charge is and reason is a correct explanation of
transferred from one atom to another atom.) assertion.
The reflected light thus contains a higher 43. (b) The electronic configurations of Europium
proportion of red and orange. (II) and cerium (II) are Eu2+ : [Xe] 4f7,5d10
41. (a) In lanthanide ions the 4 f e– are well shielded Ce2+ : [Xe] 4f1, 5d1
from external field by the overlying 5s and In Eu2+, f-subshell is half filled and d-
5p electrons. Thus the magnetic effect of subshell is completely filled and thus more
the motion of the electron in its orbital is stable.
not quenched out. Thus the magnetic moment 44. (a)
must be calculated taking into account both 45. (b) Due to larger surface area and variable
the magnetic moment from unpaired e – valencies to form intermediate absorbed
spins and that from orbital motion. complex easily, transition metals are used
g J(J 1) as catalysts.
46. (b) The magnetic moment values are lesser than
Where g 1 1 S(S 1) L(L 1) the theoreticaly predicted values due to the
2 2J(J 1) fact that 5f electrons of actinides are less
J = L – S when the shell is less than half fill effectively shielded which results in
= L + S when the shell is more than half fill. quenching of orbital contribution.
In Dy3+ ([Xe] 4f 9) f - level is more than half
47. (c) The assertion is correct but the reason is
fill hence spin and orbital motion work
false. Actually transition metal show
together, highest in lanthanoids.
variable valency due to very small difference
Hence, both assertion and reason are true
between the ns2 and (n – 1)d electrons.
and reason is correct explanation of assertion.
23 Coordination Compounds

TYPE A : MULTIPLE CHOICE QUESTIONS (a) Cu+ (b) Ag +


(c) Ni 2+ (d) Al(OH)3
1. The compound which is not coloured is [1997] 11. Which of the following is not considered as an
(a) K4[Fe(CN)6 ] (b) K3[Fe(CN)6 ] organometallic compound? [2004]
(c) Na 2[CdCl4 ] (d) Na 2[CuCl4 ] (a) cis-platin (b) Ferrocene
(c) Zeise's salt (d) Grignard reagent
2. The number of unpaired electrons in Ni(CO) 4 is
12. Which of the following does not have optical
(a) 0 (b) 1 [1997] isomer? [2004]
(c) 3 (d) 5 (a) [Co(NH3)3Cl3]
3. The organometallic compound is : [1997] (b) [Co(en)3]Cl3
(a) Ti (OCOCH 3 ) 4 (b) Ti(C2 H5 )4 (c) [Co(en)2Cl2]Cl
(c) Ti (OC 6 H 5 ) 4 (d) Ti (OC 2 H 5 ) 4
(d) [Co(en)(NH3)2Cl2]Cl
4. Mercuric chloride is soluble in KI solution due to :
13. In which of the following pairs both the
(a) the formation of complex ion [1997]
complexes show optical isomerism? [2005]
(b) common iodide ion
(a) cis-[Cr(C2O4)2Cl2]3–, cis- [Co(NH3)4Cl2]
(c) none of the above
(b) [Co(en)3]Cl3, cis-[Co(en)2Cl2]Cl
(d) both (a) and (b)
2–
(c) [PtCl(dien)]Cl, [NiCl2Br2 ]2–
5. The EAN of Zn in Zn(OH) 4 complex is: (d) [Co(NO3)3(NH3)3], cis-[Pt(en)2Cl2]
(a) 16 (b) 26 [2000] 14. An aqueous solution of CoCl2 on addition of
(c) 36 (d) 46 excess of concentrated HCl turns blue due to
6. The reagent commonly used to determine formation of : [2005]
hardness of water titrimetrically is : [2003] (a) [Co(H2O)4Cl2] (b) [Co(H2O)2Cl4]2–
(a) oxalic acid (c) [CoCl4]2– (d) [Co(H2O)2Cl2]
(b) disodium salt of EDTA 15. The diamagnetic species is : [2005]
(c) sodium citrate (a) [Ni(CN)4]2– (b) [NiCl4]2–
(d) sodium thiosulphate (c) [CoCl4]2– (d) [CoF6]2–
7. The ion which is not tetrahedral in shape is 16. The correct order for the wavelength of
(a) BF4– (b) NH4+ [2003] absorption in the visible region is : [2005]
(c) [Cu(NH3)4] 2+ (d) NiCl42–
8. The complex used as an anticancer agent is (a) [ Ni( NO 2 ) 6 ]4 [ Ni( NH 3 ) 6 ]2
(a) meso –[Co(NH3)3Cl3] [2003] [ Ni (H 2 O) 6 ]2
(b) cis –[PtCl2(NH3)2] 4
(b) [ Ni( NO 2 ) 6 ] [ Ni ( H 2 O ) 6 ]2
(c) cis–K2[PtCl2Br2]
(d) Na2CoCl4 [ Ni ( NH 3 ) 6 ]2
9. The ligand called acid is : [2003] (c) [ Ni( H 2 O ) 6 ] 2 [ Ni( NH 3 ) 6 ] 2
(a) CO (b) NH3
[ Ni( NO 2 ) 6 ]4
(c) C2O42– (d) Ethylenediamine 2
10. Which one of the following forms, with an excess (d) [ Ni( NH 3 ) 6 ] [ Ni ( H 2 O) 6 ] 2
of CN– (cyanide), a complex having co-ordination [ Ni( NO 2 ) 6 ]4
number two? [2004]
EBD_7100
C-136 Topicwise AIIMS Solved Papers – CHEMISTRY

17. The pair in which both species have same (c) optical isomerism
magnetic moment (spin only value) is : [2006] (d) none of the above
(a) [Cr(H2O)6]2+ , [CoCI4]2– 26. Among the following the compound that is both
paramagnetic and coloured is [2009]
(b) [Cr(H 2O 6 ) 2 , [Fe(H 2O) 6 ]2
(a) K2Cr2O7 (b) (NH4)2 [TiCl6]
(c) [Mn (H 2O) 6 ) 2 , [Cr(H 2O) 6 ]2
(c) VOSO4 (d) K3Cu (CN)4
(d) [CoCl 4 ) 2 , [Fe(H 2 O) 6 ]2
27. Which of the following complex has zero
18. The number of possible isomers of an octahedral
magnetic moment (spin only)? [2009]
complex [Co(C2O4)2 (NH3)2 ]–is : [2006]
(a) 1 (b) 2 (a) [Ni(NH3 )6 ]Cl2 (b) Na 3[FeF6 ]
(c) 3 (d) 4 (c) [Cr(H2O)6 ]SO4 (d) K 4 [Fe(CN)6 ]
19. The ligands in anti-cancer drug cis-platin are:
28. A chemist wants to determine the molecular
(a) NH3, CI (b) NH3, H2O [2006]
geometry of the [CoCl4]2– ion. Which of the
(c) Cl, H2O (d) NO, Cl
following gives the best suggestion for a
20. Which statement is true for ferrocene? [2007]
measurement and for the interpretation of that
(a) All Fe-C are of equal length measurement? [2009]
(b) It has sandwich type structure (a) Using absorption spectroscopy, measure
(c) It was the first discovered organometallic
max then calculate o for octahedral
compound geometry
(d) All of these. (b) Measure the molecule’s magnetic moment and
21. During estimation of nickel, we prepare nickel use the result to estimate the number of
dimethylglyoxime, a scarlet red solid. This unpaired spins in the molecule. If this number
compound is ______. [2007] is low, the geometry is likely to be square
(a) ionic (b) covalent planar; otherwise, it is likely to be tetrahedral
(c) metallic (d) non-ionic complex. (c) Measure the molecule’s magnetic moment
22. Which of the following metal ions will form and use the result to estimate the number of
complexes with the same magnetic moment and unpaired spins in the molcule.If this number
geometry irrespective of the nature of ligands? is low, the geometry is likely to be tetrahedral;
(a) Ni2+ (b) Fe2+ [2007] otherwise, it is likely to be square planar
(c) Cu 2+ (d) Co 2+
(d) Measure the molecule’s magnetic moment
23. Wavelength of red light is absorbed by the and use the result to estimate the number
complex [2007] of unpaired spins in the molecule. If this
(a) [Cu(CN)4]2– (b) [Cu(NH3)4]2+ number is low, the geometry is likely to be
(c) CuSO4 (d) Cu(CN)2 tetrahedral; otherwise, it is likely to be
octahedral
24. in the change [Cu(H 2 O 6 )] 2+ HCl
29. How many pairs of enantiomers are possible for
[CuCl(H2O)5]+, the colour changes from [2007] following complex compound, [M (AB) (CD) ef]n±
(a) blue to green (b) blue to pink (where AB, CD– Unsymmetrical bidentate ligand,
(c) pink to green (d) pink to blue. e, f–monodentate ligands) [2009]
25. [Fe(NO2)3Cl3] and [Fe(ONO)3Cl3] shows (a) 20 (b) 5
(a) linkage isomerism [2008] (c) 10 (d) 8
(b) geometrical isomerism
Coordination Compounds C-137

3
30. A complex [CoL6]n+ where L is neutral ligand 36. The [Fe(CN) 6 ] complex ion [2011]
has a magnetic moment = 4.5 B. M. Hence, (a) exhibits planar geometry
(a) Co must be in +2 oxidation state [2010] (b) is diamagnetic
(b) L must be a strong ligand (c) should be very stable
(c) The complex must be highly dist (d) has 2 unpaired electrons
(d) Co must be in +3 oxidation state 37. When AgNO 3 is added to a solution of
31. Silver chloride dissolves in: [2010] Co(NH3)5Cl3, the precipitate of AgCl shows two
(a) Water (b) Conc. HCl ionizable chloride ions. This means : [2012]
(c) NH4OH (d) CCl4 (a) Two chlorine atoms satisfy primary
32. The IUPAC name of the complex Hg [Co(CNS )4]
valency and one secondary valency
is [2010]
(b) One chlorine atom satisfies primary as well
(a) mercury tetrathiocyanatocobaltate (II)
as secondary valency
(b) mercury cobalttetrasulphocyano (II)
(c) Three chlorine atoms satisfy primary valency
(c) mercury tetrasulphocyanidecobalt (II)
(d) Three chlorine atoms satisfy secondary
(d) tetrasulphocyantocobalt mercurate (II)
valency
33. What is incorrect about homoleptic metal
38. The hypothetical complex chloro-
carbonyls? [2010]
(a) M – C, -bond is formed by donation of diaquatriamminecobalt (III) chloride can be
lone pair of electrons from CO represented as [2013]
(b) M – C, -bond is formed by back donation (a) [CoCl(NH3)3(H2O)2 ]Cl2
of electron from filled d-orbital of metal to (b) [Co(NH3)3(H2O)Cl3]
vacant p-orbital of carbon (c) [Co(NH3)3(H2O)2Cl]
(c) M – CO bonding produce synergic effect (d) [Co(NH3)3(H2O)3]Cl3
(d) Metal carbonyl contain only -bonds 39. The coordination number and the oxidation state
34. In Fe2(CO)9, the two iron atoms are [2011]
of the element ‘E’ in the complex [E(en)2(C2O4)]NO2
(a) linked only directly
(where (en) is ethylene diamine) are, respectively,
(b) linked directly along with 3 CO molecules
(a) 6 and 2 (b) 4 and 2 [2014]
as bridging ligands
(c) linked only through 3 CO molecules as (c) 4 and 3 (d) 6 and 3
bridging ligands 40. In the isoelectronic series of metal carbonyl, the
(d) joined through one CO group as bridging CO bond strength is expected to increase in the
ligands. order: [2014]
35. Which method can be used to distinguish
(a) [Mn(CO) 6 ] [Cr(CO)6 ] [V (CO) 6 ]
[Co(NH3 )6 ][Cr(NO 2 ) 6 ] and
[Cr(NH3)6][Co(NO2)6] [2011] (d) [V (CO)6 ] [Cr(CO)6 ] [Mn(CO)6 ]
(a) by measurement of their conductivity
(c) [V (CO)6 ] [Mn(CO)6 ] [Cr(CO) 6 ]
(b) by titration method
(c) by precipitation method with AgNO3 (d) [Cr(CO)6 ] [Mn(CO)6 ] [V (CO)6 ]
(d) by electrolysis of their aqueous solutions
EBD_7100
C-138 Topicwise AIIMS Solved Papers – CHEMISTRY

41. Which of the following is paramagnetic? [2014] 46. Which of the following coordination compounds
would exhibit optical isomerism? [2017]
4
(a) Fe CN (b) Ni CO 4 (a) pentamminenitrocobalt(III) iodide
6
(b) diamminedichloroplatinum(II)
2 3 (c) trans-dicyanobis (ethylenediamine)
(c) Ni CO 4
(d) CoF6
chromium (III) chloride
42. Coordination compounds have great importance (d) tris-(ethylendiamine) cobalt (III) bromide
in biological systems. In this context which of
TYPE B : ASSERTION REASON QUESTIONS
the following statements is incorrect ? [2015]
(a) Cyanocobalamin is B12 and contains cobalt Directions for (Qs. 47-52) : These questions consist
(b) Haemoglobin is the red pigment of blood of two statements, each printed as Assertion and
and contains iron Reason. While answering these questions, you are
required to choose any one of the following five
(c) Chlorophylls are green pigments in plants
responses.
and contain calcium
(a) If both Assertion and Reason are correct and
(d) Carboxypeptidase - A is an exzyme and
the Reason is a correct explanation of the
contains zinc.
Assertion.
43. Consider the following complex
[Co(NH3)5CO3]ClO4. The coordination number, (b) If both Assertion and Reason are correct but
Reason is not a correct explanation of the
oxidation number, number of d-electrons and
number of unpaired d-electrons on the metal are Assertion.
respectively [2015] (c) If the Assertion is correct but Reason is incorrect.
(a) 6, 3, 6, 0 (b) 7, 2, 7, 1 (d) If both the Assertion and Reason are incorrect.
(c) 7, 1, 6, 4 (d) 6, 2, 7, 3 (e) If the Assertion is incorrect but the Reason is
44. The deep blue colour produced on adding correct.
excess of ammonia to copper sulphate is due to 47. Assertion: AgCl dissolves in NH4OH solution.
presence of [2016] Reason: Due to formation of a complex [1998]
2
48. Assertion : NF3 is a weaker ligand than N(CH3)3
(a) Cu 2 (b) Cu(NH 3 ) 4 Reason : NF3 ionizes to give F– ions in aqueous
2 2 solution. [2003]
(c) Cu(NH 3 ) 6 (d) Cu(NH3 )2
49. Assertion : The [Ni(en)3]Cl2 (en = ethylene-
45. 0.02 mole of [Co(NH3)5Br]Cl2 and 0.02 mole of diamine) has lower stability than [Ni(NH3)6]Cl2.
[Co(NH3)5Cl]SO4 are present in 200 cc of a Reason : In [Ni(en)3]Cl2, the geometry of Ni is
solution X. The number of moles of the trigonal bipyramidal. [2004]
precipitates Y and Z that are formed when the 50. Assertion : Potassium ferrocyanide is
solution X is treated with excess silver nitrate diamagnetic, whereas potassium ferricyanide is
and excess barium chloride are respectively paramagnetic.
[2016] Reason : Crystal field splitting in ferrocyanide
(a) 0.02, 0.02 (b) 0.01, 0.02 ion is greater than that of ferricyanide ion.
(c) 0.02, 0.04 (d) 0.04, 0.02 [2005]
Coordination Compounds C-139

51. Assertion :[Co(NO2)3(NH3)3] does not show 54. Assertion : A chelating ligand must possess two
optical isomerism. or more lone pairs at such a distance that it may
Reason : It has a plane of symmetry. [2006] form suitable strain free rings at the metal ion.
52. Assertion : Copper sulphate solution is not Reason : H2N–NH2 is a chelating ligand. [2010]
stored in zinc vessel. 55. Assertion : Low spin complexes have lesser
Reason : Zinc forms complex with CuSO4. number of unpaired electrons.
[2007]
Reason : [FeF6]3– is a low spin complex. [2011]
Directions for (Qs.53-58) : Each of these questions
56. Assertion : [FeF6]3– is a low spin complex.
contains an Assertion followed by Reason. Read them
Reason : Low spin complexes have lesser number
carefully and answer the question on the basis of
of unpaired electrons. [2013]
following options. You have to select the one that
best describes the two statements. 57. Assertion : Ethylenediaminetetraacetate ion
forms an octahedral complex with the metal ion.
(a) If both Assertion and Reason are correct and
Reason : It has six donor atoms which coordinate
Reason is the correct explanation of Assertion.
simultaneously to the metal ion. [2014]
(b) If both Assertion and Reason are correct, but
3–
58. Assertion : [Fe(CN)6] is weakly paramagnetic
Reason is not the correct explanation of
Assertion. while [Fe(CN)6]4– is diamagnetic.
(c) If Assertion is correct but Reason is incorrect. Reason : [Fe(CN)6]3– has +3 oxidation state
(d) If both the Assertion and Reason are incorrect. while [Fe(CN)6]4– has +2 oxidation state.
53. Assertion : When NO reacts with FeSO4, a [2017]
brown coloured complex is formed.
Reason : In the complex, the coordination
number of Fe is 6. [2009]
EBD_7100
C-140 Topicwise AIIMS Solved Papers – CHEMISTRY

TYPE A : MULTIPLE CHOICE QUESTIONS The shape of dsp2 hybridisation is square


planar.
1. (c) In Na2[CdCl4], Cd has oxidation state +2, 8. (b) The complex used as an anticancer agent is
so, its electronic configuration is 4d104s0. cis-platin, cis- [PtCl2(NH3)2].
or all the 4 d orbitals are fully filled. H3N NH 3
Hence, there will not be d-d transition
resulting in colour. So, it is colourless.
2. (a) No. of unpaired electrons in Ni(CO)4 is zero Pt
as CO is a strong ligand which pairs up the
unpaired electrons of Ni.
Cl Cl
3d 4s
9. (a) Due to back bonding present between
Ni
metal and carbonyl ligand in metal carbonyl,
Ni in presence of CO ligand CO is termed as -acid ligand.
10. (b) Among the given metal ions only Ag+ forms
complex with CN– having co-ordination
3. (b) Organo-metallic compound is (b) as it number 2.
contains metal-carbon bonds. In others, Ag + 2CN – [Ag(CN)2 ]–
direct link of carbon with metal is not Coordination number of metals is defined
present. as the number of bonds by which ligands
4. (a) HgCl2 4Kl K 2 HgI 4 2KCl are attached to the metal atom.
11. (a) The structural formula of cis-platin is
HgCl 2 is soluble due to formation of
H 3N Cl
complex ion HgI24 ion.
5. (c) EAN for any complex ion Pt
= [At.No. of the central atom – Oxd. No. of H 3N Cl
the central atom +2 (coordination No. of Since no carbon–metal bond is present, it
central atom] is not an organometallic compound.
= 30 – 2 + 2 (4) = 36 12. (c) Cl Cl
6. (b) The reagent commonly used is EDTA
(ethylene diamine tetraacetate) which acts
en Co en en Co en
as ligand for metal ions responsible for
hardness of water. They form metal ligand
complex which renders the metal ion
inactive by making them non-ionisable. Cl Cl
7. (c) In [Cu(NH3)4]2+ the hybridisation is dsp2 As clear from the figure, the mirror image is
superimposable. So, they are not different
Cu2+ = 3d 94s0
compounds.
en en
When ammonia approaches towards the
Cu2+ it is observed that the unpaired
13. (b) en en
electron in the d-orbital gets excited to Co Co
4p-orbital resulting in dsp2 hybridisation.
en en

sp2Hybridisation [Co(en)3]+3
Coordination Compounds C-141

Cl Cl 18. (c) The complex has three isomers (cis- and


trans); cis-isomer shows optical isomerism
Cl Cl O4C2 C2O4 HN 3
CO 2 4
en Co Co en Co Co
H3N NH3 O4C2 NH3
en en
cis- (shows optical isomerism) trans-
cis-[Co(en)2Cl2]Cl 19. (a) The ligands in cis-platin [PtCl2(NH3)2] are
They form non-superimposable mirror Cl and NH3.
image, so, th ey are optically active
20. (d)
compounds.
14. (c) CoCl2 is a weak Lewis acid. It reacts with
chloride ion to produce salt containing the
tetrahedral [CoCl4]2– ion which is blue in
colour. Fe
15. (a) In [Ni(CN)4]2–, nickel has oxidation no. of
+2. So, electronic configuration of its outer
most orbital is as follows :

3d 4s Ferrocene was the first organometallic


However, CN–is a strong ligand which compound discovered by G. Wilkinson in
forces the electron to pair up. Thus two 1957. The bonding in these aromatic
unpaired electrons are forced to pair up by san dwich type structure is better
CN–. considered as -bonding involving the
lateral overlap of dxz and dyz orbitals on Fe
with the delocalized -aromatic orbital from
each cylopentadienyl ring. All the five C-
So, it becomes diamagnetic. atoms are equidistant from Fe.
16. (a) For any metal cation, the magnitude of 0 Hence all of these statements are true.
depends upon the nature of ligand. Higher 21. (d) Nickel dimethylglyoxime is a non ionic
the value of 0, lower will be the wave complex.
length absorbed. 0 is crystal field 22. (c) Cu2+ forms complexes with the same
stabilisation energy. magnetic moment and geometry
The value of 0 for ligands varies as follows irrespective of the nature of ligand. It can
H 2 O NH3 NO2 be explained by electronic configuration.
So, the wavelength absorbed will vary in 3d
reverse order or NO 2 NH 3 H 2 O 2+
17. (b) [Cr(H2O)6]2+. Here Cr is in Cr2+ form Cu
It has 9 electrons in 3d shell. Hence any
Cr 2 incoming ligand whether it is strong or
3d 4s weak will result in the formation of same
geometry (if number of ligands is same)
Fe 2 because any ligand can do nothing with
3d 4s this unpaired e– in 3d shell. Moreover, the
In [Fe(H2O)]2+, Fe is in Fe2+ form. Both will complex formed will have same magnetic
have 4 unpaired electrons. moment due to this unpaired e–.
EBD_7100
C-142 Topicwise AIIMS Solved Papers – CHEMISTRY
23. (b) The compound which appears blue green, 29. (c)
absorb red light as blue-green is e e f f
A f A f A e A e
complementary to red colour.
M M M M
Here [Cu(NH3)4]2+ appears blue-green, so B D B C B D B C
it absorbs red colour. C D C D
e e f f
24. (b) [Cu(H 2O)6 ]2 [CuCl(H 2O)5 ] B f B f B e B e
blue green pink M M M M
25. (a) The given compound contains –NO2 group A D A C A D A C
C D C D
which can donate electrons either from O– e e
or from –N . Thus, it shows linkage A C A D
isomerism. Thus the two given compounds M M
B D B C
are linkage isomers, one is nitrite –O–N = O f
f
O 30. (d) Co must be in +3 oxidation state and the
form and the other is nitro, –N form. ligand L should be a weak ligand.
O
26. (c) (d) In K3[Cu(CN)4] Cu is in + 1 oxidation Co3+ = [Ar]3d6
state hence has no unpaired electron n = 4, = 4.5 BM.
hence colourless and diamagnetic. 31. (c) AgCl is insoluble in water, conc. HCl and
(b) In (NH4)2 [TiCl6] Ti is in + 4 oxidation CCl4. It dissolves in NH4OH solution due to
state, hence has no unpaired electron the formation of complex salt.
hence colourless and diamagnetic. AgCl + 2NH4OH — [Ag(NH3)2]Cl + 2H2O
(c) In VOSO4, V is in +4 oxidation state Diamminesilver (I) chloride
hence has one unpaired electron, thus 32. (a) The IUPAC name of the given complex is
it is coloured and paramagnetic. mercury tetrathiocyanatocobaltate (II).
(a) In K2Cr2O7, Cr is in +6 oxidation state, 33. (c) It is incorrect statement.
hence has no unpaired electron and All other statements, i.e. (a), (b) and (d) are
thus it is diamagnetic. Though K2Cr2O7 correct.
has no unpaired electron but it is O
coloured. This is due to charge transfer. O C O
27. (d) [Ni(NH3)6]Cl2 sp3d2 hybridisation C C
Fe
2 unpaired electrons
Na3[FeF6] sp3d2 hybridisation 34. (b) O=C O=C C=O
3 unpaired electrons Fe
[Cr(H2O)6]SO4 d2sp3 hybridisation C
O C C O
3 unpaired electrons O
K4[Fe(CN)6] d2sp3 hybridisation 35. (d) In one case, on electrolysis of aqueous
No unpaired electron solution, the complex ion of cobalt i.e.,
[Co(NH 3 ) 6 ] 3+ of the complex
Zero magnetic moment means all the
[Co(NH3 )6 ][Cr(NO2 )6 ] moves towards
electrons paired.
cathode (i.e., negative electrode) and on this
28. (b) In the complexes of C.N. 4, square planar electrode finally cobalt would be deposited.
geometry gives rise to low spin (spin paired) In another case, on electrolysis of aqueous
configurations whereas tetrahedral solution, the complex ion of chromium
complexes display high spin configurations. i.e., [Cr(NH 3 ) 6 ] 3+ of the complex
Coordination Compounds C-143

[Cr(NH3 )6 ][Co(NO2 )6 ] moves towards electrons due to weak ligan d hence


cathode (i.e., negative electrode) and on 3
this electrode chromium would finally be CoF6 is paramagnetic.
deposited. 42. (c) The chlorophyll molecule plays an
36. (c) Both the high charge density of Fe 3 in important role in photosynthesis, contain
[Fe(CN) 6 ]3 and strong basic (Lewis)
porphyrin ring and the metal Mg not Ca.
character of the ligand CN attribute to
43. (a) [Co(NH3 )5CO3]ClO4. Six monodentate
the stability of the complex.
ligands are attached to Co hence C. N. of
37. (a) Since the precipitate of AgCl shows two
ionisable chloride ion the complex must Co = 6;
have the structure. O. N. = x + 5 × (0) + 1 × (–2) + 1× (–1) = 0
x = + 3 ; electronic configuration of
[Co( NH 3 ) 5 Cl]Cl 2 2AgNO 3
Co [Ar] 3d 6 4s 0 hence number of d
3+
[Co( NH 3 )5 Cl]( NO 3 ) 2 2AgCl electrons is 6. All d-electrons are paired
Hence two chlorine atoms satisfy the primary due to strong ligand hence unpaired
valency and one, secondary valency. electron is zero.
38. (a) The complex chlorodiaquatriammine cobalt 44. (b) CuSO 4 4NH 3 [Cu(NH3 ) 4 ]SO 4
(III) chloride can have the structure
2
[CoCl(NH3)3(H2O)2]Cl2 Blue complex due to Cu(NH 3 ) 4
39. (d) In the given complex we have two bidentate
ligands 45. (d) When excess of AgNO3 and BaCl2 are
(i.e en and C2O4), so coordination number added to solution X.
of E is 6 [Co(NH3 )5 Br]Cl2 2AgNO3
(2 × 2 + 1 × 2 = 6) 1mole 2 moles
Let the oxidation state of E in complex be x, 0.02 mole
then [Co(NH3 )5 Br](NO3 )2 2AgCl(ppt.)(Y)
[x + (–2) = 1] or x – 2 = 1 1mole 2moles
or x = + 3, so its oxidation state is + 3 0.02 2 0.04 mole
Thus option (d) is correct. [Co(NH 3 )5 Cl]SO 4 BaCl 2
40. (b) CO bond strength is reciprocal to the extent 1mole
0.02 mole
of back donation involved in synergic
[Co(NH3 )5 Cl]Cl2 BaSO 4 (ppt.) (Z)
bonding. 1mole
41. (d) Fe2+ has electronic configuration 0.02mole
3d 4s 46. (d) The optical isomers are pair of molecules
due to strong which are non superimposable mirror
ligand images of each other.
Ni has electronic configuration
4s en en
3d
due to strong
ligand en en
Ni2+ has electronic configuration Co Co
3d 4s
due to strong
en en
ligand
[Co(en)3]3+ [Co(en)3]3+
Co 3 has electronic configuration Mirror
(dextro) (laevo)
3d 4s
The two optically active isomers are
No pairing of collectivity called enantiomers.
EBD_7100
C-144 Topicwise AIIMS Solved Papers – CHEMISTRY
Type B : Assertion Reason Questions 52. (c) Copper sulphate solution is not stored in
zinc vessel as Zn is more reactive than
47. (a) AgCl dissolves in NH4OH due to formation
of complex. copper and has tendency to replace Cu from
its aqueous solution.
AgCl 2NH 4 OH [Ag(NH 3 ) 2 ]Cl 2H 2 O
48. (c) NF 3 is a weak ligand due to high Zn(s) + CuSO4(aq) ¾¾
® ZnSO4(aq) + Cu(s)
electronegativity of fluorine which
withdraws electorns from N, with the result Zn cannot form complex with CuSO4
lone pair of electrons on N atom can't be although [Zn(H2O)4]SO4 exists in aqueous
ligated. N(CH3)3 is a strong ligand because ZnSO4 solution. Hence assertion is true but
CH3 groups are electron releasing and thus reason is false.
increase electron availability on N atom. 53. (b) In the qualitative analysis of nitrate, brown
49. (d) [Ni(en) 3 ]Cl 2 is more stable than ring is formed due to the formation of
[Ni(NH3)6]Cl2 because ethylenediamine is [Fe(H2O)5(NO)]2+. The chemical reactions
a bidentate ligand, hence it forms chelating showing the formation of complex are as
ring with Ni2+ ion. follows:
50. (c) In potassium ferrocyanide, Fe is in the form
2NaNO3 + H2SO4 2NaHSO4 + 2HNO3
Fe2+ and in potassium ferricyanide, Fe is in
the form Fe3+. CN– is a strong legand. So, it 2HNO3 + 6FeSO4 + 3H2SO4
will pair up all the 3d6 electrons of Fe3+ and 3Fe2(SO4)3 + 2NO + 4H2O
make it diamagnetic. In Fe2+, all the 3d5
electrons are not paired up. One electron FeSO4 + NO + 5H2O
remains unpaired. So, it is paramagnetic. [Fe(H2O)5(NO)]SO4
Fe2+ in presence of CN– in K4 (Fe(CN)6] It is clearly seen from the formula, that the
3d 3s 4p coordination number of Fe in the brown ring
complex is six (five H2O and one NO). Thus
×× ×× ×× ×× ××××
both assertion and reason correct but
n = 0, diamagnetic, d2sp3 hybridization reason is not the correct explanation of
assertion.
Fe3+ in presence of CN– in K3 [ Fe(CN)6]
54. (c) H2N–NH2 does not act as chelating ligand.
×× ×× ×× ×× ×××× The coordination by hydrazine leads to a
three membered highly unstable strained
m = 1, paramagnetic, d 2sp3 hybridization ring and thus, it does not act as chelating
However, the reason is false because agent.
crystal field splitting in ferrocyanide is less 55. (c) [FeF6]3– is a high spin complex since F– is a
than in ferricyanide ion (higher the oxidation weak ligand.
state of the metal, greater the crystal field 56. (d) [FeF6]3– is a high spin complex since F– is
splitting). a weak ligand.
51. (a) Optical isomerism is found in octahedral
complexes with 1, 2 or 3 symmetrical 57. (a) Reason is the correct explanation of
bidentate ligands only. Since given Assertion.
compound is not having any bidentate 58. (b) Both Assertion and Reason are true but
ligand, it will not show optical isomerism. Reason is not the correct explanation of
It is because it has plane of symmetry, a statement-1. [Fe(CN) 6 ] 3– is weakly
plane which is perpendicular to equitorial paramagnetic as it has unpaired electrons
plane. Thus both A and R are true and R is while [Fe(CN)6]2– has no unpaired electron.
explantion of A.
It is diamagnetic.
Haloalkanes and Haloarenes C-145

24 Haloalkanes and Haloarenes

TYPE A : MULTIPLE CHOICE QUESTIONS conc . HNO3 conc . H 2SO 4


8. Heat
1. Chloroform and conc. HNO3 react to produce
[1997] Cl2 / FeCl 3
X Y
(a) CHCl2NO2 (b) CHCl2HNO3 The product Y is [2000]
(c) CCl3NO2 (d) CCl3NO3 (a) p-chloronitrobenzene
2. Benzene diazonium chloride reacts with (b) m-chloronitrobenzene
hypophosphorous acid to produce: [1998] (c) o-chloronitrobenzene
(a) phenol (d) o-p-dichloronitrobenzene
(b) benzene 9. Among the following, insecticide is : [2001]
(c) p-hydroxyazobenzene (a) BHC (b) Phosphene
(d) benzonitrile (c) Chloral (d) Aspirin
3. B.H.C. is used as an: [1998] 10. Which of the following is a chiral compound?
(a) Insecticide (b) Disinfectant (a) hexane [2002]
(c) Mosquito repellent(d) Antiseptic (b) n-butane
4. Which one of the following produces acyl halide
(c) methane
by treatment with PCl5? [1998]
(d) 2,3,4-trimethylhexane
(a) Alcohols
11. The reaction :
(b) Esters
Pyridine
(c) Acids C 2 H 5 OH SOCl 2 C 2 H 5Cl SO 2 HCl
(d) Carbonyl compounds is known as [2002]
5. When two halogens are attached to same carbon (a) Kharasch effect
atom, it is known as : [1997, 1999] (b) Williamson's synthesis
(a) vic-dihalide (b) gem-dihalide (c) Darzen's procedure
(c) , -dihalide (d) , -dihalide (d) Hunsdiecker reaction
6. Gammexane is : [1999] 12. Which of the following is most stable? [2002]
(a) Chloral (b) BHC (a) 1-butene (b) 1-pentene
(c) DDT (d) HCB (c) 2-butene (d) 2-pentene
7. The product obtained by treating 13. Among the following, the most reactive towards
CH 3 __ CH CH 2 HBr ? [1999] alcoholic KOH is : [2004]
(a) CH2=CHBr (b) CH3COCH2CH2Br
(a) CH 3 __ CH 2 __ CH 2 Br (c) CH3CH2Br (d) CH3CH2CH2Br
(b) CH 3 __ CH __ CH 3 14. Among the following, the dissociation constant
| is highest for : [2004]
Br (a) C6H5OH (b) C6H5CH2OH
(c) CH 2 BrCH CH 2 (c) CH 3C CH (d) CH3 NH3 Cl–
(d) __
CH3 CH CHBr
EBD_7100
C-146 Topicwise AIIMS Solved Papers – CHEMISTRY
15. Among the following which one can have a meso (a) Primary amine
form? [2006] (b) An amide
(a) CH 3CH(OH)CH(Cl)C 2 H 5 (c) Phenyl isocyanate
(d) A chain lengthened hydrocarbon
(b) CH 3CH(OH)CH(OH)CH 3 20. Which of the following is optically inactive?
[2007]
(c) C 2 H 5CH(OH)CH(OH)CH 3
H H
(d) HOCH 2 CH(Cl)CH 3 H 3C Cl Cl CH3
16. The correct increasing order of the reactivity of (a) Cl CH3 (b) H3C Cl
halides for SN1 reaction is : [2006] H H
(a) CH3 CH 2 X (CH3 )2 CH X H
CH 2 CH CH 2 X PhCH 2 X H 3C Cl
(c) H 3C Cl (d) none of these.
(b) (CH 3 )2 CH X CH3 CH 2 X H
CH 2 CH CH 2 X PhCH 2 X 21. C8H16 that can form cis-trans geometrical
isomers and also has a chiral centre, is [2008]
(c) PhCH 2 X (CH3 )2 CH X
CH3 CH 2 X CH 2 CH CH 2 X
(a) H (b) H
(d) CH 2 CH CH 2 X PhCH 2 X
(CH 3 ) 2 CH X CH 3 CH 2 X (c) Both of these (d) None of these
17. Which of the following compounds has the
22. (CH3)2C = CHCH3 + NOBr product.
highest boiling point? [2006]
The structure of the product is [2009]
(a) CH3CH 2CH 2 Cl
(a) (CH3)2 C(NO) – CH(Br)CH3
(b) CH 3CH 2CH 2 CH 2 Cl (b) (CH3)2 C(Br) – CH(NO)CH3
(c) CH 3CH(CH 3 )CH 2Cl (c) (CH3)2 CH – C(NO)(Br)CH3
(d) (CH 3 ) 3 CCl H
|
18. The major product formed in the following (d) H3C C – CH CH 3
reaction: | |
Aq.KOH
NO CH3
CH 3 CH (Cl )CH 2 CH 2 OH is:
(a) CH 3CH CH CH 2 OH [2006]
Br2 / h Alcoholic
23. Major (X)
(b) CH 2 CH CH 2OH KOH /

(c) CH 3 CH CH 2 H Br
| | Major (Y) Major (Z)
O CH 2 Peroxide
Major final product (Z) is [2009]
(d) CH 3 CH CH 2 CH 2 OH
|
OH Br
19. In the following sequence of the reations, what
is D? [2007] (a) (b)
Br
CH3 Br
[O] SOCl2
A B
Br
NaN3 Heat (c) (d)
C D
Haloalkanes and Haloarenes C-147

24. The pesticide DDT slowly changes to [2012] 29. The solution of a chemical compound reacts with
(a) CCl3-CHO and chlorobenzene AgNO3 solution to form a white preciptate of Y
(b) p, p'-Dichlorodiphenylethene which dissolves in NH4OH to give a complex Z.
When Z is treated with dilute HNO3, Y reappears.
(c) p, p'-Dichlorodiphenyldichloroethane
The chemical compound X can be: [2015]
(d) p, p'-Dichlorodiphenyldichloroethene (a) NaCl (b) CH3Cl
25. Rectified spirit is a mixture of [2012] (c) NaBr (d) NaI
(a) 95% ethyl alcohol + 5% water 30. The synthesis of alkyl fluorides is best accomplished
(b) 94% ethyl alcohol + 4.53 water by : [2017]
(c) 94.4% ethyl alcohol + 5.43% water (a) Finkelstein reaction
(b) Swarts reaction
(d) 95.87% ethyl alcohol + 4.13% water
(c) Free radical fluorination
26. Which of the following is an example of SN2
reaction? [2013] (d) Sandmeyer's reaction

(a) CH 3 Br OH CH 3 OH Br
TYPE B : ASSERTION REASON QUESTIONS
Directions for (Qs. 31-33) : These questions consist
(b) CH 3 C H CH 3 OH CH 3 C H CH 3 of two statements, each printed as Assertion and
| |
Br OH Reason. While answering these questions, you are
required to choose any one of the following five
H 2O
(c) CH 3 CH 2 OH CH 2 CH 2 responses.
(a) If both Assertion and Reason are correct and
(d) (CH 3 ) 3 C Br OH (CH 3 ) 3 COH Br
the Reason is a correct explanation of the
27. Identify Z in Assertion.
Aq. NaOH (b) If both Assertion and Reason are correct but
CH 3CH 2 CH 2 Br X
Reason is not a correct explanation of the
Al2O3 Cl2 / H 2O Assertion.
Y Z
(c) If the Assertion is correct but Reason is incorrect.
[2014]
(d) If both the Assertion and Reason are incorrect.
(a) Mixture of CH3CHClCH2Cl and
(e) If the Assertion is incorrect but the Reason is
CH3CHOHCH2Cl correct.
(b) CH3CHOHCH2Cl 31. Assertion : The presence of nitro group
(c) CH3CHClCH2OH facilitates nucleophilic substitution reactions in
(d) CH3CHClCH2Cl aryl halides.
28. Which of the following pairs is/are correctly Reason : The intermediate carbanion is
matched? [2015] stabilized due to the presence of nitro group.
Reaction Product [2006]
I. RX + AgCN RNC 32. Assertion : Chloral reacts with phenyl chloride
II. RX + KCN RCN to form DDT.
O Reason : It is an electrophilic substitution
III. RX + KNO2 R–N reaction. [2007]
O 33. Assertion : Alkyl iodide can be prepared by
IV. RX + AgNO2 R–O–N = O
treating alkyl chloride/bromide with NaI in
(a) Only I (b) I and II acetone.
(c) III and IV (d) I, II, III and IV Reason : NaCl/NaBr are soluble in acetone while
NaI is not. [2007]
EBD_7100
C-148 Topicwise AIIMS Solved Papers – CHEMISTRY
Directions for (Qs. 34-42) : Each of these questions 37. Assertion : Reimer-Tiemann reaction of phenol
contains an Assertion followed by Reason. Read them with CCl4 in NaOH at 340 K gives salicylic acid
carefully and answer the question on the basis of as the major product.
following options. You have to select the one that Reason : The reaction occurs through
best describes the two statements. intermediate formation of dichlorocarbene.
(a) If both Assertion and Reason are correct and [2010]
Reason is the correct explanation of Assertion. 38. Assertion : Cyanide (CN – ) is a strong
(b) If both Assertion and Reason are correct, but nucleophile.
Reason is not the correct explanation of Reason : Benzonitrile is prepared by the reaction
Assertion. of chlorobenzene with potassium cyanide.
(c) If Assertion is correct but Reason is incorrect. [2011]
(d) If both the Assertion and Reason are incorrect. 39. Assertion : Iodide ion combines with smaller
34. Assertion : Phenol on oxidation with KMnO4 group to avoid steric hindrance.
gives meso-tartaric acid. Reason : With HI, anisole gives iodobenzene
Reason : Pure phenol is colourless but turns and methyl alcohol. [2011]
pink due to oxidation to phenoquinone. [2009] 40. Assertion : CHCl3 is stored in dark bottles.
35. Assertion : SN2 reactions always proceed with
inversion of configuration. Reason : CHCl3 is oxidised in dark. [2015]
Reason : SN2 reaction of an optically active aryl 41. Assertion : Alkylbenzene is not prepared by
halide with an aqueous solution of KOH always Friedel-Crafts alkylation of benzene.
gives an alcohol with opposite sign of rotation. Reason : Alkyl halides are less reactive than acyl
[2010] halides. [2011, 2012, 2013, 2016]
36. Assertion : 4-Nitrochlorobenzene undergoes 42. Assertion : SN2 reaction of an optically active
nucleophilic substitution more readily than aryl halide with an aqueous solution of KOH
chlorobenzene. always gives an alcohol with opposite sign of
Reason : Chlorobenzene undergoes nucleophilic rotation.
substitution by elimination-addition mechanism Reason : SN2 reactions always proceed with
while 4-nitr ochlorobenzene undergoes retention of configuration. [2013, 2017]
nucleophilic substitution by addition-elimination
mechanism. [2010]
Haloalkanes and Haloarenes C-149

Type A : Multiple Choice Questions 11. (c) Alkyl halides can be prepared by treating
1. (c) Chloroform and HNO3 react to produce alcohol with SOCl2.
CCl 3 NO 2 which is also known as CH 3CH 2 OH SOCl 2
chloropicrin. It is used as insecticide. CH3CH 2 Cl SO2 HCl
2. (b) C6 H 5 N 2 Cl H 3 PO 2 H 2 O This reaction is known as Darzen's
C6 H 6 N2 HCl H 3 PO 3 procedure.
3. (a) B.H.C. is benzene hexachloride. It is used Note : SOBr2 and SOI2 are not used in this
as an insecticide. reaction because SOBr2 is less stable and
4. (c) Acid can give acyl halide with PCl 5 SOI2 does not exist.
CH 3 COOH PCl5 12. (c) We can explain it on the basis of
CH3 COCl POCl3 HCl hyperconjugation.
no. of -H
5. (b) CHCl 2 CH 2 Cl atoms
| | (a) CH3 – CH2 – CH = CH2 2
CH3 CH 2 Cl
(b) CH3 – CH2 – CH2 – CH = CH2 2
gem-dihalide vic–dihalide (c) CH3 – CH = CH – CH3 6
6. (b) Gammexane is commercial name of benzene (d) CH3 CH2 CH = CH CH3 5
hexachloride (B.H.C). It is an effective (c) is most stable having largest no. of
insecticide. It is also known as (6, 6, 6). hyperconjugative structures as it has
Chemically it is C6H6Cl6. largest no. of -H-atoms. (h yper-
7. (b) CH 3 CH CH 2 HBr conjugative H)
CH 3 CH CH 3 O
||
| 13. (b) In CH 3 C CH 2 CH 2 Br , the
Br
elimination reaction by alcoholic KOH will
Here Markownikoff's rule is followed which
states that negative part of addendum goes be facilitated due to presence of C = O group
to that carbon which has least hydrogen. which is electron with drawing group.
O
NO2 NO2 ||
CH 3 C CH 2 CH 2 Br KOH (alc )
Cl2 /FeCl3
8. (b) O
Cl ||
CH 3 C CH 2 CH 2 KBr H 2O
9. (a) BHC is also known as benzene hexachloride 14. (d) Dissociation constant will be highest for
(C6H6 Cl6). It is an insecticide and sold
under the brand name gammexane. It is also CH3 NH3 Cl because it is ionic in nature.
known as 6, 6, 6. 15. (b) A molecule having a plane of symmetry but
having chiral carbons will have meso form
H CH3 CH3 CH3 H H
| | | | | | CH 3
10. (d) H C C* C* C* C C H
H – C* – OH
| | | | | |
H H H H H H
H – C* – OH Plane of
Carbon atoms marked star (*) are Symmetry
CH 3
asymmetric. So it is a chiral compound.
EBD_7100
C-150 Topicwise AIIMS Solved Papers – CHEMISTRY
16. (a) SN1 reactions involve the formation of and it also has a chiral carbon. On the other
carboations, so greater the stability of the hand, structure (b) although shows cis-
carbocation formed by alkylhalide more will trans isomerism, it has no chiral carbon.
be its reactivity toward SN1 reaction. 22. (b) The reaction follows Markownikoff rule,
namely the bromide ion adds on to the
C6 H5CH 2 CH 2 CHCH 2 carbon h aving the least n umber of
hydrogen and the more positive part namely
the – NO group adds to the other carbon of
(CH3 )2 CH CH3CH 2
the double bond.
17. (b) Molecules having higher molecular weight Br
and less branching have higher boiling point. Br2 Alcoholic
23. (c)
h KOH/
Cl
(X) (Y)
| aq. KOH
18. (d) CH 3 H
CH CH 2 CH 2 OH
H – Br Br H Br
OH Peroxide
| (Z)
CH 3 CH CH 2 CH 2 OH 24. (d)
25. (d)
CH3 COOH
[O] 26. (a) Only 1° alkyl halides (i.e. CH3Br) undergo
19. (c)
SN2 reaction.
Benzoic acid 27. (b) aq. KOH
Toluene CH 3CH 2 CH 2 Br
(A)
COCl Al2O 3
SOCl2
CH 3CH 2 CH 2 OH
NaN 3 heat

Cl 2 / H 2 O
CH 3 CH CH 2 CH 3 .CHOH .CH 2 Cl
Benzoyl chloride
(B) 28. (b)
NCO 29. (a) NaCl + AgNO3 AgCl
CON3
Heat (White)
– N2 CH3Cl does not give ppt. NaBr and NaI give
(Curtius rearrangement) Phenyl isocyanate yellow and dark yellow ppt. respectively.
(C) (D) 30. (b) Alkyl fluorides are more conveniently
20. (c) It has centre of symmetry, so optically prepared by heating suitable chloro – or
inactive. bromo-alkanes with organic fluorides such
21. (a) Redrawing the given structures we get, as AsF3, SbF3, CoF2, AgF, Hg2F2 etc. This
reaction is called Swarts reaction.
H H H CH3
| | | | CH 3Br AgF CH 3 F AgBr
CH 3 – C C CH 3 CH3 C C C
| | | 2CH3CH 2 Cl Hg 2 F2
*
H3C C H H H C C H
|
3
|
2CH3CH 2 F Hg 2Cl2
H C H CH3
| Type B : Assertion Reason Questions
CH3
31. (a) Nitro group is electron-withdrawing, hence
(a) (b) it stabilises the carbanion by dispersing the
From the above structure, it is evident than negative charge on the carbon. So both the
structure (a) will show cis-trans isomerism assertion as well as reason are true and
Haloalkanes and Haloarenes C-151

reason is correct explanation of the 35. (c) Assertion is true, because aryl halides do
assertion. not undergo nucleophilic substitution under
32. (c) Chloral reacts with phenyl chloride to form ordinary conditions. This is due to
D.D.T. in presence of conc. H2SO4. A water resonance, because of which the carbon–
molecule eliminates in this reaction. halogen bond acquires partial double bond
character, hence, it becomes shorter and
stronger and thus, cannot be replaced by
H Cl nucleophiles. However, Reason is false.
36. (b) As compared to chlorobenzene, the
CCl3 – C O intermediate carbanion resulting from
4-nitrochlorobenzene is stabilized by –R-
H H Cl effect of the NO2 group.
37. (c) Dichlorocarbene (:CCl 2) attacks on the
ortho-position of the phenolate ion to form
CCl3 an intermediate which on hydrolysis gives
salicylic acid.
CH
H2SO4
38. (c) Aryl halides (chlorobenzene) do not
undergo nucleophilic substitution with KCN
–H2O because of the low reactivity of the Cl atom,
which is because of resonance in
Cl Cl
chlorobenzene. So assertion is true. Reason
D.DT
is false.
39. (c)
Hence, assertion is true but reason is false. 40. (c) CHCl3 is stored in dark bottles to prevent
33. (c) Alkyl halides on treatment with NaI in oxidation of CHCl3 in presence of sunlight.
presence of acetone forms alkyl iodide. This 41. (c) Alkyl halides give polyalkylation products.
is called Finkelstein reaction. 42. (d) Assertion is false, because aryl halides do
not undergo nucleophilic substitution
R – X + NaI ¾¾ ¾¾ ® R – I + NaX
acetone
under ordinary conditions. This is due to
Here NaI is soluble in acetone but NaBr/ resonance, because of which the carbon–
NaCl are not soluble. Hence due to chlorine bond acquires partial double bond
precipitation of salt, equilibrium is shifted character, hence it becomes shorter and
to forward direction. Hence assertion is true stronger and thus cannot be replaced by
but reason is false. nucleophiles. Also, Reason is false because
34. (b) Both assertion and reason are true and reason SN2 reactions proceeds with inversion of
is not the correct explanation of assertion . configuration.
EBD_7100
C-152 Topicwise AIIMS Solved Papers – CHEMISTRY

25 Alcohols, Phenols and Ethers

TYPE A : MULTIPLE CHOICE QUESTIONS (a) KMnO4


(b) K2Cr2O7
1. The product obtained from the reaction is: (c) CrO3
[1998] (d) PCC (pyridine chlorochromate)
OH
( i) NaOH 9. The major product formed in the following
(ii ) CO 2
reaction is : [2005]
(iii ) Hydrolysis CH 3
|
(a) Benzene (b) Toluene CH 3O –
CH 3— C — CH 2 Br
(c) Salicylic acid (d) Benzoic acid | CH 3OH
H
2. Picric acid is: [2000]
CH3
(a) Trinitrophenol (b) Trinitrotoluene |
(c) Trinitrobenzene (d) Tribromobenzene (a) CH 3—C — CH 2 OCH 3
|
3. Lucas reagent is: [2000] H
(a) anhy. AlCl3 + conc. HCl (b) CH 3— C H — CH 2 CH 3
(b) anhy. AlCl3 + conc.HNO3 |
(c) anhy. ZnCl2 OCH3
(d) anhy. ZnCl2 + conc. HCl CH 3
|
4. Lucas test is used for the detection of [2002] (c) CH 3 — C CH 2
(a) alcohols (b) alkyl halides
CH3
(c) phenols (d) aldehydes |
5. Intermolecular hydrogen bonding is strongest (d) CH3 — C — CH3
|
in : [2003]
OCH3
(a) Methylamine (b) Phenol
10. The major product obtained on the
(c) Formaldehyde (d) Methanol
monobromination (with Br 2 /FeBr 3 ) of the
6. Propan-1-ol can be prepared from propene by : following compound A is : [2006]
[2003]
OCH3
(a) H2O / H2SO4
(b) Hg(OAc)2 / H2O followed by NaBH4
(c) B2H6 followed by H2O2 CH 3
(d) CH3CO2H/H2SO4 A
7. Among the following the one which reacts most OCH3 OCH3
readily with ethanol is [2004] Br
(a) p-nitrobenzyl bromide (a) (b)
(b) p-chlorobenzyl bromide CH 3 CH 3
(c) p-methoxybenzyl bromide Br
(d) p-methylbenzyl bromide OCH 3 OCH 3
8. The most suitable reagent for the conversion of Br
(c) (d)
RCH2OH RCHO is : [2004]
Br CH 3 CH 3
Alcohols, Phenols and Ethers C-153

11. CH3OC2H5 and (CH3)3 C – OCH3 are treated with 17. Ethanol can be prepared more easily by which
hydroiodic acid. The fragments obtained after reaction ? [2011]
reactions are [2007] (i) CH 3 CH 2 Br H 2 O CH 3 CH 2 OH
(a) CH3I + HOC2H5; (CH3)3 Cl + HOCH3
(ii) CH3CH 2 Br Ag 2O (in boiling water)
(b) CH3OH + C2H5I; (CH3)3 Cl + HOCH3
(c) CH3OH + C2H5I; (CH3)3COH + CH3I CH3CH 2OH
(d) CH3I + HOC2H5; CH3I + (CH3)3 COH. (a) by (i) reaction
(b) by (ii) reaction
12. In which of the following reactions the product
(c) Both reactions proceed at same rate
obtained is t-butyl methyl ether ? [2008]
(d) by none
conc.H 2SO 4 18. An aromatic ether is not cleaved by HI even at
(a) CH 3OH HO CH 2 CH 3
CH3 525 K. The compound is [2012]
| (a) C6H5OCH3 (b) C6H5OC6H5
HO Na
(b) CH3 – C – Br CH3OH (c) C6H5OC3H7 (d) Tetrahydrofuran
|
CH3 19. The product of the following reaction is
CH3 (i ) BH3 / THF
| [2013]
(ii ) H 2 O2 , OH
(c) CH3 Br Na O – C CH 3
| (a) 1-Pentanol (b) 2-Pentanol
CH3 (c) Pentane (d) 1,2-Pentanediol
CH3 20. Ethanol when reacted with PCl5 gives A, POCl3
| and HCl. A reacts with silver nitrite to form B
(d) CH 3 – O Na CH 3 C Br
| (major product) and AgCl. A and B respectively are
CH3 (a) C2H5Cl and C2H5OC2H5 [2013]
13. Which of the following is a primary halide? (b) C2H6 and C2H5OC2H5
(a) Iso-propyl iodide [2008] (c) C2H5Cl and C2H5NO2
(b) Secondary butyl iodide (d) C2H6 and C2H5NO2
(c) Tertiary butyl bromide 21. Polyvinylalcohol can be prepared by [2013]
(d) Neohexyl chloride (a) polymerization of vinyl alcohol
14. An organic compound X on treatment with (b) alkaline hydrolysis of polyvinyl acetate
pyridinium chlorochromate in dichloromethane (c) polymerization of acetylene
gives compound Y. Compound Y reacts with I2 (d) reaction of acetylene with H2 SO 4 in
and alkali to form triiodomethane. The compound presence of HgSO4
'X' is [2008] 22. Compound 'A' of molecular formula C4H10O on
(a) C2H5OH (b) CH3CHO
treatment with Lucas reagent at room temperature
(c) CH3COCH3 (d) CH3COOH
15. The formation of diethyl ether from ethanol is gives compound ‘B’. When compound ‘B’ is
based on : [2009] heated with alcoholic KOH, it gives isobutene.
(a) Dehydrogenation reaction Compound ‘A’ and ‘B’ are respectively [2014]
(b) Hydrogenation reaction (a) 2-methyl-2-propanol and 2-methyl-2-
(c) Dehydration reaction chloropropane
(d) Heterolytic fission reaction (b) 2-methyl-1 -propanol and 1-chloro-2-
16. Chloropicrin is obtained by the reaction of methylpropane
(a) steam on carbon tetrachloride [2010]
(b) nitric acid on chlorobenzene (c) 2-methyl-1 -propanol and 2-methyl-2-
(c) chlorine on picric acid chloropropane
(d) nitric acid on chloroform (d) butan-2-ol and 2-chlorobutane
EBD_7100
C-154 Topicwise AIIMS Solved Papers – CHEMISTRY
23. Which of the following alcohols gives the best 30. In the Victor-Meyer’s test, the colour given by
yield of dialkyl ether on being heated with a trace 1°, 2° and 3° alcohols are respectively. [2017]
of sulphuric acid? [2014] (a) red, colourless, blue
(a) 2-Pentanol (b) red, blue, colourless
(c) blue, red, violet
(b) Cyclopentanol
(d) red, blue, violet
(c) 2-Methyl-2-butanol
(d) 1-Pentanol TYPE B : ASSERTION REASON QUESTIONS
24. Which of the following reagents convert Directions for (Qs. 31-35) : These questions consist
propene to 1-propanol? [2015] of two statements, each printed as Assertion and
(a) H2O, H2SO4 Reason. While answering these questions, you are
(b) Aqueous KOH required to choose any one of the following five
(c) MgSO4, NaBH4/H2O responses.
(d) B2H6, H2O2, OH– (a) If both Assertion and Reason are correct and
25. Which of the following fact(s) explain(s) as to the Reason is a correct explanation of the
why p-nitrophenol is more acidic than phenol? Assertion.
I. –I Effect of nitro group. [2015] (b) If both Assertion and Reason are correct but
Reason is not a correct explanation of the
II. Greater resonance effect of p-nitrophenoxy
Assertion.
group
(c) If the Assertion is correct but Reason is incorrect.
III. Steric effect of bulky nitro group
(d) If both the Assertion and Reason are incorrect.
(a) I and II (b) I and III
(e) If the Assertion is incorrect but the Reason is
(c) II and III (d) II alone
correct.
26. ClCH2CH2OH is stronger acid than CH3CH2OH
31. Assertion : Resorcinol turns FeCl3 solution purple.
because of: [2016]
Reason : Resorcinol have phenolic group.
(a) – I effect of Cl increases negative charge [2000]
on O atom of alcohol 32. Assertion : Phenol is a strong acid than ethanol.
(b) – I effect of Cl disperses negative charge Reason : Groups with +M effect decreases
on O atom to produce more stable cation acidity at p-position. [2002]
(c) – I effect of Cl disperses negative charge 33. Assertion : Benzyl bromide when kept in acetone
on O atom to produce more stable anion water, it produces benzyl alcohol.
(d) None of these Reason : The reaction follows SN2 mechanism.
[2003]
27. The ether that undergoes electrophilic
34. Assertion : The major products formed by
substitution reactions is [2017]
heating C6H5CH2OCH3 with HI are C6H5CH2I
(a) CH3OC2H5 (b) C6H5OCH3 and CH3OH.
(c) CH3OCH3 (d) C2H5OC2H5 Reason : Benzyl cation is more stable than methyl
28. A triglyceride can have how many different acyl cation. [2004]
groups? [2017] 35. Assertion : The ease of dehydration of the
(a) 3 (b) 2 following alcohols is
(c) 1 (d) 4 OH OH OH OH
29. In the reaction
Phenol NaOH (A) CO1402 HCl (B), here B < < <
is [2017]
(a) benzaldehyde (b) chlorobenzene Reason : Alcohols leading to conjugated alkenes
(c) benzoic acid (d) salicylic acid are dehydrated to a greater extent. [2008]
Alcohols, Phenols and Ethers C-155

Directions for (Qs.36-40) : Each of these questions 37. Assertion : ter-Butyl methyl ether is not
contains an Assertion followed by Reason. Read them prepared by the reaction of ter-butyl bromide
carefully and answer the question on the basis of with sodium methoxide.
following options. You have to select the one that Reason : Sodium methoxide is a strong
best describes the two statements. nucleophile. [2010]
(a) If both Assertion and Reason are correct and 38. Assertion : Ethers behave as bases in the
Reason is the correct explanation of Assertion. presence of mineral acids.
(b) If both Assertion and Reason are correct, but Reason : Due to the presence of lone pairs of
Reason is not the correct explanation of electrons on oxygen. [2013]
Assertion. 39. Assertion : Phenol undergo Kolbe reaction,
(c) If Assertion is correct but Reason is incorrect. ethanol does not. [2014, 2015]
(d) If both the Assertion and Reason are incorrect. Reason : Phenoxide ion is more basic than
36. Assertion: Phenyl is used as a household ethoxide ion.
germicide. 40. Assertion : Ethyl phenyl ether on reaction with
Reason : Phenyl is phenol derivative and phenol HBr form phenol and ethyl bromide.
is an effective germicide. [2010] Reason : Cleavage of C–O bond takes place on
ethyl-oxygen bond due to the more stable
phenyl-oxygen bond. [2016]
EBD_7100
C-156 Topicwise AIIMS Solved Papers – CHEMISTRY

Type A : Multiple Choice Questions 9. (a) Among the given compounds,

2. (c) OH OH CH 2 Br CH 2 Br CH 2 Br CH 2 Br
COONa
NaOH
CO2
OH NO 2 Cl OCH 3 CH 3
COOH
hydrolysis
the first one can give carbonium ion most
readily because the carbonium ion will be
Salicylic acid stabilised due to presence of NO2 group
4. (a) Picric acid is sym-trinitrophenol on the ring.
OH 10. (d) The most suitable reagent for converting
O2N NO2 alcohol to acetaldehyde is PCC. Other
reagent will convert alcohol to acid.
11. (d) The reaction is an example of SN1 reaction
CH3 CH3
NO2 | |
– Br –
5. (d) Lucas reagent is anhydrous ZnCl2 + HCl CH 3 — CH — CH 2 Br CH3 — C H — C H 2
1 carbocation
which is used to distinguish between
primary, secondary and tertiary alcohols. CH3
|
6. (a) Lucas test is used for the determination of hydrideshift
CH3 — C — CH3
primary, secondary and tertiary alcohols. 3 carbocation
7. (d) Hydrogen bondin g is formed in CH3
compounds in which H is attached to – |
OCH3
highly electronegative element like F, O and CH3 — C — CH3
|
N.
OCH3
O 12. (b) The reaction is an example of electrophilic
||
aromatic substitution. Although both OCH3
(i) In H — C— H , O is not having H atom
and CH3 groups are o,p-directing, the OCH3
so it shows very little H-bonding. group dominates. Product (b) is favoured
(ii) N is less electronegative than O, so H— because the new coming group (Br)
bond formed by amines will be weak than experiences least hindrance.
that by alcohols. 13. (a) In case of unsymmetrical ethers, the site of
(iii) C6H5OH forms weak H-bonding due to cleavage depends on the nature of alkyl
steric hinderance due to bulky phenyl group. group.
B2 H 6 e.g. CH3 – O – C2H5 + HI ¾¾ ¾®
373K
8. (c) CH 3CH CH 2 (CH3 CH 2 CH 2 )3 B
CH3I + C2H5OH
The alkyl halide is formed from the smaller
H 2O 2 /OH – alkyl group.
CH3CH 2 CH 2 OH
Propanol
Alcohols, Phenols and Ethers C-157

However, in case of tertiary alkyl ether of X which must be an alcohol (CH3CH2OH)


following reaction occurs. and thus Y is CH3CHO.
17. (c) Ethanol on dehydration forms diethyl ether.
CH 3 OC(CH3 )3 +HI ¾¾¾
373K
®
ter -butyl methyl ether C2H5O H + HO C2H5
(CH3)3C – I + CH3OH C2H5OC2H5 + H2O
The alkyl halide is formed from the tertiary 18. (d) Chloropicrin is nitrochloroform. It is
alkyl group and the cleavage of such ethers
obtained by the nitration of chloroform with
occurs by SN1 mechanism as the product is
HNO3.
controlled by the formation of more stable
intermediate tertiary carbocation from HNO3
HCCl3 O2 NCCl3
protonated ether. Chloroform Chloropicrin
14. (c) It is Williamson's synthesis. It involves the
nucleophilic attack of alkoxide ion on alkyl Chloropicrin is a liquid, poisonous and used
halide according to SN2 mechanism. In order as an insecticide and in war gas.
to prepare methyl tertiary butyl ether, we must 19. (b) Heavy metal ions, particularly Ag + ,
use methyl halide (primary) and sodium catalyse SN1 reaction because of presence
tertiary butoxide, but not sodium ethoxide of empty orbital.
an d t-alkyl halide because the latter +
undergoes elimination reaction rather than CH3CH 2Br + Ag
substitution. +
[CH3CH2 .... Br .... Ag]
15. (d) Writing the structures, we get

slow +
I CH3CH 2 OH CH3CH 2OH
| (–AgBr) from H 2O
(a) CH3 C H , Isopropyl iodide (2°)
| 20. (b) Due to greater electronegativity of
CH3
sp2-hybridized carbon atoms of the benzene
H H H ring, diaryl ethers are not attacked by
| | |
nucleophiles like I–.
(b) H C C C I , sec - Butyl iodide (2°)
| | | 21. (a) Hydroboration-oxidation leads to anti-
H H CH3
Markownikoff’s hydration, thus
CH3
(i) BH 3 /THF
|
(c) H3C C I , tert- Butyl iodide (3°) (ii) H 2O/OH
| 1-Pentanol
CH3
AgNO 2 PCl 5
22. (c) C 2 H 5 NO 2 C 2 H 5 OH
CH3 H B
| |
(d) CH3 CH 2 C C Cl , C 2 H 5 Cl POCl3 HCl
| | [A]
CH3 H 23. (b) Vinyl alcohol, CH2=CHOH, monomer of
Neohexyl chloride (1°) polyvinyl alcohol exists mainly as
Thus, (d) is a primary (1°) halide. CH3CHO; hence polyvinyl alcohol is best
PCC I 2 / alkali prepared by the alkaline hydrolysis of
16. (a) X Y CHI3
polyvinyl acetate which in turn is prepared
Compound Y must give iodoform test. by the polymerisation of vinyl acetate.
Further since Y is obtained by the oxidation
EBD_7100
C-158 Topicwise AIIMS Solved Papers – CHEMISTRY
24. (a) Reaction involved is given as : 32. (b)
CH3 CH 3 Type B : Assertion Reason Questions
HCl
CH 3 —C—OH CH 3 —C—Cl 33. (a) Phenols turn FeCl 3 solution purple
ZnCl 2
Resorcinol has phenolic group.
CH3 CH 3
(A) (B) 34. (b) Phenol is strong acid than ethanol because
phenoxide ion is resonance stabilised.
CH3 When a group having +M effect is at p-
alc. KOH position (like halide group) it decreases the
CH3 —C
acidity of phenols. So assertion and reason
CH 2 are correct but reason is not the explanation
of assertion. So correct option is (b).
25. (d) 1-pentanol
35. (c) The reaction follows SN1 mechanism for
Primary alcohols readily form ether when
two reasons :
heated with conc. H2SO4.
C 6 H 5 CH 2 Br C 6 H 5 CH 2 Br
B 2H 6
26. (d) 6 CH3 CH CH 2 1. C6H5CH2+ is stablised by resonance.
Propene ether, 0°C
2. Steric hinderance of phenyl group.
H 2O 2
2(CH3CH 2 CH 2 )3B 36. (a) As benzyl cation (C 6 H 5 CH 2 ) is more
OH
stable than methyl cation (CH 3 ) , so the
6CH3CH 2 CH 2 OH 2H 3 BO 3
Propanol product is C 6 H 5CH 2 I and CH3OH
27. (a) 37. (a) The given order of dehydration is correct
28. (c) ClCH 2 CH 2 OH is stronger acid than due to following reasons.
CH3CH 2 OH due to – I effect of Cl.
(i) Alcohols leading to conjugated alkenes are
dehydrated more readily.
Cl CH 2 CH 2 OH Cl CH 2 CH 2 O H (ii) 2-Cyclohexenol is dehydrated more easily
Stronger acid ve charge on O than 3-cyclohexenol because the carbocation
dispersed hence
conjugate base, stable from the former is more stable than the latter.
CH3CH 2OH CH3 CH 2O H OH
Weaker acid ve charge intensified,
hence conjugate +
base unstable
29. (b)
30. (a) Since glycerol has three –OH groups, it can (more stable)
have three acyl (similar or different) groups OH
OH ONa +

NaOH
31. (d)
(less stable)
Phenol (iii) Phenol is not dehydrated due to its highly
OH
stable character due to resonance.
COOH 38. (a) Phenol is an effective germicide. Phenyl is a
CO 2 + HCl
140°C derivative of phenol, that’s why it also has
germicidal property.
Salicylic acid
Alcohols, Phenols and Ethers C-159

39. (b) On using ter-butyl bromide and sodium sodium salicylate. This is known as Kolbe’s
ethoxide as reactants, the major product reaction. Ethanol does not respond to this
would be 2-methylpropene and ethanol reaction. Therefore, Assertion is true. But
(elimination reaction). the Reason that phenoxide ion is more basic
than ethoxide ion is not correct.
CH3 CH3
| |
42. (c) Alkyl aryl ethers are cleaved at the alkyl-
CH3 — C — Br CH3ONa CH 3 — C CH 2 oxygen bond due to the more stable aryl-
| oxygen bond. The reaction yields phenol
CH3
and alkyl halide
40. (a) Ethers have lone pair on oxygen, so they
O–R OH
behave as Lewis base. It forms salt known
as oxonium salt with acids.
R +H–X +R–X
R + – + –
O : + H Cl O H Cl
R R Ethers with two different alkyl groups are
also cleaved in the same manner.
41. (c) It is correct that sodium phenoxide (sodium
salt of phenol) and CO2 on heating form R O R ' HX R X R ' OH
EBD_7100
C-160 Topicwise AIIMS Solved Papers – CHEMISTRY

Aldehydes, Ketones and


26 Carboxylic Acids
TYPE A : MULTIPLE CHOICE QUESTIONS 8. Which produces ketone on treatment with
Grignard reagent ? [1999]
1. Reduction of benzoyl chloride with Pd and (a) methyl cyanide (b) acetaldehyde
BaSO 4 gives : [1997] (c) methyl alcohol (d) acetic acid
(a) benzyl chloride 9. Which of the following compound is formed
(b) benzaldehyde when CH2 = CH(CH2)2COOH reacts with HBr?
(c) benzoic acid (a) CH3CH 2CH 2 CH 2 BrCOOH [2000]
(d) benzene sulphonyl chloride
2. Acetaldehyde does not respond to [1997] (b) CH 3CHBrCH 2CH 2 COOH
(a) Tollen’s test (b) Benedict’s test (c) CH 2 BrCH 2 (CH 2 ) 2 COOH
(c) Lucas test (d) Iodoform test
(d) CH3CH 2CH 2 BrCH 2COOH
C 2 H 5ONa
3. 2CH 3COOC 2 H 5 10. Ethyl alcohol reacts with chlorine to produce :
CH 3COCH 2COOC 2 H 5 [1997] [2000]
The above reaction is known as : (a) CH 3CH 2Cl (b) CH 2ClCH 2OH
(a) Claisen condensation
(b) Perkin reaction (c) CHCl 2CH 2OH (d) CCl 3CHO
(c) Schotten-Baumann reaction 11. Aldol condensation does not take place in:
(d) Curtius reaction [2000]
4. In a ring substitution of C6H5Z, the main product (a) HCHO (b) CH 3CHO
obtained is meta, the group Z is [1997]
(a) —COOH (b) —Cl (c) CH 3CH 2CHO (d) CH 3COCH 3
(c) —NH2 (d) —CH3 12. Acetic acid on heating with P2 O5 produces :
5. Acetate ion contains: [1998]
[2000]
(a) One C – O bond and one C = O bond
(b) Two C = O bonds (a) CH 3COCH 3 (b) CH 3CHO
(c) Two C – O bonds (c) CH 3COCH 2CH 3 (d) (CH 3CO) 2 O
(d) Two C = O bonds and one C – O bond
13. Salol is : [2001]
6. NaOH CO 2 H (a) acetylsalicylic acid
Phenol X Y Z,
Z is identified as: [1999] (b) phenyl salicylate
(a) Benzoic acid (b) Benzaldehyde (c) methyl salicylate
(c) Sodium benzoate (d) Salicylic acid (d) none of the above
7. Benzaldehyde can be prepared by the hydrolysis 14. Aldehydes and ketones can be distinguished
of : [1999] by:
(a) benzonitrile (b) benzotrichloride (a) Ammonia (b) H2SO4 [2001]
(c) benzyl chloride (d) benzal chloride (c) Alkaline KMnO4 (d) Fehling solution
Aldehydes, Ketones and Carboxylic Acids C-161

COCl CHO CH3


CH3
CO2H
15. H2
Pd / BaSO4 CO2H
+ HCl
(c) (d)

The above reaction is [2001] Br Br


(a) Clemmensen reduction 24. CH3CO2C2H5 on reaction with sodium ethoxide
(b) Rosenmund reduction in ethanol gives A, which on heating in the
(c) Birch reduction presence of acid gives B. Compund B is : [2005]
(d) Walf-Kishner reduction (a) CH3COCH2COOH
16. The intermediate formed in aldol condensation (b) CH3COCH3
is [2002] O
(a) aldol (b) carbanion (c) CH2 O
(c) alcohol (d) -hydrogen ester
OC2H5
17. The compound most suitable for the preparation
(d) CH2 =C
of cyanohydrin is [2002] OC2H5
(a) C2H5COOH (b) C6H5NH2
25. Isopropylbenzene on air oxidation in the
(c) C2H5COC2H5 (d) C2H5 – C2H5
presence of dilute acid gives: [2006]
18. Action of acetylene on dilute H2SO4 gives
[2002] (a) C 6 H 5 COOH (b) C 6 H 5 COCH 3
(a) acetic acid (b) acetaldehyde (c) C 6 H 5 CHO (d) C 6 H 5OH
(c) acetone (d) acetoacetic ester
19. CH3COCH3 can be converted to CH3CH2CH3 by O COCH3 AlCl3
the action of [2002] 26. [2007]
(a) HIO3 (b) HNO3
The product obtained is/are
(c) HI (d) H3PO3 (a) o-product
20. Among the following the strongest acid is (b) m-product
(a) CH3COOH [2003] (c) o- and p-products
(b) C6H5COOH (d) o-, m- and p-products
(c) m-CH3OC6H4COOH 27. Benzoic acid is treated with lithium aluminium
(d) p-CH3OC6H4COOH hydride. The compound obtained is [2007]
21. At higher temperature, iodoform reaction is (a) benzaldehyde (b) benzyl alcohol
given by the dilute solution of [2003] (c) toluene (d) benzene.
(a) CH3 CO2 CH3 (b) CH3 CO2 C2H5 28. Maximum enol content is in [2008]
(c) CH3 CO2 C6H5 (d) C2H5 CO2 CH3
O O O
22. Th e reagent used for the separ ation of (a) (b)
acetaldehyde from acetophenone is [2004]
(a) NaHSO3 (b) C6H5NHNH2
O O O
(c) NH2OH (d) NaOH + I2
(c) (d)
23. o-Toluic acid on reaction with Br2 + Fe gives H H
[2004]
X
CH2Br CH 3
29. CH 3CH 2 C N CH 3CH 2 CHO
CO2H CO2H The compound X is [2008]
(a) (b) (a) SnCl2/HCl/H2O, boil
Br (b) H2/Pd –BaSO4
(c) LiAlH4/ether
(d) NaBH4/ ether / H3O+
EBD_7100
C-162 Topicwise AIIMS Solved Papers – CHEMISTRY
30. 3-Hydroxybutanal is formed when X reacts with
H3C
Y in dilute Z solution. What are X, Y and Z ? C–OH
(c) H3C
[2008]
X Y Z CH3–CHOH
(a) CH3CHO (CH3)2CO NaOH (d) No reaction
(b) CH3CHO CH3CHO, NaCl
34. (CH 3 ) 3 C—CHO does not undergo aldol
(c) (CH3)2CO, (CH3)2CO, HCl
condensation due to [2010]
(d) CH3CHO, CH3CHO, NaOH
31. Acid hydrolysis of which of the following (a) three electron donating methyl groups
compounds yields two different organic (b) cleavage taking place between —C— CHO
compounds? [2008] bond
(a) CH3COOH (b) CH3CONH2 (c) absence of alpha hydrogen atom in the
(c) CH3COOC2H5 (d) (CH3CO)2O molecule
32. In the Cannizzaro reaction the intermediate that (d) bulky (CH3)3 C—group
will be the best hydride donor is [2009]
H – 35. Which of the following undergoes haloform
O reaction ? [2011]
(i) CH3CH2COCH2Cl (ii) C6H5COCH3
(a) OH
(iii) C6H5COCHCl2 (iv) CH3CH2COCCl3
H – (a) only (ii) (b) (ii) and (iv)
O
(c) (i), (ii) and (iv) (d) all the four

(b) O
36. NaCN dil. H 2SO 4
(CH 3 ) 3 CCl A B.
H Compound B is [2011]

O

(a) (CH3)3CCOOH (b) (CH3)3COH
(c) O (c) (CH3)3COC(CH3)3 (d) All the three
37. When ethane –1, 2-dioic acid is heated with
MeO conc. H2SO4, it gives [2011]
H
– (a) CO + HCOOH (b) CO2 + HCOOH
O
– (c) CO+CO2+HCOOH (d) CO+CO2+H2O
(d) O
38. Which of the following statement is not true ?
[2011]
O2N
33. Identify the product for the following reaction (a) At room temperature, formyl chloride is
present in the form of CO and HCl
O (b) Acetamide behaves as a weak base as well
CH2OH HCl
CH3–C–CH3 + ? as a weak acid.
CH2OH
(c) LiAlH 4
CH 3 CONH 2 CH 3CH 2 NH 2
CH 3
COOH (d) None of the three.
(a) CH 3–CHOH + [2009] 39. Cannizzaro reaction between formaldehyde
COOH
molecules involves the formation of [2011]
H3C O–CH2 OH O
(b) C | |
H3C O–CH2 H C H H C H
(a) (b) |
|
O O
(c) Both (a) and (b) (d) none
Aldehydes, Ketones and Carboxylic Acids C-163

40. Carbonyl compounds undergo nucleophilic 45. The final product (III) obtained in the reaction
addition because of [2012] sequence –
(a) electronegativity difference of carbon and CH 3 CH 2 COOH
PCl3
I
oxygen atoms
C6 H 6 /AlCl3 NH 2 NH 2
(b) electromeric effect II III
base/heat
(c) more stable anion with negative charge on [2014]
oxygen atom and less stable carbonium ion
(d) none of the above CH2– CH – CH3
(a) 2
41. Which of the following is correct order of acidity?
[2012]
(a) HCOOH > CH3COOH > ClCH2COOH >
C2H5 COOH (b) CH – CH2– CH3
(b) ClCH2COOH > HCOOH > CH3COOH >
OH
C2H5COOH
(c) CH3COOH > HCOOH > ClCH2COOH > O
C2H5COOH ||
(c) C – CH
O 2– CH3
(d) C2H5COOH > CH3COOH > HCOOH >
ClCH2COOH
42. Which is not true about acetophenone ? [2012] O
(a) It reacts with 2,4-dinitrophenylhydrazine ||
(d) C – CH2– CH3
to form 2, 4-dinitrophenylhydrazone
(b) It reacts with Tollen’s reagent to form silver
mirror CHO
(c) It reacts with I2/NaOH to form iodoform
(d) On oxidation with alkaline KMnO4 followed Dil. NaOH
46. + CH3CHO A .
by hydrolysis it gives benzoic acid (Major)
43. Which of the following products is formed when
benzaldehyde is treated with CH3MgBr and the (A) will be – [2014]
addition product so obtained is subjected to OH
acid hydrolysis ? [2013] |
(a) A secondary alcohol (a) C6 H5 CH CH 2CHO
(b) A primary alcohol
(b) C6H5CH = CH – CHO
(c) Phenol
(d) tert-Butyl alcohol (c) C6H5CH2CH2CHO
44. Ethanoic acid on heating with ammonia forms (d) Both (b) & (c)
compound A which on treatment with bromine 47. Which of the following represents the correct
and sodium hydroxide gives compound B. order of the acidity in the given compounds?
Compound B on treatment with NaNO2/dil. HCl [2015]
gives compound C. The compounds A, B and C (a) FCH2COOH > CH3COOH > BrCH2COOH >
respectively are [2014] ClCH2COOH
(a) ethanamide, methanamine, methanol (b) BrCH2COOH > ClCH2COOH > FCH2COOH
(b) propanamide, ethanamine, ethanol > CH3COOH
(c) N-ethylpropanamide, methaneisonitrile, (c) FCH2COOH > ClCH2COOH > BrCH2COOH
methanamine > CH3COOH
(d) ethanamine, bromoethane, ethanedia- (d) CH3 COOH > BrCH2COOH > ClCH2COOH
zonium chloride > FCH2COOH
EBD_7100
C-164 Topicwise AIIMS Solved Papers – CHEMISTRY
48. An ester (A) with molecular fomula, C9H10O2 (a) Reimer- Tiemann reaction
was treated with excess of CH3MgBr and the (b) Hell-volhard Zelinsky reaction
complex so formed was treated with H2SO4 to (c) Cannizzaro reaction
give an olefin (B). Ozonolysis of (B) gave a ketone (d) Sandmeyer reaction
with molecular formula C8H8O which shows
positive iodoform test. The structure of (A) is TYPE B : ASSERTION REASON QUESTIONS
[2015] Directions for (Qs. 53-58) : These questions consist
(a) C6H5COOC2H5 of two statements, each printed as Assertion and
(b) C2H5COOC6H5 Reason. While answering these questions, you are
(c) H3COCH2COC6H5 required to choose any one of the following five
(d) p — H3CO — C6H4 — COCH3 responses.
49. In a set of reactions m-bromobenzoic acid gave (a) If both Assertion and Reason are correct and
a product D. Identify the product D. [2015] the Reason is a correct explanation of the
Assertion.
COOH
(b) If both Assertion and Reason are correct but
SOCl2 NH3 NaOH Reason is not a correct explanation of the
B C Br2
D
Assertion.
Br
(c) If the Assertion is correct but Reason is incorrect.
SO2NH2 COOH (d) If both the Assertion and Reason are incorrect.
(a) (b) (e) If the Assertion is incorrect but the Reason is
correct.
Br NH2 53. Assertion : Acetaldehyde on treatment with
alkali gives aldol.
NH2 CONH2 Reason : Acetaldehyde molecule contains
(c) (d) -hydrogen atom. [1997]
54. Assertion : Acetylene on treatment with alkaline
Br Br KMnO4 produces acetaldehyde.
50. Aldeh ydes th at do not under go aldol Reason : Alkaline KMnO4 is a reducing agent.
condensation are [2000]
1. propanal 2. trichloroethanal 55. Assertion : Hydroxyketones are not directly
3. methanal 4. ethanal used in Grignard reaction.
5. benzaldehyde Reason : Grignard reagents react with hydroxyl
[2016] group. [2003]
(a) 3 and 4 only (b) 3 and 5 only 56. Assertion : Isobutanal does not give iodoform
(c) 1, 2 and 3 only (d) 2, 3 and 5 only test.
51. Aldol condensation will not be observed in Reason : It does not have -hydrogen [2004]
(a) chloral [2017] 57. Assertion : The pKa of acetic acid is lower than
(b) phenylacetaldehyde that of phenol.
(c) hexanal Reason : Phenoxide ion is more resonance
stabilised. [2004]
(d) nitromethane
52. The reaction 58. Assertion : Acetamide has more polar C = O

Red P
group than ethyl acetoacetate.
RCH 2 CH 2 COOH R CH 2 CH COOH ..
Br2 | Reason : NH2 is more electron donating than
Br ..
OC 2 H5. [2007]
is called as [2013, 2017]
Aldehydes, Ketones and Carboxylic Acids C-165

Directions for (Qs.59-69) : Each of these questions 64. Assertion: CH 3 COCl is converted to
contains an Assertion followed by Reason. Read them CH3CONH2 on reaction with NH3.
carefully and answer the question on the basis of Reason : Cl is a stronger nucleophile and better
following options. You have to select the one that leaving group. [2011]
best describes the two statements. 65. Assertion : Aldol condensation can be catalysed
both by acids and bases.
(a) If both Assertion and Reason are correct and Reason : -Hydroxyaldehydes or ketones readily
Reason is the correct explanation of Assertion. undergo acid-catalysed dehydration. [2011]
(b) If both Assertion and Reason are correct, but 66. Assertion : 2, 2-Dimethylpropanal undergoes
Reason is not the correct explanation of Cannizzaro reaction with conc. NaOH.
Assertion. Reason : Cannizzaro reaction is a
(c) If Assertion is correct but Reason is incorrect. disproportionation reaction. [2012]
(d) If both the Assertion and Reason are incorrect. 67. Assertion : Acetoacetic ester,
59. Assertion : RCOCl, (RCO)2O and RCOOR’ all O O
react with Grignard reagents to form 3º alcohols.
Reason : RCOCl reacts with R2Cd to form ketones CH3 C CH2COC2H5 will give iodoform test
but (RCO)2O and RCOOR' do not react at all. O
[2009]
60. Assertion : Protonation of a carbonyl group Reason : It does not contains CH3 C group .
increases its electrophilic character. [2013]
Reason : Protonation of a carbonyl group 68. Assertion : Benzaldehyde is more reactive than
involves addition of an electrophile on ethanol towards nucleophilic attack.
nucleophilic oxygen. [2009] Reason : The overall effect of –I and + R effect
61. Assertion : Nitration of benzoic acid gives of phenyl group decreases the electron density
m-nitrobenzoic acid. on the carbon atom of > C = O group in
Reason : Carboxyl group increases the electron- benzaldehyde. [2014, 2015]
density at meta-position. [2009]
69. Assertion : The boiling points of aldehydes and
62. Assertion : -Keto carboxylic acids lose CO2
when heated at about 370 K. ketones are higher than hydrocarbons and
Reason : An enol is first formed by loss of CO2, ethers of comparable molecular masses.
but it readily tautomerises to the more stable Reason : There is a weak molecular association
ketone. [2010] in aldehydes and ketones arising out of the
63. Assertion : The acetate ion is resonance dipole-dipole interactions.
stabilized. [2016]
Reason : Acetate ion is more basic than the
methoxide ion. [2011]
EBD_7100
C-166 Topicwise AIIMS Solved Papers – CHEMISTRY

Type A : Multiple Choice Questions CH Cl CH2OH

1. (b) Pd / BaSO 4 H2 O
C 6 H 5 COCl C 6 H 5 CHO
This reaction is known as Rosenmund's
reduction. Benzyl chloride
2. (c) Lucas test is for primary, secondary and
tertiary alcohols and not for acetaldehyde CHCl2 CHO
3. (a)
H2 O
O
||
CH 3 C OC 2 H 5 H CH 2 COOC 2 H 5 Benzal chloride
O 8. (a)
|| R
CH3 C CH 2 COOC2 H5 |
Self condensation of ester takes place in CH 3 CN RMg X CH 3 C N Mg X
presence of strong base such as C2H5O–. R
The reaction is known as Claisen | X
2H 2O
condensation. CH3 C = O + Mg
4. (a) Group Z is meta directing. Among the given OH
groups, –COOH is the only meta directing HBr
9. (c) CH 2 CH CH 2 CH 2 COOH
group.
5. (a) Acetate ion (CH3COO–) has one C–O and CH 2 CH CH 2 CH 2 COOH
HBr
one C = O bond.
O CH 2 Br CH 2 CH 2 CH 2 COOH
||
So, the addition is anti-Markovnikov.
H3C – C– O –
10. (d) Ethyl alcohol reacts with chlorine to
OH ONa
produce CCl3CHO
CO2
6. (d) + NaOH C 2 H5 OH
Cl2
CH3CHO
3Cl2
CCl3CHO
Chloral

ONa OH 11. (a) Aldol condensation does not take place in


HCHO as it has no -carbon.
COOH H COOH 12. (d) Acetic anhydride is produced.
P2O 5
Salicylic acid
2CH 3COOH (CH3CO) 2 O
H 2O
CN COOH
13. (b) Salol is phenyl salicylate.
H2 O
7. (d) OH

Benzonitrile COOC6H 5
CCl3 COOH

H2 O
salol
It is used as intestinal antiseptic.
Benzotrichloride
Aldehydes, Ketones and Carboxylic Acids C-167

14. (d) Aldehydes and ketones can be Ethyl alcohol, so formed, will give iodoform
distinguished by Fehling solution. test.
Aldehydes can reduce it, whereas ketones C 2 H 5 COOCH 3 C 2 H 5 COOH CH 3OH
cannot reduce it.
Hence it will not give iodoform test.
15. (b) 22. (a) NaHSO3 does not react with acetophenone
16. (b) Aldol condensation is carried out on due to steric hindrance but it forms addition
aldehydes or ketones having -hydrogen product with acetaldehyde.
atom in presence of base which abstracts
23. (c) In o - toluic acid, –CH3 group is ortho-para
- H atom from aldehyde/ ketone to form directing
corresponding carbanion which is stabilised
by resonance. CH 3 CH 3
O COOH COOH
Fe
||
Br2
OH H CH 2 C H
Br
O O and –COOH group is meta-directing. So,
|| |
the resulting product will be (c) in which Br
H2O [CH 2 C H CH 2 C H]
is attached at para to –CH3 and meta to
17. (c) Ketones reacts with HCN to form addition –COOH group.
product, known as cyanohydrin compounds. O
||
C2 H 5 C2H 5 OH 24. (b) CH3COOC2 H5 CH 3 C OC 2 H 5
C O + HCN C
C2 H 5 C2 H 5 O
CN ||
C2 H5ONa
18. (b) Acetylene is hydrolysed by dil. H2SO4 to CH 3 C CH 2 COOC2 H 5
form acetaldehyde. (A)
19. (c) CH3COCH3 can be reduced by HI to alkanes.
H CH 3 COCH 2 COOH
HI is very strong reducing agent.
keto acid
CH3COCH3 + 2HI
O
CH3CH2CH3+ H2O + I2 ||
heat
20. (b) Benzoic acid is the strongest acid. Benzoate CH3 C CH3 CO2
ion is stablised by resonance. +I effect of (B)
–CH 3 or –OCH 3 group reduces the -Keto acids, on heating, undergo
possibility of ionisation of H+ from COOH. decarboxylation.
21. (b) Formation of iodoform is commonly used
CH 3 CH 3
as a test for alcohols having –CH(OH)CH3
groupin g, acetaldehyde and ketones 25. (d) CH 3 C H CH 3 C OOH
having at least one alkyl group as methyl O2 H
group, i.e. for detecting the presence of –
CHOHCH3;–COCH3 and CH3CHO. The
esters (as given in the options) are OH
O
hydrolysed into alcohols or acids. ||
H2O
+
CH 3COOC 2 H 5 CH 3 C CH 3
heat
CH 3COOH C 2 H 5 OH
EBD_7100
C-168 Topicwise AIIMS Solved Papers – CHEMISTRY
26. (c) The phenyl esters on treatment with This product is obtained when 2 molecules
AlCl3(anhy.) undergoes rearrangement to of CH3CHO are heated with concentrated
give o– and p– hydroxyketones (Fries alkali (NaOH)
rearrangement). NaOH
CH3CHO CH3 CHO
O–COCH3 'Z'
"X" "Y"
AlCl3
heat CH3C H CH 2 CHO
|
phenyl acetate OH
[Note ; The aldehydes having -hydrogen
OH atom on heating with concentrated alkali give
OH
brown resinous mass; aldol condensation]
COCH3
31. (c)
+
Acid hydrolysis
CH3COOC 2H5 CH 3COOH C 2H 5OH
ester acid alcohol
o–hydroxy
acetophenone COCH3 32. (a) In general, aldehydes containing no -H
p–hydroxy
atom undergo Cannizzaro's reaction. It
acetophenone
follows the familiar pattern of carbonyl
27. (b) When benzoic acid is treated with LiAlH4 compounds i.e. nucleophilic addition
it reduces the – COOH group of benzoic and involves two successive additions. In
acid to – CH2OH. step (i), a hydroxide ion is added to give
intermediate compound (a). The presence
COOH CH2OH of negative charge on compound (a) aids
in loss of hydride ion.
LiAlH4 + H2O
O
|| CH 2OH HCl
33. (b) CH3 C CH3 + |
28. (b) Maximum enol content is in structure (b) CH 2OH
because here the enol form is stabilised by
H-bonding. H3C O – CH2
O O O–H------O
|| || | || H3C O – CH2
CH 3 – C– CH 2 – C– CH 3 CH 3 – C CH – C– CH3
keto form enol form Cyclic ketal
(Acetylacetone)
34. (c) Aldol condensation is given by the
29. (a) Completing the given reaction
compounds which contain -hydrogen
CH3 CH 2 C N atom.
SnCl 2 / HCl 35. (d) If we observe the haloform reaction
CH 3CH 2 CH NH carefully, we see that –COCH3 group is first
H 2O,boil halogenated to the trihalo –COCX3 through
CH3CH 2 CHO NH 4Cl
(Ether) monohalogeno and dihalogeno compound.
It is Stephen's reduction. It is th e –COCX 3 part which then
30. (d) The product obtained is undergoes nucleophilic addition. The
product easily loses –CX3 since it is a very
CH3 CHCH 2 CHO
| . good leaving group.
OH
Aldehydes, Ketones and Carboxylic Acids C-169

O O 39. (c)
X2, OH
– 40. (c)
CH3 – C – CH3 CH3 – C – CH2Cl 41. (b) Recall that presence of electron-
I II
withdrawing group increases, while
presence of electron-releasing group
O O decreases the acidity of carboxylic acids.
CH3 – C – CHCl2 CH3 – C – CCl3 ClCH 2 COOH >
(electron-withdrawing gp.)
III IV
– O O
O O || ||

OH H C OH CH 3 C OH
CH3 – C – CCl3 CH3 – C – CCl3 (Electron-releasing character
increasing from Left to Right)
OH
O O
||

CH3 – C + CCl3 C2 H5 C OH
42. (b) Acetophenone (C6 H5COCH3) being a
OH ketone, does not reduce Tollen’s reagent.
Thus all compounds (I to IV) are ultimately 43. (a) Aldehydes, other than formaldehyde,
converted to CHCl3 (chloroform). when treated with RMgX give 2º alcohols
36. (b) CN – is a strong base and since the
NH 3
substrate is a tert-halide, it mainly 44. (a) CH3COOH CH3CONH 2
undergoes elimination reaction forming Ethanoic acid (A)
Ethanamide
alkene (A). In presence of dil. H2SO4,
alkenes undergo hydration in Br2 / NaOH
CH3 NH 2
NaNO2 /dil.HCl
CH 3OH
Markovnikov’s way. (B) (C)
Methanamine Methanol
CH 3 CH 2 CH3
|
CN
||
dil. H 2SO 4
| 45. (a)
CH3 C Cl CH3 C CH3 C OH
| ( HCl) | (hydration) | PCl3
CH3 CH 2 COOH CH 3 CH 2 CO Cl
CH3 CH3 CH3
A B (I)
37. (d)
conc .H 2SO 4 , heat
COOH CO CO 2 H 2O C6H/6AlCl3
| ( dehydratio n)
COOH
CO – CH2– CH3
O
||
38. (d) (a) H C Cl CO HCl

(b) HCl
CH 3 CONH 2 CH 3CO N H 3 Cl
(II)
(Acetamide as a weak base)
2 CH 3CONH 2 HgO (CH 3CONH) 2 Hg Wolf Kishner
(Acetamide as a weak acid) reduction

LiAlH 4
(c) CH 3CONH 2 CH2 – CH2– CH3
or Na / C 2 H 5OH

CH 3CH 2 NH 2 H2O
OH
Thus all the three are correct.
(III)
EBD_7100
C-170 Topicwise AIIMS Solved Papers – CHEMISTRY
46. (b) Aldol formed in aromatic aldehydes itself
loses water molecule without heating because O
double bond formed is more stable due to COOH C – Cl
conjugation with benzene ring.
C6H5CH=CHCHO is commonly known as SOCl2
49. (c)
cinnamaldehyde.
Br Br
47. (c) Electron withdrawing substituent (like
halogen, —NO2, C6H5 etc.) would disperse
O NH2
the negative charge and hence stabilise the
carboxylate ion and thus increase acidity C NH2
of the parent acid. On the other hand,
electron-releasing substituents would NH 3 NaOH
intensify the negative charge, destabilise +Br2
the carboxylate ion and thus decrease Br Br
acidity of the parent acid. 50. (d) Structures of given aldehydes
Electronegativity decreases in order CH 3CH 2 CHO
F > Cl > Br propanal
and hence –I effect also decreases in the Cl3CCHO
same order, therefore the correct option is trichloroethanal
[FCH2COOH > ClCH2COOH > BrCH2COOH HCHO
> CH3COOH] methanal

48. (a) C6 H5 COOC2 H5


CH3MgBr CH 3CHO
ethanal
OMgBr
| Mg(OC2 H5 )Br
C6 H 5 C OC2 H5 CHO
|
CH3 benzaldehyde

O Trichloroethanal, methanal and


|| benzaldehyde do not undergo aldol
Excess
C6 H 5 C CH3 condensation. Aldol condensation is not
CH3MgBr
given by aldehydes and ketones which do
OMgBr CH3
| |
not contain -hydrogen atom(s).
H2 O
C6 H5 — C — CH3 C6 H5 — C — CH3 51. (a) only those compounds which have -H give
| |
Aldol condensation
CH3 OH
O
CH3 ||
| (a) Cl3C CH (b) C 6 H 5 C H 2 CHO
Conc. H 2SO 4 Ozonolysis
C6 H 5 — C CH 2
(no -H) ( -H)
'B'
C 6 H 5 COCH 3 HCHO (c) C 4H9 C H 2 CHO (d) CH3NO2

3I 2 4 NaOH
( -H)
C 6 H 5 COCH 3 CHI 3 52. (b)
Aldehydes, Ketones and Carboxylic Acids C-171

Type B : Assertion Reason Questions Hence assertion and reason both are true
and reason is correct explanation of
53. (a) Acetaldehyde gives aldol condensation assertion.
reaction because it contains -hydrogen. 59. (b) RCOCl, (RCO)2O and RCOOR' all add two
54. (d) Both are false. KMnO4 is an oxidising agent molecules of Grignard reagents to give 3º
and acetylene on treatment with alkaline alcohols.
KMnO4 produces oxalic acid 60. (b) Both assertion and reason are true and reason
CH COOH is not the correct explanation of assertion .
||| Alk KMnO 4 | 61. (c) The correct reason : Carboxyl group only
4(O)
CH COOH marginally decreases the electron density at
55. (a) Grignard reagents are highly reactive, so m-position relative to o- and p-positions.
react with the hydroxyl group. 62. (b)
56. (c) Isobutanal does not give iodoform test 63. (c) Assertion is correct as conjugate base of a
because it does not have –COCH3 group. strong acid is weak. Since CH3COOH is a
stronger acid than CH3OH, CH3COO– is a
57. (c) Low pKa value corresponds to more acidic
weaker base than OCH3–.
compound. Acetic acid (pKa = 4.0) is more
64. (c)
acidic than phenol (pKa = 9.0). It is due to
65. (b) Correct explanation : Both carbanion
the fact that carboxylate ion is more
(formed in presence of a base) and enol form
stabilized due to resonance because its
(formed in presence of an acid) act as
reasonating structures are quivalent.
nucleophiles and hence add on the carbonyl
O
group of aldehydes and ketones to give
aldols.
58. (a) C H3 – C – NH2
66. (b) Aldehydes which do not contain -
Acetamide hydrogen undergo Cannizzaro reaction.
O O 67. (d)
68. (d) Benzaldehyde is less reactive than ethanol
O
C2H5 – C – CH2 – C – CH3 towards nucleophilic attack. The combined
Ethyl acetoacetate effect of –I and +R effect of phenyl group
is electron donating which increases the
As - NH2 is more basic than RO– hence electron density on the carbon atom of the
acetamide has more polar C = O group > C = O in benzaldehyde.
69. (a)
than OC2H5 group in ethyl acetoacetate.
EBD_7100
C-172 Topicwise AIIMS Solved Papers – CHEMISTRY

27 Amines

TYPE A : MULTIPLE CHOICE QUESTIONS 6. The product formed by the reaction of acetamide
with bromine in presence of NaOH is : [2001]
1. Identify Y in the reaction : [1998]
(a) CH 3CN (b) CH 3CHO
NaNO 2 / HCl H 2O
C 6 H 5 NH 2 X Y
dil.H 2SO 4 (c) CH 3CH 2OH (d) CH 3 NH 2

(a) C 6 H 5 N 2 Cl (b) C 6 H 5OH 7. Primary amine reacts with carbon disulphide and
HgCl2 to produce alkyl isothiocyanate. This
(c) C 6 H 5 NHOH (d) C6 H6 reaction is : [2001]
2. Dynamite is a mixture of: [1998] (a) Carbylamine reaction
(a) Nitroglycerine + raw dust (b) Hoffmann bromamide reaction
(b) Nitroglycerine + HCl (c) Perkin reaction
(c) Hydrogen bomb + H2SO4 (d) Hoffmann mustard oil reaction
(d) Glycerine + H2SO4
8. Which of the following is involved in
N 2 Cl Sandmeyer’s reaction? [2002]
3. (a) ferrous salt
(b) diazonium salt
Cl
Cu 2Cl2 , Conc.HCl (c) ammonium salt
+ N2
(d) cuprammonium salt
Chlorobenzene 9. In the reaction :
Above reaction is known as: [2000]
C6 H 5 CHO C6 H 5 NH 2
(a) Strecker's reaction
(b) Sandmeyer's reaction C6 H5 N = CHC6H5 + H2O
(c) Wohl-Ziegler reaction The compound, C6H5N = CHC6H5 is known as
(d) Stephen's reaction [2002]
4. Reduction of nitrobenzene with Sn/HCl (a) aldol (b) Schiff’s base
produces: [2001] (c) Schiff’s reagent (d) Benedict’s reagent
(a) azobenzene (b) azoxybenzene 10. The ortho/para directing group among the
(c) nitrobenzene (d) aniline following is : [2003]
5. Hinsberg's reagent is : [2001] (a) COOH (b) CN
(c) COCH3 (d) NHCONH2
(a) COOC 2 H5 11. Among the following, the weakest base is
|
COOC 2 H5 [2003]
(a) C6H5CH2NH2 (b) C6H5CH2NHCH3
(b) C 6 H 5SO 2 Cl (c) O2N.CH2NH2 (d) CH3NHCHO
12. Nitrobenzene gives N-phenylhydroxylamine by :
(c) C 6 H 5SO 2 NH 2
(a) Sn/HCl (b) H2/Pd-C [2003]
(d) CH 3COCH 2COOC 2 H 5 (c) Zn/NaOH (d) Zn/NH4Cl
Amines C-173

13. The strongest base among the following is 20. The following sequence of reactions on A gives
CH 2 CONH 2 1. Br / NaOH
2
(a) (b) [2004] 2. Heat [2006]
N COOCH 3
N H O
O
(a) (b) NH
NH2 O
(c) (d) O O
N O
H (c) NH (d)
14. Aromatic nitriles (ArCN) are not prepared by O O
reaction : [2004]
21. Nitrobenzene on treatment with zinc dust and
(a) ArX + KCN
aqueous ammonium chloride gives: [2006]
(b) ArN2 CuCN
(a) C6 H5 N NC6 H5
(c) ArCONH2 + P2O5
(d) ArCONH2 + SOCl2 (b) C 6 H 5 NH 2
15. Melting points are normally highest for :
(c) C 6 H 5 NO
(a) tertiary amides (b) secondary amides
(c) primary amides (d) amines [2004] (d) C 6 H 5 NHOH
16. Which of the following chemicals are used to 22. Which of the following statement is true? [2007]
manufacture methyl isocyanate that caused (a) Trimethyl amine forms a soluble compound
"Bhopal Tragedy"? [2005] with Hinsberg reagent and KOH.
(i) Methylamine (ii) Phosgene (b) Dimethyl amine reacts with KOH and phenol
(iii) Phosphine (iv) Dimethylamine to form an azo dye.
(a) (i) and (iii) (b) (iii) and (iv) (c) Methyl amine reacts with nitrous acid and
liberates N2 from aq. solution.
(c) (i) and (ii) (d) (ii) and (iv)
(d) None of these.
17. Among the following which one does not act as
23. Which of the following amines will not give N2
an intermediate in Hofmann rearrangement?
gas on treatment with nitrous acid (NaNO2 +
(a) RNCO (b) RCON [2005]
HCl) ? [2007]
(c) RCONHBr (d) RNC (a) C2H5NH2 (b) CH3NH2
18. Pyridine is less basic than triethylamine (c) (CH3)2CHNH2 (d) All will give N2.
because : [2005] 24. The compound which gives an oily nitrosoamine
on reaction with nitrous acid at low temperature,
(a) pyridine has aromatic character
is [2008]
(b) nitrogen in pyridine is sp2-hybridised
(a) CH3NH2 (b) (CH3)2CHNH2
(c) pyridine is a cyclic system
(c) CH3–NH–CH3 (d) (CH3)3N
(d) in pyridine, lone pair of electrons on
25. Diethyl oxalate is used for distinguishing
nitrogen is delocalised
primary, secondary and tertiary [2009]
19. C 6 H 5 CONHCH 3 can be converted into (a) alcohols
C6H5CH2NHCH3 by : [2005] (b) amines
(a) NaBH4 (b) H2-Pd/C (c) alkyl halides
(c) LiAlH4 (d) Zn-Hg/HCl (d) hydrogens in hydrocarbons
EBD_7100
C-174 Topicwise AIIMS Solved Papers – CHEMISTRY
26. Observe the following reaction : [2009] What is Y?
CH3 HOOC
(a) Acetanilide (b) Benzanilide
CH3
* (c) Azobenzene (d) Hydrazobenzene
H
H*
+ (R'R) + (R'S) 32. Which of the following gives primary amine on
(1)
NH2 Cl H reduction? [2013]
(R')-1-Phenylethylamine (R+S)-2-
Chloropropanoic acid
(a) CH 3 CH 2 NO 2
hydrolysis
(R'R) (R') + (R) (b) CH 3CH 2 O N O
separation by (3)
recrystallisation (c) C6H 5 N NC 6 H 5
(2)
(R'S)
hydrolysis
(R') + (S)
(d) CH 3CH 2 NC
(3)
33. Which of the following compound will not
Which statement is not correct about the above undergo azo coupling reaction with benzene
observation ? diazonium chloride. [2016]
(a) The product mixture of step-1 is optically (a) Aniline (b) Phenol
active (c) Anisole (d) Nitrobenzene
(b) The product R'R and R'S have identical 34. Which of the following are intermediates in
structural formula Sandmeyer reaction ? [2017]
(c) R'R is nonsuperimposable on R'S (i) C 6 H 5 N NCl (ii) C6 H5 N N
(d) R'R and R'S have same solubility in water
(iii) C 6 H 5 (iv) C6H5Cl
27. Fluorescein is an example of [2009]
(a) (ii) and (iii) (b) (i) and (iv)
(a) azo dyes
(c) (ii) and (iv) (d) (i) and (ii)
(b) phthalein dyes
(c) triphenylmethane dyes TYPE B : ASSERTION REASON QUESTIONS
(d) nitro dyes Directions for (Qs. 35-41) : These questions consist
28. Urea upon hydrolysis yields: [2010] of two statements, each printed as Assertion and
(a) acetamide Reason. While answering these questions, you are
(b) carbonic acid required to choose any one of the following five
(c) ammonium hydroxide responses.
(d) NO2 (a) If both Assertion and Reason are correct and
29. Benzamide and benzyl amine can be the Reason is a correct explanation of the
distinguished by [2011] Assertion.
(a) cold dil. NaOH (b) If both Assertion and Reason are correct but
(b) cold dil. HCl Reason is not a correct explanation of the
(c) both a & b Assertion.
(d) NaNO2, HCl, 0ºC, then -naphthol (c) If the Assertion is correct but Reason is incorrect.
30. The basic character of ethyl amine, diethyl amine (d) If both the Assertion and Reason are incorrect.
and triethyl amine in chlorobenzene is [2011] (e) If the Assertion is incorrect but the Reason is
(a) C2H5NH2 < (C2H5)2NH < (C2H5)3N correct.
35. Assertion : Amines are basic in nature
(b) C2H5NH2 < (C2H5)3N < (C2H5)2 NH
Reason : Presence of lone pair of electrons on
(c) (C2H5)3N < (C2H5)2NH < C2H5NH2
nitrogen atom. [1999]
(d) (C2H5)3N < C2H5NH2 < (C2H5)2NH 36. Assertion : Benzene diazonium chloride does
31. Consider the following reaction [2012] not give test for nitrogen.
Sn / HCl C6H5COCl Reason : Loss of N2 gas takes place during
C6H5 NO2 X Y HCl
heating. [1999]
Amines C-175

37. Assertion : p-O2N. C6H4.COCH3 is prepared by (b) If both Assertion and Reason are correct, but
Friedel Craft's acylation of nitrobenzene. Reason is not the correct explanation of
Reason : Nitrobenzene easily undergoes Assertion.
electrophilic substitution reaction. [2005] (c) If Assertion is correct but Reason is incorrect.
38. Assertion : Alkyl isocyanides in acidified water (d) If both the Assertion and Reason are incorrect.
give alkyl formamides. 42. Assertion : Nitration of aniline can be conveniently
Reason : In isocyanides, carbon first acts as a done by protecting the amino group by
nucleophile and then as an electrophile.[2005] acetylation.
39. Assertion : Anilinium chloride is more acidic Reason : Acetylation increases the electron-
than ammonium chloride. density in the benzene ring. [2010]
Reason : Anilinium ion is resonance stabilized. 43. Assertion : Acetamide reacts with Br 2 in
[2006] presence of methanolic CH3ONa to form methyl
N-methylcarbamate.
40. Assertion : Benzene diazonium salt on boiling
with water forms phenol. Reason : Methyl isocyanate is formed as an
intermediate which reacts with methanol to form
Reason : C – N bond is polar. [2007]
methyl N-methylcarbamate. [2014]
41. Assertion :Nitrobenzene is used as a solvent in 44. Assertion : Acylation of amines gives a
Friedel-Craft's reaction. monosubstituted product whereas alkylation of
Reason : Fusion of nitrobenzene with solid KOH amines gives polysubstituted product.
gives a low yield of a mixture of o-and p-
Reason : Acyl group sterically hinders the
nitrophenols. [2008] approach of further acyl groups [2016]
Directions for (Qs.42-46) : Each of these questions 45. Assertion : Aniline does not undergo Friedel-
contains an Assertion followed by Reason. Read them Crafts reaction. [2017]
carefully and answer the question on the basis of
Reason : –NH2 group of aniline reacts with AlCl3
following options. You have to select the one that
(Lewis acid) to give acid-base reaction.
best describes the two statements.
46. Assertion : Aniline is better nucleophile than
(a) If both Assertion and Reason are correct and anilium ion. [2017]
Reason is the correct explanation of Assertion. Reason : Anilium ion have +ve charge.
EBD_7100
C-176 Topicwise AIIMS Solved Papers – CHEMISTRY

Type A : Multiple Choice Questions 10. (d) –NH–CONH2 group is ortho para directing.
Nitrogen shares its lone pair with benzene
NaNO 2 HCl ring and makes this group ortho para
1. (b) C6 H5 NH 2 C 6 H5 N 2 Cl directing.
X
11. (c) As –NO2 is strong electron withdrawing
H 2O
C6H5OH
group.
Y NO 2 NHOH
2. (a) Dynamite is a mixture of nitroglycerine and
raw dust. Zn / NH 4Cl
3. (b) The given reaction is known as 12. (d)
Sandmeyer’s reaction. Phenyl hydroxylamine
4. (d) Reduction of nitrobenzene with Sn/HCl
produces aniline. 13. (c) The lone pair of electrons on nitrogen is
NO 2 NH 2 not involved in the formation of -electron
cloud of the ring.
14. (a) Aryl halide (ArX) does not undergo
Sn + HCl
+ 2H 2O nucleophilic substitution because they
have strong C—X bond due to resonance.
5. (b) Hinsberg’s reagent is C6 H5SO2Cl which 15. (d) Although amines as well as amides form
is used to distinguish primary, secondary intermolecular H-bonding, H-bonding in
O
and tertiary amines. ||
amides is less prominent because of – C –
6. (d) CH 3 CONH 2 2NaOH Br2
O
|| ••
CH3 NH 2 Na 2CO3 2NaBr H 2O group of – C – N H 2
7. (d) The reaction is known as Hoffmann mustard
oil reaction. 16. (c)
S CH3 NH 2 COCl2 CH3 NCO 2HCl
|| Phosgene Methyl isocyanate
R–NH2 + S = C = S R–NH– C –SH
17. (d) Hofmann rearrangement
HgCl2 O O
R – N = C = S + HgS + 2HCl || Br2 ||
Alkyl isothiocyanate R C NH 2 R C NHBr
8. (b) Diazonium salt is involved in Sandmeyer’s
reaction. O
||
Ar N 2 X Cu 2 Cl2 / HCl KOH R C N — Br
ArCl N 2 X (– Br )
9. (b) Primary amines react with aldehydes or O O
ketones to form compound known as || ||
: :

Schiff’s base R–C–N C=N–R


Alkyl isocyanate

–N H2 + O HC– O
|| OH
C N R H 2O
RNH 2 CO32 .
N CH
Schiff’s base
Amines C-177

18. (b) Pyridine is less basic because nitrogen in CH 3 NH CH 3 HNO 2


pyridine is sp 2 hybridised, an d sp 2 (2 amine)
hybridised orbital is more acidic in character
than sp3 hybridised orbital in (C2H5)3N. NO
|
LiAlH 4 CH3 N CH3 H 2O
19. (c) C6 H 5 CONHCH 3
dimethyl nitrosoamine
(oily)
C6 H5CH 2 NHCH3
25. (b) Diethyl oxalate is used for distinguishing
CH 2 CONH 2 1°, 2° and 3° amines as three amines react
20. (c) Br2 / NaOH differently as discussed below.
The 1° amine forms corresponding
COOCH 3
substituted oxamide which is a crystalline
solid; while 2° amine forms a diethyl oxamic
CH 2 NH 2 CH2 ester which is a liquid and 3° amine does
Heat NH not react with diethyl oxalate since it does
COOCH 3 C not contain a replacable hydrogen atom.
O COOC 2 H5 CONHR
RNH 2 | | 2C2 H5OH
NO 2 NHOH
1 amine COOC 2 H5 CONHR
oxamide
Zn / NH 4Cl crystalline
21. (d) solid

22. (c) Among the given statements only (c) is COOC 2 H5 CO NR 2


true. As methyl amine liberates N2 on R 2 NH | | C2 H5OH
2 amine COOC 2 H5 COOC 2 H5
reaction with nitrous acid.
CH3 – NH2 + 2HONO ¾¾ ® N, N dialkyl
oxamic ester
CH3 – ONO + N2+ 2H2O
While dimethyl amine and trimethyl amine COOC 2 H5
form insoluble compound with Hinsberg R3 N | No reaction
reagent and KOH. 3 amine COOC 2 H5

OH
26. (d) R'R and R'S are diastereomers and have dif-
R 2 NH C6 H 5SO 2Cl ferent physical properties like water solu-
C6 H5SO 2 NR 2 bility, B.P., M.P. etc.
Mixture of diastereomers is optically active
OH hence the product mixture in step 1 is opti-
R 3 N C6 H5SO 2Cl R3N
cally active.
insoluble
27. (b) Fluorescein also called resorcinol-
Azo dyes are not formed by secondary
phthalein is an example of phthalein dye. It
amines.
23. (d) All aliphatic primary amines liberate N2 on is prepared by heating phthalic anhydride
treatment with nitrous acid (NaNO2 + HCl) and resorcinol over a zinc catalyst, and it
crystallizes as a deep red powder.
RNH2 + HONO ¾¾ ® R – OH + N2 + H2O
O
24. (c) Out of primary, secondary and tertiary
28. (b) hydrolysis
amines, the secondary amines, on reaction NH 2 C NH 2
acid or base
with HNO 2 , produce yellow oily
compounds called nitrosoamines. H2 O
CO2 2H3 H 2CO3
Carbonic acid
EBD_7100
C-178 Topicwise AIIMS Solved Papers – CHEMISTRY
Note: The enzyme urease (occurs in 39. (c) Anilinium chloride is more acidic than
soyabean) brings about the same change. ammonium chloride because it liberates
29. (b) Cold dil. NaOH does not attack to either of aniline (resonance stabilized) when heated
the compound, while cold dil. HCl reacts with strong base.
only with benzyl amine C6H5CH2NH2. Anilinium ions does not show resonance
30. (a) In presence of chlorobenzene, hydrogen because charge dispersion at ring may
bonding is not possible between the involve pentavalent nitrogen structure.
protonated amine and the solvent and thus
the stabilization factor (solvation effect) is +
N N OH
absent. Hence basicity is explained on the
basis of the number of electron releasing
40. (b) H2O
groups in an amine.
31. (b) 32. (a) 33. (d) 34. (a) boil
benzene
Type B : Assertion Reason Questions diazonium salt

35. (a) Amines are basic in nature due to presence


Although C–N bond is polar but it is not
of a lone pair of electrons on nitrogen.
exact reason for the substitution.
36. (a) Benzene diazonium chloride does not give
test of nitrogen as nitrogen gas is evolved 41. (b) Nitrobenzene is used as a solvent in Friedel-
on heating. Craft's reaction because its –NO2 group
37. (d) Nitrobenzene undergoes electrophilic deactivates benzene ring for electrophilic
substitution reaction with difficulty substitution.
because NO2 group is electron withdrawing Although the given staement of the reason
& therefore, it deactivates the benzene ring. is correct, it is not correct explanation of
the given statement.
– H H 2O
38. (a) R N C R N CH 42. (c) Acetylation decreases the electron density
in the benzene ring thereby preventing
H O oxidation.
|| 43. (a) 44. (c) 45. (a)
R N C R NH C H
Alkyl formamide 46. (a) It is fact that aniline is better nucleophile
OH than anilium ion. Anilium ion contain +ve
In the first step due to partial nagative charge, which reduces the tendency to
charge on carbon it acts as nucleophile and donate lone pair of electron C6 H5 NH3+
therefore attacked by H+ of the acid. Once
it acquires positive charge, it becomes (Anilium ion).
electrophile as is shown in the next step.
28 Biomolecules
TYPE A : MULTIPLE CHOICE TYPE A :
MULTIPLE CHOICE QUESTIONS 7. If one strand of DNA has the sequence
ATCGTATG, the sequence in the complementary
1. Which one of the following statements is true
for protein synthesis (translation)? [2005] strand would be: [2009]
(a) Amino acids are directly recognized by (a) TAGCTTAC (b) TCACATAC
m-RNA (c) TAGCATAC (d) TACGATAC
(b) The third base of the codon is less specific 8. Fructose on reduction gives a mixture of two
(c) Only one codon codes for an amino acid alcohols which are related as [2011]
(d) Every t-RNA molecule has more than one (a) diastereomers (b) epimers
amino acid attachement (c) both (a) and (b) (d) anomers.
2. The pair in which both species have iron is : 9. -Amino acids are [2011]
(a) Nitrogenase, cytochromes [2006] (a) acidic due to –COOH group and basic due
(b) Carboxypeptidase, haemoglobin to –NH2 group
(b) acidic due to –NH3+ group and basic due
(c) Haemocyanin, nitrogenase
to –COO– group.
(d) Haemoglobin, cytochromes (c) neither acidic nor basic.
3. Lysine is least soluble in water in the pH range: (d) none is true.
(a) 3 to 4 (b) 5 to 6 [2006] 10. Denaturation of proteins leads to loss of its
(c) 6 to 7 (d) 8 to 9 biological activity by [2012]
(a) Formation of amino acids
4. Thymine is : [2006]
(b) Loss of primary structure
(a) 5-Methyluracil (b) 4-Methyluracil (c) Loss of both primary and secondary
(c) 3-Methyluracil (d) 1-Methyluracil structures
5. Which of the following statement is not correct? (d) Loss of both secondary and tertiary
[2008] structures
(a) Chlorophyll is responsible for the synthesis 11. Glucose molecule reacts with X number of
of carbohydrates in plants molecules of phenylhydrazine to yield osazone.
(b) The compound formed by the addition of The value of X is [2013]
oxygen to haemoglobin is called (a) three (b) two
oxyhaemoglobin (c) one (d) four
(c) Acetylsalicylic acid is known as aspirin 12. For osazone formation, the effective structural unit
necessary is [2014]
(d) The metal ion present in vitamin B12 is
Mg2+ (a) CH2OCH3 (b) CH 2OH
| |
6. The beta and alpha glucose have different CO
CO
specific rotations. When either is dissolved in | |
water, their rotation changes until the same fixed
value results. This is called [2008] (c) CH 2OH (d) CHO
| |
(a) epimerisation (b) racemisation
CHOCH 3 CHOCH 3
(c) anomerisation (d) mutarotation | |
EBD_7100
C-180 Topicwise AIIMS Solved Papers – CHEMISTRY
13. Which of the statements about "Denaturation" 17. Assertion : Proteins on hydrolysis produce
given below are correct ? [2015] amino acids.
(A) Denaturation of proteins causes loss of Reason : Amino acids contain –NH2 and –COOH
secondary and tertiary structures of the groups. [1998]
protein. 18. Assertion : Sucrose undergoes mutarotation.
(B) Denturation leads to the conversion of Reason : Sucrose is a disaccharide. [2000]
double strand of DNA into single strand
19. Assertion : DNA as well as RNA molecules are
(C) Denaturation affects primary structure found in the nucleus of a cell.
which gets distorted
Reason : On heating, the enzymes do not lose
Options :
their specific activity. [2002]
(a) (B) and (C) (b) (A) and (C) 20. Assertion : Haemoglobin is an oxygen carrier.
(c) (A) and (B) (d) (A), (B) and (C)
14. Which one of the following statements is Reason : Oxygen binds as O 2– to Fe of
correct? [2016] haemoglobin. [2003]
(a) All amino acids except lysine are optically 21. Assertion : Glycosides are hydrolyzed in acidic
active conditions
(b) All amino acids are optically active Reason : Glycosides are acetals. [2003]
(c) All amino acids except glycine are optically 22. Assertion : Carboxypeptidase is an
active exopeptidase.
(d) All amino acids except glutamic acids are Reason : It cleaves the N-terminal bond.
optically active [2004]
15. - D-(+)-glucose and -D-(+)-glucose are [2017] 23. Assertion : Sucrose is a non-reducing sugar.
(a) conformers (b) epimers Reason : It has glycosidic linkage. [2004]
24. Assertion : Maltose is a reducing sugar which
(c) anomers (d) enatiomers
gives two moles of D-glucose on hydrolysis.
TYPE B : ASSERTION REASON QUESTIONS Reason : Maltose has 1, 4- -glycosidic linkage
Directions for (Qs. 16-25) : These questions consist [2005]
of two statements, each printed as Assertion and 25. Assertion : Alpha ( )-amino acids exist as
Reason. While answering these questions, you are internal salt in solution as they have amino and
required to choose any one of the following five carboxylic acid groups in near vicinity.
responses. Reason : H+ ion given by carboxylic group
(a) If both Assertion and Reason are correct and (–COOH) is captured by amino group (–NH2)
the Reason is a correct explanation of the having lone pair of electrons. [2007]
Assertion. Directions for (Qs.26-31) : Each of these questions
(b) If both Assertion and Reason are correct but contains an Assertion followed by Reason. Read them
Reason is not a correct explanation of the carefully and answer the question on the basis of
Assertion. following options. You have to select the one that
(c) If the Assertion is correct but Reason is incorrect. best describes the two statements.
(d) If both the Assertion and Reason are incorrect.
(a) If both Assertion and Reason are correct and
(e) If the Assertion is incorrect but the Reason is
Reason is the correct explanation of Assertion.
correct.
(b) If both Assertion and Reason are correct, but
16. Assertion : Hydrolysis of sucrose is known as
inversion of cane sugar. Reason is not the correct explanation of
Reason : Sucrose is a disaccharide. [1997] Assertion.
Biomolecules C-181

(c) If Assertion is correct but Reason is incorrect. 29. Assertion : Treatment of D-glucose with dilute
(d) If both the Assertion and Reason are incorrect. alkali affords an equilibrium mixture consisting
of D-mannose, D-fructose and starting substance
26. Assertion :Disruption of the natural structure
D-glucose.
of a protein is called denaturation. Reason : The reaction involves an intermediate
Reason : The change in colour and appearance in which hybridisation of C2 changes from sp3
of egg during cooking is due to denaturation. to sp2. [2013]
[2008] 30. Assertion : Glucose and fructose give the same
27. Assertion : Proteins are made up of -amino osazone.
acids. Reason : During osazone formation
Reason : During denaturation, secondary and stereochemistry only at C1 and C2 is destroyed.
tertiary structures of proteins are destroyed. [2014]
[2011] 31. Assertion : Vitamin D cannot be stored in our
28. Assertion : At isoelectric point, the amino group body
does not migrate under the influence of electric Reason : Vitamin D is fat soluble vitamin and is
field. excreted from the body in urine. [2016]
Reason : At isoelectric point, amino acid exists
as a zwitterion. [2012, 2013]
EBD_7100
C-182 Topicwise AIIMS Solved Papers – CHEMISTRY

Type A : Multiple Choice Questions 12. (b)


13. (c) When the proteins are subjected to the
1. (b) The triplet of nucleotides having a specific action of heat, mineral acids or alkali, the
sequence of bases is known as codon. water soluble form of globular protein
More than one codon can specify same changes to water insoluble fibrous protein.
amino acid. Specificity of a codon is This is called denaturation of proteins.
determined by first two bases, the third base During denaturation secondary and tertiary
of a codon is not very important. structures of protein destroyed but primary
2. (d) Haemoglobin and cytochromes. structures remains intact.
NH2 14. (c) With the exception of glycine all the 19 other
| common amino acids have a uniquely
3. (c) H 2 NCH 2 CH 2 CH 2 CH 2 C HCOOH , different functional group on the central
Lysine is least soluble in water in the pH tetrahedral alpha carbon.
range 6–7. H
4. (a) Thymine is 5-methyluracil. |
5. (d) The statement is wrong or not correct. H — C — COOH
The metal ion present in vitamin B12 |
(cyanocobalamine) is cobalt. All other NH 2
statements are correct.
6. (d) This type of change in optical rotation is Glycine
called mutarotation. 15. (c) Anomers are those diastereomers that differ
7. (c) One strand of DNA : AT C GTAT G in configuration at C – 1 atom.
Complementary strand : T A G C A T A C Since D ( ) glucose and – D –
8. (c) Ketoses on reduction produce a new chiral (+) glucose differ in configuration at C – 1
carbon leading to the formation of two atom so they are anomers.
isomeric alcohols which are diastereomeric
Type B : Assertion Reason Questions
as well as C–2 epimers.
9. (b) Amino acids exist as Zwitterions in which 16. (b) Sucrose is a disaccharide with specific
rotation of +66.5°. On hydrolysis, it gives
acidic character is due to NH 3 and basic
1 mole of D– (+) glucose and 1 mole of
due to –COO– group. D–(–)– fructose. Th e hydrolysate is
R R laevorotatory hence hydrolysis is known
| | as inversion of sucrose.
acid
H3 N C HCOOH H3 N C HCOO 17. (b) Proteins are polyamides so, on hydrolysis,
R give amino acids. Further it is a fact that
base
| amino acids contain both –NH2 as well as
H 2 N C HCOO –COOH group. So assertion and reason,
10. (d) although both are correct but reason is not
11. (a) correct explanation of assertion.
18. (b) Sucrose which is a disaccharide undergoes
CHO CH=N.NHPh mutarotation because it is converted into
3PhNHNH2 glucose and fructose on hydrolysis and the
CHOH CH=N.NHPh + PhNH2+ NH 3
products have different optical activity from
(CHOH)3 (CHOH)3 that of the reactant.
19. (d) DNA is found mainly in the nucleus of the
CH2OH CH2OH cell and RNA occurs mainly in the
Biomolecules C-183

cytoplasm of the cell. So assertion given is R R


false. – + + –
Enzymes are very good biological catalysts H2N–CH–COO +H H3N–CH–COO
in certain temperature range but they lose zwitter ion
their specific activity on heating. (dipolar ion)
Hence reason is also a wrong statement. Hence assertion and reason both are true
20. (c) The assertion is correct but the reason is and reason is the correct explanation of
incorrect because oxygen binds as O2 (and assertion.
not as O2–) to Fe of haemoglobin. 26. (b) During denaturation, the protein molecule
21. (d) Glycosides are formed by treating glucose uncoils, form a more random conformation
with methanol in presence of dry HCl gas. and ultimately precipitates from the
They cannot be hydrolysed in acidic solution. Further during denaturation, a
conditions. They are not acetals but they protein molecule loses its biological activity.
are hemiacetals. Thus, reason is correct but it is not the
22. (c) It is true that carboxypeptidase is an
correct explanation of assertion.
exopeptidase because it cleaves the peptide
27. (c) Correct explanation : During formation of
chain at carboxy terminal amino acids.
proteins, –NH2 group of one amino acid
23. (a) Sucrose is a non-reducing sugar because it
condenses with –CO2H of the other with
has glycosidic linkage which has no free
elimination of a water molecule to form a
aldehyde or ketonic group
peptide bond.
24. (c) The two glucose units of maltose are linked
28. (a) Reason is the correct explanation of
through -glycosidic linkage between C–1
Assertion.
of one glucose unit and C–4 of the other.
29. (a) Reason is the correct explanation of
25. (a) All -amino acids have –NH2 and –COOH
Assertion.
groups. Since –NH2 group is basic and
30. (a) Reason is the correct explanation of
–COOH group is acidic, in neutral solution
Assertion.
it exists as internal salt which is also called
31. (d) Vitamin D is a fat soluble vitamin and can
as zwitter ion. This zwitter ion is formed
be stored in the body since it is not excreted
due to reason that proton of –COOH group
is transferred to –NH2 group. out of the body.
R
In water
H2N–CH–COOH
amino acid
EBD_7100
C-184 Topicwise AIIMS Solved Papers – CHEMISTRY

29 Polymers

TYPE A : MULTIPLE CHOICE QUESTIONS 6. Which one of the following is not a condensation
polymer ? [2017]
1. Which of the following is a polymer containing (a) Melamine (b) Glyptal
nitrogen? [2008] (c) Dacron (d) Neoprene
(a) Polyvinyl chloride (b) Bakelite
(c) Nylon (d) Terylene TYPE B : ASSERTION REASON QUESTIONS
2. Synthetic detergents are more effective in hard Directions for (Qs.7-8) : Each of these questions
water than soaps because [2009] contains an Assertion followed by Reason. Read them
(a) they are non-ionic carefully and answer the question on the basis of
(b) their Ca++ and Mg++ salts are insoluble in water following options. You have to select the one that
best describes the two statements.
(c) their Ca++ and Mg++ salts are water soluble
(a) If both Assertion and Reason are correct and
(d) they are highly soluble in water
Reason is the correct explanation of Assertion.
3. Teflon, styron and neoprene are all (b) If both Assertion and Reason are correct, but
(a) Copolymers [2012] Reason is not the correct explanation of
(b) Condensation polymers Assertion.
(c) Homopolymers (c) If Assertion is correct but Reason is incorrect.
(d) Monomers (d) If both the Assertion and Reason are incorrect.
4. Which compound/set of compounds is used in 7. Assertion : Bakelite is a thermosetting polymer.
the manufacture of nylon-66? [2014] Reason : Bakelite can be melted again and again
(a) HOOC(CH2)4 COOH + H2N(CH2)6NH2 without any change. [2015, 2016]
(b) CH2 = CH–C(CH) = CH2 8. Assertion : In vulcanisation of rubber, sulphur
(c) CH2 = CH2 cross links are introduced.
Reason : Vulcanisation is a free radical initiated
(d) HOOC COOH chain reaction. [2017]
+ HOCH2 – CH2 OH
5. The repeating unit present in Nylon 6 is [2016]
(a) — [NH(CH2)6NHCO(CH2)4CO] —
(b) — [CO(CH2)5NH] —
(c) — [CO (CH2)6NH] —
(d) — [CO (CH2)4NH] —
Polymers C-185

Type A : Multiple Choice Questions 3. (c)


1. (c) Nylon is a polymer of adipic acid and 4. (a)
hexamethylenediamine thus it contains 5. (b) Nylon 6 is
nitrogen. Polyvinyl chloride is a polymer O O
of vinyl chloride. It does not contain —(NH(CH 2 )5 —C— NH— (CH2)5 —C— )n
nitrogen.
6. (d) Neoprene is an addition polymer of
Bakelite is a resin of formaldehyde and isoprene.
phenol. It does not contain nitrogen.
Cl
Terylene is a polymer of ethylene glycol |
O 2 or peroxides
and terephthalic acid. It does not contain nCH 2 CH C CH 2
nitrogen. It is also called dacron. Chloroprene
2. (c) Structural features of soaps and detergents Cl
are almost same except that the polar end |
in detergents is OSO Na while in — CH 2 C CH CH2 —
3 Neoprene
soaps polar end is COO Na .
Detergents have an advantage over soaps Type B : Assertion Reason Questions
that its polar end sulphate and sulphonate
7. (c) Bakelite can be heated only once.
retain their efficiency in hard water, since
8. (b) Vulcanisation is a process of treating
the corresponding Ca and Mg salts are natural rubber with sulphur or some
soluble. Being salts of strong acids, they compounds of sulphur under heat so as to
yield neutral solution, in contrast, to the modify its properties. This cross-linking
soaps, which being salts of weak acids yield give mechanical strength to the rubber.
slightly alkaline solutions.
EBD_7100
C-186 Topicwise AIIMS Solved Papers – CHEMISTRY

30 Chemistry in Everyday Life


TYPE A : MULTIPLE CHOICE TYPE A :
MULTIPLE CHOICE QUESTIONS 9. Which of the following compounds is not an
antacid ? [2017]
1. Arsenicals are mainly used for treatment of: (a) Phenelzine
[1998] (b) Ranitidine
(a) Cholera (b) Jaundice (c) Aluminium hydroxide
(c) Syphilis (d) Typhoid (d) Cimetidine
2. Which of the following is a local anaesthetic?
TYPE B : ASSERTION REASON QUESTIONS
[2009]
(a) Diazepam (b) Procaine Directions for (Q. 10) : These questions consist of
(c) Chloramphenicol (d) Penicillin-G two statements, each printed as Assertion and Reason.
3. Which of the following is used as a ‘morning While answering these questions, you are required to
after pill’ [2010] choose any one of the following five responses.
(a) Norethindrone (b) Ethynylestradiol (a) If both Assertion and Reason are correct and
(c) Mifepristone (d) Bithional the Reason is a correct explanation of the
4. Identify the wrong statement in the following: Assertion.
(a) Chlorofluorocarbons are responsible for (b) If both Assertion and Reason are correct but
ozone layer depletion [2012] Reason is not a correct explanation of the
(b) Greenhouse effect is responsible for global Assertion.
warming (c) If the Assertion is correct but Reason is incorrect.
(c) Acid rain is mostly because of oxides of (d) If both the Assertion and Reason are incorrect.
nitrogen and sulphur (e) If the Assertion is incorrect but the Reason is
(d) Ozone layer does not permit infrared correct.
radiation from the sun to reach the earth 10. Assertion : Activity of an enzyme is pH-
5. Which of the following is used for inducing dependent.
sleep? [2012] Reason : Change in pH affects the solubility of
(a) paracetamol enzyme in water. [2003]
(b) chloroquine Directions for (Qs.11-14) : Each of these questions
(c) bithional contains an Assertion followed by Reason. Read them
(d) barbituric acid derivatives carefully and answer the question on the basis of
6. Arsenic drugs are mainly used in the treatment following options. You have to select the one that
of [2013] best describes the two statements.
(a) jaundice (b) typhoid (a) If both Assertion and Reason are correct and
(c) syphilis (d) cholera Reason is the correct explanation of Assertion.
7. A broad spectrum antibiotic is [2014] (b) If both Assertion and Reason are correct, but
(a) paracetamol (b) penicillin Reason is not the correct explanation of
(c) aspirin (d) chloramphenicol Assertion.
8. Which of the following is an antidiabetic drug? (c) If Assertion is correct but Reason is incorrect.
(a) Insulin (b) Penicillin [2015] (d) If both the Assertion and Reason are incorrect.
(c) Chloroquine (d) Aspirin
Chemistry in Everyday Life C-187

11. Assertion : Penicillin is an antibiotic. 13. Assertion : Tetracycline is a broad spectrum


Reason : The drugs which act on the central antibiotic.
nervous system and help in reducing anxiety are Reason : Tetracycline is effective against a
number of types of bacteria, large viruses and
called antibiotics. [2010]
typhus fever. [2016]
12. Assertion : The addition of BHA to butter 14. Assertion : Sedatives are given to patients who
increases its storage life from months to years. are mentally agitated and violent.
Reason : Butylated hydroxy anisole (BHA) is an Reason : Sedatives are used to suppress the
antioxidant. [2014] activities of central nervous system.
[2017]
EBD_7100
C-188 Topicwise AIIMS Solved Papers – CHEMISTRY

Type A : Multiple Choice Questions Type B : Assertion Reason Questions


1. (c) Arsenicals are mainly used for the treatment 10. (b) The assertion that activity of an enzyme is
of syphilis. pH dependent is true because with change
2. (b) Procaine is the only drug among the given in pH the enzymes are denatured. The
options that is used as a local anaesthetic. statement that change in pH affects
solubility of enzyme in water is also true.
Chloramphenicol and penicillin-G, both are
11. (c) Assertion is true but Reason is false.
antibiotics. Diazepam is a sedative. The drugs which act on the central nervous
3. (c) system and help in reducing anxiety are
4. (d) Ozone layer acts as a shield and does not called tranquilizers.
allow ultraviolet radiation from sun to reach
12. (a) Antioxidants are the compounds that retard
earth. It does not prevent infra-red radiation
the action of oxygen on the food and
from sun to reach earth, thus option (d) is
wrong statement and so it is the correct thereby help its preservation.
answer. 13. (a) Broad spectrum antibiotics are those
5. (d) medicines which are effective against
several different types of harmful micro
6. (c)
organisms.
7. (d)
14. (a) A small quantity of sedative produces a
8. (a) feeling of relaxation, calmness and
9. (a) Phenelzine is an antidepressant, while drowsiness.
others are antacids.
31 Analytical Chemistry

TYPE A : MULTIPLE CHOICE QUESTIONS (a) CaCl2 (b) BaCl2


(c) CuCl2 (d) None of these
1. Beilstein test is used for the detection of :
(a) N2 (b) CO2 [1999] TYPE B : ASSERTION REASON QUESTIONS
(c) Na (d) Cl Directions for (Qs. 9-17) : These questions consist
2. When 8.3 g copper sulphate reacts with excess of two statements, each printed as Assertion and
of potassium iodide then the amount of iodine Reason. While answering these questions, you are
liberated is: [2000] required to choose any one of the following five
(a) 42.3 g (b) 24.3 g responses.
(c) 4.23 g (d) 2.43 g (a) If both Assertion and Reason are correct and
3. The compound used for gravimetric estimation the Reason is a correct explanation of the
of copper(II) is: [2003] Assertion.
(a) Cu(SCN)2 (b) Cu2O (b) If both Assertion and Reason are correct but
(c) Cu2 I2 (d) Cu2CO3 Reason is not a correct explanation of the
Assertion.
4. Dimethyl glyoxime gives a red precipitate with
Ni2+, which is used for its detection. To get this (c) If the Assertion is correct but Reason is incorrect.
precipitate readily, the best pH range is :[2004] (d) If both the Assertion and Reason are incorrect.
(a) < 1 (b) 2 – 3 (e) If the Assertion is incorrect but the Reason is
correct.
(c) 3 – 4 (d) 9 – 11
9. Assertion : Potassium can be used in Lassaigne
5. Which of the following imparts green colour to
test
the burner flame? [2004]
Reason : Potassium reacts vigorously. [1997]
(a) B(OMe)3 (b) Na(OMe)
10. Assertion : During test for nitrogen with
(c) Al(OPR)3 (d) Sn(OH)2
Lassaigne extract on adding FeCl3 solution,
6. Nessler’s reagent is used to detect [2010] sometimes a red precipitate is obtained.
(a) PO4 3– (b) MnO4 –
Reason : Sulphur is also present [2001]
(c) NH4+ (d) CrO42– 11. Assertion : Sb (III) is not precipitated as
7. The reagent that can distinguish between silver sulphide when in its alkaline solution H2S is
and lead salt is [2012] passed.
(a) H2S gas Reason : The concentration of S2– ion in alkaline
(b) Hot dilute HCl solution medium is inadequate for precipitation. [2004]
(c) NH4Cl (solid) + NH4OH (solution) 12. Assertion : Borax bead test is not suitable for
Al(III).
(d) NH4Cl (solid) + (NH4)2CO3 solution
Reason : Al2O3 is insoluble in water [2005]
8. A laboratory reagent imparts green colour to the
13. Assertion : In the iodometric titration, starch is
flame. On heating with solid K2Cr2O7 and conc.
used as an indicator.
H2SO4 it evolves a red gas. Identify the reagent
Reason : Starch is a polysaccharide. [2006]
[2017]
EBD_7100
C-190 Topicwise AIIMS Solved Papers – CHEMISTRY
14. Assertion : K2 Cr 2 O7 is used as a primary Directions for (Q.18) : Each of these questions
standard in volumetric analysis. contains an Assertion followed by Reason. Read them
Reason : It has a good solubility in water. carefully and answer the question on the basis of
[2006] following options. You have to select the one that
15. Assertion : Sb2S3, is not soluble in yellow best describes the two statements.
ammonium sulphide. (a) If both Assertion and Reason are correct and
Reason : The common ion effect due to S2– ions Reason is the correct explanation of Assertion.
reduces the solubility of Sb2S3 [2006] (b) If both Assertion and Reason are correct, but
Reason is not the correct explanation of
16. Assertion : Fe3+ can be used for coagulation of Assertion.
(c) If Assertion is correct but Reason is incorrect.
As2S3 sol.
(d) If both the Assertion and Reason are incorrect.
Reason : Fe3+ reacts with As2S3 to give Fe2S3
18. Assertion : Cu2+ and Cd2+ are separated from
[2006]
each other by first adding KCN solution and
17. Assertion : Change in colour of acidic solution
then passing H2S gas.
of potassium dichromate by breath is used to
Reason : KCN reduces Cu2+ to Cu+ and forms a
test drunk drivers.
complex with it. [2013]
Reason : Change in colour is due to the
complexation of alcohol with potassium
dichromate. [2006]
Analytical Chemistry C-191

Type A : Multiple Choice Questions determination of ammonia (or NH4 +) in


1. (d) Beilstein test is used for the detection of solution. It gives a yellow colour or brown
halogens. A green or blue colour in flame precipitate of oxydimercuric ammonium
Hg
indicates the presence of halogen. The O NH 2I even wi t h
colour is produced due to the formation of iodide Hg
copper halides. concentra-tion as low as 1 part per
2. (c) 2CuSO4 .5H 2O + 4 KI million of NH3.
498g 7. (b) Hot HCl will produce precipitate of AgCl
with Ag+ only. PbCl2 will not precipitate
Cu 2 I 2 2K 2 SO 4 I2 10H 2 O
254 g
because it is soluble in hot solution.
498 g of CuSO4 liberate I2 = 254 g 8. (b) The reagent is BaCl2 which imparts green
colour to flame. BaCl 2 forms chromyl
254 chloride (which is red in colour), when
8.3 g of CuSO4 liberate I2 = 8.3
498 treated with K2Cr2O7 and conc. H2SO4.
= 4.23 g 2BaCl2 K 2Cr2O7 3H 2SO4
3. (a) Gravimetrically, Cu2+ ions are estimated as
Cu(SCN)2, copper thiocyanate. K 2SO4 2BaSO4 2CrO2Cl 2 3H 2 O
Chromyl chloride
CH3 —C = NOH (red gas)
2+
4. (d) + Ni
CH3 —C = NOH Type B : Assertion Reason Questions
O H—O
9. (e) Potassium is not used in Lassaigne’s test
CH3 —C = N 2+ N = C — CH3 + because of its higher reactivity.
Ni + 2H
CH3 —C = N N = C — CH3 10. (a) Due to presence of sulph ur, sodium
thiocyanate is produced which produces
O —H O
blood red coloured precipitate with Fe3+ ion.
The reaction indicates that protons are
released during reaction. Hence a basic Na C S N NaSCN
medium is needed to facilitate the reaction.
Thus the best pH range is 9-11. If medium Fe3 3 NaSCN Fe(SCN) 3 3Na .
is kept acidic then nitrogen atom of the
11. (d) Sb (III), placed in II group of qualitative
ligand will start donating lone pair to H+
ion of the acid and not to central metal ion. analysis is precipitated as Sb2S3 by passing
5. (a) In the qualitative analysis of BO33–, mixture H2S in presence of HCl which decreases
is heated with conc. H2SO4 and little alcohol ionisation of H2 S (due to common ion
when trialkyl borate, R3BO3 or B(OR)3 is effect). This is done to avoid precipitation
formed. of metals of higher groups.
2BO3– 3H2SO4 2H3BO3 3SO2–
3 4 H 2S 2H S2–
H 3 BO3 3ROH R 3 BO3 3H 2 O
The vapours of trialkyl borate, B(OMe)3 HCl H + +Cl –
impart green colour to the burner flame Thus when Sb(III) is precipitated by low
6. (c) Nessler’s reagent (K2 [HgI4 ]) is used for
concentration of H2S, it will easily be
t he detect ion a nd quan ti t at ive
EBD_7100
C-192 Topicwise AIIMS Solved Papers – CHEMISTRY
precipitated when concentration of S2– is ammonium thioantimonate.
high, i.e., by H2S in presence of alkali. Sb 2S3 3(NH 4 ) 2 S + 2S
2–
H 2S 2H S 2(NH 4 )3 SbS4
H +
OH –
H2 O The common ion effect due to S2– ion does
not reduce the solubility of Sb2S3. So both
Thus here both assertion and reason are
false. A as well as R, are false.
12. (b) Metals form coloured compound 16. (c) Fe3+ can be used to coagulate As2S3 as sol
(metaborates) with borax. On the basis of of As2 S3 is negatively charged due to
adsorption of S2– ions. The positive charge
colour of metaborates, we can identify the
of Fe3+ ion neutralises the negative charge
metal. Aluminium does not form coloured
of sol particles. Reason as given in the
metaborate, so this test is not valid for Al3+
question does not explain A.
ion. Al2O3 is insoluble in water but it is not
17. (c) A is true but R is wrong because change in
explanation of A
colour of potassium dichromate is due to
13. (b) Starch is used as an indicator because it
oxidation of alcohol by dichromate.
forms blue coloured complex with iodine.
K2Cr2O7 + 4H2SO4
Starch is a polysaccharide but this is not
explanation of A. K2SO4 + Cr2(SO4)3 + 4H2O + 3O
14. (c) K2Cr2O7 is used as primary standard in C2H5OH + O CH3COOH
volumetric analysis because its standard
solution can be prepared. A is true but R is 18. (b) KCN forms complexes with Cu+ and Cd2+
wrong. as K 3 [Cu(CN) 4 ] and K 2 [Cd(CN) 4 ]
respectively. On passing H2S, only Cd2+
15. (d) Antimony sulphide (Sb2S3) is soluble in
complex is decomposed to give yellow CdS
yellow ammonium sulphide forming precipitate.
1 The Living World

TYPE A : MULTIPLE CHOICE QUESTIONS 7. The system of classification based on


evolutionary and genetic relationships among
1. Linnaeus system of classification is [1999]
organ isms, ignoring the morphological
(a) Natural (b) Artificial
similarities or differences, is called [2009]
(c) Phylogenetic (d) Progressive
(a) cladistics
2. The basic unit of classification is [2000]
(b) phenetics
(a) genus (b) species
(c) classical systematics
(c) variety (d) subspecies
(d) new systematics
3. Binomial nomenclature was introduced by
8. Scientific name of Mango plant is Mangifera
(a) Linnaeus [2000] indica (Linn.) Santapau. In the above name
(b) Darwin Santapau refers to [2012]
(c) Bentham and Hooker (a) Variety of Mango
(d) Aristotle (b) A taxonomist who proposed the present
4. The usage of binomial names, for plant species nomenclature in honour of Linnaeus
was accepted by all after the publication of the (c) A scientist who for the first time described
work by [2001] Mango plant
(a) Hooker (b) Linnaeus (d) A scientist who changed the name
(c) Bentham (d) Darwin proposed by Linnaeus and pr oposed
5. Which one of the following is correctly matched present name
regarding an Institute and its location? [2004] 9. The classification of Linnaeus was mainly based
(a) National Institute of Virology - Pune on [2012]
(b) National Institute of Communicable disease (a) Sepals (b) Carpels
- Lucknow
(c) Petals (d) Stamens
(c) Central Drug Research Institute - Kasauli
Institute 10. Which of the following is less general in
(d) National Institute of Nutrition - Mumbai characters as compared to genus? [2013]
6. “Ordines Anomali” of Bentham and Hooker (a) Species (b) Division
includes [2006] (c) Class (d) Family
(a) seed plants showing abnormal forms of 11. Read the following statements
growth and development. (i) Lower the taxon, more are the
characteristics that the members within the
(b) plants represented only in fossil state.
taxon share.
(c) plants described in the literature but which (ii) Order is the assemblage of genera which
Bentham and Hooker did not see in original. exhibit a few similar characters.
(d) a few orders which could not be placed (iii) Cat and dog are included in the same family
satisfactory in the classification. Felidae.
EBD_7100
B-2 Topicwise AIIMS Solved Papers – BIOLOGY
(iv) Binomial Nomenclature was introduced by (a) If both Assertion and Reason are correct and
Carolus Linnaeus. [2014] the Reason is a correct explanation of the
Which of the following statements are NOT Assertion.
correct? (b) If both Assertion and Reason are correct but
(a) (i), (ii) and (iii) (b) (ii), (iii) and (iv) Reason is not a correct explanation of the
(c) (i) and (iv) (d) (ii) and (iii) Assertion.
12. Choose the correct one [2015] (c) If the Assertion is correct but Reason is
(i) Growth cannot be taken as a defining incorrect.
(d) If both the Assertion and Reason are incorrect.
property of living organism.
(e) If the Assertion is incorrect but the Reason is
(ii) Dead organism does not grow.
correct.
(iii) Reproduction cannot be an all inclusive
14. Assertion: Living organisms are regarded as
defining characteristic of living organisms.
closed systems.
(iv) No non-livin g object is capable of
Reason: Energy of living organisms can not be
replicating itself.
lost or gained from external environment. [2002]
(v) Metabolism in a test tube is non-living.
15. Assertion: Systematics is the branch of biology
(vi) Metabolism is a defining feature of all living that deals with classification of living organisms.
organisms.
Reason: The aim of classification is to group
(a) (i) and (iii) (b) All except (v) the organisms. [2002]
(c) All except (iii) (d) All of these Directions for (Q. 16) : Each of these questions
13. Match column I with column II and choose the contains an Assertion followed by Reason. Read them
correct option. [2017]
carefully and answer the question on the basis of
Column-I Column-II
following options. You have to select the one that
A. Family I. tuberosum
best describes the two statements.
B. Kingdom II. Polymoniales
C. Order III. Solanum (a) If both Assertion and Reason are correct and
D. Species IV. Plantae Reason is the correct explanation of Assertion.
E. Genus V. Solanaceae (b) If both Assertion and Reason are correct, but
(a) A – IV; B – III; C – V; D – II; E – I Reason is not the correct explanation of
(b) A – V; B – IV; C – II; D – I; E – III Assertion.
(c) A – IV; B – V; C – II; D – I; E – III (c) If Assertion is correct but Reason is incorrect.
(d) A – V; B – III; C – II; D – I; E – IV (d) If both the Assertion and Reason are incorrect.
16. Assertion : Taxon and category are different
TYPE B : ASSERTION REASON QUESTIONS
things.
Directions for (Qs. 14-15) : These questions consist Reason : Category shows hierarchical
of two statements, each printed as Assertion and
classification. [2013, 14]
Reason. While answering these questions, you are
required to choose any one of the following five
responses.
The Living World B-3

Type A : Multiple Choice Questions “cladogram” suggests that the lengths of


the branches in the diagram are arbitrary,
1. (b) Carolus Linnaeus system of classification
while in a “phylogeny,” the branch lengths
is artificial.
indicate the amount of character change.
2. (b) The basic unit of classification is species
8. (d) Collection of rules regarding scientific
which consists of a natural population of
nomenclature of plants is known as ICBN
individuals having similar, morphological,
or International Code of Botanical
anatomical, biochemical, cytological and
Nomenclature. It was first accepted in 1961.
reproductive characters so that they can
According to ICBN, if any scientist has
interbreed freely and produce fertile
proposed wrong name then his name
offsprings.
should be written in bracket and the
3. (a) Carolus Linnaeus gave the binomial system
scientist who corrected the name should
of nomenclature.
be written after the bracket.
4. (b) Carolus Von Linnaeus in his book Genera
9. (d) Classification proposed by Linnaeus is
Plantarum made use of the artificial system
artificial. Linnaeus classified plant kingdom
of classification. He distinguished between
on the basis of only two characters
the natural and artificial systems. He used
(1) stamens (2) style
the binomial nomenclature system and
classified organisms into genus and 10. (a) Species is the lowest taxonomic category.
species. Class is a category made of one or more
5. (a) National Institute of Virology is located in related orders possessing similar correlated
Pune. characters. Family is composed of one to
The National Institute of Virology (NIV) is many related genera. Division comprises
a premier virus research laboratory in India. of several related classes.
It is one of the major Institutes of the Indian 11. (d) Order being higher category is the
Council of Medical Research (ICMR). It was assemblage of families which exhibit a few
established at Pune, similar characteristics.
6. (d) Bentham and Hooker system was
Dog (Canis familaris) and Cat (Felis
published well before there were
domesticus) belong to two different
internationally accepted rules for botanical
families—Cancideae and Felidae
nomenclature. In this system, termination
respectively.
for families was not so as it is now.
12. (b) Metabolism takes place only inside the
7. (a) “Cladogram” emphasizes that the diagram
body of a living organism.
represents a hypothesis about the actual
13. (b) A - V; B - IV; C - II; D - I; E - III
evolutionary relationships of a group. This
history is deduced on the basis of certain Type B : Assertion Reason Questions
shares like homologous characters. that are
thought to indicate common ancestry. 14. (d) Living organisms are regarded as open
While “phylogenies” represent true system as energy flow is the key function
evolutionary history. To other biologists, of an ecosystem.
EBD_7100
B-4 Topicwise AIIMS Solved Papers – BIOLOGY

15. (b) Systematics is the science of identification, repr esent an y level of grouping of
naming and classification of the organisms organisms based on certain common
into groups. characteristics. There is some confusion
16. (a) A category is a rank or level in the in the use of taxon and category, for example
hierarchial classification of organisms. Bryophyta is a taxon while division is a
Taxon is a unit in classification which may category.
2 Biological Classification

TYPE A : MULTIPLE CHOICE QUESTIONS 11. Powdery mildews of crops are caused by [2001]
(a) bacteria (b) ascomycetes
1. The symbiotic relationship of algae and fungus
is found in [1997] (c) basidiomycetes (d) phycomycetes
(a) lichen (b) mycorrhiza 12. Pullorum disease of poultry is caused by [2001]
(c) pneumatophore (d) bacteriophage (a) Mycobacterium (b) Salmonella
2. The virus that infects bacteria is called [1997] (c) Clostridium (d) Haemophilus
(a) cyanophage (b) mycophage 13. Viroids have [2003]
(c) bacteriophage (d) none of these (a) single stranded RNA not enclosed by
3. Citrus canker is caused by [1997] protein coat.
(b) single stranded DNA not enclosed by
(a) bacterium (b) fungus protein coat.
(c) alga (d) virus (c) double stranded DNA enclosed by protein
4. Which of the following is a cyanobacterium ? coat.
(a) Nostoc (b) Chara [1997] (d) double stranded RNA enclosed by protein
coat.
(c) E. coli (d) Polysiphonia
14. Myxomycetes are [2006]
5. Virus has [1998] (a) saprobes or parasites, having mycelia,
(a) DNA asexual reproduction by fragmentation.
(b) RNA (b) slimy mass of multinucleate protoplasm,
(c) both (a) and (b) having pseudopodia like structures for
(d) either DNA or RNA engulfing food, reproduction through
6. Hormogonia are vegetative reproductive fragmentation or zoospores.
structure of [1998] (c) prokaryotic organisms, cellular or acellular,
saprobes or autotrophic, reproduce by
(a) Spirogyra (b) Ulothrix
binary fission.
(c) Oscillatoria (d) Yeast (d) eukaryotic, single-celled or filamentous,
7. Which is a unicellular fungus ? [1998] saprobes or autotrophic, asexual
(a) Yeast (b) E.coli reproduction by division of haploid
(c) Nostoc (d) Albugo individuals, sexual reproduction by fusion
8. Bacterial cell wall is made up of [1999] of two cells or their nuclei.
(a) xylan (b) chitin 15. Among rust, smut and mushroom all the three
(c) cellulose (d) murein [2006]
(a) are pathogens (b) are saprobes
9. Cell wall is absent in [2000]
(c) bear ascocarps (d) bear basidiocarps
(a) Amoeba (b) Chara 16. "Fairy rings" in lawns result from
(c) yeast (d) E. coli outward,spreading circles of mycelia of
10. Which of the following is a fungus? [2000] mushrooms producing, at their periphery,
fruiting bodies called [2009]
(a) Nostoc (b) E. coli
(a) ascocarps (b) basidiocarps
(c) Yeast (d) Chara (c) sorocarps (d) pseudocarps
EBD_7100
B-6 Topicwise AIIMS Solved Papers – BIOLOGY
17. Which one of the following statements is The correct combination is –
correct? [2010] (a) A – II; B – I; C – IV; D – III
(a) Prions are the smallest free-living cells. (b) A – II; B – IV; C – I; D – III
(b) The cell wall of Mycoplasmas is made up (c) A – IV; B – I; C – II; D – III
of amino sugars.
(d) A – IV; B – III; C – II; D – I
(c) Viroids consist of single-stranded RNA
molecule. 22. Which of the following is correct about the slime
(d) Rickettsiae lack cell wall. mould ? [2015]
18. Bacterial cell wall is composed of peptidoglycan, (i) Its thalloid body, Plasmodium has
a complex of oligosaccharides and proteins. The pseudopodia for locomotion and engulfing
oligosaccharide component consist of [2010] organic matter.
(a) linear chain of alternating NAG and NAM (ii) During unfavourable conditions
linked by (1 – 4) linkage. Plasmodium differentiates and produces
(b) linear chains of alternating NAG and NAM fruiting bodies, sporangium.
linked by (1 – 4) linkage. (iii) Spores posses no true cell wall.
(c) linear chain of glucose linked by (1 – 4) (iv) They are dispersed by air current.
linkage. (v) Being extremely resistant, spores survive for
(d) linear chain of glucose linked by (1 – 4) many years.
likage. (vi) Plasmodium can grow upto several feet.
19. Choose the correct names of the different bacteria Choose the answer from the following options
according to their shapes. [2013]
(a) (i),(ii), (iv), (v) and (vi)
(b) (i),(ii) and (iii)
(c) (i),(ii), (iii) and (vi)
(d) (ii),(iii) and (vi)
23. (i) Green algae occur in fresh water, brackish
(a) A – Cocci, B – Bacilli, C – Spirilla, D – Vibrio water, salt water.
(b) A – Bacilli, B – Cocci, C – Spirilla, D – Vibrio (ii) Habitat of Brown algae-fresh water (rare),
(c) A – Spirilla, B – Bacilli, C – Cocci, D – Vibrio brackish water, salt water
(d) A – Spirilla, B – Vibrio, C – Cocci, D – Bacilli (iii) Some red algae are found in fresh water,
20. Which pair of the following belongs to mostly occur in salt water, some are in
Basidiomycetes? [2013] brackish water.
(a) Puffballs and Claviceps (iv) Most of the red algae are multicellular.
(b) Peziza and Alternaria (v) Red alga may occur in both well lighted
(c) Morchella and mushrooms regions close to water-surface and also at
(d) Birds nest fungi and puffballs great depths in oceans where light
21. Match column-I with column-II and select the penetration is little.
option . [2014] (vi) Cell wall of red algae consists of cellulose
Column-I Column-II + agar.
(Classes of fungi) (Common name) (vii) 2 – 8, equal and apical flagella in green
A. Phycomycetes I. Sac fungi algae [2015]
B. Ascomycetes II. Algal fungi (a) All are correct
C. Basidiomycetes III. Fungi imperfecti (b) All are false
D. Deuteromycetes IV. Club fungi (c) (i) and (vi) are correct
(d) (ii), (iii) and (v) are correct
Biological Classification B-7

24. Fungi are filamentous with the exception of "X" 28. Assertion : Bacteria have three basic shapes,
which is unicellular. Identify X. [2017] i.e., round , rod, spiral.
(a) Yeast (b) Albugo Reason : Cocci and Bacilli may form clusters
(c) Mucor (d) Lichen or chain of a definite length. [2000]
29. Assertion : Aflatoxins are produced by
25. Which of the following statements is not correct Aspergillus flavus.
for viruses? [2017] Reason : These toxins are useful to mankind.
(a) Viruses are obligate parasites. [2000]
(b) Viruses can multiply only when they are 30. Assertion : Escherichia coli, Shigella sp. and
inside the living cells. Salmonella sp. are all responsible for diarrhoeal
(c) Viruses cannot pass through bacterial diseases. [2006]
filters. Reason : Dehydration is common to all types of
(d) Viruses are made up of protein and DNA or diarrhoeal diseases and adequate supply of
RNA (never both DNA and RNA). fluids and electrolytes should be ensured.
26. Which of the following statements regarding 31. Assertion : Gram-negative bacteria do not retain
cyanobacteria is incorrect? [2017] the stain when washed with alcohol.
(a) It is also called blue green algae. Reason : The outer face of the outer membrane
(b) They are chemosynthetic heterotrophs. of gram-negative bacteria contains
(c) It forms blooms in polluted water bodies. lipopolysaccharides, a part of which is integrated
into the membrane lipids. [2006]
(d) It is unicellular, colonial or filamentous,
marine or terrestrial bacteria. 32. Assertion : Neurospora is commonly called
water mould.
TYPE B : ASSERTION REASON QUESTIONS Reason : It belongs to basidomycetes fungi.
Directions for (Qs. 27-32) : These questions consist [2007]
of two statements, each printed as Assertion and Directions for (Qs. 33-34) : Each of these questions
Reason. While answering these questions, you are contains an Assertion followed by Reason. Read them
required to choose any one of the following five carefully and answer the question on the basis of
following options. You have to select the one that
responses.
best describes the two statements.
(a) If both Assertion and Reason are correct and
(a) If both Assertion and Reason are correct and
the Reason is a correct explanation of the Reason is the correct explanation of Assertion.
Assertion. (b) If both Assertion and Reason are correct, but
(b) If both Assertion and Reason are correct but Reason is not the correct explanation of
Reason is not a correct explanation of the Assertion.
Assertion. (c) If Assertion is correct but Reason is incorrect.
(d) If both the Assertion and Reason are incorrect.
(c) If the Assertion is correct but Reason is
33. Assertion : Chemosynthesis is an autotrophic
incorrect.
nutrition.
(d) If both the Assertion and Reason are incorrect.
Reason : Chemoautotrophs contain chlorophyll
(e) If the Assertion is incorrect but the Reason is pigments. [2009]
correct. 34. Assertion : TMV is a virus which causes mosaic
27. Assertion: Viruses are not considered organism. disease.
Reason: Viruses are nucleoproteins and lack cell Reason : TMV has RNA as genetic material.
organelle, etc. [1998] [2001, 2017]
EBD_7100
B-8 Topicwise AIIMS Solved Papers – BIOLOGY

Type A : Multiple Choice Questions 8. (d) Bacterial cell wall consists of


peptidoglycan/murein/mucopeptide which
1. (a) Lichens are the symbiotic association contains polymers of modified sugars
between algae and fungi. Fungi provides (N-acetyl glucosamine and N-
the raw material and algae manufacture the acetylmuramic acid) cross linked by short
food.
peptides.
2. (c) The virus which attacks bacteria is called 9. (a) Cell wall is a characteristic of bacteria,
bacteriophage, e.g. T II phage, M13, 174 plants and fungi.
etc. 10. (c) Heterotrophs were the first to be evolved
and fungi in the plant kingdom are
3. (a) Citrus canker is caused by bacterium,
heterotrophs. Yeast with its characteristic
Xanthomonas citri. Bacterium is rod
absence of chlorophyll and its vegetative
shaped, motile, flagellate and gram
propagation th rough buddin g and
negative.
saprophytic nature shows it is a fungus.
4. (a) Nostoc is a cyanobacterium. Cyanobacteria 11. (b) Powdery mildews are caused by fungi
are gram (–) ve prokaryotes which perform belonging to the family Erysiphaceae
oxygenic photosynthesis like plants. (class-Ascomycetes). Ascomycetes
Cyanobacteria can be unicellular (e.g. spores are harmful and it makes the crop
Spirulina), colonial. (e.g. Nostoc) or vulnerable to its toxin. The stem becomes
filamentous (e.g. Oscillatoria) a powdery mass.
5. (d) Virus are obligate intracellular parasite 12. (b) Pullorum disease (Salmonellosis), a
which can reproduce only by invading and contagious disease of young birds, is
taking over other cells as they lack the caused by bacteria Salmonella pullorum.
cellular machinery for self reproduction. It is usually transmitted by infected hens
Virus has either DNA or RNA and never through their eggs. Its symptoms are loss
both. of appetite and thirst, diarrhoea with white
6. (c) Hormogonia is a part of a filament of a faeces, drooping of wings etc.
cyanobacterium that detaches and grows 13. (a) Viroids are the smallest known agents of
by cell division into a new filament. infectious diseases comprising of small
Cyanobacteria mostly multiply by asexual single stranded molecule of RNA. Viroids
means like binary fission, small fragments lack capsid and have no proteins
(hormogones), hormospores, akinetes, associated with them.
endospores etc. Common cyanobacteria 14. (b) Myxomycetes are acellular plasmodial
are-Nostoc, Anabaena, Oscillatoria etc. slime molds i.e. a group of 4 fungus which
7. (a) Yeast is the only member in kingdom fungi reproduces through fragmentation or
which is unicellular. zoospore.
Nostoc : Blue green algae 15. (d) Rust, smut and mushroom bear short lived
E. coli : Bacterium reproductive bodies called basidiocarps.
Albugo : Parasitic fungi
Biological Classification B-9

16. (b) Some basidiomycetes or club fungi (e.g., only replicate inside the cells of another
Agaricus campestris) produce organism. Viruses are not considered as
basidiocarps. These aerial structures also organisms as they have no independent
called mushrooms. They occur in rings machinery.
called 'fairy rings'. Basidiocarps of some 28. (e) Bacteria are microscopic organisms whose
fungi are edible. single cells have neither a membrane
17. (c) Viroid is a small infective segment of enclosed nucleus nor other membrane
nucleic acid, usually RNA. enclosed orgnelles like mitochondria and
Prion is an infectious pathogenic agent chloroplast. Bacteria possess various
devoid of nucleic acid and composed mainly forms and shapes like coccus (round),
of proteins. bacillus (rod shaped), vibrio (comma
Mycoplasmas lack a cell wall while shaped) and spirullum (spiral like cork
Rickettsiae possess a cell wall. screw).
18. (b) The oligosaccharide component of bacterial 29. (c) Aflatoxin, produced by Aspergillus flavus,
cell wall consists of linear chains of is a carcinogenic fungal toxin. The conidia
alternating NAG and NAM linked by not only causes allergies in human beings
(1 – 4) linkage. but also causes death of birds.
19. (a) 20. (d) 30. (b) E. coli, Shigella sp. and Salmonella sp.
21. (a) A – II; B – I; C – IV; D – III causes diarrhoea. It is due to intake of
22. (a) contaminated water.
23. (a) All the given statements about algae are
31. (a) Gram negative bacteria contain
correct.
lipopolysaccharide in its membrane.
24. (a) Yeast being a unicellular fungus does not
show filamentous nature. It is a microscopic 32. (d) Neurospora belongs to class ascomycetes
fungus consisting of a single oval cell that in which ascospores are produced in sac
reproduces by budding. like asci . Neurospora is commonly called
25. (c) Virus is a small infectious agent that as pink or red bread mould. It can be grown
replicates only inside the living cells of easily in laboratory conditions so, used in
other organisms. Viruses can infect all genetics experiments.
types of life forms, from animals and plants 33. (b) Chemoautotrophs developed the ability to
to microorganisms, including bacteria and synthesize organic molecules from
archaea. Viruses can pass through bacterial inorganic raw materials. Chlorophyll
proof filters as they are smaller than
pigments are present in photoautotrophs.
bacteria.
The photo-autotrophs utilize the solar
26. (b) Cyanobacteria are photosynthetic
energy absorbed by them in synthesizing
(containing a blue photosynthetic and
organic compounds. Certain chemo-
chemosynthetic pigment) autotrophs.
autotrophs include green sulphur bacteria.
They are prokaryotic and represent the
earliest known form of life on the Earth. 34. (b) In TMV, the chromosome consists of single
stranded, linear RNA molecule coiled into
Type B : Assertion Reason Questions a regular spiral. TMV causes mosaic
27. (a) Virus is a small infectious agent that can disease in tobacco and some other plants.
EBD_7100
B-10 Topicwise AIIMS Solved Papers – BIOLOGY

Chapter

3 Plant Kingdom
9. In Dryopteris, the opening mechanism of
TYPE A : MULTIPLE CHOICE QUESTIONS
sporangium is effectively operated by [1999]
1. Meiosis in Dryopteris takes place during [1997] (a) stalk (b) stomium
(a) spore formation (c) annulus (d) peristome
(b) gametic formation 10. 13 celled male gametophyte of Selaginella has
(c) spore germination [2000]
(d) zygote formation (a) 12 cells of antheridium + 1 prothallial cell
2. Sometimes a ladder like structure in Spirogyra (b) 10 cells of antheridium + 3 prothallial cell
is formed due to [1998] (c) 9 cells of antheridium + 4 prothallial cell
(a) lateral conjugation (d) 8 cells of antheridium + 6 prothallial cell
(b) asexual reproduction 11. Which of the following is found in algal zone of
Cycas coralloid roots? [2000]
(c) scalariform conjugation
(a) Blue green algae (b) Red algae
(d) direct conjugation
(c) Diatoms (d) Brown algae
3. In which group of the following would you place
12. Sex organs in Funaria develop [2001]
the plants having vascular tissue and lacking
(a) in protonema.
seeds? [1998]
(b) outside capsule.
(a) Algae (b) Fungi
(c) in the axil of leaf.
(c) Bryophytes (d) Pteridophytes
(d) at the tip of gametophore.
4. Largest ovule in plant kingdom are found in
13. In which portion of Cycas diploxylic vascular
(a) Pinus (b) Cycas [1998] bundles are found ? [2001]
(c) Thuja (d) Gnetum (a) Root (b) Stem
5. Plant material, which is used in culture medium (c) Leaflet (d) Rachis and leaflet
is obtained from [1998] 14. Funaria gametophyte is [2001]
(a) Cycas (b) coconut milk (a) dioecious
(c) Pinus (d) mango (b) heteroecious
6. Which is a saprophytic angiosperm? [1998] (c) autoecious
(a) Cuscuta (b) Neottia (d) monoecious and autoecious
(c) Agaricus (d) Yeast 15. Sometimes, the fern plant arises from fern
7. Bryophytes do not possess [1999] prothallus without fertilization. This is an example
(a) vascular tissue of [2001]
(a) apospory (b) apogamy
(b) gametophyte
(c) parthenocarpy (d) gametogenesis
(c) alternation of generation
16. Zygospore of Spirogyra at the time of meiosis
(d) spores
is divided into 4 nuclei. How many nuclei
8. The heart shaped fern prothallus is [1999] degenerate out of these four ? [2001]
(a) gametophyte (b) sporophyte (a) One (b) Two
(c) saprophyte (d) gamete (c) Three (d) Four
Plant Kingdom B-11

17. Cycas is [2001] 26. In the following table identify the correct
(a) monoecious (b) bisexual matching of the crop, its disease and the
(c) dioecious (d) hermaphrodite corresponding pathogen [2006]
18. The drug bellodona is obtained from [2001] Crop Disease Pathogen
(a) Atropa (b) Opium (a) Citrus Canker Pseudomonas
(c) Rauwolfia (d) Solanum rubrilineans
19. Algae are useful because they [2002] (b) Potato Late blight Fusarium udum
(a) purify the atmosphere (c) Brinjal Root-knot Meloidogyne
(b) are large in number incognita
(c) are used in fermentation (d) Pigeon pea Seed gall Phytophthora
(d) are used to study respiration infestans
20. The plant body of Funaria is [2002]
27. Plants of which one of the following groups of
(a) sporophyte
genera are pollinated by the same agency?
(b) gametophyte
(a) Triticum, Cocos, Mangifera [2006]
(c) predominantly sporophyte with
(b) Ficus, Kigelia, Casuarina
independent gametophyte
(c) Salvia, Morus, Euphorbia
(d) predominantly gametophyte with
(d) Bombax, Butea, Bauhinia
dependent sporophyte.
28. Which of the following match is correct? [2007]
21. Elaters help in dispersal of spores of [2002]
Disease Pathogen
(a) Riccia (b) Marchantia
(a) Wilt disease Synchytrium
(c) Dryopteris (d) Funaria
(b) Citrus canker Xanthomonas
22. The nature of megasporophyll of Cycas is similar (c) Red rot of sugarane Ustilago
to [2002]
(d) Powdery mildew Fusarium
(a) stamen (b) carpel
29. People recovering from long illness are often
(c) sepal (d) petal advised to include the alga Spirulina in their
23. Which of the following helps in respiration of diet because it [2003, 2008]
lichens? [2002] (a) makes the food easy to digest.
(a) Isidia (b) Soredia (b) is rich in proteins.
(c) Cyphella (d) Cephalodia (c) has antibiotic properties.
24. In Ulothrix, meiosis takes place in [2004] (d) restores the intestinal microflora.
(a) cells of the filament 30. Botanical name of Sanjeevani is [2009]
(b) holdfast (a) Selaginella chrysocaulos
(c) zygote (b) Selaginella bryopteris
(d) zoospores (c) Selaginella chrysorhizos
25. Mosses and ferns are found in moist and shady (d) None of the above
places because both [2004] 31. In Pinus, the third tier of embryonal cells from
(a) require presence of water for fertilization. below is known as [2009]
(b) do not need sunlight for photosynthesis. (a) rosette tier (b) suspensor tier
(c) embryonal tier (d) free-nuclear tier
(c) depend for their nutrition on micro-
organisms which can survive only at low 32. Which of the following is found in algal zone of
Cycas coralloid roots? [2007, 2011]
temperature.
(a) Blue green algae (b) Red algae
(d) cannot compete with sun-loving plants.
(c) Diatoms (d) Brown algae
EBD_7100
B-12 Topicwise AIIMS Solved Papers – BIOLOGY
33. Which among the following is a rootless (a) (i) is correct, but (ii) and (iii) are incorrect
plant? [2007, 2011] (b) (i) and (ii) are correct, but (iii) is incorrect
(a) Nymphaea (b) Sagittaria (c) (i) and (iii) are correct, but (ii) is incorrect
(c) Ceratophyllum (d) Vallisneria
(d) (iii) is correct, but (i) and (ii) are incorrect
34. Sporocarp is a reproductive structure of
(a) Some algae [2012] TYPE B : ASSERTION REASON QUESTIONS
(b) Some aquatic ferns having sori Directions for (Qs. 39-43) : These questions consist
(c) Angiosperms having spores of two statements, each printed as Assertion and
(d) Bryophytes Reason. While answering these questions, you are
35. Coralloid roots of Cycas is distinguished from
required to choose any one of the following five
angiosperm roots by [2013]
responses.
(a) absence of pith
(b) having xylem tissue (a) If both Assertion and Reason are correct and
(c) absence of algal zone the Reason is a correct explanation of the
(d) presence of algal zone Assertion.
36. If the cells of root in wheat plant have 42 (b) If both Assertion and Reason are correct but
chromosomes, then the no. of chromosome in Reason is not a correct explanation of the
the cell of pollen grain is [2014] Assertion.
(a) 14 (b) 21
(c) If the Assertion is correct but Reason is
(c) 28 (d) 42
37. Match the following incorrect.
Column-I Column-II (d) If both the Assertion and Reason are incorrect.
(Classes) (Examples) (e) If the Assertion is incorrect but the Reason is
A. Psilotopsida I. Dryopteris, Pteris,
correct.
Adiantum
B. Lycopsida II. Equisetum 39. Assertion : Mosses are evolved from algae.
C. Sphenopsida III. Selaginella Reason : Protonema of mosses is similar to some
D. Pteropsida IV. Lycopodium green algae. [2001]
V. Psilotum 40. Assertion : Coconut tree is distributed in coastal
(2016) areas over a large part of the world.
(a) A – V; B – III; C – II; D – I
Reason : Coconut fruit can float and get
(b) A – I; B – II; C – III; D – IV
dispersed over thousands of kilometers before
(c) A – IV; B – III; C – II; D – I
(d) A – III; B – V; C – I; D – II losing viability. [2004]
38. Consider the following statements regarding the 41. Assertion : The fungi are widespread in
major pigments and stored food in the different distribution and they even live on or inside other
groups of algae and choose the correct option plants and animals.
(i) In chlorophyceae, the stored food material Reason : Fungi are able to grow anywhere on
is starch and the major pigments are land, water or on other organisms because they
chlorophyll-a and d.
have a variety of pigments, including chlorophyll,
(ii) In phaeophyceae, laminarian is the stored
food and major pigments are chlorophyll-a carotenoids, fucoxanthin and phycoerythrin.
and b. [2005]
(iii) In rhodophyceae, floridean starch is the 42. Assertion : Algae and fungi are classified as
stored food and the major pigments are
thallophytes.
chlorophyll-a, d and phycoerythrin.
[2017] Reason : They both are autotrophs. [2007]
Plant Kingdom B-13

43. Assertion : Conifer trees produce a large 44. Assertion : Red algae contributes in producing
quantity of wind borne pollen grains. coral reefs. [2004, 2011]
Reason : The pollen grains have wings. [2007] Reason : Some red algae secrete and deposit
Directions for (Qs. 44-46) : Each of these questions calcium carbonate over their walls.
contains an Assertion followed by Reason. Read them 45. Assertion: The peristome is a fringe of teeth-
carefully and answer the question on the basis of like projections found at the mouth of the
following options. You have to select the one that capsule. [2004, 2011]
best describes the two statements.
Reason: It may be of two types nematodontous
(a) If both Assertion and Reason are correct and and orthodontus.
Reason is the correct explanation of Assertion.
46. Assertion : Members of phaeophyceae vary in
(b) If both Assertion and Reason are correct, but
colour from olive green to various shades of
Reason is not the correct explanation of
brown
Assertion.
(c) If Assertion is correct but Reason is incorrect. Reason : Phaeophceae possess chlorophyll a,
(d) If both the Assertion and Reason are incorrect. c, carotenoids and xanthophysis. [2016]
EBD_7100
B-14 Topicwise AIIMS Solved Papers – BIOLOGY

Type A : Multiple Choice Questions The sperms of Cycas are also the largest
(300 µm) in the biological world.
1. (a) In sporangium, 5. (b) Coconut milk is rich in compounds
Meiosis chemically similar to cytokinin and promote
Spore mother cell (2n)
callus growth by inducing cell division.
dispersed by 6. (b) Neottia grows on soil rich humus. It has
spores (n) germination
an underground stem with a cluster of roots.
Prothallus(n) gamete fusion The roots are associated with endotrophic
mycorrhiza. The fungus absorbs its food
zygote Main plant body from humus and the cortical cells of the
(2n). roots of Neottia absorb food from humus.
2. (c) Conjugation is exchange of genetic material Neottia is actually a parasite on fungus.
i.e. mode of sexual reproduction in lower 7. (a) Bryophytes are terrestrial plants found in
organisms. Conjugation occurs by two moist places and shady localities .They are
methods– rightly called the amphibians of the plant
(i) Scalariform conjugation kingdom .Since fertilization does not occur
It takes place during the night and without water. Being in moist places, water
between the recently divided cells of and mineral salts are being absorbed by
the conjugating participating rhizoids that have the main function of
filaments. Opposite cells of two fixing the plant. Absorption also occurs due
filaments develop conjugation tube to diffusion. Moreover, these plants are
by lying side by side and parallel to very small. Hence, Bryophytes do not
each other. The conjugation tube possess the vascular tissue.
between two filament looks like a 8. (a) The heart shaped prothallus is the
ladder, through which gamete from characteristic feature of ferns and mainly
one gametangia passes through to belong to polypodiaceae.They show
fuse with the passive gamete of polarity and dorsoventrality. The
another filament. prothallus contains both the antheridia and
(ii) Lateral conjugation archegonia which are embedded in the
prothallus. Since it forms the male and
Two adjacent cells of same filament
female gametes the prothallus is
function as male and female cells. It is
considered as the gametophyte.
commonly reported in S. affinis and
9. (b) When the capsule of the sporangium in
S. fenuissima.
Dryopteris matures, about 4 lower median
3. (d) Pteridophytes are seedless vascular plants.
cells of the jacket stretch tangentially. The
They are also called vascular cryptogams
two median ones identify the place from
as they possess xylem and phloem.
where the capsule opens. This is called as
4. (b) Largest ovule in plant kingdom is found in stomium. The other cells of the jacket
Cycas. Female cone consists of develop a thickening . At maturity, the
megasporophylls. Each megasporophyll indusium dries exposing the sorus. Due to
bears 2-12 reddish ovules in the middle loss of water in the annulus, the upper walls
fertile part. Ovules of Cycas are the largest, contract and inner ones straighten, the
6-7 cm in diameter. annulus coils. This feature exerts pressure
Plant Kingdom B-15

on the wall resulting in the breaking of the to germination, the diploid zygospore
capsule, between the cells of the stomium, nucleus undergoes meiosis and forms four
thereby, releasing the spores. nuclei. Three of the four nuclei degenerate
10. (a) The structure and development of male since they are smaller in size. The last cell
gametophyte was described by Slagg is bigger and grows using the reserve food
(1932). The 13 celled male gametophyte has substances.
12 antheridial cells and one prothallial cell 17. (c) Cycas is dioecious in nature since it
as a result of segmentation. This is to produces micro and megaspores in the
increase the number of biflagellate micro and megasporangia. The micro and
antherozoids. megasporangia are born on
11. (a) The algal zone consists of blue green algae microsporophyll and megasporophyll
in the coralloid roots. The relationship is a respectively.
mutual relationship. The algae get the 18. (a) Atropa bellandona produces the drug
dwelling place, while Cycas can use the belladona. It is an alkaloid which is atropine
food produced by the algae. and hyoscyamine. The leaves and root
12. (d) Sex organs in Funaria develop at the tip of contain the alkaloid. It is used to cure and
gametophore since the plant itself relieve pain.
constitutes the gametophytic phase. After 19. (a) Algae is a collective term for all those
fertilization develops the capsule from chlorophyll bearing organisms which are
which protonema develops from the spores thalloid. So, during photosynthesis by
that dehisces from the capsule. The capsule taking in CO2 and giving out O2, they purify
constitutes the sporophytic phase. The the atmosphere.
plant shows an alternation of generation,
20. (d) In the life cycle of Funaria, two distinct
between the sporophytic and
individuals occur. One of these is haploid
gametophytic phase.
and is independent leafy plant. The other
13. (d) The rachis and leaflet have diploxylic
is diploid, leafless sporogonium. It is
vascular bundles. At the region of the stem
partially dependent on the leafy
cortex, the leaf trace has only centrifugal
gametophyte for its nutrition.
xylem, hence, it is endarch. These traces
21. (b) The diploid bispiral elaters are hygroscopic
enter the leaf as the rachis separates, the
and they help in the dispersal of spores.
centripetal xylem appears and it is in the
exarch condition. The leaflet has a single 22. (b) Megasporophyll of Cycas bears ovules
midrib bundle and is in the diploxylic similar to carpel.
condition. 23. (c) Cyphella are aerating organs in the form of
14. (d) Funaria is both monoecious and organised breaks, which occur in the lower
autoecious. The antheridium (male sex cortex and appear as concave circular
organ) are born on different heads of the depression where white medulla is exposed.
same plant. Hence, it is both autoecious 24. (c) Ulothrix being an haploid organism, shows
and monoecious. zygotic meiosis. Zygote is the only diploid
15. (b) Apogamy is the formation of sporophyte structure which undergoes meiosis to form
from a gametophytic cell other than egg four cells. Each develops into Ulothrix.
without fertilization. The prothallus is only 25. (a) Mosses and ferns are called the amphibians
a part of gametophytic cell. Hence, the of plants which are land plants but require
development is apogamy. a film of water for antherozoids to reach
16. (c) The united protoplasmic mass of two archegonium.
gametes is called zygote or zygospore. Prior
EBD_7100
B-16 Topicwise AIIMS Solved Papers – BIOLOGY
26. (c) Root knot/root gall in brinjal is caused by 32. (a) The algal zone consists of blue green algae
nematode Meloidogyne incognita. in the coralloid roots. The relationship is a
27. (c) Salvia, Morus, Euphorbia are pollinated mutual relationship. The algae get the
by insects (entomophily). dwelling place, while cycas can use the
28. (b) Citrus canker is the disease caused by an food produced by the algae.
aerobic, r od shaped bacterium, 33. (c) Nymphaea, Sagittaria, Ceratophyllum
Xanthomonas citri. and Vallisneria are hydrophytes which
29. (b) Some dieters say that blue green algae grow in water. Nymphaea, Sagittaria and
helps satisfy appetite and supplies good Vallisnaria are rooted hydrophytes while
Ceratophyllum is a submerged floating
nutrition while dieting. Some dieters say
hydrophyte. It remains completely under
that blue-green algae Spirulina, a well-
water but is not rooted in mud.
known example, is a group of 1,500 species
Ceratophyllum lacks roots even in
of microscopic aquatic plants. The two
embryonic stage.
most common species used for human
consumption are Spirulina maxima and 34. (b) Sporocarp is a reproductive structure of
Spirulina platensis. Spirulina is particularly some aquatic ferns like Marsilea.
rich in protein and also contains Sporocarps are functionally and
carotenoids, vitamins, minerals, and developmentally modified leaf which bears
essential fatty acids. Blue-green algae several sori. Each sori consists of
(Spirulina) is a concentrated source of sporangia of two types -megasporangia
nutrients compared to most foods, but it is producing simple large female megaspore
expensive compared to other supplemental and microsporangia producing many small
male microspore.
sources of protein, vitamins, and minerals.
30. (b) Selaginella bryopteris (L.), popularly 35. (d) In Cycas, coralloid roots are bluish green
known as Sanjeevani, is a plant with in colour and have blue-green algae in their
medicinal properties. Sanjeevani grows on cortex. This character distinguishes them
the hills of tropical areas, particularly the from angiosperms.
Arawali mountain terrains from east to west 36. (b) Number of chromosome in root cells of
in India. Traditionally plants have been used wheat plant is
to: (i) relief from heat stroke and the burning
2n = 42
sensation during urination; (ii) restorating
menstrual irregularities to normal and applied 42
n= = 21
topically to pregnant women, aids easy 2
delivery; and (iii) for treating jaundice. The number of chromosome in the cell of
31. (a) In Pinus, the third tier of embryonal cells pollen grain is 21 because pollen grains are
from below is known as rosette tier. The haploid (n) in nature.
four lower most cells which are far from the 37. (a)
micropolar end are called the embryonal tier.
These develop into an embryo. The cells 38. (d) In chlorophyceae, the stored food material
above the embryonal tier called the is starch and the major pigments are
suspensor tier and the third tier from below chlorophyll- a and b. In phaeophyceae,
laminarian is the stored food and major
is called the rosette tier. The free nuclei tier
pigments are chlorophyll a, c and
are formed of 4 nuclei. It is the fourth upper
most tier. fucoxanthin.
Plant Kingdom B-17

Type B : Assertion Reason Questions 44. (a) Red algae generally grow attached to rocky
stones. Some deep water red algae are
39. (a) Mosses that belong to the bryophytes
calcareous and build up hard stony thalii
have evolved from algae. The fact that
responsible for the production of lime
protonema has a thallus like body shows
stones and coral reefs.
that mosses have evolved from algae.
45. (b) The peristome teeth are present at the
40. (a) Coconut tree is distributed in coastal areas
mouth of capsule. The teeth may be solid
since it floats on saline water because salt
is more denser than freshwater, so it floats. cellular tissue or composed only of the
thickened portions of the cell walls of
41. (c) Fungi lack pigments and are therefore,
adjacent cells. When the teeth of peristome
heterotrophic. The mode of nutrition is
are solid structures composed of bundles
either parasitic/saprophytic.
of dead cells, it is termed nematodontous
42. (c) Thallophyte includes plants in which body peristome are solid structures composed
is not differentiated into root, stem and
of bundles of dead cells, it is termed
leaves. Algae and fungi have thallose plant
nematodyntous peristome. It is found in
body. Algae are autotrophs i.e. they can
polytrichum, Pogonatum and Tetraphis. If
prepare their own food by photosynthesis.
peristome composed of thin, membranous,
while fungi are heterotrophs.
transversely barred teeth, each tooth is
43. (a) In pinus, a conifer tree, the microspores are made up of the thickened portions of the
produced by microsporogenesis in
cell walls of adjacent cells. Such a peristome
microsporangium. Each microsporangium
is called orthodontous.
has an inner nourishing layer known as
tapetum. A large number of dusty and two 46. (a) Phaeophyceae possesses chlorophyll a, c,
winged microspores are present. On carotenoids and xanthophylls. Members of
maturity the microsporangium wall bursts phaeophyceae show variations in colour
and microspores are released in the air which from olive green to different shades of
is called "shower of sulphur."They are brown depending upon the amount of
dispersed by wind due to presence of wings. xanthophyll pigments.
EBD_7100
B-18 Topicwise AIIMS Solved Papers – BIOLOGY

4 Animal Kingdom

TYPE A : MULTIPLE CHOICE QUESTIONS 10. The long bones are hollow and connected by
air passage. They are characteristic of [1998]
1. Pouched mammals are known as [1997] (a) aves (b) mammalia
(a) prototherians (b) metatherians (c) reptilia (d) sponges
(c) eutherians (d) therians 11. Aristotle’s lantern is found in [1998]
2. Sponges capture food with the help of [1997] (a) Asteroidea (b) Echinoidea
(a) pinacocytes (b) choanocytes
(c) Holothuroidea (d) Ophiuroidea
(c) trophocytes (d) theocytes
12. Sympathetic nerves in mammals arise from
3. Chloragogen cells of earthworm are similar to
(a) sacral region [1998]
which organ of vertebrates ? [1997]
(a) Liver (b) Lung (b) cervical region
(c) Kidney (d) Spleen (c) 3rd, 7th, 9th, 10th cranial nerves
4. Haemocoel is found in [1997] (d) thoracico-lumbar region
(a) Hydra and Aurelia 13. Which of the following statement is true
(b) Taenia and Ascaris regarding corals ? [1999]
(c) Cockroach and Pila (a) Form branch colonies.
(d) Herdmania and Balanoglossus (b) Solitary or colonial.
5. Aquatic reptiles are [1997] (c) Grow as massive bodies.
(a) ureotelic (d) All of the above
(b) ureotelic on land 14. Water current in Leucosolenia is produced
(c) ammonotelic by [1999]
(d) uricotelic in water (a) pinacocytes (b) choanocytes
6. Larva of house-fly lacks [1997] (c) archaeocytes (d) collencytes
(a) eyes (b) wings 15. Which is the example of platyhelminthes ?[1999]
(c) spiracles (d) none of these (a) Entamoeba (b) Plasmodium
7. Single filament of Nostoc without mucilage
(c) Wuchereria (d) Schistosoma
sheath is called as [1998]
16. Paired spermathecae occur in Pheretima in
(a) colony (b) mycelium
which of the following segments ? [1999]
(c) trichome (d) hyphae
(a) 4, 5, 6, 7 (b) 5, 6, 7, 8
8. Organisms, attached to substratum generally
possess [1998] (c) 6, 7, 8, 9 (d) 3, 4, 5, 6
(a) radial symmetry 17. Weberian ossicles are found in [1999]
(b) asymmetrical body (a) frog (b) snakes
(c) single opening of digestive canal (c) fishes (d) birds
(d) cilia to create water current 18. The vertebrae in birds are mostly [1999]
9. Hydra recognizes its prey by [1998] (a) procoelous (b) heterocoelous
(a) nematocyst (c) amphicoelous (d) acoelous
(b) special organ 19. Basket star belongs to class [1999]
(c) chemical stimulus (a) Ophiuroidea (b) Echinoidea
(d) mechanical stimulus (c) Crinoidea (d) Asteroidea
Animal Kingdom B-19

20. The egg case in female cockroach is formed by 33. What is left, when bath sponges dries up?[2002]
secretion of [2000] (a) Spicules (b) Hold fast
(a) collaterial gland (b) mushroom gland (c) Spongin fibres (d) Tentacles
(c) conglobate gland (d) prothoracic gland 34. Hydra receives impulses and stimuli
21. Power of regeneration in sponges is due to through [2002]
[2000] (a) nerve cells (b) sensory cells
(a) theocytes (b) archaeocytes (c) neuron cell (d) nematocysts
(c) amoebocytes (d) sclerocytes 35. Which of the following are uricotelic animals?
22. The poisonous fluid present in nematocyst of (a) Rohu, Frog [2002]
Hydra is [2000] (b) Camel, Frog
(c) Lizard, Crow
(a) toxin (b) venom
(d) Eagles, Earthworm
(c) hematin (d) hypnotoxin
36. In Entamoeba histolytica, the presence of
23. Life cycle of Taenia is [2000]
chromatid bodies is characteristic of [2002]
(a) monogenetic (b) digenetic (a) precystic stage
(c) polygenetic (d) hexogenetic (b) trophozoite stage
24. Pigment haemocyanin is found in [2000] (c) mature binucleate stage
(a) chordata (b) annelida (d) both (a) and (b)
(c) porifera (d) mollusca 37. Just as Xenopsylla is a Yersenia pestis, so is
25. Antedon belongs to which of the following [2003]
class? [2000] (a) Glossina palpalis to Wuchereria bancrofti
(a) Asteroidea (b) Ophiuroidea (b) Culex to Plasmodium falciparum
(c) Crinoidea (d) Echinoidea (c) Homo sapiens to Taenia solium
26. Scales in Chondrichthyes are [2000] (d) Phlebotomus to Leishmania donovani
(a) placoid (b) ctenoid 38. Which of the following feature is common to
(c) cycloid (d) all of these leech, cockroach and scorpion? [2004]
27. Which of the following snake is not poisonous? (a) Nephridia (b) Ventral nerve cord
(a) Naja- Naja (b) Python [2000] (c) Cephalization (d) Antennae
(c) Hydrophis (d) Bungarus 39. Which one of the following statements is correct
28. Birds are [2000] with respect to salt water balance inside the body
(a) cold blooded (b) homeothermal of living organisms? [2005]
(c) poikilothermal (d) homeopoiesis (a) When water is not available camels do not
29. Which of the following substances is at its produce urine but store urea in tissues.
lowest level in fish food ? [2000] (b) Salmon fish excretes lot of stored salt
through gill membrane in fresh water.
(a) Actin (b) Myosin
(c) Paramecium discharges concentrated salt
(c) Cholesterol (d) Tissue fluid
solution by contractile vacuoles.
30. How many ovaries are found in birds ? [2000]
(d) The body fluids of fresh water animals are
(a) One (b) Two generally hypotonic to surrounding water.
(c) Three (d) Many 40. Which one of the following groups of
31. Gemmule formation in sponges are useful in structures/organs have similar function?
(a) asexual reproduction [2001] [2005]
(b) sexual reproduction (a) Typhlosole in earthworm, intestinal villi in
(c) parthenogenesis rat and contractile vacuole in Amoeba.
(d) parthenocarpy (b) Nephridia in earthworm, Malpighian
32. The places of first, second and third moulting of tubules in cockroach and urinary tubules
Ascaris larva are [2002] in rat.
(a) soil, alveoli, lung (c) Antennae of cockroach, tympanum of frog
(b) liver, soil, stomach and clitellum of earthworm.
(c) soil, lung, liver (d) Incisors of rat, gizzard (proventriculus) of
(d) soil, intestine, lung cockroach and tube feet of starfish.
EBD_7100
B-20 Topicwise AIIMS Solved Papers – BIOLOGY
41. Which one of the following features is common (c) Morphallaxis - Healing up of a wound
in silverfish, scorpion, dragonfly and prawn? in the skin.
[2005] (d) Epimorphosis - Regeneration of
(a) Three pairs of legs and segmented body. crushed and filtered
(b) Chitinous cuticle and two pairs of antennae. out pieces of a
(c) Jointed appendages and chitinous Planaria into as many
exoskeleton. new Planarians.
(d) Cephalothorax and trachae. 47. Which statement best explains why
42. Which one of the following animals is correctly invertebrates regenerate lost tissue more readily
matched with its one characteristic and the than most vertebrates do? [2009]
taxon? [2006]
(a) Invertebrates contain specialized cells that
Animal Characteristic Taxon produce the hormones necessary for this
(a) Millipede Ventral nerve Arachnida process.
cord (b) Invertebrate cells exhibit a higher degree
(b) Duckbill Oviparous Mammalia of uncontrolled cell division than
vertebrate cells do.
platypus (c) Invertebrate animals reproduce asexually,
(c) Silverfish Pectoral & Chordata but vertebrate animals reproduce sexually.
Pelvic fins
(d) Invertebrate animals have more
(d) Sea anemone Triploblastic Cnidaria
undifferentiated cells than vertebrate
43. All mammals without any exception are animals have.
characterized by [2006] 48. Which of the following is correctly matched?
(a) viviparity and biconcave red blood cells. (a) Human -Renal portal system [2009]
(b) extra-abdominal testes and a four- (b) Earthworm - Closed circulatory system
chambered heart. (c) Cockroach - Nephridia
(c) heterodont teeth and 12 pairs of cranial (d) None of the above
nerves.
49. Which set of terms would most likely be used in
(d) a muscular diaphragm and milk producing
a description of the nervous system of
glands.
chordates? [2009]
44. Which of the following does not come under
the class mammals? [2007] (a) Brain, dorsal nerve cord, highly developed
(a) Flying fox (b) Hedgehog receptors
(c) Manatee (d) Lamprey (b) Brain, fused ganglia, ventral nerve cord
45. Which one feature is common to leech, (c) No brain, fused ganglia, tympana
cockroach and scorpion? [2008] (d) No brain, nerve net, modified neurons
(a) Nephridia (b) Ventral nerve cord 50. Which one of the following statements is not
(c) Cephalization (d) Antennae correct? [2010]
46. Based on cellular mechanisms there are two major (a) All echinoderms are viviparous.
types of regeneration found in the animals.
(b) Roundworm has no circulatory system.
Which one of the following is the correct
example of the type mentioned? [2008] (c) In bony fishes, swim bladder is usually
(a) Morphallaxis - Regeneration of two present.
transversely cut equal (d) In cartilaginous fishes, fertilization is
pieces of a Hydra into internal.
two small hydras 51. Ticks and mites are actually [2010]
(b) Epimorphosis - Replacement of old and (a) arachnids (b) crustaceans
dead erythrocytes by (c) insects (d) myriapods
the new ones.
Animal Kingdom B-21

52. "Portuguese man of war" is [2012] 55. Cockroaches are brown or black bodied animals
(a) Soldier of world war I that are included in class _______ of phylum
_______. [2014]
(b) Portuguese soldier
(a) Reptilia; Annelida
(c) A sponge (b) Insecta; Arthropoda
(d) A polymorphic, colonial, coelenterata (c) Insecta; Annelida
53. Which are exclusively viviparous ? [2012] (d) Reptilia; Arthropoda
(a) Bony fishes Ectoderm
56. Mesoglea
(b) Cartilagenous fishes Endoderm
(c) Sharks
(d) Whales
54. Given are the four matches of phyla with their
characteristic cells [2013]
A. Coelenterata - Nematocytes Mesoderm
B. Porifera - Choanocytes A B

C. Ctenophora - Solenocytes The above diagram shows the germs layer. The
animals having structures shown in the figures A
D. Platyhelminthes - Nephrocytes
and B are respectively called [2014]
Mark the option that has both correct matches (a) Diploblastic, Triploblastic
(a) A and B (b) B and C (b) Triploblastic, Diploblastic
(c) C and D (d) B and D (c) Diploblastic, Diploblastic
(d) Triploblastic, Triploblastic

57. Given below are pie diagrams A, B and C related to proportionate number of species of major taxa of
invertebrates, vertebrates and plants respectively. Critically study and fill in the blanks I, II, III and IV

Other animal
groups Mosses
Mammals
Ferns and
Crustaceans Fishes Birds allies

I Insects III
IV
II Reptiles
Lichens

A B Algae C
Invertebrates Vertebrates Plant

[2015]
(a) I- Molluscs, II-Amphibians, III-Fungi, IV-Angiosperms
(b) I- Molluscs, II-Amphibians, III-Angiosperms, IV-Fungi
(c) I- Hexapoda, II-Amphibians, III-Fungi, IV-Angiosperms
(d) I- Turtles, II-Amphibians, III-Fungi, IV-Angiosperms
EBD_7100
B-22 Topicwise AIIMS Solved Papers – BIOLOGY
58. Which of the following statements are true/false? TYPE B : ASSERTION REASON QUESTIONS
(i) In Torpedo, the electric organs are capable
of generating strong electric shock to Directions for (Qs. 61-69) : These questions consist
paralyze the prey. of two statements, each printed as Assertion and
(ii) Bony fishes use pectoral, pelvic, dorsal anal Reason. While answering these questions, you are
and caudal fins in swimming.
required to choose any one of the following five
(iii) Amphibian skin is moist and has thick
scales. responses.
(iv) Birds are poikilothermic animals. (a) If both Assertion and Reason are correct and
(v) The most unique mammalian characteristic the Reason is a correct explanation of the
is the presence of milk producing mammary Assertion.
glands by which the young ones are
nourished. [2015] (b) If both Assertion and Reason are correct but
(a) (i), (ii) and (iii) are true; (iv), (v) are false Reason is not a correct explanation of the
(b) (i), (ii) and (v) are true; (iii) and (iv) are false Assertion.
(c) (i), (iv) and (v) are true; (ii) and (iii) are false
(c) If the Assertion is correct but Reason is
(d) (i), (ii) and (iv) are false; (iii) and (v) are true
59. Column-I contains organisms and column-II incorrect.
contains their excretory structures. Choose the (d) If both the Assertion and Reason are incorrect.
correct match form the options given below. (e) If the Assertion is incorrect but the Reason is
Column- I Column -II
correct.
(Organism) (Excretory
structures) 61. Assertion : Cold blooded animals do not have
A. Cockroach I. Nephridia fat layer.
B. Cat fish II. Malpighian Reason : Cold blooded animals use their fat for
tubules metabolic process during hibernation. [1997]
C. Earthworm III. Kidneys 62. Assertion : Birds have one ovary.
D. Balanoglossus IV. Flame cells
E. Flatworm V. Proboscis gland Reason : This reduces the body weight for flight.
[2017] [1999]
(a) A – I; B – III; C – II; D – IV; E – V 63. Assertion: Lateral line system is found in fishes
(b) A – III; B – I; C – II; D – V; E – IV and aquatic larval amphibians.
(c) A – II; B – I; C – III; D – V; E – IV Reason: Lateral line system has receptor of
(d) A – II; B – III; C – I; D – V; E – IV sensory cells derived from ectoderm. [2002]
60. In which one of the following the genus name,
64. Assertion : Bats and whales are classified as
its two characters and phylum are not correctly
matched ? mammals.
Reason : Bats and whales have four-chambered
Genus Two characters Phylum heart. [2003]
name
65. Assertion : Holoblastic cleavage with almost
(i) Body segmented
(a) Pila Mollusca equal sized blastomeres is a characteristic of
Mouth with radula placental animals.
(ii) Spiny skinned Reason : Eggs of most mammals, including
(b) Asterias Echinodermata
Water vascular system humans, are of centrolecithal type. [2003]
(iii) Pore bearing 66. Assertion : All birds, except the ones like koel
(c) Sycon Porifera (cuckoo) build nests for retiring and taking rest
Canal system
during night time (day time for nocturnal).
(iv) Jointed appendages
(d) Periplaneta Arthropoda Reason : Koel lays its eggs in the nests of tailor
Chitinous exoskeleton
bird. [2003]
[2017]
Animal Kingdom B-23

67. Assertion : The honey bee queen copulates 71. Assertion : Acraniata is a group of organisms
only ones in her life time. which do not have distinct cranium.
Reason : The honey bee queen can lay fertilized Reason : It includes small marine forms without
as well as unfertilized eggs. [2004]
head. [1997, 2012]
68. Assertion : Torsion can be seen in ctendium.
Reason : Ctenidium acts as the respiratory 72. Assertion : Sponges belong to Porifera.
organ. [2007] Reason : Sponges have canal system.
69. Assertion : Tapeworm, roundworm and pinworm
are endoparasites of human intestine. [1998, 2014]
Reason : Improperly cooked food is the source 73. Assertion : The duck-billed Platypus and the
of intestinal infections. [2004, 2008] spiny ant-eater, both are egg-laying animals yet
Directions for (Qs. 70-76) : Each of these questions they are grouped under mammals.
contains an Assertion followed by Reason. Read them Reason : Both of them have seven cervical
carefully and answer the question on the basis of
vertebrae and 12 pairs of cranial nerves.
following options. You have to select the one that
best describes the two statements. [2005, 2015]
74. Assertion : Typhlosole increases the effective
(a) If both Assertion and Reason are correct and
area of absorption in the intestine.
Reason is the correct explanation of Assertion.
(b) If both Assertion and Reason are correct, but Reason : Typhlosole, present in the intestine, is
Reason is not the correct explanation of the characteristic feature of cockroach.
Assertion. [2016]
(c) If Assertion is correct but Reason is incorrect. 75. Assertion : Ambulacral system plays a major
(d) If both the Assertion and Reason are incorrect. role in locomotion of echinoderm.
70. Assertion : Sponges have body organization of
Reason : Hydraulic pressure of fluid and
"cellular level".
Reason : There is some physiological division contraction of muscle of tube feet make possible
of labour. [2009] movement of echinoderm. [2010, 2017]
EBD_7100
B-24 Topicwise AIIMS Solved Papers – BIOLOGY

Type A : Multiple Choice Questions 11. (b) Aristotle’s lantern is a chewing toothed
apparatus in class echinoidea of phylum
1. (b) Pouched mammals are metatherians or echinodermata.
marsupials because their female has a pouch 12. (d) Sympathetic nerves in mammals arise from
or marsupium containing the teats for thoracico–lumbar region. Sympathetic
rearing the young ones. e.g., Kangaroo and nervous system is represented by a chain
Wallaby. of 21 sympathetic ganglion on either side
2 (b) Sponges capture food with the help of of spinal cord. It receives preganglionic
flagellated cells/collar cells/choanocytes. sympathetic fibres from the spinal cord
These cells maintain a flow of water through which make these exit along with thoracic
the body and improves both respiratory and lumbar region.
and digestive functions, pulling in oxygen 13. (b) Corals belong to class anthozoa which exist
and nutrients and allowing a r apid as solitary or colonial. Corals is the rock
expulsion of CO2 and other waste products. like external skeletons. Coral animals secrete
external skeletons of calcium carbonate.
3. (a) Chloragogen cells of earthworm are similar
14. (b) Water current in Leucosolenia is produced
to the liver of vertebrates because of the
by flagellated choanocytes.
connection with storage and synthesis of
15. (d) Platyhelminthes are flat worms, e.g. blood
glycogen and fat, deamination and urea
fluke, Schistosoma.
formation.
16. (c) Four pairs of small spermathecae are
4. (c) Presence of haemocoel (a blood filled body present on the intersegmental groove of 5
cavity) is a characteristic of arthropods and & 6, 6 & 7, 7 & 8 and 8 & 9 segments. They
molluscs, e.g. cockroach and Pila. receive sperms from another worm during
5. (c) Aquatic reptiles are ammonotelic. Their copulation.
main excretory product is nitrogenous 17. (c) Weberian ossicles are the chain of 4 small
ammonia. Availability of water makes them bones that connect air bladder and internal
ammonotelic. ear of teleost fishes. They serve to enhance
6. (b) Larva of housefly is legless, headless and hearing by conducting pressure changes
wingless maggot. produced by externally originating sound
7. (c) Nostoc is a colonial cyanobacterium. It waves from the swim bladder to the ear.
contains a number of interwined filaments 18. (b) The vertebrae in birds are of heterocoelous
on the periphery, a mucilage filled hollow type.
interior and a dense mucilage covering on 19. (a) Basket star (Astropecten) belongs to class
the outside. ophiuroidea of phylum echinoderm.
8. (a) The organisms attached to substratum 20. (a) A pair of unequal, branched collaterial
glands (opening separately into genital
generally possess radial symmetry.
chamber) form an egg case ootheca.
Radial animals are usually sessile, freely
21. (b) In sponges, archaeocytes are totipotent
floating or weakly swimming.
cells which can transform into sclerocytes,
9. (c) Hydra recognizes its prey by a chemical spongocytes or collencytes. They also
stimulus. have a role in nutrient transport and sexual
10. (a) Presence of air passages in the bones reproduction.
(pneumatic bones) is a characteristic of 22. (d) Penetrant, the largest nematocysts in Hydra
birds. produce hypnotoxin.
Animal Kingdom B-25

23. (b) Life cycle of Taenia is digenetic. Their 36. (a) Chromatid bodies are present in the
primary host is pig and secondary is man. precystic stage in Entamoeba histolytica.
24. (d) Copper containing pigment, hemocyanin These bodies disappear as the cyst mature.
is found in molluscs. 37. (d) Phlebotomus (sand fly) transmits
25. (c) Antedon (sea lily/feather star) is found in Leishmania donovani which causes dum
the class crinoidea of phylum echino- dum fever/kala azar.
dermata. 38. (b) In leech, cockroach and scorpion, double
ventral nerve cord is present.
26. (a) Chondrichthyes have placoid scales and
teleosts have cycloid and ctenoid type of 39. (a) When water is not available, the camels
scales. produce dry faeces and concentrated urine.
One of the best sources of metabolic water
27. (b) Python is the largest non-poisonous
in camels is oxidation of food in the hump.
snake.
40. (b) Nephridia in earthworm, malpighian
28. (b) Birds are warm blooded/homeothermal tubules in cockroach and urinary tubules
animals. Their body temperature is in the rat are excretory in function.
independent of that of their external
41. (c) Silverfish, scorpion, dragonfly and prawn
environment.
are arthropods. They are characterised by
29. (c) Fish meal has good nutritive value because jointed appendages and chitinous
it contains easily digestive proteins (55 – exoskeleton.
77%), minerals (10 – 20%) and moisture (6
42. (b) Millipede is myriapod, silver fish is insecta
– 12%).
and sea anemone is diploblastic.
30. (a) Birds have only one ovary (left) which Duck bill platypus is a small semiaquatic
reduces their body weight. It is an oviparous mammal.
adaptation for flight.
Ornithorhynchus anatinus, the Duckbill
31. (a) Gemmule is a type of internal bud formed platypus, is a unique mammal native to
in sponges to pass the unfavourable Queensland, New South Wales, Victoria,
season. It is helpful in asexual reproduction. South Australia, and Tasmania. This animal
32. (a) The rhabditoid larva moults within egg is about the size of a house cat and is
shell in soil to form second stage rhabditoid covered by thick waterproof hair. It has a
(capable of infection). Egg hatches in the beak like a duck, webbed forelimbs for
intestine, bores through epithelium of swimming, clawed hind feet for aid in
intestine to enter the circulatory system burrowing, a common opening for the
and then it reaches alveoli where it moults reproductive, excretory and digestive
for the second time. After its 3rd moult, it systems, and a broad, flat tail. In addition,
leaves the lungs through trachea. the males have a single spur on each hind
33. (c) Spongin is an organic horny elastic ankle that contains venom, and the females
substance. They are fibres insoluble, lay eggs.
chemically inert and resistant to protein 43. (d) Mammals are characterised by muscular
digesting enzymes. diaphragm and milk producing glands.
34. (b) Hydra has diffused type of nervous 44. (d) A lamprey (sometimes also called as
system. It receives stimulus through lamprey eel) is a jawless fish that belongs
sensory cells scattered in to the class cyclostomata. Lampreys
epitheliomuscular layer. possess toothed funnel-like sucking
35. (c) Animals excreting uric acid are called mouth, having scaleless slimy skin with
uricotelic. It is least toxic and causes least seven pairs of circular gill slits on each side.
loss of body water, e.g. land reptiles, birds They are well known as those species
etc. which bore into the flesh of other fish to
suck their blood.
EBD_7100
B-26 Topicwise AIIMS Solved Papers – BIOLOGY
45. (a) Nephridia are invertebrate organs which systems made up of a brain and a dorsal
function similarly to kidneys. They remove (back) nerve cord. We all have highly devel-
metabolic wastes from an animal’s body. oped receptors to detect changes in the en-
They are of two basic types, metanephridia vironment for us to respond to. Fused
and protonephridia. A metanephridium is a ganglia is a primitive brain found in earth-
type of excretory gland or nephridium found worms, tympana are primitive ears found in
in annelids, arthropods and molluscs. A grasshoppers, and nerve nets are branched
protonephridium is a network of dead-end nerves without direction found in the Hydra.
tubules lacking internal openings. The ends 50. (a) The animals of phylum echinodermata are
are called flame cells or solenocytes; they oviparous. They have the ability of regen-
functions in osmoregulation. eration.
46. (a) Morphallaxis refers to the type of
51. (a) Scorpion, spider, tick and mites comes un-
regeneration in which lost body parts are
der class arachnida of phylum arthropoda.
replaced by the remodeling of the remaining
tissue. In this type of regeneration, little or 52. (d) Portuguese man of war is a polymorphic,
no cellular proliferation takes place during colonial coelenterate Physalia physalis. It
the regeneration process. A classic example is named so due to its appearance which
of an organism that regenerates using this resembles with portuguese vessels sailing
mechanism is the Hydra. When a Hydra is in sea in 15th and 16th centuries
cut into two pieces, two Hydra as will be
53. (d) Whales are exclusively viviparous.
regenerated, both smaller than the parental
Hydra. Once regeneration is completed, the In viviparous animals (majority of mammals
two Hydra can continue to grow and reach including human beings), the zygote
the size of their original parent. Growth develops into a young one inside the body
requires cellular proliferation but during the of the female organism. After attaining a
regenerative process very little cellular certain stage of growth, the young ones are
proliferation takes place. delivered out of the body of the female
47. (d) Invertebrates, animals without a backbone organism.
such as an earthworm, are simpler than
54. (a) 55. (b)
vertebrates such as a human. Therefore, their
cells are not as complex and they may be 56. (a) Diploblastic animals have two germinal
able to regenerate or grow lost body parts layers, outer ectoderm and inner endoderm,
easier than a more complex animal. e.g., Porifera and Coelenterate. Triploblastic
48. (b) Earthworm has closed circulatory system. animals have three germinal layers – outer
The blood circulates in definite walled blood ectoderm, middle mesoderm and inner
vessels. Renal portal system is characteristic endoder m, e.g., Platyhelminthes,
of reptiles. In reptiles, renal portal system Aschelminthes, Annelida, Arthropoda,
brings blood from internal organs to the
Molluscs, Echinodermata and Chordata.
kidneys because their digestive tract and
urinary tract only have cloaca to eliminate 57. (a)
wastes. Humans have hepatic portal system 58. (c) The amphibian skin is moist.
that brings all the venous flow from digestive 59. (d)
system into the liver. In cockroach, the 60. (a) Molluscans are soft bodied animals. Their
excretory organs are malpighian tubules not body is unsegmented with a distinct head,
nephridia. muscular foot and visceral hump. In Pila,
49. (a) Chordates are a group of animals such as the buccal cavity contains a rasping organ,
fish, frogs, snakes, birds, dogs, monkeys and the radula with transverse rows of teeth.
humans. We all have central nervous
Animal Kingdom B-27

Type B : Assertion Reason Questions 69. (b) Tapeworm, roundworm & pinworm are all
endoparasites. The main cause of the
61. (a) Cold blooded organisms utilize their stored intestinal infection is improperly cooked
food at the time of hibernation and food. However, tapeworm infection occur
aestivation. by eating improperly cooked food,
62. (a) Birds have only left ovary which is an roundworm is transmitted by contaminated
adaptation to reduce the weight for flight. food & water and pinworm or ringworm is
transmitted through food or improper
63. (b) Lateral line system is made up of sensory sanitary condition.
cells of ectodermal origin. It is meant for
70. (b) Sponges are multicellular but they have
balancing the body while swimming. So, is cellular level of body organization i.e., true
found in fishes and larval forms of tissue, movable parts, or appendages are
amphibians. not formed. Although, there is some
64. (b) Bats and whales are classified as mammals. physiological division of labour,
They have milk secreting mammary glands. accompanied with structural differentiation
amongst body cells. But here, similar cells
Whales and bats are mammals. Whales are
are arranged neither in permanent layer nor
warm blooded, breath air through lungs and
masses to form tissues.
give birth to live young that are suckled on
71. (b) Cephalochordates and urochordates are
milk secreted from the mother’s mammary
acraniates. These are marine animals
glands. Bats have hair, give birth to live without cranium, jaws, vertebral column
young and feed these young on milk and paired appendages. Notochord is
produced in mammary glands. They are present and they are less developed than
the only true flying mammals and are so crainates.
unique that they have been placed in an 72. (b) Sponges belong to Porifera and they have
order of their own chiroptera. 4-chambered characteristic canal system.
hearts in mammals and birds keep fully 73. (b) Both duck billed platypus and spiny ant
oxgenated. eaters are mammals because of their
65. (c) Cleavage in placental mammals is constant body temperature and presence
holoblastic because of microlecithal eggs. of diaphragm.
66. (c) Koel (Eudynemis) lays eggs in crow’s nest 74. (d) Typhlosole is the characteristic feature of
for incubation and rearing. earthworm. It can be defined as an extra
flap of tissue or an infolding along the inner
67. (a) Honey bee queen copulates only once in
wall of intestine. The typhlosole in
her life span (nupital flight) and stores all earthworm increases the surface area of the
the sperms in her seminal receptacles. It intestin e for efficient secretion and
depends on her whether she releases absorption during digestion.
sperms while laying eggs or not, so there 75. (a) The water vascular system is a unique
are both fertilized and unfertilized eggs. organ system that functions in locomotion,
68. (b) Ctenidium is a gill situated on the right side feeding, respiration an d excretion.
of the branchial chamber. It helps in Ambulacral canal is connected to outside
respiration by beating cilia. During through external tube feet. Hydraulic
development, ctenidium shifts from left side pressure of fluid and contraction of muscle
of tube feet make possible movement of
to right side which is called "torison". It is
Echinoderm.
characteristic feature of gastropods.
EBD_7100
B-28 Topicwise AIIMS Solved Papers – BIOLOGY

5 Morphology of Flowering Plants


10. Clove is [2001]
TYPE A : MULTIPLE CHOICE QUESTIONS
(a) flower bud (b) axillary bud
1. Fern character of Cycas is [1997] (c) thalamus (d) ovule
(a) coralloid root
11. Pollinia are found in [2001]
(b) tap root
(a) wheat (b) madar
(c) parallel venation
(d) circinate venation (c) mango (d) banana
2. The desert plants in order to tolerate water stress 12. Monocarpic plants flower [2001]
show [1997] (a) once (b) twice
(a) sunken stomata (c) many times (d) never
(b) reduced leaves 13. Nodules with nitrogen fixing bacteria are found
(c) well developed root system in [2001]
(d) all of these (a) cotton (b) gram
3. Perisperm is [1997] (c) mustard (d) wheat
(a) remnant of endosperm 14. In which family (9) + 1 androecium condition is
(b) persistant nucellus found ? [2001]
(c) remnant of embryo
(a) Malvaceae (b) Papilionaceae
(d) part of endosperm
4. In moss capsule, the number of peristome whorls (c) Solanaceae (d) Poaceae
are [1998] 15. Which of following type of anther is found in
(a) 1 (b) 2 Malvaceae? [2002]
(c) 3 (d) 4 (a) Monothecous (b) Dithecous
5. A mature ligule, having a prominent basal (c) Polythecous (d) Without thecous
portion, is called [1998] 16. Potato and sweet potato [2004]
(a) glossopodium (b) rhizophore
(a) have edible parts which are homologous
(c) trichome (d) None of these
organs.
6. Inflorescence of Ficus is [1999]
(b) have edible parts which are analogous
(a) spike (b) hypanthodium
organs.
(c) raceme (d) verticillaster
7. The edible part of cauliflower is [2000] (c) have been introduced in India from the
(a) inflorescence (b) leaf same place.
(c) flower (d) stem (d) are two species of the same genus.
8. Most reduced form of stem is found in [2000] 17. The sugarcane plant has [2004]
(a) bulb (b) rhizome (a) dumb-bell shaped guard cells
(c) tree (d) stem (b) pentamerous flowers
9. In Opuntia, spines are modification of [2000] (c) reticulate venation
(a) stem (b) root (d) capsular fruits
(c) leaf (d) flower
Morphology of Flowering Plants B-29

18. The family containing mustard and its main 25. Of the following, which instrument is most
characters are [2005] commonly used to observe the external features
(a) Brassicaceae - Tetramerous flowers, six of a grasshopper's abdomen? [2009]
stamens, bicarpellary gynoecium, siliqua (a) Ultracentrifuge
type fruit (b) Microdissection instrument
(b) Brassicaceae - Pentamerous flowers, many (c) Dissecting microscope
stamens, pentacarpellary gynoecium, (d) Electron microscope
capsule type fruit 26. In a bisexual flower, if androecium and
(c) Solanaceae - Pentamerous flowers, five gynoecium mature at different times, the
stamens, bicarpellary gynoecium, berry phenomenon is known as a [2010]
type fruit (a) dichogamy (b) herkogamy
(d) Poaceae - Trimerous flowers, three (c) heterogamy (d) monogamy
stamens, monocarpellary gynoecium, 27. Which of following type of anther is found in
caryopsis type of fruit Malvaceae? [2011]
19. Velamen present in orchids help in [2007] (a) Monothecous (b) Dithecous
(a) absorbing water from support (c) Polythecous (d) Without thecous
(b) respiration 28. Parachute type dispersal occurs in [2011]
(c) absorption of moisture from air (a) tomato (b) mustard
(d) synthesizing food (c) pea (d) cotton
20. Composite fruit develops from [2007]
29. Prickles of rose are [2012]
(a) single ovary (b) inflorescence
(a) Modified leaves
(c) apocarpous ovary (d) pericarp
(b) Modified stipules
21. If the anthers are fused together forming a
tubular structure while the filaments remain free, (c) Exogenous in origin
the condition is found in which one of the (d) Endogenous in origin
following family? [2009] 30. Which of the following are not characteristic
(a) Malvaceae (b) Cucurbitaceae features of fabaceae? [2013]
(c) Solanaceae (d) Asteraceae (a) Tap root system, compound leaves and
22. Floral diagram fails to indicate [2009] raceme inflorescence.
(a) epiphylly and epipetaly (b) Flowers actinomorphic, twisted aestivation
(b) aestivation and placentation and gamopetalous.
(c) position of ovary on the thalamus (c) Stamens 10, introrse, basifixed, dithecous.
(d) cohesion of carpels and stamens (d) Monocarpellary, ovary superior and bent
23. Aggregate fruit develops from [2009] stigma.
(a) syncarpous ovary 31. Which one of the following is correctly
(b) multicarpellary, syncarpous ovary matched? [2014]
(c) unilocular ovary (a) Onion – Bulb
(d) multicarpellary, apocarpous ovary
(b) Ginger – Sucker
24. The presence of cilia, an oral groove, and food
vacuoles, and the absence of chloroplasts in a (c) Chlamydomonas – Conidia
unicellular organism indicate that the organism (d) Yeast – Zoospores
carries on [2009] 32. Read the following statements.
(a) sexual reproduction (i) Gynoecium is situated in the centre and
(b) autotrophic nutrition other parts of the flower are located on the
(c) extracellular digestion rim of the thalamus almost at the same level.
(d) heterotrophic nutrition
EBD_7100
B-30 Topicwise AIIMS Solved Papers – BIOLOGY
(ii) Ovary is half-inferior. C. Indusium III. An unbranched
(iii) Examples are plum, rose and peach. columnar stem with a
Which condition of flowers is being described crown of leaves.
by the above statements ? [2014] D. Caudex IV. Protective covering of
(a) Hypogyny (b) Perigyny radicle
(c) Epigyny (d) None of these V. Protective structure of a
33. Which one of the option is correct? sorus. [2016]
(a) A – V; B – II; C – IV; D – I
Seed coat C (b) A – IV; B – I; C – V; D – III
Raphe D (c) A – III; B – V; C – II; D – IV
A E (d) A – II; B – III; C – I; D – V
B 36. Which of the following is a modified stem for
Seed the protection of plants from browsing animals?
Seed opened
(a) Tendrils (b) Thorns
[2015] (c) Rhizome (d) Tuber
(a) A - Hilum, B - Micropyle, C - Radicle, [2017]
D - Cotyledon, E - Plumule 37. Leaves of dicotyledonous plants possess
(b) A - Hilum, B - Micropyle, C - Plumule, _________ venation, while _________
D - Cotyledon, E - Radicle venation is the characteristic of most
(c) A - Micropyle, B - Hilum, C - Plumule, monocotyledons. [2017]
D - Cotyledon, E - Radicle (a) reticulate and parallel
(d) A - Hilum, B - Micropyle, C - Plumule, (b) parallel and reticulate
D - Radicle, E - Cotyledon (c) reticulate and perpendicular
34. Seeds are adaptively important because [2015] (d) obliquely and parallel
1. they maintain dormancy TYPE B : ASSERTION REASON QUESTIONS
2. they protect young plants during
vulnerable stages Directions for (Qs. 38-40) : These questions consist
of two statements, each printed as Assertion and
3. they store food for young plants, and
facilitate dispersal Reason. While answering these questions, you are
required to choose any one of the following five
(a) 1 and 3 (b) 2 and 3
responses.
(c) 1 and 2 (d) All of these
(a) If both Assertion and Reason are correct and
35. Match the following-
the Reason is a correct explanation of the
List-I List-II
Assertion.
A. Coleorhiza I. Development of sporo-
phyte directly from ga- (b) If both Assertion and Reason are correct but
metophyte without in- Reason is not a correct explanation of the
tervention of gametes Assertion.
B. Apogamy II. Development of game- (c) If the Assertion is correct but Reason is
tophyte directly from incorrect.
sporophyte (d) If both the Assertion and Reason are incorrect.
without the involvement (e) If the Assertion is incorrect but the Reason is
of reduction division. correct.
Morphology of Flowering Plants B-31

38. Assertion : In hemianatropous ovule, the funicle (a) If both Assertion and Reason are correct and
lies parallel to body of ovule. Reason is the correct explanation of Assertion.
Reason : Here, body of ovule is rotated by 90°. (b) If both Assertion and Reason are correct, but
[1999] Reason is not the correct explanation of
39. Assertion : Many plants are propagated Assertion.
(c) If Assertion is correct but Reason is incorrect.
vegetatively even though they bear seeds.
(d) If both the Assertion and Reason are incorrect.
Reason : Potatoes multiply by tubers, apple by
41. Assertion : In fabaceae family monocarpellary,
cutting etc. [2001]
unilocular ovary is present. [2010]
40. Assertion : Ginger has a prostrate growing
rhizome. Reason : In fabaceae, placentation is parietal.
Reason : Shoot growth is not effected by gravity. 42. Assertion : Apical meristem of root is
[2004] subterminal.
Directions for (Qs. 41-43) : Each of these questions Reason : At the terminal end of root, root cap is
contains an Assertion followed by Reason. Read them present. [2014]
carefully and answer the question on the basis of 43. Assertion : A simple leaf has undivided lamina.
following options. You have to select the one that Reason : Leaves showing pinnate and palmate
best describes the two statements. venation have various types of incisions.
[2016]
EBD_7100
B-32 Topicwise AIIMS Solved Papers – BIOLOGY

Type A : Multiple Choice Questions and spice has more of the medicinal
property at the bud stage.
1. (d) In circinate venation, leaves are coiled
11. (b) Pollinia is generally seen in fused anthers.
when immature and gradually rolled with
Each anther has pollinia. Madar shows the
maturity. This coiling protects the growing
characteristic feature of pollinia.
point.
12. (a) Monocarpic flowers have a single carpel
2. (d) Desert plants have well developed root
that can mature only once in their life time.
system so that they can absorb water from
Hence, they flower only once.
the deeper layers of soil. They have sunken
stomata and reduced leaves which reduce 13. (b) Grams are leguminous plants. Nitrogen
the rate of water loss through transpiration. fixing bacteria is seen in leguminous plants.
They convert atmospheric N2 to nitrate that
3. (b) The nucellus is generally used up during
can easily be absorbed by plants.
the development of embryo but in some
cases it remains outside the endosperm in 14. (b) In papilionaceae- the androceium is seen
in diadelphous condition. 10 stamens are
the form of a thin layer, called perisperm.
seen in two bundles- 9 + 1. 9 together form
4. (b) In moss, peristome consists of 32 acellular one bundle whereas the single stamen
teeth arr anged in 2 whorls, outer forms another bundle.
hygroscopic and inner hygroscopic. 15. (a) The filament of stamen bears one celled
5. (a) Glossopodium is a mature ligule. It is anther.
generally present in grasses. The ligule is 16. (b) Potato is the modified underground stem
an outgrowth between leaf base and lamina. whereas sweet-potato is the modified root
Leaves with ligule are called ligulates. for storage of food. These are analogous
6. (b) Ficus has hypanthodium type of organs which have different origin but
inflorescence i.e. cup shaped, fleshy serve the same functions.
receptacle bearing flowers on the inner wall 17. (a) Sugarcane being a monocot plant is
of the cavity. characterized by the presence of dumb-bell
7. (a) Cauliflower is a crossbreed between 2 shaped guard cells. In dicots, guard cells
are kidney shaped.
varieties of cabbage. It is evident that the
edible part is the inflorescence due to the
presence of small florets.
8. (a) Bulb has the most reduced form of stem
since the stem is discoid in nature – a
flattened disc. e.g. Allium cepa, Allium Stomata Stomata
sativum. Dicot Monocot
9. (c) In Opuntia, the entire leaf is modified into 18. (a) Brassicaceae (cruciferae)
the spine, to reduce water loss due to Other members are : Raddish, cabbage,
transpiration as Opuntia is a xerophyte. cauliflower.
10. (a) The structure of clove itself resembles the Floral formula :
bud where we are, infact, able to see the
calyx also. Clove that is used as a condiment Ebr, , K2 + 2, C4 A2 + 4, G(2)
Morphology of Flowering Plants B-33

19. (c) Orchids are the epiphytes that posses aerial manipulate microscopic organelles (such
roots. These roots contain a spongy tissue as transferring nuclei), and electron micro-
called velamen whose main function is to scopes can magnify an image up to 250,000x
absorb moisture from the air. and are used to see microscopic details.
20. (b) Composite fruits develop from the 26. (a) In a bisexual flower, if androecium and
complete inflorescence and are also known gynoecium mature at different times, the
as multiple fruit. These are of 2 types- phenomenon is known as dichogamy.
sorosis and syconus. Sorosis develops 27. (a) The filament of stamen bears one celled
from spike, spadix or catkin inflorescence, anther.
e.g. Ananas, Arto carpus. Syconus 28. (b) Dispersal is a universal biological need. For
develops from hypanthodium non-aquatic, terrestrial plants, the wind is
inflorescence, e.g. Ficus. an obvious supplier of en ergy for
21. (d) This condition is called syngenesious. It movement, and many plant adaptations
is found in members of family asteraceae exist that clearly take advantage of this fact.
(or compositae), e.g., disc florets of This type of seed dispersal is not efficient,
sunflower. but very effective. Appendages of some
22. (c) Floral diagram illustrates the whorls and fruits & seeds act as parachute like seeds
number of parts in each of the sets of of cotton possess hair that help in dispersal
organs comprising a flower. It shows the of these seeds. Anoth er well-known
position of floral parts in relation to mother example is the dandelion.
axis but although position of ovary on the 29. (c) Prickles of rose develop only from cortex
thalamus is not shown by it. and epidermis and found at the nodes or
23. (d) A muticarpellary ovary may be syncarpous internodes. It helps in climbing. It is
(when carpels are fused) or apocarpous exogenous in origin.
(when the carpels are free). A syncarpous 30. (b)
ovary gives rise to a simple fruit while in an 31. (a) Onion - Bulb - Undeground stem , Ginger -
apocarpous ovary, each carpel changes Rhizome, Chlamydomonas - Zoospore,
into a fruitlet. The collection or aggregate Yeast - Ascospores
of these fruitlets is known as etaerio.
32. (b) 33. (b)
24. (d) Cilia are hair-like bristles on a Paramecium
34. (d) Seeds perform all the given functions.
used in locomotion and to find food. An
35. (b)
oral groove is a mouth for a Paramecium,
36. (b) Thorn is a stiff, sharp-pointed woody
and food vacuoles store food in the cell
projection on the stem or other part of a
body of the Paramecium. All of these are
plant. Thorns are found in many plants
cell organelles used to ingest, digest, and
such as Citrus, Bougainvillea. They
egest preformed food, which is
protect plants from grazing animals.
heterotrophic nutritrion. An autotroph can
37. (a) Leaves of dicotyledonous plants possess
make its own food using chloroplasts.
reticulate venation while parallel venation
25. (c) The dissecting microscope allows to view
is the characteristics of most
3D images up to 50x magnification. It is
monocotyledonous. In reticulate venation,
commonly used in dissections. The
the main veins of leaf form numerous
ultracentrifuge spins liquids and separates
irregular branches and as a result a net like
the contents by their density, micro-
arrangements is formed. Reticulate
dissection in struments are used to
EBD_7100
B-34 Topicwise AIIMS Solved Papers – BIOLOGY
venation is the most common vein emerges from axils of scaly leaves.
formation in leaves. It can be found in the Response to light by plants is called
leaves of maple trees, oak trees and rose phototropism. In this sense, shoot shows
bushes. In parallel venation, veins are positive phototropism and root shows
arranged parallel to each other. negative phototropism.
Type B : Assertion Reason Questions 41. (c) In fabaceae, ovary is present. Placentation
is marginal with many ovules.
38. (e) In hemianatropus ovule, the funicle lies at
42. (a)
right angles to the body of the ovule. The
body of the ovule is rotated by 90º. 43. (b) A leaf having a single or undivided lamina
is called simple leaf, the lamina can have
39. (c) Plants do propagate more by vegetative
different types of incisions, which may
means since they multiply faster
reach upto half, more than half or near the
vegetatively.
base or midrib. Depending upon the pinnate
40. (b) Ginger is an example of rhizome (e.g. or palmate venation, the incisions are known
prostrate stem creeping horizontally under as pinnatifid, palmatifid, pinnatipartite,
soil surface). There is no effect of gravity. palmatipartite, pinnatisect and palmatisect,
Rhizome of ginger contains nodes, etc.
internodes and scaly leaves. Buds are
6 Anatomy of Flowering Plants

TYPE A : MULTIPLE CHOICE QUESTIONS 10. Passage cells are found in [2002]
(a) endodermis (b) pericycle
1. Cycas stem shows [1997]
(c) cortex (d) epiblema
(a) porous wood (b) manoxylic wood
(c) pycnoxylic wood (d) ring porous wood 11. Fascicular cambium is the cambium of vascular
2. Aerenchyma is found in [1997] bundle of [2002]
(a) parenchyma (b) xylem (a) monocot stem (b) dicot stem
(c) phloem (d) sclerenchyma (c) monocot leaf (d) dicot leaf
3. Which of the following tissue is absent in 12. Mesophyll is usually differentiated in [2002]
vascular bundles of monocot stem ? [1997] (a) monocot leaf (b) isobilateral leaf
(a) Xylem (b) Phloem
(c) dorsiventral leaf (d) both ‘a’ and ‘b’
(c) Cambium (d) All of these
13. In a dicotyledonous stem, the sequence of
4. Cork cambium is a [1999]
tissues from the outside to the inside is[2003]
(a) lateral meristem
(b) apical meristem (a) phellem-pericycle-endodermis-phloem
(c) intercalary meristem (b) phellem-phloem-endodermis-pericycle
(d) primitive meristem (c) phellem-endodermis-pericycle-phloem
5. Endodermis is a part of [1999] (d) pericycle-phellem-endodermis-phloem
(a) cortex (b) pericycle 14. The quiescent centre in root meristem serves as a
(c) medulla (d) epidermis (a) site for storage of food which is utilized
6. Lateral root in higher plants arise from [1999] during maturation. [2003]
(a) cortex (b) pericycle
(b) reservoir of growth hormones.
(c) epidermis (d) endodermis
(c) reserve for replenishment of damaged cells
7. Cambium of root is an example of [2000]
of the meristem.
(a) apical meristem
(d) region for absorption of water.
(b) intercalary meristem
(c) primary meristem 15. In a plant organ which is covered by periderm
(d) secondary meristem and in which the stomata are absent, some
gaseous exchange still takes place through
8. Which of the following is enucleate at maturity?
[2004]
[2000]
(a) aerenchyma (b) trichomes
(a) Companion cell (b) Meristematic cell
(c) pneumatophores (d) lenticels
(c) Parenchyma (d) Sieve tube cell
16. Companion cells in plants are associated with
9. Porous wood contains [2001]
[2004]
(a) vessels (b) tracheids
(a) vessels (b) sperms
(c) fibres (d) parenchyma
(c) sieve elements (d) guard cells
EBD_7100
B-36 Topicwise AIIMS Solved Papers – BIOLOGY
17. Cork cambium results in the formation of cork 23. If a stem is girdled [2012]
which becomes impermeable to water due to the (a) Root dies first
accumulation of [2004] (b) Shoot dies first
(a) resins (b) suberin (c) Both die together
(c) lignins (d) tannins (d) None of the above would die
18. Which one of th e following statements 24. Which of the following statement(s) is/are true?
pertaining to plant structure is correct? [2005]
(A) Uneven th ickening of cell wall is
(a) Cork lacks stomata but lenticels carry out
characteristic of sclerenchyma. [2013]
transpiration.
(B) Periblem forms cortex of the stem and the
(b) Passage cells help in transfer of food from
root.
cortex to phloem.
(C) Tracheids are the chief water transporting
(c) Sieve tube elements possess cytoplasm but
elements in gymnosperms.
no nuclei.
(d) The shoot apical meristem has a quiescent (D) Companion cell is devoid of nucleus at
centre. maturity.
19. In which one of the following would you expect (E) The Commercial cork is obtained from
to find glyoxysomes ? [2005] Quercus suber.
(a) Endosperm of wheat (a) A and D only (b) B and E only
(b) Endosperm of castor (c) C and D only (d) B, C and E only
(c) Palisade cells in leaf 25. Sclerenchyma usually_______ and ________
(d) Root hairs protoplasts. [2014]
20. Grafting is successful in dicots but not in (a) live, without (b) dead, with
monocots because the dicots have [2006] (c) live, with (d) dead, without
(a) vascular bundles arranged in a ring 26. T.S. of dicot leaf passing through the midrib is
(b) cambium for secondary growth given below, certain parts have been indicated
(c) vessels with elements arranged end to end by alphabets. Choose the correct option.
(d) cork cambium
21. In the sieve elements, which one of the following
is the most likely function of P-proteins?[2006]
(a) Deposition of callose on sieve plates.
(b) Providing energy for active translocation.
(c) Autolytic enzymes.
(d) Sealing mechanism on wounding.
22. Two cross-sections of stem and root appear
simple, when viewed by naked eye. But under
microscope, they can be differentiated by
[2009]
(a) exarch condition of root and stem
[2015]
(b) endarch condition of stem and root
(a) A – Epidermis, B – Spongy parenchyma,
(c) endarch condition of root and exarch
C – Palisade parenchyma, D – Stomata, E –
condition of stem
Guard cells, F – Phloem, G – Metaxylem, H
(d) endarch condition of stem and exarch
– Protoxylem
condition of root
Anatomy of Flowering Plants B-37

(b) A – Epidermis, B – Palisade parenchyma, (a) If both Assertion and Reason are correct and
C – Spongy parenchyma, D – Sub-stomatal the Reason is a correct explanation of the
cavity, E – Stoma, F – Phloem, G – Assertion.
Metaxylem, H – Bundle sheath (b) If both Assertion and Reason are correct but
(c) A – Epidermis, B – Palisade parenchyma, Reason is not a correct explanation of the
C – Spongy parenchyma, D – Stomata, E – Assertion.
Guard cells, F – Epidermis, G – Xylem, H – (c) If the Assertion is correct but Reason is
Phloem incorrect.
(d) A – Epidermis, C – Palisade parenchyma, (d) If both the Assertion and Reason are incorrect.
C – Spongy parenchyma, D – Stomata, E – (e) If the Assertion is incorrect but the Reason is
Guard cells, F – Phloem, G – Metaxylem, H correct.
– Protoxylem 30. Assertion : Thick cuticle is mostly present in
27. Contractile tissues have the following features disease resistant plants.
(i) Mesodermal in origin
Reason : Disease causing agents cannot grow
(ii) They contain stretch receptors.
(iii) Rhythmic contractions are seen in them on cuticle and cannot invade the cuticle. [1997]
(iv) They do not fatigue during the life of the 31. Assertion: Cambium is a lateral meristem and
animal cause growth in width.
Which of the above are characteristics of Reason: Cambium is made up of fusiform and
sphincters? [2015]
ray initials in stem. [1998]
(a) All the four
(b) Only (i), (ii) and (iii) 32. Assertion : Higher plants have meristematic
(c) Only (i), (ii) and (iv) regions for indefinite growth.
(d) Only (i), (iii) and (iv) Reason : Higher plants have root and shoot
28. Meristematic tissue responsible for increase in apices. [1999]
girth of tree trunk is [2016] 33. Assertion : In collateral vascular bundles,
(a) Apical meristem phloem is situated towards inner side.
(b) Intercalary meristem Reason : In monocot stem, cambium is present.
(c) Lateral meristem [2000]
(d) Phellogen 34. Assertion : Collenchyma is thick walled dead
29. In stems, the protoxylem lies towards the tissue.
_____________ Reason : Collenchymatous cells show
and the metaxylem lies towards the thickenings of pectin. [2002]
____________ of the organ. 35. Assertion: The two cotyledons in seed are
(a) centre; periphery embryonic leaves.
(b) periphery; centre
Reason: The embryo contains radicle and
(c) periphery; periphery
plumule. [2002]
(d) centre; centre [2017]
36. Assertion : In angiosperms, the conduction of
TYPE B : ASSERTION REASON QUESTIONS water is more efficient because their xylem has
vessels. [2006]
Directions for (Qs. 30-38) : These questions consist
Reason : Conduction of water by vessel
of two statements, each printed as Assertion and
elements is an active process with energy
Reason. While answering these questions, you are
supplied by xylem parenchyma rich in
required to choose any one of the following five
mitochondria.
responses.
EBD_7100
B-38 Topicwise AIIMS Solved Papers – BIOLOGY
37. Assertion : In woody stems, the amount of heart (b) If both Assertion and Reason are correct, but
wood continues to increase year after year. Reason is not the correct explanation of
Reason : The cambial activity continues Assertion.
uninterrupted. [2007] (c) If Assertion is correct but Reason is incorrect.
(d) If both the Assertion and Reason are incorrect.
38. Assertion : Petroplants produce large amount
39. Assertion : Vessels are more efficient for water
of latex.
conduction as compared to tracheids. [2010]
Reason : The latex contains long chain
Reason : Vessels are dead lignified.
hydrocarbons. [2007]
40. Assertion: Bulliform cells are useful in the
Directions for (Qs. 39-41) : Each of these questions
unrolling of leaf.
contains an Assertion followed by Reason. Read them
carefully and answer the question on the basis of Reason: Bulliform leaves store water. [2011]
following options. You have to select the one that 41. Assertion : In stem, pericycle take active part in
best describes the two statements. secondary growth.
(a) If both Assertion and Reason are correct and Reason : In dicots, pericycle has the capacity to
Reason is the correct explanation of Assertion. produce lateral roots. [2013]
Anatomy of Flowering Plants B-39

Type A : Multiple Choice Questions 7. (d) The cambium is secondary in its functional
aspect since it forms secondary tissues like
1. (b) Cycas stem shows monoxylic wood with the secondary xylem and secondary
broad parenchymatous rays and often phloem. It is, however, primary in origin.
contain abundant resinous cells and resin 8. (d) Sieve tube cell is enucleate at maturity due
canals. Towards the periphery of the stem, to the degeneration of its nucleus during
cycads produce a 'leaf armour' consisting its developmental process. The companion
of the tightly packed, helically arranged leaf cell-that develops from the same initial as
bases. the sieve tube cell, possesses the nucleus
2. (a) Aerenchyma is the modification of throughout its life. The companion cell
parenchyma tissue in which cells are carries out the function of the sieve tube
arranged in such a way that - large air filled cell in the event of its degeneration.
spaces are formed. Aerenchyma is found 9. (a) Porous wood contains vessels, with sieve
in aquatic plants to produce buoyancy. cells, which contains passages for
3. (c)
movement of substances.
Monocot vascular Dicot vascular
bundle bundle 10. (a) Passage cells are found in endodermis
which allow a limited transfer of materials
between the cortex and the vascular
Phloem Phloem
cylinder.
Cambium 11. (b) In dicot stem, fascicular cambium and
Xylem Xylem interfascicular cambium join to form a
No secondary growth, secondary growth, complete ring which helps in secondary
cambium absent V.B. is open. growth.
i.e. V.B. is closed 12. (c) In a dorsiventral leaf, mesophyll is
4. (a) Cork cambium (phellogen) is a secondary differentiated into two layers i.e. palisade
lateral meristem which develops from parenchyma and spongy parenchyma.
permanent tissues in the region of 13. (c) In a dicotyledonous stem, the sequence of
epidermis, hypodermis, cortex and even in tissues from outside to the inside is
outer layers of phloem. phellem-endodermis-pericycle-phloem.
5. (a) Endodermis is the inner most layer of cortex. 14. (c) The concept of Quiescent Centre was
The cells are characterized by the presence proposed by Clowes in 1961. On the basis
of casparian strips. of autoradiographic studies of DNA
6. (b) The lateral roots arise from the cell of synthesis in the root tip of zea, he found a
pericycle and hence, the root branches are reservoir of cells having low DNA, RNA
said to be endogenous in origin (arising and protein concentration. He called it as
from a layer inner to endodermis). The Quiescent Centre. They may or may not
lateral roots help in absorption of water and divide. It is resistant to damages.
mineral salts from the soil. The meristematic 15. (d) During secondary growth, where epidermis
cells of the lateral root push through the is replaced by tough and hard periderm,
endodermis and cortex and then pierce the cracked/ruptured epidermis forms the
through the epidermis to come out to form small holes called lenticels which help in
the lateral root. gaseous exchange.
EBD_7100
B-40 Topicwise AIIMS Solved Papers – BIOLOGY
16. (c) Companion cells are long elongated living 23. (a) If a stem is girdled, root dies first, as the
cells, that lie on the sides of the sieve tubes food synthesized by leaves is not able to
in phloem. Companion cells control the reach to the roots.
activities of the sieve tube through
24. (d)
plasmodesmata.
17. (b) Phellogen produces cork or phellem on the 25. (d) Sclerenchyma consists of long, narrow cells
outer side. It consists of dead and with thick and lignified cell walls having a
compactly arranged rectangular cells that few or numerous pits. They are usually
possess suberised cell walls. dead and without protoplasts.
Lenticels 26. (b) 27. (b) 28. (c)
29. (a) The first formed primary xylem elements are
called protoxylem and the later formed
Cork cells primary xylem is called metaxylem. In stems,
(suberized) the protoxylem lies towards the centre
Phellogen (pith) and the metaxylem lies towards the
periphery of the organ. This type of
Secondary
cortex primary xylem is called endarch.
18. (c) Sieve tube elements possess cytoplasm but Type B : Assertion Reason Questions
lack nucleus at maturity. Its metabolic
activities are regulated by the nucleus of a 30. (e) Plant cuticles are a protective waxy
closely associated cell called companion covering produced only by the epidermal
cell. cells of leaves, young shoots and all other
19. (b) Glyoxysomes are found in the plant cells aerial plant organs. In addition to its
particularly in the cells of germinating fatty function as a permeability barrier for water
seeds, e.g. endosperm of castor. and other molecules, the micro and nano-
20. (b) Grafting is a horticultural technique structure of the cuticle confer specialized
whereby tissues from one plants are surface properties that prevent
inserted into those of another so that the contamination of plant tissues with external
sets of vascular tissues may join together. water, dirt and micro-organisms. The waxy
Grafting is successful in dicots because sheet of cuticle also functions in defence,
vascular bundles are arranged in a ring and forming a physical barrier that resists
have cambium for secondary growth. penetration by virus particles, bacterial
21. (a) In the sieve elements, P-proteins deposit cells, and the spores or growing filaments
callose on sieve plates. of fungi.
22. (d) The cross sections of stem and root appear 31. (b) Fusiform initials are vertically elongated cells
simple, when viewed by naked eye but that produce xylem and phloem elements.
under microscope they can be Ray initials are isodiametric and produce
differentiated as endarch condition in stem parenchymatous rays in secondary xylem
and exarch condition in root. In endarch, and phloem.
protoxylem is present towards the center 32. (a) The root apex an d shoot apex are
of stem while metaxylem towards the meristematic in nature. These meristematic
pericycle. In the exar ch condition, tissues are embryonic in origin. They are
protoxylem is present towards pericycle primary in origin because it develops from
and metaxylem towards the center of the embryonic tissues and primary in function
root.
Anatomy of Flowering Plants B-41

because they form the primary structure of 38. (a) Petroplants are plants having large amount
the plant cell, the root apex and shoot apex, of latex with long chain hydrocarbons. Latex
that live till the death of the whole plant. of these plants are a good substitute for
Hence, plants have the feature of indefinite liquid fuels or petroleum. Cultivation of
growth. petroplants is a part of energy- cropping.
33. (d) Collateral vascular bundles have the xylem Dr. Calvin was the scientist who identified
pointing towards the inner side of the petrocrops. They have pr oper ty of
phloem. In the same way in monocots, converting large amount of their
cambium is absent. Collateral vascular photosynthates into latex along with
bundles are present in stems and leaves of hydrocarbons. Some important petrocrops
angiosperms and gymnosperms. are Euphorbia antisyphilitica, E. lathyris
34. (e) Collenchyma is made up of living cells with Calotropis procera etc.
unevenly thickened cell wall. Their cell wall 39. (b) Vessels are more efficient for water
is made up of cellulose and pectin. conduction as compared to tracheids.
Collenchyma are present beneath the Vessels resemble tracheids very much in
epidermis of young stem, petioles and structure and function. But unlike tracheids
midrib of leaves etc. These are absent in
these are like long tubes arranged in vertical
underground tissues and leaves and stems
row formed of cylindrical cells arranged to
of monocots.
end with their end walls completely
35. (b) During epigeal germination, cotyledons dissolved. These ar e also dead and
come out of the soil. The green cotyledons
lignified.
function as leaves of the seedling. They
40. (b) In isobilateral leaves, the upper epidermis
manufacture food and sustain the young
seedling till the plumule gives rise to new contains specialized cells, i.e., bulliform or
leaves. motor cells. They are highly vacuolate and
can store water, if available. However, in
36. (d) Xylem is the water conducting tissue. It
consists of living cells like parenchyma and case of water deficiency, the bulliform cells
dead cells like tracheary elements. lose water and become flaccid. As a result,
the leaf gets rolled up to reduce the exposed
37. (a) In woody trees, the central portion of stem
is dark in colour. It is hard and tough due surface. The bulliform cells are also useful
to deposition of resins, tannins, gums and in the unr olling of leaf during its
formation of tyloses. This central hard development.
portion is called heart wood. It is formed 41. (c) Pericycle is the outermost layer of stele. In
by secondary growth. Due to cambial dicot stems, pericycle strengths the stem
activity secondary xylem becomes non- and provides protection to the vascular
functional and forms heart wood or bundles. In angiosperms (dicots), pericycle
duramen. It is more durable and little gives rise to lateral roots and contribute to
susceptible to attack of pathogens. The the vascular cambium often diverging into
cambial activity continues in this region. a work cambium.
EBD_7100
B-42 Topicwise AIIMS Solved Papers – BIOLOGY

7 Structural Organisation in Animals


(b) administered large amounts of thyroxine.
TYPE A : MULTIPLE CHOICE QUESTIONS (c) reared on a diet rich in egg yolk.
1. In frog, gastrulation process involves [1997] (d) reared on a diet rich in both egg yolk and
(a) epiboly (b) emboly glucose.
(c) invagination (d) all of these 10. Which of the following type of cell junction is
2. Structure which remains unchanged during not found in animal tissues ? [2013]
metamorphosis of frog's tadpole is [1997] (a) Adhering junction (b) Tight junction
(a) lung (b) heart (c) Gap junction (d) Plasmodesmata
(c) nervous system (d) intestine 11. Identify the figure with its correct function
3. Which gland plays a key role in metamorphosis
of frog ? [1999]
(a) Adrenal (b) Thyroid
(c) Thymus (d) Pancreas
4. Major protein of connective tissue is [2001]
(a) myosin (b) collagen
(c) melanin (d) keratin
5. Outer covering of cartilage is called [2001]
(a) perichondrium (b) periosteum
(c) endosternum (d) peritoneum
6. The protoplasmic segment of a striated muscle Fig :. Adipose connective tissue
fibre is termed as [2001] (a) Areolar connective tissue – Serves as a
support framework for epithelium
(a) sarcoplasm (b) sarcomere
(b) Adipose tissue – Store fats and act as heat
(c) neuromere (d) metamere
insulators
7. Sharpey’s perforating fibres are related with (c) Dense regular tissue – Provide flexibility
(a) heart contraction [2002] (d) Dense irregular tissue – Provide strength
(b) muscle relaxation and elasticity [2014]
(c) fixing of teeth 12. Which of the following statement about cell
(d) none of these
junctions is false? [2015]
8. The type of epithelial cells which line the inner
surface of fallopian tubes, bronchioles and small (i) All the cells of the epithelium are held
bronchi are known as [2006] together with little intercellular materials.
(a) squamous epithelium (ii) In almost all animal tissues specialized
(b) columnar epithelium junction provide both structural and
(c) ciliated epithelium functional link between its individual cells.
(d) cubical epithelium (iii) Tight junctions help to stop substances
9. Tadpoles of frog can be made to grow as giant from leaking across a tissue.
sized tadpoles, if they are [2006] (iv) Adhering junctions provide cementing to
(a) administered antithyroid substance like
keep neighbouring cells together.
thiourea.
Structural Organisation in Animals B-43

(v) Gap junctions provide cytoplasmic (a) smell (b) hearing


channels between cells for passage of ions, (c) respiration (d) touch
small molecules and sometimes big 17. In earthworms, setae are present in all segments
molecules. except [2017]
(a) (ii) and (iii) (b) (i) and (ii) (a) first and the last segments
(b) first segment and the clitellum
(c) Only (v) (d) None of these
(c) first segment
13. i. The shape of the cells may vary with the (d) clitellum and last segments
function they perform
TYPE B : ASSERTION REASON QUESTIONS
ii. Human RBC is about 7.0 m in diameter
iii. Cytoplasm is the main arena of cellular Directions for (Q. 18) : These questions consist of
activities two statements, each printed as Assertion and Reason.
iv. Various chemical r eactions occur in While answering these questions, you are required to
choose any one of the following five responses.
cytoplasm to keep the cell in the living state
(a) If both Assertion and Reason are correct and
[2015]
the Reason is a correct explanation of the
(a) All are correct Assertion.
(b) Only I and II are correct (b) If both Assertion and Reason are correct but
(c) Only IV is correct Reason is not a correct explanation of the
(d) All are wrong Assertion.
14. The figure given below shows the head region (c) If the Assertion is correct but Reason is
of cockroach. Identify A to F. incorrect.
A
(d) If both the Assertion and Reason are incorrect.
Antennae
B (e) If the Assertion is incorrect but the Reason is
correct.
D 18. Assertion : Cartilage and bone are rigid
C connective tissues.
E F Reason : Blood is a connective tissue [2001]
[2016] Directions for (Q. 19) : Each of these questions
(a) A- Compound eye, B-Ocellus, C-Maxilla, contains an Assertion followed by Reason. Read them
D-Mandible, E-Labrum, F-Labium carefully and answer the question on the basis of
(b) A- Ocellus, B-Compound eye, C-Mandible, following options. You have to select the one that
D-Maxilla, E-Labrum, F-Labium best describes the two statements.
(c) A- Ocellus, B-Compound eye, C-Mandible, (a) If both Assertion and Reason are correct and
D-Maxilla, E-Labium, F-Labrum Reason is the correct explanation of Assertion.
(d) A- Ocellus, B-Compound eye, C-Maxilla, (b) If both Assertion and Reason are correct, but
D-Mandible, E-Labrum, F-Labium Reason is not the correct explanation of
15. Male cockroach can be identified from the Assertion.
female by the presence of [2017] (c) If Assertion is correct but Reason is incorrect.
(a) long antennae (d) If both the Assertion and Reason are incorrect.
(b) wingless body 19. Assertion : The squamous epithelium is made
(c) elongated abdomen of a single thin layer of flattened cells with
irregular boundaries.
(d) anal styles
Reason : They are found in walls of blood
16. The sensory papillae in frogs are associated vessels and air sacs of wings. [2017]
with [2017]
EBD_7100
B-44 Topicwise AIIMS Solved Papers – BIOLOGY

Type A : Multiple Choice Questions Endosternum is a collective name for the


apodemes or interior processes of the
1. (d) Gastrulation is the process of formation of sternum in the thoracic of an insect. The
three layers i.e. ectoderm, endoderm and peritoneum is thin membrane that lines the
mesoderm. In frog, these layers are formed abdominal and pelvic cavities and covers
by the processes of epiboly, emboly and most abdominal viscera.
invagination.
6. (b) Sarcomere is the smallest contractile unit
2. (c) Frog’s nervous system once differentiated of striated muscle fibre. Sarcomere occurs
remains as such throughout life. as repeating units along the length of a
3. (b) Thyroxine hormone produced by thyroid myofibril, occupying the region between Z
gland plays a very important role in the lines of the myofibril.
metamorphosis of frog. Sarcoplasm is the cytoplasm of a muscle
4. (b) Collagen is the major protein of connective fibre. It is a water solution containing ATP
tissue. Collagen, in the form of elongated and phosphogens, as well as the enzymes
fibrils is mostly found in fibrous tissues of intermediate and product molecules
such as tendon, ligament and skin, and is involved in many metabolic reactions.
also abundant in cornea cartilage, bone , Neuromeres is a metameric segment of CNS.
blood vessels, the gut, and intervertebral Metamere is a linear series of primitively
disc. Collagen performs a very important similar segments into which the body of
role in ageing processes. higher invertebrate and vertebrate is
Myosin, commonest protein in muscle cells, divisible.
is responsible for the elastic and contractile 7. (c) Sharpey's fibres are collagenous fibres that
properties of muscle. It combines with actin pass from the periodontal membrane into
to form actomyosin. Melanin is a black or the cementum and the jaw bones, fixing the
dark brown pigment that is responsible for teeth firmly in the sockets.
the dark colour of the skin, hair, scales,
8. (c) Columnar ciliated epithelium posses fine
feathers and eyes of animals. Keratin is a
hair like ongrowths, cilia on thier free
fibrous scleroprotein that occurs in the
surfaces. This epithelium lines the nasal
outer layer of the skin and in horny tissues
passages, oviduct (fallopion tube), terminal
such as hair, feathers, nails and hooves.
bronchiole etc. Its major function is
5. (a) Cartilage is a type of connective tissue protection and movement of mucus, urine
consisting of cells (called chondrocytes) and egg in a particular direction.
and though flexible matrix made of collagen,
protein, and sugar. The cartilage is covered Squamous epithelium forms the lining of
on the outside by death of white fibrous cavities such as the mouth, blood vessels,
connective tissue called perichondrium. heart and lungs and make up the outer
layers of the skin. Columnar epithelium
Periosteum is a membrane that lines the forms the lining of the stomach and
outer surface of all bones, except at the intestines. Some columnar cells are
joints of long bones. It contains the blood
specialized for sensory reception such as
vessels and nerves that provide
in the nose, ears and the taste buds of the
nourishment and sensation.
Structural Organisation in Animals B-45

tongue. Cubical epithelium is found in 16. (d) Frog has different types of sense organs
glands and in the lining of the kidney like organs of touch (sensory papillae),
tubules as well as in the ducts of the glands. taste (taste buds), smell (nasal epithelium),
They also constitute the germinal vision (eyes) and hearing (tympanum with
epithelium which produces the egg cells in internal ears).
the female ovary and the sperm cells in the 17. (d) Except the first, the last and clitellar segment
male testes. in each segment bear a ring of tiny curved,
9. (b) Thyroxine helps in the metamorphosis of chitinous structure known as setae. Setae
tadpole. helps in locomotion and copulation.

10. (d) Type B : Assertion Reason Questions


11. (b) Adipose tissue is another type of
18. (e) Cartilage is a semi rigid connective tissue
connective tissue located mainly beneath
that is weaker than bone, but more flexible
the skin. The cells of this tissue are resilient. Cartilage serves to provide
specialised to store fats. structure and support to the body's other
12. (d) All the given statements about cell tissues and also provide a cushioning effect
functions are true. in points. Bone is rigid connective tissue
and forms the skeleton of the body. It is
13. (a) All the given statements are correct.
composed chiefly of calcium phosphate
14. (b) A - Ocellus; B - Compound eye; and calcium carbonate. It also serves as a
C - Mandible; D - Maxilla; E - Labrum; storage area for calcium, playing a large
F - Labium. role in calcium balance in the blood. Blood
15. (d) Both the sexes of cockroach have anal cerci is a fluid connecting tissue. Blood delivers
necessary substances, such as nutrients
which are jointed structures. But in the
and oxygen, to the body's cells (in animals)
male, in addition, there is a paired unjointed
and transports waste products away from
needle-like anal style, which serve to
those same cells.
distinguish between the male and the
female. 19. (b)
EBD_7100
B-46 Topicwise AIIMS Solved Papers – BIOLOGY

8 Cell : The Unit of Life


10. Chromosomes with equal arms are called [2000]
TYPE A : MULTIPLE CHOICE QUESTIONS
(a) metacentric (b) telocentric
1. In many bacteria, cell membrane is invaginated (c) acentric (d) polycentric
and folded to form [1997]
11. A prokaryotic cell lacks [2001]
(a) pili (b) cristae
(a) true nucleus
(c) flagella (d) mesosome
(b) nuclear membrane
2. Cristae helps in [1998]
(c) membrane bound organelles
(a) respiration (b) photosynthesis
(c) transpiration (d) guttation (d) all of the above
3. In nucleoplasm, a spherical body attached to a 12. Which of the following is a single membranous
particular chromosome on a definite position is structure ? [2001]
called [1998] (a) Lysosome (b) Nucleus
(a) nucleolus (b) karyolymph (c) Mitochondria (d) Chloroplast
(c) plasmid (d) reticulum 13. In bacteria, site of respiration is [2001]
4. Which of the following is responsible for (a) mesosome (b) episome
mechanical support and enzyme transport ? (c) plasmid (d) cytoplasm
[1999]
14. Which of the following organelle is related with
(a) Dictyosome (b) Cell membrane
photorespiration? [2002]
(c) E. R. (d) Mitochondria
(a) Peroxisome (b) Nucleus
5. Which of the following is present between cell
(c) Cell wall (d) Lysosome
walls of the plant cells ? [1999]
15. The phagocytosis was first of all seen by
(a) Lomasome (b) Microsome
(c) Lysosome (d) Middle lamella (a) Huxley (b) Haeckel [2002]
6. Rough E. R. differs from smooth E. R. due to the (c) Metchnikoff (d) Darwin
presence of [2000] 16. A chromosome with centromere at one end is
(a) DNA (b) nucleus called [2002]
(c) ribosome (d) enzyme (a) telocentric (b) metacentric
7. Electron microscope was invented by [2000] (c) excentric (d) apocentric
(a) Robert Hooke 17. Plasmodesmata connections help in [2003]
(b) Knoll and Ruska (a) cytoplasmic streaming.
(c) Pasteur (b) synchronous mitotic divisions.
(d) Schwann and Schleiden (c) locomotion of unicellular organisms.
8. Double membrane structure of cell are [2000] (d) movement of substances between cells.
(a) nucleus (b) chloroplast 18. DNA is present in [2004]
(c) mitochondria (d) all of these (a) chromosomes and dictyosomes
9. Hydrolytic enzymes are found in [2000] (b) chloroplasts and lysosomes
(a) peroxisomes (b) lysosomes (c) mitochondria and chloroplasts
(c) lepdosomes (d) lomasomes (d) mitochondria and endoplasmic reticulum
Cell : The Unit of Life B-47

19. Three of the following statements regarding cell (b) lysosomes and peroxisomes
organelles are correct while one is wrong. Which (c) Golgi bodies and' smooth endoplasmic
one is wrong? [2005] reticulum
(a) Lysosomes are double membraned vesicles (d) plastids and inherited via male gamete
budded off from Golgi apparatus and 25. What is common between chloroplasts,
contain digestive enzymes. chromoplasts and leucoplasts ? [2008]
(b) Endoplasmic reticulum consists of a (a) Presence of pigments.
network of membranous tubules and helps (b) Possession of thylakoids and grana.
in transport, synthesis and secretion. (c) Storage of starch, proteins and lipids.
(c) Leucoplasts are bound by two membranes, (d) Ability to multiply by a fission-like process.
lack pigment but contain their own DNA 26. Molecules that are too large to pass through
and protein synthesizing machinery. the pores of a cell membrane may enter the cell
(d) Sphaerosomes are single membrane bound by a process known as [2009]
and are associated with synthesis and (a) hydrolysis (b) pinocytosis
storage of lipids. (c) cyclosis (d) synthesis
20. What is common between chloroplasts, 27. Three morphological forms of golgi complex are
chromoplasts and leucoplasts? [2006]
(a) Lamellae, tubules and vesicles [2012]
(a) Presence of pigments. (b) Cisternae, tubules and vesicles
(b) Possession of thylakoids and grana. (c) Cisternae, tubules and lamellae
(c) Storage of starch, proteins and lipids. (d) Granum, thalykoids and vesicles
(d) Ability to multiply by a fission-like process. 28. Which chromosome may lost during cell
21. In prokaryotes, chromatophores are [2006] division? [2012]
(a) specialized granules responsible for (a) Giant chromosome
colouration of cells. (b) Acentric chromosome
(b) structures responsible for organizing the
(c) Polycentric chromosome
shape of the organism.
(d) Telocentric chromosome
(c) inclusion bodies lying free inside the cells
29. Choose the incorrect match [2013]
for carrying out various metabolic
(a) Nucleus — RNA
activities.
(b) Lysosome — Protein synthesis
(d) internal membrane systems that may
become extensive and complex in (c) Mitochondria — Respiration
photosynthetic bacteria. (d) Cytoskeleton — Microtubules
30. Which of the following statements are correct ?
22. Which of the following is responsible for the
mechanical support, protein synthesis and (i) In prokaryotic cells, a special membranous
enzyme transport? [2007] structure formed by the extension of the
plasma membrane into the cell is known as
(a) Cell membrane
polysome.
(b) Mitochondria
(ii) The smooth endoplasmic reticulum is the
(c) Dictyosome major site for synthesis of glycoproteins.
(d) Endoplasmic reticulum (iii) RuBisCO is the most abundant protein in
23. “Omnis-cellula-e-cellula” was given by[2007] the whole biosphere.
(a) Virchow (b) Hooke (iv) Mitochondria, chloroplasts and
(c) Leeuwenhoek (d) Brown peroxisomes are not considered as part of
24. Genes present in the cytoplasm of eukaryotic endomembrane system. [2016]
cells, are found in [2005, 2008]
(a) (iii) and (iv) (b) (i) and (ii)
(a) mitochondria and inherited via egg
(c) (ii) and (iii) (d) (i) and (iv)
cytoplasm
EBD_7100
B-48 Topicwise AIIMS Solved Papers – BIOLOGY

TYPE B : ASSERTION REASON QUESTIONS 36. Assertion : Eukaryotic cells have the ability to
adopt a variety of shapes and carry out directed
Directions for (Qs. 31-38) : These questions consist movements.
of two statements, each printed as Assertion and Reason : There are three principal types of
Reason. While answering these questions, you are protein filaments-microfilaments, microtubules
required to choose any one of the following five and intermediate filaments, which constitute the
responses. cytoskeleton. [2006]
(a) If both Assertion and Reason are correct and Directions for (Qs. 37-40) : Each of these questions
the Reason is a correct explanation of the contains an Assertion followed by Reason. Read them
Assertion. carefully and answer the question on the basis of
(b) If both Assertion and Reason are correct but following options. You have to select the one that
Reason is not a correct explanation of the best describes the two statements.
Assertion. (a) If both Assertion and Reason are correct and
(c) If the Assertion is correct but Reason is Reason is the correct explanation of Assertion.
incorrect. (b) If both Assertion and Reason are correct, but
(d) If both the Assertion and Reason are incorrect. Reason is not the correct explanation of
(e) If the Assertion is incorrect but the Reason is Assertion.
correct. (c) If Assertion is correct but Reason is incorrect.
(d) If both the Assertion and Reason are incorrect.
31. Assertion : Power house of cell is mitochondria.
37. Assertion : Lipids present in the outer and inner
Reason : ATP is produced in mitochondria.
side of the bilayer membrane are commonly
[2001] different.
32. Assertion : Cell wall is not found in animal cell. Reason : Oligosaccharides are attached to
Reason : Animal cells are covered by cell external surface as well as inner surface of a
membrane. [2001] biomembrane. [2009]
33. Assertion: Organisms are made up of cells. 38. Assertion : Mitochondria and chloroplasts are
Reason: Cells are structural unit of living semi autonomous organelles.
organisms. A cell keeps its chemical composition Reason : They are formed by division of pre-
steady within its boundary. [2002] existing organelles as well as contain DNA but
34. Assertion: Specialization of cells is useful for lack protein synthesizing machinery.
organism. [2005, 2014]
39. Assertion : A cell membrane shows fluid
Reason: It increases the operational efficiency
behaviour.
of an organism. [2002]
Reason : A membrane is a mosaic or composite
35. Assertion: The number of cells in a multicellular
of diverse lipids and proteins.
organism is inversely proportional to size of
[2003, 2008, 2015]
body.
40. Assertion : Centrosomes and centrioles are
Reason: All cells of biological world are alive.
related to each other.
[2002]
Reason : Centrosome usually contains two
cylindrical structures called centrioles.
[2016]
Cell : The Unit of Life B-49

Type A : Multiple Choice Questions 12. (a) Lysosome consists of hydrolytic enzymes,
enclosed in a unit membrane.
1. (d) In prokaryotic cell, cell membrane 13. (a) In bacteria, mesosome helps in cell
invaginate to form mesosomes. They have
respiration.
enzymes, which are useful for respiration.
14. (a) Leaf peroxisomes are associated with
2. (a) Cristae form a part of mitochondria which
endoplasmic reticulum, chloroplast and
helps in cellular respiration.
mitochondria and are involved in
3. (a) A small spherical body attached to the
photorespiration. Photorespiration is light
particular chromosome (nucleolar
induced CO 2 liberation from a C 2
chromosome) in the nucleoplasm is
compound (glycolic acid) of dark phase of
nucleolus. Nucleolus is the principal or
active site for the development of ribosomal photosynthesis.
RNAs & it is essential for spindle formation 15. (c) The phagocytosis was first of all seen by
of ribosomes. Metchnikoff in 1893. Phagocytosis is a
4. (c) ER provides mechanical support and process whereby certain cells & unicellular
enzyme transport in a cell. organisms are capable of ingesting and
5. (d) Adjacent cells in a plant tissue are held digesting solid material.
together by a thin, sticky, amorphous layer 16. (a) When centromere is located at the tip of
of cementing tissue called middle lamella. the chromosome, it is said to be telocentric.
It is made up of Ca and Mg pectate. 17. (d) Plasmodesmata are small fuses that
6. (c) Rough endoplasmic reticulum is rough due connect plant cells to each other, providing
to the presence of ribosomes at its surface. living bridges between cells.
Their attachment to the ER is by means of Plasmodesmata brings the exchange of
protein ribophorin I and II. substances between cells.
7. (b) Electron microscope was invented by Knoll 18. (c) DNA is present in mitochondria and
and Ruska in 1932. chloroplasts (extranuclear DNA). So these
8. (d) Double membrane organelles of the cell are organelles are self replicating.
mitochondria, chloroplast and nucleus. 19. (a) Lysosomes are single membrane bound
9. (b) Lysosomes are spherical, single membrane organelles rich in hydrolytic enzymes.
limited vesicles, containing hydrolytic
20. (a) C4 pathway/Hatch and Slack pathway
enzymes working at acidic pH. Lysosome
ensures the Calvin cycle to be operated
is also known as suicidal bag/sac as they
contain hydrolytic enzymes. only in bundle sheath cell. It is an
adaptation to photorespiratory loss.
10. (a) Metacentric chromosomes have equal
Therefore, C 4 plants are adapted to
sized chromatids i.e. they have centromere
in the centre. photorespiratory loss.
11. (d) Prokaryotic cells, among the tiniest of all 21. (d) In prokaryotes, chromatophores are
cells, has one envelope system with no internal membrane system that may
membrane lined internal organelles except become extensive and complex in
thylakoid if present. A prokaryotic cell photosynthetic bacteria. It is structurally
lacks membrane bound organelles, nuclear and functionally similar to eukaryotic
membrane and histone proteins. chloroplast.
EBD_7100
B-50 Topicwise AIIMS Solved Papers – BIOLOGY
22. (d) Endoplasmic reticulum (ER) is a system of molecules. Hydrolysis, is the process of
flattened membranes running through the using water to split molecules apart.
cytoplasm. Rough ER containing Cyclosis is a mechanism for transporting
ribosomes is mainly concerned with materials within a cell, by the cytoplasm
protein synthesis and transport of swirling. Synthesis is the process of
enzymes. It also functions as cytoskeleton building up molecules within the cell.
by giving mechanical support to the 27. (b) Three morphological forms of golgi
cytoplasm. complex are cisternae, tubules and vesicles.
Varied number of cisternae are present in a
23. (a) The cell is the basic structural and
Golgi complex.
functional unit of living organisms. In 1855,
Rudolf Virchow showed that all cells arise The Golgi cisternae are concentrically
from the pre-existing cells by cell division arranged near the nucleus with distinct
or Omnis-cellula-e-cellula. Robert Hooke convex cis or the forming face and concave
trans or the maturing face.
was the first one to find out the basic units
of life and termed them as cells. Anton van Tubules are long flattened structure while
Leeuwenhoek was the one who observed vesicles are round or oval structure.
unicellular organisms including bacteria. 28. (b) Acentric chromosome may be lost during
Robert Brown described the nucleus as a cell division as centromere is absent in
characteristic spherical body in plant cells. them. During metaphase and anaphase,
spindle fibres are not attached with them
24. (a) Genes pr esent in the cytoplasm of
and so they are not able to reach to the
eukaryotic cells are found in mitochondria
poles.
and inherited via egg cytoplasm.
29. (b) Protein synthesis takes place in ribosomes,
25. (c) Chromoplasts are plastids responsible for
which are attached to surface of
pigment synthesis and storage. They, like
endoplasmic reticulum by ribophorin-I and
all other plastids (including chloroplasts
ribophorin-II. About 50 hydrolytic enzymes
and leucoplasts), are organelles found in are found in the lysosome. They include
specific photosynthetic eukaryotic proteases, nucleases, glycosidases, lipases,
species. Chloroplasts con duct phospholipases, phosphatases and
photosynthesis. Chloroplasts absorb light sulphatases. All lysosomal enzymes are acid
and use it in conjunction with water and hydrolases and optimally active at pH-5.0
carbon dioxide to produce sugars.
30. (a) The special membranous structure formed
Leucoplasts are non-pigmented, in contrast
by the extension of prokaryotic plasma
to other plastids such as the chloroplast. membrane is known as mesosome while
Lacking pigments, leucoplasts are not polysome is structure formed by
green, so they are predictably located in combination of many ribosomes.
roots and non-photosynthetic tissues of
SER is the major site of synthesis of lipids.
plants. They may become specialized for
The site of protein synthesis is RER.
bulk storage of starch, lipid or protein and
are then known as amyloplasts, elaioplasts, Type B : Assertion Reason Questions
or proteinoplasts respectively.
31. (b) Mitochondria are called power house of a
26. (b) Pinocytosis, or pinching in of the cell
cell because they produce large amount of
membrane, allows cells, such as the
energy in the form of ATP.
Paramecium, to capture larger food
Cell : The Unit of Life B-51

32. (a) Animal cells are covered by semipermeable 38. (c)


plasma membrane. Cell wall is absent in 39. (a) The cell membrane also called the plasma
animal because cell wall is incompatible with membrane, plasmalemma, or “phospholipid
the way in which an animal moves and bilayer” is a selectively permeable lipid
grows.
bilayer found in all cells. It contains a wide
33. (a) Cells are the basic structural and functional variety of biological molecules, primarily
unit of organism. proteins and lipids, which are involved in a
34. (a) Specialization of the cell increases the vast array of cellular processes such as cell
efficiency of the cell for a particular adhesion, ion channel conductance and
function. cell signaling. The plasma membrane also
35. (d) The size and shape of the cell in serves as the attachment point for both the
multicellular organism depends upon the intracellular cytoskeleton and, if present,
location and function performed by them. the extracellular cell wall.
36. (b) Eukaryotic cells contain three types of 40. (a) The centrosome is the main place where
filaments as microtubules, microfilament cell microtubules get organized.
and intermediate filament which give
Centrosome usually contains two
definite shape to the cell and also helps in
cylindrical structure called centrioles.
directional movement.
Centrioles are composed of grouping of
37. (c) Lipids present in the outer and inner side of
microtubules arranged in 9 + 3 pattern. The
the bilayer are commonly different, e.g.,
pattern is so named because a ring of 9
lecithin on the outer side and cephalin on
microtubule "triplets" are arranged at right
the inner side of erythrocyte membrane.
angles to one another. Centrioles, found
Oligosaccoharides are attached to external
in animal cells, help to organize the
surface of lipids and proteins of a
bio-membrane. They are absent on the inner assembly of microtubules during cell
side. division. Centrioles replicate during the
interphase stage of mitosis and meiosis.
EBD_7100
B-52 Topicwise AIIMS Solved Papers – BIOLOGY

9 Biomolecules

TYPE A : MULTIPLE CHOICE QUESTIONS 10. Which of the following set of three items are not
true as each set belongs to the category
1. Which is the derivative of amino acid ? [1999] mentioned against them [2005]
(a) Epinephrine (b) Estrogen (a) Lysine, glycine, thiamine - Amino acids
(c) Progesterone (d) All of these (b) Myosin, oxytocin and gastrin - Hormones
2. High energy bond of ATP are present in between (c) Rennin, helicase and hyaluronidase -
[1999] Enzyme
(a) C – C (b) C – O (d) Optic nerve, occulomotor, vagus - Sensory
(c) C – N (d) O – P nerves
3. Who coined the term zymase? [1999] 11. The figure given below show three velocity-
(a) Pasteur (b) Buchner substrate concentration curves for an enzyme
reaction. What do the curves a, b and c depict
(c) Kuhne (d) Sumner
respectively? [2006]
4. Apoenzyme is [2000]
(a) protein (b) lipid
(c) sugar (d) vitamin
a b
5. Gamma globulins are synthesized inside c
Initial velocity V0

(a) liver [2000]


(b) kidney
(c) bone marrow
(d) lymph and lymphoid tissues Substrate concentration [S]

6. Proteins are [2002]


(a) a -normal enzyme reaction,
(a) polysaccharides (b) polyamides
b- competitive inhibition,
(c) polynucleotides (d) polyglycol
c- non-competitive inhibition.
7. Which of the following gives Fehling’s test?
(b) a -enzyme with an allosteric modulator
(a) Pectin (b) Sucrose [2002]
added,
(c) Cellulose (d) Glucose b - normal enzyme activity,
8. The nicotinamide is synthesized in our body c - competitive inhibition.
from [2002] (c) a - enyzme with an allosteric stimulator,
(a) tryptophan (b) tryosine b - competitive inhibitor added,
(c) valine (d) alanine c - normal enzyme reaction.
9. An example of competitive inhibition of an (d) a - normal enzyme reaction,
enzyme is the inhibition of [2003] b - non-competitive inhibitor added,
(a) succinic dehydrogenase by malonic acid c - allosteric inhibitor added.
(b) cytochrome oxidase by cyanide 12. Which of the following contain -l, 4 linkage?
(c) hexokinase by glucose-6-phosphate (a) Maltose (b) Sucrose [2007]
(d) carbonic anhydrase by carbon dioxide (c) Lactose (d) Fructose
Biomolecules B-53

13. Which statement is true? [2007] 20. The Km value of the enzyme is the value of
(a) Adenine has 4 nitrogen atoms. the substrate concentration at which the
(b) Cytosine has 3 nitrogen atoms. reaction reaches to [2014]
(c) Guanosine has 3 nitrogen atoms. (a) Zero (b) 2 Vmax
(d) Uracil has 5 nitrogen atoms. (c) ½ Vmax (d) ¼ Vmax
14. Michaelis constant Km is equal to [2010] 21. The following diagrams represent the
K1 K 2 K3 nitrogenous bases of nucleic acid molecules.
(a) K – K (b) K1
2 3 Identify the correct combination
K 2 – K3 K1 K2
(c) K1 (d) K3
15. Alpha-keratin is a protein present in [2010]
(a) blood (b) skin
(c) lymph (d) eggs
16. Which one of th e following statements
regarding starch and cellulose is not correct?
[2010]
(a) Both of them are of plant origin.
(b) Both of them are polymers.
(c) Both of them give colour with iodine.
(d) Both of them are made up of glucose
molecules.
17. Which of the following type of enzyme is not
matched correctly with the molecule that it breaks [2015]
down? [2013]
(a) A-uracil, B-adenine, C-thymine, D-guanine,
(a) Amylase–starch
(b) Lipase–starch E- cytosine
(c) Protease–proteins (b) A - uracil, B-guanine, C-cytosine,
(d) Disaccharidase–sugars D-adenine, E-thymine
18. The diagram illustrates energy changes in an (c) A-uracil, B - guanin e, C-thymine,
enzyme controlled reaction. [2013]
D-adenine, E-cytosine
(d) A-thymine, B-guanine, C-uracil, D-adenine,
Z E-cytosine.
energy

reactants
Y
22. The given graph shows the effect of substrate
products X concentration on the rate of reaction of the
reaction enzyme green -gram -phosphatase. What does
Which of the following represents the lowering the graph indicate ?
of the activation energy?
(a) X (b) Y
Velocity (v)

(c) Z (d) Z – Y
19. Which one of the following is a non - reducing
carbohydrate? [2014]
(a) Maltose
(b) Sucrose
O
(c) Lactose Substrate Concentration (s)
(d) Ribose 5 - phosphate
[2015]
EBD_7100
B-54 Topicwise AIIMS Solved Papers – BIOLOGY

(a) The rate of enzyme reaction is directly (c) If the Assertion is correct but Reason is
proportional to the substrate concentration incorrect.
(d) If both the Assertion and Reason are incorrect.
(b) Presence of an enzyme inhibitor in the
(e) If the Assertion is incorrect but the Reason is
reaction mixture
correct.
(c) Formation of an enzyme-substrate complex
26. Assertion : Enzymes have active sites and
(d) At higher substrate-concentration the pH substrates have reactive sites on their surface
increases. respectively.
23. Inorganic catalyst work efficiently at Reason : Active and reactive sites push the
________temperature and ________ pressure. enzyme and substrate molecules away from each
[2016] other. [1999]
(a) high, low (b) low, low 27. Assertion : Vegetable oils are fats which are
(c) low, high (d) high, high present in plant cells in soluble form.
24. Refer the given structure of adenylic acid. In Reason : Vegetable oils occur only in cells of
this identify A. embryo. [2007]
O Directions for (Qs. 28-37) : Each of these questions
|| O contains an Assertion followed by Reason. Read them
HO – P – OCH2 N-base carefully and answer the question on the basis of
|
OH A following options. You have to select the one that
best describes the two statements.
(a) If both Assertion and Reason are correct and
[2016] Reason is the correct explanation of Assertion.
(a) Glycosidic bond (b) Phosphate bond (b) If both Assertion and Reason are correct, but
(c) Ester bond (d) Ionic bond Reason is not the correct explanation of
25. Nucleotides are building blocks of nucleic acids. Assertion.
Each nucleotide is a composite molecule formed (c) If Assertion is correct but Reason is incorrect.
by [2017] (d) If both the Assertion and Reason are incorrect.
(a) base-sugar-phosphate. 28. Assertion : Human diet should compulsorily
(b) base-sugar-OH. contain glycine, serine and tyrosine. [2010]
(c) (base-sugar-phosphate)n. Reason : Essential amino acids can not be
(d) sugar-phosphate. synthesized in the human body.
TYPE B : ASSERTION REASON QUESTIONS 29. Assertion : Unsaturated fats are more reactive
compared with the saturated fats. [2010]
Directions for (Qs. 26-27) : These questions consist Reason : Unsaturated fats have only single
of two statements, each printed as Assertion and bonds in their structure.
Reason. While answering these questions, you are
30. Assertion : The amino acid glycine comes under
required to choose any one of the following five
the category of nonessential amino acids.
responses.
Reason : This is due to the fact that it can not be
(a) If both Assertion and Reason are correct and
synthesised in the body. [2011]
the Reason is a correct explanation of the
Assertion. 31. Assertion : Allosteric enzymes show feed back
inhibition.
(b) If both Assertion and Reason are correct but
Reason is not a correct explanation of the Reason : The inhibitor is competitive. [2012]
Assertion.
Biomolecules B-55

32. Assertion : Coenzymes serve as co-factors in a Reason : A complete, catalytically active enzyme
number of different enzyme catalyzed reactions. together with its bound prosthetic group is
Reason : Coenzymes and prosthetic groups are called apoenzyme. [2016]
cofactors. [2013] 36. Assertion : Glycosidic bonds are formed by
33. Assertion : Enzymes lower the activation dehydration.
energy. Reason : In polysaccharides, individual
Reason : A substrate molecule can be acted upon monosaccharide is linked by glycosidic bond.
by a particular enzyme. [2014] [2016]
34. Assertion : Comparative biochemistry provides
37. Assertion : In a DNA molecule, A–T rich parts
a strong evidence in favour of common ancestory
melt before G–C rich parts.
of living beings.
Reason: In between A and T there are three
Reason : Genetic code is universal. [2015]
H–bond, whereas in between G and C there are
35. Assertion : A co-enzymes or metal ions that is
two H-bonds. [2017]
very tightly bound to enzyme protein is called
prosthetic group.
EBD_7100
B-56 Topicwise AIIMS Solved Papers – BIOLOGY

Type A : Multiple Choice Questions competitive inhibitor decreases the vmax of


the reaction i.e. it can not be overcome by
1. (a) Epinephrine is derived from tyrosine amino increase in substrate concentration.
acid.
12. (c) Lactose or milk sugar, found exclusively in
2. (d) High energy bonds of ATP are between
milk, contain -1,4 linkage. It is a
O~P. disaccharide formed by combination of
3. (b) Buchner coined the term zymase for the galactose and glucose by means of a
complex of biocatalysts extracted from chemical reaction called as condensation
yeast and taking part in alcoholic reaction.
fermentation.
13. (a) Nucleotides contain carbon, hydrogen,
4. (a) Apoen zyme is the protein part of oxygen, nitr ogen an d phosphorous.
holoenzyme. Nucleotides are either purines or
5. (a) Gamma globulins ( ) are synthesized by pyrimidines. Adenine and guanine are the
B-lymphocytes and stem cells found in the two purines which are the 9-membered
liver during foetal stage and bone marrow double ringed compound where each ring
cells in the adult stage. possesses four nitrogen atoms. Thymine,
6. (b) Proteins are the polymers of basic units uracil, and cytosine are the pyrimidines
amino acids and hence are polyamides. which are single ringed nitrogenous
compounds.
7. (d) Glucose is an aldehydic sugar and their free
–CHO part converts Cu +2 to Cu +1 14. (b) Mich aels constant K m is equal to
(Fehling's reagent). K 2 K3
8. (a) The enzyme nicotinamide can be K1 .
synthesized in small quantities from amino
15. (b) Alpha-keratin is present in high quantity in
acid, tryptophan.
skin and epidermal appendages like hair and
9. (a) Enzyme inhibition caused by a substance
nail.
resembling substrate molecule through
16. (c) Starch is the reserve substance in plant cells
blocking its active site is competitive
whereas cellulose is the most important
inhibition. Malonate closely resembles
structural component of the cell wall of
succinate in structure that inhibits the
plants. Both starch and cellulose are
action of succinic dehydrogenase.
polymers of glucose. Starch is a branched
10. (c) Thiamine is a nitrogen base; myosin is a polymers of -D glucose units which are
muscle protein; occulomotor nerve is motor linked by –1, 4 glycosidic bonds (but at
and vagus is mixed type. branching, by –1, 6 glycosidic bonds).
11. (a) The effect of a competitive inhibitor is Cellulose is a linear polymer of –D–
reversed by increasing substrate glucose units connected through -1, 4
concentration. At a sufficiently high glycosidic bonds. In contrast to starch and
substrate concentration, the reaction glycogen, cellulose is insoluble in ordinary
velocity reaches the vmax observed in the solvents and is not hydrolysed by boiling
absence of inhibitor whereas non- dilute acids. It gives no colour with iodine.
Biomolecules B-57

17. (b) Lipases are enzymes found in the small 27. (d) Vegetable oils and fats are present in plants
intestine of humans that help in the in insoluble form. They are extracted mostly
breakdown of fats. from seeds. In several cereals, they are
18. (d) The energy required to initiate a reaction is obtained from embryo. Olive and palm oils
known as the activation energy, Ea. The are obtained from flesly pericarp of the fruit.
bold curve shows the uncatalysed reaction Sometimes oils are also extracted from
with Ea = Z, whilst Y represents the Ea of roots, stem and leaves.
the catalysed reaction (dotted curve). The 28. (d) Essential amino acids are those which are
Ea is thus lowered by (Z – Y). taken from food and not synthesized in the
19. (b) Sucrose is classified under non-reducing body whereas non-essential amino acids
sugar because it does not have any free need not be supplied in the diet and are
aldehyde or keto group. synthesized in the body. Glycine, serine and
tyrosine are non-essential amino acids.
20. (c) The concentration of substrate at which
29. (c) Compound having double bond in their
velocity of enzymatic action reaches half
structure are more unstable compound in
of its maximum value, is called Km value or
comparison to single bond holder
Michaelis constant.
compounds. Unsaturated fats those have
21. (a)
double bonds in their structures are more
22. (b) Even though the substrate concentration
reactive than saturated fats.
increases the velocity is decreasing there
by showing a presence of inhibitor. 30. (c) Non-essential amino acids are those amino
acids which need not be supplied in the diet
23. (d) Inorganic catalyst work efficiently at high
because they can be synthesised by the
temperature and high pressure. Inorganic
body, particularly from carbohydrate
catalysts speed up reactions, but they do
metabolites, Glycine is one such non
not have carbon-hydrogen atoms. An
essential amino acid. On the contrary,
example of this is magnesium sulphate,
essential amino acids are those amino acids
which is a compound used to speed up
which can not be synthesised in the animal
some reactions in the chemistry lab.
body and must be supplied with food in
24. (c) Phosphate is bound to pentose sugar by
adequate amounts. Out of twenty amino
ester bond.
acids, eight are considered essential in
25. (a) Nucleotides are the building blocks of human diet.
nucleic acid. Each nucleotide consists of
31. (c) Feed back inhibition is a type of reversible
three parts: a sugar (ribose for RNA and
inhibition found in allosteric enzymes. The
deoxyribose for DNA), a phosphate, and a
inhibitor is noncompetitive and is usually a
nitrogenous base.
low molecular intermediate or product of
Type B : Assertion Reason Questions metabolic pathway having a chain of
reactions involving a number of enzymes.
26. (d) Enzymes are biocatalyst that increases the
32. (b) Cofactor may be inorganic or organic in
speed of a chemical reaction without
nature. Organic cofactors are of two types,
themselves undergoing any permanent
coenzymes and prosthetic groups.
chemical change. Enzymes have active
Coenzymes are easily separable nonprotein
sites to bind the substrate during catalyzed
organic cofactors. Prosthetic groups are
reaction.
EBD_7100
B-58 Topicwise AIIMS Solved Papers – BIOLOGY

non-protein organic cofactors firmly a protein that forms an active enzyme system
attached to apoenzymes (protein part of by combination with a coenzyme and
enzyme). determines the specificity of this system for
33. (b) Activation energy is an external supply of a substrate.
energy which is needed for the initiation of 36. (b) In polysaccharides, individual
the chemical reaction. Activation energy monosaccharide is linked by glycosidic
required for such a large number of reactions bond. This bond is formed between two
cannot be provided by living systems. carbon atoms of two adjacent
Enzymes lower the activation energy monosaccharides. A glycosidic bond is a
required for a reaction. Enzymes are type of covalent bond that joins a
generally specific for their substrates. carbohydrate molecule to another group,
34. (b) Comparative biochemistry provides a strong which may or may not be another
evidence for common ancestors of living carbohydrate. Glycosidic bonds are formed
beings (e.g. proteins lymph, enzymes, by dehydration.
hormones, blood groups etc.) 37. (c) In a DNA molecule, A-T rich parts melt
35. (c) Prosthetic group is an organic substance before G-C rich parts because there are two
which is thermostable and firmly attached H-bond between A and T whereas in
to the protein or apoenzyme portion during between G and C, there are three H-bond.
the complete catalytic cycle. Apoenzyme is
Cell Cycle and Cell Division B-59

10 Cell Cycle and Cell Division

7. Many cells function properly and divide


TYPE A : MULTIPLE CHOICE QUESTIONS mitotically even though they do not have
1. The correct sequence in cell cycle is [1999] [2011]
(a) S - G1 - G2 - M (b) S - M - G1 - G2 (a) plasma membrane
(b) cytoskeleton
(c) G1 - S - G2 - M (d) M1 - G1 - G2 - S
(c) mitochondria
2. Colchicine prevents the mitosis of cells at which (d) plastids
of the following stage? [2000] 8. The stage of meiosis where centromere separate
(a) Anaphase (b) Metaphase [2013]
(c) Prophase (d) Interphase (a) metaphase I (b) metaphase II
3. Spindle fibres of mitotic cells are made up of (c) anaphase I (d) anaphase II
(a) tubulin (b) actin [2001] 9. During meiosis I, the chromosomes start pairing
(c) myosin (d) collagen at [2014]
4. When synapsis is complete all along the (a) Leptotene (b) Zygotene
chromosome, the cell is said to have entered a (c) Pachytene (d) Diplotene
10. How many mitotic divisions are needed for a
stage called [2005]
single cell to make 128 cells? [2016]
(a) zygotene (b) pachytene (a) 7 (b) 14
(c) diplotene (d) diakinesis
(c) 28 (d) 64
5. Which one of the following precedes re-
formation of the nuclear envelope during M- 11. Match the description (given in column I) with
phase of the cell cycle? [2008] correct stage of prophase I (given column II)
(a) Decondensation from chromosomes and and choose the correct option. [2017]
reassembly of the nuclear lamina. Column I Column II
(b) Transcription from chromosomes and A. Chromosomes are I. Pachytene
reassembly of the nuclear lamina. moved to spindle
(c) Formation of the contractile ring and equator
formation of the phragmoplast. B. Centromere splits and II. Zygotene
(d) Formation of the contractile ring and chromatids move apart
transcription from chromosomes. C. Pairing between III. Anaphase
6. During which stages (or prophase I substages) homologous
of meiosis do you expect to find the bivalents chromosomes takes
and DNA replication respectively? [2009] place
(a) Pachytene and interphase (between two D. Crossing between IV. Metaphase
meiotic divisions) homologous
(b) Pachytene and interphase (just prior to chromosomes
prophase I) (a) A – I; B – II; C – III; D – IV
(c) Pachytene and S phase (of interphase just
(b) A – II; B – III; C – IV; D – I
prior to prophase I)
(d) Zygotene and S phase (of interphase prior (c) A – IV; B – III; C – II; D – I
to prophase I) (d) A – III; B – I; C – IV; D – II
EBD_7100
B-60 Topicwise AIIMS Solved Papers – BIOLOGY

TYPE B : ASSERTION REASON QUESTIONS (a) If both Assertion and Reason are correct and
Reason is the correct explanation of Assertion.
Directions for (Q. 12) : These questions consist of (b) If both Assertion and Reason are correct, but
two statements, each printed as Assertion and Reason. Reason is not the correct explanation of
While answering these questions, you are required to Assertion.
choose any one of the following five responses. (c) If Assertion is correct but Reason is incorrect.
(a) If both Assertion and Reason are correct and (d) If both the Assertion and Reason are incorrect.
the Reason is a correct explanation of the 13. Assertion : Meiosis II is known as equational
Assertion. or homotypic division. [2010]
(b) If both Assertion and Reason are correct but Reason :Meiosis II produces same number of
Reason is not a correct explanation of the chromosome in cell.
Assertion. 14. Assertion : Interphase is resting stage.
(c) If the Assertion is correct but Reason is Reason : The interphase cell is metabolically
incorrect. inactive. [2012]
(d) If both the Assertion and Reason are incorrect. 15. Assertion : During zygotene, chromosomes
show bivalent stage.
(e) If the Assertion is incorrect but the Reason is
correct. Reason : Bivalent is half the number of
chromosomes. [2013]
12. Assertion: Meiosis results in production of
haploid cells. 16. Assertion : The stage between two mitotic
Reason: Synapses occurs during leptotene. divisions is called interkinesis.
[1998] Reason : Interkinesis is generally short lived.
Directions for (Qs. 13-17) : Each of these questions [2016]
contains an Assertion followed by Reason. Read them 17. Assertion : Diplotene is characterized by the
carefully and answer the question on the basis of presence of chiasmata.
following options. You have to select the one that Reason : Diplotene can last for months and years
best describes the two statements. in oocytes of some vertebrates. [2016]
Cell Cycle and Cell Division B-61

Type A : Multiple Choice Questions phenomenon is called synapsis and it


occurs durin g zygotene stage. DNA
1. (c) The correct sequence of cell cycle is G1, S,
replication occurs during S phase or
G2 and M.
synthetic phase which is the second phase
2. (c) Colchicine is an alkaloid derived from of interphase.
(colchicum autumnale) arrests the spindle 7. (d) Many cells function properly and
formation at the end of prophase. Hence, it
divide mitotically even in the absence of
is also called 'Mitotic Poison'. plastids.
3. (a) Spindle fibres of mitotic cells are made up 8. (d)
of microtubules. It consists of protein
9. (b) During zygotene, a substage of Prophase I
tubulin that maintain cell shape, serve a
of meiosis I, where chromosomes start
tracks for organelle movement & help in
pairing together is called synapsis. Such
cell division by getting attached to the
paired chromosomes are called as
centromeres of bivalents. They are
homologous chromosomes. A complex
arranged on equator due to congression
structure i.e., synaptonemal complex is
movements. The movement of bivalents or
formed by a pair of synapsed homologous
chromosomes occur towards the poles as
chromosomes called a bivalent or a tetrad.
a result of spindle fibres contraction.
10. (a) In mitosis, a single cell divides to form two
4. (b) Synapse stabilizes the paired condition of
daughter cells. So, the number of mitotic
chromosomes in zygotene stage. After this
divisions can be calculated by 2n.
the cell enters the pachytene stage.
where, n is the number of division of cell.
5. (a) In most eukaryotes, the nuclear envelope
2n = 128
that separates the DNA from the cytoplasm
2n = 27
disassembles. The chromosomes align
themselves in a line spanning the cell. n=7
Microtubules, essentially miniature strings, Thus, 7 mitotic divisions are needed for a
pulls out from opposite ends of the cell single cell to make 128 cells.
and shorten, pulling apart the sister 11. (c) A – IV; B – III; C – II; D – I
chromatids of each chromosome. As a Metaphase – Chromosomes are moved
matter of convention, each sister chromatid to spindle fibre.
is now considered a chromosome, so they Anaphase – Centromere splits an d
are renamed to sister chromosomes. As the chromatids move apart.
cell elongates, corresponding sister Zygotene – Pairing between
chromosomes are pulled toward opposite
homologous chromosomes
ends. A new nuclear envelope forms
takes place.
around the separated sister chromosomes.
Pachytene – Crossing between
6. (d) In bivalent formation of chromosomes
homologous
durin g meiosis, the homologous
chromosomes occurs.
chromosomes are arranged in pairs. The
EBD_7100
B-62 Topicwise AIIMS Solved Papers – BIOLOGY

Type B : Assertion Reason Questions 15. (b) During zygotene, because of the pairing of
the homologues, the nucleus contains half
12. (c) Synapsis occurs during zygotene stage. the number of chromosomes. Each unit is a
Synapsis is the pairing of homologous bivalent composed of two homologous
chromosomes which leads to formation of chromosomes.
bivalents. 16. (d) Interkinesis or interphase II is a period of
13. (a) Meiosis II is known as equational or rest that cells of some species enter during
homotypic division like mitosis. It ensures meiosis, between meiosis I and meiosis II.
the maintenance of constant number of No DNA replication occurs during
chromosomes from generation to interkinesis however it does occur during
generation on a species. the interphase I stage of meiosis.
14. (c) Previously interphase is called resting stage Interkinesis is generally short lived.
because there is no apparent activity 17. (b) Diplotene is the longest and most active
related to cell division. The interphase cell subphase of prophase I of meiosis. The
is metabolically quite active. Interphase beginning of diplotene is recognized by
consist of three subphases (G1 , G2 and S). the dissolution of synaptonemal complex
Synthesis of DNA occurs in S phase. G1 is and the tendency of the recombined
the period between the end of mitosis and homologous chromosomes of the
the start of S phase. G2 is the phase bivalents to separate from each other
between S phase and the next mitosis. As except at the sites of crossovers. These
the synthesis of DNA and proteins occurs X-shaped structures are called chiasmata.
in interphase so, it is considered as Diplotene can last for months and years in
metabolically active phase. oocytes of some vertebrate.
11 Transport in Plants

8. Water potential of pure water and its solution


TYPE A : MULTIPLE CHOICE QUESTIONS
are [2013]
1. Translocation of organic materials in plants is (a) 0 and 1
explained by [1997] (b) 0 and 0
(c) 0 and more than one
(a) active transport
(d) 0 and less than 1.
(b) transpiration pull 9. In which method of transport in plasma
(c) inhibition theory membrane does not require carrier molecule?
(d) mass flow hypothesis [2014]
2. The main function of phloem is translocation of (a) Active transport
(b) Facilitated diffusion
(a) food (b) water [1998] (c) Simple diffusion
(c) mineral (d) air (d) Na+ – K+ pump
3. In rainy season, door gets swelled due to [2001] 10. Seed increase in its volume by the adsorption of
(a) imbibition (b) diffusion water through [2014]
(c) transpiration (d) respiration (a) Osmosis (b) Plasmolysis
(c) Imbibition (d) Diffusion
4. Which of the following helps in ascent of sap?
11. Minerals are known to enter the plant root by
[2007] means of a number of mechanisms, including all
(a) Root pressure (b) Transpiration except one of the following. Which one of the
( c) Capillarity (d) All of these following is NOT a mechanism for moving
5. Hydroponics is [2007] minerals into roots? [2015]
(a) Foliar feeding (b) Active transport
(a) nutrient less culture
(c) Proton (H+) pump (d) Cation exchange
(b) water less culture
12. A botanist discovered a mutant plant that was
(c) soilless culture unable to produce materials that form casparian
(d) none of these strip. This plant would be [2015]
6. During Na+ – K+ pump [2010] (a) unable to transport water or solutes to the
(a) 3Na+ and 2K+ are transported leaves.
(b) unable to use its sugar as a sugar sink.
(b) 1Na+ and 2K+ are transported
(c) able to exert greater root pressure than the
(c) 3 Na+ and 3K+ are transported normal plant.
(d) Depends on requirement of cell (d) unable to control amounts of water and solutes
7. Excessive loss of water causes wilting of leaves, it absorbs.
it can be prevented by : [2012] 13. If a cell A with D.P.D. 4 bars is connected to cell
B, C, D whose O.P. and T.P. are respectively 4
(a) Keeping the plant in bright light
and 4, 10 and 5 and 7 and 3 bars, the flow of
(b) Spraying the plant with alcohol water will be [2015]
(c) Applying vaseline on the leaf surface (a) A and D to B and C
(d) Adding high amounts of fertilizers to the (b) A to B, C and D
soil (c) B to A, C and D
(d) C to A, B and D
EBD_7100
B-64 Topicwise AIIMS Solved Papers – BIOLOGY
14. A boy is studying transport of a certain type of (d) If both the Assertion and Reason are incorrect.
molecules into cell. He finds that transport slows (e) If the Assertion is incorrect but the Reason is
down when the cells are poisoned with a correct.
chemical that inhibits energy production. Under 16. Assertion : Waxy and cutin coating on plant
normal circumstances, the molecules studied by parts reduce the transpiration.
the boy is probably transported by [2016] Reason : These adaptation are found in
(a) simple diffusion xerophytes. [1999]
(b) osmosis 17. Assertion : Water and mineral uptake by root
(c) active transport hairs from the soil occurs through apoplast until
(d) facilitated diffusion it reaches endodermis.
15. Which of the following statements is/are not Reason : Casparian strips in endodermis are
incorrect? [2017] suberized. [2003]
(i) Water and minerals, and food are generally 18. Assertion :When the ambient temperature is
moved by a mass or bulk flow system. high and soil contains excess of water, the plants
(ii) Bulk flow can be achieved either through a tend to lose water in the form of droplets from
positive hydrostatic pressure gradient or a lenticels.
negative hydrostatic pressure gradient. Reason : Root pressure regulates the rate of loss
(iii) The bulk movement of substances through of water form lenticels. [2006]
the conducting tissues of plants is called Directions for (Qs.19-21) : Each of these questions
translocation. contains an Assertion followed by Reason. Read them
(iv) Xylem translocates organic and inorganic carefully and answer the question on the basis of
solutes, mainly from roots to the aerial parts following options. You have to select the one that
of the plants.
best describes the two statements.
(v) Phloem translocates water, mineral salts,
some organic nitrogen and hormones, from (a) If both Assertion and Reason are correct and
the leaves to other parts of the plants. Reason is the correct explanation of Assertion.
(b) If both Assertion and Reason are correct, but
(a) (ii), (iii) and (v)
Reason is not the correct explanation of
(b) (ii), (iii) and (iv)
Assertion.
(c) (iv) and (v) (c) If Assertion is correct but Reason is incorrect.
(d) (ii) and (v) (d) If both the Assertion and Reason are incorrect.
TYPE B : ASSERTION REASON QUESTIONS 19. Assertion : Upward movement of water is called
ascent of sap.
Directions for (Qs. 16-18) : These questions consist
Reason : Upward movement of water occurs
of two statements, each printed as Assertion and
through xylem and phloem. [2013]
Reason. While answering these questions, you are
required to choose any one of the following five 20. Assertion : Long distance flow of photo
responses. assimilates in plants occurs through sieve tubes.
Reason : Mature sieve tubes have parietal
(a) If both Assertion and Reason are correct and
cytoplasm and perforated sieve plates.
the Reason is a correct explanation of the
[2012, 2015]
Assertion.
21. Assertion : Light is very important factor in
(b) If both Assertion and Reason are correct but
transpiration.
Reason is not a correct explanation of the
Reason : Light induces stomatal opening and
Assertion.
darkness closin g of stomata. Therefore,
(c) If the Assertion is correct but Reason is transpiration increases in light and decreases in
incorrect.
dark. [1999, 2015]
Transport in Plants B-65

Type A : Multiple Choice Questions 9. (c)


1. (a) Active transport is the mediated transport 10. (c) Imbibition is the process of adsorption of
of biochemicals, and other atomic/ water by hydrophilic surfaces of a
molecular substances, across membranes. substance without forming a solution. It is
Unlike passive transport, this process a type of diffusion by which movement of
requires chemical energy in the form of water takes place along a diffusion gradient.
adenosine triphosphate(ATP). In this form The solid particles which adsorb water or
of transport, molecules move against either any other liquid are called Imbibants. The
an electrical or concentration gradient liquid which is imbibed is known as
(collectively termed an electrochemical Imbibate. Examples are absorption of water
gradient). by seeds and dry wood.
2. (a) Phloem is the chief food conducting tissue 11. (a) Potassium is accumulated by passive
of vascular plan ts responsible for transport. Some solutes are pumped across
translocation of organic solutes. membranes using active transport. The role
3. (a) In rainy season, door gets swelled due to of proton pumps in the transport process
the phenomenon of imbibition. It is the of plant cells is a specific application of
process of absorption of water without
chemiosmosis, a transmembrane proton
forming a solution.
gradient that links energy-releasing
4. (d) Upward movement of water (sap) through
processes to energy-consuming ones like
xylem against the force of gravity is called
active transport.
ascent of sap. All three help in ascent of
12. (d) The casparian strips function in regulation
sap.
of the flow of water between outer tissues
5. (c) Cultivation of plants by placing the roots
and the vascular cylinder.
in the nutrient solution without any soil is
13. (c) DPD = OP – TP.
called hydroponics. It is also known as
soilless culture/ water culture/ solution DPD (A) = 4 bar.
culture. It is used to determine which DPD (B) = 0 bar.
elements are essential for plant growth and DPD (C) = 10 – 5 = 5 bars.
what symptoms are produced by the DPD (D) = 7 – 3 = 4 bars.
absence or deficiency of essential elements. Water always flows from lower DPD
6. (a) During sodium-potassium pump, the (Diffusion Pressure Deficit) to higher DPD.
concentration of sodium ions will be about Since the DPD of cell B is the lowest (O) the
14 times more in extra cellular fluid (outside) water will flow from B to A and then to C.
and concentration of potassium ions will 14. (c) Active transport uses energy (ATP) to
be about 28-30 times more in axoplasm pump molecules against a concentration
(inside). Thus, 3Na+ and 2K+ are transported gradient. Cells undergoing active transport
during the process. bear abundant mitochondria to provide
7. (c) Excessive loss of water from the leaves can ATP, needed to power active transport. So,
be prevented by applying vaseline on leaf the production of ATP is blocked or
surface. It will close the stomata and check decreased and active transport is blocked
transpiration. or slow down.
8. (d) 15. (c) Statements (iv) and (v) are not correct.
EBD_7100
B-66 Topicwise AIIMS Solved Papers – BIOLOGY
(iv) Xylem is associated with the sap from cut stems and guttation of water
translocation of mainly water, mineral from leaves. The pressure is generated by
salts, some organic nitrogen and the concentration of solutes in the xylem
hormones from roots to the aerial parts of the root and stem which is then causes
of the plants. water to move into the xylem by osmosis.
19. (c) Sap is water with dissolved ingredients.
(v) Phloem translocates a variety of
The upward movement of water from roots
organic and inorganic solutes mainly
towards the tips of stem branches and their
from the leaves to other parts of the leaves is called ascent of sap. It occurs
plants. through the tracheary elements of xylem.
Type B : Assertion Reason Questions 20. (a) The parietal cytoplasm and perforated
sieve plates help in the transport of
16. (a) Waxy and cutin coating does reduce photoassimilates that are required for
transpiration. This adaptive feature is seen photosynthesis. The parietal cytoplasm is
in xerophytic and in plants of semi-arid
the streaming of cytoplasm. The streaming
region. This adaptive feature is seen to
reduce water loss by transpiration. cytoplasm moves throughout the cell, thus,
17. (a) The radial and the inner walls of the cells helping in the transport of photoassimilates
of the endodermis are greatly thickened. from one cell to another through the sieve
These are called as casparian strips and plates.
these are impervious to water. Apart from 21. (a) Light is an important factor in transpiration.
the casparian strips, and suberization also The stomata opens well on days when light
does not allow the water to reach the
endodermis. Hence, water moves through is brighter. It is also evident on cloudy days
the apoplast which are passage cells. that the stomata does not open well. Hence,
18. (d) Root pressure is a pressure produced in light induces stomatal opening and
the roots of plants, causing exudation of darkness closing.
12 Mineral Nutrition

TYPE A : MULTIPLE CHOICE QUESTIONS D. Chlorine IV. Required to


activate
1. Leghaemoglobin helps in [2007] respiratory
(a) nitrogen fixation enzyme.
(b) protecting nitrogenase from O2 E. Manganese V. Required for
(c) destroys bacteria synthesis
(d) transport of food in plants of mitotic spindle.
2. Which of the following is correct set of (a) A – 1; B – 2; C – 3; D – 4; E – 5
micronutrient for plants? [2007, 2011]
(b) A – 5; B – 4; C – 3; D – 2; E – 1
(a) Mg, Si, Fe, Cu, Ca
(b) Cu, Fe, Zn, B, Mn (c) A – 4; B – 1; C – 5; D – 3; E – 2
(c) Mg, Fe, Zn, B, Mn (d) A – 5; B – 3; C – 2; D – 1; E – 4
(d) Mo, Zn, Cl, Mg, Ca 7. Which of the following is an INCORRECT
3. In glycolysis, glucose molecule is converted into: match of essential element and function?
[2011] [2014]
(a) Manganese - structural component of
(a) PEP (b) RuBP
chlorophyll.
(c) Acetyl CoA (d) Pyruvic acid (b) Calcium - component of the middle lamella.
4. Which one of the following elements is not an (c) Zinc - enzyme activator.
essential micronutrient for plant growth? [2012] (d) Iron - component of ferredoxin.
(a) Ca (b) Mn 8. Which of the following is the mismatched pair?
(c) Zn (d) Cu Mineral Form that is
5. Which element plays an important role in elements absorbed
nitrogen fixation? [2012] by plant
(a) Mn (b) Mo (a) Nitrogen NO3–
(c) Zn (d) Cu (b) Phosphorus H2PO4–
6. Match column-I and Column-II and choose the (c) Sulphur H2SO4
correct option given below the columns.[2013] (d) Iron Fe3+
Column-I Column-II [2016]
(Element) (Function)
TYPE B : ASSERTION REASON QUESTIONS
A. Calcium I. Required for
ionic- balance. Directions for (Qs. 9-10) : These questions consist
B. Boron II. Essential for of two statements, each printed as Assertion and
constitution Reason. While answering these questions, you are
of nucleic acid required to choose any one of the following five
responses.
C. Phosphorus III. Required for
absorption of (a) If both Assertion and Reason are correct and
calcium. the Reason is a correct explanation of the
Assertion.
EBD_7100
B-68 Topicwise AIIMS Solved Papers – BIOLOGY
(b) If both Assertion and Reason are correct but Directions for (Q.11) : Each of these questions
Reason is not a correct explanation of the contains an Assertion followed by Reason. Read them
Assertion. carefully and answer the question on the basis of
(c) If the Assertion is correct but Reason is following options. You have to select the one that
incorrect. best describes the two statements.
(d) If both the Assertion and Reason are incorrect. (a) If both Assertion and Reason are correct and
(e) If the Assertion is incorrect but the Reason is Reason is the correct explanation of Assertion.
correct. (b) If both Assertion and Reason are correct, but
9. Assertion : Plants lack excretory organs. Reason is not the correct explanation of
Reason : Plant usually absorb essential nutrients Assertion.
and lead a passive life. [1997] (c) If Assertion is correct but Reason is incorrect.
(d) If both the Assertion and Reason are incorrect.
10. Assertion : Plants absorb sulphur in the form of
11. Assertion : Deficiency of sulphur causes
sulphate ions.
chlorosis in plants.
Reason : Sulphur bacteria are required for the Reason : Sulphur is a constituent of chlorophyll,
formation of sulphate. [2007] protein and nucleic acids. [2004, 2014]
Mineral Nutrition B-69

Type A : Multiple Choice Questions Molybdenum availability varies with soil


type, being highest in organic soil, less in
1. (b) The root nodules of leguminous plants
clay, least in sandy soil.
contain a symbiotic nitrogen fixing bacteria
Rhizobium. Root nodules are small irregular 6. (d)
outgrowth of the roots which are pinkish 7. (a) Magnesium is a constituent of the ring
internally due to presence of a pigment structure of chlorophyll. Function of
called leghaemoglobin. It is related to blood manganese is to activate many enzymes
pigment haemoglobin. The cells of root involved in photosynthesis, respiration and
nodules are tetraploid and contain nitrogen metabolism. The best defined function
polyhedral bacteria called bacteroids. of manganese is in the splitting of water to
Leghaemoglobin is an oxygen scavenger liberate oxygen during photosynthesis.
and protects the nitrogen fixing enzyme 8. (c) Plants obtain sulphur in the form of
nitrogenase of bacteroids. sulphate (SO42–).
2. (b) The essential elements are divided into
Type B : Assertion Reason Questions
macroelements and microelements based
on the quantity in which they are required 9. (b) Plants do lack excretory organs, but it is
by the plants. Mn, Cu, Mo, Zn, B, and Cl not due to absorption pattern or passive
are the micronutrients needed in very small life. The carbon dioxide during respiration
quantities by the plants. C, H, O, N, P, S, K, passes out through the stomata. The other
Ca, Mg, Fe are the macronutrients required waste materials comes out in the form of
alkaloids, gums and resins.
in more quantity.
10. (a) Sulphur is a constituent of amino acids
3. (d) In this process, one molecule of glucose (cystein and cystine and methionine).
undergoes partial oxidation to form 2 Sulphur is present in the soil in the form of
molecules of pyruvic acid. oxides. Sulphur bacteria convert them into
sulphate ions.The plants absorb sulphur
4. (a) Micronutrient elements are those element
in the form of sulphate ions.
which is required in less quantity. These
11. (c) Due to deficiency of sulphur plant shows
are Cu, Zn, Mn, B, Cl, Mo and Ni. Some chlorosis (i.e., yellowing due to degradation
physiologist consider Fe as micronutrient. of chlorophyll) followed by anthocyanin
5. (b) Molybdenum (Mo) is required for development. The younger leaves show
symbiotic nitrogen fixation by legumes. chlorosis before older ones. Sulphur is not
Plants requires molybdenum 0.1 to 2.5 ppm the constituent of chlorophyll. The main
in their tissue for normal growth. constituent of chlorophyll is magnesium.
EBD_7100
B-70 Topicwise AIIMS Solved Papers – BIOLOGY

13 Photosynthesis in Higher Plants

8. Select the incorrect statement [2011]


TYPE A : MULTIPLE CHOICE QUESTIONS
(a) C 4 path way for CO 2 fixation were
1. In C4 plants, the first carbon dioxide acceptor is discovered by Hatch and Slack
(a) pyruvate [1997]
(b) CO2 is essential for photosynthesis
(b) phosphoenol pyruvate
(c) Addition of sodium carbonate in water
(c) ribulose biphosphate
retards photosynthetic rate in vallisneria
(d) ribulose 5, phosphate
2. In photosynthesis, splitting of water and release (d) Phloem is the principal pathway for
of oxygen occurs during [1998] translocation of solutes
(a) photolysis (b) red drop 9. The family in which many plants are C4 type
(c) Pasteur effect (d) Calvin cycle [2012]
3. Blackman’s law of limiting factor is applied (a) Malvaceae (b) Solanaceae
to [2001] (c) Crucifereae (d) Graminae
(a) respiration (b) transpiration 10. In the electron transport chain during terminal
(c) photorespiration (d) photosynthesis oxidation, the cytochrome, which donates
4. Hill reaction occurs in [2003] electrons to O2 is [2012]
(a) high altitude plants (a) Cytochrome-b (b) Cyto-C
(b) total darkness (c) Cyto-a3 (d) Cyto-f
(c) absence of water 11. Which one does not differ between a C3 and a
(d) presence of ferricyanide C4 plant? [2013]
5. Which one of the following categories of I. Initial CO2 acceptor.
organ isms do not evolve oxygen during II. Extent of photorespiration.
photosynthesis? [2004] III. Enzyme catalyzing reaction that fixes CO2.
(a) Red algae IV. Presence of Calvin cycle.
(b) Photosynthetic bacteria V. Leaf anatomy.
(c) C4 plants with Kranz anatomy (a) I and V (b) IV
(d) Blue green algae (c) II and III (d) II
12. The total requirement of ATP & NADPH for each
6. What is PAR range? [2007]
molecule of CO 2 fixed & reduced in
(a) 200 nm - 800 nm (b) 400 nm - 700 nm photosynthesis in the Calvin cycle is [2014]
(c) 350 nm - 550 nm (d) 600 nm - 100 nm (a) 2 ATP & 2 NADPH
7. Through the use of oxygen-18 (heavy oxygen), (b) 2 ATP & 3 NADPH
scientists have found that the oxygen released (c) 3 ATP & 2 NADPH
(d) 4 ATP & 3 NADPH
during photosynthesis comes from molecules of
13. Consider the following statements regarding
[2009] photosynthesis.
(a) carbon dioxide (b) water [2015]
(c) glucose (d) chlorophyll (A) ATP formation during photosynthesis is
termed as photophosphorylation.
Photosynthesis in Higher Plants B-71

(B) Kranz anatomy pertains to leaf. Reason : Oxygen started accumulating in the
(C) Reduction of NADP+ to NADPH occurs atmosphere after the non-cyclic pathway of
during Calvin cycle. photosynthesis evolved. [2004]
(D) In a chlorophyll molecule, magnesium is 19. Assertion : C4 photosynthetic pathway is more
present in phytol tail. Of the above efficient than the C3 pathway.
statements [2015] Reason : Photorespiration is suppressed in C4
(a) A and B are correct plants. [2005]
(b) C and D are correct 20. Assertion : The atmospheric concentration of
(c) A and C are correct CO2 at which photosynthesis just compensates
(d) A and D are correct for respiration is referred to as CO 2
compensation point.
TYPE B : ASSERTION REASON QUESTIONS Reason : The CO2 compensation point is
Directions for (Qs. 14-23) : These questions consist reached when the amount of CO2 uptake is less
of two statements, each printed as Assertion and than that generated through respiration because
Reason. While answering these questions, you are the level of CO2 in the atmosphere is more than
that required for achieving CO2 compensation
required to choose any one of the following five
point. [2005]
responses.
21. Assertion : Under conditions of high light
(a) If both Assertion and Reason are correct and intensity and limited CO 2 supply,
the Reason is a correct explanation of the photorespiration has a useful role in protecting
Assertion. the plants from photo-oxidative damage.
(b) If both Assertion and Reason are correct but Reason : If enough CO2 is not available to utilize
Reason is not a correct explanation of the light energy for carboxylation to proceed, the
Assertion. excess energy may not cause damage to plants.
(c) If the Assertion is correct but Reason is [2006]
incorrect. 22. Assertion : Photosynthetically C4 plants are less
(d) If both the Assertion and Reason are incorrect. efficient than C3 plants. [2006]
Reason : The operation of C4 pathway requires
(e) If the Assertion is incorrect but the Reason is
the involvement of only bundle-sheath cells.
correct.
23. Assertion : Dark reaction is purely enzymatic
14. Assertion: C 4 pathway of CO 2 fixation is reaction. [2007]
found in some tropical plants. Reason : It occurs only in absence of light.
Reason: In this pathway, CO 2 is fixed by 3C Directions for (Qs.24-30) : Each of these questions
contains an Assertion followed by Reason. Read them
compound. [1998]
carefully and answer the question on the basis of
15. Assertion : Mitochondria helps in
following options. You have to select the one that
photosynthesis
best describes the two statements.
Reason : Mitochondria have enzymes for dark
reaction. [1999] (a) If both Assertion and Reason are correct and
16. Assertion: Bacterial photosynthesis occurs by Reason is the correct explanation of Assertion.
utilizing wavelength longer than 700 nm. (b) If both Assertion and Reason are correct, but
Reason: Here reaction centre is B-890. [2002] Reason is not the correct explanation of
17. Assertion : Rhoeo leaves contain anthocyanin Assertion.
pigments in epidermal cells. (c) If Assertion is correct but Reason is incorrect.
Reason : Anthocyanins are accessory (d) If both the Assertion and Reason are incorrect.
photosynthetic pigments. [2003] 24. Assertion : Dark reaction occurs only at night
18. Assertion : Cyclic pathway of photosynthesis in the stroma of chloroplast.
first appeared in some eubacterial species. Reason : CO2 fixation occurs only during C3
cycle. [2009]
EBD_7100
B-72 Topicwise AIIMS Solved Papers – BIOLOGY
25. Assertion : Amaranthus and sugarcane are Reason : Oxygen started accumulating in the
called as Hatch & Slack plants. atmosphere after the non-cyclic pathway of
Reason : One glucose is formed by fixation of 6 photosynthesis evolved. [2012]
CO2 in the plants. [2010] 29. Assertion : Each molecule of ribulose-1,
26. Assertion : D.C.M.U. is a photosynthetic 5-bisphosphate fixes one molecule of CO2.
inhibitor. [2010] Reason : Three molecules of NADPH and two
Reason : D.C.M.U. inhibits a photolysis of water. ATP are required for fixation of one molecule of
27. Assertion: The stromal thylakoids are rich in CO2. [2013]
both PS I and PS II. 30. Assertion: Six molecules of CO2 and twelve
Reason: The stroma membranes are rich in ATP molecules of NADPH+ + H+ and 18 ATP are
synthetase. [2011] used to form one hexose molecule.
28. Assertion : Cyclic pathway of photosynthesis Reason: Light reaction results in formation of
first appeared in some eubacterial species. ATP and NADPH2. [2002, 2015]
Photosynthesis in Higher Plants B-73

Type A : Multiple Choice Questions 9. (d) C4 cycle occurs in 1500 species of 19


angiospermic families but most of the plants
1. (b) In C4 plants, the first CO2 acceptor is 3C are monocots which belong to graminae
compound, phosphoenol pyruvate. and cyperaceae family.
2. (a) In photosynthesis, release of oxygen 10. (c) In ETS or respiratory chain, there are five
occurs during photolysis of water in light cytochromes cyto-b, cyto-c1, cyto-c, cyto-
reaction. a, cyto-a 3 . Cytochrome a 3 is last
3. (d) Blackman’s law is applicable to cytochrome of ETS which denotes
photosynthesis. It states that when a electrons to O2 due to this metabolic water
process is conditioned as to its rapidity by
is formed.
number of factors, its rate is limited to the
pace of slowest factor. Taking temperature 11. (b)
into consideration, rate of photosynthesis 12. (c) For every CO2 molecule entering the Calvin
is maximum at optimum temperature. cycle, 3 molecules of ATP and 2 molecules
4. (a) Hill reaction or light reaction depends upon of NADPH are required.
light. It involves photolysis of water and 13. (a) During Calvin cycle, NADPH is oxidised
production of assimilatory power in the to NADP. In a chlorophyll molecule,
form of NADPH and ATP. magnesium is present in the porphyrin ring.
5. (b) Photosynthetic bacteria do not use H2O Type B : Assertion Reason Questions
as electron donor but some other 14. (b) C4 pathway is an adaptation of tropical
compound like Fe2+, H2S. plants to reduce/avoid the photorespiratory
In photosynthesis, splitting of water and loss. In C4 pathway, first acceptor of CO2 is
liberation of O2 by chlorophyll in presence a 3 carbon compound - phosphoenol
of light and hydrogen acceptor is called pyruvate.
photolysis of water. 15. (d) Mitochondria helps in cellular respiration
1 by transferring energy from organic
H 2O 2H 2e O2
2 compounds to ATP. Chloroplast helps in
[Photolysis of water] photosynthesis. Dark reaction takes part
6. (b) PAR (Photosynthetically active radiations) in the stroma of the chloroplast.
ranges between the wavelength 400- 16. (b) In bacteria, photosynthesis utilizes light
700nm. wavelength more than 700 nm and their
7. (b) Oxygen which is liberated during reaction centre is B-890.
photosynthesis comes form water.In 17. (c) Anthocyanin pigments only give
photosynthesis, the light energy is colouration since the epidermal cells mainly
captured by chlorophyll in the chloroplasts have potential colouring pigments. It is
in plant leaves. This energy is used to split responsible of blue, red, pink and purple
water apart in a process called photolysis. colours, observed in different parts of plants
4H 2 O 4H+ + 4OH- such as petals, stamens and fruits etc.
+ -
Anthocyanin are also important for
4OH- ¾¾¾¾ Mn ,Cl
® 2H 2 O + O2 - +4e- attracting insects for pollination and seed
8. (c) C4 pathway for CO2 fixation were discovered dispersal. Hence, Anthocyanin pigments
by Hatch and Slack. This pathway has more are not accessory photosynthetic
effective method of CO2 fixation and is seen pigments.
in plants like sugarcane. CO2 is essential for 18. (b) Photosynthetic bacteria have a substance
photosynthesis. It is the source of carbon. called bacteriochlorophyll. The bacterio-
The phloem or leptome is the pathway for chlorophyll pigment absorbs light in the
movement of solutes. extreme UV and infra-red parts of the
EBD_7100
B-74 Topicwise AIIMS Solved Papers – BIOLOGY
spectrum which is outside the range used CO2 is added by the enzyme, RuBisco to a
by normal chlorophyll, seen in plants. 5 carbon compound RuBP that is converted
19. (a) C4 pathway/Hatch and Slack pathway to 2 molecules of 3-carbon PGA. In C4 cycle,
ensures the Calvin cycle to be operated the first product of CO2 fixation (takes
only in bundle sheath cell. It is an place in mesophyll) is a 4-carbon
adaptation to photorespiratory loss. compound, oxaloacetic acid. It is seen in
Therefore, C 4 plants are adapted to some tropical plants.
photorespiratory loss. 25. (b) Amaranthus sp and sugarcane are known
20. (c) Compensation point is that value or point as Hatch and Slack plants. In Hatch and
in the light intensity and atmospheric CO2 Slack pathway, one glucose molecule is
concentration when the rate of formed by fixation of 6CO2 in the plants.
photosynthesis is just equivalent to the 26. (a) DCMU (Dichlorophenyl dimethyl urea) is
rate of respiration in the photosynthetic a herbicide that can prevent non cyclic
organ. So that there is not net gaseous photophosphorylation and oxygen
exchange. production. It inhibits photolysis of water.
21. (c) Photorespiration is the uptake of O2 and 27. (d) The grana stacks of membranes are
release of CO2 in light and results from the enriched in PS II and LHC (Light harvesting
biosynthesis of glycolate in chloroplasts centre), while there is little ATP synthetase.
and subsequent metabolism of glycolate On the other hand, a fraction of stroma
acid in the same leaf cell. During thylakoids is rich in PS I and ATPase and
photorespiration loss of carbon takes place poor in PS II and LHC.
in the form of CO2. 28. (b) Cyclic pathway of photosynthesis is
22. (d) C4 plants are more efficient in picking up appeared first in some eubacterial species.
CO 2 even when it is found in low It is supposed to be the first evidence of
concentration because of its high affinity production of ATP in the presence of light.
for PEP. They show kranz anatomy i.e. During non-cyclic photophosphorylation
vascular bundle is surrounded by bundle photolysis of water takes place. Under the
sheath and mesopyll cells. influence of light energy and the catalytic
23. (e) During photosynthesis, assimilatory power action of chlorophyll, water is split up into
ATP and NADPH2 are produced which oxygen an d hydrogen. Non-cyclic
require light. This reaction is called light photophosphorylation is the only natural
reactions or Hill's reaction. Assimilatory process which adds molecular oxygen to
powers are required for the reduction of the atmosphere.
CO 2 . This reaction is enzymatic and 29. (c) Each molecule of ribulose-1, 5-biphosphate
independent of light. It is called dark fixes one molecule of carbon dioxide with
reaction which takes place in stroma of the addition of water, thereby resulting in
chloroplast. So, dark reaction is the formation of two molecules of
independent of presence or absence of 3-phosphoglyceric acid (3-PGA). The
light. fixation and reduction of one molecule of
24. (d) Dark reaction is also known as light- CO2 requires three molecules of ATP and
independent phase. Unlike, light reaction, two of NADPH, coming from the
it does not require light as an essential photochemical reactions.
factor. Thus, can take place both in the 30. (b) Light reaction or Hill reaction results in the
presence or absence of light. The term dark formation of ATP and NADPH2, 6CO2 ,
reaction does not mean that it takes place 6H2O, ATP and NADPH2 are utilised to
only in dark period or at night. CO2 fixation produce one molecule of glucose.
occurs in both C3 and C4 cycle. In C3 cycle,
14 Respiration in Plants

TYPE A : MULTIPLE CHOICE QUESTIONS (c) Isocitrate dehydrogenase


(d) Malate dehydrogenase
1. Glycolysis occurs in [2000] 8. Which of the metabolites is common to
(a) mitochondria (b) chloroplast respiration mediated breakdown of fats,
(c) cytoplasm (d) peroxisome
carbohydrates and proteins? [2014]
2. Anaerobic respiration, after glycolysis is also
(a) Fructose 1, 6 - bisphosphate
called as [2002]
(b) Pyruvic acid
(a) fermentation (b) fragmentation (c) Acetyl CoA
(c) restoration (d) multiplication (d) Glucose - 6 - phosphate
3. In glycolysis, glucose molecule is converted into 9. Which of the following representation correctly
[2002] explain the function of mitochondrion? [2015]
(a) PEP (b) RuBP
(c) acetyl CoA (d) pyruvic acid
O2 H2O
4. Photorespiration in C3 plants starts from [2003]
(a) phosphoglycerate ADP M ATP
(a)
(b) phosphoglycolate
(c) glycerate Phosphate CO2
(d) glycine
5. Which of the following is the connecting link H2O
O2
between glycolysis and Krebs cycle? [2007]
(a) Acetyl CoA ADP M AMP
(b)
(b) Oxalosuccinic acid Phosphate CO2
(c) Pyruvic acid
(d) Citric acid
6. Pyruvate kinase enzyme catalyses [2010] CO2 H2O
(a) first irreversible step of glycolysis ADP ATP
(c) M
(b) second irreversible step of glycolysis
Phosphate O2
(c) third irreversible step of glycolysis
(d) fourth irreversible step of glycolysis
7. An enzymes of TCA cycle are located in the CO2 H2O
mitochondrial matrix except one which is located
in inner mitochondrial membrane in eukaryotes ADP M AMP
and in cytosol in prokaryotes. This enzyme is (d)
[2012] Phosphate O2
(a) Succinate dehydrogenase
(b) Lactate dehydrogenase
EBD_7100
B-76 Topicwise AIIMS Solved Papers – BIOLOGY
10. Refer the figure and answer the question. (iii) Glucose is phosphorylated to give rise to
glucose - 6 - phosphate by the activity of
Glucose the enzyme phosphofructokinase.
ATP
(iv) The scheme of glycolysis was given by
ADP Gustav Embden, Otto Morrison, and J.
Glucose-6-Phosphate Parnas and is often referred to as the EMP
pathway.
P Q R + (v) ATP is utilized at two steps: first in the
NAD
+ conversion of glucose into glucose 6-
NADH + H
phosphate & second in the conversion of
S
fructose - 6- phosphate to fructose 1,
2 × 3 PGA 6-disphosphate. [2016]
(a) (i), (iv) and (v) (b) (iii) and (v)
2 × 2 PGA (c) (iv) and (v) (d) (ii) and (iv)
13. Refer the given equation and answer the
H2O
question.
PEPA
ADP 2(C51H 98 O6 ) 145O 2 102CO 2
ATP 98H 2O Energy
2 × C3H4O3 The R.Q of above reaction is [2017]
(a) 1 (b) 0.7
Choose the correct names of P, Q, R, and S. (c) 1.45 (d) 1.62
[2015] 14. In alcoholic fermentation, NAD+ is produced
during the [2017]
P Q R S (a) reduction of acetyldehyde to ethanol.
(a) 1,3 di PGA 3 PGAld Fr.1,6 di P Fr. 6 P (b) oxidation of glucose.
(b) 3 PGA1d 1,3 di PGA Fr. 1,6 di P Fr.6 P (c) oxidation of pyruvate to acetyl coA.
(c) Fr. 1,6 di P Fr. 6 P 3 PGAld 1,3 di PGA (d) hydrolysis of ATP to ADP.
(d) Fr.6 P Fr. 1,6 di P 3 PGAld 1,3 di PGA
TYPE B : ASSERTION REASON QUESTIONS
11. By which of the following complex, proton is Directions for (Q. 15) : These questions consist of
pumped to reach ATP synthase to participate in two statements, each printed as Assertion and Reason.
ATP synthesis? While answering these questions, you are required to
(a) Cytochrome b6 f choose any one of the following five responses.
(b) Cytochrome c oxidase (a) If both Assertion and Reason are correct and
(c) Cytochrome a - a3 the Reason is a correct explanation of the
(d) Cytochrome bc Assertion.
[2016] (b) If both Assertion and Reason are correct but
12. Which of the following statements (i to v) Reason is not a correct explanation of the
regarding glycolysis are correct. Assertion.
(i) It is ten enzymatic reactions that convert a (c) If the Assertion is correct but Reason is
six-carbon molecule to a three carbon incorrect.
pyruvate and result in a net gain of 2 ATP (d) If both the Assertion and Reason are incorrect.
molecules.
(e) If the Assertion is incorrect but the Reason is
(ii) Glucose undergoes partial oxidation to form
correct.
one molecule of pyruvic acid.
Respiration in Plants B-77

15. Assertion : Stomata are absent in submerged 16. Assertion : Glycolysis is the first step of
hydrophytes. respiration in which glucose completely breaks
Reason : Respiration occurs by means of air into CO2 and H2O.
chambers in submerged plants. [1997] Reason : In this process, there is net gain of
Directions for (Qs.16-18) : Each of these questions twenty four molecules of ATP. [2009]
contains an Assertion followed by Reason. Read them 17. Assertion : The inner membrane of
carefully and answer the question on the basis of mitochondria contains systems involving
following options. You have to select the one that electron transport.
best describes the two statements. Reason : The mitochondrial matrix contains
(a) If both Assertion and Reason are correct and enzymes of Kreb's cycle. [2013]
Reason is the correct explanation of Assertion. 18. Assertion : Glycolysis occurs in cytoplasm.
(b) If both Assertion and Reason are correct, but Reason : Enzymes for glycolysis are found in
Reason is not the correct explanation of cytoplasm. It is common in aerobic/anaerobic
Assertion. respiration. [2002, 2014]
(c) If Assertion is correct but Reason is incorrect.
(d) If both the Assertion and Reason are incorrect.
EBD_7100
B-78 Topicwise AIIMS Solved Papers – BIOLOGY

Type A : Multiple Choice Questions 9. (a) The main function of mitochondrion is the
1. (c) All the enzymes required for the different generation of ATP from ADP and inorganic
steps of glycolysis (Ist step in cellular phosphate during cellular respiration.
respiration) are present in the cytoplasm. 10. (d)
2. (a) Glycolysis is a common step for both 11. (a) With the help of cytochrome b6 f, proton is
aerobic and anaerobic respiration. Next step pumped to reach ATP synthetase to
after glycolysis is fermentation in anaerobic participate in ATP synthesis.
respiration which leads to the formation of 12. (a) Glucose undergoes partial oxidation to form
ethanol or lactic acid. two molecules of pyruvic acid, Glucose is
3. (d) In glycolysis, one molecule of glucose phosphorylated to give rise to glucose-6-
undergoes partial oxidation to form 2 phosphate by the activity of the enzyme
hexokinase. The scheme of glycolysis was
molecules of pyruvic acid.
given by Gustav Embden, Otto Meyerhof,
4. (a) In C3 plants, photorespiration starts from and J. Parnas, and is often referred to as
3-phosphoglycerate. Photorespiration is the EMP pathway.
also called glycolate pathway. 13. (b) The ratio of the volume of CO2 liberated to
5. (a) Acetyl CoA is the connecting link between the volume of oxygen absorbed per
glycolysis and Krebs cycle. Acetyl CoA molecule during respiration is called
combines with oxalo acetic acid to form Respiratory Quotient (RQ). The value of
citric acid during Krebs cycle. RQ indicates the types of respiratory
6. (c) Pyruvate kinase enzyme catalyses third substrate.
irreversible step of glycolysis. Volume of CO2 evolved
2 Phosphoenol pyruvic acid + 2ADP RQ =
Volume of O 2 consumed
Pyruvate kinase
Pyruvic acid + ATP.. 102
Mg 2 , K RQ 0.7
145
7. (a) All the oxidative enzymes of TCA are located
in matrix except succinic dehydrogenase. 14. (a) Alcoholic fermentation is a process in
This enzyme catalysed the conversion of which molecules such as glucose etc. are
succinic acid into fumaric acid. The enzyme converted into cellular energy and thereby
is an integral protein complex that is tightly produce ethanol and carbon dioxide as
bound to the inner mitochondrial metabolic waste products. During alcoholic
membrane. Infact this enzyme is the
fermentation, NAD+ is produced when
preferred marker enzyme for inner
acetaldehyde is reduced to ethanol.
membranes when doing mitochondrial
fractionations. Type B : Assertion Reason Questions
8. (c) Acetyl CoA is common to respiration
mediated breakdown of fats, carbohydrates 15. (b) Stomata are absent since gaseous exchange
and proteins. Fats are broken down to fatty takes place through diffusion in submerged
acid and glycerol and again fatty acid plants.
degraded to acetyl CoA . Protein first 16. (d) Glycolysis is the process of breakdown of
degraded by proteases to indiviual amino glucose or similar hexose sugar into two
acids which deaminated to pyruric acid and molecules of pyruvic acid through a series
further decarboxylised to acetyl CoA. of enzyme mediated reactions, releasing
Respiration in Plants B-79

energy (ATP) and reducing power 17. (b) Electron transport system is a series of co-
(NADH2). It is the first step of respiration, enzymes and cytochromes that takes part
which occurs inside the cytoplasm and is in the passage of electrons from a chemical
independent of O2 . In glycolysis, two to its ultimate receptor. The mitochondrial
molecules of ATP are consumed during matrix contains all the soluble enzymes of
double phosphorylation of glucose to form the citric acid or Kreb's cycle and those
fructose 1, 6 diphosphate. Four molecules involved in the oxidation of fatty acids.
of ATP are produced in the conversion of 1, 18. (a) Glycolysis occurs in cytoplasm as all
3-diphosphoglycerate to 3-phospho- necessary enzymes are found in it. This
glycerate and phosphenol pyruvate to process is common in aerobic/anaerobic
pyruvate whereas, two molecules of respiration. In this process, one glucose
NADH2 are formed during oxidation of molecule is converted into 2 molecules of
glyceraldehyde 3-phosphate to pyruvic acid.
1,3-diphosphoglycerate. Since, each
NADH is equivalent to 3 ATP, so net gain
in glycolysis is 8 ATP.
EBD_7100
B-80 Topicwise AIIMS Solved Papers – BIOLOGY

15 Plant Growth and Development

TYPE A : MULTIPLE CHOICE QUESTIONS 12. Induction of cell division and delay in
senescence is done by [2011]
1. In plant, auxin synthesis occurs in [1997]
(a) Cytokinins (b) Auxins
(a) cortex
(c) GA (d) CoA
(b) xylem
13. Gibberellins can promote seed germination
(c) phloem
because of their influence on [2005, 2012]
(d) root and shoot tips
(a) Rate of cell division
2. Which of the following hormone induces cell
division ? [1997] (b) Production of hydrolyzing enzymes
(a) Auxin (b) Gibberellin (c) Synthesis of abscisic acid
(c) Cytokinin (d) Trypsin (d) Absorption of water through hard seed
3. Which is a weedicide ? [1998] coat.
(a) IAA (b) 2,4-D 14. Study the following statements. [2013]
(c) IBA (d) NAA I. “X” hormone promotes root growth and
4. Which of the following is gaseous hormone ? root hair formation thus helping the plants
to increase their absorption surface.
(a) Auxin (b) Ethylene [1998]
II. “Y” hormone induces flowering in mango
(c) Cytokinin (d) GA
and also promotes rapid internode/petiole
5. A plant cell has potential to develop into a full
elongation in deep plants and hence
plant. This is called [1998]
helping leaves or upper parts of shoot
(a) totipotency (b) gene cloning
above water.
(c) tissue culture (d) regeneration III. “Z” hormone inhibits the seed germination,
6. Which of the following induces dormancy ? increase the tolerance of plant to various
(a) Auxin (b) Cytokinin [1999] stresses, play import in seed development,
(c) Both (a) and (b) (d) Abscisic acid maturation and dormancy.
7. The plant hormone controlling fruit ripening is Identify the correct names of hormones marked
(a) IAA (b) GA [1999] as ‘X’, ‘Y’ & ‘Z’.
(c) ABA (d) Ethylene (a) Y = ABA; X = Auxin; Z = GA
8. Induction of cell division and delay in (b) Z = GA; X = Auxin; Y = C2H4
senescence is done by [2001] (c) Y = Auxin; X = C2H4; Z = GA
(a) cytokinins (b) auxins (d) Y = C2H4; X = C2H4; Z = ABA
(c) GA (d) CoA 15. Identify the correct and incorrect statements from
9. Curling of tendrils is due to [2001] the following.
(a) thigmotropism (b) phototropism (i) 17,500 new cells are produced per hour by
(c) chemotropism (d) nyctinasty a single maize root apical meristem.
10. Bud dormancy can be induced by [2002] (ii) With the help of length, growth of pollen tube
(a) IAA (b) GA is measured.
(c) ABA (d) ethylene (iii) The growth of the leaf is measured in term
11. Avena curvature test is a bioassay for examining of volume.
the activity of [2006] (iv) Cells in a watermelon may increase in size
(a) auxins (b) gibberellins by upto 3,50,000 times. [2016]
(c) cytokinins (d) ethylene
Plant Growth and Development B-81

(a) (i), (ii), (iii) are correct and (iv) is incorrect. 16. Assertion : Dark period plays more important
(b) (i), (ii), (iv) are correct and (iii) is incorrect. part in flowering than light period.
(c) (ii), (iii) are correct and (i), (iv) are incorrect. Reason : Flowering occurs in short-day plant if
(d) (i), (iv) are correct and (ii), (iii) are incorrect. the dark period is interrupted by light break.
TYPE B : ASSERTION REASON QUESTIONS [2013]
17. Assertion : Photomodulation of flowering is a
Directions for (Qs. 16-19) : These questions consist phytochrome regulated process.
of two statements, each printed as Assertion and Reason : Active form of phytochrome (PFR)
Reason. While answering these questions, you are directly induces floral induction in shoot buds.
required to choose any one of the following five [2015]
responses. 18. Assertion (A) : Vernalization is acceleration of
(a) If both Assertion and Reason are correct and subsequent flowering by low temperature
the Reason is a correct explanation of the
treatment.
Assertion.
Reason (R) : Site of vernalization is apical
(b) If both Assertion and Reason are correct but
meristem.
Reason is not a correct explanation of the
Assertion. [2015]
19. Assertion : Auxins help to prevent fruit and leaf
(c) If the Assertion is correct but Reason is
drop at early stages.
incorrect.
Reason : Auxins promote the abscission of older
(d) If both the Assertion and Reason are incorrect. mature leaves and fruits.
[2017]
EBD_7100
B-82 Topicwise AIIMS Solved Papers – BIOLOGY

Type A : Multiple Choice Questions 10. (c) Abscisic acid (ABA) is a plant hormone
1. (d) Auxin is a plant hormone which promote which promotes dormancy in seeds and
and regulate its growth and development. buds.
Auxin are produced in the meristem of 11. (a) Went (1928) performed Avena curvature
shoot tips and move down the plant test for auxins.
causing various effects. 12. (a) Cytokinins promote cell division and inhibit
2. (c) Cytokinins are substances which act the degradative reactions in detached
primarily on cell division and have little or leaves and slow down senescence in intact
no effect on extension/growth. It delays leaves. The effect of cytokinin in retarding
the senescence of leaves. ageing is called the Richmond Lang Effect.
3. (b) 2, 4-D (dichlorophenoxyacetic acid) is a
common systemic pesticide/herbicide. It is 13. (b) Gibberellins can promote seed germination
a synthetic auxin (plant hormone) and is in cereals due to production of hydrolyzing
used for killing broad leaved weeds enzymes like amylase and proteases.
(generally dicot). 14. (d) Gibberellins stimulate the production of
4. (b) Ethylene is the only gaseous natural plant some mRNAs and then hydrolytic enzymes
growth regulator produced by all plant like amylase, lipase ribonuclease and
organs but its maximum production occurs proteases. The enzymes solubilise the
in ripening fruits and during senescence. reserve food of the seeds and the same is
5. (a) Any cell which has an ability to develop transferred to embryo axis for its growth.
into a complete organism is totipotent. 15. (b) The growth of the leaf is measured in term
6. (d) Abscisic acid (ABA) induces dormancy.
of surface area.
Robinson and Warming (1964), isolated a
substance responsible for dormancy in Type B : Assertion Reason Questions
Acer pseudoplatanus, and named it as
dormin. This hormone was similar to 16. (c) It has been demonstrated that flowering in
absicissin. It is a naturally occurring plant is more of a response to the dark
hormone that is present in all vascular period than to the light period. In short day
plants and some mosses but is not present plants, the plants can flower in complete
in bacteria, algae, fungi and liverworts. This
hormone is responsible for dormancy. darkness if supplied with exogenous
7. (d) Fruit ripening is controlled by hormone, nutrients. Flowering is prevented in them
ethylene. Ethylene is a gaseous hormone. if dark period below the critical level is
It affects the growth, development, ripening interrupted by a flash of light. Interruption
and senescence (aging) of all plants. of light by dark inhibits flowering under
8. (a) Cytokinins promote cell division and inhibit normal photoperiods.
the degradative reactions in detached
leaves and slow down senescence in intact 17. (a) Active form of PFR is responsible for
leaves. The effect of cytokinin in retarding inducing flowering. Phytochrome, protein
ageing is called the Richmond Lang Effect. pigment, exists in two inter convertible
9. (a) The curling of tendrils is a growth forms.
movement induced due to contact or
touch. Such a movement is called Red
thigmotropic movement. When the tip of Pr Pfr
the tendril comes in contact with an uneven Far red
surface, it coils around the support. This is
due to the differential growth in the tendril. Dark
Plant Growth and Development B-83

18. (b) The physiological mechanism of flowering 19. (b) Auxin delays abscission of young leaves
in plants is controlled by two factors__ light and fruits. Its effect is through non-
period and low temperature. The cold formation of abscission zone below a leaf
treatment of plants to induce flowering is or fruit. Abscission zone cuts off nutrients
called vernalization. Term vernalization was and water supply. However, auxin promotes
first given by T.D.Lysenko(1928). As a the abscission of mature or older leaves
result of vernalization a flowering hormone and fruits.
called vernaline is formed. Site of
vernalization is apical meristem.
EBD_7100
B-84 Topicwise AIIMS Solved Papers – BIOLOGY

16 Digestion and Absorption

TYPE A : MULTIPLE CHOICE QUESTIONS 11. Wharton’s duct is the duct of [2002]
(a) Parotid gland
1. Tocopherol stands for [1997]
(a) Vitamin A (b) Vitamin E (b) Sublingual gland
(c) Vitamin C (d) Vitamin K (c) Submaxillary gland
2. Scurvy is caused due to deficiency of vitamin (d) Pancreatic gland
(a) A (b) B [1997] 12. Continued consumption of a diet rich in butter,
(c) E (d) C red meat and eggs for a long period may lead to
3. The contraction of gall bladder is due to [1998] (a) vitamin A toxicity [2003]
(a) gastrin (b) secretin (b) kidney stones
(c) cholecystokinin (d) enterokinase (c) hypercholesterolemia
4. The function of rennin is [1999]
(d) urine laden with ketone bodies
(a) vasodilation
13. Which one of the following pairs of the cells
(b) reduce blood pressure
(c) degradation of angiotensinogen with their secretion is correctly matched?
[2006]
(d) none of the above
5. Liver in our body stores [1999] (a) Oxyntic cells - A secretion with pH between
(a) Vitamin A (b) Vitamin D 2.0 and 3.0
(c) Vitamin B12 (d) All of these (b) Alpha cells of Islets of Langerhans -
6. Vitamin C is also called as [2000] Secretion that decreases blood sugar level.
(a) ascorbic acid (b) glutamic acid (c) Kupffer cells - A digestive enzyme that
(c) aspartic acid (d) enolic acid hydrolysis nucleic acids.
7. Brunner’s glands are present in [2001] (d) Sebaceous glands - A digestive enzyme
(a) duodenum (b) oesophagus that hydrolysis nucleic acids
(c) ileum (d) stomach 14. Which match is true? [2007]
8. Curdling of milk in small intestine takes place Vitamin deficiency Vitamin Source
due to [2002] disease
(a) rennin (b) trypsin (a) Severe bleeding Tocopherol Milk,
(c) chymotrypsin (d) ptyalin egg
9. Which of the following has minimum pH?[2002] (b) Anaemia Ascorbic Lemon,
(a) Bile (b) Saliva acid orange
(c) Night blindness Retinol Carrot,
(c) Gastric juice (d) Pancreatic juice
milk
10. Which of following teeth are lophodont? [2002]
(d) Sterility Calciferol Milk,
(a) Incisor and canine butter
(b) Premolar and molar 15. A child took sugar cane and sucked its juice.
(c) Canine and premolar Regarding this which of the following match is
(d) Premolar and incisor correct? [2007]
Digestion and Absorption B-85

Substrate Enzyme Site of Products (a) polyunsaturated fatty acids


of enzyme secretion formed (b) saturated fatty acids
(a) Proteins Pepsin Duodenum Polypeptides (c) essential vitamins
(b) Starch Amyase Salivary Glucose (d) more carbohydrates and proteins
glands
21. Endoscopy, a technique used to explore the
(c) Lipids Lipase Pancreas Fat
globules
stomach or other inner parts of the body, is based
(d) Sucrose Invertase Duodenum Glucose + on the phenomenon of [2010]
Fructose (a) total internal reflection
16. Which one of the following pairs of the kind of (b) interference
cells and their secretion are correctly matched? (c) diffraction
[2008] (d) polarization
(a) Oxyntic cells – A secretion with 22. Lathyrism is caused by excessive consumption
pH between 2.0 of [2010]
and 3.0 (a) khesari dal (b) mustard oil
(b) Alpha cells – Secretion of (c) polished rice (d) mushrooms
of (Nutrition) Langerhans that 23. The normal temperature of human body on the
islets decreases blood Kelvin scale is [2010]
sugar level (a) 280 (b) 290
(c) Kupffer cells – A digestive (c) 300 (d) 310
enzyme that 24. Parotid salivary glands are present [2012]
hydrolyses (a) Below the tongue
(b) Below the cheeks
nucleic acids
(c) In the angle between two jaws
(d) Sebaceous – A secretion that
(d) Below the eye orbits
glands evaporates for
25. If for some reason the parietal cells of the gut
cooling
epithelium become partially non-funcitonal, what
17. Fat present below the skin surface in our body, is likely to happen? [2015]
acts as a barrier against [2010]
(a) The pancreatic enzymes and specially the
(a) loss of heat from the body trypsin and lipase will not work efficiently
(b) loss of essential body fluids (b) The pH of stomach will fall abruptly
(c) loss of salts from the body (c) Steapsin will be more effective
(d) entry of harmful micro-organisms from the (d) Proteins will not be adequately hydrolysed
environment by pepsin into proteoses and peptones
18. The nutritional deficiency condition that needs 26. A healthy person eats the following diet - 5 gm
to be given top priority for remedial action in raw sugar, 4 gm albumin, 10 gm pure buffalo ghee
India today is [2010] adultrated with 2 gm vegetable ghee
(hydrogenated vegetable oil) and 5 gm lignin.
(a) scurvy (b) rickets How many calories he is likely to get?
(c) xerophthalmia (d) pellagra [2014, 2016]
19. What is the average fat content of buffalo milk? (a) 144 (b) 126
(a) 7.2% (b) 4.5% [2010] (c) 164 (d) 112
(c) 9.0% (d) 10.9% 27. Which of the following statement is true ? [2017]
(a) Pepsin cannot digest casein.
20. Consumption of fish is considered to be healthy
(b) Trypsin can digest collagen.
when compared to flesh of other animals because
(c) Pepsin cannot digest collagen.
when compared to flesh of other animals, fish
(d) Chymotrypsin can digest casein.
contains [2010]
EBD_7100
B-86 Topicwise AIIMS Solved Papers – BIOLOGY

TYPE B : ASSERTION REASON QUESTIONS 29. Assertion : In the condition of obstructive


jaundice, large amounts of unabsorbed fats are
Directions for (Q. 28) : These questions consist of eliminated out of the body.
two statements, each printed as Assertion and Reason. Reason : Entry of bile into the small intestine is
While answering these questions, you are required to prevented during obstructive jaundice. [2009]
choose any one of the following five responses.
30. Assertion : Pancreatic amylase digest starch to
(a) If both Assertion and Reason are correct and maltose. [2010]
the Reason is a correct explanation of the
Reason : Pancreatic amylase breaks the peptide
Assertion.
bond of protein.
(b) If both Assertion and Reason are correct but 31. Assertion : Trypsin helps in blood digestion of
Reason is not a correct explanation of the predator animals.
Assertion.
Reason : Trypsin hydrolyzes fibrinogen.
(c) If the Assertion is correct but Reason is [2011]
incorrect.
32. Assertion : Lipases of bile help in the
(d) If both the Assertion and Reason are incorrect. emulsification of fats.
(e) If the Assertion is incorrect but the Reason is Reason : Lipases can break large fat droplets
correct. into smaller ones. [2011]
28. Assertion : Scurvy is caused by deficiency of 33. Assertion : In the condition of obstructive
vitamin. jaundice, large amounts of unabsorbed fats are
Reason : Deficiency of ascorbic acid causes scurvy.
eliminated out of the body.
[2001]
Reason : Entry of bile into the small intestine is
Directions for (Qs.29-35) : Each of these questions
prevented during obstructive jaundice.
contains an Assertion followed by Reason. Read them
carefully and answer the question on the basis of [2015]
following options. You have to select the one that 34. Assertion : Starch is hydrolysed by ptyalin to
best describes the two statements. maltose.
(a) If both Assertion and Reason are correct and Reason : Sucrase hydrolyses sucrose to lactose.
Reason is the correct explanation of Assertion. [2016]
(b) If both Assertion and Reason are correct, but 35. Assertion : Water and electrolytes are almost
Reason is not the correct explanation of fully absorbed in the large intestine.
Assertion. Reason : In large intestine, haustral contractions
(c) If Assertion is correct but Reason is incorrect. (slow segmenting movements) roll the forming
(d) If both the Assertion and Reason are incorrect. faeces over and over, causing absorption of
water and electrolytes. [2017]
Digestion and Absorption B-87

Type A : Multiple Choice Questions from dudonum's glands, called the crypts
1. (b) Tocopherol or vitamin E is fat soluble of Lieberkuh n, it converts inactive
vitamin. It is antisterility factor and trypsinogen into active trypsin.
antioxidant for membrane lipids, skin, and 4. (d) Rennin (also called rennet or chymosin) is
hair etc. It reduces atherosclerosis and an coagulating enzyme produced from
inhibits oxidation of vitamin A and stomach of human body. It catalyzes the
unsaturated fatty acid. Retinol, ascorbic coagulation of milk by converting milk with
acid and phylloquinone stands for vitamin soluble protein caesin into insoluble semi
A, C and K respectively. fluid calcium paracaesinate. This is called
2. (d) Deficiency of vitamin C (which is necessary curdling of milk. Rennin produced in the
for collagen production and iron infants immediately after birth. As the child
absorption) causes scurvy. It is grows, rennin production goes down and
characterised by bleeding of gums, is replaced by pepsin digestive enzymes.
disturbance of protein metabolism and Renin is an enzyme which acts as hormone
increased susceptibility to infections. secreted by juxtaglomerular cells. It converts
Scurvy is most frequently seen in order, angiotensinogen into angiotensin.
malnourished adults. Scurvy commonly is 5. (d) Liver is the largest organ in the body. It
associated with sailors in the 16th to 18th helps in digestion and removes waste
centuries who navigated long voyages products and worn-out cells from the
without enough vitamin C frequently blood. Liver performs many functions, like
perished from the condition. it forms and secretes bile that contains bile
Deficiency of vitamin A, B and E results in acids to aid in the intestinal absorption
xeropthalmia, beri-beri and miscarriage (taking in ) of fats and the fat-soluble
respectively. vitamins as A, D, E, K and B12. Hence, it
stores vitamins as A, D, E, K and B12.
3. (c) The contraction of gall bladder occurs due
to hormone, cholecystokinin. 6. (a) Vitamin C (also called as ascorbic acid) is a
water soluble vitamin. It helps the body to
Cholecystokinin is a gastrointestinal
make collagen, an important protein used
hormone that is secreted by cells in the
to make skin, cartilage, tendons, ligaments
duodenum. Its release is stimulated by the
and blood vessels. Vitamin C is essential
presence of fatty acids and amino acids in
for healing wounds, and for repairing and
the small intestine. It stimulates the release
maintaining bones and teeth.
of bile into the intestine by contracting gall
bladder and the secretion of pancreatic 7. (a) Brunner's glands are compound tubular
enzymes. Gastrin and secretin are glands found only in the sub-mucosa of
polypeptide hormones. Gastrin, secreted by duodenum. They produce mucoid fluid
certain cells of the pyloric glands, which protects the duodenal mucosa from
stimulates secretion of gastric juice into the acidic chyme coming from the stomach.
bloodstream and churning movement of 8. (a) Trypsin changes chymotrypsinogen to
stomach. Secretin is secreted by the chymotrypsin and procarboxypeptidase to
mucosa of the duodenum and jejunum. It carboxypeptidase. Chymotrypsin changes
stimulates secretion of water an d caesin of milk into paracaesin. Ptyalin or
bicarbonates in bile and activates pancreas salivary amylase converts starch and
to secrete pancreatic juice. Enterokinase of glycogen into limit dextrins, maltose and
enteropeptidase is an enzyme, secreted isomaltose.
EBD_7100
B-88 Topicwise AIIMS Solved Papers – BIOLOGY
9. (c) pH of bile is 8; pH of saliva is 6.7, pH of 14. (c) Retinol is the chemical name of the vitamin
pancreatic juice is 8.8 and that of stomach A, which is mostly found in carrots, milk,
is 2. cheese, fish etc. Retinol is well adapted for
10. (b) Premolar and molar are lophodont teeths. light absorption in animals where it is
Lophodont teeth with the cusps elongated converted into the light-absorbing
to form narrow ridges. The molars in molecule called retinal. Deficiency of
elephants and horses have cusps fused by vitamin A mostly affects the rods
containing retinal and leads to a disease
means of intermediate masses of dentine
called as night blindness or poor night
to form ridges or lophs. vision. Tocopherol is the chemical name of
11. (c) Wharton's duct is the duct of the vitamin E, found mostly in wheat germ oil,
submaxillary or submandibular gland that brown flour etc. Its deficiency causes
occurs in the mouth on a papilla at the side sterility in rats. Vitamin C, also called as
of the frenulum of the tongue. The purpose ascorbic acid, is found mostly in citrus
of this duct is to drain the saliva. fruits, potatoes, tomatoes etc. Its deficiency
causes painful disease of the joints and
12. (c) Continued consumption of fat rich gums called scurvy.
diet causes hypercholesterolemia. 15. (d) Sugarcane and sugarbeet are the richest
Hypercholesterolemia is the presence of sources of sucrose, a disaccharide. It is
high levels of cholesterol in the blood. High most commonly found in plants, where it is
cholesterol raises your risk for heart transported in large quantity by phloem
disease, heart attack, and stroke. Kidney tissue. In humans, enzyme invertase
stones are solid mass made up of tiny (sucrase) present in duodenum of the small
crystals. There are different types of kidney intestine hydrolyses sucrose into one
stones. The exact cause depends on the molecule of glucose and one molecule of
type of stone like, calcium stones, uric acid fructose. Pepsin is proteolytic enzyme that
stone etc. Vitamin A toxicity or hydrolyses many proteins into smaller
hypervitaminosis A is having too much of molecules of peptones. Saliva of humans
vitamin A in the body. Ketonuria is contains salivary amylase (ptyalin) that
condition in which ketone bodies are hydrolyses starch into maltose, isomaltose
present in urine. Body productes excess and small dextrins. Lipases hydrolyse
ketone bodies as an alternate source of triglyceride fat into diglycerides, and then
energy during starvation or diabetes into monoglycerides alongwith fatty acids
mellitus (type 1). at each step.
13. (a) Oxyntic cells or Parietal cells, are the 16. (a) Parietal cells also called oxyntic cells are the
stomach epithelium cells that secrete gastric stomach epithelium cells that secrete gastric
acid intrinsic factor. These cells secrete acid and intrinsic factor. A cell of the gastric
hydrochloric acid (HCl) which makes the glands that secretes hydrochloric acid.
gastric juice acidic.(pH = 2.0-3.0). 17. (a) Fats present below the skin surface in our
Alpha cells of islets of Langerhans secretes body, is called subcutaneous fat deposition.
glucagon hormone which increase the It acts as insulator of body and prevent loss
glucose level in the blood by converting of heat from the body.
glycogen to glucose in liver cells. Kupffer's 18. (c) Xerophthalmia is caused by vitamin A
cells are specialized cells in the liver that deficiency. This generally occurs in poorer
destroy bacteria, foreign proteins, and section of the society because often they do
worn-out blood cells. Sebaceous glands not have adequate amount of fruits in diet.
and microscopic glands in the skin that 19. (a) The average fat content in buffalo milk is
secrete an oily/waxy matter (called sebum) 7.2% which is higher than human milk.
to lubricate the skin and hair of mammals. Lactose is higher in human milk than cow
and buffalo’s milk.
Digestion and Absorption B-89

20. (a) Fish has more polyunsaturated fatty acids 30. (c) Pancreatic amylase is a starch splitting
which act as natural antioxidant. enzymes similar to ptyalin by hydrolysing
21. (a) Endoscopy, a technique used to explore the starch and glycogen to maltose, isomaltose
stomach or other inner parts of the body, is and limit dixtrins.
based on total internal reflection.
31. (a) Trypsin is protein digesting enzyme present
22. (a) Lathyrism is caused by excessive
in the intestine of animals. Though it cannot
consumption of khesari dal.
digest casein (a milk protein), in predator
23. (d) Normal body temperature is 37 degree cen-
animals drinking the blood of their prey,
tigrade but in Kelvin scale 0ºC = 0ºC + 273 =
trypsin hydrolyses fibriongen of blood into
273ºK. When we convert 37ºC into Kelvin,
fibrin, leading to blood coagulation thus
it becomes 37 + 273 = 310ºK.
help in blood digestion. It also activates
24. (b) There are three pairs of salivary glands.
other pancreatic proteases.
These are parotids (below the cheeks), sub
maxillary/ or sub mandibular (lower jaw) and 32. (d) It is not lipases but the bile salts which are
the sublinguals (below the tongue) responsible for the emulsification of fats.
Bile salts are steroids secreted by the liver
25. (d)
in the bile. In the intestinal lumen, they
26. (a) Physiological value of carbohydrates is 4.0 reduce the surface tension of fat dropletes,
kcal/g, proteins 4.0 kcal/g and of fats is 9.0 causing their breakdown into many smaller
kcal/g. Hence, ones. A stable fine emulsion of fat is thereby
5 g raw sugar will yield formed. On the other hand, lipases are the
5 × 4.0 = 20.0 kcal enzymes which hydrolyse fats and oils.
4 g albumin (protein) will yield Lipases can digest fat in significant amounts
4 × 4.0 = 16.0 kcal only when large fat droplets are broken into
10 + 2 g of fat will yield tiny droplets to form a fine emulsion.
12 × 9.0 = 108.0 kcal Emulsificaion of fats by bile salts thus,
increases the lipase action on fats.
Total yield = 144 kcal.
33. (a) In the condition of obstructive jaundice the
27. (d) Milk protein can be digested by pepsin and entry of bile into the small intestine is
chymotrypsin prevented due to an obstruction in the bile
duct. As we know that bile salts help in the
Type B : Assertion Reason Questions
digestion of fats by emulsification and also
28. (a) Deficiency of ascorbic acid/vitamin C in their absorption by the formation of
causes scurvy. water soluble droplets called micelles from
whom fatty acids, glycerides, sterols and
29. (a) In the condition of obstructive jaundice the
fat soluble vitamins are absorbed into the
entry of bile into the small intestine is
intestinal cells. Therefore, in the absence
prevented due to an obstruction in the bile
of bile, the fats remain unabsorbed and
duct. As we know that bile salts helps in
consequently are eliminated out of the
the digestion of fats by emulsification and
body in the faeces.
also in their absorption by the formation of
water soluble droplets called micelles from 34. (c) Sucrase hydrolyses sucrose to glucose and
whom fatty acids, glycerides, sterols and fructose.
fat soluble vitamins are absorbed into the 35. (a)
intestinal cells. Therefore, in the absence
of bile, the fats remain unabsorbed and
consequently are eliminated out of the
body in the faeces.
EBD_7100
B-90 Topicwise AIIMS Solved Papers – BIOLOGY

Breathing and Exchange


17 of Gases
(a) Pulmonary ventilation of ‘A’ and ‘B’ is
TYPE A : MULTIPLE CHOICE QUESTIONS same.
1. Oxidative breakdown of respiratory substrates (b) Alveolar ventilation of ‘A’ and ‘B’ is same.
with the help of O2 is called as [1997] (c) Pulmonary ventilation of ‘A’ is greater than
(a) fermentation ‘B’.
(b) anaerobic respiration (d) Alveolar ventilation of ‘A’ is greater than
(c) R. Q. ‘B’.
(d) aerobic respiration 6. Oxyhaemoglobin can transport [2010]
(a) 8 ml of CO2/100 ml blood
2. Severe Acute Respiratory Syndrome (SARS)
(a) is caused by a variant of Pneumococcus (b) 5 ml of CO2/100 ml blood
pneumoniae. [2004] (c) 3 ml of CO2/100 ml blood
(b) is caused by a variant of the common cold (d) 2 ml of CO2/100 ml blood
virus (corona virus). 7. Which of the following match is correct?[2011]
(c) is an acute form of asthma. (a) Emphysema: reduction of surface area of
(d) affects non-vegetarians much faster than alveoli and bronchi
the vegetarians. (b) Pneumonia: occupational disease with
3. The diagram below represents part of a capillary asbestos
in a specific region of the human body. The (c) Silicosis: inflammation of alveoli
region labeled X represents part of [2009] (d) Asthma: excessive secretion of bronchial
mucus
8. Volume of air breathed in and out during normal
breathing is called [2012]
(a) Vital capacity (b) IRV
(c) ERV (d) Tidal volume
9. Much developed larynx of human male is called
(a) Aristole's lanttern (b) Syrinx [2012]
(c) Adam's apple (d) Muller's organ
(a) a glomerulus (b) an alveolus 10. The presence of CO2 in blood will lower pH
(c) a villus (d) the liver because CO2 combines with____, with the rate
4. In humans, the concentration of carbon dioxide of reaction increased by_____ . [2013]
in the plasma [2009] (a) H2O to form H+ and HCO3–, carbonic
(a) causes in creased production of anhydrase
hydrochloric acid. (b) H2 O to form only HCO 3 – , carbonic
(b) regulates gastric acid production by anhydrase
forming carbonic acid. (c) H2O to form only H+, carbonic ions
(c) regulates breathing rate by its effect on the (d) H+ to form HCO3–, oxyhaemoglobin
medulla. 11. Approximately seventy percent of carbon-
(d) causes inflammation of the tissues of the dioxide absorbed by the blood will be
bronchial tubes. transported to the lungs [2014]
(a) as bicarbonate ions
5. If the respiratory rate of ‘A’ is 35 breaths/min
and tidal volume 185 cc/breath and of ‘B’ is (b) in the form of dissolved gas molecules
25 breaths/min and tidal volume 259 cc/breath (c) by binding to RBC
then [2009] (d) as carbamino - haemoglobin
Breathing and Exchange of Gases B-91

12. During oxygen transport the oxyhaemoglobin 14. Assertion : Severe Acute Respiratory Syndrome
at the tissue level liberates oxygen to the cells (SARS) is originated in China.
because in tissue [2016] Reason : China is the most populated country
(a) O2 concentration is high and CO2 is low of the world. [2003]
(b) O2 concentration is low and CO2 is high
Directions for (Qs.15-17) : Each of these questions
(c) O2 tension is high and CO2 tension is low
contains an Assertion followed by Reason. Read them
(d) O2 tension is low and CO2 tension is high
carefully and answer the question on the basis of
TYPE B : ASSERTION REASON QUESTIONS following options. You have to select the one that
best describes the two statements.
Directions for (Qs. 13-14) : These questions consist
of two statements, each printed as Assertion and (a) If both Assertion and Reason are correct and
Reason. While answering these questions, you are Reason is the correct explanation of Assertion.
required to choose any one of the following five (b) If both Assertion and Reason are correct, but
responses. Reason is not the correct explanation of
(a) If both Assertion and Reason are correct and Assertion.
the Reason is a correct explanation of the (c) If Assertion is correct but Reason is incorrect.
Assertion. (d) If both the Assertion and Reason are incorrect.
15. Assertion : Oxyhaemoglobin dissociates near
(b) If both Assertion and Reason are correct but
the organ tissue due to Bohr effect and oxygen
Reason is not a correct explanation of the
is released. [2010]
Assertion.
Reason :Increased CO2 concentration reduces
(c) If the Assertion is correct but Reason is
the affinity of haemoglobin for oxygen.
incorrect.
16. Assertion: In mammals, complex respiratory
(d) If both the Assertion and Reason are incorrect.
system has developed. [2011]
(e) If the Assertion is incorrect but the Reason is Reason: Mammalian skin is impermeable to gases
correct. 17. Assertion : Inspiration occurs due to muscular
13. Assertion : Many visitors to the hills suffer from relaxation.
skin and respiratory allergy problems. Reason : During inspiration, the diaphragm and
Reason : Conifer trees produce a large quantity exter nal intercostal muscle contract
of wind-borne pollen grains. [2003] simultaneously. [2012]
EBD_7100
B-92 Topicwise AIIMS Solved Papers – BIOLOGY

Type A : Multiple Choice Questions 8. (d) Volume of air breathed in and out during
normal breathing is called tidal volume. It
1. (d) The oxidative breakdown of respiratory
is approximately 500 ml i.e., a healthy
substrates (like car bohydr ates, fats,
person can expire or inspire approximately
proteins) into CO2 and water, occurs in the
6000 to 8000 ml of air per minute as
presence of oxygen. It is called aerobic
breathing rate is 12-16 times/minute.
respiration.
2. (b) Severe acute respiratory syndrome (SARS) 9. (c) Larynx is a cartilaginous box which helps
is caused by mutant of Influenza/common in sound production and hence is called
cold virus (Corona virus). the sound box. Much developed larynx of
3. (b) An alveolus is a tiny air sac in the lung. It human male is called Adam's apple.
is the actual part of the lung that exchanges 10. (a) Carbon dioxide combines with H2O in the
atmospheric oxygen with carbon dioxide plasma to form H+ and HCO3–. The enzyme
from the blood. A glomerulus is a ball of carbonic anhydrase catalyzes the reaction.
capillaries found in the nephrons of the
kidneys (to filter nitrogenous wastes), a 11. (a) CO2 from the respiratory tissues to the
villus is a tiny projection of a capillary in lungs is transported by the blood in 3 ways:
the small intestine (to absorb digested (i) In dissolved state or as a physical
food), and the liver is the site of bile solution: Very small amount is physically
production and breakdown of amino acids. dissolved in plasma (7% i.e. 0.3 ml of CO2
4. (c) The medulla oblongata at the base of the by each 100 ml of blood).
brain increases breathing rate if the amount
(ii) Bicarbonate ions: About 70% (i.e. @
of carbon dioxide increases. It does not
2.5 ml per 100 ml of blood) CO2 diffuses in
regulate breathing rate by checking oxygen
plasma & then into RBCs where it (in the
content.
presence of carbonic anhydrase) combines
5. (c) The process by which a continuous
exchange of gases is maintained across with H2O to form carbonic acid which is
respiratory surface is often called external almost spontaneously dissociated into
respiration. The ventilation rate of an hydrogen ion and bicarbonate ions.
animal is the volume of air breathed per (iii) Carbaminohaemoglobin : 23%
minute i.e tidal volume x number of (i.e. 1 ml of CO2 per 100 ml of blood)
breathes per minute. It can be measured combines with haemoglobin forming an
with the aid of a respirometer. unstable compound.
6. (c) Oxyhaemoglobin can transport about 3ml 12. (d)
of carbon dioxide per 100 ml of blood.
7. (a) Cigarette smoking leads to the disease Type B : Assertion Reason Questions
emphysema. In this disease, terminal 13. (b) The skin problem could be due to pollen
bronchioles get obstructed. This reduces allergy and respiratory problem could be
the ventilation of the alveoli connected to due to the decrease in oxygen content,
them. Many alveoli coalesce together to since the atmosphere becomes thin as one
form large chambers due to destruction of goes up the hill.
their walls. This change of smaller alveoli
14. (b) The first patient of SARS was reported in
to large chambers reducing the area of
alveolar surface across which gases are February 2003 in China. Its causing agent
exchanged. All these changes reduce both is human corona virus (type of Influenza
oxygen uptake an d carbon dioxide virus) which spreads through contact,
elimination. respiratory secretions and cockroaches.
Breathing and Exchange of Gases B-93

15. (a) Bohr ,s effect is the effect of CO 2 on animals; so they need a more extensive
oxyhaemoglobin. Body tissues obtain respiratory surface. Thus, a complex
oxygen from oxyhaemoglobin because of mammalian respiratory system consists of
its dissociation caused by low O2 and high the nasal cavity, nasopharynx, larynx,
CO2 concentration. The increased CO2 trachea, bronchi, bronchiole and lungs.
concentration reduces the affinity of 17. (c) Inspiration is the result of muscular
haemoglobin for oxygen. contraction. The diaphragm and external
16. (b) Mammalian skin is impermeable so that intercostal muscles contract simultaneously.
water loss through it is minimised. But
The lateral thoracic wall moves outward and
mammals need far more oxygen to maintain
their high metabolic rates than lower upward.
EBD_7100
B-94 Topicwise AIIMS Solved Papers – BIOLOGY

18 Body Fluids and Circulation


TYPE A : MULTIPLE CHOICE QUESTIONS (b) It is coronary artery which is blocked by a
plaque and the same is being cracked.
1. Anti-serum contains [1997] (c) It is coronary vein in which the defective
(a) antigens (b) antibodies valves are being opened.
(c) leucocytes (d) RBCs (d) It is coronary vein blocked by a parasite
2. Which enzyme induces lysis of fibrinogen to (blood fluke) that is being removed.
fibrin during fibrinolysis ? [1997]
8. Hirudin is [2006]
(a) Plasmin (b) Thrombin
(a) A protein produced by Hordeum vulgare,
(c) Fibrin (d) Trypsin which is rich in lysine.
3. Which of the following blood group can be given (b) A toxic molecule isolated from Gossypium
to any patient ? [1999] hirusutum, which reduces human fertility.
(a) A (b) B (c) A protein produced from transgenic
(c) O (d) AB Brassica napus, which prevents blood
4. Which of the following chamber of heart has clotting.
the thickest muscular wall ? [1999] (d) An antibiotic produced by a genetically
(a) Left auricle (b) Left ventricle engineered bacterium, Escherichia coli.
(c) Right ventricle (d) Right auricle 9. The component of blood which prevents its
5. In which of the following pairs the two items coagulation in the blood vessels is [2007]
mean one and the same thing? [2004] (a) haemoglobin (b) plasma
(a) Malleus - Anvil (c) thrombin (d) heparin
(b) SA node - Pace maker 10. Thickening of arteries due to cholesterol
(c) Leucocytes - Lymphocytes deposition is [2007]
(d) Haemophilia - Blood cancer (a) arteriosclerosis (b) rheumatic heart
6. An artificial pace maker is implanted (c) blood pressure (d) cardiac arrest.
subcutaneously and connected to the heart in 11. Which one of the following is a matching pair?
[2003, 2008]
patients [2004]
(a) Lubb - Sharp closure of AV valves at the
(a) having 90% blockage of the three main beginning of ventricular systole.
coronary arteries. (b) Dup - Sudden opening of semilunar valves
(b) having a very high blood pressure. at the beginning of ventricular diastole.
(c) with irregularity in the heart rhythm. (c) Pulsation of the radial artery valves in the
(d) suffering from arteriosclerosis. blood vessels.
7. The figure below shows an angioplasty of the (d) Purkinje fibres-Initiation of the heart beat.
coronary blood vessel. Which one of the 12. A malfunction of the lymph nodes would most
following statements correctly describes, what likely interfere with the [2009]
is being done? [2006] (a) release of carbon dioxide into the lymph
(b) filtering of glucose from the lymph
(c) release of oxygen into the lymph
(d) filtering of bacteria from the lymph
13. Arteries supplying blood to the heart are
called [2010]
(a) It is coronary artery which has a cancerous (a) carotid arteries (b) hepatic arteries
growth that is being removed. (c) coronary arteries (d) pulmonary arteries
Body Fluids and Circulation B-95

14. A man whose blood group is not known meets (ii) RBC, WBC and blood platelets are
with a serious accident and needs blood produced by bone marrow.
transfusion immediately, which one of the blood (iii) Neutrophils bring about destruction and
groups readily available in the hospital will be detoxification of toxins of protein origin.
safe for transfusion? [2010] (iv) The important function of lymphocytes is
(a) O, Rh– (b) O, Rh+ to produce antibodies.
(c) AB, Rh – (d) AB, Rh+ (a) (i) and (ii) only (b) (i) and (iv) only
15. With reference to the blood in a normal
(c) (i) and (iii) only (d) (ii) and (iii) only
person, which one of the following statements
is correct? [2010] 20. The diagram given here is the standard ECG of
(a) Compared to arteries, veins are less a normal person, the P-wave represents the :
numerous and hold less of the body’s
R
blood at any given time.
(b) Blood cells constitute about 70 percent of
the total volume of the blood. P Q S T
(c) White blood cells (WBC) are made by
lymph nodes only. (a) Initiation of the ventricular contraction
(d) The blood has more platelets than WBC. (b) Beginning of the systole
16. Which of the following organs is the blood (c) End of systole
bank? [2011] (d) Contraction of both the atria [2014]
(a) Heart (b) Lungs 21. Bulk of carbon dioxide (CO2) released from body
(c) Spleen (d) Liver
tissues into the blood is present as [2015]
17. Which one of the following is a matching pair of
a certain body feature and its value/count in a (a) 70% carbamino-haemoglobin and 30% as
normal human adult? [2003, 2008, 2011] bicarbonate
(a) Urea 5-10 mg / 100 ml of blood (b) carbamino-haemoglobin in RBCs
(b) Blood sugar (fasting) - 70-100 mg/100 ml (c) bicarbonate in blood plasma and RBCs
(c) Total blood volume - 5-6 (d) free CO2 in blood plasma
(d) ESR in Wintrobe method - 9-15 mm in males 22. Given below is the ECG of a normal human.
and 20-34 mm in females
Which one of its components is correctly
18. Given below is the ECG of a normal human.
Which one of its components is correctly interpreted below?
interpreted below? [2013] R
R
T [2015]
P Q S
P Q S T

(a) Peak P and Peak R together - systolic and (a) Peak P and Peak R together - systolic and
diastolic blood pressures diastolic blood pressures
(b) Peak P - Initiation of left atrial contraction (b) Peak P - Initiation of left atrial contraction only
only (c) Complex QRS - One complete pulse
(c) Complex QRS - One complete pulse (d) Peak T - Initiation of total cardiac
(d) Peak T - Initiation of total cardiac contraction
contraction
19. Which of the following statements are wrong?
TYPE B : ASSERTION REASON QUESTIONS
[2013] Directions for (Qs. 23-29) : These questions consist
(i) Leucocytes disintegrate in the spleen and of two statements, each printed as Assertion and
liver. Reason. While answering these questions, you are
EBD_7100
B-96 Topicwise AIIMS Solved Papers – BIOLOGY
required to choose any one of the following five 29. Assertion : Smaller the organism higher is the
responses. rate of metabolism per gram weight.
(a) If both Assertion and Reason are correct and Reason : The heart rate of a six month old baby is
the Reason is a correct explanation of the much higher than that of an old person. [2007]
Assertion.
Directions for (Qs.30-34) : Each of these questions
(b) If both Assertion and Reason are correct but contains an Assertion followed by Reason. Read them
Reason is not a correct explanation of the carefully and answer the question on the basis of
Assertion. following options. You have to select the one that
(c) If the Assertion is correct but Reason is best describes the two statements.
incorrect.
(a) If both Assertion and Reason are correct and
(d) If both the Assertion and Reason are incorrect. Reason is the correct explanation of Assertion.
(e) If the Assertion is incorrect but the Reason is (b) If both Assertion and Reason are correct, but
correct. Reason is not the correct explanation of
23. Assertion : Muscle fibres of SA node possess Assertion.
the lowest rhythmicity among all cardiac (c) If Assertion is correct but Reason is incorrect.
muscles. (d) If both the Assertion and Reason are incorrect.
Reason : Due to this fact, it can initiate excitatory
30. Assertion : Prothrombinase enzyme act as
waves at the highest rate. [1999]
24. Assertion : Saline water is not given to patients antiheparin. [2010]
of hypertension. Reason : Heparin prevent coagulation of blood
Reason : Saline water can cause vomiting and in blood vessels.
may drop blood pressure suddenly causing 31. Assertion : Blood is coloured in the insects.
cardiac arrest. [2000] Reason : Insect blood has no role in O2 transport.
25. Assertion : Blood pressure is arterial blood [2012, 2013]
pressure. 32. Assertion : When there is a fall in the blood
Reason : Blood pressure is measured by pressure due to loss of blood volume, this is
sphygmomanometer. [2000] compensated by vasoconstriction of veins.
26. Assertion: WBCs accumulate at site of wounds Reason : Veins hold the extra amount of blood
by diapedesis. which can be shifted to the arteries as required.
Reason: It is squeezing of leucocytes from [2010, 2015]
endothelium. [2002] 33. Assertion : Lub is a heart sound which is
27. Assertion : Persons suffering from haemophilia produced during each cardiac cycle.
fail to produce blood clotting factor VIII. Reason : It is associated with the closure of the
Reason : Prothrombin producing platelets in tricusp and bicuspid valves.
such persons are found in very low [2016]
concentration. [2005]
28. Assertion (A) : Blood coagulates in uninjured
blood vessels.
Reason (R) : Uninjured blood vessels release an
anticoagulant heparin. [2007]
Body Fluids and Circulation B-97

Type A : Multiple Choice Questions walls of blood vessels. This causes the
arteries to lose their elasticity and get
1. (b) Anti-serum is a blood serum that contains
stiffened. This is called arteriosclerosis or
antibodies. hardening of arteries.
2. (b) During blood clotting, lysis of fibrinogen 11. (a) Lubb sound is caused partly by the closure
to fibrin occurs by plasma thrombin. Later of the bicuspid and tricuspid valves and
on fibrin entangles corpuscles to form a partly by the contraction of the muscles in
clot. Thrombin is not mormally found in the ventricles. Lubb is the first heart sound.
circulating blood, but instead it is repressed 12. (d) The lymphatic system not only regulates
by its inactive precursor prothrombin. The fluid amounts in the blood but also helps
conversion of prothrombin to thrombin to fight infection. The lymph nodes
requires blood platelets, calcium ions and produce white blood cells and filter harmful
thromboplastin. bacteria, that is why they tend to enlarge
3. (c) Blood group ‘O’ is universal donor. Group when we have an infection.
‘O’ blood, with no antigens, can be given 13. (c) Coronary arteries supply the blood to the
heart. Carotid arteries supply the blood to
to anyone.
head region. Hepatic arteries supply the
4. (b) Left ventricle of the heart has the thickest blood to liver while the pulmonary artery
wall because it has to pump the blood to supplies blood to the lungs.
the farthest end of the body. 14. (b) Accordin g to ABO system of blood
5. (b) Malleus is hammer shaped; lymphocytes grouping, O type blood can be given to
are type of leucocytes and hemophilia is person of all types of blood, i.e., O, A, B
bleeder’s disease (genetic) disorder). SA and AB. Hence, a person with O type blood
node acts as pacemaker because it is called universal donor.
functions as originator of cardiac impulse. Accordin g to Rh system of blood
6. (c) Artificial pace maker is implanted to the grouping, most of the people have blood
patients where heart beat level drops in which there is a substance called Rh
abnormally low (30-40) due to disease or factor. Rh stands for rhesus. This type of
operations. blood can be donated only in rarest of rare
7. (b) In this figure, coronary angioplasty is being chances. In India, about 97% of people are
done where atherosclerotic plaque, which Rh+. So, if an Rh+ person is transfused with
builds up and clogs the coronary arteries, Rh+ blood, then it is safe. But if an Rh
is compressed against the vessel wall by negative (Rh–) person is transfused with
expanding a balloon like device inserted Rh+ blood then he/she will develop anti-
through a catheter that has been threaded Rh factor i.e., antibodies in his/her blood,
through the artery. and there might beno harm is, but may kill
the recipient if a second Rh + transfusion is
8. (c) Hirudin is an anticlotting agent produced
done.
by transgenic Brassica napus.
9. (d) Blood contains an anticoagulant called 15. (d) The number of blood platelets per cubic
mm in human blood is 3 lacs while the
heparin, which prevents the activation of
number of WBCs are 5000/cubic mm of
prothrombin. The latter is an inactive
blood. Veins are as complex as the arteries.
globulin which is required during blood
Veins and arteries both are types of blood
coagulation. Heparin is released from the
vessels. Arteries carry blood from heart to
mast-cell granules. different organs while vein carries blood
10. (a) High proportion of cholesterol in blood from different organs to heart. At any given
leads to deposition of cholesterol on the
EBD_7100
B-98 Topicwise AIIMS Solved Papers – BIOLOGY
time in a healthy human, the blood amount 24. (c) Saline water increases the blood pressure
is same in both, as the circulation of blood due to the presence of NaCl in it. Hence,
never stops. the reason for the assertion is false.
Blood consists of two parts: 25. (b) Blood pressure is the arterial pressure of
The plasma (water, proteins, inorganic salts blood exerted on the wall of arteries with
and other elements) constitutes 55-60% of each heart beat. It is measured form the
blood while cellular part constitutes 40- brachial artery in the elbow pit. It is
45% of total blood. WBCs are produced in expressed as
red bone marrow, lymph nodes and systolic pressure (mm/Hg)
sometimes even in liver and spleen.
diastolic pressure (mm/Hg)
16. (c) Spleen is referred to as a blood bank of the
body because it is the organ having the Arterial (superficial; arteries) blood
function of making and storing pressure is measured by
lymphocytes and red corpuscles. These are sphygmomanometer.
squeezed out into the blood-stream when 26. (b) WBC’s are wandering cells capable of
the body needs more in circulation, as for coming out of blood capillar ies by
instance in haemorrhage or shock. amoeboid movement called diapedesis.
17. (b) Fasting glucose level is 70-110 mg/decilitre. 27. (a) Haemophilia is caused by lack of activity
Total blood volume in normal adult human of blood clotting factor VIII or IX and they
is 5-6 litres. Blood is a fluid connective show platelet function disorder.
tissues. Its cells consists of corpuscles. 28. (d) When an injury is caused to a blood vessel,
Plasma represents matrix of blood. Blood bleeding starts which is stopped by blood
is mesodermal in origin and salty in taste. clotting. At the site of injury blood platelets
Its pH is 7.3 – 7.4. release platelet factor - 3 and injured tissues
18. (c) 19. (c) release thromboplastin. The two combine
to form prothrombinase enzyme which
20. (d) The P-wave represents the electrical exci-
converts prothrombin to thrombin. The
tation (or depolarisation) of the atria, which
latter stimulates formation of fibrin thread
leads to the contraction of both the atria.
or clot. Blood contains an anticoagutant
The QRS complex represents the
heparin which prevents blood clotting in
depolarisation of the ventricles, which ini-
uninjured vessels.
tiates the ventricular contraction. The con-
traction starts shortly after Q and marks 29. (b) The basal metabatic rate is defined as the
the beginning of the systole. energy requirement of human body at rest.
21. (c) BMR of smallest animals are generally
22. (c) higher then larger animals. Peoples with
higher metabolism means that they have
Type B : Assertion Reason Questions higher heart rate. Heart rate of baby is
23. (e) The sinoatrial or sinoauricular node 70-190 times/minute, whereas adults
determines the rate of heart beat by (including serious) is 60-100 times/ minute.
determining the rate of discharge of cardiac
30. (b) Prothrombinase enzyme is necessary for
impulse. It is called the pace maker. It is
blood clotting. It acts as antiheparin.
formed of specialized cardiac muscles and
Coagulation of blood in vessels is
is located in the right atrial wall near the
prevented by heparin, a quick acting
opening of superior venacava. These
anticoagulant. It inhibits conversion of
muscles are self excitable. Since it is self
prothrombin to thrombin and is used in
excitable, it can produce waves at highest
open-heart surgery.
rate. Hence, the assertion is incorrect.
Body Fluids and Circulation B-99

31. (b) Insect blood is colourless and does not 33. (b) Lub and dub are two heart sounds, which
play any role in transport of oxygen. Insects occurs due to the closure of cuspid valves
have tracheal respiration. and semilunar valves respectively. Lub is
32. (a) When the blood pressure of an individual the first heart sound which is formed due
decreases due to loss of blood volume, then to closure of atrioventricular valves at the
vasoconstriction of veins occurs. This beginning of ventricular systole. It is low
shifts the little amount of blood from veins pitched of long duration (0.15 sec).
to arteries.
EBD_7100
B-100 Topicwise AIIMS Solved Papers – BIOLOGY

Excretory Products and their


19 Elimination
TYPE A : MULTIPLE CHOICE QUESTIONS 8. If Henle’s loop were absent from mammalian
nephron which of the following is to be expected
1. Podocytes occur in [1998] [2015]
(a) large intestine (a) there will be no urine formation
(b) glomerulus of kidney
(b) there will be hardly any change in the
(c) wall of capillaries
quality and quantity of urine formed
(d) neck region
(c) the urine will be more concentrated
2. The end product of ornithine cycle is [1999]
(d) the urine will be more dilute.
(a) urea (b) uric acid
9. Which blood vessel in mammals would normally
(b) NH3 (d) CO2 carry the largest amount of urea? [2016]
3. Reabsorption in tubules of nephrons occurs by (a) Dorsal aorta (b) Hepatic vein
[2000] (c) Hepatic portal vein (d) Renal vein
(a) osmosis (b) diffusion 10. In ornithine cycle, enzyme arginase breaks down
(c) active transport (d) both (b) & (c) arginine into [2016]
4. Toxic substances are detoxified in human body (a) Citrulline and ammonia
in [2001] (b) Ornithine and ammonia
(a) kidney (b) lungs (c) Ornithine and urea
(c) liver (d) stomach (d) Citrulline and urea.
5. In which of the following minimum content of TYPE B : ASSERTION REASON QUESTIONS
urea is present ? [2012]
(a) Hepatic portal vein Directions for (Qs. 11-13) : These questions consist
(b) Portal vein of two statements, each printed as Assertion and
(c) Renal vein Reason. While answering these questions, you are
(d) Vena cava required to choose any one of the following five
6. Duct of Bellini is concerned with [2012] responses.
(a) Filtration of urine (a) If both Assertion and Reason are correct and
(b) Purification of urine the Reason is a correct explanation of the
(c) Conduction of urine Assertion.
(b) If both Assertion and Reason are correct but
(d) All the above
Reason is not a correct explanation of the
7. Which one of the following statements in regard Assertion.
to the excretion by the human kidneys is
(c) If the Assertion is correct but Reason is
correct? [2013]
incorrect.
(a) Ascending limb of Loop of Henle is
(d) If both the Assertion and Reason are incorrect.
impermeable to electrolytes (e) If the Assertion is incorrect but the Reason is
(b) Descending limb of Loop of Henle is correct.
impermeable to water 11. Assertion : During physiology of excretion,
(c) Distal convoluted tubule is incapable of deamination does not take place in liver.
reabsorbing HCO3– Reason : Deamination is a process to make use
(d) Nearly 99 per cent of the glomerular filtrate of excess of amino acids which cannot be
is reabsorbed by the renal tubules incorporated into protoplasm. [2001]
Excretory Products and their Elimination B-101

12. Assertion : Secreting hypotonic urine is 14. Assertion : Ultrafiltration takes place in presence
effective in reducing urinary loss of water. of effective filtration pressure.
Reason : Hypotonic urine is more concentrated Reason : In ultrafiltration process, blood is filtered
and higher in osmotic pressure than the blood. in Bowman’s capsule, filtered fluid contain protein
[2007] & blood corpuscles also. [2010]
13. Assertion : Aldosterone is a steroid hormone 15. Assertion : In vertebrates, the liver is also
and is important in the control of sodium and referred as an accessory excretory organ.
potassium ion concentration in mammals. Reason : Liver helps kidneys in the secretion of
Reason : It upgrades sodium ion concentration urine. [2012]
in the ECF by promoting reabsorption of sodium 16. Assertion : Main constituent of human urine is
ions from renal tubules and excretion of ammonia.
potassium ions in urine. [2007] Reason : If human urine is allowed to stand for
some time, it smells strongly of ammonia. [2013]
Directions for (Qs.14-18) : Each of these questions
17. Assertion : Hemodialysis can save and prolong
contains an Assertion followed by Reason. Read them the life of uremic patients.
carefully and answer the question on the basis of Reason : Waste products like urea can be
following options. You have to select the one that removed from the blood by the process of
best describes the two statements. hemodialysis. [2014]
(a) If both Assertion and Reason are correct and 18. Assertion : In the descending limb of loop of
Reason is the correct explanation of Assertion. Henle, the urine is hypertonic, whereas in
(b) If both Assertion and Reason are correct, but ascending limb of loop of Henle, the urine is
Reason is not the correct explanation of hypotonic.
Assertion. Reason : Descending limb is impermeable to
(c) If Assertion is correct but Reason is incorrect. sodium, while ascending limb is impermeable to
(d) If both the Assertion and Reason are incorrect. water. [2016]
EBD_7100
B-102 Topicwise AIIMS Solved Papers – BIOLOGY

Type A : Multiple Choice Questions 12. (d) Hypotonic urine means concentration of
urine is less than that of blood. Hypertonic
1. (b) Podocytes are specialized visceral
urine is more concentrated than blood and
epithelial cell in the Bowman's capsule in
has high osmotic pressure than the blood,
the kidneys that wraps around the
therefore it helps in reducing the loss of
capillaries of the glomerulus. It helps filter
water with urine. The urine is filtered by
blood in the glomerulus of the kidney.
the Bowman's capsule. The tubules of
2. (a) The ornithine cycle occurs in the
nephrons reabsorb a large quantity of water
mitochondria of liver cells. It is meant for
making urine more concentrated.
urea formation.
13. (a) Aldosterone is one of the important
3. (d) Tubular r eabsorption is the second
mineralocorticoids in humans secreted by
process in urine formation through filtrate.
adrenal cortex. Its main function is to
In this, most of the filtrate passes out of
regulate sodium content of the body . It
rephron tubule and returns to the blood
increases sodium ion concentration in the
through peritubular capillaries. Tubular
blood by absorbing sodium ions from renal
reabsorption occurs by diffusion and
tubules. Excessive production of
active transport.
aldosterone causes a disease aldosteronism.
4. (c) Liver is the primary site of detoxification
Its symptoms include high blood pressure,
and elimination of body wastes and
high blood volume.
poisons. Liver detoxifies endotoxins, e.g.
14. (c) Ultrafiltration takes place in renal corpuscle
toxic NH3 combined with CO2 to form less
of uriniferous tubule. It takes place in
toxic urea. It also detoxifies alcohol and
presence of effective filtration pressure.
convert them to acetaldehyde and then
During the process, blood is filtered and
harmless acetyl CoA.
contains only blood plasma – proteins. The
5. (c) Renal veins connects the kidney to inferior
filtered blood entering into Bowman’s
vena cava. They carry the blood purified by
capsule is called glomerular filtrate.
the kidney and carry minimum quantity of urea.
Glomerular filtrate = Blood – (Blood
6. (c) Renal collecting tubules are also known as
corpuscles + plasma proteins)
duct of bellini. These are the long narrow
15. (c) In vertebrates, the lungs, liver & skin are
tubes in the kidney that conduct urine from
referred as accessory excretory organs
the nephrons to larger ducts that leads to
because besides the urinary system these
urinary bladder.
organs also participate in the removal of
7. (d)
waste products from the body. The liver is
8. (d) Henle's loop is responsible for the
the principal organ for the excretion of
reabsorption of water and sodium chloride cholesterol, bile pigments (bilirubin and
from the urine. Hence, in the absence of biliverdin) and inactivated products of
Henle's loop the urine will become more steroid hormones, some vitamins and many
dilute. drugs. It secretes these substances in the
9. (b) 10. (c) bile and indirectly helps by formation of
urea through amino acids in ornithine cycle.
Type B : Assertion Reason Questions
It has no role in secretion of urine.
11. (d) Deamination is the process of converting 16. (d) Urea is the chief nitrogenous constituent
amino acid to keto acid with the release of of human urine, though it possesses small
amount of ammonia. But when the urine is
NH3. It occurs in the liver.
allowed to stand for sometime, bacterial
Excretory Products and their Elimination B-103

degradation occurs and it leads to the hemodialysis saves and prolongs the life
production of ammonia from urine. And of many uremic patients.
thus it smells strongly.
18. (a) Descending limb of Henle is permeable to
17. (a) The blood urea level rises abnormally water but not to sodium. Consequently
(uremia) in patients suffering from renal water moves out into interstitium and
failures. In uremia patients an artificial concentration of sodium in tubular filtrate
kidney is used for removing accumulated rises making filtrate hypertonic. Ascending
waste products like urea from the blood by loop is impermeable to water but permeable
to sodium and makes the filtrate hypotonic.
a process called hemodialysis. In this way,
EBD_7100
B-104 Topicwise AIIMS Solved Papers – BIOLOGY

20 Locomotion and Movement

TYPE A : MULTIPLE CHOICE QUESTIONS (a) tibia and tarsals


(b) femur and fibula
1. Cranium of human contains [2000]
(c) fibula and phalanges
(a) 12 bones (b) 8 bones
(d) tarsals and femur
(c) 14 bones (d) 20 bones
7. A cricket player is fast chasing a ball in the field.
2. Which of the following is made up of a single Which one of the following groups of bones are
bone in mammal ? [2001]
directly contributing in this movement?[2006]
(a) Dentary (b) Hyoid
(a) Femur, malleus, tibia, metatarsals
(c) Upper jaw (d) All of these
3. Sella turcica is found [2001] (b) Pelvis, ulna, patella, tarsals
(a) near pituitary (b) in bone (c) Sternum, femur, tibia, fibula
(c) in joints (d) near thyroid (d) Tarsals, femur, metatarsals, tibia
4. Which one of the following is a sesamoid bone? 8. The shoulder blade is made of [2007]
[2003] (a) clavicle (b) humerus
(a) Pelvis (b) Patella (c) ilium (d) scapula
(c) Pterygoid (d) Pectoral girdle 9. The sensation of fatigue in the muscles after
5. Two of the body parts which do not appear in prolonged strenuous physical work, is caused
MRI may be [2005] by [2010]
(a) molar teeth and eye lens (a) a decrease in the supply of oxygen
(b) scapula and canines (b) minor wear and tear of muscle fibres
(c) ligaments and ribs (c) the depletion of glucose
(d) tendons and premolars (d) the accumulation of lactic acid
6. Given below is a diagram of the bones of the left 10. Sesamoid bone is derived from- [2012]
(a) Cartilage (b) Areolar tissue
human hindlimb as seen from front. It has certain
(c) Tendon (d) Ligament
mistakes in labelling. Two of the wrongly labelled 11. Select the correct matching of the type of the
bones are [2005] joint with the example in human skeletal system:
[2014]
Type of joint Example
(a) Cartilaginous between frontal and
Femur joint parietal
(b) Pivot joint between third and
fourth cervical
vertebrae
Fibula (c) Hinge joint between humerus and
Tibia pectoral girdle
(d) Gliding joint between carpals
12. Wish bone in birds is formed from the bones of
Tarsals
[2016]
Phalanges (a) Shoulder girdle (b) Hip girdle
(c) Keeled sternum (d) Skull bones
Locomotion and Movement B-105

13. The given figure represents the cross bridge 14. Assertion: Fatigue is inability of muscle to relax.
cycle in skeletal muscle. What does the step B Reason: It is due to lactic acid accumulation by
in the figure represents? [2017] repeated contractions. [1998]
Directions for (Qs.15-19) : Each of these questions
Myosin head
(high-energy
ADP contains an Assertion followed by Reason. Read them
P1
configuration)
carefully and answer the question on the basis of
A following options. You have to select the one that
Thin filament
best describes the two statements.
ADP
(a) If both Assertion and Reason are correct and
ATP ADP Thick filament
hydrolysis P1 Reason is the correct explanation of Assertion.
B
(b) If both Assertion and Reason are correct, but
D
Reason is not the correct explanation of
ATP
Assertion.
Myosin head
ATP (low-energy
configuration)
(c) If Assertion is correct but Reason is incorrect.
(d) If both the Assertion and Reason are incorrect.
C
15. Assertion : Ball and socket joints are the most
(a) Attachment of myosin head to actin
mobile joints.
forming cross bridge.
(b) Release of phosphate. Myosin changes Reason : Synovial fluid is present here.
shape to pull actin. [2012]
(c) Attachment of new ATP to myosin head. 16. Assertion : Intercalated discs are important
The cross bridge detaches. regions of cardiac muscle cells.
(d) Splitting of ATP into ADP and Pi. Myosin Reason : Intercalated discs function as boosters
cocks into its high energy conformation.T for muscle contraction waves. [2012]
17. Assertion : Arthritis or inflammation of a joint
TYPE B : ASSERTION REASON QUESTIONS
makes the joint painful.
Directions for (Qs. 14-15) : These questions consist Reason : Some toxic substances are deposited
of two statements, each printed as Assertion and at the joint. [2013]
Reason. While answering these questions, you are 18. Assertion : The phase of muscle contraction
required to choose any one of the following five occurs when myosin binds and releases actin.
responses. Reason : Muscle contraction is initiated by a
(a) If both Assertion and Reason are correct and signal sent by the peripheral nervous system
the Reason is a correct explanation of the via motor neuron. [2016]
Assertion. 19. Assertion : Inflammation of a skeletal joint may
(b) If both Assertion and Reason are correct but immobilize the movements of the joint.
Reason is not a correct explanation of the Reason :Uric acid crystals in the joint cavity
Assertion. and ossification of articular cartilage lead to this.
(c) If the Assertion is correct but Reason is [2006, 2017]
incorrect.
(d) If both the Assertion and Reason are incorrect.
(e) If the Assertion is incorrect but the Reason is
correct.
EBD_7100
B-106 Topicwise AIIMS Solved Papers – BIOLOGY

Type A : Multiple Choice Questions joints allow the bones to glide past one
another in any direction along the plane of
1. (b) Cranium of human body consists of 8 bones. the joint - up and down, left and right, and
There are 1 frontal, 2 parietal, 2 temporal, 1 diagonally. Many gliding joints are formed
occipital, 1 sphenoid and 1 ethmoid bone. in the appendicular skeleton between the
2. (b) Hyoid is a horse shoe shaped bone present carpal bones of the wrist; between the
in neck between lower law and sound box carpals and the metacarpals of the palm;
(larynx). It is not articulated to any bone, between the tarsal bones of the ankle; and
but is simply suspended, from temporal between the tarsals and the metatarsals of
bones by means of ligaments. Hyoid the foot.
provides surface for the attachment of 12. (a) Two clavicles fuse with one inter-clavicle
tongue muscles. to form ‘Wish bone’ or ‘Bone of merry
3. (b) Sella turcica/turkish saddle/pituitary fossa thought’.
is a depression in sphenoid bone of skull
in which pituitary gland lies. 13. (b) Step A: Attachment of myosin head to actin
4. (b) Sesamoid bones are formed in the tendons forming cross bridge.
at the joints, e.g. patella. Step B: Release of phosphate. Myosin
5. (b) MRI machine does not show face image of changes shape to pull actin.
bone and calcium, e.g. scapula, canine. It Step C: Attachment of new ATP to myosin
is also not suitable for patients with cardiac head. The cross bridge detaches.
pace makers. Step D: Splitting of ATP into ADP and Pi.
6. (c) Fibula and phalanges marked parts are Myosin cocks into its high energy
actually tibia and metatarsals. conformation.
7. (d) In a fast chasing cricketer, foot and leg
bones contribute directly in this movement Type B : Assertion Reason Questions
e.g. femur, tibia, tarsal and metatarsals. 14. (a) Due to the accumulation of lactic acid,
8. (d) Shoulder blade is made up of scapula. It is muscles do not respond to a stimuli. After
a flat, triangular-shaped largest bone a prolonged previous activity.
present in each half of the pectoral 15. (b) Synovial fluid is a thick sticky fluid secreted
(shoulder) girdle. It is joined to the clavicle by synovial membranes into the synovial
in front. The clavicle is well developed in cavity. Though the presence of synovial
humans that links scapula to the sternum. fluid is one of the reasons behind the
9. (d) The sensation of fatigue in the muscles mobility of the joints, but the most accurate
after prolonged strenuous physical work reason is the arrangement of the bones at
is caused by the accumulation of lactic the joint, the spheroidal ball-like end of one
acid. bone articulates here with the cup-shaped
10. (c) Sesmoid bones are embedded in tendons. depression in another. This allows the bone
Sesmoids are found in locations where a with the ball head to be moved freely in
tendon passes over a joint, such as the many planes. Shoulder joints and hip joints
hand, knee and foot. Functionally they act are the ball-and-socket joints.
to protect the tendon and its mechanical 16. (a) Cardiac muscle cells are short cylindrical
effect. cells joined end to end and by side
11. (d) A gliding joint is a common type of synovial branching to form a network. Intercalated
joint formed between bones that meet at discs are the dense junctions formed in
flat or nearly flat articular surfaces. Gliding between the cardiac muscle cells where
Locomotion and Movement B-107

they meet each other. Intercalated discs are 18. (c) The phase of muscle contraction occurs
the specialised regions of the cell when myosin binds and releases actin.
membranes. As cardiac muscle possesses Muscle contraction is initiated by a signal
considerable rhythmicity and generates its sent by the central nervous system via a
own wave of excitation, these discs motor neuron. A motor neuron along with
function as boosters for muscle contraction the muscle fibres connected to it constitutes
wave. a motor unit.
17. (c) Arthritis or inflammation of a joint makes 19. (a) Painful inflammation of the synovial
the joint painful and may even immobilise membrane of the joints results in stiffening
the movements at the joint. This may result of joints and painful movement Uric acid
from a lack of the synovial fluid at the joint. accumulation in the joints can lead to
The ossification of the articular cartilage, painful movement of joint.
deposition of uric acid crystals in the joint
cavity or other changes at the joint.
EBD_7100
B-108 Topicwise AIIMS Solved Papers – BIOLOGY

21 Neural Control and Coordination

9. Excessive stimulation of vagus nerve in humans


TYPE A : MULTIPLE CHOICE QUESTIONS may lead to [2003]
1. The vagus nerve is the cranial nerve numbering (a) hoarse voice
[1997] (b) peptic ulcers
(a) 10th (b) 9th (c) efficient digestion of proteins
(c) 5th (d) 8th (d) irregular contraction of diaphragm
2. Sensation of stomach pain is due to [1998] 10. A person is wearing spectacles with concave
(a) interoceptors (b) exteroceptors lenses for correcting vision. While not using
(c) teloreceptors (d) all of these the glasses, the image of a distant object in his
3. Which is the example of conditioned reflex ? case will be formed [2003]
[1999] (a) on the blind spot
(a) Eyes closed when anything enter into it. (b) behind the retina
(b) Hand took up when piercing with needle. (c) in front of the retina
(c) Salivation in a hungry dog in response to (d) on the yellow spot
ringing of a bell. 11. Unidirectional transmission of a nerve impulse
(d) Digestion food goes forward in alimentary through nerve fibre is due to the fact that[2004]
canal. (a) nerve fibre is insulated by a medullary
4. Otorhinolaryngology is the study of [1999] sheath.
(a) brain cells (b) sodium pump starts operating only at the
(b) bird anatomy cyton and then continues into the nerve fibre.
(c) locomotary organs (c) neurotransmitters are released by dendrites
(d) ENT and not by axon endings.
5. If frog’s brain is crushed, even then its leg moves (d) neurotransmitters are released by the axon
on pinpointing . It is called [2001] endings and not by dendrites.
(a) simple reflex 12. Examine the diagram of the two cell types A and
(b) conditional reflex B given below and select the correct option:
(c) neurotransmitter function
[2006]
(d) autonomic nerve conditions
6. Which of the following is not a mental disorder?
(a) Epilepsy (b) Neurosis [2001]
(c) Psychosis (d) Plague
7. The 5th cranial nerve of frog is called
[1998, 2001]
(a) optic nerve (b) vagus nerve A B
(c) trigeminal nerve (d) olfactory nerve
8. The crystal of lead zirconate is a key component (a) Cell A is the rod cell found evenly all over
of [2003] retina.
(a) electroencephalography (b) Cell A is the cone cell more concentrated in
(b) electrocardiography the fovea centralis.
(c) magnetoencephalography (c) Cell B is concerned with colour vision in
bright light.
(d) sonography
(d) Cell A is sensitive to low light intensities.
Neural Control and Coordination B-109

13. A person, who shows unpredictable moods, 20. Multipolar nerve cells are present in [2012]
outbursts of emotion, quarrelsome behaviour (a) Cochlea
and conflicts with others, is suffering from (b) Dorsal root ganglia of spinal cord
(a) borderline personality disorder (BPD) (c) Retina of eye
(b) mood disorder [2006] (d) Brain
(c) addictive disorder 21. Neurons receive signals thr ough their
(d) schizophrenia __________ and send signals to other neurons
14. Which of the following is an eye disease? through their __________. [2013]
(a) hepatitis (b) measles (a) dendrites ... receptors
(c) glaucoma (d) bronchitis (b) end feet ... cell bodies and dendrites
15. Given below is a table comparing the effects of (c) cell bodies and dendrites ... axons
sympathetic and parasympathetic nervous
(d) transmitter vesicles ... axons
system for four features (1-4). [2006]
Which one feature is correctly described? 22. Which of the following ions are required for
nerve conduction ? [2016]
(a) Ca++, Na+ and K+ (b) Ca++ and Mg++
Sympathetic (c) Mg++ and K+ (d) Na+ and K+
Parasympahtetic
Feature nervous 23. The following diagram indicates the reflex arc.
nervous system
system Identify the parts labelled as A, B, C, D, E, F and
(a) Salivary Stimulates G. Choose the correct option
Inhibits secretion
glands secretion
(b) Pupil of
Dilate Constricts
the eye
(c) Heart rate Decrease Increase
(d) Intestinal
Stimulates Inhibits
peristalsis

16. Hearing impairment affects which part of


[2016]
brain? [2007]
(a) A = sense organ; B = sensory nerve;
(a) Frontal lobe (b) Parietal lobe
C = dorsal horn;
(c) Temporal lobe (d) Cerebellum
D = interneuron; E = ventral horn;
17. The black pigment in the eye which reduces the
F = motor nerve; G = effector
internal reflection is located in [2007]
(b) A = sense organ; B = sensory nerve;
(a) retina (b) iris C = ventral horn;
(c) cornea (d) sclerotic D = interneuron; E = dorsal horn;
18.. Bipolar nerve cells are present in [2012] F = motor nerve; G = effector
(a) Skin tactile corpuscles (c) A = effector; B = motor nerve;
(b) Spinal cord C = dorsal horn;
(c) Retina of eye D = interneuron; E = ventral horn;
(d) All the above F = sensory nerve; G = effector
19. Fenestra ovalis is the opening of [2012] (d) A = effector; B = motor nerve;
C = ventral horn; D = interneuron;
(a) Cranium (b) Tympanum
E = dorsal horn; F = sensory nerve;
(c) Tympanic cavity (d) Brain
G = sense organ.
EBD_7100
B-110 Topicwise AIIMS Solved Papers – BIOLOGY

TYPE B : ASSERTION REASON QUESTIONS Directions for (Qs. 27-31) : Each of these questions
contains an Assertion followed by Reason. Read them
Directions for (Qs. 24-26) : These questions consist carefully and answer the question on the basis of
of two statements, each printed as Assertion and following options. You have to select the one that
Reason. While answering these questions, you are best describes the two statements.
required to choose any one of the following five (a) If both Assertion and Reason are correct and
responses. Reason is the correct explanation of Assertion.
(a) If both Assertion and Reason are correct and (b) If both Assertion and Reason are correct, but
the Reason is a correct explanation of the Reason is not the correct explanation of
Assertion. Assertion.
(b) If both Assertion and Reason are correct but (c) If Assertion is correct but Reason is incorrect.
Reason is not a correct explanation of the (d) If both the Assertion and Reason are incorrect.
Assertion. 27. Assertion : The brain stem contains centres for
(c) If the Assertion is correct but Reason is controlling activities.
incorrect. Reason : Brain stem is very sensitive. [2012]
(d) If both the Assertion and Reason are incorrect. 28. Assertion : The chemical stored in the synaptic
(e) If the Assertion is incorrect but the Reason is vesicles are termed as neurotransmitters.
correct. Reason : Synaptic vesicles release these
chemicals in the synaptic cleft. [2013]
24. Assertion : Transmission of nerve impulse 29. Assertion: The imbalance in concentration of
across a synapse is accomplished by Na+, K+ and proteins generates resting potential.
neurotransmitters. Reason: To maintain the unequal distribution of
Reason : Transmission across a synapse usually Na+ & K+, the neurons use electrical energy.
requires neurotransmitters because there is a [2002, 2015]
small space, i.e., synaptic cleft, that separates 30. Assertion : The axonal membrane of the neuron
one neuron from another. [1999] is more permeable to sodium ion (Na+) and nearly
impermeable to potassium (K+).
25. Assertion : Tongue is a gustatoreceptor.
Reason : In a resting state neuron does not
Reason : Receptors for gustatory sensations are
conduct any impulse. [2016]
located in taste buds. [2000]
31. Assertion : A cerebellum is related with skillful
26. Assertion : Astigmatism is due to uneven voluntary movement and involuntary activity
curvature of lens. like body balance, equilibrium, etc.
Reason : It is treated with cylindrical Reason : It is part of hind brain and it is situated
lenses. [2007] behind the pons. [2010, 2017]
Neural Control and Coordination B-111

Type A : Multiple Choice Questions 11. (d) Transmission of nerve impulse is always
from axon of one neuron to the dendrite of
1. (a) Vagus is the 10th cranial nerve. Vagus is another neuron i.e. unidirectional because
the longest cranial nerve. It has maximum neurotransmitters are produced by axons
branches and also called as wandering and not by dendrites.
nerve.
12. (b) Cell A is the cone cell more concentrated in
2. (a) The sensation of stomach pain is due to the fovea centralis/yellow spot of the eye.
enteroceptors (visceroreceptors). Cone cells are sensitive to bright light hence
3 (c) Conditioned reflexes are acquired reflexes helps in differentiating colours and give
and dependent on past experiences, high resolution. These cells are specialized
training and learning I.R. Pavlov for colour vision.
demonstrated conditioned reflexes in a 13. (a) Borderline personality disorder is an
hungry dog. He called food and salivation emotionally unstable personality disorder
in response to it as unconditioned stimulus charactarised by impulsivity, unpredictable
and sound of bell and salivation in moods, outburst of emotion, behavioural
response to bell as conditioned reflexes. explosions, quarrelsome behaviour and
4. (d) Otorhinolaryngology is the study of ENT. conflicts with others.
5. (a) The withdrawl of leg in a decapitated frog 14. (c) Glaucoma is an eye disease characterized
when pin pointed is a type of simple-reflex. by increased occular pressure within the
6. (d) Plague is a bacterial disease of rat, caused eye ball. Glaucoma is a group of diseases
by Pasteurella pestis. Their vector is rat of the optic nerve involving loss of retinal
flea (Xenopsylla cheopsis). Bubonic plague ganglion cells in a characteristic pattern of
affects lymph nodes. Pneumonic plague optic neuropathy. Untreated glaucoma
affects lungs and septicemic plague causes leads to permanent damage of the optic
anaemia. nerve and resultant visual field loss that
7. (c) The 5th cranial nerve in frog is trigeminal. can progress to blindness.Measles is a
highly infectious viral disease that usually
8. (d) Lead zirconate is a key component of
spread by droplet infection. Bronchitis is
sonography. These crystals are housed in
the inflammation of the membrane lining of
a transducer which gets excited and starts
the bronchial tubes.
vibrating when an electric potential is
applied to it. These vibrations are the 15. (b) Sympathetic nervous system inhibits
source of ultrasound. salivary gland secretion, accelerate heart
rate, decreases intestinal peristalsis and
9. (d) Vagus nerve is a mixed nerve. It controls
dilate pupil of the eye. Whereas
the visceral sen sations and viscer al
parasympathetic nervous system
movements, e.g. respiratory movements.
stimulates salivary gland secretion, slows
10. (c) Concave lenses correct the eye condition heart rate, sitmulates intestinal peristalsis
of near sightedness i.e. myopia by bringing and constricts pupil of the eye. The
the light rays to a focus on retina. In such sympathetic and the parasympathetic
cases light rays converge at a point in front nervous system are parts of what is
of the retina. commonly called the autonomic nervous
EBD_7100
B-112 Topicwise AIIMS Solved Papers – BIOLOGY
system. These systems work in balance 18. (c) Bipolar nerve cells are present in retina of
with each other and directly or indirectly eye. Bipolar nerve cell posses one axon and
affect almost every structure in the body one dendron.
(e.g. heart frequency, heart capacity, lumbar 19. (c) Fenestra ovalis is membrane covered
function, kidneys, blood vessels, stomach opening leading from tympanic cavity into
and intestines). The sympathetic nervous the vestibule of the internal ear.
system has an active “pushing” function,
20. (b) Multipolar nerve cells are present in dorsal
the parasympathetic has mainly a relaxing
root ganglia of spinal cord.
function.
21. (c) Dendrites generally receive inputs and
Sympathetic Structure Parasympathetic conduct signals toward the cell body,
Rate increased Heart Rate decreased whereas axons conduct signals away from
Force increased Heart Force decreased the cell body.
Bronchial muscle Bronchial muscle 22. (a) 23. (a)
Lungs
relaxed contracted Type B : Assertion Reason Questions
Pupil dilation Eye Pupil constriction
Motility reduced Intestine Digestion increased 24. (a) Transmission of nerve impulse access
Sphincter closed Bladder Sphincter relaxed synapses is accomplishes by
neurotransmitter because synapses
Decreased urine Increased urine
Kidneys comprises of a synaptic cleft between the
secretion secretion
end of one nerve fibres and the beginning
16. (c) Forebrain is the largest part of the brain of the next.
consisting of two halves called cerebral
25. (a) Gustatoreceptors are chemoreceptors,
hemispheres separated by longitudinal
enclosed within taste buds.
fissures. Each cerebral hemisphere is
divided into four lobes-frontal lobe, parietal 26. (b) Astigmatism is a kind of defect of vision in
which the image of an object is distorted. It
lobe, temporal lobe and occipital lobe.
is because all the light rays do not come to
Temporal lobe has cells that bring to
focus on retina. It is due to abnormal
consciousness, the sensations of hearing
curvature of the lens . It can be corrected
and smell. The frontal lobe has centers that
are concerned with voluntary movements by wearing cylindrical lenses.
and personality. The parietal lobe is 27. (b) The brain stem consists of pons varoli,
concerned with general sensations like medulla oblongata, mid brain and
temperature, touch, pressure, pain, and diencephalon. The brain stem is the
proprioception. The occipital lobe has connection between brain and spinal cord.
centers of visual sense. It contains centres for controlling many
vital activities like respiration, body
17. (a) The inner layer of the posterior two-thirds temperature, urge for eating and drinking
of the eyeball consists of a light sensitive
etc. It also carries nerve tracts between the
layer, called retina that possesses two types
spinal cord and the higher brain structure.
of photoreceptors called the rods and the
28. (b) The axon terminal of the neuron contains
cone cells. Retina reduces the internal
many membrane bound vesicles called
reflection so any damage to it leads to
synaptic vesicles, in its cytoplasm. Within
greater internal reflection of light often
these vesicles, chemical substances such
causing an increase in light sensitivity.
as adrenaline and acetylcholine remain
Neural Control and Coordination B-113

stored. These chemicals are called nearly impermeable to sodium ions (Na+).
neurotransmitters, because they help to In a resting state neuron does not conduct
transmit nerve impulses across the any impulse. In the resting state the period
synapses. When a nerve impulse passes the during which a neuron is not conducting
axon terminal, its synaptic vesicles release the fluids outside the cell membrane carry
their stored chemicals to the synaptic cleft. a relatively high positive charge. The fluids
These diffuse through the cleft to reach the inside the cell membrane. Carry a less
membrane of the next neuron, stimulating positive, or relatively negative, charge.
the latter. This causes the nerve impulse to 31. (b) Hind brain consists of cerebellum located
be transmitted along the next neuron. dorsally to medulla oblongate and pons
29. (c) Resting potential is due to differential varolii. It contains centres for maintenance
distribution of ions on two sides of cell of posture and equilibrium of the body and
membrane. for the muscle tone. All activities of the
30. (d) The axonal membrane of neuron is more cerebellum are involuntary but may involve
permeable to potassium ions (K+ ) and learning in their early stages.
EBD_7100
B-114 Topicwise AIIMS Solved Papers – BIOLOGY

22 Chemical Co-ordination and Integration

TYPE A : MULTIPLE CHOICE QUESTIONS (a) a reduction in insulin secretion from


pancreas
1. To yield more milk, cow is injected with [1997] (b) a reduction in vasopressin secretion from
(a) sorbitol (b) prolactin posterior pituitary
(c) gonadotrophs (d) stilbesterol (c) a fall in the glucose concentration in urine
2. Treatment with alloxan destroys [1998] (d) an increase in secretion of glucagon
(a) stilt cells
10. The source of somatostatin is same as that of
(b) -cells of Langerhans
(a) Thyroxine and calcitonin [2003]
(c) sertoli cells
(b) Insulin and glucagon
(d) cells of Leydig
(c) Somatotropin and prolactin
3. Addison's disease result from [1998]
(a) hyposecretion of gonads (d) Vasopressin and oxytocin
(b) hyposecretion of adrenal gland 11. Which one of the following four secretions is
(c) hypertrophy of kidney correctly matched with its source, target and
(d) hyposecretion of pituitary gland nature of action? [2005]
4. The development of adult characteristics in a Se c re t io n So ur c e Target Act io n
moulting insect is promoted by [1998]
(a) Thyroxine (b) Ecdysone (a) Gastrin Stomach Oxyntic Production
lining cells of HCl
(c) Pheromone (d) None of these
5. Heart beat increases at the time of interview because (b) Inhibin Sertoli Hypotha - Inhibition of
of [1999] cells lamus secretion of
(a) renin (b) rennin gonadotropin
(c) adrenaline (d) Diuretic ADH releasing
hormone
6. Conn’s disease is caused by the over secretion
of [1999] (c) Entero- Duodenum Gall Release of
(a) ADH (b) ACTH kinase bladder bile juice
(c) Oxytocin (d) Aldosterone (d) Atrial Sinu atrial Juxta- Inhibition of
7. Acromegaly is due to hypersecretion of [2001] Natriuretic node (SAN) glomerular release of
(a) Insulin (b) Thyroxine Factor M-cells of apparatus renin
(c) Growth hormone (d) None of these (ANF) Atria (JGA)
8. Steroid hormones are similar in structure to
(a) Tryosine [2001] 12. Which of the following match is correct?[2007]
(b) Cholesterol Hormone Effect
(c) Coenzyme A (a) Oxytocin Milk ejection hormone
(d) Glycerol (b) Glucagon Decreases blood sugar
9. A person passes much urine and drinks much level
water put his blood glucose level normal. This (c) Adrenaline Decreases heart rate
condition may be the result of [2003]
(d) Thyroxine Decreases BMR
Chemical Co-ordination and Integration B-115

13. Which of the following statements regarding 17. Which one of the following hormones contains
glucagon is false? [2007] iodine? [2010]
(a) It is secreted by -cells of Langerhans. (a) Thyroxine (b) Testosterone
(b) It acts antagonistically to insulin. (c) Insulin (d) Adrenaline
(c) It decreases blood sugar level. 18. The pituitary gland by virtue of its tropic
(d) The gland responsible for its secretion is hormones controls the secretory activity of other
heterocrine gland. endocrine glands. Which one of the following
14. Which one of the following four gland is endocrine gland can function independently of
correctly matched with the accompanying the pituitary gland? [2010]
description? [2005, 2008] (a) Thyroid (b) Gonads
(a) Thyroid - Hyperactivity in young (c) Adrenals (d) Parathyroid
children causes cretinism 19. Match List-I (Endocrine glands) with List-II
(b) Thymus - Starts undergoing (Hormones secreted) and select the correct
atrophy after puberty answer using the codes given below [2010]
(c) Parathyroid - Secretes para-thormone, List- I List-II
which promotes A. Gonads I. Insulin
movement of calcium ions B. Pituitary II. Progesterone
from blood into bones C. Pancreas III. Growth
during calcification
hormones
(d) Pancreas - Delta cells of the 'islets of
D. Adrenal IV. Cortisone
Langerhans' secrete a
Codes :
hormone, which
stimulates glycolysis in (a) A – III; B – II; C – IV; D – I
liver (b) A – II; B – III; C – IV; D – I
15. Which row in the chart contains the words that (c) A – II; B – III; C – I; D – IV
best complete this statement? The (I) glands (d) A – III; B – II; C – I; D – IV
produce (II), which are transported by the (III) 20. Which gland is concerned with salt equilibrium
system. [2009] in body? [2012]
(a) Anterior pituitary (b) Pancreas
Row I II III (c) Adrenal (d) Thyroid
A digestive hormones circulatory 21. Which of th e following hormones have
B endocrine enzymes lymphatic antagonistic (opposing) effects? [2013]
C endocrine hormones circulatory (a) Thyroxine and calcitonin
D digestive enzymes lymphatic (b) Insulin and glucagon
(c) Growth hormone and epinephrine
(a) A (b) B (d) ACTH and glucocorticoids
(c) C (d) D 22. Select the corr ect option describing
16. The blood glucose level is commonly expressed gonadotropin activity in a normal pregnant
as [2010] female: [2014]
(a) mm. of Hg (a) High level of FSH and LH stimulate the
(b) milligram per deci litre thickening of endometrium.
(c) parts per million (b) High level of FSH and LH facilitate
(d) gram (mg/dl) per litre implantation of the embryo.
EBD_7100
B-116 Topicwise AIIMS Solved Papers – BIOLOGY
(c) High level of hCG stimulates the synthesis Reason : Anti-diuretic hormone (ADH) is
of estrogen and progesterone. secreted by the posterior lobe of pituitary gland.
(d) High level of hCG stimulates the thickening [2004]
of endometrium.
27. Assertion : Our body secretes adrenaline in
23. Which endocrine gland is called ‘the Throne of intense cold. [2006]
immunity’? [2016]
Reason : Adrenaline raises metabolic rate.
(a) Spleen (b) Thymus
Directions for (Qs. 28-31) : Each of these questions
(c) Pineal (d) Adrenal medulla
contains an Assertion followed by Reason. Read them
24. Which of the following hormone acts upon the
carefully and answer the question on the basis of
renal tubule and blood capillaries ? [2017] following options. You have to select the one that
(a) Glucagon (b) Aldosterone best describes the two statements.
(c) Vasopressin (d) Glucocorticoids
(a) If both Assertion and Reason are correct and
TYPE B : ASSERTION REASON QUESTIONS Reason is the correct explanation of Assertion.
(b) If both Assertion and Reason are correct, but
Directions for (Qs. 25-27) : These questions consist
Reason is not the correct explanation of
of two statements, each printed as Assertion and
Assertion.
Reason. While answering these questions, you are
(c) If Assertion is correct but Reason is incorrect.
required to choose any one of the following five
(d) If both the Assertion and Reason are incorrect.
responses.
28. Assertion : Mammary glands are apocrine
(a) If both Assertion and Reason are correct and
glands. [2009]
the Reason is a correct explanation of the
Assertion. Reason : The distal part containing secretory
(b) If both Assertion and Reason are correct but granules break down and leaves as a secretion.
Reason is not a correct explanation of the 29. Assertion : Hormone calcitonin has antagonistic
Assertion. effect to that of parathormone. [2009]
(c) If the Assertion is correct but Reason is Reason : Calcitonin decreases blood calcium
incorrect. level while parathormone increases blood
(d) If both the Assertion and Reason are incorrect. calcium level.
(e) If the Assertion is incorrect but the Reason is 30. Assertion : The person with diabetes insipidus
correct. feels thirsty.
25. Assertion: The regulation of RBC production is Reason : A person with diabetes insipidus
accomplished by FSH. suffers from excess secretion of vasopressin.
[2010]
Reason: Erythropoietin hormone circulates to
31. Assertion: Failure of secretion of hormone
red bone marrow where it increases stem cell
vasopressin causes diabetes mellitus in the
mitosis and speed up development of RBCs.
patient. [2011]
[2002]
Reason: Vasopressin increases the volume of
26. Assertion : Diabetes insipidus is marked by urine by increasing the reabsorption of water
excessive urination and too much thirst of water. from the urine.
Chemical Co-ordination and Integration B-117

Type A : Multiple Choice Questions 5. (c) Adrenaline (amine hormone) is secreted by


adrenal medulla on stimulation of
1. (b) Prolactin (also called leuteotrophic) sympathetic nervous system for meeting
hormone is associated with lactation an emergency or stress condition like fear
(secretion of milk from the mammary gland). injury accident etc. Hence, it also called as
Therefore, to yield more milk cow is injected emergency hormone. It increases blood
with prolactin hormone. Sorbitol is a pressure, respiration rate, sugar level in
sweetner found in some fruits (like apple, blood etc. It prepares the body to face stress
peers etc). Gonadotrophs a type of basophil at the time of interview by increasing heart
in the adenohypohysis (anterior pituitary beat. So it is also called stress hormone.
gland) whose granules secrete FSH Renin is proteolytic enzyme
(follicele-stimulating hormone) and LH synthesized,stored, and secreted by the
(luteinizing hormone). Stilbesterol is juxtaglomerular cells of the kidney. It plays
synthetic estrogen used in hormone a role in regulation of blood pressure by
therapy, as a post-coital contraceptive, and catalyzing the conversion of
as a growth-promoting agent for livestock. angiotensinogen to angiotensin I and II
2. (b) Alloxan treatment damages -cells of Islets which in turn stimulates the release
of langerhans which leads to aldosterone from adrenal gland. Rennin, a
hyperglycaemia and glycosuria. coagulating enzyme produced from the
3. (b) Addisons disease occurs due to stomach of human body, catalyzes the
coagulation of milk by converting milk
hyposecretion of both mineralocorticoids
protein, caesin into paracaesinate. ADH
(aldosterone) and glucocorticoids (cortisol)
(antidiuretic hormone) is secreted by the
from the layers of adrenal cortex called zona
posterior portion of the pituitary gland that
glomerulosa and zona fasciculata
constricts blood vessels raises blood
respectively. The disease is characterized
pressure, and reduces excretion of urine.
by excessive loss of Na+, Cl– and HCO3
6. (d) Conn's syndrome/aldosteronism is due to
increased K+ level in blood, weight loss, hypersecretion of aldosterone. Aldosetrone
muscle weakness, fatigue, low blood is secreted from zona glomerulosa layer of
pressure, and sometimes darkening of the adrenal cortex. The principle action of the
skin in both exposed and nonexposed parts aldosterone is retention of sodium. Conn’s
of the body. syndrome is characterized by rise in blood
4. (b) Ecdysone is a moulting hormone of insects. volume and blood pressure; muscular
Ecdysone is produced from prothoracic weakness; high sodium and low potassium
glan d that tr iggers moulting and level in the blood plasma resulting in kidney
metamorphosis. Thyroxine is thyroid damage with polyuria and tetany and
hormone that stimulates body metabolism metabolic disorder.
and helps regulate body growth and 7. (c) Acromegaly is due to hyposecretion of
development. Pheromone is a chemical growth/somatotrophic hormone in adults
screted by an animal that influences the after the closure of epiphysed plate at the
behavior or development of other members end of long bones. Growth hormone is
of the same species. secreted by artesian pituitary gland.
EBD_7100
B-118 Topicwise AIIMS Solved Papers – BIOLOGY
It is characterised by abnormal elongation Vasopressin and oxytocin are secreted from
of limbs and lower jaw giving gorilla like posterior pituitary gland.
appearance. Hypersecretion of insulin 11. (d) Atrial natriuretic factor (ANF) is produced
results in hypoglycemia, hunger, sweating by cardiocytes of atria of heart in response
and double vision. Hypersecretion of to an increased return of the deoxygenated
thyroxine results in Grave's disease (also blood. It inhibits the release of renin from
called exopthalmic goiter) which is juxta-glomerular apparatus and thereby,
characterised by increased BMR, heart rate, inhibits NaCl reabsorption by the collecting
pulse rate; protrusion of eyes etc. duct and reduces aldosterone release from
8. (b) Steroid hormones are fat soluble and have the adrenal cortex. Inhibin is an endocrine
sterol group. They are derived from hormone, produced from ovary and testes.
cholesterol, e.g. hormones of adrenal When inhibin is secreted, it inhibits the
cortex, testis and ovaries. Amine hormones production of follicle stimulating hormone
are derived form tyrosine amino acids and (FSH). It also limits the release of
have amino group, e.g. thyroxin e, gonadotropin releasing hormone. (For
epinephrine and nor epinephrine. other hormones refer answer no. 4)
Coenzyme A is essential to metabolism of 12. (a) Oxytocin is the hormone secreted by
carbohydrates and fats and some amino posterior pituitary that causes contraction
acids. Glycerol is a clear, colorless, viscous, of the smooth muscles of myometrium
sweet-tasting liquid organic compound of during child birth and ejection of milk from
the alcohol family. the mammary glands. Glucagon is secreted
9. (b) When a person passes more urine and by the -cells of islets of langerhans of
drinks more water, he is suffering from pancreas. Its main fuction is to increase
diabetes insipidus. It is caused by blood glucose level. Adrenaline
reduction in vasopressin (ADH) secretion (epinephrine) is secreted by adrenal
from post pituitary gland. medulla. It increases the rate and force of
10. (b) The source of somatostatin is same as that heart beat. Thyroid gland secretes
of insulin and glucagons. All these thyroxine that regulates basal metabolic
hormones are secreted from endocrine part rate.
(called Islets of Langerhans) of pancreas. 13. (c) Pancreas is a heterocrine gland comprising
These endocrine parts contain different both endocrine and exocrine parts. Its
types of endorcrine cells which secretes endocrine part consists of small masses of
different hormones. hormone secreting cells called islets of
langerhans. The -cells of latter secrete
Endocrine cells Hormones
glucagons and its -cells secrete insulin.
cell Glucagon These two hormones have antagonistic
cell Insulin effects on the glucose level in the blood
cell Gastrin which means that insulin decreases the
blood glucose level while glucagon
cell Somatostatin increases blood glucose level.
F cell Pancreatic polypeptides 14. (b) The thymus is an organ located in the upper
anterior portion of the chest cavity just
Thyroxine and calcitonin are secreted by
behind the sternum. The thymus continues
thyroid gland. Somatotropin and prolactin
to grow between birth and puberty and then
are secreted from anterior pituitary gland.
begins to atrophy. Proportional to thymic
Chemical Co-ordination and Integration B-119

size, thymic activity is most active before 24. (c) ADH (or vasopressin) is secreted by
puberty. Upon atrophy, the size and activity posterior pituitary gland. It acts on kidney
are dramatically reduced, and the organ is tubule and blood capillaries and
primarily replaced with fat. The atrophy is concentrates the urine by promoting the
due to the increased circulating level of sex reabsorption of water and salts into the
hormones, and chemical or physical cortical collecting ducts.
castration of an adult result in the thymus
increasing in size and activity. Type B : Assertion Reason Questions
15. (c) The endocrine glands produce hormones,
which are transported by the circulatory 25. (e) The regulation of RBC production is
system. The digestive system makes accomplished by erythropoietin hormone
enzymes that are secreted via ducts to the (EPO). Erythropoietin, a glycoprotein, is
organ that needs them. produced by the kidney when the oxygen
16. (b) Blood glucose level is commonly expressed level of low EPO then stimulates the bone
as milligram per deci litre. marrow to produce more red cells and
17. (a) The main secretion of thyroid gland is thereby increase the oxygen-carrying
called thyroxine. Thyroxine contains iodine. capacity of the blood. Follicle-stimulating
When thyroid gland becomes inactive, the hormone (FSH) is a gonadotropic hormone
lack of iodine causes goitre. that is secreted by the anterior pituitary
18. (d) Parathyroid gland secretes parathormone gland. FSH causes gametogenesis and
hormone, which regulates Ca++ and PO 24 stimulates estrogen production from
ovaries.
ion in body. This gland works
independently to pituitary gland. 26. (b) Diabetes insipidus (DI) occurs when the
19. (c) Gonads, pituitary, pancreas & adrenal are kidneys are unable to conserve water as
all endocrine glands which secretes they perform their function of filtering
progresterone, growth hormone, insulin and blood. The amount of water conserved is
coltisone hormone respectively. controlled by antidiuretic hormone (ADH)
20. (c) Adrenal glands are concerned with salt also called vasopressin which is secreted
equilibrium in the body. Mineralorcoticoids by posterior lobe of pituitary gland.
secreted from adrenal cortex. Aldosterone Diabetes inscipidus is characterised by
is the main mineralocorticoid in our body. excessive urination and thirst. This
Aldosterone acts mainly at the renal problem appears due to the increase in
tubules and stimulates reabsorbtion of Na+ permeability of collecting tubules.
an d water an d excretion of K + and
27. (a) Adrenaline is an emergency hormone
phosphate ions.
whose concentration increases under
21. (b) Insulin lowers blood sugar levels. While
stress conditions. e.g. cold, stress.
Glucagon raises blood sugar levels.
Adrenaline is secreted from adrenal
22. (c) Synthesis of estrogen and progesterone
medulla. It initiates many bodily responses,
due to high level of hCG is a normal
including the stimulation of heart action
gonadotropic activity in a normal pregnant
female. and an increase in blood pressure,
metabolic rate, and blood glucose
23. (b) Thymus stimulates T-cells which regulate
the production of antibodies. concentration.
EBD_7100
B-120 Topicwise AIIMS Solved Papers – BIOLOGY
28. (a) Based on the mode of secretion, the glands calcitonin hormone increases excretion of
are of three types : mesocrine, apocrine and calcium in urine.
holocrine. Mammary glands that are present 30. (c) Vasopressin or antidiuretic hormone is
in mammals to feed the young ones with secreted by posterior pituitary gland. The
milk are the example of apocrine glands. In deficiency of vassopressin results in a
apocrine glands, the secretion accumulates disorder known as diabetes insipidus. The
as secretory granules in the distal part of main symptoms of diabetes insipidus are
the cell. This part later breaks down and increase in thirst and increase in urination.
leaves as a secretion. 31. (d) Vasopressin or anti-diuretic hormone
29. (a) Calcitonin or thyrocalcitonin is secreted by (ADH) reduces the volume of urine by
parafollicular cells of thyroid stroma. It increasing the reabsorption of water from
retards bone dissolution and stimulates the urine in the distal convoluted tubules,
excretion of calcium in urine. Thus, it lowers collecting tubules and collecting ducts in
calcium level in extra cellular fluid (ECF). the kidney. It does so by rendering the walls
Parathormone is secreted by chief cells of of these tubules leads to diabetes insipidus
parathyroid gland and is also known as
(increased urination). Although the volume
Collip's hormone. It maintains blood calcium
of urine is increased. No glucose appears in
level by increasing its absorption from food
the urine of such patients.
in intestine and its reabsorption from
Diabetes mellitus is a disease which is
nephrons in the kidney. Maintenance of
caused due to the failure of insulin hormone
proper calcium level is in fact, a combined
secretion by the pancreatic islets. The
function of parathormone and calcitonin.
When calcium level falls below normal osmotic effect of glucose in the urine
parathormone maintains it by promoting its considerably increases the volume of urine,
absorption, reabsorption and also by due to which thirst is also enhanced. In
demineralisation of bones. When blood extreme cases, the patient suffers from coma
calcium level exceeds above normal then and may die.
23 Reproduction in Organisms

4. Identify the events (A, B, D and E) in life of


TYPE A : MULTIPLE CHOICE QUESTIONS
general reproduction-
1. Based on cellular mechanisms there are two major
types of regeneration found in the animals.
Which one of the following is the correct
example of the type mentioned? [2005]
(a) Morphallaxis - Regeneration of two
transversely cut equal pieces of a Hydra [2015]
into two small Hydras. (a) A-Gamete transfer, B-Gametogeneis,
D-Zygote formation, E-Embryogenesis
(b) Epimorphosis - Replacement of old and
dead erythrocytes by the new ones. (b) A-Gametogeneis, B-Gamete transfer,
D-Zygote formation, E-Embryogenesis
(c) Morphallaxis - Healing up of a wound in
(c) A-Gametogeneis, B-Zygote formation,
the skin.
D-Gamete transfer, E-Embryogenesis
(d) Epimorphosis - Regeneration of crushed
(d) A-Gametogeneis, B-Gamete transfer,
and filtered out pieces of a Planaria into as
D-Embryogenesis, E-Zygote formation.
many new Planarians. from chapter 25
5. Which of the following statements is incorrect ?
2. Which form of reproduction is correctly
(i) Bamboo species flower only once in their
matched? [2007]
life time, generally after 50-100 years and
(a) Euglena transverse binary fission
produce large number of fruits and die.
(b) Paramecium longitudinal binary
(ii) In animals, the juvenile phase is followed
fission
by morphological and physiological
(c) Amoeba multiple fission
changes prior to active reproductive
(d) Plasmodium binary fission
behaviour.
3. Which reproductive adaptation is characteristic
(iii) The reproductive phase is of same duration
of most terrestrial vertebrates but not of most
in all organisms.
aquatic vertebrates? [2009]
(iv) Juvenile phase is the period of growth
(a) External fertilization
between the birth of an individual till it
(b) Internal fertilization
reaches reproductive maturity. [2016]
(c) Motile gametes
(a) Only (i) (b) Only (ii)
(d) External development
(c) Only (iii) (d) Only (iv)
EBD_7100
B-122 Topicwise AIIMS Solved Papers – BIOLOGY

6. In a practical test, a student has to identify the carefully and answer the question on the basis of
organisms in which syngamy does not occur. In following options. You have to select the one that
those organisms the female gamete undergoes best describes the two statements.
development to form new organisms without (a) If both Assertion and Reason are correct and
fertilization. This phenomenon is called "X". Reason is the correct explanation of Assertion.
Identify the organisms and the phenomenon (b) If both Assertion and Reason are correct, but
"X". [2017] Reason is not the correct explanation of
(a) Frog, Parthenogenesis Assertion.
(b) Lizards, Gametogenesis (c) If Assertion is correct but Reason is incorrect.
(c) Rotifers, Embryogenesis (d) If both the Assertion and Reason are incorrect.
(d) Honeybee, Parthenogenesis 7. Assertion : Leaves of Bryophyllum, Begonia

TYPE B : ASSERTION REASON QUESTIONS help in vegetative multiplication.


Reason : Leaves of these plants possess
Directions for (Q. 7) : Each of these questions
adventitious buds. [2014]
contains an Assertion followed by Reason. Read them
Reproduction in Organisms B-123

Type A : Multiple Choice Questions sperm directly into the female body. This is
advantageous since sperm need liquid to
1. (a) Morphallaxis is the production of an entire swim and the moist female reproductive
animal from a small fragment whereas tract provides this. Aquatic vertebrate
epimorphosis is the replacement of the lost animals live in water so sperm can easily
part. be deposited in the water and swim to the
female reproductive tract.
2. (c) Reproduction is the production of a new
4. (b) Gametogenesis leads to production of
generation of individuals of the same
gametes (sperm and ovum). Male gametes
species. It involves transfer of genetic are then transferred to the site of
material from one generation to the next. fertilization. Fertilization results in zygote
Asexual and sexual are the two types of formation. The zygote then gives rise to
reproduction. Fission and budding are two embryo.
most common forms of asexual 5. (c) Statement (iii) is not correct. The
reproduction in animals. During adverse reproductive phase is not of same duration
conditions, amoeba reproduces by multiple in all organisms.
fission that gives rise to many amoeba. 6. (d) Parthenogenesis is a form of reproduction
Euglena reproduces by longitudinal binary in which an unfertilized egg develops into
fission, Paramecium reproduces by a new individual, occurring commonly
transverse binary fission and Plasmodium among insects and certain other
reproduces by multiple fission. arthropods.

3. (b) Most vertebrate animals that live on land Type B : Assertion Reason Questions
have an adaptation that allows internal
fertilization, whereby the male penis inserts 7. (a)
EBD_7100
B-124 Topicwise AIIMS Solved Papers – BIOLOGY

24
Sexual Reproduction in
Flowering Plants
9. Xenia refers to [2002]
TYPE A : MULTIPLE CHOICE QUESTIONS
(a) effect of pollen on endosperm
1. Asexual reproduction is called as [1997] (b) effect of pollen on stems
(a) apomixis (b) fragmentation (c) effect of pollen on taste of fruits
(c) self fertilization (d) cross fertilization (d) effect of pollen on vascular tissue
2. Pollination by snail and slug is called as 10. Ploidy of ovum of angiosperms is [2002]
[1998] (a) haploid (b) diploid
(a) entomophilous (c) triploid (d) polyploid
(b) malacophilous 11. Pollen grains are able to withstand extremes of
(c) ornithophilous temperature and dessication because their exine
(d) chiropterophilous is composed of [2003]
3. In angiosperm, the endosperm is [1998] (a) cutin (b) suberin
(a) diploid (b) triploid (c) sporopollenin (d) callose
(c) haploid (d) polyploid 12. The pollen tube usually enters the embryo sac
4. Female gametophyte of angiosperm is [1999] (a) between the egg cell and synergid [2004]
(b) by directly penetrating the egg
(a) 7 celled (b) 8 celled
(c) between one synergid and antipodal cell
(c) 11 celled (d) 5 celled
(d) by knocking off the antipodal cells
5. Anemophilous flowers have [1999]
13. Double fertilization involves [2005]
(a) sessile stigma
(a) fertilization of egg by two male gametes
(b) small, smooth stigma (b) fertilization of two eggs in the same embryo
(c) coloured and scented flowers sac by two sperms brought by one pollen tube
(d) large feathery stigma (c) fertilization of the egg and the central cell
6. Growth of pollen tube towards embryo is [2000] by two sperms brought by different pollen tubes
(a) geotropism (b) chemotaxis (d) fertilization of the egg and the central cell
(c) phototaxis (d) thigmotaxis by two sperms brought by the same pollen tube
7. Which of the following statement is true? 14. In which one of the following combinations
[2000] (a - d) the number of chromosomes of the present
(a) Spores are gametes day hexaploid wheat is correctly represented?
(b) Spores and gametes are diploid [2006]
(c) Gametes are always haploid Comb- Mono- Haploid Nulli- Tri-
(d) Spores are always diploid ination somic somic somic
8. Which part of embryo comes out first during (a) 21 28 42 43
seed germination ? [2001] (b) 7 28 40 42
(c) 21 7 42 43
(a) Radicle (b) Plumule
(d) 41 21 40 43
(c) Hypocotyl (d) Epicotyl
Sexual Reproduction in Flowering Plants B-125

15. Apomixis is [2007] 21. Which of the following statement is correct?


(a) formation of seeds by fusion of gametes. [2016]
(b) formation of seeds without syngamy and (a) Sporopollenin can withstand high
meiosis. temperatures but not strong acids.
(c) formation of seeds with syngamy but no (b) Sporopollenin can be degraded by
meiosis. enzymes.
(d) None of the above (c) Sporopollenin is made up of inorganic
materials.
16. The plant part which consists of two
(d) Sporopollenin can withstand high
generations one within the other, is [2008]
temperature as well as strong acids and
(a) germinated pollen grain
alkalis.
(b) embryo
(c) unfertilized ovule TYPE B : ASSERTION REASON QUESTIONS
(d) seed Directions for (Qs. 22-24) : These questions consist
17. Chasmogamy refers to the condition where of two statements, each printed as Assertion and
[2012] Reason. While answering these questions, you are
(a) Flowers remains closed required to choose any one of the following five
(b) Flowers are absent responses.
(c) Flowers are open (a) If both Assertion and Reason are correct and
(d) Flower are gamopetalous the Reason is a correct explanation of the
18. What is common between vegetative Assertion.
reproduction and apomixis? [2013] (b) If both Assertion and Reason are correct but
(a) Both are applicable to only dicot plants Reason is not a correct explanation of the
(b) Both bypass the flowering phase Assertion.
(c) Both occur round the year (c) If the Assertion is correct but Reason is
(d) Both produces progeny identical to the incorrect.
parent (d) If both the Assertion and Reason are incorrect.
19. Emasculation is not required when flowers are (e) If the Assertion is incorrect but the Reason is
[2013] correct.
(a) bisexual (b) intersexual 22. Assertion : If pollen mother cells has 42
(c) unisexual (d) either (a) or (b) chromosomes, the pollen has only 21
20. Geitonogamy involves: chromosomes.
(a) fertilization of a flower by the pollen from Reason : Pollens are formed after meiosis in
another flower of the same plant. pollen mother cell. [1997]
(b) fertilization of a flower by the pollen from 23. Assertion: The megaspore mother cell divide
the same flower. mitotically to produce four spores.
(c) fertilization of a flower by the pollen from a Reason: Megaspore mother cells are diploid and
flower of another plant in the same megaspore is haploid. [2002]
population. 24. Assertion : Insects visit flower to gather honey.
(d) fertilization of a flower by the pollen from a Reason : Attraction of flowers prevents the
flower of another plant belonging to a insects from damaging other parts of the plant.
distant population. [2014] [2004]
EBD_7100
B-126 Topicwise AIIMS Solved Papers – BIOLOGY
Directions for (Qs. 25-28) : Each of these questions 26. Assertion : Chasmogamous flowers require
contains an Assertion followed by Reason. Read them pollinating agents.
carefully and answer the question on the basis of Reason : Cleistogamous flowers do not expose
following options. You have to select the one that their sex organs. [2012]
best describes the two statements.
27. Assertion : Double fertilization is characteristic
(a) If both Assertion and Reason are correct and feature of angiospersms.
Reason is the correct explanation of Assertion.
Reason : Double fertilization involves two
(b) If both Assertion and Reason are correct, but
fusions. [2016]
Reason is not the correct explanation of
Assertion. 28. Assertion : Endosperm is a nutritive tissue and
(c) If Assertion is correct but Reason is incorrect. it is triploid.
(d) If both the Assertion and Reason are incorrect. Reason: Endosperm is formed by fusion of
25. Assertion : Pollen mother cells (PMCs) are the secondary nucleus to second male gamete. It is
first male gametophytic cells. [2009] used by developing embryo. [1998, 2017]
Reason : Each PMC gives rise to two pollens.
Sexual Reproduction in Flowering Plants B-127

Type A : Multiple Choice Questions 8. (a) The radicle comes out first since it grows
towards the earth. During seed germination
1. (c) In asexual reproduction, single parent is the radicle comes out first due to
involved. It usually includes amitosis or gravitational force and further more it
mitotic division. results in a differential growth.
2. (b) Pollination by insects is entomophily, 9. (a) Xenia is the effect of pollen genes on the
pollination by birds is ornithophily,
development of the fruit or seed.
pollination by bats is chiropterophily and
10. (a) Ovum is a female gamete and is always
pollination by molluscs (snow, slugs) is
haploid.
malacophily.
11. (c) Sporopollenin, which is the hardest
3. (b) Endosperm is formed as a result of triple
fusion male gamete (n) + seconday nucleus substance, helps the pollen grains to
(2n) = Primary endosperm nucleus (3n) withstand extremes of temperatures. It
Secondary nucleus is formed by the fusion avoids transpiration or water loss. This
of 2 polar nuclei. hard proteinaceous substance present in
4. (a) The female gametophyte of angiosperms the exine makes it also spinous in nature.
is eight nucleated and seven celled. The 12. (a) The synergids direct the growth of pollen
organized embryo sac comprises a 3 celled tube by secreting some chemical
egg apparatus, three antipodal cells and a substances. The tip of pollen tube enters
bipolar central cell. The embryo sac into one synergid.
although eight nucleated has only seven
13. (d) Double fertilization involves fertilization of
cells.
the egg/oosphere (2n) and that of
5. (d) Anemophilous flowers have feathery
secondary nucleus (3N) by two different
stigma. It is the characteristic feature of
sperms produced in the same pollen tube.
Gramineae family (grass). In grasses, the
stigma, that is plumose, works as an 14. (d) 1n = 21; monosomic (2n – 1) = 42 – 1 = 41;
efficient pollen catcher. Hence, nullisomic (2n – 2) = 42 – 2 = 40. Trisomic
anemophilous flowers have feathery or (2n + 1) = 42 +1 = 43
plumose stigma. 15. (b) In plants, normal sexual reproduction
6. (b) Growth of pollen tube towards embryo is includes meiosis and fertilization . It is
chemotaxis due to the stimulus being called amphimixis. But in some plants
chemical in nature. The chemical stimulus abnormal sexual reproduction called
is supplied in the form of Ca++ ions. apomixis has been observed. Apomixis
7. (c) Gametes are always haploid in order to includes abnormal sexual reproduction in
preserve the species genetically, anatomically which egg or other cells like synergids and
and morphologically also. The embryo or antipodals develop into embryo without
zygote is formed due to the union of male fertilization and meiosis. The term apomixis
and female gametes. (n + n = 2n). Hence, any was given be Winkler (1908) eg. Citrus,
species which is 2n is diploid in nature. Rannunculus.
EBD_7100
B-128 Topicwise AIIMS Solved Papers – BIOLOGY

16. (b) The plant which consists of two Type B : Assertion Reason Questions
generations one within the other is embryo.
In botany, a seed plant embryo is part of a 22. (a) Pollen mother cells undergo meiosis and
produce pollen grains. The pollen grains
seed, consisting of precursor tissues for
have haploid number of chromosomes.
the leaves, stem and root as well as one or
23. (e) Megaspore mother cell is a prominent cell
more cotyledons. Once the embryo begins
in the nucellus. It divides by meiosis and
to germinate, grow out from the seed, it is forms a row of four haploid megaspores.
called a seedling. Plants that do not produce
24. (d) Honey bee visit flowers to gather nectar
seeds, but do produce an embryo, include and turn it into honey. Visiting of insects
the bryophytes and ferns. In these plants, for nectar helps in pollination.
the embryo is a young plant that grows 25. (d) Primary sporogenous cell gives rise to
attached to a parental gametophyte. microspore mother cells or pollen mother
17. (c) Chasmogamous flowers are always open. cells (PMCs). They are sporophytic in nature
In same bisexual plants like Commelina & i.e., diploid. These cells undergo meiosis
Viola. Chasmogamous and cleistogamous (reduction division) which gives rise to 4
microspores or pollens and this formation
flowers (which never opens throughout the
of microspores or pollens is called
life) are found. microsporo-genesis. Microspores represent
18. (d) Vegetative reproduction and apomixis both the beginning of the gametophytic phase
are asexual methods of reproduction, and they are haploid in nature.
which gives the progeny genetically similar 26. (b) The majority of angiosperms bear
to parent. chasmogamous flowers, which means the
flowers expose their mature anthers and
19. (c) In unisexual flowers, the female flower buds
stigma to the pollinating agents. There is
are bagged before the flowers open. When
another group of plants which set seeds
the stigma become receptive, pollination is without exposing their sex organs. Such
carried out using the desired pollen and flowers are called cleistogamous and the
the flower rebagged. Hence, there is no phenomenon is cleistogamy.
need of emasculation in these flowers. 27. (b) Double fertilization is a characteristic
20. (a) Geitonogamy is the transfer of pollen grains feature of angiosperms. It involves two
in different flowers of same plant. fusions in which one female gametes fuse
with egg cell to form zygote and other male
21. (d) Pollen grains are generally spherical and
gamete fuses with the diploid secondary
prominent two-layered wall. The hard outer nucleus to produce triploid primary
layer (called the exine) is made up of endosperm nucleus.
sporopollenin which is one of the most 28. (a) Male gamete (n) + secondary nucleus (2n)
resistant organic material known. It can = primar y endosperm nucleus which
withstand high temperatures and strong develops into endosperm (3n)
acids and alkali. Endosperm is the reserve food used by
developing embryo.
25 Human Reproduction

TYPE A : MULTIPLE CHOICE QUESTIONS 10. Which one of the following statements with
regard to embryonic development in humans is
1. Prostate gland secretion helps in formation of correct? [2003]
(a) larva (b) semen [1997]
(a) Cleavage division s bring about
(b) cocoon (c) none of these considerable increase in the mass of
2. Immediately after ovulation, the mammalian egg protoplasm.
is covered by a membrane called as [1998]
(b) In the second cleavage division, one of the
(a) chorion (b) corona radiata
two blastomeres usually divides a little
(c) zona pellucida (d) none of these
sooner than the second.
3. The extra-embryonic membranes of mammalian
(c) With more cleavage divisions, the resultant
embryo are derived from [1999]
blastomeres become larger and larger.
(a) trophoblast (b) follicle cells
(c) inner cell mass (d) formative cells (d) Cleavage division results in a hollow ball
of cells called morula.
4. Acrosome of sperm is formed by [1999]
(a) nucleus (b) golgi bodies 11. Women who consumed the drug thalidomide
(c) lysosome (d) E. R. for relief from vomiting during early months of
5. Cumulus covers [1999] pregnancy gave birth to children with [2004]
(a) ovary (b) ovum (a) no spleen
(c) embryo (d) sperm (b) hare-lip
6. Cessation of menstrual cycle in women is called (c) extra fingers and toes
(a) menopause (b) lactation [2001] (d) under developed limbs
(c) ovulation (d) parturition 12. A cross section at the midpoint of the middle
7. Both corpus luteum and macula lutea are [2003] piece of a human sperm will show [2005]
(a) found in human ovaries (a) centriole, mitochondria and 9 + 2
(b) a source of hormones arrangement of microtubules.
(c) characterized by a yellow colour (b) centriole and mitochondria.
(d) contributory in maintaining pregnancy (c) mitochondria and 9 + 2 arrangement of
microtubules.
8. The early human embryo distinctly possesses
(d) 9 + 2 arrangement of microtubules only.
(a) gills [2003]
13. Which one of the following events is correctly
(b) gill slits
matched with the time period in a normal
(c) external ear (pinna) menstrual cycle? [2005]
(d) eyebrows (a) Release of egg : 5th day
9. The phase of menstrual cycle in humans that (b) Endometrium regenerates : 5-10 days
lasts for 7-8 days, is [2003] (c) Endometrium secretes nutrients for
(a) follicular phase (b) ovulatory phase implantation : 11-18 days
(c) luteal phase (d) menstruation (d) Rise in progesterone level : 1-15 days
EBD_7100
B-130 Topicwise AIIMS Solved Papers – BIOLOGY
14. Which of the following is true regarding 22. All of the following statements concerning
sperm? [2007] pregnancy are accurate EXCEPT [2015]
(a) Fertilizin: For penetrating egg membrane
(a) the detection of human chorionic
(b) Hyalurodinase: For penetrating egg
gonadotropin in the urine forms the basis
membrane
(c) Acrosin: Dissolves corona radiata for pregnancy tests.
(d) Capacitation: Takes place in penis (b) the cyclic release of pituitary
15. Both corpus luteum and macula lutea are [2008] gonadotropins and ovarian steroids is
(a) found in human ovaries continued.
(b) a source of hormones (c) the mammary gland tissue of the pregnant
(c) characterized by a yellow colour woman is stimulated to develop by
(d) contributory in maintaining pregnancy placental hormones.
16. In humans, what is the ratio of the number of (d) the corpus luteum of pregnancy maintains
gametes produced from one male primary sex the uterus until the placenta is well
cell to the number of gametes produced from
established.
one female primary sex cell? [2009]
(a) 1:3 (b) 1:4 23. The following graph of relative concentrations
(c) 3:1 (d) 4:1 of the four hormones present in the blood plasma
17. Corpus luteum is a mass of cells found in[2010] of a woman during her menstrual cycle. Identify
(a) brain (b) ovary the hormones. [2015]
(c) pancreas (d) spleen
18. Cells of leydig are found in [1997, 2011]
increasing concentration

(a) Testes of frog (b) Testes of rabbit C D


(c) Kidney of frog (d) Kidney of rabbit
of hormone

B
19. Meroblastic cleavage refers to which type of A
division of egg [2001, 2011]
(a) Complete (b) Spiral
(c) Incomplete (d) Horizontal day
20. Which of the following organ is differentiated
first during development? [2012] A B C D
(a) Heart (b) Skin
(a) FSH Progesterone LH Oestrogen
(c) Brain (d) Neural tube
(b) LH Progesterone FSH Oestrogen
21. The correct sequence of spermatogenetic stages
(c) FSH Oestrogen LH Progesterone
leading to the formation of sperms in a mature
human testis is: [2013] (d) LH Oestrogen FSH Progesterone
(a) spermatogonia-spermatid-spermatocyte- 24. Ejaculation of human male contains about
sperms 200 – 300 million sperms, of which for normal
(b) spermatocyte-spermatogonia-spermatid- fertility ____ % sperms must have normal shape
sperms and size and at least ____% must show
(c) spermatogonia-spermatocyte-spermatid- energetic motility. [2016]
sperms
(a) 40, 60 (b) 50, 50
(d) spermatid-spermatocyte-spermatogonia-
sperms (c) 60, 40 (d) 30, 70
Human Reproduction B-131

25. The given figure shows the human foetus within (c) C: Grafian follicle, mature follicle which
the uterus with few structures marked as A, B, C ruptures to release secondary oocyte.
and D. (d) D: Tertiary follicle, a large number of this
B follicle degenerates during the phase from
birth to puberty.
Cavity
of uterus TYPE B : ASSERTION REASON QUESTIONS
A C
Embryo Directions for (Qs. 27-31) : These questions consist
of two statements, each printed as Assertion and
Reason. While answering these questions, you are
required to choose any one of the following five
D responses.
Which of the following options shows the (a) If both Assertion and Reason are correct and
correct labeling? [2016] the Reason is a correct explanation of the
(a) A Umbilical cord with its veins, B Assertion.
Chorionic villi, C Antrum, D Plug of (b) If both Assertion and Reason are correct but
mucus in cervix Reason is not a correct explanation of the
(b) A Umbilical cord with its vessels, Assertion.
B Fimbriae, C Oocyte, D Plug of (c) If the Assertion is correct but Reason is
mucus in vagina incorrect.
(c) A Umbilical cord with its vessels, (d) If both the Assertion and Reason are incorrect.
B Placental villi, C Yolk sac, D Plug (e) If the Assertion is incorrect but the Reason is
of mucus in cervix correct.
(d) A Umbilical cord with its veins, 27. Assertion : During fertilization only head of
B Placental villi, C Trophoblast, spermatozoa enters egg.
D Plug of mucus in vagina [2016] Reason : If several spermatozoa hit the egg at
26. The figure given below shows the sectional same time, all can enter the egg. [1997]
view of ovary. Select the option which gives 28. Assertion : In morula stage, cells divide without
correct identification of marked structure (A to D) increase in size.
and its feature. [2017] Reason : Zona pellucida remains undivided till
A B cleavage is complete. [1997]
29. Assertion : Death is one of the important
C
regulatory process on earth.
Reason : It avoids over-crowding caused by
continuous reproduction. [2002]
30. Assertion : Old age is not an illness. It is a
[2
]10
7 continuation of life with decreasing capacity for
adaptation.
Reason : Cessation of mitosis is a normal
D
genetically programmed event. [2003]
(a) A: Primary follicle, it is also called gamete 31. Assertion : Senescence is the time when age
mother cell. associated defects are manifested. [2005]
(b) B: Corpus luteum, it cannot be formed and Reason : Certain genes may be undergoing
added after birth. sequential switching on and off during one's
life.
EBD_7100
B-132 Topicwise AIIMS Solved Papers – BIOLOGY
Directions for (Qs. 32-38) : Each of these questions Reason : The eggs of mammal are microlecithal
contains an Assertion followed by Reason. Read them and telolecithal.
carefully and answer the question on the basis of 35. Assertion : Head of sperm consists of acrosome
following options. You have to select the one that and mitochondria.
best describes the two statements. Reason : Acrosome contains spiral row of
(a) If both Assertion and Reason are correct and mitochondria. [2014]
Reason is the correct explanation of Assertion. 36. Assertion : Females have less stature than males
(b) If both Assertion and Reason are correct, but after puberty.
Reason is not the correct explanation of Reason : This happens because of the presence
Assertion. of hCG in the blood of females. [2014]
(c) If Assertion is correct but Reason is incorrect. 37. Assertion : Testicular lobules are the
(d) If both the Assertion and Reason are incorrect. compartments present in testis.
32. Assertion : Corpus luteum degenerates in the Reason : These lobules are involved in the
absence of fertilization. [2009] process of fertilization. [2016]
Reason : Progesterone level decreases. 38. Assertion : Interstitial cell is present in the region
33. Assertion : Clitoris is not remnant of penis in outside the seminiferous tubule called interstitial
females. [2009] spaces.
Reason : It also has high blood supply and Reason : Interstitial cells provide nutrition to
erectile tissue. the sertoli cells. [2016, 2017]
34. Assertion : Mammalian ova produces
hyaluronidase. [2009]
Human Reproduction B-133

Type A : Multiple Choice Questions The period of menopause is between 45-55


years.
1. (b) The prostate gland is a chestnut shaped
7. (c) Corpus luteum is the fluid filled yellow
gland which lies at the base of bladder and
body in the ovary and macula lutea is the
surrounds the first part of the urethra. This
yellow spot present in the eyes.
gland secretes a slightly alkaline fluid
8. (c)
which forms the important component of
9. (b) In menstrual cycle, menstrual phase lasts
semen. This fluid constitutes the major
for 4 days, proliferating/ovulating phase
portion of seminal fluid which carries sperm
for about 10 days and secretory phase for
and sperms move freely in this fluid.
14 days.
Prostate gland secretion contains lipids,
10. (a) Repeated cleavage in the zygote brings
small amount of citric acid, HCO 3 ions about the distribution of the cytoplasm of
and a few enzymes. They activate and the zygote among blastomere and
provides nutrition to sperms and neutralise increases mobility of the protoplasm which
the acidity of urine which may kill the facilitates morphogenetic movements for
sperms. They form about 30% part of cell differentiation.
semen. 11. (d) Woman who took the drug thalidomide in
2. (d) Immediately after ovulation, mammalian early pregnancy gave birth to children with
eggs are covered by vitelline membrane. severe birth defects such as missing or
3. (a) Trophoblast in mammalian embryo shortened limbs.
produces extra embryonic membranes 12. (c) A cross section at the midpoint of the
which provide protection and nourishment middle piece of human sperm will show
to foetus. These are of 4 types – chorion, mitochondria and 9+2 arrangement of
amnion, allantois & yolk sac embryonic microtubules.
membranes. 13. (b) The proliferative phase lasts for about 14
4. (b) Acrosome of sperm is formed from Golgi days in which the endometrium becomes
bodies and contains hydrolysing enzymes thicker by rapid cell multiplication.
for sperm penetration.
5. (b) Cumulus covers the ovum. The ovum at
Cycle begins 5 6 7
the matured conditions has a massy cloud 4 8
here 9
formed with a flat base and rounded 3
2 10
outlines piled up like a mountain. A Menstrual
1 Phase 11
granulosa cell is a somatic cell found
closely associated with the developing 28 12
Proliferative
female gamete (oocyte or egg) in the ovary 27 Phase 13
of mammals. Granulosa cells form a single Secretory 14
26
flattened layer around the oocyte in the Phase
25 15
primordial ovarian follicle and later in follicle 16
24
development they advance to form a multi 23 17
22 18
layered cumulus surrounding the oocyte. 21 20 19
Corpus luteum
6. (a) Menopause is the period when ovulation Ovulation
degenerates
and menstrual cycle stop in human females.
EBD_7100
B-134 Topicwise AIIMS Solved Papers – BIOLOGY
14. (b) Sperm is the male gamete that fuses with during pregnancy inhibit the cyclic release
female gamete and produce a diploid cell of pituitary gonadotropins and prevent the
called zygote. During fertilization, menstrual cycles. Pregnancy hormones
acrosome of the sperm releases some include hCG, which maintains the corpus
enzymes, particularly hyaluronidase, that luteum of pregnancy and forms the basis
facilitates the penetration of the sperm into for pregnancy diagnosis urine tests.
ovum. These enzymes dissolve th e Ovarian and placental estrogen and
membrane enveloping the ovum and help progesterone, human placental lactogen,
the sperm head to enter the ovum. and pituitary prolactin stimulate
15. (c) Both are characterized by a yellow colour. development of the ducts and alveoli in
The corpus luteum is a temporary the mammary glands.
endocrine structure in mammals, involved 23. (c)
in production of progesterone, which is 24. (c) Ejaculation of human male contains about
needed to maintain pregnancy. The macula 200 – 300 million sperms, of which for
or macula lutea is an oval yellow spot near normal fertility 60 % sperms must have
the center of the retina of the human eye. normal shape and size and at least 40 %
16. (d) Four viable sperm cells are produced from must show energetic motility.
one primary sex cell, whereby only one 25. (c) A–Umbilical cord with its vessels, B–
viable egg cell is produced, due to the Placental villi, C–Yolk sac, D–Plug of
unequal division of cytoplasm and the mucus in cervix
formation of polar bodies, which wither and 26. (c) Oogonia are called as gamete mother cell.
die. Corpus luteum is formed as a temporary
17. (b) Corpus luteum is a yellow coloured mass endocrine structure after the ovulation. It
of cells found in ovary. Corpus luteum is involved in the production of relatively
secretes progesterone hormone, which is high levels of progesterone and moderate
essential for maintaining pregnancy and levels of estradiol and inhibin A to maintain
therefore also called as “pregnancy pregnancy. A large number of primary
hormone.” follicles degenerate during the phase from
18. (b) Leydig cells are the characteristic of birth to puberty.
mammalian testis. They produce hormone,
testosterone meant for developement of Type B : Assertion Reason Questions
secondary sexual characters in males. 27. (c) Fertilization is the fusion of male and female
19. (c) Zygote divides partially in meroblastic gametes to form zygote. During fertilization
cleavage. only head of the sperm enters egg. After
20. (a) In human beings, after one month of that polyspermy is avoided by fertilization
pregnancy, the embryo's heart is formed. membrane.
By the end of second month of pregnancy 28. (a) Morula involves cleavage of cells till 32
limbs and digits are developed. By the end cell stage is formed. It is still surrounded
of 12 weeks (first trimester) most of the by Zona pellucida.
major organ systems are formed. 29. (a) Death is th e ultimate goal of every
21. (c) organism. This is caused by the wear and
22. (b) The high levels of estrogen and tear of organs which constitute the body
progesterone in the maternal circulation of a living being.
Human Reproduction B-135

30. (c) Old age is the progressive deterioration in 35. (c) Head of a sperm has acrosome but the
the structure and functioning of cells, spiral row of mitochondria are present in
tissues and organs and cessation of the mid (connecting) piece of the sperm.
immune system.
36. (c) Males have more stature than females
31. (a) According to programmed senescence because of the action of male sex hormone-
theory of ageing, ageing is a result of testosterone which is secreted by testis in
switching on and off of certain genes. B males. Body starts secreting testosterone
and T- lymphocytes undergo programmed from the age of puberty. Its secretion is
cell death called apoptosis. under the influence of Luteinizing Hormone
32. (b) In female, Graafian follicle forms corpus (LH) of the anterior lobe of pituitary gland.
luteum after ovulation. The cells of corpus Testosterone controls the development of
luteum are called luteal cells. The cytoplasm secondary sexual characters in males like
of luteal cells have yellow granules called hoarseness of voice, development of facial
lutein which secrete the hormone hairs, bone growth, calcium retention,
progesterone to maintain pregnancy if closing of epiphysial cartilage. The total
fertilization takes place. In the absence of quantity of bone matrix increases. The
fertilization, corpus luteum degenerates and pelvic outlet is narrowed and lengthened.
forms corpus albicans and there is decrease The strength of the pelvic bones increases
in progesterone level as well. to carry more loads. That is why, males have
33. (c) Clitoris is a female reproductive organ. It is more stature than females after puberty
homologous to penis of males. It is not when this hormone is present in the blood.
remnant of penis. It is devoid of erectile
hCG (Human Chorionic Gonadotropin) is
tissue and high blood supply as in penis,
the hormone secreted by human placenta
penis is the copulatory organ of males.
during pregnancy. hCG enlarges the corpus
34. (d) Hyaluronidase, a hydrolytic enzyme is an luteum in the mother's ovary and stimulates
acrosomal content in mammalian sperm. It it to secrete progesterone.
helps at the time of fertilization during the
37. (d) Testicular lobules are the compartments
penetration of the sperm into the ovum.
Based on the amount of yolk mammalian present in the testes, are not involved in
eggs are alecithal means egg without yolk. the process of fertilization as whole.
Microlecithal eggs contain very little yolk Fusion of male and female gametes is
e.g., sea urchin, starfish. On the basis of called fertilization.
distribution of yolk telolecithal eggs are 38. (c) Leydig cells, also known as interstitial cells,
those eggs in which the yolk is are found adjacent to the seminiferous
concentrated towards the vegetal pole and tubules in the testicle. They produce
cytoplasm and nucleus lie near the animal testosterone in the presence of luteinizing
pole, e.g., birds and reptiles. hormone (LH).
EBD_7100
B-136 Topicwise AIIMS Solved Papers – BIOLOGY

26 Reproductive Health

5. Match Column -I with Column - II. [2015]


TYPE A : MULTIPLE CHOICE QUESTIONS
Column I Column II
1. GIFT is [2009] Method Mode of Action
(a) transfer of a sperm in fallopian tube of a A. The pill I. Prevents sperms
female with the help of injections. reaching cervix
(b) transfer of a zygote fertilized in vitro in the B. Condom II. Prevents implantation
fallopian tube of female incapable to C. Vasectomy III. Prevents ovulation
conceive. D. Copper T IV. Semen contains no
(c) transfer of an ovum collected from a donor sperms
into another females fallopian tube who (a) A – III; B – I; C – IV; D – II
can’t produce an ovum but can provide a (b) A – IV; B – I; C – II; D – III
good environment for further development. (c) A – III; B – IV; C – I; D – II
(d) embryo is developed in vitro and then (d) A – II; B – III; C – I; D – IV
6. Select the correct match of the techniques given
transferred into female’s tract.
in column I with its feature given in column II.
2. What is the function of copper-T ?
Column I Column II
(a) Checks mutation [2012] A. ICSI I Artificially introduction of
(b) Stops fertilization semen into the vagina or uterus.
(c) Stops zygote formation B. IUI II Transfer of ovum collected
(d) Stops oblituation of blastocoel from a donor into the fallopian
tube where fertilization occur
3. Progestasert and LNG-20 are [2013]
(a) Implants C. IUT III Formation of embryo by
directly injecting sperm into the
(b) Copper releasing IUDs
D. GIFT ovum of the zygote or early
IV Transfer
(c) Non-medicated IUDs
embryo (with upto 8
(d) Hormone releasing IUDs blastomeres) into a fallopian
4. What is the figure given below showing in tube.
particular ? [2014] E. ZIFT V Transfer of embryo with more
than 8 blastomeres into the
uterus

[2016, 2017]
(a) A – V; B – IV; C – I; D – III; E – IV
(b) A – I; B – II; C – III; D – IV; E – V
(a) Ovarian cancer (b) Uterine cancer
(c) A – III; B – V; C – II; D – IV; E – I
(c) Tubectomy (d) Vasectomy
(d) A – III; B – I; C – V; D – II; E – IV
Reproductive Health B-137

TYPE B : ASSERTION REASON QUESTIONS 7. Assertion : Cu-T and Cu-7 do not suppresses
sperm-motility.
Directions for (Qs. 7-9) : Each of these questions
Reason : Hormones released by them affect
contains an Assertion followed by Reason. Read them
sperm motility. [2009]
carefully and answer the question on the basis of
8. Assertion : HIV infection can be avoided by
following options. You have to select the one that
use of condoms.
best describes the two statements.
Reason : Condoms secrete anti-viral interferons.
(a) If both Assertion and Reason are correct and
[2014]
Reason is the correct explanation of Assertion.
(b) If both Assertion and Reason are correct, but 9. Assertion : Copper -T is an effective
Reason is not the correct explanation of contraceptive device in human females.
Assertion. Reason : Copper-T prevents passage of sperms
(c) If Assertion is correct but Reason is incorrect. from vagina upwards into fallopian tubes.
(d) If both the Assertion and Reason are incorrect. [2011, 2014]
EBD_7100
B-138 Topicwise AIIMS Solved Papers – BIOLOGY

Type A : Multiple Choice Questions IUI (intrauterine insemination) - Artificial


introduction of semen into the vagina or
1. (c) GIFT is an ovum donation programme and
uterus
its purpose is to provide ovum to the
women who suffer from infertility due to IUT (Intra uterine transfer) - Transfer of
primary and premature ovarian failure embryo with more than 8 blastomeres into
which is incurable. A woman in the uterus
reproductive age denotes ova to a woman GIFT (Gamete intra fallopian transfer) -
which cannot produce ova but she could Transfer of ovum collected from a donor
provide good environment for embryo
into the fallopian tube where fertilization
development. The ovum from donor mother
occurs
is transferred to the recepient or would be
mother’s fallopian tube where it is fertilized ZIFT (Zygote intra fallopian transfer) -
by sperm and develops into embryo. Transfer of the zygote or early embryo
2. (b) Copper-T is copper releasing intra uterine (with upto 8 blastomeres) into a fallopian
devices (IUD). It increases phagocytosis tube.
of sperms within the uterus and suppress
sperm motility and fertilising capacity of Type B : Assertion Reason Questions
sperms.
3. (d) 7. (c) Cu-7 and Cu-T are intr auterine
contraceptive devices for females. They do
4. (c) The figure shows the tubectomy. This is a
not suppress sperm motility. Their mode of
surgical method to prevent pregnancy in
action is different. Cu-T and Cu-7 discharge
women. In tubectomy, small part of the 50-75 micrograms of ionic copper into the
fallopian tube is removed or tied through a uterus daily. These copper ions interfere
small cut in the abdomen or through with life-sustaining functions that regulate
vagina. It is very effective method but implantation in the uterus. No any hormone
reversibility is very poor. is released by them.
5. (a) A. The pill — Prevents ovulation 8. (c) The use of condoms has been shown to
B. Condom — Prevents sperm decrease the transmission of AIDS because
reaching cervix
condoms is contraceptic.
C. Vasectomy — Semen contains no
sperms 9. (c) Intra-uterine device (IUD) Copper-T is
D. Copper-T — Prevent implantation. plastic or metal object placed in the uterus
6. (d) ICSI (Intracytoplasmic sperm injection) - by a doctor. Copper-T preven t the
Formation of embryo by directly injecting fertilization of the egg or implantation of
sperm into the ovum the embryo.
Principles of Inheritance and Variation B-139

Principles of Inheritance and


27 Variation
TYPE A : MULTIPLE CHOICE QUESTIONS (a) essential features
(b) acquired characters
1. The formation of multivalents at meiosis in
(c) non-essential changes
diploid organism is due to [1998]
(a) monosomy (d) mutations
(b) deletion 8. Mirabilis jalapa shows [2001]
(c) inversion (a) codominance
(d) reciprocal translocation (b) incomplete dominance
2. If a homozygous tall plant is crossed with (c) dominance
homozygous dwarf plant, the offsprings will be (d) complementary genes
[1999] 9. Frame shift mutation occurs when [2002]
(a) all tall plants (b) all dwarf plants (a) base is added
(c) half tall plants (d) half dwarf plants (b) base is deleted
3. XO chromosomal abnormality in humans causes (c) base is added or deleted
[1999] (d) none of the above
10. Pure line breed refers to [2002]
(a) Turner’s syndrome
(a) homozygosity (b) heterozygosity
(b) Down’s syndrome
(c) linkage (d) both b & c
(c) Patau’s syndrome
11. If a homozygous red flowered plant is crossed
(d) Klinefelter’s syndrome
with a homozygous white flowered plant, the
4. Polygenic genes show [2000] offsprings would be [2002]
(a) similar genotype (a) all red flowered
(b) different phenotype (b) half red flowered
(c) different karyotype (c) half white flowered
(d) different genotype (d) all white flowered
5. Which disease has XXY chromosome constitution? 12. Genes of which one of the following is present
(a) Down’s syndrome [2000] exclusively on the X-chromosome in humans?
(b) Turner’s syndrome [2003]
(c) Klinefelter’s syndrome (a) Baldness
(d) Okazaki syndrome (b) Red-green colour blindness
6. Barr-body in mammals represents [2001] (c) Facial hair/moustaches in males
(a) One of the two X chromosomes in somatic (d) Night blindness
cells of females. 13. Given below is a pedigree chart of a family with
(b) All heterochromatin of male & female cells. five children. It shows the inheritance of attached
earlobes as opposed to the free ones. The squares
(c) Y chromosomes of male.
represent the male individuals and circles the
(d) All heterochromatin of female cells female individuals. Which one of the following
7. Discontinuous variations are [2001] conclusions drawn is correct? [2004]
EBD_7100
B-140 Topicwise AIIMS Solved Papers – BIOLOGY
17. Primary source of allelic variation is [2005]
(a) independent assortment
(b) recombination
(c) mutation
(d) polyploidy
18. Given below is a pedigree chart showing the
inheritance of a certain sex-linked trait in humans.

Generation 1
Free Attached

Ear-lobes Ear-lobes
(a) The parents are homozygous recessive. Generation 2
(b) The trait is Y-linked.
(c) The parents are homozygous dominant.
(d) The parents are heterozygous. Generation 3
14. Given below is a representation of a kind of Key :
chromosomal mutation. What is the kind of
mutation represented? Unaffected female
male Unaffected male
female

Affected male Affected male


A B C D E F G H
The trait traced in the above pedigree chart is
[2005]
(a) dominant X-linked
A E F G H
(b) recessive X-linked
(c) dominant Y-linked
D C B (d) recessive Y-linked
19. The "Cri-du-Chat" syndrome is caused by
[2004] change in chromosome structure involving
(a) deletion [2005]
(b) duplication (a) deletion (b) duplication
(c) inversion (c) inversion (d) translocation
(d) reciprocal translocation
20. Given below is a highly simplified representation
15. How many different types of gametes can be of the human sex ch romosomes from a
formed by F1 progeny, resulting from the karyotype. [2006]
following cross Tt × Rr? [2004]
(a) 4 (b) 8
(c) 27 (d) 64
Gene a
16. Grain colour in wheat is determined by three pairs Gene b
of polygene. Following the cross AABBCC (dark
colour) × aabbcc (light colour), in F2 generation
what proportion of the progeny is likely to
resemble either parent? [2005]
(a) Half (b) Less than 5 percent
(c) One third (d) None of these
Principles of Inheritance and Variation B-141

The genes a and b could be of 27. Bird females have chromosome arrangement as
(a) colour blindness and body height [2009]
(b) attached ear lobe and Rhesus blood group (a) XY (b) XO
(c) haemophilia and red-green colour (c) WZ (d) WW
blindness 28. Gene pool is referred to [2009]
(d) phenylketonuria and haemophilia (a) the genetic drift caused in a population
21. In India, we find mangoes with different flavours, (b) aggregate of all genes and their alleles in a
colours, fibre content, sugar content and even population.
shelf-life. The large variation is on account of (c) deletion of non essential genes.
[2006] (d) induce cell division
(a) species diversity 29. Mother and father both have blood group ‘A’.
(b) induced mutations They have two children one with blood group
(c) genetic diversity ‘O’ and second one with blood group ‘A’.
(d) hybridization They have [2009]
22. Which one of the following pairs of features is a (a) mother has homozygotic gene father has
good example of polygenic inheritance? [2006] heterozygote I A IA.
(a) Human height and skin colour. (b) both are homozygotic (IAIA).
(b) ABO blood group in humans and flower (c) mother is heterozygotic (IAi) and father is
colour of Mirabilis jalapa. homozygotic (IAIA).
(c) Hair pigment of mouse and tonque rolling (d) both are heterozygotic (IAi).
in humans. 30. When one gene controls two or more different
(d) Human eye colour and sickle cell anaemia. characters simultaneously, the phenomenon is
23. Gene which suppresses other gene’s activity called [2010]
but does not lie on the same locus is called as (a) apomixis (b) pleiotropy
[2007] (c) polyploidy (d) polyteny
(a) epistatic (b) supplementary 31. Three children in a family have blood types O,
(c) hypostatic (d) codominant AB and B respectively. What are the genotypes
of their parents? [2013]
24. XO-chromosomal abnormality in human beings
(a) IA i and IBi (b) IAIB and i i
causes [2007]
(c) IBIB and IAIA (d) IAIA and IBi
(a) Turner’s syndrome
32. If both parents are carriers for thalassemia, which
(b) Down’s syndrome
is an autosomal recessive disorder, what are the
(c) Klinefelter’s syndrome
chances of pregnancy resulting in an affected
(d) none of the above
child? [2014]
25. A normal woman whose father was colour (a) 50% (b) 25%
blind, is married to a normal man. The sons (c) 100% (d) no chance
would be [2008] 33. In Huntington’s disease, the unaffected persons
(a) 75% colour blind are homozygous for normal allele h. The
(b) 50% colour blind following is erroneous because [2015]
(c) all normal
(d) all colour blind
26. Mating of an organism to a double recessive in
1 2
order to determine whether it is homozygous or
heterozygous for a character under
consideration is called [2008] 3 4 5 6 7
(a) reciprocal cross (b) test cross
(c) dihybrid cross (d) back cross
8 9 10 11
EBD_7100
B-142 Topicwise AIIMS Solved Papers – BIOLOGY
(a) it shows both male and female affected by (a) 98.7% and 62.8%
Huntingtons disease (b) 1.3% and 37.2%
(b) either person 6 or 7 should have the (c) 37.2 and 1.3%
disease, if individual 11 shows the disease.
(d) 62.8% and 98.7%
(c) at least one of the 2 children (8, 9) should
have the disease TYPE B : ASSERTION REASON QUESTIONS
(d) all of these Directions for (Qs. 35-38) : These questions consist
34. The experiment shown in the given figure has of two statements, each printed as Assertion and
been carried out by Morgan to show the Reason. While answering these questions, you are
phenomenon of linkage and recombination. If in required to choose any one of the following five
cross I, genes are tightly linked and in cross II, responses.
genes are loosely linked then what will be the (a) If both Assertion and Reason are correct and
percentage of recombinants produced in cross I the Reason is a correct explanation of the
and cross II respectively? Assertion.
(b) If both Assertion and Reason are correct but
Cross I
Reason is not a correct explanation of the
Assertion.
(c) If the Assertion is correct but Reason is
Parents
incorrect.
Yellow White Wild type
(d) If both the Assertion and Reason are incorrect.
body eyes (e) If the Assertion is incorrect but the Reason is
correct.
35. Assertion : The genetic complement of an
F1 generation organism is called genotype.
Reason : Genotype is the type of hereditary
Wild type Yellow White
body eyes
properties of an organism. [1999]
36. Assertion : Phenylketonuria is a recessive
hereditary disease caused by body’s failure to
oxidize an amino acid phenylalanine to tyrosine,
because of a defective enzyme.
Cross II
Reason : It results in the presence of
phenylalanine acid in urine. [2000]
37. Assertion : In humans, the gamete contributed
White Miniature Wild type
by the male determines whether the child
body wings produced will be male or female.
Reason : Sex in humans is a polygenic trait
depending upon a cumulative effect of some
genes on X-ch romosome and some on
Y-chromosome. [2005]
Wild type White Miniature 38. Assertion : Haemophilia is a recessive sex linked
body wings disease.
Reason : Haemophilia occurs due to mutation
[2016] of a structural gene on chromosome 15.
[2007]
Principles of Inheritance and Variation B-143

Directions for (Qs.39-44) : Each of these questions 41. Assertion: Aneuploidy may be of hypoploidy
contains an Assertion followed by Reason. Read them or hyperploidy type.
carefully and answer the question on the basis of Reason: Monosomy lacks one pair of chromo-
following options. You have to select the one that
somes. [2011]
best describes the two statements.
42. Assertion : Cross of F1 individual with recessive
(a) If both Assertion and Reason are correct and
homozygous parent is test cross.
Reason is the correct explanation of Assertion.
(b) If both Assertion and Reason are correct, but Reason : No recessive individual are obtained
Reason is not the correct explanation of in the monohybrid test cross. [2012]
Assertion. 43. Assertion : In Mirabilis, selfing of F1 pink
(c) If Assertion is correct but Reason is incorrect. flower plants produces same phenotypic &
(d) If both the Assertion and Reason are incorrect. genotypic ratio.
39. Assertion : Persons suffering from haemophilia Reason : Flower colour gene shows incomplete
fail to produce blood clotting factor VIII. dominance. [2014]
Reason : Prothrombin producing platelets in 44. Assertion : In humans, the gamete contributed
such persons are found in very low by the male determines whether the child
concentration. [2008] produced will be male or female.
40. Assertion : In case of incomplete linkage, linked Reason : Sex in humans is a polygenic trait
gene show new combination along with parental
depending upon a cumulative effect of some
combination.
genes on X-ch romosome and some on
Reason : In case of incomplete linkage, linked
Y-chromosome. [2015, 2017]
genes are separated by crossing over. [2010]
EBD_7100
B-144 Topicwise AIIMS Solved Papers – BIOLOGY

Type A : Multiple Choice Questions 10. (a) The self pollinated progeny of a
homozygous plant constitutes a pure line.
1. (d) Translocation is the separation of a
chromosome segment and its union to a 11. (a) As per Mendel’s law in F1 generation only
non homologous chromosome. In dominant phenotypes appear.
reciprocal translocation 2 non-homologous
chromosomes exchange segments among Parents RR X rr (White)
themselves. Reciprocal translocation (Red)
produces duplication.
Gametes R R r r
2. (a) Homozygous Tall × Homozygous dwarf
TT tt F1 generation Rr (Heterozygous Red)

Gamets T T t t
12. (b) Red and green colourblindness is a sex
Tt
All Tall linked inheritance. Its genes are present on
X-chromosome.
3. (a) XO chromosomal abnormality in humans
13. (d) Such types of results are obtained only if
is due to monosomy of sex chromosome.
parents are heterozygous for the trait.
Their total number of chromosomes are 45.
This condition is found in Turner’s 14. (c) The kind of mutation is paracentric
syndrome. inversion. In this a segment of a
4. (b) Polygenic genes show intermediate chromosome separates and rejoins in an
phenotypes, e.g. skin colour. inverted position.
5. (c) Down’s syndrome is due to trisomy of 21st 15. (a) F1 generation is always heterozygous, e.g.
chromosome; Turner’s syndrome having TtRr, so there are 4 types of gamete
XO genotype is caused by the absence of formation i.e. TR, Tr, tR, tr.
X chromosome in females; klinefelter’s
syndrome (XXY) is due to trisomy of sex aabbcc
16. (b) AABBCC
chromosome.
6. (a) Barr body is th e inactive one X-
ABC abc Gametes
chromosome in somatic cells of female
(Dosage compensation). The number of
Barr bodies are always one less than the AaBbCc F 2 generation
total number of X-chromosome.
7. (d) Discontinuous variations lead to The F2 generation will show the intermediate
mutations. colour because of quantitative inheritence. In
8. (b) Mirabilis jalapa (4O’ clock plant) shows case of crossing between AA BB CC (dark
incomplete dominance because the genes colour) and aa bb cc (light colour), in F2
for red and white colour do not mix in the generation seven phenotypes will be obtained
F1 pink hybrids as both the pure characters with ratio 1 : 6 : 15 : 20 : 15 : 6 : 1. The total number
reappear in the F2 plants. of progeny is 64, out of which only two will be
9. (c) A mutation in which there is deletion or likely resemble with either parents. Hence, their
insertion of one or few nucleotides is called percentage in F2 generation would be 3.12 i.e
frameshift mutation. less than 5%.
Principles of Inheritance and Variation B-145

17. (c) Primary source of allelic variation is F2 generation :


mutation. Gametes CA Ca cA ca
18. (b) The genes for such traits are recessive and CCAA CCAa CcAA CcAa
located on the X-chromosome. The CA Agouti Agouti Agouti Agouti
character appears more often in males in CCAa CCaa CcAa Ccaa
Ca Agouti Coloured Agouti Coloured
hemizygous condition, but also in females
with homozygous condition. Affected CcAA CcAa ccAA ccAa
cA Agouti Agouti Albino Albino
males receive their defective gene from
carrier mothers who may have affected CcAa CcAa ccAa ccaa
ca
Agouti Coloured Albino Albino
father. These exhibit criss- cross
inheritance. Agouti - 9
19. (a) Cri-du-chat/cat cry syndrome is due to the Coloured - 3
deletion of large part of the small or one of Albino - 4
the 5th chromosome. Ratio - 9 : 3 : 4
20. (c) Genes a and b lie very close to each other. 24. (a) Turner's syndrome is caused due to the
So, they are representing linked genes. e.g. missing X chromosome. The genotype is
genes of haemophilia and colour blindness therefore XO instead of the normal XX and
show sex linked inheritance. the sufferer person possesses 45
21. (c) Genetic diversity describes an attribute chromosomes instead of 46. Patients with this
which is commonly held to be condition can best be described as
advantageous for survival that there are incompletely developed females, although
there are often no obvious external
many different versions of otherwise similar
differences compared with normal females.
organisms, e.g. different varieties of
Affected females lack ovaries so they are
mangoes. infertile.
22. (a) Polygenic inheritance is the trait under the
control of more than one pair of genes, e.g. 25. (b)
skin colour (trigenic) and human height. Genotype of
normal woman with Genotype of
23. (a) When one gene masks the effect or activity colourblind father normal man
of another gene which does not lie on the h X Y
X X
same locus, it is called epistasis. Epistasis
refers to non- allelic interactions. Like coat h
X X XY
colour in mice is controlled by epistatic Colourblind h
XX Normal son
X Y
gene. When coloured (CCaa) mouse is daughter Normal
Colourblind
crossed with albino (ccAA), agouti mice son daughter
(ccAa) appeared in F1 generation. Agouti, 50% son would be colourblind.
coloured and albino mice are obtained in 26. (b) In genetics, a test cross, first introduced
9 : 3 : 4 ratio in F2 generation. by Mendel, is used to determine if an
individual exhibiting a dominant trait is
Parent : CCaa ccAA homozygous or heterozygous for that trait.
Coloured × Albino Test crosses involve breeding the
individual in question with another
Gametes : Ca cA individual that expresses a recessive
version of the same trait. If all offspring
F1 generation : CcAa display the dominant phenotype, the
Agouti individual in question is homozygous
EBD_7100
B-146 Topicwise AIIMS Solved Papers – BIOLOGY
dominant; if the offspring display both that superficially resembles normal sexual
dominant and recessive phenotypes, then reproduction but there is no fusion of
the individual is heterozygous. In some gametes.
sources, the “test cross” is defined as 31. (a)
being a type of back cross between the
recessive homozygote and F1 generation. 32. (b) Genotype of carrier parents is – Aa (male
27. (c) In birds sex is determined by parent) × Aa (female parent)
morphologically dissimilar pair of
A a
chromosomes called sex chromosomes. Z
and W are two sex chromosomes of birds. A AA Aa
A male bird has ZZ (homogenetic sex) a Aa *aa
arrangement while a female bird has ZW
(heterogenetic sex) arrangement of AA= normal child (25%)
chromosomes. Number of chromosomes in
Aa = carriers child (50%)
birds is 69.
28. (b) Gene pool is the sum total of genes and their aa = affected child (25%)
alleles in the reproductive gametes of a 33. (b) Since the offspring number 11 is affected
population. The gene pool is transferred (diseased) either of the two parents 6 or 7
from one generation to the other using have to be affected.
gametes from genetic pool. These gametes 34. (b) The percentage of recombinants produced
will form zygotes of next generation. Gene in cross I and cross II are respectively 1.3%
pool of a population will consist of a large and 37.2%.
number of genes which vary in their
frequencies. Type B : Assertion Reason Questions
29. (d) In the mentioned case, one child with blood 35. (a) Genotype of the organism include all
group ‘O’ and second with blood group dominant and recessive characters.
‘A’ are born to parents with heterozyous 36. (b) Phenylketonuria is an recessive autosomal
condition of genes for blood group A . i.e., gene disorder. It occurs due to the absence
of enzyme phenylalanine hydroxylase
IAI0. Both mother and father have blood
which changes phenylalanine to tyrosine.
group A but their genotypes indicate 37. (c) In humans, sex of a child depends upon
heterozygotic condition. the gametes produced by the male (X, Y).
A 0 A 0
38. (c) Haemophilia also known as bleeder disease
I I I I is an example of recessive sex linked
inheritance in human beings. It is masked
Mother Father in heterozygous condition. The person
suffering from this disease lack factors VIII
Children A A A 0 A 0 0 0 and IX responsible for blood clotting. A
I I I I I I I I
small cut may lead to bleeding till death.
Blood group A A A O
Men are affected by this disease while
30. (b) Pleiotropy is the condition in which a single women are the carriers.
gene influences more than one trait. Mutation of a structural gene on
Polyploidy is a condition in which individuals chromosome number 15 causes Marfan
have more than two complete sets of syndrome. This disease results in formation
chromosomes. of abnormal form of connective tissues and
Apomixis is a reproductive process in plants characteristic extreme loosseness of joints.
Principles of Inheritance and Variation B-147

39. (c) Haemophilia bleeding disorder is a group due to addition of one or mor e
of hereditary genetic disorders that impair chromosomes to complete chromosome
the body’s ability to control blood clotting complement (hyperploidy). Hypopliody is
or coagulation. In its most common form, mainly due to loss of a single chromosomes,
Hemophilia A, clotting factor VIII is absent. monosomes (2n – 1) or due to loss of one
In Haemophilia B, factor IX is deficient. pair of chromosomes, nullisomes (2n – 2).
Factor VIII participates in blood 42. (c) In the monohybrid test-cross both
coagulation; it is a cofactor for factor IXa dominant and recessive traits are obtained
which, in the presence of Ca +2 and in 1 : 1 ratio.
phospholipids forms a complex that
43. (a) F2 phenotypic and genotypic ratio in
converts factor X to the activated form Xa.
monohybrid cross involving incomplete
Defects in this gene results in hemophilia
dominance is
A, a common recessive X-linked
coagulation disorder. Proth rombin 1 : 2 : 1
producing platelets in such persons are not RR Rr rr
found in very low concentration. (red) (pink) (white)
40. (a) In case of incomplete linkage, the linked 44. (c) In human, the gamete contributed by the
gene shows new combination along with male determines whether the child pro-
parental combination due to crossing over duced will be male or female. Sex in
between chromatids. humans is a polygenic trait depending
41. (c) Aneuploidy can be either due to loss of upon cumulative effect of some genes
one or more chromosomes (hypoploidy) or present on Y-chromosome. Only sex in
human is amonogenic trait.
EBD_7100
B-148 Topicwise AIIMS Solved Papers – BIOLOGY

28 Molecular Basis of Inheritance

TYPE A : MULTIPLE CHOICE QUESTIONS 8. Okazaki fragments form [2000]


(a) leading strand (b) lagging strand
1. The process through which the amount of DNA, (c) non-sense strand (d) senseful strand
RNA and protein can be known at a time is called 9. Wobble hypothesis was given by [2002]
[1997] (a) F.H.C. Crick (b) Nirenberg
(a) autoradiography (c) Holley (d) Khorana
(b) tissue culture 10. Which one of the following pairs of terms/names
mean one and the same thing? [2003]
(c) cellular fractioning
(a) Gene pool-genome
(d) phase contrast microscopy
(b) Codon-gene
2. Balbiani rings are found in [1997]
(c) Cistron-triplet
(a) polysomes
(d) DNA fingerprinting - DNA profiling
(b) polytene chromosomes 11. What is true about t-RNA? [2003]
(c) autosomes (a) It binds with an amino acid at it 3' end.
(d) nonsense chromosomes (b) It has five double stranded regions.
3. In DNA helix, cytosine is paired with guanine (c) It had a codon at one end which recognizes
by [1997] the anticodon on messenger RNA.
(a) covalent bond (d) It looks like clover leaf in the three
(b) phosphate bond dimensional structure.
(c) three hydrogen bonds 12. Which one of the following codons codes for
the same information as UGC? [2003]
(d) two hydrogen bonds
(a) UGU (b) UGA
4. Which RNAs pick up specific amino acid from
(c) UAG (d) UGG
amino acid pool in the cytoplasm to ribosome
13. During protein synthesis in an organism, at one
during protein synthesis ? [1998]
point the process comes to a halt. Select the
(a) tRNA (b) mRNA group of the three codons from the following,
(c) rRNA (d) hnRNA from which anyone of the three could bring about
5. The structure of DNA is [1998] this halt. [2006]
(a) linear (b) double helix (a) UUU, UCC, UAU (b) UUC, UUA, UAC
(c) single helix (d) triple helix (c) UAG, UGA, UAA (d) UUG, UCA, UCG
6. Transposon was discovered by [1998] 14. The total number of nitrogenous bases in human
(a) Sutton (b) Strassburger genome is estimated to be about [2004, 2008]
(c) Fischer (d) B.Mc Clintock (a) 3.5 million (b) 35 thousand
(c) 35 million (d) 3.1 billion
7. Root cell of wheat has 42 chromosomes. What
15. Which one of the following pairs is correctly
would be the number of chromosomes in the
matched with regard to the codon and the amino
synergid cell ? [1999]
acid coded by it ? [2004, 2008]
(a) 7 (b) 14 (a) UUA-valine (b) AM-lysine
(c) 21 (d) 28 (c) AUG-cysteine (d) CCC-alanine
Molecular Basis of Inheritance B-149

16.
5 3'
5'

3 (d) 5'
3'
3
5
3'
5'
5
3
19. Select the correct option: [2014]
What is the error in above diagram? [2009]
Direction of RNA Direction of reading of
(a) Arrows are wrongly depicted.
synthesis the template DNA strand
(b) Polarity is incorrect. (a) 5´—3´ 3´—5´
(c) Both arrows and polarity are incorrect. (b) 3´—5´ 5´—3´
(d) None of the above. (c) 5´—3´ 5´—3´
17. TATA box of eukaryotic promotor lies [2010] (d) 3´—5´ 3´—5´
(a) about 25 bp upstream of the transcription 20. Which one of the following represents a
start site. palindromic sequence in DNA? [2014]
(b) about 50 bp upstream of the transcription (a) 5' - GAATTC - 3'
start site. 3' - CTTAAG - 5'
(c) about 75 bp upstream of the transcription (b) 5' - CCAATG - 3'
start site. 3' - GAATCC - 5'
(d) about 200 bp upstream of the transcription (c) 5' - CATTAG - 3'
start site. 3' - GATAAC - 5'
18. Which one of the following correctly represents (d) 5' - GATACC - 3'
the manner of replication of DNA ? [2003, 2012] 3' - CCTAAG - 5'
21. Thirty percent of the bases in a sample of DNA
5'
3'
extracted from eukaryotic cells is adenine. What
percentage of cytosine is present in this DNA?
(a) 3' [2015]
5' (a) 10% (b) 20%
5' (c) 30% (d) 40%
3' 22. There are three genes a, b, c. Percentage of
crossing over between a and b is 20%, b and c is
3' 28% and a and c is 8%. What is the sequence of
5' genes on chromosome? [2015]
5'
(a) b, a, c (b) a, b, c
(b)
3' (c) a, c, b (d) None of these
3'
23. Which one of the following group of codons is
5' called as degenerate codons? [2016]
(a) UAA, UAG and UGA
3' (b) GUA, GUG, GCA, GCG and GAA
5' (c) UUC, UUG, CCU, CAA and CUG
5'
(d) UUA, UUG, CUU, CUC, CUA and CUG
(c)
3' 24. The given figure shows the structure of
nucleosome with their parts labelled as A, B &
5'
3' C. Identify A, B and C. [2017]
EBD_7100
B-150 Topicwise AIIMS Solved Papers – BIOLOGY
(c) If the Assertion is correct but Reason is
A B
incorrect.
(d) If both the Assertion and Reason are incorrect.
C (e) If the Assertion is incorrect but the Reason is
correct.
26. Assertion : Histones are basic proteins of major
Core of histone molecules importance in packaging of eukaryotic DNA.
DNA and histones comprise chromatin forming
(a) A – DNA; B – H1 histone; the bulk of eukaryotic chromosome.
C – Histone octamer Reason : Histones are of five major types
(b) A – H1 histone; B – DNA;
C – Histone octamer H1 , H 2 A H 2 B, H 3 and H 4 . [2000]
(c) A – Histone octamer; B – RNA; 27. Assertion: mRNA attaches to ribosome through
C – H1 histone its 3' end.
(d) A – RNA; B – H1 histone; Reason: The mRNA has F-capsular nucleotide
C – Histone octamer and bases of lagging sequence. [2002]
25. Match the codons given incolumn I with their
28. Assertion : Replication and transcription occur
respective amino acids given in column II and
in the nucleus but translation in the cytoplasm.
choose the correct answer. [2017]
Reason : m-RNA is transferred from the nucleus
into the cytoplasm where ribosomes and amino
Column -I Column -II acids are available for protein synthesis. [2005]
(Codons) (Amino acids)
29. Assertion: An organism with lethal mutation
A UUU I. Serine may not even develop beyond the zygote.
B GGG II. Methionine [2006]
C UCU III. Phenylalanine Reason: All types of gene mutations are lethal.
D CCC IV. Glycine 30. Assertion: Polytene chromosomes have a high
E AUG V. Proline amount of DNA.
Reason: Polytene chromosomes are formed by
(a) A – III; B – IV; C – I; D – V; E – II repeated replication. [2006]
(b) A – III; B – I; C – IV; D – V; E – II Directions for (Qs.31-35) : Each of these questions
(c) A – III; B – IV; C – V; D – I; E – II contains an Assertion followed by Reason. Read them
(d) A – II; B – IV; C – I; D – V; E – III carefully and answer the question on the basis of
TYPE B : ASSERTION REASON QUESTIONS following options. You have to select the one that
best describes the two statements.
Directions for (Qs. 26-30) : These questions consist (a) If both Assertion and Reason are correct and
of two statements, each printed as Assertion and Reason is the correct explanation of Assertion.
Reason. While answering these questions, you are (b) If both Assertion and Reason are correct, but
required to choose any one of the following five Reason is not the correct explanation of
responses. Assertion.
(a) If both Assertion and Reason are correct and (c) If Assertion is correct but Reason is incorrect.
the Reason is a correct explanation of the (d) If both the Assertion and Reason are incorrect.
Assertion. 31. Assertion : DNA is associated with proteins.
(b) If both Assertion and Reason are correct but Reason : DNA binds around histone proteins
Reason is not a correct explanation of the that form a pool and the entire structure is called
Assertion. a nucleosome. [2013]
Molecular Basis of Inheritance B-151

32. Assertion : The uptake of DNA during 34. Assertion : In a DNA molecule, A–T rich parts
transformation is an active, energy requiring melt before G–C rich parts.
process. Reason : In between A and T there are three
Reason : Transformation occurs in only those H–bond, whereas in between G and C there are
bacteria, which possess the enzymatic two H-bonds. [2010, 2015]
machinery involved in the active uptake and 35. Assertion : Replication and transcription occur
recombination [2014] in the nucleus but translation takes place in the
33. Assertion : UAA, UAG and UGA terminate cytoplasm.
protein synthesis. Reason : mRNA is transferred from the nucleus
into cytoplasm where ribosomes and amino acids
Reason : They are not recognised by tRNA.
are available for protein synthesis.
[2011, 2014]
[2008, 2015, 2017]
EBD_7100
B-152 Topicwise AIIMS Solved Papers – BIOLOGY

Type A : Multiple Choice Questions of a triplet codon on mRNA have a precise


1. (a) Autoradiography is the use of X-ray films pairing with the bases of the tRNA
to detect radioactive material. It produces anticodon.
a permanant record of positions and relative 10. (d) Gene pool is the sum total of genes present
intensities of radiolabeled bands in a gel or in the inbreeding population whereas
slot. Typically, biomolecules are labeled genome is a complete set of genes in the
with 32P or 35 S, and detected by over night haploid case of chromosomes. Codon is a
film exposure. triplicate of nucleotide whereas gene is a
Autoradiography technique may be used linear segment of DNA. Cistron is a segment
to determine the tissue localization of a of the DNA molecule carrying information
radioactive substance, either introduced for the production of one polypeptide
into a metabolic pathway, bound to a chain. DNA fingerprinting is also known
receptor or enzyme, or hybridized to a as DNA profiling or DNA typing.
nucleic acid. 11. (d) t-RNA has a clover leaf like structure.
2. (b) Balbiani rings are temporary swellings in 12. (a) UGC and UGU codes for cysteine.
polytene chromosome having uncoiled 13. (c) Protein termination occurs by nonsense
active DNA that transcribes RNA. codons i.e. UAA, UGA ,UAG.
14. (d) Genome size is usually stated as the total
3. (c) In DNA helix, cytosine always pairs with number of base pairs; the human genome
guanine by three hydrogen bonds. contains roughly 3.1 billion base pairs
4. (a) There are specific types of tRNA (because organized into 24 distinct, physically
of anticodon) for specific amino acid. They separate microscopic units called
transport the amino acids from the chromosomes. All genes are arranged
cytoplasm to the site of protein synthesis. linearly along the chromosomes. The
5. (b) DNA is double helical in structure. Its both complete set of instructions for making an
strands are complementary to each other. organism is called its genome.
6. (d) Mc Clintock discovered transposon or 15. (a) Valine is one of 20 proteogenic amino acids.
(jumping genes) in maize plants in 1983. Its codons are GUU, GUC, GUA, and GUG.
Cysteine codons are UGU and UGC. With a
7. (c) The root cell in wheat is a somatic cell and,
thiol side chain, cysteine is classified as a
hence, has 2n number of chromosomes
hydrophilic amino acid. Alanine codons are
which is a diploid condition. The synergid
GCU, GCC, GCA, and GCG. It is classified as a
cell that is formed after meiosis along with
non-polar amino acid. L-alanine is second
the ovum has 21 chromosomes which is
only to leucine. Lysine codons are AAA and
the haploid condition.
AAG. Lysine is a base, as are arginine and
8. (b) Okazaki fragments are short pieces of newly histidine.
synthesized DNA, which are generated
16. (b) The figure below is the replicating fork of
durin g th e normal process of DNA
DNA. The DNA replication takes place in
replication. They are linked up by the
5 to 3 direction always. On the leading
enzyme DNA ligase after replacing RNA
strand DNA replication is continuous while
primers with deoxribonucleotides. This will
on lagging strand DNA replication is
form lagging strand.
discontinuous. The polarity of lagging
9. (a) Wobble hypothesis was given by F.H.C. strand is incorrect in the given figure. The
Crick. It states that the first two positions correct figure should be
Molecular Basis of Inheritance B-153

nd 20% 8%
stra 22. (a) b a c
ng 5
g gi 28%
La
23. (a) Degenerate codons (also called as non -
3 sense codons or terminator codons) do not
3 code for any amino acids. Three types of
5
degenerate codons are UAG (amber), UAA
Leading (ochre) and UGA (opal).
strand 5
3 24. (a) Nucleosome is a structural unit of a
eukaryotic chromosome which consists of
Both the strands are antiparallel. In one a length of DNA coiled around a core of
strand carbon of sugar are in 3 – 5 histones and are thought to be present only
direction and in other the carbon of sugar during interphase of the cell cycle. In the
are in 5 – 3 direction. given figure of nucleosome structure, the
17. (a) TATA box of eukaryote promoter lies about parts marked as A, B and C are respectively
25 bp upstream of the transcription start site. DNA, H1 histones and histone octamer.
18. (d) The new strands of DNA are formed in the 25. (a) UUU – Phenylalanine
GGG – Glycine
5' 3' direction from the 3' 5' template
DNA by the addition of UCU – Serine
deoxyribonucleotides to the 3' end of primer CCC – Proline
RNA. AUG – Methionine
19. (a) Synthesis of RNA exhibits several features Type B : Assertion Reason Questions
that are synonymous with DNA replication.
RNA synthesis requires accurate and 26. (a) Histones are basic proteins found in the
efficient initiation, elongation proceeds in eukaryotic chromosomes. These are rich
the 5'–3' direction (i.e. the polymerase in the amino acids lysine and arginine.
moves along the template strand of DNA Histonie proteins are basic proteins
in the 3'–5' direction), and RNA synthesis consisting of 5 types – H1, H2A, H2B, H3,
requires distinct and accurate termination. H4. DNA is coiled around it. It exists as
Transcription exhibits several features that octamers linked with H1.
are distinct from replication. 27. (d) mRNA is attached to the ribosome by
20. (a) A palindromic sequence is a nucleic acid means of protein ribophorin I & II. The
sequence (DNA or RNA) that is the same sequence of nucleotides on mRNA is called
whether read 5' (five-prime) to 3' (three codon.
prime) on one strand or 5' to 3' on the 28. (a) DNA is the master copy which transcribes
complementary strand with which it forms to form working copy in the form of mRNA
a double helix. which translates in the form of peptide
5 - GAATTC - 3 chain in the cytoplasm. It is also called
3 - CTTAAG - 5 central dogma.
It is a palindromic sequence of DNA cut by 29. (c) Organisms with lethal mutation bear lethal
restriction enzyme EcoRI. genes that result in the death of the
21. (b) If 30 percent of DNA is adenine, then by individual which carries them. The
Chargaff’s rule 30 percent will be thymine. completely lethal genes usually cause death
The remaining 40 percent of the DNA is of the zygote. Mutation is a sudden heritable
cytosine and guanine. Since the ratio of change in the sequence of gene occuring
cytosine to guanine must be equal, then on the chromosomes. Mutation may be
each accounts for 20 percent of the bases. beneficial, normal, sublethal or lethal.
EBD_7100
B-154 Topicwise AIIMS Solved Papers – BIOLOGY
30. (a) Polytene chromosomes show endomitosis 33. (a) Synthesis of polypeptide terminates when
and they have high DNA content. a nonsense codon of mRNA reaches the
31. (a) A chain of DNA has 140 base pairs, make A– site. There are three nonsense codons–
3 UAA, UAG and UGA. These codons are
1 turns and twist around a histone not recognised by any of the tRNAs.
4
octamer forming nucleosome. The core of Therefore, no more aminoacyl tRNA
nucleosome consists of 4 histones H2A, reaches the A site. The P– site tRNA is
H2B, H3 and H4. hydrolysed and the completed polypeptide
32. (a) Transformation does not involve passive is released in the presence of a release
entry of DNA molecules through permeable factor.
cell walls and membranes. It does not occur 34. (c) In a DNA molecule, A-T rich parts melt
'naturally' in all species of bacteria, only in before G-C rich parts because there are two
those species possessing the enzymatic H-bond between A and T whereas in
machinery involved in the active uptake between G and C, there are three H-bond.
and recombination processes. Even in 35. (a) In eukaryotes, the replication and
these species, all cells in a given population transcription takes place in the nucleus.
are not capable of active uptake of DNA. mRNA comes out from the nucleus through
Only competent cells, which possess a so the nuclear pore. In cytoplasm, translation
called competence factor are capable of occurs. In prokaryote, there is no nuclear
serving as recipients in transformation. membrane, so replication, transcription and
translation all occur in the cytoplasm.
29 Evolution

TYPE A : MULTIPLE CHOICE QUESTIONS 9. Wings of pigeon, mosquito and bat show
[1999, 2007, 2011]
1. The branch of science dealing with process of (a) divergent evolution
improvement of human race by selective
(b) atavism
breeding is called [1997]
(a) Eugenics (b) Euthenics (c) convergent evolution
(c) Euphenics (d) Obstetrics (d) all of these
2. The connecting link between annelida and 10. Which of the following cannot be explained by
mollusca is [1998] Lamarckism? [2012]
(a) Neoplina (b) Nautilus (a) Absence of lips in snakes
(c) Glochidium (d) Velliger larva (b) Long neck of giraffe
3. Theory of Pangenesis was given by [1998] (c) Degeneration of visual apparatus in cave
(a) Lamarck (b) Oparin dwellers
(c) Darwin (d) De Vries (d) Dull progeny of noble laureate
4. The cranial capacity was largest among the 11. Thorn of Bougainvillea and tendril of Cucurbita
[2002] are example of [2013]
(a) Peking man (b) African man (a) analogous organs
(c) Java Ape man (d) Neanderthal man (b) homologous organs
5. A baby has been born with a small tail. It is the
(c) vestigial organs
case exhibiting [2004]
(a) retrogressive evolution (d) retrogressive evolution
(b) mutation 12. Forelimbs of cat, lizard used in walking; forelimbs
(c) atavism of whale used in swimming and forelimbs of bats
(d) metamorphosis used in flying are an example of [2014]
6. “Homo sapiens” implies [2007] (a) Analogous organs
(a) human race (b) human beings (b) Adaptive radiation
(c) modern man (d) none of these (c) Homologous organs
7. The study of homologous structures in mature (d) Convergent evolution
organisms provides evidence for the 13. The diagram represents Miller's experiment.
evolutionary relationships among certain groups Choose the correct combination of labelling.
of organisms. Which field of study includes this [2015]
evidence of evolution? [2009]
(a) Comparative cytology
(b) Biochemistry
(c) Geology
(d) Comparative anatomy
8. Darwin’s finches were a good example of [2009]
(a) convergent evolution
(b) adaptive radiation
(c) mutation
(d) none of the above
EBD_7100
B-156 Topicwise AIIMS Solved Papers – BIOLOGY
(a) A–electrodes, B– NH3 + H2 + H2O + CH4, (b) If both Assertion and Reason are correct but
C–cold water, D–vacuum, E–U trap Reason is not a correct explanation of the
(b) A–electrodes, B–NH4 + H2 + CO2 + CH3, Assertion.
C–hot water, D–vacuum, E–U trap (c) If the Assertion is correct but Reason is
(c) A–electrodes, B–NH3 + H2O, C–hot water, incorrect.
D–tap, E–U trap
(d) If both the Assertion and Reason are incorrect.
(d) A–electrodes, B–NH3 + H2 + H2O + CH4,
C–steam, D– vacuum, E–U trap (e) If the Assertion is incorrect but the Reason is
14. A population is in Hardy- weinberg equilibrium correct.
for a gene with only two alleles. If the gene 17. Assertion: We have lost all the direct evidence
frequency of an allele A is 0.7, the genotype of origin of life.
frequency of Aa is [2014, 2016]
Reason: The persons responsible for protecting
(a) 0.21 (b) 0.42
evidences were not skilled. [1998]
(c) 0.36 (d) 0.7
18. Assertion : Ginkgo biloba is a living fossil.
15. According to Hardy-Weinberg principle, allele
and genotype frequencies in a population will Reason : Organism which have persisted and
remain constant from generation to generation remain unchanged for the past several million
in the absence of other evolutionary influences. years while their relatives disappeared. [2000]
It makes several assumptions which were given 19. Assertion : Among the primates, chimpanzee is
below. [2017] the closest relative of the present day humans.
i. Random Mating Reason : The banding pattern in the autosome
numbers 3 and 6 of man and chimpanzee is
ii. Sexual Reproduction
remarkably similar. [2004]
iii. Non-overlapping Generations 20. Assertion : From evolutionary point of view,
iv. Occurrence of Natural Selection human gestation period is believed to be
v. Small size of population shortening.
Identify two assumptions which do not meet for Reason : One major evolutionary trend in
a population to reach Hardy-Weinberg humans has been the larger head undergoing
Equilibrium? relatively faster growth rate in the foetal stage.
(a) iv and v (b) ii and iv [2004]
21. Assertion : Coacervates are believed to be the
(c) iii, iv and v (d) i, ii and iii
precursors of life.
16. Which of the following was most similar to
Reason : Coacervates were self-duplicating
modern man? [2017]
aggregates of proteins surrounded by lipid
(a) Java man
molecules. [2004]
(b) Neanderthal man
22. Assertion : Human ancestors never used their
(c) Homo habilis
tails and so the tail expressing gene has
(d) Cro-Magnon man
disappeared in them.
TYPE B : ASSERTION REASON QUESTIONS Reason : Lamarck's theory of evolution is
Directions for (Qs. 17-26) : These questions consist popularly called theory of continuity of germ
plasm. [2005]
of two statements, each printed as Assertion and
Reason. While answering these questions, you are 23. Assertion : Comparative biochemistry provides
a strong evidence in favour of common ancestory
required to choose any one of the following five
of living beings.
responses.
Reason : Genetic code is universal. [2005]
(a) If both Assertion and Reason are correct and
24. Assertion : Darwin's finches show a variety of
the Reason is a correct explanation of the
beaks suited for eating large seeds, flying
Assertion.
insects and cactus seeds.
Evolution B-157

Reason : Ancestral seed-eating stock of Darwin's 27. Assertion : The primitive atmosphere was
finches radiated out from South American reducing one i.e., without oxygen. [2009]
mainland to different geographical areas of the Reason : In the primitive atmosphere, oxygen
Galapagos Islands, where they found was involved in forming ozone.
competitor-free new habitats. [2005]
28. Assertion : Jave Ape-man, Peking man and
25. Assertion : The earliest organisms that appeared Heidelberg man are the fossils of Homo erectus.
on the earth were non-green and presumably
anaerobes. Reason : Homo erectus evolved from Homo
habilis. [2009]
Reason :The first autotrophic organisms were
the chemoautotrophs that never released 29. Assertion : Natural selection is the outcome of
oxygen. [2006] difference in survival and reproduction among
individuals that show variation in one or more
26. Assertion : The earliest fossil form in the traits.
phylogeny of horse is eohippus.
Reason : Adaptive forms of a given trait tend to
Reason : Eohippus lived during the early become more common; less adaptive ones
pliocene epoch. [2007] become less common or disappear. [2004, 2012]
Directions for (Qs.27-30) : Each of these questions 30. Assertion : Organic compounds first evolved
contains an Assertion followed by Reason. Read them in earth required for origin of life were protein
carefully and answer the question on the basis of and nucleic acid.
following options. You have to select the one that Reason : All life forms were in water environment
best describes the two statements. only. [2016]
(a) If both Assertion and Reason are correct and
Reason is the correct explanation of Assertion.
(b) If both Assertion and Reason are correct, but
Reason is not the correct explanation of
Assertion.
(c) If Assertion is correct but Reason is incorrect.
(d) If both the Assertion and Reason are incorrect.
EBD_7100
B-158 Topicwise AIIMS Solved Papers – BIOLOGY

Type A : Multiple Choice Questions birds of that Island. He studied the finches
1. (c) Eugenics deals with factors related to the there, which were called Darwin’s finches.
improvement of human race by controlled The finches showed differences in their
selective breeding. It is the improvement beaks based on their different feeding
of human race by improvin g body habits and are examples of natural selection.
functions and treatment of defective Darwin’s Finches are good examples of
heredity by genetic engineering. Euthenics adaptive radiation ( also called divergent
deals with the improvement of human race evolution). Different types of beaks in these
by providing better conditions of life. Finches show adaptive radiation.
Obstetrics is the study of the reproductive 9. (c) Wings of birds, insects, mammals are
process within the female body including analogous organs i.e. have same function
fertilization, pregnancy and childbirth. and different structure. The similarity
2. (a) Neoplina is a connecting link between developed in distantly related groups is an
annelids and molluscs. adaptation for the same function. So it is
analogy or convergent evolution.
3. (c) The theory of Pangenesis was given by
Darwin. 10. (d) According to Lamarck, during the lifetime
of an organism new characters are
4. (d) The cranial capacity of Neanderthal man developed due to internal vital forces, effect
was the largest. The cranial capacity was of environment, new needs, use and disuse
about 1450 cc, roughly equal to that of of organs. These acquired characters are
modern man. inherited from one generation to another.
5. (c) Atavism is the reappearance of ancestral According to him, intelligence should also
and not parental characters in an organisms be inherited and so it fails to explain dull
which do not occur normally. A baby that progeny of nobel laureate .
has been born with a small tail it is exhibiting 11. (b) Thorn of Bougainvillea and tendril of
the case of atavism. Atavism is the sudden Cucurbita are examples of homologous
appearance of some ancestor characters chromosomes.
which are lost during course of evolution.
12. (c) Organs which have a common fundamental
6. (b) Homo sapiens imply human beings.
anatomical plan and similar embryonic
Humans belong to an order of mammals
origin whatever various functions they
called primates and placed in the family
may perform are regarded as homologous
hominidae. Hominidae includes modern
organs. For example, the flippers of a whale,
and archaric human beings and also
a bat's wing, fore-limb of a horse, a bird's
consists of neanderthal, a sub species of
wing and forelimbs of human ar e
Homo sapiens. Humans and their ancestors
structurally as well as functionally
are more closely related to the apes.
different.
7. (d) Comparative cytology is the field of study 13. (a)
involving examination of similarities in cells
14. (b) For a gene with two alleles, A (dominant)
of different organisms Biochemistry
and a (recessive), if the frequency of A is p
compares DNA and proteins made from
and the frequency of a is q, then the
DNA. Geology is the study of the earth.
frequencies of the three possible
8. (b) During voyage on the ship M.S. Beagle in genotypes (AA, Aa, and aa) can be
1931, Darwin visited Galopagos Islands of expressed by the Hardy-Wein berg
South America and studied the climate and
Evolution B-159

equation : Complete development will not take place


before the gestation period of 270 – 290
p2 2 pq q 2 1 days in humans.
where, p2 = frequency of AA (homozygous 21. (d) Coacervates are large colloidal aggregates
dominant) individuals, 2pq = frequency of formed due to intermolecular attraction from
Aa (heterozygous) individuals and large organic molecules synthesized
q 2 = frequen cy of aa (homozygous abiotically on primitive earth. It mainly
recessive) individuals. The equation can consists of proteins, polysaccharides and
be used to calculate allele frequencies if water. They do not fulfil the requirement
the numbers of homozygous recessive for probable precursors of life.
individuals in the population is known.
A coacervate is a tiny spherical droplet of
Here, p = 0.7 and q = 0.3 (given) assorted organic molecules which is held
2pq (frequency of heterozygote) together by hydrophobic forces from a
= 2 × 0.7 × 0.3 = 0.42 surrounding liquid. Coacervates possess
15. (a) Occurrence of natural selection and small osmotic properties and form
size of population do not meet the criteria spontaneously from certain dilute organic
for a population to reach Hardy-Weinberg solutions. They were even once suggested
Equilibrium. For Hardy-Weinberg to have played a significant role in the
equilibrium to be reached, natural selection evolution of cells and, therefore, of life
should not be occurring. If populations are itself. They are interesting not only in that
undergoing natural selection at the locus they provide a locally segregated
under consideration, allele frequencies will environment but also in that their
be continuously changing in a specific boundaries allow the selective absorption
direction and Hardy-Weinberg Equilibrium of simple organic molecules from the
predicts that allele frequencies will stay surrounding medium. Coacervates do not
constant. It assumes that population size have lipid outer membrane, hence they
is very large. cannot reproduce.
16. (d) The skeleton of Cro-Magnon was almost 22. (c) According to Lamarck’s theory, continuous
identical to the modern man. disuse of organs makes them weak. The
Type B : Assertion Reason Questions theory of continuity of germplasm was
given by Weismann.
17. (c) We have lost all the direct evidences of
If humans share ancestry with other
origin of life only due to destruction of
primates such as prosimian, monkeys etc,
fossils on account of climatic changes and
then remnants of that common ancestory
not due to any person/(s).
should be present in our genes.
18. (a) Ginkgo biloba, belong to the Ginkgoales
evolved during the Jurassic period and is 23. (b) Comparative biochemistry provides a
still existing without any morphological or strong evidence for common ancestors of
anatomical changes. Hence, both assertion living beings (e.g. proteins lymph,
and reason are correct. enzymes, hormones, blood groups etc.)
19. (a) The banding pattern seen on stained 24. (a) Darwin finches found on Galapagos
chromosomes from humans and islands differ primarily in body size, feather
chimpanzee show striking similarities which colour, bill shape as adaptation to type of
indicates that they have evolutionary food available. It is a type of divergent
relationships (cytogenetic evidence). evolution.
20. (d) The human gestation period cannot be 25. (b) The earliest organisms that appeared on
shortened unless there is really a problem. the earth were heterotrophic because of
EBD_7100
B-160 Topicwise AIIMS Solved Papers – BIOLOGY
reducing atmosphere and the first 28. (b) The fossil of Java Ape-man was discovered
autotrophs were chemoautotrophs. from pleistocene rocks in central Java. The
fossil of Peking man was discovered from
An anaerobic organism does not require
the lime stone caves of Choukoution near
oxygen for growth and may even die in its
Peking while that of Heidelberg man was
presence. Chemotrophs are the first
discovered in mid pleistocene. All these three
organisms that appeared on earth & obtain
fossils come under the category of Homo
energy by the oxidation of electron
erectus. Homo erectus appeared about 1.7
donating molecules in their environments.
million years ago in the middle pleistocene.
These molecules can be organic
H. erectus evolved from Homo habilis. He
(organotrophs) or inorganic (lithotrophs).
was about 1.5-1.8 metres tall. He had erect
The chemotrophs utilize solar energy and
posture. His skull was flat than that of the
can be either autotrophic or heterotrophic.
modern man. He had protruding jaws,
26. (c) Eohippus is the earliest fossil form in the projecting brow ridges, small canines and
phylogeny of horse. Origin of horse took large molar teeth. He made more elaborate
place in the eocene period. The first fossil tools of stones and bones, hunted big
of horse was found in North America which animals and perhaps knew the use of fire.
was named Eohippus or "Dawn Horse. ",
29. (a) The Darwin's theory of Natural Selection
that later renamed as hyracotherium.
can be generalised as the change in species
It was found during eocene period not by the survival of an organism exhibiting a
during pliocene. natural variation that gives it an adaptive
27. (c) The lightest atoms of nitrogen, carbon etc. advantage in an environment. Thus,
formed the primitive atmosphere. Hydrogen leading to a new environmental equilibrium.
atoms were most numerous and most The idea of the survival of the fittest
reactive in primitive atmosphere. Hydrogen explains the above evolution by natural
atoms combined with all oxygen atoms to selection. According to survival of the
form water leaving no free oxygen. Thus fittest, some of the variations exhibited by
primitive atmosphere was reducing (without living things make it easier for them to
free oxygen) unlike the present oxidizing survive and reproduce. Thus, more
atmosphere (with free oxygen). adaptive forms increase. Those which are
not fit (or less adaptive) are eliminated.
Formation of ozone layer is the
consequen ce of modern oxidizing 30. (b) Organic compounds that first evolved in
atmosphere having plenty of free oxygen. earth which required for origin of life were
As more oxygen accumulated in the protein and nucleic acid. All life forms were
atmosphere (due to photosynthesis) ozone in aquatic environment only.
began to appear in the top layers.
30 Human Health & Diseases

TYPE A : MULTIPLE CHOICE QUESTIONS 10. Sporogony of malarial parasite occurs in [1999]
(a) liver of man
1. Gambusia fish has been introduced in lakes and
(b) RBCs of man
ponds of India to control a deadly disease. It
(c) stomach wall of mosquito
feeds on larva of [1997]
(d) salivary glands of mosquito
(a) nepenthes (b) anopheles
11. Malignant tertain malaria is caused by [2000]
(c) dragon fly (d) house-fly
2. Anti-viral substance is [1997] (a) P. vivax (b) P. malariae
(a) antigen (b) antibody (c) P. ovale (d) P. falciparum
(c) interferon (d) antibiotic 12. HIV has a protein coat and genetic material
3. Which malarial parasite has longest incubation [2000]
period? [1997] (a) ss RNA (b) ds RNA
(a) Plasmodium vivax (c) ss DNA (d) ds DNA
(b) Plasmodium falciparum 13. Cyclosporine is used [2002]
(c) Plasmodium malariae (a) For allergy
(d) Plasmodium ovale (b) As immunodepressent
4. The type of antibodies present in colostrum (c) Prophylactic for virus
secreted from mammary gland is [1997] (d) None of the above
(a) IgM (b) IgD 14. Lysis of foreign cell is mediated through [2002]
(c) IgE (d) IgA (a) IgM (b) IgA
5. Which of the following disease is due to an (c) IgE (d) IgM & IgG
allergic reaction ? [1998] 15. The treatment of snake-bite by antivenom is an
(a) Goitre (b) Hay fever example of [2004]
(c) Skin cancer (d) Rheumatic fever
(a) artificially acquired active immunity
6. Influenza is caused by [1998]
(b) artificially acquired passive immunity
(a) virus (b) bacteria
(c) naturally acquired passive immunity
(c) alga (d) fungus
(d) specific natural immunity
7. Which type of cancer is found in lymph nodes
and spleen? [1998] 16. Electron beam therapy is a kind of radiation
(a) Carcinoma (b) Sarcoma therapy to treat [2004]
(c) Lymphoma (d) Leukemia (a) enlarged prostate gland
8. Amoebiasis is caused by [1999] (b) gall bladder stones by breaking them
(a) Entamoeba histolytica (c) certain types of cancer
(b) Taenia solium (d) kidney stones
(c) Plasmodium vivax 17. A young drug addict used to show symptoms
(d) E. coli of depressed brain activity, feeling of calmness,
9. Inflammatory response, in allergy is caused by relaxation and drowsiness. Possibly he was
the release of [1999] taking [2005]
(a) antigen (b) histones (a) Amphetamine (b) Marijuana
(c) histamines (d) antibodies (c) Pethadine (d) Valium
EBD_7100
B-162 Topicwise AIIMS Solved Papers – BIOLOGY
18. When children play bare footed in pools of dirty 25. Which of the following is a pentameric
water and flood water, they may suffer from immunoglobulin and is produced first in a
diseases like [2006] primary response to an antigen? [2010]
(a) leptospirosis and bilharizia (a) Ig G (b) Ig M
(b) malaria, amoebic dysen tery and (c) IgA (d) Ig E
leptospirosis 26. Cattle fed with spoilt hay to sweet clover which
(c) bilharizia, infective hepatitis and diarrhoea contains dicumarol [2011]
(d) guinea worm infection, elephantiasis and (a) are healthier due to a good diet
amoebic dysentery (b) catch infections easily
19. Which one of the following is not a matching (c) may suffer vitamin K deficiency and
pair of a drug and its category ? [2004, 2008] prolonged bleeding
(a) Amphetamines - stimulant (d) may suffer from beri-beri due to deficiency
(b) Lysergic acid - narcotic of vitamin-B
dimethyl amide 27. Opium is obtained from [2011]
(c) Heroin - psychotropic (a) Oryza sativa
(d) Benzodiazepam - pain killer (b) Coffea arabica
20. An insect bite may result in inflammation of that (c) Thea sinensis
spot. This is triggered by the alarm chemicals (d) Papaver somniferum
such as [2005, 2008] 28. Match the following bacteria with the diseases
(a) histamine and dopamine Column-I Column-II
(b) histamine and kinins A. Treponema pallidum I. Plague
(c) interferons and opsonin B. Yersinia pestis II. Anthrax
(d) interferons and histones C. Bacillus anthracis III. Syphilis
21. Antigen binding site in an antibody is found D. Vibrio IV. Cholera
between [2005, 2008] [2012]
(a) two light chains (a) A – III; B – I; C – II; D – IV
(b) two heavy chains (b) A – IV; B – I; C – II; D – III
(c) one heavy and one light chain (c) A – III; B – II; C – I; D – IV
(d) either between two light chains or between (d) A – I; B – III; C – II; D – IV
one heavy and one light chain depending 29. Which one of the following is a correct match?
upon the nature of antigen [2013]
22. The antigen-binding site are present where on (a) Bhang – Analgesic
the antibody molecule [2009]
(b) Cocaine – Opiate narcotics
(a) on light chain as well as on heavy chain.
(c) Morphine – Hallucinogen
(b) on light chain only.
(d) Barbiturate – Tranquiliser
(c) on variable region and constant region of
30. Which of the following is an autoimmune
light chain.
disorder? [2013]
(d) on heavy chain only.
23. Which one of the following antimicrobial drugs (a) Myasthenia gravis
is suitable for treatment of both tuberculosis and (b) Osteoporosis
leprosy? [2010] (c) Muscular dystrophy
(a) Isoniazid (d) Gout
(b) R-aminosalicylic acid 31. Which of the following is based upon the
(c) Streptomycin principle of antigen-antibody interaction?
(d) Rifampicin [2014]
24. Antigen is a substance which [2010] (a) PCR
(a) lowers body temperature (b) ELISA
(b) destroys harmful bacteria (c) r-DNA technology
(c) triggers the immune system (d) RNA
(d) is used as an antidote to poison
Human Health & Diseases B-163

32. Identify the molecules (A) and (B) shown below 36. Which one of the following pairs of diseases is
and select the right option giving their source viral as well as transmitted by mosquitoes?
and use. [2014,2015] [2017]
(a) Elephantiasis and dengue
CH3
N O
(b) Yellow fever and sleeping sickness
CH3 (c) Encephalitis and sleeping sickness
(A) O (d) Yellow fever and dengue
H
O TYPE B : ASSERTION REASON QUESTIONS
H Directions for (Qs. 37-44) : These questions consist
O
of two statements, each printed as Assertion and
OH
Reason. While answering these questions, you are
required to choose any one of the following five
responses.
(B) (a) If both Assertion and Reason are correct and
O
H the Reason is a correct explanation of the
Assertion.
(b) If both Assertion and Reason are correct but
Molecule Source Uses
Reason is not a correct explanation of the
(a) (A) Cocaine Erythroxylum Accelerates Assertion.
coca the transport
(c) If the Assertion is correct but Reason is
of dopamine
incorrect.
(b) (B) Heroin Cannabis Depressant
(d) If both the Assertion and Reason are incorrect.
sativa and slows
(e) If the Assertion is incorrect but the Reason is
down body
correct.
functions
(c) (B) Cannabinoid Atropa Produces 37. Assertion: There is no chance of malaria to a
hallucinations
man on the bite of male Anopheles mosquito.
belladona
(d) (A) Morphine Papaver Sedative and Reason: It carries a non-virulant strain of
pain killer
Plasmodium. [1998]
somniferum
38. Assertion : Rabies is an infection of mammals,
33. Pasteurella/Yersinia pestis (causal agent of it involves central nervous system which may
Bubonic Plague) is transmitted by [2016] result in paralysis and finally death.
(a) Cimex (b) Xenopsylla Reason : This is caused by neurotropic bacteria
(c) Pediculus (d) Aedes in saliva of rabies animal. [2000]
34. Which of the following pairs is not correctly 39. Assertion : Plasmodium vivax is responsible
matched ? [2016] for malaria.
(a) Cholera – Vibrio cholerae Reason : Malaria is caused by polluted water.
(b) German measles – Rubella virus [2001]
(c) Whooping cough – Bordetella pertussis 40. Assertion: Histamine is related with allergic and
(d) Tetanus – Pasteurella pestis inflammatory reactions.
35. Human immuno deficiency virus (HIV) has a
Reason: Histamine is a vasodilator. [2002]
protein coat and a genetic material which is
41. Assertion : Organ transplantation patients are
[2017]
given immunosuppressive drugs.
(a) Single stranded DNA.
(b) Single stranded RNA. Reason : Transplanted tissue has antigens
(c) Double stranded RNA. which stimulate the specific immune response
(d) Double stranded DNA. of the recipient. [2005]
EBD_7100
B-164 Topicwise AIIMS Solved Papers – BIOLOGY
42. Assertion: LSD and marijuana are clinically used Reason : Allergens in the environment on
as a analgesics. [2006] reaching human body stimulate mast cells in
Reason:Both these drugs suppress brain certain individuals. [2003, 2008]
function. 49. Assertion : Cannabis sativa is a powerful
43. Assertion : A person who has received a cut anti-depressant.
and is bleeding needs to be given anti-tetanus Reason : Hashish and Marijuana are derived
treatment. [2006] from it. [2009]
Reason : Anti-tetanus injection provides
50. Assertion: Epstein– Barr virus is an oncovirus.
immunity by producing antibodies for tetanus.
44. Assertion (A) : Antigen can be easily recognized Reason: It stimulates the growth of cancer.
because it has antigenic determinants. [2011]
Reason (R) : The recognition ability is innate. 51. Assertion: HIV infected person are prone to
[2007] oppurtunistic diseases.
Directions for (Qs.45-57) : Each of these questions Reason: Immune system weakens during HIV
contains an Assertion followed by Reason. Read them infection. [2011]
carefully and answer the question on the basis of 52. Assertion : Histamine is involved in allergic and
following options. You have to select the one that inflammatory reactions.
best describes the two statements. Reason : Histamine is a vasodilator. [2012]
(a) If both Assertion and Reason are correct and 53. Assertion : The antibodies separted from serum
Reason is the correct explanation of Assertion. are homogenous.
(b) If both Assertion and Reason are correct, but Reason : Monoclonal antibodies are
Reason is not the correct explanation of homogenous immunological reagents. [2013]
Assertion. 54. Assertion : Interferons are effective against
(c) If Assertion is correct but Reason is incorrect. viruses.
(d) If both the Assertion and Reason are incorrect. Reason : Proteins which can be synthesized only
45. Assertion : Escherichia coli, Shigella sp. and by genetic engineering are effective against
Salmonella sp. are all responsible for diarrhoeal viruses. [2013, 2014]
diseases. 55. Assertion : Cancer cells are virtually immortal
Reason : Dehydration is common to all types of until the body in which they reside dies.
diarrhoeal diseases and adequate supply of Reason : Cancer is caused by damage to genes
fluids and electrolytes should be ensured. regulating the cell division cycle. [2006, 2015]
[2008] 56. Assertion : Cocaine has a potent stimulating
46. Assertion : Dope test is used to estimate the action on central nervous system, producing a
level of blood alcohol by analyzing the breath sense of euphoria and increased energy.
of persons drinking alcohol. Reason : Injecting the microbes intentionally
during immunisation or infectious organisms
Reason : A drunken person usually feels tense
gaining access into body during natural infection
and less talkative. [2004, 2008]
induces active immunity. [2016]
47. Assertion : Interferons are a type of'antibodies
57. Assertion: Artificially acquired passive immunity
produced by body cells infected by bacteria.
results when antibodies or lymphocytes produced
Reason : Interferons stimulate inflammation at
outside the host are introduced into a host.
the site of injury. [2004, 2008]
Reason: A bone marrow transplant given to a
48. Assertion : Mast cells in the human body release
patient with genetic immunodeficiency is an
excessive amounts of inflammatory chemicals,
example of artificially acquired passive immunity.
which cause allergic reactions.
[2017]
Human Health & Diseases B-165

Type A : Multiple Choice Questions 14. (c) IgE immunoglobulin acts as mediator in
allergic response.
1. (b) Gambusia fish is larvicidal i.e. feed on insect 15. (b) Artificial passive immunity is the resistance
larva. They are introduced into lakes and passively transferred to a recipient by
ponds to eat mosquito larvae (e.g. administration of antibodies, e.g.
Anopheles) and control malaria. administration of antivenom.
2. (c) Interferons are glycoproteins produced by 16. (c) Electron beam therapy is a kind of radiation
viral infected cell. They make the cells therapy to treat certain types of cancer. It
resistant to viral infection. is used as a therapeutic treatment for cancer.
3. (c) Incubation period of Plasmodium vivax is 17. (d) Valium is the anti depressant used by
14 days, P. malariae is 30 days, P. ovale is addicts that produces feelings of calmness,
14 days, and P. falciparum is 12 days. relaxation and drowsiness.
4. (d) Colostrum (also called first milk) is the thin 18. (a) In leptospirosis and bilhariziasis diseases,
yellowish fluid secreted by the mammary cercaria of the fluke penetrates the human
glands at the time of parturition and which body through the skin during bath in rivers
precedes the production of true milk. It and canals.
provides a nursing infant with essential
19. (d) Amphetamine is pap pill/antisleep drug
nutrients and infection-fighting antibodies
which stimulates CNS. Lysergic acid
(called immunoglobulin). IgA is the major
diethylamide (LSD) is the most potent
immunoglobulin in colostrum. It provides
hallucinogen. Benzodiazepam is a sedative
protection from inhaled and ingested
which switches of the CNS and is
pathogens.
antianxiety. Heroin is a psychotropic drug
5. (b) Hay fever is due to allergic reaction.
which acts on the brain and alters behavior,
6. (a) Influenza is caused by virus Myxovirus
consciousness and capacity of perception.
influenzae. It is an acute respiratory tract
infection. 20. (b) An insect bite may result in inflammation of
7. (c) Cancer of lymphatic tissue i.e. lymph that spot. This is triggered by the alarm
nodes, spleen, is lymphoma. chemicals such as histamine and kinins.
8. (a) Amoebiasis or amoebic dysentry is caused Histamine is a biogenic amine involved in
by Entamoeba histolytica. local immune responses as well as regulating
9. (c) Inflamatory response is due to the release physiological function in the gut and acting
of histamines by the damaged mast cells. as a neurotransmitter. It is found in virtually
The vascular fluid comes out of the blood all animal body cells. New evidence also
vessels causing swelling of the region. indicates that histamine plays an important
10. (d) Sporogony of Plasmodium occurs in the role in chemotaxis of white blood cells. Kinin
salivary glands of female Anopheles. is any of various structurally related
11. (d) Malignant tertain malaria is caused by polypeptides, such as bradykinin and
Plasmodium falciparum. kallikrein. They are members of the autacoid
12. (a) HIV (AIDS virus) consists of glycoprotein family. They act locally to induce
coat, double layer of lipid membrane of two vasodilation and contraction of smooth
protein coats. It contains ss RNA and muscles.
reverse transcriptase. 21. (c) Antigen binding site is Y-shaped showing
13. (b) Cyclosporin is used as immunodepressant lock and key pattern made up of one heavy
especially for the patients having organ and one light chain.
transplantation. It suppresses T-lymphocytes 22. (a) The Y-shaped antibody molecule is
activity in the immune response. composed of 4 polypeptide chains - two
EBD_7100
B-166 Topicwise AIIMS Solved Papers – BIOLOGY
identical light chains and two heavy chains allows for easy visualization of results.
held together by disulphide bonds. 32. (d) Molecule (A) represents structure of
Antigen-binding site is formed on antibody morphine. Morphine is the most abundant
molecule where a heavy chain variable and alkaloid found in Opium, the dried sap
a light chain variable come closer. (latex) derived from shallowly slicing the
23. (d) Leprosy is caused by Mycobacterium unripe seedpods of the Opium, or common
leprae, while tuberculosis is caused by and/or edible poppy Papaver somniferum.
bacteria Streptococcus or Staphylococcus. Morphine is a potent opiate analgesic drug
Rifampicin is a common medicine in that is used to relieve severe pain.
treatment of both the diseases.
33. (b)
24. (c) Antigen is the only foreign substance
34. (d) Tetanus is caused by Clostridium tetani
which triggers immune system by way of
while plague is caused by Pasteurella
antibody formation. These antibodies
pestis.
neutralize antigen by antigen-antibody
reaction. 35. (b) The human immunodeficiency virus is a
lentivirus that causes the acquired
25. (b) IgM is a pentameric immunoglobulin that
immunodeficiency syndrome, a condition
is produced first in a primary response to
in humans in which progressive failure of
an antigen. IgA, IgD and IgE consist of
the immune system allows life-threatening
single monomers.
opportunistic infections and cancers to
26. (c) Hemorrhagic syndrome of cattle are
thrive. HIV has a protein coat and a genetic
associated with the feeding of sweet clover
material which is single stranded RNA.
hay containing dicumarol.
27. (d) Opium is a narcotic formed from the latex 36. (d) Yellow fever and dengue are viral diseases,
released by lacerating the immature seed and they are transmitted by mosquitoes.
pods of opium poppies (Papaver Type B : Assertion Reason Questions
somniferum). Morphine is the most preva-
lent alkaloid in opium, about 10%-16% of 37. (c) Male Anopheles mosquito do not have
the total alkaloids. It is responsible for most piercing and sucking type of mouth parts.
of the harmful effects such as lung edema, So, they can not inject malarial parasite into
respiratory difficulties, coma, or cardiac or man.
respiratory collapse. 38. (c) Rabies (hydrophobia) is caused by rabies
28. (a) Plague is a deadly infectious disease that virus. Its vectors are raboid animals
is caused by the enterobacteria Yersinia especially dogs. It leads to encephalitis,
pestis (formely know as Pasteurella pestis). fear of water (hydrophobia), high fever,
Anthrax is an infectious bacterial disease severe headache, spasm of throat & chest,
which involves skin, gastrointestinal tract leading to death.
or lungs. Syphilis is a sexually transmitted 39. (b) Malaria is caused by Plasmodium whose
infection caused by bacterium Treponema sexual phase occurs in the mosquito
pallidium. Cholera is an infection in the Anopheles. When female Anopheles feed
small intestine caused by the bacterium on blood, they can serve as vector host for
Vibrio cholerae that causes a large amount malarial parasite.
of watery diarrhea and vomiting. 40. (a) Histamine is produced by mast cells in
29. (d) response to the allergy and inflammatory
30. (a) reaction. When histamine is released from
mast cells, it causes vasodilation and an
31. (b) The ELISA is a fundamental tool of clinical increase in permeability of the blood vessel
immunology, and is used as an initial screen walls. These effects, in turn cause the
for HIV detection. Based on the principle common symptoms of allergy including a
of antigen- antibody interaction, this test running nose and watering eyes.
Human Health & Diseases B-167

41. (a) Success of organ transplant depends on 46. (d) Blood alcohol test measures the amount of
proper matching of histocompatibility of alcohol (ethanol) in the body. Alcohol is
antigens that occurs in all cells of the body. quickly absorbed into the blood and can
As there are antigens which are likely to be be measured within minutes of having an
attacked by recipient's T-cells and alcoholic drink. The amount of alcohol in
antibodies, the recipient of organ transplant the blood reaches its highest level about
is always given immunosuppressants to an hour after drinking. But food in the
prevent immune rejection of th e stomach may increase the amount of time
transplanted tissue. it takes for the blood alcohol to reach its
42. (c) LSD and marijuana are anti inflammatory, highest level. About 90% of alcohol is
sedative, anticonvulsive and laxative in broken down in the liver. The rest of it is
action. passed out of the body in urine and your
43. (c) Anti tetanus is a disinfectant i.e. it prevents exhaled breath. Symptoms of alcohol
the infection due to the entry of bacteria intoxication include confusion, lack of
through wounds. coordination, unsteadiness that makes it
44. (b) Antigen is a substance that when hard to stand or walk, or erratic or unsafe
introduced in the body, stimulate the driving. Dope test is not related to alcohol.
production of antibody. They are mostly Dope test is a blood test to know whether
proteins but may be carbohydrates, lipids, a person used any drug to improve their
nucleic acids etc. One antigen can bind with performance.
many antibodies. Every antigen has many 47. (d) Interferons are natural proteins produced
antigenic determinants called epitopes. by the cells of the immune system in
The recognition ability of antibody is response to challenges by foreign agents
innate and develops without exposure to such as viruses, parasites and tumor cells.
the antigen. Interferons belong to the large class of
45. (b) Diarrhoeal disease conditions include glycoproteins known as cytokines.
frequent and excessive discharge of watery Interferons are produced by a wide variety
material from the bowel. Such diseases mostly of cells in response to the presence of
result from ingestion of harmful germs with double-stranded RNA, a key indicator of
food and water. E. coli, Shigella sp. & viral infection. Interferons assist the immune
response by inhibiting viral replication
salmonella sp. causes diarrhoea. Diarrhoea
within host cells, activating natural killer cells
caused by virus, bacteria or parasites
and macrophages, increasing antigen
possesses two characteristics- firstly, the
presentation to lymphocytes, and inducing
offending organisms colonise the intestine
the resistance of host cells to viral infection.
and as a consequence cause inflammation When the antigen is presented to matching
of the intestine or enteritis; and secondly, T and B cells, those cells multiply and
they upset the balance of intestinal fluid strategically and specifically wipe out the
absorption and secretion mechanism, often foreign substance. That is why antigen
enhancing the latter very considerably, presentation is so important to the immune
which is then manifested as watery stool response.
discharged frequently in large volumes. 48. (a) A mast cell contains many granules rich in
Shigella sp, Salmonella sp. are quite histamine serotonin and heparin. Although
closely related genera that are responsible best known for their role in allergy and
for diarrhoeal diseases. Dehydration is anaphylaxis, mast cells play an important
common to all types of diarrhoeal diseases protective role as well, being intimately
& adequate supply of fluids & electrolytes involved in wound healing and defense
that provides ions, should be ensured. against pathogens. The symptoms of an
allergic reaction develop in response to
EBD_7100
B-168 Topicwise AIIMS Solved Papers – BIOLOGY
histamine. Mast cells release a large amount vertebrates. When released, it has the effect
of histamine into the blood stream & it also of dilating capillaries and lowering blood
act as initiator of the inflammatory response, pressure. Histamine is involved in allergic
which aids the arrival of leucocytes at a site and inflammatory reactions also.
of infection. Histamine stimulates capillary 53. (d) From hyperimmunized animals, the blood
dilation increased capillary permeability, serum may be taken and antibodies may be
closure of bronchial tubes, mucus secretion, isolated from this serum. However, the
pain & swelling. antibodies, whenever separated from serum
49. (d) Cannabis sativa or Hemp is an annual herb after induction due to an antigen, are
of cannabaceae family. It is mainly the source usually heterogenous, because the cells
of fibre. Cannabis sativa is not an keep on producing a variety of antibodies.
antidepressant. Antidepressant is a Monoclonal antibodies (Mabs), on the
psychiatric medication used to alleviate other hand are homogeneous immunological
mood disorders such as major depression. reagents of defined specificity so that these
Hashish and Marijuana are drugs derived can be utilized for diagnosis and screening
from it. Preparations of flowers of Cannabis of disease.
is called marijuana while preparations of the
54. (c) Interferons are proteins that are effective
resinous extract is called hashish. Both are
against most viruses. They are naturally
consumed by smoking, vapourising and oral
produced by virus infected cells. The
ingestion and are hallucinogen drugs that
proteins interact with adjacent cells and
cause acute panic anxiety reaction.
make them resistant to virus attack. Now
50. (a) Oncoviruses are cancer – causing viruses interferons are also being manufactured
and may be DNA or RNA virus e.g., Epstein through genetic engineering.
– Barr – Virus, Herpes simplex type 2 virus
etc. This shows that the development of Interferons control the multiplication of
cancerous tumour is associated with virus particles by inhibiting their protein
certain genes. synthesis.
51. (a) Usually, person shows symptoms of HIV 55. (b) Cancer cells divide and redivide mitotically
infection within 2 to 6 weeks of exposure to and is due to the activation of pro-
the virus. But in some persons, the virus oncogenes.
may remain silent for long periods (upto 10 56. (c) Cocaine is obtained from the cocaplant,
years) before symptoms of full blown AIDS erthyroxylum coca. Cocaine is commonly
are observable. The symptoms of HIV called as coke or crack which is usually
infection include fever, lethargy, pharyngitis, snored. It has a potent stimulating action
nausea, headache, rashes etc. Persons of euphoria and increased energy. It
suffering from AIDS have a weakened interferes with the transport of the
immune system due to depletion of T- helper neurotransmitter dopamine. Its increased
cells. Such persons show opportunistic dosages cause hallucinations.
infections, i.e., infection by those fungi,
57. (b) Artificially acquired passive immunity
bacteria and viruses to which a person with
results when antibodies or lymphocytes
normal immune system is expected to be
that have been produced outside the host
resistant. Therefore, persons prone to
are introduced into a host. This type of
opportunistic infections may be suspected
immunity is immediate short lived, lasting
to be infected by HIV, particularly, if the
only a few weeks to a few months. An
count of T helper cells in 200/ ml or lower.
example is bone marrow transplant given
52. (a) Histamine is a derivative of the amino acid
to a patient with genetic immunodeficiency.
histidine produced by damaged cells of
Strategies for Enhancement in
31 Food Production
TYPE A : MULTIPLE CHOICE QUESTIONS increasing amounts of cholesterol. Results
obtained are shown in the given graph.
1. The chemical which are produced by host plants [2005, 2008]
due to infection as a defence reaction to
pathogen, are called [1997]
(a) phytotoxin (b) toxin

(Khapra beetle) mg
(c) phytotron (d) phytoalexins

Wt of insect larva
2. Auxanometer is used to measure [1998] 2
(a) length (b) respiration
(c) transpiration (d) ascent of sap
3. Crop rotation is used to increase [1999] 1
(a) soil fertility
(b) pore size and soil particle
(c) organic content of soil O
(d) viscosity of soil water 1 2 3 4 5
µg cholesterol/g basal diet
4. Haploid cultures can be obtained by culturing
(a) pollen grains (b) embryo [2000] The graph indicates
(c) shoot apex (d) root apex (a) cholesterol is an essential dietary
5. Azolla is used as a biofertilizer because it[2003] requirement of khapra beetle.
(a) multiplies very fast to produce massive biomass (b) growth of khapra beetle is directly
(b) has association of nitrogen-fixing Rhizobium proportional to cholesterol concentration.
(c) has association of nitrogen-fixing Cyanobacteria (c) cholesterol concentration of 2 µg/g diet is
(d) has association of mycorrhiza the optimum level.
6. Pruning of plants promotes branching because (d) growth of khapra beetle is inhibited when
the axillary buds get sensitized to [2004] cholesterol concentration exceeds 5 µg/g
(a) ethylene (b) gibberellin diet.
(c) cytokinin (d) indole acetic acid 10. A scion is grafted to a stock. The quality of
7. Somaclonal variation can be obtained by[2004] fruits produced will be determined by the
(a) application of colchicine genotype of [2006]
(b) irradiation with gamma rays
(a) stock
(c) tissue culture
(d) hybridisation (b) scion
8. Somaclonal variation appears in [2005] (c) both stock and scion
(a) organisms produced through somatic (d) neither stock nor scion
hybridization. 11. Cocoa is the plant from which chocolate is made.
(b) plants growing in highly polluted Which part is used to extract it? [2007]
conditions. (a) Flower (b) Fruit
(c) apomictic plants. (c) Seeds (d) Bark
(d) tissue culture raised plants. 12. Bean seeds were planted and put on a sunny
9. In an experiment freshly hatched larvae of an windowsill. As the plants grew, their stems bent
insect (Khapra beetle) were reared on a basal toward the window. This bending was most likely
diet (complete diet without cholesterol) with caused by an [2009]
EBD_7100
B-170 Topicwise AIIMS Solved Papers – BIOLOGY
(a) unequal distribution of auxin in the stem. 20. Which of the following is a viral disease of poultry
(b) unequal distribution of a neurotransmitter birds? [2017]
in the stem. (a) Anthrax (b) Ranikhet
(c) equal distribution of auxin in the stem. (c) Coccidiosis (d) None of these
(d) equal distribution of a neurotransmitter in
TYPE B : ASSERTION REASON QUESTIONS
the stem.
13. Phytotron is a facility to [2010] Directions for (Qs. 21-25) : These questions consist
(a) grow plants under disease-free conditions. of two statements, each printed as Assertion and
(b) conserve endangered species of plants. Reason. While answering these questions, you are
(c) grow plants under controlled conditions. required to choose any one of the following five
(d) induce mutations. responses.
14. Essential oils are those which [2011] (a) If both Assertion and Reason are correct and
(a) are essential to the plant itself the Reason is a correct explanation of the
(b) are used as lubricants Assertion.
(c) produce perfumes (b) If both Assertion and Reason are correct but
(d) are essential for human beings Reason is not a correct explanation of the
Assertion.
15. Coconut water is rich in [2011]
(c) If the Assertion is correct but Reason is
(a) auxins (b) gibberellins
incorrect.
(c) abscisic acid (d) cytokinin
(d) If both the Assertion and Reason are incorrect.
16. Which of the following is the pair of
(e) If the Assertion is incorrect but the Reason is
biofertilizers? [2014]
correct.
(a) Azolla and BGA
21. Assertion : In plant tissue culture somatic
(b) Nostoc and legume
embryos can be induced from any plant cell.
(c) Rhizobium and grasses
Reason : Any viable plant cell can differentiate
(d) Salmonella & E. coli
into somatic embryos. [2003]
17. Hisardale is a new breed of sheep developed in
22. Assertion : Use of fertilizers greatly enhances
Punjab by one of the breeding technique in
crop productivity.
which superior male of one breed is mated with
Reason : Irrigation is very important in increasing
superior females of another breed.
crop productivity. [2003]
Identify the breeding technique from the option
23. Assertion : Fish meal is a rich source of protein
given below. [2016]
(a) Inbreeding (b) Out crossing for cattle and poultry.
(c) Out breeding (d) Cross breeding Reason : Fish meal is produced from non-edible
18. Biodiversity loss occurs due to the reasons parts of fishes like fins, tail etc. [2004]
given below. [2017] 24. Assertion : Cattle breeds can be improved by
(i) Habitat loss and fragmentation super ovulation and embryo transplantation.
(ii) Co-extinction Reason : Superovulation in high milk-yielding
(iii) Over-exploitation cows is induced by hormonal injection. [2004]
(iv) Alien species invasion 25. Assertion (A) : Vernalization is acceleration of
Identify the correct reasons. subsequent flowering by low temperature
(a) (i) and (ii) (b) (i), (ii) and (iii) treatment.
(c) (ii), (iii) and (iv) (d) (i), (ii), (iii) and (iv) Reason (R) : Site of vernalization is apical
19. Explant is required to be disinfected before meristem. [2007]
placing in culture. This is done by [2017] Directions for (Qs. 26-31) : Each of these questions
(a) autoclaving contains an Assertion followed by Reason. Read them
(b) ultra-violet rays carefully and answer the question on the basis of
(c) clorax or hypochlorite following options. You have to select the one that
(d) X-rays best describes the two statements.
Strategies for Enhancement in Food Production B-171

(a) If both Assertion and Reason are correct and Reason : Hybrid vigour is mostly used in
Reason is the correct explanation of Assertion. vegetatively propagated plants. [2013]
(b) If both Assertion and Reason are correct, but 29. Assertion : Yeasts such as Saccharomyces
Reason is not the correct explanation of cerevisiae are used in baking industry.
Assertion. Reason : Carbon dioxide produced during
(c) If Assertion is correct but Reason is incorrect. fermentation causes bread dough to rise by
(d) If both the Assertion and Reason are incorrect. thermal expansion. [2015]
26. Assertion : In plant tissue culture, somatic 30. Assertion : Somatic embryos can be induced
embryos can be induced from any plant cell. from any cell in plant tissue culture.
Reason : Any viable plant cell can differentiate Reason : Any living plant cell is capable of dif-
into somatic embryos. [2008] ferentiating into somatic embryos. [2017]
27. Assertion : Cattle breeds can be improved by 31. Assertion : A major advantage of tissue culture
superovulaton and embryo transplantaion. is protoplast fusion.
Reason : Superovulation in high milk-yielding Reason : A hybrid is formed by the fusion of
cows is induced by hormonal injection. [2013] naked protoplasts of two plants. [2017]
28. Assertion : In case of vegetatively propagated
crops, pure-line selection is not required.
EBD_7100
B-172 Topicwise AIIMS Solved Papers – BIOLOGY

Type A : Multiple Choice Questions that control cell division, shoot meristem
1. (d) Phytoalexins are produced by plants that initiation, leaf and root differentiation,
are under attack. Phytoalexins produced in senescence.
plants act as toxins to the attacking 7. (c) Somaclonal variations are differences in
organism. They may puncture the cell wall, plants that are raised from the callus by
tissue culture. If these variations are of
delay maturation, disrupt metabolism or
economic value, e.g. , induce tolerance of
prevent reproduction of the pathogen in
pests, diseases, etc, such plants are
question. However, phytoalexins are often
selected and multiplied. This technique has
targeted to specific predators; a plant that been used in wheat, rice, potato and
has anti-insect phytoalexins may not have tomato.
the ability to repel a fungal attack.
8. (d) Somaclonal variations refer to heritable
2. (a) Auxanometer is an apparatus for measuring
changes which accumulate in callus (tissue
increase or rate of growth in plants. It
automatically detects and measures plant culture) from a somatic explant and express
growth and stores information in a data in the progeny of in vitro regeneration
logger. obtained from callus.
These variations have been used to
develop several useful.
9. (a) According to graph, growth of Khapra
beetle is directly proportional to cholesterol
concentration.
10. (b) The quality of fruits produced by the
grafted plant is determined by the genotype
of scion.
11. (c) Cocoa is obtained from seeds of cocoa plant
3. (a) Crop rotation increases the soil fertility by Theobroma. It belongs to family
sowing different crops, usually legume and sterculiaceae. This plant is a native of
non-legume, in successive seasons on the tropical America and its seeds are used to
same piece of land. prepare cocoa and chocolate.
4. (a) Pollen grains are used for haploid cultures 12. (a) The plant hormone, auxin, is more
since they possess n -number of distributed on the side away from the
chromosomes that are required for haploid unilateral illumination causing cells to grow
cultures. faster in the darkerside, which in turn,
5. (c) Azolla, has cyanobacteria which is the causes the plant to bend toward the light. If
nitrogen fixing bacteria. Due to this Azolla the distribution were equal, the plant would
is able to fix the nitrogen as nitrates, thereby grow just upwards. Neuro-transmitters are
making it available to plants in the soluble chemicals secreted by multicellular animals
form of nitrogen. and are used in transmitting impulses in the
6. (c) Pruning causes the cutting of plant apex nervous system.
which lowers the amount of auxin and
13. (c) Phytotron is a chamber in which plants can
relatively an increase in cytokinin content.
be grown in controlled condition.
In the plant, cytokinin promotes growth of
axillary buds causing branching. 14. (c) Essential oils-These are volatile oils and
Cytokinins are essential plant hormones possess strong aromatic smell.
Strategies for Enhancement in Food Production B-173

15. (d) Coconut water is rich in cytokinin. 23. (a) Fish meal is made from nonedible parts of
Cytokinin initiates cell division and is found the fish and is rich source of protein.
in dividing tissues. So, coconut water is Fish is a rich source of protein and many of
used as a medium in tissue culture of plant the vitamins and minerals which is required
tissues where it initiates cell division. for good health. Fish also has many other
Coconut water is liquid endosperm. benefits including helping to protect
16. (a) Azolla and BGA are biofertilizer which against heart disease and a range of other
increases the fertility of soil. illnesses. Fish is also a source of zinc, which
17. (d) Hisardale is a new breed of sheep is needed for a healthy immune system,
developed in Punjab by crossing Bikaneri iodine, needed for a healthy metabolism
ewes and Marino rams. Cross breeding is a and vitamins A and B12. Oil-rich fish are an
method in which superior male of one breed important source of omega 3 fats. Your
is mated with superior females of another body cannot make these special fats so you
breed. It allows the desirable qualities of need to eat foods containing omega 3
two different breeds to be combined. everyday. Fish meal, is a commercial
18. (d) Biodiversity refers to the variety found in product made from the waste of fish oil and
biota due to the genetic make-up of plants the bones from processed fish. It is a brown
and animals to cultural diversity. The main powder or cake obtained by pressing the
cause of the loss of biodiversity can be whole fish or fish trimmings to remove the
attributed to the influence of human beings fish oil. The major use of fish meal is as a
on the world's ecosystem. The important high-protein supplement in aquaculture
factors causing loss of biodiversity are -
feed.
habitat loss, habitat fragmentation,
disturbances, over exploitation of 24. (a) Superovulation is done by hormone
resources, pollution, exotic species, co- injection. During artificial insemination,
extinction, alien species invasion, intensive 4-10 embryos are transplanted into carrier
agriculture and forestry. cows. The seven days old embryos can be
19. (c) Before transferring on the culture preserved at a temperature of –196ºC for
medium, the explant is first of all disinfected several years. The assertion & reason are
by surface sterilization using clorax water, true and the reason is the correct
sodium or calcium hypochlorite solution explanation for the assertion.
or methiolate. Too much care must be taken 25. (b) The physiological mechanism of flowering
in this operation so that the cells do not in plants is controlled by two factors__ light
die. period and low temperature. The cold
20. (b) Coccidiosis is a protozoan disease. treatment of plants to induce flowering is
Type B : Assertion Reason Questions called vernalization. Term vernalization was
first given by T.D.Lysenko(1928). As a
21. (a) Any living plant cell which is viable can be result of vernalization a flowering hormone
used to culture somatic embryos. These called vernaline is formed. Site of
embryos have the characteristic features vernalization is apical meristem.
of the parent plant. The somatic cultures
26. (a) Any living plant cell which is viable can be
are viable from the stem, root or leaves and
used to culture somatic embryos. These
the most viable part is the meristematic cell.
embryos have the characteristic features
22. (b) Irrigation relates to the supply of water to of the parent plant. The somatic cultures
the crops. Fertilizers which are mainly NPK are viable from the stem root, or leaves &
are required to increase the harvest of
the most viable part is the meristematic cell.
crops.
EBD_7100
B-174 Topicwise AIIMS Solved Papers – BIOLOGY
27. (b) Cattle breeds can be improved by super profitably used in vegetatively propagated
ovulation and embryo transfer technique. crops because they do not involve sexual
It is also known as Multiple Ovulation reproduction and hence no loss of hybrid
Embryo Transfer Technology (MOET). In superiority.
this method, a cow is administered 29. (a) Yeast is a fermentation agent. It is a known
hormones with FSH like activity, to induce fact that yeast raises bread dough to rise
super ovulation instead of one egg, which and hence, yeast is also used to increase
they normally yield per cycle they produce the volume, making the dough porous and
6-8 eggs. The animal is either mated with the product soft. It is the carbon dioxide
an elite bull or artificially inseminated. The that is produced by the yeast that helps
fertilised eggs at 8-32 cells stages are the dough to rise.
30. (a) Somatic embryos are non- zygotic embryo
removed non surgically and transferred to
like structures that develop into from any
surrogate mothers. The genetic mother is
type of tissue in plant tissue culture.
available for another round of super
31. (b) An important technique of tissue culture,
ovulation.
somatic hybridization results in the
28. (b) In case of vegetatively propagated crops, production of somatic hybrid plants. Two
pure line selection is not required. Pure- different plant varieties each with a
line selection is useful only for sexually desirable character can be made to undergo
reproducing plants. Hybrid vigour is most protoplast fusion, which further can be
grown into a new plant.
32 Microbes in Human Welfare

TYPE A : MULTIPLE CHOICE QUESTIONS C. Bacillus thuringiensis III. Production of


human insulin
1. Which of the following antibiotic was discovered D. Pseudomonas putida IV. Biological
by Alexander Flemming ? [2000] control of
(a) Streptomycin (b) Tetracycline fungal
(c) Penicillin (d) Terramycin disease
2. Which one of the following pairs is correctly V. Bio-
matched? [2003] decomposed
(a) Rhizobium - Parasite in the roots of insectiside
leguminous plants. [2016]
(b) Mycorrhizae - Mineral uptake from soil. (a) A – III; B – I; C – V; D – IV
(c) Yeast - Production of biogas. (b) A – I; B – II; C – III; D – IV
(d) Myxomycetes - The ringworm diseases. (c) A – II; B – I; C – III; D – IV
3. The bacteria Pseudomonas is useful because of (d) A – III; B – I; C – V; D – II
its ability to [2004] 8. What would happen if oxygen availability to
(a) transfer genes from one plant to another. activated sludge flocs is reduced?
(b) decompose a variety of organic compounds. [2016]
(c) fix atmospheric nitrogen in the soil. (a) It will slow down the rate of degradation of
(d) produce a wide variety of antibiotics. organic matter.
4. Chloramphenicol and erythromycin (broad (b) The centre of flocs will become anoxic,
spectrum antibiotics) are produced by [2014] which would cause death of bacteria and
(a) Streptomyces (b) Nitrobacter eventually breakage of flocs.
(c) Rhizobium (d) Penicillium (c) Flocs would increase in size as anaerobic
5. A patient brought to a hospital with myocardial bacteria would grow around flocs.
infarction is normally immediately given: [2014] (d) Protozoa would grow in large numbers.
(a) Penicillin (b) Streptokinase 9. Which one of the following statement regarding
BOD is true? [2017]
(c) Cyclosporin-A (d) Statins
(a) The greater the BOD of waste water, more
6. Microbes are used in [2015] is its polluting potential.
1. primary treatment of sewage (b) The greater the BOD of waste water, less is
2. secondary treatment of sewage its polluting potential.
3. anaerobic sludge digester (c) The lesser the BOD of waste water, more is
4. production of bioactive molecules its polluting potential.
(a) 1, 3 and 4 (b) 1, 2, 3 and 4 (d) The lesser the BOD of waste water, less is
(c) 2, 3 and 4 (d) 3 and 4 its polluting potential.
7. Choose the right combination 10. The free-living fungus Trichoderma can be used
Column-I Column-II for [2017]
A. Escherichia coli I. Nif gene (a) killing insects
B. Rhizobium melilotae II. Digestive (b) biological control of plant diseases
hydrocarbon (c) controlling butterfly caterpillars
of crude oil (d) producing antibiotics
EBD_7100
B-176 Topicwise AIIMS Solved Papers – BIOLOGY

TYPE B : ASSERTION REASON QUESTIONS Directions for (Qs.13-15) : Each of these questions
contains an Assertion followed by Reason. Read them
Directions for (Qs. 11-12) : These questions consist carefully and answer the question on the basis of
of two statements, each printed as Assertion and following options. You have to select the one that
Reason. While answering these questions, you are best describes the two statements.
required to choose any one of the following five
(a) If both Assertion and Reason are correct and
responses.
Reason is the correct explanation of Assertion.
(a) If both Assertion and Reason are correct and (b) If both Assertion and Reason are correct, but
the Reason is a correct explanation of the Reason is not the correct explanation of
Assertion. Assertion.
(b) If both Assertion and Reason are correct but (c) If Assertion is correct but Reason is incorrect.
Reason is not a correct explanation of the (d) If both the Assertion and Reason are incorrect.
Assertion.
13. Assertion : Lichen is important for chemical
(c) If the Assertion is correct but Reason is industries.
incorrect. Reason : Litmus and Orcein are formed from
(d) If both the Assertion and Reason are incorrect. lichens. [2009]
(e) If the Assertion is incorrect but the Reason is 14. Assertion : Yeasts such as Saccharomyces
correct. cerevisiae are used in baking industry.
11. Assertion : Leguminous plants are nitrogen [2003, 2011]
fixers. Reason : Carbon dioxide produced during
Reason : Leguminous plants have Rhizobium fermentation causes bread dough to rise by
in their root nodules. [1997] thermal expansion.
12. Assertion : Nitrogen-fixing enzyme in legume 15. Assertion : Vitamins B2 is found in cereals, green
root nodules function at low oxygen vegetables, brewer's yeast, egg white, milk and liver.
concentration. Reason : It can be commercially produced by
Reason : Low oxygen concentration is provided some yeasts. [2014]
by leghaemoglobin. [2004]
Microbes in Human Welfare B-177

Type A : Multiple Choice Questions the quantity of organic wastes in the water
supply is high then the number of
1. (c) Alexander Flemming discovered Penicillin decomposing bacteria present in the water
from a fungus penicillium accidentally. It is will also be high. As a result, BOD value
a natural antibiotic.
will increase.
2. (b) Mycorrhiza is a symbiotic relationship
between fungi and roots of higher plants. 10. (b) Trichoderma is a free-living saprophytic
Mycorrhizae present in the soil help in the fungi that most commonly lives on dead
organic matter in the soil and rhizosphere
uptake of minerals from soil. These
organisms also help in the binding of roots (root ecosystem). It inhibits pathogens
to the soil. Since it is active transport in the through release of gliotoxin, viridin,
roots, the mycorrhizae help in this active gliovirin and trichodermin like substances.
transport of minerals. Type B : Assertion Reason Questions
3. (b) Pseudomonas is a gram negative rod
shaped bacteria. It is useful because of its 11. (a) The Rhizobium bacteria converts the
ability to decompose a variety of organic atmospheric nitrogen into soluble nitrates
compounds. Other bacteria which help in that is absorbed by the plants.
purification, decay are Streptococci, Soybean root nodules, contain billions of
Clostridium, Micrococus, Proteus, etc. Bradyrhizobium bacteria.
4. (a) 12. (a) Leghaemoglobin is the oxygen scavenger
5. (b) Streptokinase is immediately given to and it protects nitrogen fixing enzyme
dissolve the thrombus carring myocardial nitrogenase from oxygen.
13. (a) Litmus is an important and widely used dye
infarction.
in chemical laboratories as an acid-base
6. (c) Microbes are not used in the primary
indicator. It is obtained from Rocella
treatment of sewage which basically
montaignei. Orcein, a biological stain, is
involves settling down of sludge through
obtained from Rocella tinctoria.
sedimentation.
14. (a) Yeast is a fermentation agent. It is a known
7. (d)
fact that yeast raises bread dough to rise
8. (b) In the secondary treatment of sewage, if
and hence, Yeast is also used to increase
oxygen availability to activated sludge
the volume, making the dough porous and
flocs (masses of bacteria associated with
the product soft. It is the yeast that helps
fungal filaments to form mesh like structure)
the dough to rise.
is reduced; the centre of flocs will becomes
15. (b) Besides cereals, green vegetables, brewer's
anoxic, which would cause death of
yeast, egg white, vitamin B2 is also
bacteria and eventually breakage of flocs.
produced by intestinal bacteria. The
9. (a) BOD is the method of determining the
vitamin was first obtained in 1938 using
amount of oxygen requir ed by
wild strain of mould Ashbhya gossypii.
microorganisms to decompose the waste
Vitamin B2 is essential for normal growth
present in the water supply. It is a measure
and reproduction in a number of laboratory
of organic matter present in the water. If
animals.
EBD_7100
B-178 Topicwise AIIMS Solved Papers – BIOLOGY

Biotechnology: Principles and


33 Processes
TYPE A : MULTIPLE CHOICE QUESTIONS (c) making transient pores in the cell
membrane to introduce gene constructs.
1. Restriction endonucleases are used as [1998] (d) purification of saline water with the help of
(a) molecular build up at nucleotides. a membrane system.
(b) molecular degradation to DNA breakup. 7. What is the first step in the Southern blot
(c) molecular knives for cutting DNA at technique? [2004, 2008]
specific sites. (a) Denaturation of DNA on the gel for
(d) molecular cement to combine DNA sites. hybridization with specific probe.
2. In genetic engineering, which of the following (b) Production of a group of genetically
is used ? [2001] identical cells.
(a) Plasmid (b) Plastid (c) Digestion of DNA by restriction enzyme.
(c) Mitochondria (d) E.R. (d) Denaturation of DNA from a nucleated cell
3. Introduction of foreign gene for improving such as the one from the scene of crime.
genotype is called [2002] 8. The polymerase chain reaction (PCR)
(a) tissue culture technology was discovered by [2009]
(b) vernalization (a) Karry Mullis
(c) genetic engineering (b) Saiki et al
(d) eugenics (c) Craig Venter
4. An example of gene therapy is [2004] (d) Maxam and Gilbert
(a) production of injectable hepatitis B vaccine. 9. After 4 PCR cycles how many DNA molecules
(b) production of vaccines in food crops like are formed from one DNA template molecule ?
potatoes which can be eaten. [2012]
(c) introduction of gen e for adenosin e (a) 4 (b) 32
deaminase in persons suffering from Severe (c) 16 (d) 8
Combined Immuno Deficiency (SCID). 10. Human Genome Project (HGP) is closely
(d) production of test tube babies by artificial associated with the rapid development of a new
insemination and implantation of fertilized area in biology called as [2013]
(a) biotechnology (b) bioinformatics
eggs.
(c) biogeography (d) bioscience
5. c-DNA probes are copied from the messenger
11. Identify the correct match for the given
RNA molecules with the help of [2005]
apparatus. [2013]
(a) restriction enzymes
(b) reverse transcriptase
(c) DNA polyermase
(d) adenosine deaminase
6. Electroporation procedure involves [2005]
(a) fast passage of food through sieve pores
in phloem elements with the help of electric
stimulation.
(b) opening of stomatal pores during night by
artificial light.
Biotechnology: Principles and Processes B-179

Apparatus Function (a) ‘Ori’ is a sequence responsible for


(a) Gene gun Vectorless direct gene controlling the copy number of the linked
transfer DNA.
(b) Column Separation of (b) Selectable marker selectively permitting the
chromatograph chlorophyll growth of the non-transformants.
pigments (c) In order to link the alien DNA, the vector
(c) Stirred tank Carry out fermentation needs to have single recognition site for
bioreactor process the commonly used restriction enzymes.
(d) Respirometer Finding out rate of (d) The ligation of alien DNA is carried out at
respiration a restriction site present in one of the two
12. Genes of interest can be selected from a genomic antibiotic resistance genes.
library by using [2014]
TYPE B : ASSERTION REASON QUESTIONS
(a) Restriction enzymes
(b) Cloning vectors Directions for (Qs. 16-20) : These questions consist
(c) DNA probes of two statements, each printed as Assertion and
(d) Gene targets Reason. While answering these questions, you are
13. Choose the correct option. [2015] required to choose any one of the following five
responses.
Cla I Hind III (a) If both Assertion and Reason are correct and
A
Pvu I the Reason is a correct explanation of the
Pst I B Assertion.
C
R
(b) If both Assertion and Reason are correct but
tet
pBR 322 Sal I Reason is not a correct explanation of the
Assertion.
D (c) If the Assertion is correct but Reason is
rop
incorrect.
(d) If both the Assertion and Reason are incorrect.
Pvu II
(e) If the Assertion is incorrect but the Reason is
A B C D correct.
(a) Hind I EcoR I ampR ori 16. Assertion : Plasmids are extrachromosomal
(b) Hind I BamH I kanR ampR DNA.
(c) BamH I Pst I ori ampR Reason : Plasmids are found in bacteria and are
(d) EcoR I BamH I ampR ori useful in genetic engineering. [2001]
14. Which one of the following palindromic base 17. Assertion: Plasmids are single-stranded extra
sequences in DNA can be easily cut at about chromosomal DNA.
the middle by some particular restriction Reason: Plasmids are usually present in
enzyme? [2016] eukaryotic cells. [2002]
(a) 5'.............CGTTCG.............3' 18. Assertion: Clones are produced by sexual
3'.............ATGGTA.............5' reproduction.
(b) 5'.............GATATG.............3' Reason: These are prepared by group of cells
3'.............CTACTA.............5' descended from many cells or by inbreeding of
(c) 5'.............GAATTC.............3' a heterozygous line. [2002]
3'.............CTTAAG.............5' 19. Assertion : In recombinant DNA technology
(d) 5'.............CACGTA.............3' human genes are often transferred into bacteria
3'.............CTCAGT.............5' (prokaryotes) or yeast (eukaryote).
15. Which of the following statement is not correct Reason : Both bacteria and yeast multiply very
about cloning vector ? [2017] fast to form huge population which expresses
the desired gene. [2005]
EBD_7100
B-180 Topicwise AIIMS Solved Papers – BIOLOGY
20. Assertion : Agrobacterium tumefaciens is 21. Assertion : Restriction enzymes cut the strand
popular in genetic engineering because this of DNA to produce sticky ends. [2009]
bacterium is associated with the roots of all Reason : Stickiness of the ends facilitates the
cereal and pulse crops. action of the enzyme DNA polymerase.
Reason : A gene incorporated in the bacterial 22. Assertion : “DNA finger printing” has become
chromosomal genome-gets automatically a powerful tool to establish paternity and identity
transferred to the crop with which the bacterium of criminals in rape and assault cases. [2010]
is associated. [2005] Reason : Trace evidences such as hairs, saliva
Directions for (Qs. 21-24) : Each of these questions and dried semen are adequate for DNA analysis.
contains an Assertion followed by Reason. Read them 23. Assertion : In recombinant DNA technology,
carefully and answer the question on the basis of human genes are often transferred into bacteria
following options. You have to select the one that (prokaryotes) or yeast (eukaryote). .
best describes the two statements. Reason : Both bacteria and yeast multiply very
fast to form huge population, which express the
(a) If both Assertion and Reason are correct and
desired gene. [2008, 2015]
Reason is the correct explanation of Assertion.
24. Assertion : Insertion of recombinant DNA
(b) If both Assertion and Reason are correct, but
within the coding sequence of -galactosidase
Reason is not the correct explanation of results in colourless colonies.
Assertion. Reason : Presence of insert results in
(c) If Assertion is correct but Reason is incorrect. inactivation of enzyme -galactosidase known
(d) If both the Assertion and Reason are incorrect. as insertional inactivation. [2017]
Biotechnology: Principles and Processes B-181

Type A : Multiple Choice Questions 8. (a) Developed in 1984 by Kary Mullis, PCR is
1. (c) Restriction endonuclease cleaves DNA now a common and often indispensable
duplex at specific points in such a way that technique used in medical and biological
single stranded free ends project from each research labs for a variety of applications.
fragment of DNA duplex called sticky ends. These include DNA cloning for
These sticky ends can join similar sequencing, DNA-based phylogeny, or
complementary ends of DNA fragment functional analysis of genes; the diagnosis
from some other source. of hereditary diseases; the identification
2. (a) Plasmid (extrachromosomal part in bacteria) of genetic fingerprints (used in forensic
plays important role in recombinant DNA sciences and paternity testing); and the
technology. detection and diagnosis of infectious
3. (c) The process of introduction of foreign gene diseases. In 1993, Mullis won the Nobel
for obtaining the desirable trait is called Prize in Chemistry for his work on PCR.
genetic engineering.
9. (c) From a single DNA template molecule, it is
4. (c) Gene therapy is a new system of medicine.
possible to generate 2n DNA molecules
Excellent example of gene therapy is SCID.
after n number of cycles in polymerase chain
They have defective gene for the enzyme
reaction.
adenosine deaminase (ADA). They lack T-
lymphocytes which fail to combat the 10. (b) Human Genome Project (HGP) is closely
infecting pathogen. Lymphocytes are associated with the rapid development of
extracted from the patients bone marrow a new area in biology called Bioinformatics
and normal functioning copy of gene which is used for storage and analysis of
coding for ADA is introduced into these enormous amount of data.
lymphocytes with the help of retrovirus. 11. (c)
The cells so treated are reintroduced into
12. (c) A hybridization probe is a fragment of DNA
patients bone marrow which reactivate
patients immune system for life. of variable length which is used in DNA
5. (b) c DNA probes are copied from the mRNA samples to detect the presence of nucleotide
molecules with the help of r everse sequence (the DNA target) that are
transcriptase. complementary to the sequence in the
6. (c) Electroporation is the method of making probe. The probe hybridize to single–
cell membrane permeable for the entry of stranded DNA whose base sequence allow
recombinant DNA into the bacteria. probe target base-pairing due to
7. (c) The Southern blot is used to detect and complementary between the probe and
identify certain DNA sequences in a sample target.
of bodily fluid. It uses single-stranded DNA 13. (d)
to search out their complementary strands. 14. (c) Palindromic sequences in DNA molecule
When a Southern blot is performed on are group of bases that forms the same
DNA, the first step is digestion of DNA
sequence when read in both forward and
with restriction enzymes. Restriction
backward direction. In the given question,
enzymes cut DNA at known sequences, and
produces DNA fragments of a certain only option (c) represent a palindromic
length. Once the DNA is cut into pieces, sequence.
scientists conduct electrophoresis to 15. (b) Selectable marker selectively permitting the
separate them by size. growth of the transformants.
EBD_7100
B-182 Topicwise AIIMS Solved Papers – BIOLOGY

Type B : Assertion Reason Questions hydrogen bonds with their complementary


counter parts i.e., they can join similar
16. (b) Plasmids are the extrachromosomal part in
complementary ends of DNA fragment from
the bacteria and are useful in recombinant
some other source with the help of DNA
DNA technology.
ligase. This stickness of the ends facilitates
17. (c) Plasmid is a extra chromosomal DNA the action of the enzyme DNA ligase, not
present in prokaryotes e.g., bacteria. DNA polymerase.
18. (d) Cloning is shortcut method to amplify the 22. (a) DNA finger printing has become a powerful
number of organisms with a desirable tool due to its ability in tracing evidences
constitution. No sex is involved in their of crime and to establish the paternity.
production. Members of clone are These evidences can be collected from
genetically identical as they are derived hairs, semen and saliva.
from single parent.
23. (a) Recombinant DNA is a form of synthetic
19. (a) In r ecombinant DNA technology, DNA that is engineered through the
recombinants DNA are usually transferred combination or insertion of one or more
to E. coli, yeast because of their rapid DNA strands, thereby combining DNA
multiplication. This technology is sequences that would not normally occur
employed for combining DNA from two together. In terms of genetic modification,
different organisms to produce recombinant DNA is produced through the
recombinant DNA. addition of relevant DNA into an existing
20. (d) A soil inhabiting, plant pathogenic bacteria organismal genome, such as the plasmid
Agrobacterium tumefaciens infects broad of bacteria, to code for or alter different
leaved crops including tomato, soyabean, traits for a specific purpose, such as
sunflower and cotton but not the cereals. immunity. It differs from genetic
Tumour formation (crown galls) is induced recombination, in that it does not occur
by its plasmid (Ti) into the chromosomal through processes within the cell or
DNA of its host plant. The T-DNA causes ribosome, but is exclusively engineered.
tumors. As gene transfer occurs without Recombinant protein is protein that is
human efforts the bacteria is known as derived from recombinant DNA.
natural genetic engineer of plants. 24. (a) Alternative markers have been developed
21. (c) Restriction enzyme, a type of that can differentiate recombinants from
endonuclease, functions by "inspecting" non-recombinants based upon their ability
the length of a DNA sequence. Once it finds to produce colour in presence of a
a recognition sequence, it binds and cut chromogenic substrate. The plasmid in the
each of the two strands of the double helix bacteria, lacking an insert produces blue
at specific point leaving single stranded coloured colonies, while those plasmids
portions at the ends. This results in with an insert do not produce any colour
overhanging stretches called sticky ends. due to insertional inactivation of the
These are named so because they form enzyme, -galactosidase.
34 Biotechnology & its Applications

TYPE A : MULTIPLE CHOICE QUESTIONS


6. The scientific process by which crop plants are
1. A tumour inducing plasmid widely used in the enriched with certain desirable nutrients is called
production of transgenic plant is that of [2005] [2013]
(a) Escherichia coli (a) crop protection (b) breeding
(b) Bacillus thuringiensis (c) bio-fortification (d) bio-remediation
7. Which of the following is a variety of Brassica
(c) Staphylococcus aureus
resistance to white rust disease? [2014]
(d) Agrobacterium tumefaciens (a) Himgiri
2. Which one of the following is a correct (b) Pusa Kamal
statement? [2005] (c) Pusa Swarnim (Karan rai)
(a) "Bt" in "Bt-cotton" indicates that it is a (d) Pusa Sadabahar
genetically modified organism produced 8. The first clinical gene therapy was given in 1990
through biotechnology. to a 4 years old girl with enzyme deficiency of
(b) Somatic hybridization involves fusion of [2014]
two complete plant cells carrying desired
genes. (a) Adenosine deaminase (ADA)
(c) The anticoagulant 'hirudin' is being (b) Tyrosine oxidase
produced from transgenic Brassica napus (c) Monamine oxidase
seeds. (d) Glutamate dehydrogenase
(d) "Flavr Savr" variety of tomato has 9. Select the correct statement(s)- [2015]
enhanced the production of ethylene
(1) IARI has released a mustard variety rich in
which improves its taste.
vitamin C.
3. Cultivation of Bt cotton has been much in the
news. The prefix Bt means [2008] (2) Pusa Sawani variety of Okra is resistant to
(a) barium-treated cotton seeds aphids.
(b) bigger thread variety of cotton with better (3) Hairiness of leaves provides resistance to
tensile strength insect pests.
(c) produced by biotechnology usin g (4) Agriculture accounts for approximately
restriction enzymes and ligases 33% of India's GDP and employs nearly 62%
(d) carrying an endotoxin gene from Bacillus of the population.
thuringiensis (a) (1) and (2) (b) (2) and (3)
4. Golden rice is a transgenic crop of the future (c) (1), (3) and (4) (d) None of these
with the following improved trait [2012] 10. Read the following four statements (1-4) about
certain mistakes in two of them
(a) High lysine (essential amino acid) content
1. The first transgenic buffalo, Rosie
(b) Insect resistance
produced milk which was human alpha-
(c) High protein content lactal albumin enriched.
(d) High vitamin-A content 2. Restriction enzymes are used in isolation
5. How many varieties of rice has been estimated of DNA from other macro-molecules.
to be present in India? [2013] 3. Downstream processing is one of the steps
(a) 2,000 (b) 20,000 of R-DNA technology.
(c) 200,000 (d) 2,000,000
EBD_7100
B-184 Topicwise AIIMS Solved Papers – BIOLOGY
4. Disarmed pathogen vectors are also used TYPE B : ASSERTION REASON QUESTIONS
in transfer of R-DNA into the host.
Which are the two statements having mistakes? Directions for (Q. 13) : Each of these questions
[2015] contains an Assertion followed by Reason. Read them
(a) Statement 2 and 3 (b) Statement 3 and 4 carefully and answer the question on the basis of
following options. You have to select the one that
(c) Statement 1 and 3 (d) Statement 1 and 2
best describes the two statements.
11. A transgenic food crop which may help in solving
the problem of night blindness in developing (a) If both Assertion and Reason are correct and
Reason is the correct explanation of Assertion.
countries is [2016]
(b) If both Assertion and Reason are correct, but
(a) golden rice (b) Bt soyabean
Reason is not the correct explanation of
(c) flavr - savr tomato (d) starlink maize Assertion.
12. Which variety of rice was patented by a U.S. (c) If Assertion is correct but Reason is incorrect.
company even though the highest number of (d) If both the Assertion and Reason are incorrect.
varieties of this rice is found in India ? [2017] 13. Assertion : Insect resistant transgenic cotton
(a) Sharbati Sonara (b) Co-667 has been produced by inserting Bt gene. [2010]
(c) Basmati (d) Lerma Roja Reason : The Bt gene is derived from a bacterium.
Biotechnology & its Applications B-185

Type A : Multiple Choice Questions


deficiency or ADA-SCID is an autosomal
1. (d) A tumour (crown gall) inducing plasmid recessive metabolic disorder that causes
widely used in the production of transgenic immunodeficiency. ADA deficiency is due
plant is Agrobacterium tumefaciens. to a lack of the enzyme adenosine
2. (c) Hirudin is a protein that stops blood deaminase.
clotting. The gene encoding hirudin was 9. (c)
chemically synthesized. This gene was then 10. (d) Transgenic Rosie is actually cow.
transferred into Brassica napus where
Restriction enzymes cut the DNA at
hirudin accumulates in seeds. This hirudin
specific sites.
is purified & used as a medicine.
11. (a) A transgenic food crop which may help in
3. (d) Bacillus thuringiensis, or Bt, is a bacterium
solving the problem of night blindness in
that occurs naturally in the soil, produces
developing countries is golden rice. Golden
a protein that is toxic to certain insect pests,
rice is genetically modified rice that has
and is widely used as a pest control agent.
been engineered to have elevated levels of
It is also extremely host-specific.
beta carotene in it. It is a pre cursor of
4. (d) Golden rice is a variety of rice produced
vitamin A, which gives it a characteristic
through genetic engineering to golden colour.
biosynthesize beta-carotene, a precusor of 12. (c) Basmati rice was patented by a US
pro-vitamin A. company even though the highest number
5. (c) 6. (c) 7. (c) of varieties of this rice is found in India.
8. (a) Gene therapy is an experimental technique
that uses genes to treat or prevent disease.
Type B : Assertion Reason Questions
The first clinical gene therapy was given 13. (b) Bt-cotton is a transgenic crop. Transgenic
for treating adenosine deaminase plants are those plants, which have foreign
deficiency. A four-year old girl became the gene incorporated in their DNA. This insect
first gene therapy patient on September 14, resistant gene is derived from a bacterium,
1990 at the NIH Clinical Center. Adenosine Bacillus thuringiensis.
deaminase deficiency, also called ADA
EBD_7100
B-186 Topicwise AIIMS Solved Papers – BIOLOGY

35 Organisms and Populations

8. Which one of the following is a matching pair of


TYPE A : MULTIPLE CHOICE QUESTIONS
certain organism(s) and the kind of association?
1. Territoriality occurs as a result of [1998] [2003]
(a) competition (b) parasitism (a) Shark and sucker fish - Commensalism
(c) predation (d) co-operation (b) Algae and fungi in lichens - Mutualism
2. Obligate parasites live on [1999] (c) Orchids growing on trees - Parasitism
(d) Cuscuta (dodder) growing on other -
(a) living host only
flowering plants - Epiphytism
(b) living host and dead organic matter
9. The great barrier reef along the east coast of
(c) dead organic matter only Australia can be categorised as [2004]
(d) artificial liquid medium (a) population (b) community
3. Mycorrhiza help in absorption of [1999] (c) ecosystem (d) biome
(a) calcium (b) nutrients 10. Which one of the following correctly represents
(c) metals (d) none of these an organism and its ecological niche ? [2005]
4. The plants which can withstand narrow range (a) Vallisneria and pond
of temperature tolerance are called [2000] (b) Desert locust (Schistocerca) and desert
(a) stenothermal (b) eurythermal (c) Plant lice (aphids) and leaf
(d) Vultures and dense forest
(c) mesothermal (d) monothermal
11. Keystone species deserve protection because
5. Abundance of a species in a population, within
these [2006]
habitat is called [2001]
(a) are capable of surviving in harsh environ-
(a) niche density
mental conditions.
(b) absolute density
(b) indicate presence of certain minerals in the
(c) relative density
soil.
(d) geographic density
(c) have become rare due to overexploitation.
6. The maintenance of internal favourable conditions,
(d) play an important role in supporting other
by a self regulated mechanisms inspite of the fact
species.
that there are changes in environment, is known
12. A lizard-like member of reptila is sitting on a tree
as [2001]
with its tail coiled around a twig. This animal
(a) entropy (b) enthalpy could be [2006]
(c) homoeostasis (d) steady state
(a) Hemidactylus showing sexual dimorphism
7. July 11 is observed as [2003]
(b) Varanus showing mimicry
(a) World Population Day
(c) Garden lizard (Calotes) showing
(b) No Tobacco Day
camouflage
(c) World Environment Day
(d) Chamaeleon showing protective
(d) World Health Day colouration
Organisms and Populations B-187

13. Carrying capacity is [2009] 20. The figure given below is a diagrammatic
(a) the capacity of an individual to produce representation of response of organisms to
young ones. abiotic factors. What do A, B and C represent
(b) availability of resources in a given habitat respectively? [2015]
to support a certain no of individuals of
population, beyond which no further
growth is possible.
(c) gene frequency from one generation to
next.
(d) gene frequency in same generation.
14. Within biological communities, some species are
important in determining the ability of a large
number of other species to persist in the
A B C
community. Such species are called [2010]
(a) conformer regulator partial regulator
(a) keystone species (b) regulator partial conformer
(b) allopatric species regulator
(c) sympatric species (c) partial regulator conformer
(d) threatened species regulator
15. Presence of flagellated protozoans in the gut of (d) regulator conformer partial regulator
termites are the example [2012] 21. The salinity in sea water in parts per thousand
(a) Symbiosis (b) Parasitism (ppt) ranges between [2016]
(c) Antibiosis (d) Commensalism (a) 5-15% (b) 30-35%
16. The formula for exponential population growth (c) 50-75% (d) more than 100%
is [2013] 22. In Urn shaped age pyramid of the population
(a) dN/rN = dt (b) rN / dN = dt the trend of growth is [2017]
(c) dN / dt = rN (d) dt / dN = rN (a) Rapid (b) Stable
(c) Declining (d) Stationary
17. The rate of formation of new organic matter by
rabbit in a grassland, is called [2014] TYPE B : ASSERTION REASON QUESTIONS
(a) Net productivity Directions for (Qs. 23-25) : These questions consist
(b) Secondary productivity of two statements, each printed as Assertion and
(c) Net primary productivity Reason. While answering these questions, you are
(d) Gross primary productivity required to choose any one of the following five
18. If 4 individuals in a laboratory population of 40 responses.
fruitflies died during a specified time interval (i.e.,
(a) If both Assertion and Reason are correct and
a week), the death rate in the population during
the Reason is a correct explanation of the
that period is [2014]
Assertion.
(a) 1 (b) 0.1
(b) If both Assertion and Reason are correct but
(c) 0.01 (d) 0.4 Reason is not a correct explanation of the
19. A population of 500 that experiences 55 births Assertion.
and 5 deaths during a one-year period. What is
(c) If the Assertion is correct but Reason is
the reproductive rate for the population during
incorrect.
the one-year period ? [2015]
(d) If both the Assertion and Reason are incorrect.
(a) 0.01/year (b) 0.05/year
(e) If the Assertion is incorrect but the Reason is
(c) 0.1/year (d) 50/year
correct.
EBD_7100
B-188 Topicwise AIIMS Solved Papers – BIOLOGY
23. Assertion : Leaf butterfly and stick insect show 26. Assertion : Thick cuticle is mostly present in
mimicry to dodge their enemies. disease resistant plants.
Reason : Mimicry is a method to acquire body Reason : Disease causing agents cannot grow
colour blending with the surroundings. [2003] on cuticle and cannot invade the cuticle.[2012]
24. Assertion : Animals adopt different strategies 27. Assertion : In sigmoid growth curve, population
to survive in hostile environment. finally stabilizes itself.
Reason : Praying mantis is green in colour which Reason : Finally, the death rate increases than
merges with plant foliage. [2004] the birth rate. [2013]
25. Assertion : The sex ratio of Kerala is highest in 28. Assertion : Tropical rain forests are
India. disappearing fast from developing countries
Reason : In countries like India the population such as India.
is increasing at a rapid rate. [2005] Reason : No value is attached to these forests
Directions for (Qs. 26-30) : Each of these questions because these are poor in biodiversity.
contains an Assertion followed by Reason. Read them [2015]
carefully and answer the question on the basis of 29. Assertion : Flora contains the actual account
following options. You have to select the one that of habitat and distribution of plants of a given
best describes the two statements. area.
(a) If both Assertion and Reason are correct and Reason : Flora helps in correct identification.
Reason is the correct explanation of Assertion. [2016]
(b) If both Assertion and Reason are correct, but 30. Assertion : Species are groups of potentially
Reason is not the correct explanation of interbreeding natural populations which are
Assertion. isolated from other such groups.
(c) If Assertion is correct but Reason is incorrect. Reason : Distinctive morphological characters
(d) If both the Assertion and Reason are incorrect. are displayed due to reproductive isolation.
[2017]
Organisms and Populations B-189

Type A : Multiple Choice Questions 9. (c) Coral reefs occur in clear, shallow, warm
water where temperature rarely falls below
1. (a) Competition exists between individuals of 20°C and there is enough light for
the same or different species for food, light, photosynthesis.
space, shelter and mate. Establishment of
10. (c) Ecological niche is the status of an
territories by animals is meant for ensuring
organism within its environment and
availability of their needs.
community (affecting its survival as a
2. (a) Obligate parasites lead only parasitic life species).
and are host specific. 11. (d) Keystone species has disproportionate
3. (b) The roots of higher plants possess effects on its environment relative to its
symbiotic mycorrhizae. It is a symbiotic, biomass. Such species play a critical role
non-pathogenic association between the in maintaining the structure of an ecological
roots and soil fungi. Several conifers lack community and help to determine the types
root hair and here the mycorrhizae perform and numbers of various other species in
this function. The mycorrhizae are of the community.
two types : ectomycorrhizae and endo-
12. (d) Chamaeleon (girgit) shows protective
mycorrhizae with an intermediate category
colouration with its surrounding e.g. twig.
as ectondomycorrhizae. The most common
13. (b) Carrying capacity is the maximum number
endomycorrhizae are Vesicular Arbuscular
of individuals of population that can be
Mycorrhizae (VAM), generally present in
sustained by available resources in a given
herbaceous plants. Mycorrhizae perform
habitat beyond which there is no further
the function of root hair, i.e. they absorbs
growth. When population reaches the
essential ions.
carrying capacity then mortality < natality.
4. (a) The climatic conditions in the stenothermal 14. (a) Within biological communities, some
plants have made them to get adapted to a species may be important in determining
small variations in temperature.
the ability of large number of other species
5. (a) Density of a population is the number of to persist in the community. These crucial
individual species in a given area. Niche is species are known as key stone species.
a suitable habitat of a species. Niche Allopatric species are species having
density is the number of a particular species exclusive areas of geographic distribution.
in a given area or suitable habitat. Sympatric species are species having
6. (c) For metabolic processes to continue, the overlapping area of geographical
living things need to remain in a steady distribution.
state maintained by self regulatory 15. (a) Presence of flagellated protozoans in the
mechanism called homeostasis. gut of termites are the example of
7. (a) July 11 has been observed as World symbiosis. In this type of association both
Population Day. partners are mutally benefited from each
8. (b) Algae and fungi in lichens show mutualism. other. Termites are dependent on protozoa
Fungi provide fixation, water, minerals and for breaking down their food stuff and the
shelter to the algae. The algae manufactures protozoa are dependent on the termites as
food for itself and for fungi. host organisms.
EBD_7100
B-190 Topicwise AIIMS Solved Papers – BIOLOGY
16. (c) The formula of exponential growth is Type B : Assertion Reason Questions
dN dN
rN where is the rate of change 23. (a) Leaf butterfly is green in colour and stick
dt dt
insect also mimics in order to escape from
in population size, r is the biotic potential
the enemies and also to catch prey. The
and N is the population size.
camouflaging mechanism helps the
17. (b) At the trophic level of consumers the rate organism to get adapted to its
at which food energy is assimilated is called surroundings. The Leaf butterfly is seen
secondary productivity. Rabbit is a more near the leaves while the stick insect
camouflages with its surroundings by
consumer. living on the branches.
18. (b) Mortality or Death rate refers to the death 24. (a) Animals blend with the surroundings or
of individuals in a population. back ground to remain unnoticed for
protection and aggression.
No. of deaths
Death rate 25. (b) Kerala has a sin ratio of 1058 Indian
Total population population is growing with 1.2% animal
change.
4
0.1 individuals per fruitfly 26. (a) Disease resistant plants possess thick
40
per week cuticle. Infectious organisms can not grow
19. (c) The rate of growth, r, equals (55 births – 5 or invade cuticle.
deaths)/ 500 per year, or 0.1/year. 27. (c) In sigmoid growth curve, finally, growth
20. (d) In the graph, the line A represents regulator, rate becomes stable because mortality and
line B represents conformer and line C natality rates become equal to each other
represents partial regulator. Organisms that and finally the population shows zero
are able to maintain homeostasis by growth rate as birth rate equals death rate.
physiological means that ensures constant 28. (c) Tropical rain forests have disappeared
body temperature are called regulators. mainly due to man’s activities. Due to over
Organism that are not able to maintain a population in countries like India, rain
constant internal temperature are called forests are cut to make place available for
man to live and build houses. To build
conformers.
buildings and factories man has
Partial regulators are organisms that have
incessantly cut down trees. This has
the ability to regulate, but only over a
caused the depletion of rain forests.
limited range of environmental conditions,
29. (b) Flora contains the actual account of habitat
beyond which they simply conform.
and distribution of plants of a given area.
21. (b) Sea water typically has a salinity of around
It provides the index to the plant species
35 g/kg although lower values are typical
found in particular area.
near coasts where river enter the ocean.
Rivers and lakes can have a wide range of 30. (b) A group of individuals resembling each
salinities, from less than 0-0.1 g/kg to a few other in morphological, physiological,
g/kg, although there are many places where biochemical and behavioural characters
even higher salinities are found. The dead constitute a species. Such individuals can
sea has a salinity of more than 200 g/kg. breed among themselves but cannot breed
22. (c) In Urn Shaped pyramid the individuals with members other than their own to
below the reproductive age are fewer in produce fertile offsprings. New species are
number than th e individuals of formed mainly due to repr oductive
reproductive age. isolation.
36 Ecosystem

9. Mr. X is eating curd/yoghurt. For this food


TYPE A : MULTIPLE CHOICE QUESTIONS
intake in a food chain Mr. X should be
1. The food chain in which microbes breakdown considered as occupying [2003]
energy rich compounds synthesized by
(a) first trophic level
producers is called [1999]
(b) second trophic level
(a) ecosystem
(c) third trophic level
(b) parasitic food chain
(c) detritus level chain (d) fourth trophic level
(d) predator food chain 10. Given below is one of the types of ecological
2. 10 % law of energy transfer was given by pyramids. This type represents [2005]
(a) Lindemann (b) Tansley [2000]
(c) Stanley (d) Darwin
3. Food chain starts with [2000]
(a) autotrophs (b) herbivores
(c) carnivores (d) decomposers
4. Flora and fauna in lake or ponds is [2000]
(a) lentic biota (b) lotic biota (a) pyramid of numbers in a grassland
(c) abiotic biota (d) field layer (b) pyramid of biomass in a fallow land
5. The enzyme responsible for the reduction of (c) pyramid of biomass in a lake
molecular nitrogen to the level of ammonia in (d) energy pyramid in a spring
the leguminous root nodule is [2000]
11. The function of leghaemoglobin during
(a) nitrogenase (b) nitrate reductase
biological nitrogen fixation in root nodules of
(c) nitrite reductase (d) ammoneases
legumes is to [2006]
6. The role of bacteria in carbon cycle is [2000]
(a) photosynthesis (a) convert atmospheric N2 to NH3
(b) chemosynthesis (b) convert ammonia to nitrite
(c) decomposition of organic compounds (c) transport oxygen for activity of nitrogenase
(d) evolution of O2 (d) protect nitrogenase from oxygen
7. Trophic levels are formed by : [2001] 12. An ecosystem, such as an aquarium, is self-
(a) plants sustaining if it involves the interaction between
(b) animals organisms, a flow of energy, and the presence
(c) organisms linked in food chain of [2009]
(d) carnivores
(a) equal numbers of plants and animals
8. Desert can be converted into green land by
planting [2001] (b) more animals than plants
(a) oxylophytes (b) psammophytes (c) materials cycles
(c) halophytes (d) trees (d) pioneer organisms
EBD_7100
B-192 Topicwise AIIMS Solved Papers – BIOLOGY
13. The graph below shows the changes in two (c) the desert insects eat away flowers during
populations of herbivores in a grassy field. A day time.
possible reason for these changes is that [2009] (d) the desert insects are active during night
time.
18. Whale is [2012]
(a) Primary producer
(b) Carnivorous, secondary consumer
(c) A decomposer
(d) Herbivorous
(a) all of the plant populations in this habitat 19. Which one of the following is not a function of
decreased. an ecosystem? [2013]
(b) population B competed more successfully (a) Energy flow (b) Decomposition
for food than population A did. (c) Productivity (d) Stratification
(c) population A produced more offspring 20. How much portion of the Photosynthetically
than population B did. Active Radiation (PAR) is captured by the
(d) population A consumed the members of plants? [2016]
population B. (a) 5 – 10% (b) 7 – 10%
14. A scorpion stalks, kills, and then eats a spider. (c) 8 – 10% (d) 2 – 10%
Based on its behavior, which ecological terms
21. Arrange the following ecosystems in increasing
describe the scorpion? [2009]
order of mean NPP (Tonnes / ha / year)
(a) producer, herbivore, decomposer A. Tropical deciduous forest
(b) producer, carnivore, heterotroph B. Temperate coniferous forest
(c) predator, carnivore, consumer C. Tropical rain forest
(d) predator, autotroph, herbivore D. Temperate deciduous forest [2017]
15. In the vast marine ecosystem, certain sea (a) B < A < D < C
develop red colouration. This red colour is due (b) D < B < A < C
to the presence of large population of which (c) A < C < D < B
one of the following organisms? [2009] (d) B < D < A < C
(a) Trichodesmium erythrium TYPE B : ASSERTION REASON QUESTIONS
(b) Physarium
(c) Dinoflagellates Directions for (Q. 22) : These questions consist of
(d) Diatoms and members of red algae two statements, each printed as Assertion and Reason.
16. The xerophytic plants conserve water by storing While answering these questions, you are required to
it in [2009] choose any one of the following five responses.
(a) intercellular spaces (a) If both Assertion and Reason are correct and
(b) normal parenchymatous cells the Reason is a correct explanation of the
(c) intercellular spaces and parenchymatous Assertion.
cells (b) If both Assertion and Reason are correct but
(d) parenchymatous cells specialized for this Reason is not a correct explanation of the
purpose Assertion.
17. Most of the desert plants bloom during night (c) If the Assertion is correct but Reason is
time because [2010] incorrect.
(a) their blooming is controlled by low (d) If both the Assertion and Reason are incorrect.
temperature. (e) If the Assertion is incorrect but the Reason is
(b) they are sensitive to the phases of moon. correct.
Ecosystem B-193

22. Assertion: Insectivorous habitat of plants is to 24. Assertion : Pyramid of energy may be upright
cope up O 2 deficiency. or inverted. [2011]
Reason: Only 20% of energy goes to next trophic
Reason: Insectivorous plants are partly
level.
autotrophic and partly heterotrophic. [1998]
25. Assertion : Biotic community has higher
Directions for (Qs. 23-28) : Each of these questions position than population in ecological hierarchy.
contains an Assertion followed by Reason. Read them Reason : Population of similar individuals
carefully and answer the question on the basis of remains isolated in the community. [2012]
following options. You have to select the one that 26. Assertion : Net primary productivity is gross
best describes the two statements. primary productivity minus respiration.
(a) If both Assertion and Reason are correct and Reason : Secondary productivity is produced
by heterotrophs. [2013]
Reason is the correct explanation of Assertion.
27. Assertion : Net primary productivity is gross
(b) If both Assertion and Reason are correct, but
primary productivity minus respiration.
Reason is not the correct explanation of Reason : Secondary productivity is produced
Assertion. by heterotrophs. [2016]
(c) If Assertion is correct but Reason is incorrect. 28. Assertion : In a food chain, members of
(d) If both the Assertion and Reason are incorrect. successive higher levels are fewer in number.
23. Assertion : A network of food chains existing Reason : Number of organisms at any trophic
together in an ecosystem is known as food web. level depends upon the availability of organisms
Reason : An animal like kite cannot be a part of which serve as food at the lower level.
a food web. [2006, 2008, 2011] [2003, 2017]
EBD_7100
B-194 Topicwise AIIMS Solved Papers – BIOLOGY

Type A : Multiple Choice Questions second trophic level and Mr. X will occupy
third place (curd eater).
1. (c) In n ature, detritus food chains are
10. (c) The given figure shows the pyramid of
indispensable as the dead organic matter biomass in a lake. An ecological pyramid
of grazing food chains is acted upon by of biomass shows the relationship between
the detritivores (bacteria, protozoa, biomass and trophic level by quantifying
nematodes) to recycle the inorganic the amount of biomass present at each
elements into the ecosystem. trophic level of an ecological community
2. (a) 10 % law of energy transfer (pyramid of at a particular moment in time.
energy) was given by Lindemann. 11. (d) Leghaemoglobin is an oxygen scavanger.
In this, only 10% of total energy received It combines with oxygen and protects
by one trophic level is transferred to next nitrogenase which catalyses the fixation of
trophic level. nitrogen under anerobic conditions.
3. (a) All trophic levels in an ecosystem are 12. (c) If a ecosystem is to be self-sustaining,
connected by transfer of food as energy. materials such as oxygen, carbon dioxide,
The transfer of food and its contained water and nitrogen must to recycle
energy from one trophic level to the next between the organisms.
trophic level is called food chain. 13. (b) If population B increased while population
Food chain always starts with producers A decreased, these organisms were
(autotrophs) Herbivores probably in competition for the same food
Carnivorous Detrivores are placed at (grass) and population B was better adapted.
the top of the food chain. Hence, population A is competitively being
4. (a) Lentic relates to still waters such as lakes excluded from the population.
and ponds. Hence, the flora and fauna 14. (c) Because the scorpion stalks, kills and eats
constitue the lentic biota. its food, it is a predator. Because it eats a
5. (d) The enzyme responsible for the reduction spider it is a carnivore. Because it ingests
of nitrogen to ammonia is ammoneases and food it is a consumer. A producer is an
the process is ammonification. e.g. autotroph as it is an organism that makes
actinomycetes, Bacillus ramosus, B. its own food from inorganic substances. A
vulgaris etc. decomposer breaks down dead matter and
6. (c) The excretory wastes of living organisms a herbivore eats only plants.
have accumulated carbon compounds and 15. (a) Trichodesmium erythreum is a
they are decomposed after their death by cyanobacteria (blue green alga). Although
micro-organisms in the soil to release CO2 a blue green alga, it possesses a pigment,
back into the environment for its recycling. phycoerythrin, which is red in colour and
7. (c) The producers and consumers in imparts red colour to the water of the sea in
ecosystem are arranged into several which it is found, hence named Red Sea.
feeding groups/levels called trophic levels. 16. (d) The xerophytic plants conserve water by
8. (b) Psammophytes are those plants that can storing it in parenchymatous cells
grow in desert and mainly in sandy soil. specialized for this purpose. Xerophytes
Hence, psammophytes can be used to plants are specially adapted to succeed in
convert desert into a green land. an arid climate. They are typically able to
withstand long periods of drought and the
9. (c) Producers occupy first trophic level, drying effects of desert winds. Some plants
primary consumers i.e. herbivores (cow have adapted to arid lands by developing
produce milk formating curd) are placed at the ability to store water.
Ecosystem B-195

17. (d) In desert condition, most of the activity of 24. (d) Energy flow in the ecosystem is a
the plants and animals happens during night unidirectional manner. There is a decline in
because of very high temperature in day the amount of energy passing from one
time. As a result the desert insects make trophic level to the next. Thus the pyramid
themselves active and pollinate the flowers of energy is always upright. According to
at night. To attract the insects, most of the
Lindemann, only 10% of energy goes to
desert plants bloom during night.
next trophic level.
18. (b) Whale is carnivorous and feeds on primary
consumer and occupies the third trophic 25. (c) The organisms of all the species that live
level of the ecosystem. in a particular area and interact in various
19. (d) Four important functional aspects of the ways with one another form biotic
ecosystem are community. Biotic community is a grouping
(i) Productivity that is higher than population in ecological
(ii) Decomposition, hierarchy. It is an assemblage of all the
(iii) Energy flow and populations of different organisms
(iv) Nutrient cycling. occurring in an area. The different
20. (d) The main source of energy for an populations of a community do not remain
ecosystem is the radiant energy or light isolated. They show interactions and inter-
energy derived from the sun. 50% of the dependence.
total solar radiation that falls on earth is 26. (b) Net primary productivity is the rate of
Photosynthetically Active Radiation organic matter built up or stored by
(PAR). producers in their bodies per unit time and
The light energy is converted into area. Net productivity is equal to gross
chemical energy in the form of sugar by primary productivity minus loss due to
photosynthesis. respiration and other reasons. Rate of
6H2O + 6CO2 + Light 6C6H12O6 + 6O2 increase in energy containing organic
Plants utilize 2-10% of PAR in matter or biomass by heterotrophs or
photosynthesis. consumers per unit time and area is known
21. (d) Net primary productivity (NPP) is the as secondary productivity.
biomass or storage of energy by green
plants. It is equal to the gross primary 27. (b) Net primary productivity is the rate of
productivity minus loss due to respiration. organic matter build up or stored by
The productivity generally increases from producers in their bodies per unit time and
polar regions toward the tropics, because area. Net productivity is equal to gross
of the increasing sunlight and temperature. primary productivity minus loss due to
Type B : Assertion Reason Questions respiration and other reasons. Rate of
increase in energy containing organic
22. (e) Insectivorous plants are those plants which matter or biomass by heterotrophs or
capture and digest live prey (normally consumers per unit time and area is known
insects) to obtain nitrogen compounds that as secondary productivity.
are lacking in its usual marshy habitat. 28. (d) When food is made available, automatically
These plants are partly autotrophic and
the next higher level of organism in the
partly heterotrophic.
hierarchy should increase. This is because
23. (c) In the food web, different food chains are
when the forest cover got depleted it led to
interconnected. Each chain is
interconnected and consists of different the increase in the number of endangered
trophic levels i.e. producers, consumers and species. If the deer population is more, it
detrivorous. So, kite can also be a part of automatically leads to an increase in the
food web. tiger population.
EBD_7100
B-196 Topicwise AIIMS Solved Papers – BIOLOGY

37 Biodiversity and its Conservation

6. Which one of the following is correct matching


TYPE A : MULTIPLE CHOICE QUESTIONS of a plant, its habit and forest type where it
1. Heavy rainfall during summer produces [1998] normally occurs? [2005]
(a) Prosopis, tree, scrub
(a) desert (b) grassland (b) Saccharum officinarum, grass, forest
(c) forest (d) wetland (c) Shorea robusta, herb, tropical rain forest
2. The trees occurring in two seasons is the (d) Acacia catechu, tree, coniferous forest
characteristic feature of [1998] 7. One of the ex-situ conservation methods for
(a) temperate deciduous forest endangered species is [2005]
(b) tropical savannah (a) wildlife sanctuaries
(c) grassland (b) biosphere reserves
(d) coniferous forest (c) cryopreservation
3. The map given below indicates the former and (d) national parks
8. Genetic diversity in agricultural crops is
the present distribution of an animal. [2003]
threatened by [2005]
(a) introduction of high yielding varieties.
(b) intensive use of fertilizers.
(c) extensive intercropping.
(d) intensive use of biopesticides.
9. The Montreal protocol refers to [2006]
(a) persistent organic pollutants
(b) global warming and climate change
(c) substances that deplete the ozone layer
(d) biosafety of genetically modified
organisms
Former distribution 10. Biosphere reserves differ from National parks
Present distribution and Wildlife sanctuaries because in the
former [2006]
Which animal could it be? (a) human beings are not allowed to enter.
(a) Wild ass (b) Nilgai (b) people are an integral part of the system.
(c) Black buck (d) Lion (c) plants are paid greater attention than the
4. If the high altitude birds become rare or extinct, animals.
the plants which may disappear along with them (d) living organisms are brought from all over
are [2004] the world and preserved for posterity.
(a) pine (b) oak 11. Which part of the world has a high density of
(c) orchids (d) Rhododendrons
organism? [2007]
5. Which one of the following is a pair of
endangered species? [2004] (a) Deciduous forests
(a) Garden lizard and Mexican poppy (b) Grasslands
(b) Rhesus monkey and sal tree (c) Tropical rain forests
(c) Indian peacock and carrot grass (d) Savannahs
(d) Hornbill and Indian aconite
Biodiversity and its Conservation B-197

12. Beta diversity is diversity [2007]


200
(a) in a community
100
(b) between communities
50

S : Number of bird species


(c) in a mountain gradient
30
(d) on a plain
13. Which one of the following pairs of 20
Slope
geographical areas show maximum biodiversity 10
0.15
in our country ? [2008] 5
S = 1.89
(a) Sunderbans and Rann of Kutch
2
(b) Eastern Ghats and West Bengal 0.001 0.01 0.1 1 10 100 1000 10,000 100,000
(c) Eastern Himalaya and Western Ghats
A : Island land area (km2) [2016]
(d) Kerala and Punjab.
(a) 17 percent of the bird species will be lost.
14. A tree species in Mauritus failed to reproduce
because of the extinction of a fruit-eating bird. (b) 20 percent of the bird species will be lost.
Which one of the following was that bird? (c) All of bird species will be lost.
(a) Dove (b) Dodo [2010] (d) 93 percent of the bird species will be lost.
(c) Condor (d) Skua TYPE B : ASSERTION REASON QUESTIONS
15. Tectonic is the study of [2011]
Directions for (Q. 20) : These questions consist of
(a) volcanos (b) earth’s crust
two statements, each printed as Assertion and Reason.
(c) sand dunes (d) Sun
While answering these questions, you are required to
16. If the Bengal tiger becomes extinct
choose any one of the following five responses.
[2004, 2012]
(a) If both Assertion and Reason are correct and
(a) Hyenas and wolves will become scare
the Reason is a correct explanation of the
(b) The wild area will be safe for man and
Assertion.
domestic animals
(b) If both Assertion and Reason are correct but
(c) Its gene pool will be lost for ever
Reason is not a correct explanation of the
(d) The population of beautiful animals like
Assertion.
deers will be stabilized.
17. Which of the following is considered a hot-spot (c) If the Assertion is correct but Reason is
of biodiversity in India ? [2013] incorrect.
(a) Indo-Gangetic Plain (d) If both the Assertion and Reason are incorrect.
(b) Eastern Ghats (e) If the Assertion is incorrect but the Reason is
(c) Aravalli Hills correct.
(d) Western Ghats 20. Assertion : In tropical rain forests. O-horizon
18. The largest Tiger reserve in India is [2014] and A-Horizon of soil profile are shallow and
(a) Nagarhole nutrient-poor.
(b) Valmiki Reason : Excessive growth of micro-organisms
(c) Nagarjunsagar-Srisailam in the soil depletes its organic content.[2006]
(d) Periyar Directions for (Qs. 21-24) : Each of these questions
19. Using the figure, determine the percentage of contains an Assertion followed by Reason. Read them
bird species that will be lost if the island's carefully and answer the question on the basis of
inhabitable land area is reduced from 100,000 following options. You have to select the one that
km2 to 1 km2. best describes the two statements.
EBD_7100
B-198 Topicwise AIIMS Solved Papers – BIOLOGY
(a) If both Assertion and Reason are correct and 22. Assertion : Diversity observed in the entire
Reason is the correct explanation of Assertion. geographical area is called gamma diversity.
(b) If both Assertion and Reason are correct, but Reason : Biodiversity decreases from high
Reason is not the correct explanation of altitude to low altitude. [2014]
Assertion.
23. Assertion : A sanctuary is formed for the
(c) If Assertion is correct but Reason is incorrect. conservation of animals only.
(d) If both the Assertion and Reason are incorrect.
Reason : Restricted human activities are allowed
21. Assertion : Tropical rain forests are
in sanctuaries. [2014]
disappearing fast from developing countries
such as India. 24. Assertion: Communities that comprise of more
Reason : No value is attached to these forests species tend to be more stable.
because these are poor in biodiversity. Reason: A higher number of species results in
[2012, 2013] less animal variation in total biomass. [2017]
Biodiversity and its Conservation B-199

Type A : Multiple Choice Questions 10. (b) Biosphere reserves are a special category
of protected areas of land or coastal
1. (b) During summer the land becomes dry and environments where people are an integral
rainfall induces formation of grass and component of the system.
hence, heavy rainfall during summer 11. (c) Tropical rain forests have a high density
produces grasslands. of organisms. Tropical rain forests are
2. (a) In the deciduous forests, the leaves mainly found in America, South America,
generally fall off during autumn. These Cargo river basin of Africa, South East Asia.
trees are seen in temperate lands. In this biome, rainfall and warmth are
3. (a) This animal which had a wide distribution abundant. Plant growth is luxuriant. This
has now considerably decreased in biome possess more than half of the flora
number. This is due to the depletion of and fauna of the world. Productivity is
forest areas due to man’s intrusion. This very high. Diversity of life is so high that a
area has now become a protected area and hectare of the forest may have 200 species
hence, this animal is now considered an of trees. 70–80% of all insects and 80 –
endangered species. 85% of all birds are known from tropical
4. (b) The oak plant may disappear since they forests.
are seen at a high altitude and their 12. (b) , , are the three types of ecosystem
dispersal is dependant on these birds. diversities where diversity is the diversity
5. (d) Endangered species are species with low operating between communities.
population numbers that ar e in 13. (c) The Eastern Himalayas Region includes
considerable danger of becoming extinct Bhutan, north-eastern India, and southern,
Hornbill and Indian Aconite are endagered central, and eastern Nepal. It is home to
species. 163 globally threatened species, including
6. (a) Prosopis fuliflora is a shrub or small tree Asia’s three largest herbivores—the Asian
native to Mexico, South America and the elephant (Elephas maximus), the greater
Caribbean. Its uses include forage, wood one-horned rhinoceros (Rhinoceros
and environmental management. unicornis), and the wild water buffalo
(Bubalus bubalis)—and its largest
7. (c) Ex situ conservation is the method of
carnivore, the tiger (Panthera tigris), as
selecting plants or animals in places outside
well as several large birds such as vultures,
their natural homes, e.g. cryopreservation,
adjutant storks, and hornbills. The Western
tissue culture etc. They are the sources of
Ghats are amongst the world’s biodiversity
genetic material for breeders and genetic
Hotspots. The various human-induced
engineering.
threats to the rich biodiversity and the large
8. (d) Genetic diversity is liable to undergo number of endemic species have been
degradation and prone to mass scale identified and highlighted by the recently
destruction due to fungal and insects completed National Biodiversity Strategy
attacks and intensive use of biopesticides. and Action Plan; Western Ghats Ecoregion.
9. (c) Montreal protocol (1987) was a landmark 14. (b) Dodo became extinct in 14th century from
international agreement to protect the Mauritius because of large-scale hunting.
stratospheric ozone by agreeing to limit the Dodo was helpful in pollinating and
production and use the ozone-depleting propagating seed of this tree species.
substances.
EBD_7100
B-200 Topicwise AIIMS Solved Papers – BIOLOGY
15. (b) Tectonic is the study of the earth's crust. It and has abundant minerals mixed with
was considered that earth was made up of humus.
tectonic plates. 21. (c) Tropical rain forests are located in the
16. (c) If bengal tiger becomes extinct, its gene equatorial regions where the annual rainfall
pool will be lost forever. There are various exceeds 140 cm. They are also called
methods for conservation of biodiversity jungles and cover one twelveth of earth's
like formation of botanical gardens, surface but contain more than half of the
zoological park, sperm bank, gene bank etc. earth's flora and fauna (i.e., rich in
17. (d) Hot spot are those areas which were rich in biodiversity). Now-a-days these forests are
biodiversity but now under threat due to disappearing due to excessive cutting of
direct or indirect interference of human forests for domestic purposes like fuel,
activites. These regions are on the edge to furnitures, accomodations, cloths, resin,
get some of their species extinct due to gum, etc.
humans. Western Ghats in India are under
threat due to continuous developmental 22. (a)
activities and Doon valley is under threat 23. (a) A sanctuary is an area which is reserved
due to continuous mining activities. for the protection of wild animals only. The
18. (c) Nagarjunsagar-Srisailam Tiger Reserve is activities like harvesting of timber,
the largest Tiger reserve in India and the collection of minor forest products and
only Tiger Reserve in Andhra Pradesh private ownership rights are allowed,
state. however, such activities should not have
any adverse effect on animals.
19. (d)
24. (a) Communities with higher number of
Type B : Assertion Reason Questions species are more stable as it can resist
occasional disturbances. A stable
20. (c) O-horizon occupies the topmost soil and community should show less variation in
is rich in mineral and decomposed organic productivity from year to year and
matter (humus). A-horizon is dark coloured resistance towards alien species.
38 Environmental Issues

9. Photochemical smog formed in congested


TYPE A : MULTIPLE CHOICE QUESTIONS
metropolitan cities mainly consists of [2003]
1. Which of the following is biodegradable (a) ozone, peroxyacetyl nitrate and NOx
pollutant? [1997] (b) smoke, peroxyacetyl nitrate and SO2
(a) Plastic (b) Asbestos (c) hydrocarbons, SO2 and CO2
(c) Sewage (d) Polythene (d) hydrocarbons, ozone and SOx
2. Positive pollution of soil is due to [1997] 10. Biological Oxygen Demand (BOD) is a measure
(a) reduction in soil productivity of [2003]
(b) addition of waste to soil (a) industrial wastes poured into water bodies
(b) extent to which water is polluted with
(c) excessive use of fertilizers
organic compound
(d) all of the above
(c) amount of carbon monoxide inseparably
3. Which of the following metal pollution causes combined with haemoglobin
sterility in human beings? [1998] (d) amount of oxygen needed by green plants
(a) Mercury (b) Arsenic during night
(c) Manganese (d) Chromium 11. Drinking of mineral water with very low level of
4. Which of the following is dissolved in water to pesticides (about 0.02 ppm) for long periods may
make Bordeaux mixture ? [1998] (a) produce immunity against mosquito [2003]
(a) Calcium chloride (b) Copper sulphate (b) cause leukemia (blood cancer) in most
(c) Sodium chloride (d) None of these people
5. Acid rain is due to pollution of [2000] (c) cause cancer of the intestine
(a) dust (b) pesticides (d) lead to accumulation of pesticide residues
(c) SO2 and NO2 (d) carbon particle in body fat
6. Deforestation causes [2001] 12. Minimata disease is pollution related disease. It
results from [2001, 2004]
(a) soil erosion (b) soil pollution
(a) oil spills in sea
(c) noise pollution (d) air pollution
(b) DDT pollution
7. Green mufler is useful against [2002]
(c) release of industrial waste containing
(a) air pollution mercury in fishing water
(b) noise pollution (d) accumulation of arsenic
(c) soil pollution 13. A lake with an inflow of domestic sewage rich in
(d) radioactive pollution organic waste may result in [2004]
8. In almost all Indian metropolitan cities like Delhi, (a) drying of the lake very soon due to algal
the major atmospheric pollutant(s) is / are[2003] bloom
(a) suspended particulate matter (SPM) (b) an increased production of fish due to lot
(b) oxides of sulphur of nutrients
(c) carbon dioxide and carbon monoxide (c) death of fish due to lack of oxygen
(d) oxides of nitrogen (d) increased population of aquatic food web
organisms
EBD_7100
B-202 Topicwise AIIMS Solved Papers – BIOLOGY
14. Nitrogen oxides, produced from the emission of 20. Which one of the following statement pertaining
automobiles and power plants, are the source of to pollutants is correct? [2011]
fine air borne particles which lead to [2004] (a) DDT is a non-biodegradable pollutant
(a) photochemical smog (b) Excess fluoride in drinking water causes
(b) dry acid deposition osteoporosis hardening of bones, stiff
(c) industrial smog joints
(d) wet acid deposition (c) Excess cadmium in drinking water causes
15. Formation of non-functional methaemoglobin
black foot disease
causes blue-baby syndrome. This is due
(d) Methylmercury in water may cause "Itai
to [2005]
(a) excess of arsenic concentration in drinking Itai" disease
water 21. In the environment, ozone is known for its
(b) excess of nitrates in drinking water (a) Harmful effects [2012]
(c) deficiency of iron in food (b) Useful effects
(d) increased methane content in the (c) Both (a) and (b)
atmosphere (d) Inert nature
16. Pollution from animal excreta and organic waste 22. Which one of the following statement is true?
from kitchen can be most profitably minimized
[2013]
by [2006]
(a) The greater the BOD of waste water, more
(a) storing them in underground storage tanks
is its polluting potential.
(b) using them for producing biogas
(c) vermiculture
(d) using them directly as biofertilizers (b) The greater the BOD of waste water, less is
17. Which one of the following is an environment- its polluting potential.
related disorder with the correct main cause? (c) The lesser the BOD of waste water, more is
[2006] its polluting potential.
(a) Black lung disease (pneumoconiosis) (d) The lesser the BOD of waste water, less is
found mainly in workers in stone quarries its polluting potential.
and crushers. 23. Which one of the following pairs is mismatched?
(b) Blue baby disease (methaemoglobinaemia) (a) Fossil fuel burning - release of CO2[2013]
due to heavy use of nitrogen rich fertilizers (b) Nuclear power - radioactive wastes
in the area. (c) Solar energy - green house effect
(c) Non-Hodgkin’s Lymphoma found mainly (d) Biomass burning - release of CO2
in workers involved in manufacture of neem 24. The two gases making the highest relative
based pesticides. contribution to the greenhouse gases are [2014]
(d) Skin cancer mainly in people exposed to (a) CO2 and CH4 (b) CH4 and N2O
benzene and methane. (c) CFC and N2O (d) CO2 and N2O
18. Ozone in stratosphere extends [2007] 25. A lake near a village suffered heavy mortality of
(a) 10-20 km (b) 20-25 km fishes within a few days. Consider the following
(c) 15-30 km (d) 25-40 km reasons for this:
19. Which one of the following organisms is likely 1. Lots of urea and phosphate fertilizer were
to show the highest concentration of DDT, once used in the crops in the vicinity
it has been introduced into the ecosystem?
2. The area was sprayed with DDT by an
[2010]
aircraft
(a) Grasshopper (b) Toad
3. The lake water turned green and stinky
(c) Snake (d) Cattle
Environmental Issues B-203

4. Phytoplankton populations in the lake 29. Assertion : Inhabitants close to very busy
declined initially thereby greatly reducing airports are likely to experience health hazards.
photosynthesis. Reason : Sound level of jet aeroplanes usually
Which two of the above were the main causes exceeds 160 dB. [2003]
of fish mortality in the lake? [2015] 30. Assertion : Organochlorine pesticides are
(a) 2 and 3 (b) 3 and 4 organic compounds that have been chlorinated.
(c) 1 and 3 (d) 1 and 2 Reason : Fenitrothion is one of the
26. DDT residues are rapidly passed through food organochlorine pesticides. [2003]
chain causing biomagnification because DDT 31. Assertion : Agricultural output increased
is [2015] several times after introduction of DDT.
(a) moderately toxic Reason : DDT was the first insecticide used on
a wide scale. [2004]
(b) non-toxic to aquatic animals
32. Assertion : A suspended particulate matter
(c) water soluble
(SPM) is an important pollutant released by
(d) lipo soluble
diesel vehicles.
27. Two lakes, A and B are identical in all aspects Reason : Catalytic converters greatly reduce
except that lake A has higher temperature. pollution caused by automobiles. [2005]
Which of the following is true ? [2016] 33. Assertion : Presently, the global atmosphere is
(a) A has higher rate of oxygen dissolution. warming up.
(b) B has higher rate of oxygen dissolution. Reason : The depletion of stratospheric ozone
(c) Oxygen dissolution of both is the same. layer has resulted in increase in ultraviolet
(d) Both the lakes have same BOD. radiations reaching the earth. [2005]
28. Euro II norms stipulate that sulphur be controlled 34. Assertion : Deforestation is one main factor
at _______ ppm in diesel and _______ ppm in contributing to global warming.
petrol. [2017] Reason : Besides CO2, two other gases methane
(a) 350; 150 (b) 150; 350 and CFCs are also included under green house
gases. [2006]
(c) 350; 250 (d) 150; 250
35. Assertion : UV radiation causes photo-
TYPE B : ASSERTION REASON QUESTIONS dissociation of ozone into O 2 and O, thus
causing damage to the stratospheric ozone layer.
Directions for (Qs. 29-38) : These questions consist
Reason : Ozone hole is resulting in global
of two statements, each printed as Assertion and
warming and climate change. [2006]
Reason. While answering these questions, you are
36. Assertion : The concentration of methane in
required to choose any one of the following five
the atmosphere has more than doubled in the
responses.
last 250 years.
(a) If both Assertion and Reason are correct and
Reason : Wetlands and rice fields are the major
the Reason is a correct explanation of the
sources of methane. [2006]
Assertion.
37. Assertion (A) : Pollution is always caused by
(b) If both Assertion and Reason are correct but
human activities.
Reason is not a correct explanation of the
Reason (R) : Pollution is not different from
Assertion.
contamination. [2007]
(c) If the Assertion is correct but Reason is
38. Assertion (A) : Chlorofluorocarbons are
incorrect.
responsible for ozone depletion.
(d) If both the Assertion and Reason are incorrect.
Reason (R) : Ozone level decreases by as much
(e) If the Assertion is incorrect but the Reason is as 67% every year. [2007]
correct.
EBD_7100
B-204 Topicwise AIIMS Solved Papers – BIOLOGY
Directions for (Qs. 39-43) : Each of these questions Reason : Nitrates are responsible for blue baby
contains an Assertion followed by Reason. Read them syndrome.
carefully and answer the question on the basis of 41. Assertion : Water pollutants are measured by
following options. You have to select the one that BOD.
best describes the two statements. Reason : If BOD is more, the water is polluted.
(a) If both Assertion and Reason are correct and [2013]
Reason is the correct explanation of Assertion. 42. Assertion : Methane, component of green
(b) If both Assertion and Reason are correct, but house gases, contributing to global warming is
Reason is not the correct explanation of about 20 percent.
Assertion. Reason : Introduction of multi-point fuel
(c) If Assertion is correct but Reason is incorrect. injection engines in automobiles has decreased
(d) If both the Assertion and Reason are incorrect. methane content in the exhausts.
39. Assertion : Secondary succession takes place [2005, 2015]
in recently denuded area. [2009] 43. Assertion : Eutrophication shows increase in
Reason : It is caused due to baring of an area. productivity in water. [2013, 2017]
40. Assertion : Excess of nitrates in drinking water Reason : With increasing eutrophication, the
are harmful for infants. [2009] diversity of the phytoplankton increases.
Environmental Issues B-205

Type A : Multiple Choice Questions weight of organisms with the rise in trophic
level due to their accumulation in fat
1. (c) Sewage is a biodegradable pollutant i.e. (biomagnification).
capable of being removed by microbial
12. (c) Mercury compounds in waste water are
action.
converted by bacterial action into extremely
2. (d) Soil pollution is of two types : positive soil toxic compound, methyl mercury.
pollution and negative soil pollution. Biomagnification of mercury into fishes
When there is addition of undesirable through the food chain is responsible for
substance in the soil then it is called as large number of deaths due to Minamata
positive pollution. diseases. It is characterised by numbers of
3. (c) Manganese causes sterility in human limbs, lips and tongue, deafness, blurred
beings. It enters the food chain and finally vision, mental retardation etc.
reaches human beings & has an effect on 13. (c) A lake rich in (domestic sewage) nutrients
the reproductive system. accelerates the growth of algae which use
4. (b) Bodeaux mixture is a combination of copper oxygen at night and deoxygenate the water
sulphate and hydrated lime. It is used enough to kill the fish and other animals
mainly to control garden, vineyard and farm (eutrophication).
infestations of fungus. 14. (a) Nitrogen oxides form peroxyacyl nitrate
5. (c) Acid rain is rainfall and other form of (PAN) by reacting with hydrocarbons. It
precipitation with a pH of less than 5. It is leads to photochemical smog formation.
due to the excess of SO2 and NO2 in the air
15. (b) Excess nitrate in drinking water, leaves and
which along with water form sulphurous
fruits, changes into nitrite in alimentary
and nitric acid.
canal. It reacts with haemoglobin and
6. (a) In India, deforestation is the main cause of
produces non-functional met haemoglobin.
soil erosion. The quality of top soil loss in
This reduces O2 carrying capacity of blood.
India is about 18.5% of the total global loss.
The disorder is called methaemoglobinea.
7. (b) Green muffler or green belt vegetation is It causes blue baby syndrome and
rows of trees and shrubs grown and breathlessness in adults.
maintained to serve as noise absorbers.
16. (c) Vermiculture means artificial seating of
8. (a) Suspended particulate matter (SPM) is worms whose excreta is rich in humus.
maximum in metropolitan cities. These worms eat farmyard manure along
9. (c) Photochemical smog or oxidizing type of with other farm wastes and pass out it
pollution is characterized by the presence through their bodies and the process
of large concentration of ozone, oxides of converts it into vermicompost for the
nitrogen and various hydrocarbons. It betterment of human beings.
occurs in Los Angeles. 17. (b) Blue baby disease is caused by nitrate
10. (b) BOD is the measure of degree of impurity poisoning (presence of methaemoglobin)
of water due to organic matter. resulting in organs & cell tissues that are
11. (d) Pesticides in a drinking water pass into food deprived of oxygen & skin with the
chain and they increase in amount per unit characteristic blush tint.
EBD_7100
B-206 Topicwise AIIMS Solved Papers – BIOLOGY
18. (b) Stratosphere extends from 20-25 km above 24. (a) The gases that makes highest relative
tropospheres. It is also called ozonosphere contribution to the green house gases are
due to presence of ozone. Temperature carbon dioxide (CO2) and methane (CH4).
increases in this zone (upto 90°C) due to 25. (c) Lots of urea and phosphate fertilizer were
ozone formation. Ozone protects Earth from used in the crops in the vicinity and the
harmful ultraviolet radiations of the sun. lake water turned green and stinky. Due to
Oxides of Nitrogen + Unburnt Hydrocarbons this, lake near a village suffered heavy
mortality of fishes within a few days.
UV rays
O3 PAN 26. (d) DDT residues are rapidly passed through
(Ozone) (Peroxy acetyl nitrate) food chain causing biomagnification
19. (c) Biomagnification is phenomenon of because DDT is soluble in lipids.
Biomagnification, also known as
increasing concentration of a compound in
bioamplification or biological
the tissues of chain, usually as a result of
magnification, is the increase in
food intake. In this case, the concentration
concentration of a substance, such as the
of compound increases with increasing
pesticide DDT, that occurs in a food chain
trophic level.
as a consequence of:
Pollutant increases in concentration from • Persistence (slow to be broken down
Producers Primary consumers by environmental processes)
Secondary consumers Tertiary • Food chain energetics
consumers Top consumer. In the given • Low (or non-existent) rate of internal
question, only snake represents the tertiary degradation/excretion of the
consumer, so the concentration of DDT will substance (often due to water-
be highest in snake. insolubility)
20. (a) Excess fluorine in drinking water causes 27. (b) Lake A and B are identical in all aspects
hardening of bones and stiffness of joints, except that Lake A has higher temperature.
black foot disease is due to arsenic and On the basis of this difference, it is clear
itai-itai disease is due to cadmium in that Lake B has higher rate of oxygen
contaminated water. dissolution
21. (c) In the environment, ozone is known for its 28. (a) The Govt. of India through a new auto fuel
both harmful and useful effects. policy has laid out a roadmap to cut down
Ozone present in stratosphere is useful as the vehicular pollution in Indian cities. For
it act like shield and protect the earth from example, Euro II norms stipulate that
harmful UV-rays. Ozone present in sulphur be controlled at 350 ppm in diesel
troposphere act as pollutant and have many and 150 ppm in petrol.
deleterious effects. Type B : Assertion Reason Questions
22. (a)
23. (c) Solar energy coming to the earth is not 29. (a) Sound level of jet aeroplanes is about 150
responsible for green house effect. It is the dB beyond the level of human audible
increase in green house gases in capacity (80 dB). It affects the hearing and
atmosphere like CO2 which is released by general health of man.
complete combustion of fossil fuels or 30. (c) Organochlorine are persistent pesticides
biomass in industries or transportation (e.g. DDT) which pass into food chain and
vehicles that prevent the reradiation of increase in amount per unit weight of
infrared radiation from the earth and result organisms with the rise in trophic level.
Fenitrothion is organophosphate.
in increase in temperature of the earth.
Environmental Issues B-207

31. (a) DDT (dichloro diphenyl trichloroethane) is made. Natural pollution includes volcanic
a organochlorine contact insecticide that eruptions, soil erosion, UV- rays etc.
kills by acting as a nerve poison. DDT was Pollution is different from contamination.
originally used during world war II to Contamination is the presence of harmful
control typhus which was spread by body organisms causing disease.
louse. Since then it has been used to control 38. (b) Stratosphere zone of Earth's atmosphere
mosquito borne malaria and was used contains a layer of ozone which protects
extensively as a general agricultural us from harmful ultraviolet radiations of the
insecticide. sun.A group of chlorine containing
32. (b) Catalytic converters are involved in compounds called chlorofluoro carbon
reducing gaseous pollutions by converting (CFCs) used as coolants in air conditioners
CO CO2, NO2 N2 etc. Thus, and refrigerators are primary chemicals
decrease the amount of pollutant. They can responsible for ozone depletion. After their
not reduce emission of SPM. release into troposphere, CFCs go to
33. (b) Global warming is due to the increase in stratosphere where they are broken down
concentration of green house gases by UV radiations releasing chlorine.In
resulted in increase in global temperature. presence of sunlight, chlorine breaks O3
These global gases prevent the escape of into O2. Due to ozone depletion, its levels
long wave radiations into space. decreases by 67% every year causing
34. (b) Deforestation results in increase in green higher levels of UV radiations reaching
house gases which retains more and more earth which may cause eye cataracts, skin
UV radiations and leads to global warming. cancer etc.
Global warming is the warming/heating up 39. (a) Biotic or ecological succession is the
of the earth's atmosphere due to depletion formation of a series of biotic communities
of ozone in the stratosphere. at the same site over a period of time one
35. (c) Ozone in stratosphere is responsible for after the other, till a stable climax community
the protection of earth from high energy develops over the area. It occurs generally
UV rays i.e. it acts as life saving screen. in bare areas. Primary succession takes place
Ozone layer found in troposphere protects on a biological sterile soilless primary barren
from warming effect of earth. area. Secondary succession takes place in a
Due to human activities, the ozone layer in recently denuded area which still contains a
the stratosphere starts thinning, which is lot of organic debris, remains and propagules
also called ozone hole. Ozone hole is of previous living organisms. The area has
resulting in rain failure, increase in become bared due to destruction of the
radiation, cancer (skin) and reduction in community previously present. The baring
crop production. of an area can be caused due to forest fire,
36. (a) Wetland and rice fields are the major deforestation for wood, timber and habitat,
sources of methane. It is a green house overgrazing, landslides or earthquakes,
gas whose concentration is double now excessive and repeated droughts, following
than it was 250 years ago. a cropland and repeated floods.
37. (d) Pollution may be defined as an undesirable 40. (a) Excess of nitrates in drinking water are
change in physical, chemical or biological harmful for human health and may be fatal
characteristics of air, water and land for infants. Excessive use of fertilizers often
causing harmful effects on living leads to accumulations of nitrates in water.
organisms. Pollution can be natural or man In infants, excess nitrate reacts with
EBD_7100
B-208 Topicwise AIIMS Solved Papers – BIOLOGY
haemoglobin to form nonfunctional methane (CH4) and chlorofluoro carbons
methaemoglobin that impairs oxygen (CFCs) is 20% and 14% respectively. N2O
transport. This condition is termed as also contributes 6% in total global warming.
methaemoglobinemia or blue baby Efficient engine such as multi point fuel
syndrome. This disease can damage injection engine can reduce the unburnt
respiratory and vascular systems and even hydrocarbon (methane) in auto-emissions.
cause suffocation. 43. (b) Eutrophication is a natural process which
41. (a) Water pollutants are commonly measured literally means well nourished or enriched.
by their main common denominator, called It is a natural state in many lakes and ponds
BOD (Biochemical Oxygen Demand), i.e., which have a rich supply of nutrients.
the amount of free oxygen absorbed by Eutrophication becomes excessive, when
extraneous substances from water. If water abnormally high amount of nutrient from
is polluted, it will consume more oxygen, sewage, fertilizers, animal wastage and
thereby enhancing the BOD of water. detergent, enter streams and lakes causing
42. (b) Methane is produced by incomplete excessive growth or blooms of
biomass combustion, incomplete microorganisms. With increasing
decomposition mostly by anaerobic eutrophication, the diversity of the
methanogens. phytoplankton community of a lake
Carbon dioxide contributes about 60% of increases and the lake finally becomes
the total global warming and share of dominated by blue - green algae.
1 G.K. – History

1. Doctors before starting their service take oath 12. In which year was English recommended as the
in the name of a scientist to work honestly : medium of instruction for higher education in
(a) Hippocrate (b) Darwin [1997] India by Lord Macaulay? [2001]
(c) Plato (d) Socrates (a) 1833 (b) 1835
2. Confucianism is famous in : [1997] (c) 1859 (d) 1825
(a) Japan (b) China 13. 'Gayatri Mantra' is related with : [2002]
(c) Myanmar (d) Malaysia (a) Athar Veda (b) Rig Veda
3. The tomb of Qutub Shahi is situated in : [1997] (c) Yajur Veda (d) Sam Veda
(a) Aligarh (b) Agra 14. The founder of 'Khalsa' was : [2002]
(c) Hyderabad (d) Allahabad (a) Guru Gobind Singh
4. “Statue of Liberty” was gifted to USA by : [1997] (b) Guru Nanak Dev
(a) Germany (b) Canada (c) Guru Ram Das
(c) Greece (d) France (d) Guru Tegh Bahadur
5. The dynasty of Bahadur Shah (II) Jafar was in : 15. Who was the last Viceroy of India : [2002]
[1998] (a) Lard David
(a) 1658 to 1707 A.D. (b) 1837 to 1857 A.D. (b) Lord Wavell
(c) 1857 to 1862 A.D. (d) 1800 to 1829 A.D.
(c) Lord Mountbattern
6. Swami Vivekanand was born in which one of the
(d) Wellington
following year ? [1998]
16. Euthanasia (mercy killing) was first legalized in :
(a) 1860 (b) 1882
(a) Switzerland [2003]
(c) 1897 (d) 1863
(b) Netherlands (Holland)
7. The Simon Commission had come in which of
(c) France
the following year ? [1998]
(d) Italy
(a) 1937 (b) 1938
(c) 1927 (d) 1942 17. Which city was gifted to Charles II by the
8. The Fifth Day commission was headed by justice : Portuguese when he married the sister of the
(a) Verma (b) Ahmadi [1999] King of Portugal in 1662? [2003]
(c) Anand (d) Vadhwa (a) Bombay (b) Paris
9. Grand Trunk road was built by : [2000] (c) Lisoon (d) Castille
(a) Sher Shah Suri (b) Shah Jahan 18. Who is the mother of Bharat in the epic
(c) Lord Bentic (d) Lord Mount Battan 'Ramayana'? [2003]
10. At the first time, the song Vande Mataram has (a) Kaushalya (b) Sumitra
sung in : [2001] (c) Urmila (d) Kaikayee
(a) Indian National Congress session 1986 19. Which of the 'Nawab of Bengal' is supposed to
(b) Indian National Congress session 1896 be responsible for 'Back Hole Tragedy' of
(c) Quit India Movement 1942 Calcutta (Kolkata)? [2003]
(d) Congress session 1911 (a) Mir Jafer (b) Sirajuddaula
11. From where did Mahatma Gandhi start the (c) Alivardi Khan (d) Sarfaraj Khan
famous Dandi March? [2001] 20. Who among the following is known as Fuehrer?
(a) Surat (b) Mumbai (a) Stalin (b) Lenin [2003]
(c) Bardoli (d) Ahmedabad (c) Hitler (d) Bismarck
EBD_7100
C-2 Topicwise AIIMS Solved Papers – GENERAL KNOWLEDGE
21. Since the Britishers wanted India to produce and (a) Mahatma Gandhi
supply raw materials to feed English factories, (b) Jawaharlal Nehru
the pushed a policy that encouraged cultivation (c) Lokmanya Tilak
of : [2004] (d) Muhammad Iqbal
(a) jute (b) indigo 32. The earliest Iron Age in India has been
(c) cotton (d) commercial crop associated with [2010]
22. ‘Brahmo Samaj’ was founded by : [2005] (a) Painted grey ware
(a) Devendra Nath Tagor (b) Black and red ware
(b) Keshv Chandra Sen (c) Ochre Coloured Pottery (OCP)
(c) Raja Ram Mohan Roy (d) Northern black polished ware
(d) Annie Besant 33. Who among the following was the ruler of Delhi
23. Who was the first woman ruler of India? [2005] at the time of the invasion of Nadir Shah?[2010]
(a) Raziya Sultan (b) Noor Jahan (a) Muhammad Shah
(c) Chand Biwi (d) Durga Devi (b) Raj Raj I, the Chola
24. Who were the first to issue gold coins in India? (c) Bahadur Shah
[2005] (d) Alamgir II
(a) The Kushan's (b) The Gupta's 34. Most of the Ajanta Paintings were done during
(c) The Mauryan's (d) The Indo Greeks the period of :
25. Which sea is referred to in our National Anthem? (a) Harshavardhana (b) Guptas
(a) Bay of Bengal [2006]
(c) Mauryas (d) Kushan
(b) Indian ocean
35. Who was the first Indian ruler to face the
(c) Arabian sea
Mahmud Ghaznavi ? [2011]
(d) No sea is mentioned in it
(a) Prithviraj Chauhan (b) King Jaipal
26. The national calendar of India is based on :
(c) Jaichand (d) Firdausi
(a) Gragorian calendar [2006]
36. Which one of the following countries is “Orange
(b) Hizrr Era
(c) Saka- Era Revolution” associated ? [2012]
(d) one of the old Indian Era (a) Brazil (b) Sudan
27. Gaya is associated with Lord buddha, where he (c) Turkey (d) Ukraine
(a) was born [2007] 37. Who was the Governor General of India during
(b) attained enlightenment the Revolt of 1857 ? [2012]
(c) died (a) Lord Canning (b) Lord Dalhousie
(d) delivered his first sermon (c) Lord Dufferin (d) Lord Hardinge
28. Which of the following places was known as a 38. Who founded the Brahma Samaj? [2012]
centre of learning in ancient India? [2007] (a) Debendranath Tagore
(a) Nalanda (b) Ujjain (b) Keshab Chandra Sen
(c) Allahabad (d) none of these (c) Raja Rammohan Roy
29. In violation of the Salt Laws, Gandhiji started a (d) Ishwar Chandra Vidyasagar
movement called [2008] 39. Consider the following Mughal Emperors:
(a) Non-Cooperation Movement i. Farrukhshiyar ii. Jahandar Shah
(b) Swadeshi Movement iii. Muhammad Shah iv. Shah Alam II
(c) Civil Disobedience Movement What is the correct chronological sequence of
(d) None of the above their ascending the throne ? [2012]
30. The Cabinet Mission came to India in [2008] (a) i, ii, iii, iv (b) ii, i, iii, iv
(a) 1943 (b) 1944 (c) i, ii, iv, iii (d) ii, i, iv, iii
(c) 1945 (d) 1946 40. Match the following : [2013]
31. Who was the founder-editor of the famous A. Sarojini Naidu 1. Muslim League
newspaper ‘Kesari’ during the national struggle? B. M.A. Jinnah 2. Indian National
[2009] Congress
G.K. – History C-3

C. Sir Tej Bahadur 3. Hindu Mahasabha (c) 1919 - Jallianwala Bagh Massacre
Sapru (d) None of the above
D. V.D. Savarkar 4. Liberal Party 45. The first meeting of Indian National Congress
A B C D A B C D was held in Bombay in1885 A.D. under the
(a) 2 1 4 3 (b) 2 1 3 4 leadership of __? [2015]
(c) 2 4 1 3 (d) 4 1 3 2 (a) Dadabhai Naoroji
41. Identify the medical trio of Ancient India from (b) Sir C. Sankaran Nair
the following names. [2013] (c) Womesh Chandra Banerjee
(a) Charaka, Susruta and Vagbhata (d) Badruddin Tyabji
(b) Charaka, Vatsyayana and Vagbhata 46. When did Vasco-da-Gama come to India ? [2016]
(c) Charaka, Susruta and Bharata (a) 1492 (b) 1494
(d) Charaka, Susruta and Patanjali (c) 1496 (d) 1498
42. Who among the following first propounded the 47. The Qutub Minar at Delhi was built by ? [2016]
idea of Basic Education? [2014] (a) Qutbuddin Aibak (b) Shahjhan
(a) Jawahar Lal Nehru (c) Aleu-din-khilji (d) Chandragupta
(b) Raja Ram Mohan Roy 48. Who was the first Governor - General of India
(c) Mahatma Gandhi (a) Robert Clive [2016]
(d) Dayanand Saraswati (b) Sir Charles Metcalfe
43. 'Tebhega' movement is associated with which (c) William Bentinck
state ? [2015] (d) Warren Hastings
(a) Maharashtra (b) Tamil Nadu 49. ‘Do or Die’ (Karo ya Maro) slogan was given
(c) Karnataka (d) Bengal by which freedom fighter in 1942 ? [2017]
44. When was our national song sung for the first (a) Mahatma Gandhi
time? [2015] (b) Subhash Chand Bose
(a) 1896 session of Indian National Congress (c) Bal Gangadhar Tilak
(b) 1857 revolt (d) Bhagat Singh

ANSWER KEY
1 (a) 2 (b) 3 (c) 4 (d) 5 (b) 6 (d) 7 (c) 8 (a) 9 (a) 10 (b)
11 (d) 12 (a) 13 (b) 14 (a) 15 (c) 16 (b) 17 (a) 18 (d) 19 (b) 20 (c)
21 (b) 22 (c) 23 (a) 24 (d) 25 (d) 26 (c) 27 (b) 28 (a) 29 (c) 30 (d)
31 (c) 32 (a) 33 (a) 34 (b) 35 (b) 36 (d) 37 (a) 38 (c) 39 (b) 40 (a)
41 (d) 42 (c) 43 (d) 44 (a) 45 (c) 46 (d) 47 (a) 48 (d) 49 (a)
EBD_7100
C-4 Topicwise AIIMS Solved Papers – GENERAL KNOWLEDGE

2 G.K. – Indian Polity

1. Sixty five year plan to promote significant (a) C. Raj Gopalachari (b) Radha Krishana
expansion of employment opportunities was (c) Y. C. Grace (d) V. V. Giri
scheduled in the year of : [1998] 10. National Anthem "Jana Gana Mana" was
(a) 1980-1985 (b) 1669-1674 adopted on [2001]
(c) 1974 -1979 (d) 1961-1966 (a) 26 Jan. 1950 (b) 26 July 1947
2. Sixty first amendment in the Constitution states (c) 15 August 1947 (d) 24 Jan. 1950
about [1998] 11. Who is the chairman of Rajya Sabha? [2001]
(a) Extended President rule in Punjab (a) Speaker of Lok Sabha
(b) Increasing the ceibling of profession tax (b) Home Minister
(c) Reservation of seats for scheduled castes (c) President
and scheduled tribes in parliament (d) Vice President
(d) Reducing the voting age from 21 years to 12. Which one of the following determines the
18 years salary of attorney General? [2001]
3. Which of the following state, becomes the 22nd (a) Speaker of Lok Sabha
state of Indian union ? [1998] (b) Home Minister
(a) Tripura (b) Sikkim (c) President of India
(c) Meghalaya (d) Assam (d) Prime Minister
4. At present, the total membership of Lok Sabha :
13. The oath of office to a Supreme Court Judge is
(a) 552 (b) 525 [1998]
administered by : [2001]
(c) 527 (d) 545
(a) The Chief Justice
5. Fundamental duties were introduced in the
(b) The President of India
constitution by : [1999]
(c) The Chief Justice of India
(a) 42nd amendment (b) 40th amendment
(d) The Law minister
(c) 48th amendment (d) 53rd amendment
6. The person who served as the President of India 14. Who addressed the U. N. General Assembly for
twice, was : [1999] the first time in Hindi? [2001]
(a) Radha Krishnan (a) Rajendra Prasad (b) Atal Bihari Vajpai
(b) Dr. Rajendra Prasad (c) Jawahar Lal Nehru (d) Swarn Singh
(c) Zakir Hussain 15. How many languages are recognised by the
(d) V.VGiri constitution of India in the 8th schedule? [2002]
7. President of India gives his resignation to the : (a) 12 (b) 14
(a) Chief Justice (b) Parliament [1999] (c) 16 (d) 18
(c) Vice President (d) Prime Minister 16. Who is known as the 'Iron Man of India'? [2002]
8. The contribution of sarkaria commission was (a) Jawaharlal Nehru
related between : [2000] (b) Bal Gangadhar Tilak
(a) state and centre (c) Sardar Vallabhbhai Patel
(b) centre and union territories (d) Mahatma Gandhi
(c) one state to other state 17. The design of the National Flag was adopted by
(d) none of these the constituent assembly of India on : [2002]
9. Who was appointed as the first governor general (a) 26 January, 1949 (b) 26 January, 1950
of India ? [2000] (c) 22 July, 1947 (d) 15 August, 1947
G.K. – Indian Polity C-5

18. When was the golden jubilee of Indian Parliament 29. Who is the constitutional head of the Government
celebrated ? [2002] of India ? [2008]
(a) 1st January, 1997 (b) 26th January, 2002 (a) President
(c) 13th may, 2002 (d) 15th August, 1997 (b) Prime Minister
19. Dravida Munnetra Kargam (DMK) was founded (c) Chief Justice of India
by : [2004] (d) Attorney General
(a) M.G. Ramachandran 30. Which part of Indian Constitution has been
(b) C.N. Annadurai described as the soul of the Constitution ?
(c) Kumar Swami Kamraj (a) Fundamental Rights [2008]
(d) Lalithambika Antharjanam (b) Directive Principle of State Policy
20. The Planning Commission was set up in : [2005] (c) The Preamble
(a) January, 1950 (b) March, 1950 (d) Right to Constitutional Remedies
(c) January, 1952 (d) March, 1952 31. The President of India can nominate [2008]
21. How many spokes are in Indian National Flag? (a) 10 members to Rajya Sabha
(a) 22 (b) 24 [2005] (b) 2 members to Rajya Sabha
(c) 26 (d) 28 (c) 15 members to Rajya Sabha
22. Who was elected as the permanent President of (d) 12 members to Rajya Sabha
constituent assembly ? [2005]
32. The Constitution of India was adopted by the
(a) Dr. Sachchidanand Sinha
Constituent Assembly on [2008]
(b) Dr. Rajendra Prasad
(a) August 15, 1947
(c) Dr. B. R. Ambedkar
(b) June 30, 1948
(d) C. Rajgopalachari
(c) November 26, 1949
23. How many articles and schedule are there in
(d) January 26, 1950
originally constitution ? [2005]
(a) 391 articles and 7 schedules 33. Who is the Chairman of the Planning Commission ?
(b) 395 articles and 8 schedules (a) President [2008]
(c) 400 articles and 10 schedules (b) Prime Minister
(d) 444 articles and 12 schedules (c) Finance Minister
24. The Tenure of first planning holiday was [2005] (d) Governor of Reserve Bank
(a) 1964 -1967 (b) 1965 -1968 34. Which Plan give emphasis on removal of poverty
(c) 1966 -1969 (d) 1978 -1981 for the first time? [2009]
25. World Trade Organisation established in : [2005] (a) Fourth (b) Fifth
(a) 1954 (b) 1988 (c) Sixth (d) Seventh
(c) 1994 (d) 1995 35. The Council of Ministers does not include
26. The supreme command of the defence forces is (a) Cabinet Ministers [2009]
vested with the : [2006] (b) Ministers of State
(a) Field Marshal (c) Cabinate Secretary
(b) Commander-in-chief (d) Ministers without portfolio
(c) Prime minister 36. Attorney -General of India is appointed by
(d) President of India (a) Chief Justice of Supreme Court [2010]
27. India's first battle field missile is : [2006] (b) Parliament
(a) Akash (b) Prithvi (c) Law Minister
(c) Agni (d) Nag (d) President
28. Who conducts the State assembly elections? 37. Which one of the following Article provide
[2007] ‘Right to equality’? [2010]
(a) Chief Justice of the High Court concerned (a) Article - 14
(b) Chief Justice of the Supreme Court (b) Article -19
(c) Chief Election Commissioner (c) Article - 20
(d) Governor of the state concerned (d) Article - 18
EBD_7100
C-6 Topicwise AIIMS Solved Papers – GENERAL KNOWLEDGE
38. First speaker of Lok Sabha was : [2011] Choose the correct option from the codes given
(a) Sardar Hukum Singh below:
(b) G. V. Mavlankar (a) Only 1 (b) Only 2
(c) Neelam Sanjeevan Reddy (c) Only 1 & 2 (d) Only 1 & 3
(d) Bali Ram Bhagat 43. In Lok Sabha of India, the "Leader of the House"
39. What is the intermediate tier of the Panchayati is nominated by ___: [2015]
Raj System called? [2012] (a) President (b) Lok Sabha Speaker
(a) Zilla Parishad (b) Taluka Panchayat (c) Prime Minister (d) Deputy Speaker
(c) Panchayat Samiti (d) Gram Sabha 44. The President of India can nominate how many
40. Which of the following is associated with members to Rajya Sabha and Lok Sabha
Panchayati Raj ? [2013] respectively? [2015]
(a) Nanavati Commission (a) 10, 3 (b) 12, 2
(b) Balwant Rai Mehta Committee (c) 10, 2 (d) 12, 3
(c) Librahan Commission 45. Supreme Court judge retires upon attaining the
(d) Shah Commission age of [2016]
41. How many articles are there in the Indian (a) 65 years (b) 60 years
constitution? [2014] (c) 55 years (d) 50 years
(a) 395 (b) 396 46. How many times has National Emergency been
(c) 398 (d) 399 declared? [2016]
42. The Vice President of India is elected by the (a) Six times (b) Three times
electoral collage consisting of the members of (c) Five times (d) Four times
_______? [2015] 47. A Municipal Corporation is set up in a city with
1. Rajya Sabha how much population of not less than? [2016]
2. Lok Sabha (a) 2 lakhs (b) 5 lakhs
3. State Legislatures (c) 10 lakhs (d) 15 lakhs

ANSWER KEY
1 (b) 2 (a) 3 (c) 4 (a) 5 (a) 6 (b) 7 (b) 8 (a) 9 (a) 10 (d)
11 (d) 12 (c) 13 (c) 14 (b) 15 (d) 16 (c) 17 (c) 18 (c) 19 (b) 20 (b)
21 (b) 22 (b) 23 (b) 24 (c) 25 (d) 26 (d) 27 (b) 28 (c) 29 (a) 30 (d)
31 (d) 32 (c) 33 (b) 34 (a) 35 (c) 36 (d) 37 (a) 38 (b) 39 (c) 40 (b)
41 (a) 42 (c) 43 (c) 44 (b) 45 (a) 46 (b) 47 (c)
G.K. – Geography C-7

3 G.K. – Geography

1. Dachigan Wild life santuary in Kashmir is (a) The factory where India’s newly designed
associated with which of the following animal ? battle tanks are being manufactured
(a) Hangul (b) Panther [1997] (b) The samadhi of Indira Gandhi
(c) Horned toed Deer (d) Sagui (c) The nuclear reactor at KalpaKam at Chennai
2. ‘Principality of Liechtenstion’ is situated (d) none of these
between Switzerland and [1997] 12. Khushi Nagar, the famous Buddhist pilgrimage
(a) Austria (b) France centre in the state of : [2000]
(c) Italy (d) Germany (a) U.P. (b) M.P
3. The largest producer of rubber is : [1997] (c) Bihar (d) Orissa
(a) Sri Lanka (b) India 13. Nasic is situated on the bank of : [2000]
(c) Japan (d) Malaysia (a) Narmada (b) Krishna
4. Kalpakkam in Tamilnadu is known for its : (c) Kauvery (d) Godavari
(a) temples [1997] 14. Which city is known as Pink city ? [2000]
(b) textiles mills (a) Jaipur (b) Paris
(c) handicrafts (c) New York (d) London
(d) atomic power plants 15. An indian river, that does not form any delta is :
5. OPEC is : [1997] (a) Cauvery (b) narmada [2002]
(a) Organisation of petroleum Exporting (c) Yamuna (d) Singh
Companies 16. How many islands are there in lakshadweep?
(b) Oil and Petroleum exporting Corporation (a) 47 (b) 36 [2002]
(c) Oil and petroleum Exporting Countries (c) 27 (d) 17
(d) Organisation of Petroleum Exporting 17. From which of the following places the
Countries international dateline crosses? [2003]
6. Sambalpur is situated on the bank in which of (a) Atlantic ocean
the following river ? [1998] (b) Pacific ocean
(a) Jammu (b) Saraswati (c) Greenwich
(c) Sagar (d) Mahandi (d) Gape of Good Hope
7. Seoul is the capital of : [1999] 18. A man-made tunnel in India transfers water from
(a) Japan (b) South Korea which one river to another? [2004]
(c) Afganistan (d) Philippines (a) Narmada to Tapti
8. Hirakund dam is constructed on which of the (b) Betwa to Sone
following river ? [1999] (c) Beas to Sutlej
(a) Mahanadi (b) Ganga (d) Godawari to Krishna
(c) Yamuna (d) Kosi 19. The grand Fisher Bank is situated off : [2004]
9. ‘Sun city’ is in : [2000] (a) the coast of New Foundland
(a) USA (b) South Africa (b) the Chilean coast
(c) France (d Denmark (c) the Spanish coast
10. Which one of the least populated state in India? (d) the coast of Great Britain
[2000] 20. Which one of the following organisation's iron
(a) Nagaland (b) Himachal Pradesh and steel plant was build to use charcoal as a
(c) Orissa (d) Sikkim source of power, to start with, but later switched
11. Shakti sthal is the name given to : [2000] over to hydroelectricity ? [2004]
EBD_7100
C-8 Topicwise AIIMS Solved Papers – GENERAL KNOWLEDGE
(a) The Tata Iron Steel Company 31. ‘Equinox’ means [2007]
(b) The Indian Iron and Steel Company (a) days are longer than nights
(c) Mysore Iron and Steel Company (b) days and nights are equal
(d) Hindustan Steel Limited (c) days are shorter than nights
21. Rishikesh is famous for the production of (d) none of the above
(a) antibiotics [2004] 32. Which country leads in production of aluminium
(b) heavy electricals and aluminium goods ? [2007]
(c) fertilizers (a) Australia (b) U.S.
(d) transistorized radios (c) Russia (d) Japan
22. Nepanagar is famous for : [2004] 33. Which is an ore of aluminium? [2007]
(a) paper board industries (a) chromite (b) cuprite
(b) craft paper industries (c) bauxite (d) siderite
34. The southern tip of India is [2008]
(c) carbon paper industries
(a) Cape Comorin (Kanyakumari)
(d) newsprint paper industries
(b) Point Calimere
23. Where is 'Indira Gandhi Rashtriya Uran
(c) Indira Point in Nicobar Islands
Akadami's situated? [2004] (d) Kovalam in Thiruvananthapuram
(a) Dehradun (b) Raebareli 35. The major coffee producing state in India is
(c) Allahabad (d) Mussoorie (a) Kerala (b) Karnataka [2008]
24. Where is 'National Defence Academy' situated? (c) Tamil Nadu (d) West Bengal
[2005] 36. Bauxite is an ore of [2008]
(a) New Delhi (b) Khadakvasla (a) Aluminium (b) Boron
(c) Dehradun (d) Pune (c) Lead (d) Silver
25. Which river is called 'Bengal's Sorrow'? [2005] 37. Which of the following methods is used to
(a) Hughli (b) Ganga determine the age of the earth ? [2008]
(c) Damodar (d) Koshi (a) Carbon dating (b) Germanium dating
26. 'Maoris' are : [2005] (c) Uranium dating (d) All of the above
(a) inhabitants of New Zealand 38. The coastal tract of Andhra Pradesh and Tamil
(b) inhabitants of Hungary Nadu is called [2009]
(c) inhabitants of North America (a) Konkan (b) Coromandel
(d) inhabitants of North-West Asia (c) East Coast (d) Malabar Coast
27. 'Chittaranjan' is famous for the [2005] 39. Diamond bearing rocks occur in [2010]
(a) railway coaches (b) locomotives (a) Panna of Madhya Pradesh
(c) iron and steel (d) heavy machinery (b) Mysore of Karnataka
28. Sardar Sarovar Dam is built on the river [2006] (c) Waltair of Andhra Pradesh
(a) Jhelam (b) Narmada (d) Ajmer of Rajasthan
(c) Tapti (d) Vyas 40. Highest mustard seed producing state is[2010]
29. Lagoon refers to : [2006] (a) Maharashtra (b) Rajasthan
(a) A full moon (c) U.P. (d) Gujarat
(b) The sea breaking into the land and then 41. The highest city of the world is- [2011]
separated by the sand dunes (a) Wan Chan (China) (b) La Paz (Bolivia)
(c) A spot in a desert made fertile by presence (a) Tokyo (Japan) (d) New York (USA)
42. Kalahari desert is in [2011]
of water
(a) North Africa (b) South Africa
(d) Horse shoe shaped corel reef
(c) South America (d) Australia
30. Ecology deals with : [2006]
43. Which of the following latitudes passes through
(a) The earth and planets
India ? [2011]
(b) The relationship between organisms and (a) Equator
environment (b) Arctic Circle
(c) The life under the sea (c) Tropic of Capricorn
(d) Economical growth of poor people (d) Tropic of Cancer
G.K. – Geography C-9

44. Ghataprabha is a tributary of which one of the (b) magnetic poles


following rivers ? [2012] (c) geographic north pole
(a) Ganga (b) Indus (d) latitude 45º
(c) Godavari (d) Krishna 52. Which place in India is called “The Golden Mine
45. Kiel Canal connects [2012] of Liverworts”? [2014]
(a) Baltic Sea and North Sea (a) Eastern Himalayas
(b) Red Ocean and Mediterranean Sea (b) Western Himalayas
(c) Caribbean Sea and Pacific Sea (c) Western Ghats
(d) Atlantic Ocean and Pacific Ocean (d) Eastern Ghats
46. The Victoria Falls in Africa is located on which 53. As per data, which two Indian cities are at
river? [2012] highest risk of being devastated by an earthquake?
(a) Zaire (b) Orange (a) Guwahati and Nagpur [2015]
(c) Zambezi (d) Niger (b) Guwahati and Srinagar
47. The project similar to T.V.A. (Tennessee Valley (c) Jamnagar and Guwahati
Authority) of U.S.A. in India is [2013] (d) Nagpur and Srinagar
(a) Damodar Valley Project 54. Which is the hottest planet of the solar system.
(b) Mahanadi Delta Project (a) Mercury (b) Venus [2016]
(c) Ramganga Multipurpose Project (c) Jupiter (d) Saturn
(d) Idukki Project 55. Which one of the following zones of the
48. Catch crops are [2013] atmosphere is rich in Ozone gas [2016]
(a) crops palnted to attract certain insect pests (a) Mesosphere (b) Troposphere
to be destroyed (c) Stratosphere (d) Ionosphere
(b) crops planted to attract certain useful insects 56. A sand deposit extending into the mouth of a
to be used for biological control of pests bay is a ? [2016]
(a) Headland (b) Sea Stack
(c) crops to be cut and fed green to the cattle
(c) Split (d) Berm
(d) substitute crops planted after the regular
57. Which is the Highest milk producer in India?
crop has failed.
[2016]
49. Which one of the following National Park/
(a) Madhya Pradesh (b) Andhra Pradesh
Sanctuary is not in Rajasthan ? [2013]
(c) Uttar Pradesh (d) Rajasthan
(a) Sariska National Park
58. Which is the longest bridge in India? [2017]
(b) Sambar Wildlife Sanctuary
(a) Bandra Worli Sea Link
(c) Rajaji National Park (b) Mahatma Gandhi Setu
(d) Rhanthambore National Park (c) Dhola-Sadia Bridge
50. The black part of the moon is always calm and (d) Arrah-Chhapra Bridge
dark which is called [2014] 59. Asia’s longest bi-direction road tunnel is located
(a) Sea of tranquility (b) Ocean of storms in—————. [2017]
(c) Area of storms (d) None of these (a) Jammu & Kashmir
51. The earth’s magnetic field always has a vertical (b) Sikkim
component except at the [2014] (c) Maharashtra
(a) magnetic equator (d) Himachal Pradesh

ANSWER KEY
1 (a) 2 (a) 3 (d) 4 (d) 5 (d) 6 (a) 7 (b) 8 (a) 9 (b) 10 (a)
11 (b) 12 (a) 13 (d) 14 (a) 15 (b) 16 (b) 17 (b) 18 (c) 19 (a) 20 (a)
21 (a) 22 (d) 23 (b) 24 (b) 25 (c) 26 (a) 27 (b) 28 (b) 29 (b) 30 (b)
31 (b) 32 (c) 33 (c) 34 (a) 35 (b) 36 (a) 37 (c) 38 (b) 39 (a) 40 (b)
41 (b) 42 (b) 43 (d) 44 (d) 45 (a) 46 (c) 47 (a) 48 (d) 49 (c) 50 (a)
51 (a) 52 (b) 53 (b) 54 (b) 55 (c) 56 (c) 57 (c) 58 (c) 59 (a)
EBD_7100
C-10 Topicwise AIIMS Solved Papers – GENERAL KNOWLEDGE

4 G.K. – General Science

1. Halleys comet will be seen in : [1997] 10. Which one of the following function of the
(a) 2062 (b) 2060 platelets occurs in our body? [2001]
(c) 2068 (d) 2066 (a) It helps in breathing
2. Skylab was launched into space by the US in : (b) It helps in strengthening of gums
(a) 1975 (b) 1974 [1997] (c) It helps in circulation of blood
(c) 1973 (d) 1979 (d) It helps in clotting of blood
3. Which one of the following is responsible for 11. Which one of the following vitamin can be most
the disease ‘dropsy’ ? [1999] easily synthesised inthe human body? [2001]
(a) Argemone maxicana (a) Vitamin B (b) Vitamin C
(b) Brassica oleracea (c) Vitamin A (d) Vitamin D
(c) Oenothera lamarckiana 12. The spinning of the earth on its imaginary axis
(d) Brassica campestris is known as : [2002]
4. Which of the following vitamin is required in (a) rotation (b) circulation
bone formation ? [1999] (c) orbiting (d) revolution
(a) D (b) B 13. Which organ of the body purifies the blood ?
(c) C (d) A (a) Heart (b) Lungs [2002]
5. Ecology is the branch of science which deals with : (c) Kidneys (d) Pancreas
[1999] 14. Positron Emission Tomography (PET) is one of
(a) cell structure (b) soils surface the best methods of functional imaging because
(c) balance of nature (d) human anatomy [2003]
6. The disease rheumatism effects : [1999] (a) isotopes of basic body elements are used
(a) legs (b) ears for imaging
(c) lungs (d) joints (b) isotopes with long half-lives are used
7. Weight of blood in the body is : [2000] (c) isotopes with short half-lives are used
(a) about 7 litres in normal body of 7% of the (d) positrons are directly involved used in
total body weight imaging
(b) about 5 litres in normal body or 5% of the 15. Magnetic Resonance (MR) images are derived
total body weight from the proton -bearing species present
(c) about 10 litres in normal body or 10% of principally from water and [2003]
the body weight (a) long alkane chain protons of the fatty acid
(d) none of these moieties
8. The earthquake is measured by : [2001] (b) short alkane chain protons of the fatty acid
(a) Lactometer (b) Seismograph moieties
(c) Hygrometer (d) Barometer (c) long alkene chain protons of the fatty acid
9. AIDS is caused by : [2001] moieties
(a) Helminth (b) Protozoa (d) short alkene chain protons of the fatty acid
(c) Virus (d) Bacteria moieties
G.K. – General Science C-11

16. The following separation technique depends on 26. Biometry refers to : [2006]
the molecular size of the protein : [2003] (a) Identification of humans by scanning face
(a) chromatography on a carboxymethyl (CM) and fingerprints
cellulose column (b) Measurement of mechanical displacement
(b) iso-electric focusing in humans
(c) gel filtration chromatography (c) A method of lie detection
(d) chromatography on a diethylaminoethyl (d) Body length relationships across the
(DEAE) cellulose column evolutionary scale
17. The approximate number of genes contained in 27. Which one of the following is one of the two
the genome of Kalpana Chawala was : [2003] days when the sun rises exactly in the east?
(a) 40,000 (b) 30,000 (a) 14th January (b) 21st March [2006]
(c) 80,000 (d) 1,00,000 (c) 21st June (d) 23rd December
18. Thanatology is the science that deals with 28. X-rays were discovered by : [2006]
(a) death in all its aspects [2003] (a) Wilhelm K. Roentgen
(b) solving paternity of child (b) H. Kissinger
(c) identification of living (c) Sir C.V. Raman
(d) detection of lie (d) Meghnad Saha
19. The disease, Tetanus also known as [2003] 29. One ream of paper equal to : [2006]
(a) Gangrene (b) Shingles (a) 100 –110 sheets
(c) Lockjaw (d) Whooping cough (b) 256 sheets
20. It lives underwater for up to three years as (c) 180 -500 sheets
nymph before emerging as a flying insect, Fossils (d) 1000 sheets
of this insect dating back about 300 million years 30. Which of the following gases is most toxic?
have been found : [2004] [2006]
(a) scorpion fly (b) stone fly (a) Carbon dioxide (b) Carbon monoxide
(c) caddis fly (d) May fly (c) Sulpher dioxide (d) None of these
21. Rayon fibre is manufactured from : [2004] 31. The gas used in the manufacture of vanaspati
(a) petroleum (b) wood and pulp ghee is : [2006]
(c) chemicals (d) naphtha (a) Helium (b) Oxygen
22. Maria Montessori's name is associated with : (c) Nitrogen (d) Hydrogen
(a) Christian mission [2004] 32. Which mirror is used as a rear view mirror in
(b) Child education vehicles? [2007]
(c) Women's rights (a) plain (b) convex
(d) Mission hospitals (c) concave (d) spherical
23. The noise produced in office is normally at the 33. Chemical change does not take place in
level of : [2004] (a) souring of milk into curd [2007, 2011]
(a) 20 db (b) 30 db (b) rusting of iron in atmosphere
(c) 40 db (d) 60 db (c) burning of magnesium ribbon in air
24. Ascorbic acid is the chemical name of : [2005] (d) emitting of light by a red hot platinum wire
(a) vitamin A (b) vitamin B 34. The process of transfer of heat by matter but
(c) vitamin C (d) vitamin D without actual movement of the particles
25. The chemical formulae of Plaster of Paris' is themselves is called [2007]
(a) CaSO4. 2H2O (b) Ca(OH)2 [2005] (a) conduction (d) convection
(c) CaSO4. 1/2H2O (d) C2H5OH (c) radiation (b) none of these
EBD_7100
C-12 Topicwise AIIMS Solved Papers – GENERAL KNOWLEDGE
35. Only zero and one are used for operating 47. The purest form of water is [2009]
[2007, 2011] (a) Tap water (b) Rain water
(a) Calculator (b) Computer (c) Ground water (d) Distilled water
(c) Abacus (d) Type writer 48. Which of the following means ‘change’ in Greek
36. Transistor is [2007] (a) Orangutam (b) Goat [2009]
(a) semi conductor (b) inductor (c) Amoeba (d) Hydra
(c) modulator (d) demodulator 49. The velocity of light in a medium is maximum for
37. Computer cannot [2007] that colour for which refractive index is [2010]
(a) send message (b) read files (a) minimum (b) maximum
(c) abstract thought (d) play music (c) optimum (d) very high
38. Which of the following is not a carbohydrate ? 50. Which of the following is/ are the law(s) of
(a) wax (b) starch [2007] reflection of light? [2010]
(c) sucrose (d) maltose (a) The angle of incidence is equal to the angle
39. Which of the following is an eye disease? of reflection.
(a) hepatitis (b) measles [2007] (b) The incident ray, the normal to the mirror at
(c) glaucoma (d) bronchitis the point of incidence and the reflected ray,
40. Which of the following is the vaccine for all lie in the same plane.
tuberculosis? [2007] (c) Both (a) and (b).
(a) OPT (b) BCG (d) The angle of incidence is never equal to
(c) salk vaccine (d) rubella vaccine the angle of reflection.
41. Horns, nails and hair are [2007] 51. Which of the following halo-organic compounds
(a) soluble fats is used as insecticide, germicide, soil fumigant
(b) insoluble carbohydrates and deodorant? [2010]
(c) keratin proteins (a) Benzene hexachloride
(d) complex lipids (b) Chlorobenzene
42. If the blood group of one parent is AB and that (c) p-chlorobenzene
of the other O, the possible blood group of their (d) All of the above
child would be [2008] 52. Electronic configuration of deuterium atom is
(a) A or B (b) A or B or O (a) 1s2 (b) 2s1 [2010]
(c) A or AB or O (d) A or B or AB or O (c) 1s 1 (d) 2s2

43. The vitamin that helps to prevent infections in 53. Which of the material interactions will not
the human body is [2008] promote co-evolution? [2010]
(a) vitamin A (b) vitamin B (a) Commensalism (b) Parasitism
(c) vitamin C (d) vitamin D (c) Heterothallism (d) All of these
44. The gas which turns into liquid at the lowest 54. Bence Jones protein is associated with
temperature among the following is [2008] (a) Lymphosarcoma
(a) hydrogen (b) oxygen (b) Leukaemia
(c) helium (d) nitrogen (c) Multiple myeloma
45. Which of the following disease is caused by a (d) All of these
virus ? [2008] 55. A MODEM is connected in between a telephone
(a) Diphtheria (b) Malaria line and a [2010]
(c) Cholera (d) Hepatitis (a) Serial port
46. An ECG shows the functioning of the [2009] (b) Computer
(a) Brain (b) Heart (c) Network
(c) Lungs (d) Kidneys (d) Communication adaptor
G.K. – General Science C-13

56. A sequence of precise and unambiguous (b) hypoactivity of thymus


instructions for solving a problem in a finite (c) hypoactivity of thyroid
number of operations is referred as [2010] (d) hyperactivity of thymus
(a) algorithm (b) address 67. Which one among the following is the correct
(c) advice (d) all of these order of power consumption for light of equal
57. Polio vaccine was first prepared by [2011] intensity ? [2014]
(a) J. Salk (b) L. Pasteur (a) CFL tube < Fluorescent tube < Incan-
(c) G. J. Mendel (d) Watson descent bulb < Light emitting diode
58. The Saturn rings were discovered by [2011] (b) Light emitting diode < CFL tube < Fluores-
(a) Copernicus (b) Newton cent tube < Incandescent bulb
(c) Galileo (d) none of these (c) CFL tube < Fluorescent tube < Light emit-
ting diode < Incandescent bulb
4
59. coulomb of charge contains ________ (d) Incandescent bulb < Light emitting diode
25
< Fluorescent tube < CFL tube
electrons. [2011] 68. When a particle and an antiparticle come in
(a) 1015 (b) 1018 contact with each other, they [2014]
(c) 1020 (d) none of these (a) repell each other
60. Fish can survive inside deep frozen ponds (b) annihilate each other
because [2011] (c) go undisturbed
(a) Fish are cold blooded (d) spin about a common axis
(c) They hibernate 69. Photoelectric effect is [2014]
(c) Ice is a good conductor of heat (a) an instantaneous process
(d) There is water at 4°C below frozen ice (b) delayed process
61. Which one of the following diseases is not (c) emission of protons
caused by virus? [2012] (d) emission of neutrons
(a) Chicken pox (b) Measles 70. Candles contains a mixture of [2014]
(c) Poliomyelitis (d) Tetanus (a) Bees wax and paraffin wax
62. In the plant body, the water and minerals are (b) Bees wax and stearic acid
transported by: [2012] (c) Paraffin wax and stearic acid
(a) Bast (b) Collenchyma (d) Higher fatty acid
(c) Parenchyma (d) Xylem 71. In chemical terms, what are alums used for
63. What is the main constituent of natural gas ? purifying water for drinking purposes ? [2014]
(a) Methane (b) Ethane [2012] (a) Hydrated chlorides
(c) Butane (d) Hydrogen (b) Double nitrate
64. Which one among the following has the highest (c) Double sulphates
first ionisation energy ? [2012] (d) Nitrates of aluminium
(a) Carbon (b) Fluorine 72. Each body segment of Earthworm is called [2014]
(c) Nitrogen (d) Oxygen (a) Proglottid (b) Metamere
65. Chemical weathering is at its maximum in : (c) Scolex (d) Rostellum
(a) hot and dry regions [2012] 73. The hydraulic brake used in automobiles is a
(b) cold and humid regions direct application of [2014]
(c) hot and humid regions (a) Archimedes’ principle
(d) cold and dry regions (b) Torricellean law
66. Grave's disease is caused due to: [2013] (c) Bernoulli’s Theorem
(a) hyperactivity of thyroid (d) Pascal’s law
EBD_7100
C-14 Topicwise AIIMS Solved Papers – GENERAL KNOWLEDGE

74. Of the following, which is the fastest? [2014] (a) Tin (b) Mercury
(a) CD-ROM (b) RAM (c) Lead (d) Zinc
(c) Registers (d) Cache 80. The Bipolar disorder is related to which among
75. ‘Splenic fever’ is another name for [2014] the following? [2015]
(a) FMD (b) Anthrax (a) Heart (b) Lungs
(c) Cow pox (d) Mastitis (c) Brain (d) Liver
76. Big Bang theory explains _______? [2015] 81. Name the first antibiotic medicine discovered.
(a) Origin of Universe (b) Origin of Sun. [2016]
(c) Laws of physics. (d) None of above. (a) Penicillin (b) Auromycin
77. Which are the main gases present in Sun? (c) Streptomycin (d) Ampicilin
(a) Hydrogen and Helium [2015] 82. The disease scarlet fever is caused by [2016]
(b) Hydrogen and Argon (a) culex mosquito
(c) Argon and Helium (b) housefly
(d) Hydrogen and Carbon Dioxide (c) anopheles mosquito
78. Which of the following is a non metal that (d) haemolytic streptococcal infection
remains liquid at room temperature? [2015] 83. In India, person unable to count fingers from a
(a) Phosphorous (b) Bromine distance of how many metres is categorised as
(c) Chlorine (d) Helium blind? [2017]
79. Which of the following metals forms an amalgam (a) 3 (b) 6
with other metals? [2015] (c) 7 (d) 8

ANSWER KEY
1 (a) 2 (c) 3 (a) 4 (a) 5 (c) 6 (d) 7 (a) 8 (b) 9 (c) 10 (d)
11 (d) 12 (a) 13 (b) 14 (c) 15 (a) 16 (c) 17 (b) 18 (a) 19 (c) 20 (b)
21 (b) 22 (b) 23 (c) 24 (c) 25 (c) 26 (b) 27 (b) 28 (a) 29 (c) 30 (b)
31 (d) 32 (b) 33 (d) 34 (a) 35 (c) 36 (a) 37 (c) 38 (a) 39 (c) 40 (b)
41 (c) 42 (a) 43 (c) 44 (a) 45 (d) 46 (b) 47 (d) 48 (c) 49 (a) 50 (c)
51 (c) 52 (a) 53 (a) 54 (d) 55 (b) 56 (a) 57 (a) 58 (c) 59 (b) 60 (d)
61 (d) 62 (d) 63 (a) 64 (b) 65 (c) 66 (a) 67 (b) 68 (b) 69 (a) 70 (c)
71 (c) 72 (d) 73 (d) 74 (c) 75 (b) 76 (a) 77 (a) 78 (b) 79 (b) 80 (c)
81 (a) 82 (d) 83 (b)
G.K. – General Awareness C-15

5 G.K. – General Awareness

1. In which of the following year Olympic Games 12. The largest airport in the world is situated in
were not played ? [1997] (a) Saudi (b) USSR [1998]
(a) 1936 (b) 1925 (c) Russia (d) Denmark
(c) 1916 (d) 1932 13. Martyr day is observed on : [1998]
2. The male cricketer who scored maximum runs in (a) 18 April (b) 12 May
one day cricket match : [1997] (c) 29 August (d) 30 January
(a) Azharuddin (b) Vivian Richards 14. National Housing Bank is subsidary of :[1998]
(c) Sachin Tendulkar (d) Sayeed Anwar (a) R.B.I. (b) I.D.B.I.
3. Writer of Future Shock is [1997] (c) I.C.I.C.I. (d) S.B.I.
(a) Bernad shaw (b) Bertrand 15. What is Nikkie? [1998]
(c) Sewitzer (d) Alwyn Toffler (a) Index of shares in Tokyo stock exchange
4. Deep Blue is a : [1997] (b) Rate of interest by Bank of tokyo
(a) computer which gives weather report (c) A private firm situated in Japan
(b) computer operating system (d) Currency of Korea
(c) blue whale 16. ‘Frank Worrel was associated with which of the
(d) computer which plays chess following sport ? [1998]
5. The ‘AIDS’ day is observed on : [1997] (a) Hockey (b) Football
(a) 10th Dec. (b) 21st Dec. (c) Swimming (d) Cricket
(c) 20 Dec. (d) 1st Dec. 17. India’s multi largest surface to air missile is
6. Asian Games were held in India : [1997] known as : [1998]
(a) 1961 and 1984 (b) 1956 and 1986 (a) Nag (b) Agni
(c) 1962 and 1984 (d) 1957 and 1982 (c) Prithvi (d) Akash
7. Ruble is the currency of : [1997] 18. Beighton cup in India is associated with : [1998]
(a) USSR (b) Denmark (a) Cricket (b) Foot ball
(c) Volley ball (d) Hockey
(c) Germany (d) Japan
19. Which one of the following book is written by
8. The trade name IBM associated with : [1997]
Sarojini Naidu ? [1998]
(a) scooters (b) refrigerators
(a) Gandevata (b) Chitra
(c) cars (d) computers
(c) Broken wing (d) Great Tragedy
9. Which plant was responsible for Bhopal Gas
20. The common wealth games of 1998 were hosted by
Tragedy? [1997]
(a) Singapore (b) England [1998]
(a) BHEL (c) Malaysia (d) Australia
(b) Union Carbide 21. The old name of Thailand is : [1998]
(c) Hindustan Insecticides (a) Combodia (b) Siam
(d) None of these (c) Zaire (d) Persia
10. Libra is the currency of which of the following 22. Which one of the following country is not the
country? [1998] member of SAARC ? [1999]
(a) Spain (b) Vietnam (a) Maldeiv (b) Bangladesh
(c) Philippines (d) Turkey (c) Nepal (d) Myanmar
11. Which one of the following is the biggest cave 23. Hari Prasad Chaurasia is related to which of the
temple in India ? [1998] following instrument ? [1999]
(a) Ajanta (b) Tuljapur (a) Tabla (b) Flute
(c) Ellora (d) Parli (c) Violin (d) Santoor
EBD_7100
C-16 Topicwise AIIMS Solved Papers – GENERAL KNOWLEDGE
24. Which of the following missile of India has the (c) Florence Nightinagale
longest range ? [1999] (d) None of these
(a) Akash (b) Prithvi 38. Rial is the currency of : [2000]
(c) Pinaka (d) Both (b) and (c) (a) Afganistan (b) Iran
25. Which one of the following is the cave temple in (c) Saudi Arabia (d) Jordan
India ? [1999] 39. The writer of "Daughter of East" is : [2001]
(a) Ellora (b) Ajanta (a) Indira Gandhi (b) Benazier Bhutto
(d) Parli (d) Tuljapur (c) Amrita Pritam (d) Marget Tatcher
26. Which one of the following pair is incorrect ? 40. Which one of the following is the largest bridge?
(a) Kapil – Cricket [1999] (a) Seawise (b) Petronas [2001]
(b) M. F. Husain – Actor (c) Strahov (d) George
(c) Abul Fazal – Author 41. Teacher day is celebrated on : [2001]
(d) Feroz Gandhi – Politics (a) 5th September (b) 16 August
27. The great poetry ‘Madhushala’ was composed by : (c) 21 September (d) 1st April
(a) Mulk Raj Anand [1999] 42. Who discovered the sea route to India?[2001]
(b) Harivansh Rai Bachchan (a) Vasco de Gama (b) Columbus
(c) Mahadevi Verma (c) Magellan (d) Hopkins
(d) Surender Sharma
43. Garba dance is a dance style of : [2001]
28. Grand prix is a term associated with : [1999]
(a) Gujrat (b) Uttar Pradesh
(a) Chess (b) Table tennis
(c) Nagaland (d) Bihar
(c) Hockey (d) Badminton
44. A former cricketer after whose name no
29. William’s cup is related to : [2000]
(a) basket ball (b) table tennis championship has been started in India : [2001]
(c) volley ball (d) foot ball (a) G. K. Naidu (b) Daleep Singh
30. Full form of H.T.T.P. is : [2000] (c) Lala Amarnath (d) Vijay Merchant
(a) Hyper Terminal Transformation 45. Currency note bearts the signature of the : [2001]
(b) Hyper Text Transfer Protocol (a) Finance Minister
(c) High Technology Test Principles (b) Governor, Reserve Bank of India
(d) Hyper Text Training Program (c) Cabinet Secretary
31. Tallest tower in the world is : [2000] (d) President
(a) C.N.Tower (b) Kutub Minar 46. When was the first football world cup held?
(c) Angel (d) None of these (a) 1930 (b) 1950 [2002]
32. ‘The Satanic Verses’ a controversial book is (c) 1954 (d) 1968
written by : [2000] 47. 'Human Organ Development Centre for
(a) Gyani Jail Singh (b) Khuhwant Singh Transplantation' is going to be established in
(c) Kuldip Nayyar (d) Salman Rushdie India at : [2002]
33. World Tourism day was declared on : [2000] (a) Vellore (b) Mumbai
(a) 1st October (b) 11th February (c) Hyderabad (d) Chennai
(c) 27th September (d) none of these 48. July 11 is celebrated as [2002]
34. Which one of first Indian missile (earth to earth) (a) Doctor's Day
was tested successfully from Shri Hari Kota ? (b) Van Mahotsava Day
(a) Prithvi (b) Nag [2000] (c) AIDS Day
(c) Agni (d) none of these (d) World Population day
35. “Divine” comedy was written by : [2000] 49. Which one of the classical dance forms
(a) Goethe (b) Milton originated in Andhar Pradesh? [2002]
(c) Dante (d) Shakespears (a) Odissi (b) kathakali
36. ‘Abhigyan shakuntalam’ was written by : [2000] (c) Kuchipudi (d) Bharat Natyam
(a) Surdas (b) Tulsidas 50. Who is called Nightingale of India [2002]
(c) R.N. Tagore (d) Kalidas (a) Indira Gandhi
37. Who was known as the “ Lady of the Lamp”? (b) Lata mangeshker
(a) Sarojini Naidu [2000] (c) Asha Bhonsle
(b) Joan of Arc (d) Sarojini Naidu
G.K. – General Awareness C-17

51. In Internet what does 'http' mean? [2003] (c) it is interdisciplinary i.e., involves sociologists,
(a) High Transfer Text Protocol economist, computer sciences etc.
(b) Highest Transfer Text Protocol (d) it plans to produce wearable computer
(c) Hyper Text Transfer Protocol 62. Who one of the following is a medical doctor ?
(d) Hyper Transfer Text Protocol (a) Samuel Taylor Coleridge [2004]
52. The India-born US physicist who was awarded (b) John Webster
the Nobel Prize in Physics for his work on (c) Somerset Maugham
astrophysics is : [2003] (d) Thomas Gray
(a) H. G. Khorana 63. Who was the first Indian to win Nobel Prize?
(b) Subrahmanyam Chandrashekhar (a) Mother Terresa [2005]
(c) Sivaramakrishna Chandrashekhar (b) C.V. Raman
(d) C.V Raman (c) Ravindra Nath Tagore
53. Which German Physicist invented the electron (d) Amartya Sen
microscope which won him the 1986 Nobel Prize 64. Oldest religious text in the world is : [2005]
in Physics ? [2003] (a) Rig Veda (b) Sama Veda
(a) E. Ruska (b) Van't Hoff (c) Yajur Veda (d) Atharva Veda
(c) J. H. D. Jensen (d) Eugene P. Wigner 65. Who is the inventor of 'Insulin'? [2005]
54. Who was the first Indian to be awarded the world (a) Loard Lister (b) Jonos Salk
Food Prize in 1987? [2003] (c) Ronald Ross (d) Banting and Best
(a) M. S. Swaminathan 66. Who is the author of 'An Area of Darkness'?
(b) Sunderlal Bahuguna (a) Nirad C. Choudhari [2005]
(c) Anna Hazare (b) Vikram Seth
(d) B. R. Barwale (c) V.S. Naipaul
55. Israel's Prime Minister Yitzhak Rabin won the (d) B.C. Chatterjee
67. Which one of the following literary titles is
Nobel Prize for : [2003]
correctly matched with its author? [2006]
(a) Peace (b) Literature
(a) Ramayan – Tulsidas
(c) Chemistry (d) Economics
(b) Mahabharat – Vedvyas
56. Who was the world's first space tourist? [2003]
(c) Kumarsambhav – Ravidas
(a) Desmond Rickett (b) Dennis Tito
(d) Shakuntala – Bhushan
(c) Igor Kajlnikov (d) Li Wang
68. The jungle in Rudyard Kippling's Jungle book,
57. How many "World Cultural Hertitage Sites" are
describes which part of Indian forest ? [2006]
in India ? [2003] (a) Central Indian forest near Satpura range
(a) 10 (b) 17 (b) Uttranchal thick forest
(c) 14 (d) 15 (c) Himalayan Forest in Himachal
58. Birbal Sahni was a : [2004] (d) Nilgiri Jungles
(a) zoologist 69. Which of the following honour is given by
(b) founder of Central Drug Research Institute UNESCO? [2006]
(c) ornithologist (a) The Kalinga Prize
(d) paleobotanist (b) Magasay Award
59. Primary sector refers to : [2004] (c) Pulitzer Prize
(a) industry (b) agriculture (d) Order of the Golden Ark Award
(c) trade (d) banks 70. 'Body line' in the cricket refers to : [2006]
60. All of the following won the title of 'Miss World' (a) Bowling that hits the body
except : [2004] (b) The line of body chose to wicket line
(a) Lara Dutta (b) Aishwarya Rai (c) The white line on ground within which the
(c) Yukta Mukhi (d) Priyanka Chopra player sands
61. All of the following are correct about 'Media (d) The line of moving ball
Lab' except : [2004] 71. 'Hindu view of life' is written by : [2006]
(a) the third media lab is situated in india (a) S.Radhakrishnan (b) R.K. Narayan
(b) it is supported by government funding (c) V.D. Savarkar (d) John Ruskin
EBD_7100
C-18 Topicwise AIIMS Solved Papers – GENERAL KNOWLEDGE
72. The compilation “Meri Ekyawan Kavitayen’s is 84. Environment Day is celebrated on [2009]
by [2007] (a) 5th June (b) 11th Sep.
(a) A.B. Vajpayee (c) 20th Feb. (d) 5th Oct.
(b) Harivanshrai Bachchan 85. Among which of the below is a script? [2009]
(c) Dharam Vir Bharti (a) Hindi (b) English
(d) Shiv Mangal Singh Suman (c) Gurumukhi (d) Sanskrit
73. Who was known as “Nightingale of India”? 86. On which cartoon character’s name a gene is
[2007] also named : [2009]
(a) Vijaylaxmi Pandit (b) Sarojini Naidu (a) Tintin (b) Sonic
(c) Suraiya (d) None of these (c) Asterix (d) Obedix
74. Kalidas was [2007] 87. Which of the following river’s name means
(a) A poet during the Gupta period ‘Elephant -river’ [2009]
(b) A dramatist during Harshvardhana’s reign (a) Krishna (b) Errabadi
(c) An astronomer during Gupta period (c) Godavari (d) Mahanadi
(d) None of the above 88. Reserve Bank of India’s emblem carries the
75. India's first human DNA Bank has been setup in sketch of a tiger and a tree. What kind of tree is
(a) Patna (b) New Delhi [2008] depicted in this emblem? [2010]
(c) Lucknow (d) Kolkata
(a) Palm tree (b) Cactus
76. What is the motto incorporated under our
(c) Banyan (d) Banana
National Emblem ? [2008]
89. The term 'smash' in sports is associated with
(a) Satyam Shivam
(a) Lawn Tennis (b) Badminton
(b) Satyam Shivam Sundaram
(c) Satyameva Jayate (c) Volleyball (d) Hockey
(d) Jai Hind 90. Match List-1 (personality) with List-II (their field
77. Bhabha Atomic Research Centre is situated in of activity) and select the correct combination/
(a) Delhi (b) Mumbai [2009] option : [2010]
(c) Chennai (d) Hyderabad List - I List - II
78. FERA in India has been replaced by [2009] (A) Yamini Krishnamurthy (1) Paintings
(a) FEPA (b) FEMA (B) Wasim Jaffar (2) Politics
(c) FENA (d) FETA (C) Raja Ravi Verma (3) Bharat Natyam
79. The missile Agni II of India is a [2009] (D) Rahul Gandhi (4) Cricket
(a) Nuclear missile Code :
(b) Surface-to -air missile (A) (B) (C) (D)
(c) Surface-to-surface missile (a) 3 4 1 2
(d) Surface-to-sea missile (b) 3 4 2 1
80. Amjad Ali Khan is a maestro with which (c) 2 4 3 1
instrument? [2009] (d) 4 2 1 3
(a) Violin (b) Sitar 91. Which of the following sites has been included
(c) Sarod (d) Sarangi in UNESCO’s list of World Heritage sites?[2010]
81. With which sport do you associate the name of (a) Akbar’s Tomb at Sikandara
Geet Sethi? [2009] (b) Gateway of India (Mumbai)
(a) Golf (b) Billiards (c) Agra Fort
(c) Lawn Tennis (d) Cricket (d) Bibi ka Maqbara (Aurangabad)
82. In which of the following games, left hand is not 92. With which program, the slogan ‘Do Boond
to be used? [2009] Jindgi Ki’ is associated? [2010]
(a) Tennis (b) Hockey (a) Blood Donation (b) Save Water
(c) Polo (d) cricket (c) Pulse Polio (d) Pollution Control
83. Among countries given below, Albert Einstein 93. On whose birthday is Teacher’s Day celebrated?
had citizenship of which country along with (a) S. Radhakrishnan [2011]
Germany and US? [2009] (b) Maulana Abul Kalam Azad
(a) Sweden (b) Austria (c) Rajendra Prasad
(c) Israel (b) Netherlands (d) Jawaharlal Nehru
G.K. – General Awareness C-19

94. Dronacharya Award is given to [2011] 105. Which of the following is not correctly paired?
(a) fire fighting operation (a) Jwala Gutta — Tennis [2013]
(b) archery (b) Virat Kohli — Cricket
(c) outstanding coaching in sports and games (c) Harbhajan Singh — Kabaddi
(d) outstanding coaching in athletics (d) Saina Nehwal — Badminton
95. Yuan is the currency of [2011] 106. Which one of the following is an example for a
(a) China (b) Indonesia non-economic good? [2013]
(c) Thailand (d) Japan (a) Doctor’s service (b) Teacher’s service
96. National game of Australia is [2011] (c) Mother’s service (d) Banker’s service
(a) base -ball (b) cricket 107. Which one of the following does not match?
(c) rugby football (d) hockey (a) Hindu Marriage Act : 1955 [2013]
97. Which one of the following pairs is not correctly (b) Medical Termination of Pregnancy Act : 1971
matched ? [2012] (c) Domestic Violence on women Act : 1990
State/U.T. High Court (d) Cruelty against Women : 1995
(a) Goa – Bombay 108. Ward Cunningham is famous for developing for
(b) Andaman and – Calcutta the first time [2013]
Nicobar Islands (a) the free encyclopedia on Internet
(c) Sikkim – Guwahati (b) a computer language called 'Java'
(d) Pondicherry – Madras (c) a digital camera
98. Who is the author of Das Kapital ? [2012] (d) a software which can take Hebrew language
(a) Karl Marx (b) Friedrich Engels as input
(c) Joseph Stalin (d) Vladimir Lenin 109. Which of the following is/are instance(s) of
99. Which National Highway connects Delhi and violation of human rights? [2013]
Mumbai? [2012] 1. A person was detained by the security
(a) NH 6 (b) NH 8 forces while going for casting vote in
(c) NH 10 (d) NH 12 Parliamentary Election.
2. A civilian was killed by the army while
100. Who among the following advocated scientific
undertaking combing operation.
socialism? [2012]
Select the correct answer using the code given
(a) Robert Owen
below:
(b) Proudhon Pierre Joseph
Code:
(c) Karl Marx (a) 1 only (b) 2 only
(d) Saint Simon Henri Claude (c) Both 1 and 2 (d) Neither 1 nor 2
101. Where are the headquarters of the UNO ? [2012] 110. Navroze is a festival celebrated in India by the
(a) Geneva (b) The Hague (a) Hindus (b) Muslims [2013]
(c) New York (d) Paris (c) Parsis (d) Christians
102. For controlling inflation, the central bank should 111. Who is the author of the book ‘Conquest of Self’?
(a) sell Government securities in the open (a) Aurobindo Ghosh [2014]
market [2013] (b) Rabindra Nath Tagore
(b) lower the bank rate (c) Mahatma Gandhi
(c) purchase Government securities in the open (d) S. Radhakrishnan
market 112. Who is known as the ‘Piccaso of India’? [2014]
(d) lower the reserve ratio of the banks (a) Amrita Shergil (b) M.F. Hussain
103. United Nations Conference on Environment and (c) Sudhir Vyas (d) Shafqat Hussain
Development is called [2013] 113. Which is the capital of Mali ? [2014]
(a) Earth Summit (b) Water Summit (a) Mopti (b) Bamako
(c) Air Summit (d) Resource Summit (c) Cairo (d) Nairobi
104. The five permanent members of the U.N. 114. Which one of the following though called a
Security Council are [2013] garden is infact, not a garden? [2014]
(a) China, France, Russia, U.K. and U.S.A. (a) Vrindavan Garden of Mysore
(b) China, Canada, France, U.S.A and Germany (b) Hanging Garden of Mumbai
(c) China, Germany, Russia, U.K. and U.S.A. (c) Eden Garden of Kolkata
(d) China, Germany, U.S.A., U.K and Canada (d) Shalimar Garden of Kashmir
EBD_7100
C-20 Topicwise AIIMS Solved Papers – GENERAL KNOWLEDGE
115. Who advocated the adoption of ‘PURA’ model (c) Liquid Ratio
to eradicate rural poverty? [2014] (d) Statutory Ratio
(a) Dr. A. P. J. Abdul Kalam 122. Which movie has won maximum awards at the
(b) Sri Abhijit Sen International Indian Film Awards? [2015]
(c) Maulana Abdul Kalam Azad (a) Queen (b) Ek Villain
(d) Prof. A. M. Patha (c) Haider (d) PK
116. Who caught the wicket of Sachin Tendulkar in 123. Who has made the first attempt to initiate
his last match? [2014] economic planning in India [2016]
(a) Chris Gayle (a) M. Visvesvaraya (b) Zakir Hussain
(b) Darren Sammy (c) Amarty Sen (d) Narendra MOdi
(c) Shiv Narayan Chandrapaul 124. When was decimal coinage introduced in India?
(d) Dwane Bravo (a) 1949 (b) 1965 [2016]
117. Gilt-edged market stands for…..? [2015] (c) 1935 (d) 1957
(a) bullion market 125. Who founded the Ayurveda system of medicine?
(b) market of government securities [2016]
(c) market of guns (a) Susruta (b) Thirumoolar
(d) market of pure metals (c) Atreya (d) Agasthiya
118. The best indicator of economic development of 126. What is the full form of GST? [2017]
a country is _________? [2015] (a) General Sales Tax
(a) Its agriculture (b) Goods and Sales Tax
(b) Its transport (c) Goods and Services Tax
(c) Its gross production (d) General Services Tax
(d) Its per capita income 127. ICMR signs agreement to provide healthcare
119. The data of estimation of India's National income is through solar based solutions.What is the full
issued by ? [2015] form of ICMR [2017]
(a) Planning Commision (a) Indian Council of Medical Research
(b) National Data Center (b) International Council of Medical Research
(c) Central Statistical Organisation (c) India Council of Medical Research
(d) None of above (d) Indian Counciling of Medical Research
120. First five year plan in India was from? [2015] 128. Which tiger reserve in Madhya Pradesh has
(a) 1947 – 1952 (b) 1949 – 1954 become the first tiger reserve in India in officially
(c) 1950 – 1955 (d) 1951 – 1956 introduce a mascot which has been named
121. The banks are required to maintain a certain ratio Bhoorsingh the Barasingha? [2017]
between their cash in hand and total assets. This (a) Kanha Tiger Reserve
is called ? [2015] (b) Kaziranga Tiger Reserve
(a) Statutory Liquid Ratio (c) Buxa Tiger Reserve
(b) Cash Reserve Ratio (d) Sunderbans Tiger Reserve

ANSWER KEY
1 (c) 2 (d) 3 (d) 4 (d) 5 (d) 6 (d) 7 (a) 8 (d) 9 (b) 10 (b)
11 (b) 12 (a) 13 (a) 14 (a) 15 (b) 16 (b) 17 (b) 18 (a) 19 (b) 20 (c)
21 (b) 22 (d) 23 (b) 24 (b) 25 (a) 26 (b) 27 (b) 28 (a) 29 (a) 30 (b)
31 (a) 32 (d) 33 (c) 34 (a) 35 (c) 36 (d) 37 (c) 38 (b) 39 (b) 40 (d)
41 (a) 42 (a) 43 (a) 44 (c) 45 (b) 46 (a) 47 (c) 48 (d) 49 (a) 50 (d)
51 (c) 52 (b) 53 (a) 54 (a) 55 (a) 56 (b) 57 (b) 58 (d) 59 (b) 60 (a)
61 (d) 62 (c) 63 (c) 64 (a) 65 (b) 66 (c) 67 (b) 68 (a) 69 (a) 70 (a)
71 (a) 72 (a) 73 (b) 74 (a) 75 (c) 76 (c) 77 (b) 78 (b) 79 (b) 80 (c)
81 (b) 82 (c) 83 (b) 84 (a) 85 (c) 86 (b) 87 (b) 88 (a) 89 (b) 90 (a)
91 (c) 92 (c) 93 (a) 94 (c) 95 (a) 96 (b) 97 (c) 98 (a) 99 (b) 100 (c)
101 (c) 102 (a) 103 (a) 104 (a) 105 (c) 106 (c) 107 (c) 108 (a) 109 (c) 110 (c)
111 (c) 112 (b) 113 (b) 114 (c) 115 (a) 116 (b) 117 (b) 118 (d) 119 (c) 120 (d)
121 (a) 122 (c) 123 (a) 124 (d) 125 (a) 126 (c) 127 (a) 128 (a)
G.K. – Current Affairs G-21

6 G.K. – Current Affairs

1. Noble Prize for physiology and medicine for the 10. In the year (2003) the chemistry Nobel Prize was
year 1998 was given for the discovery of : awarded to the following work : [2004]
(a) Prion (b) Viagra [1999] (a) aquaporins (b) Na++ channels
(c) Streptomycin (d) Invading germs (c) Ca++channels (d) methyl chavicol
2. How many countries adopted Euro currency? 11. Who is known as 'Deshbandhu'? [2004]
(a) 11 (b) 6 [1999] (a) Chandra Shekhar (b) C.R Das
(c) 9 (d) 8 (c) A.O. Hume (d) Annie Besant
3. Present speaker in 12th Lok Sabha is : [1999] 12. The Ramakrishna Mission was established by
(a) G. M. C. Balayogi (a) Swami Vivekananda [2004]
(b) Nazma Haptullah (b) Ramakrishna Paramhansa
(c) P. A. Sangama (c) Swami Dayanand Saraswati
(d) Murali Manohar Joshi (d) None of these
4. Miss Universe event 2000 was held at [2001] 13. In which space Shuttle Kalpana Chawla killed
(a) Peuotro Rico (a) Discovery (b) Columbia [2005]
(b) London (c) Sputnik (d) None of the above
(c) Paris 14. A very much publicized treatment method
(d) Nicosia (Cyprus) "DOTS" is being adopted for the cure of :
5. Who was chosen ‘Time Magazines Person’ for (a) Dementia [2006]
the year 2001? [2002] (b) Tetanus
(a) Collin Powel (b) Mike Monore (c) Tuberculosis
(c) George Bush (d) Rudolf Guilani (d) Sexually transmitted disease
6. Which of the following Hindi Indian movies 15. Which of the following Indian circket player after
Language film for Oscar Award 2002? [2002] India-Pakistan ODI (one-day International) at
(a) Mansoon Wedding Abudhabi became no. 1 ODI batsman in the ICC
(b) Dil Chahata Hai (international Cricket Club) ranking : [2006]
(c) Gadar Ek Prem Katha (a) Rahul Dravid (b) Yuvraj
(d) Lagaan (c) Sachin Tendulkar (d) M.S. Dhoni
7. Men's Single US Open, 2001 Championship won
16. Who is the highest wicket taker in Indian Cricket
by : [2002]
team ? [2007]
(a) Leyton Herwitt (b) Pete Sampras
(a) Javagal Srinath (b) Anil Kumble
(c) Safin (d) Stefan Edberg
(c) Maninder Singh (d) Kapil Dev
8. Who is CEAT International Cricketer of the year
17. India test-fired Agni III on [2008]
2000 -2001? [2002]
(a) May 7, 2008 (b) March 20, 2008
(a) Sachin Tendulkar
(c) May 20, 2008 (d) March 7, 2008
(b) Muttiah Muralitharan
18. The train which was started on April 14, 2008
(c) Shane Warne
(d) Brayan Lara between Kolkata (India) and Dhaka
9. When seen from earth, which of the following (Bangladesh) has been named ? [2008]
planet eclipsed (crossed a cross) of the sun on 7 (a) Shanti Express
May, 2003? [2003] (b) Maitri Express
(a) Mercury (b) Uranus (c) Aman Express
(c) Saturn (d) Jupiter (d) Samjhauta Express
EBD_7100
G-22 Topicwise AIIMS Solved Papers – GENERAL KNOWLEDGE
19. Which cricketer holds the record for scoring 29. Schemes run under the National Rural
highest number of runs in a test match innings? Employment Guarantee Act are sponsored[2013]
[2008] (a) by the Central Government alone
(a) Gary Sobers (b) Vivian Richards (b) partly by the Central Government and
(c) Sunil Gavaskar (d) Brian Lara partly by the State Government
20. What is the name given to the common currency (c) by Centre, State and Panchayat bodies
of the European Union? [2009] together
(a) Rupee (b) Euro (d) on public-private partnership basis
(c) Rouble (d) Franc
30. ‘Catch Me If You Can’ is a Hollywood movie
21. Who has written “Two Lives’? [2010]
made on the life of Frank Abagnale who was
(a) Vikram Seth (b) Kiran Desai
(c) Arundhati Roy (d) Khushwant Singh (a) a famous lawyer [2013]
22. Who amongst the following is the ICC Cricketer (b) a famous swimmer
of the year? [2011] (c) a famous sprinter
(a) Sachin Tendulkar (b) Kevin Pietersen (d) an infamous imposter
(c) M. S. Dhoni (d) Sanath Jayasuriya 31. Who among the following was awarded the first
(e) None of these Tagore Award for Cultural Harmony for the year
23. ‘Goodbye Shahzadi’ is a book written by- 2012 ? [2013]
[2011] (a) Jasraj (b) Ravi Shankar
(a) Shyam Bhatia (b) Ashok Mehra (c) Nikhil Banerjee (d) Bhimsen Joshi
(c) Janardhan Thakur (d) Arun Gandhi 32. Which team has won the Men's National Hockey
(e) None of these Championship ? [2015]
24. Who is Robert Zoelick? [2011] (a) Karnataka (b) Telangana
(a) IMF Chief (c) Indian Railways (d) Madhya Pradesh
(b) World Bank President 33. Which bank won the Asian Banker Achievement
(c) ADB President
Award ? [2015]
(d) Chief UNIDO
(a) ICICI Bank
(e) None of these
(b) Bharatiya Mahila Bank
25. Which country’s Parliament has banned ‘Burqa’
by passing a law of 14th September, 2010? (c) State Bank of Mysore
(a) USA (b) France [2011] (d) Axis Bank
(c) Italy (d) Holland 34. Which among the following state does not
26. Which among the following is not correct with implement the National Food Security Act
regard to Sampoorna Garmeena Rozgar Yojana? recently empowers 87% of the priority
(1) The cash component of the programme is households? [2016]
borne exclusively by the Central (a) Manipur
Government. (b) Sikkim
(2) Foodgrains are provided free of costs to (c) Nagaland
the States/Union Territories. (d) Arunachal Pradesh
Select the answer unsing the code given below: 35. Smt. Maneka Sanjay Gandhi launched Beti
(a) (1) only (b) (2) only [2012] Bachao Beti Padhao Scheme in additional how
(c) Both (1) and (2) (d) Neither (1) nor (2) many districts of the country? [2016]
27. Which of the following countries does not (a) 66 (b) 67
belong to the group of G-8 nations? [2012]
(c) 51 (d) 61
(a) Italy (b) Canada
36. Which Government banned digging of bore
(c) Germany (d) Australia
28. Who is author of One Night @ the Call Centre ? wells beyond 200ft to prevent the declining of
[2013] Ground water level? [2016]
(a) Vikram Seth (b) Chetan Bhagat (a) Tamilnadu (b) B.Karnataka
(c) Anurag Mathur (d) Robin Sharma (c) Madhya Pradesh (d) Maharashtra
G.K. – Current Affairs G-23

37. India is collaborated with which country on (b) It has been developed by the National
tackling offshore tax evasion and increase Co- Payments Corporation of India
operation in sharing of cross-border tax (c) The receiver of transferred money needs
Information? [2016] to compulsorily be a BHIM user
(a) United Kingdom (b) United States (d) A person without a bank account cannot
(c) Japan (d) China use BHIM
38. Which country has been declared as drought 42. WannaCry’, recently seen in the news, is which
Emergency for first time ever in the History? one of the following types of malware? [2017]
[2016] (a) Virus (b) Worm
(a) Hungary (b) South Africa (c) Trojan Horse (d) Ransomware
(c) Zimbabwe (d) El Salvador 43. According to Urban Jyoti Abhiyaan or Urja app
39. Who is the defence minister of India? [2017] which states top the list in providing
(a) Sushma Swaraj (b) George Anthony uninterrupted power? [2017]
(c) Prithviraj Chauhan (d) Arun Jaitley (a) Jharkhand, Uttarakhand and Assam
40. Who is the new chief of the Chief of the Army (b) Tamil Nadu, Rajasthan and West Bengal
Staff (COAS) of the Indian Army? [2017] (c) Kerala, Maharashtra and Rajasthan
(a) Balbir Singh Sandhu (d) Punjab, Assam and Andhra Pradesh
(b) Velu Nair 44. Who has been honoured as the Wisden’s leading
(c) Bipin Rawat cricketer in the World for 2016? [2017]
(d) Amit Sarin (a) Chris Woakes (b) Misbah-ul-Haq
41. The Central government has recently announced (c) Younis Khan (d) Virat Kohli
a new digital payments app called BHIM. Which 45. How many ‘priority pathogens’ are listed by
of the following statements about BHIM is not the World Health Organization for the first time
true? [2017] that pose the greatest threat to human life?
(a) The BHIM digital payments app is based (a) 10 (b) 11 [2017]
on the Unified Payments Interface (c) 12 (d) 15

ANSWER KEY
1 (b) 2 (a) 3 (c) 4 (a) 5 (d) 6 (d) 7 (c) 8 (b) 9 (a) 10 (a)
11 (b) 12 (a) 13 (b) 14 (c) 15 (d) 16 (b) 17 (a) 18 (b) 19 (d) 20 (b)
21 (a) 22 (a) 23 (a) 24 (b) 25 (b) 26 (a) 27 (d) 28 (b) 29 (a) 30 (d)
31 (b) 32 (c) 33 (b) 34 (b) 35 (d) 36 (d) 37 (b) 38 (d) 39 (d) 40 (c)
41 (d) 42 (d) 43 (c) 44 (d) 45 (c)
EBD_7100
7 G.K. – Logical Thinking

1. A duck, a goose, a goat, and a horse all entered 4. Find out the number of students who play only
the bar n at different times one day last week. cricket. [2017]
[2017] 40
(1) A mammal entered the bar n first.
Cricket Tennis
(2) The duck entered before the goose. Players 16 Players
(3) The goose entered ahead of the horse. 25 22
Who entered the bar n first?
(a) A goat (b) A duck (a) 25 (b) 18
(c) A goose (d) A horse (c) 9 (d) 41
2. Choose or find odd word [2017] 5. Arrange the following words as per order in the
(a) Piano (b) Guitar dictionary [2017]
(c) Sitar (d) Violin 1. Eyelid 2. Eyeless 3. Eyesore 4. Eyesight
3. If Ram lives east of a post office. In the north of (a) 2, 1, 4, 3 (b) 2, 1, 3, 4
post office is big bazar. Then what is the position (c) 4, 3, 1, 2 (d) 4, 2, 3, 1
of Ram’s house with respect to the post office? 6. If the day before yesterday was Thursday, when
[2017] will Sunday be? [2017]
(a) North-west (b) North-east (a) Today
(c) South-west (d) South-east (b) Two days after today
(c) Tomorrow
(d) Day after Tomorrow

ANSWER KEY
1 (d) 2 (a) 3 (d) 4 (a) 5 (a) 6 (c)
Mock AIIMS
Time : 3½ Hrs. Questions : 200 Maximum Marks : 200
Negative Marking : –1/3 for each incorrect answer.

6. A charge q is fixed. Another charge Q is brought


SECTION I - PHYSICS
near it and rotated in a circle of radius r around
1. A block is kept on a inclined plane of inclination it. Work done during rotation is
of length . The velocity of particle at the bottom
Q.q
of inclined is (the coefficient of friction is ) (a) zero (b)
4 0r
(a) [2g ( cos sin )]1 / 2
Q .q
(b) 2g (sin cos ) (c) (d) None of these
2 0r
(c) 2g (sin cos )
7. Advantage of optical fibre
(d) 2g (cos sin ) (a) high bandwidth and EM interference
2. If earth is supposed to be a sphere of radius R, if (b) low band width and EM interference
g 30 is value of acceleration due to gravity at (c) high band width, low transmission capacity
lattitude of 30° and g at the equator, the value of and no EM interference
g – g30 is (d) high bandwidth, high data transmission
1 2 3 2 capacity and no EM interference.
(a) R (b) R
4 4 8. In an electromagnetic wave, direction of
1 2 propagation is in the direction of
(c) 2 (d)
R R
2 (a) E (b) B
3. An organ pipe open at one end is vibrating in
first overtone and is in resonance with another
(c) E B (d) None of these
pipe open at both ends and vibrating in third
harmonic. The ratio of length of two pipes is 9. F1 and F2 are focal length of objective and eyepiece
(a) 1 : 2 (b) 4 : 1 (c) 8 : 3 (d) 3 : 8 respectively of the telescope. The angular
4. A coil takes 15 min to boil a certain amount of
water, another coil takes 20 min for the same magnification for the given telescope is equal to
process. Time taken to boil the same amount of F1 F2
water when both coil are connected in series, (a) (b)
(a) 5 min (b) 8.6 min F2 F1
(c) 35 min (d) 30 min
5. Two capillary of length L and 2L and of radius R F1 F2 F1 F2
and 2R are connected in series. The net rate of (c) (d)
F1 F2 F1F2
flow of fluid through them will be (given rate to
10. Critical velocity of the liquid
PR 4
the flow through single capillary, X = ) (a) decreases when radius decreases
8 L
(b) increases when radius increases
8 9 5 7 (c) decreases when density increases
(a) X (b) X (c) X (d) X
9 8 7 5 (d) increases when density increases
EBD_7100
2 Topicwise AIIMS Solved Papers
11. An organ pipe, open from both end produces 5 18. A period of a planet around Sun is 27 times that
beats per second when vibrated with a source of Earth. The ratio of radius of planet’s orbit to
of frequency 200 Hz. The second harmonic of the radius of Earth’s orbit is
the same pipes produces 10 beats per second (a) 4 (b) 9 (c) 64 (d) 27
with a source of frequency 420 Hz. The 19. 3 particles each of mass m are kept at vertices of
fundamental frequency of organ pipe is an equilateral triangle of side L. The gravitational
(a) 195 Hz (b) 205 Hz field at centre due to these particles is
(c) 190 Hz (d) 210 Hz (a) 1.5 V (b) 2.0 V (c) 2.5 V (d) 5 V
12. Two rings of radius R and nR made up of same
material have the ratio of moment of inertia about
3GM 9GM 12 GM
an axis passing through centre as 1 : 8. The value (a) zero (b) (c) (d)
2
of n is L 2
L 3 L2

1 20. A solid sphere of radius R is rolling with velocity


(a) 2 (b) 2 2 (c) 4 (d) v on a smooth plane. The total kinetic energy of
2
13. One drop of soap bubble of diameter D breaks sphere is
into 27 drops having surface tension . The 7 3
(a) mv 2 (b) mv 2
change in surface energy is 10 4
(a) 2 D2 (b) 4 D2 1 1
(c) mv 2 (d) mv 2
2 4
(c) D2 (d) 8 D2
21. A diode having potential difference 0.5 V across
14. The gas having average speed four times as that its junction which does not depend on current,
of SO 2 (molecular mass 64) is is connected in series with resistance of 20
(a) He (molecular mass 4) across source. If 0.1 A current passes through
(b) O2 (molecular mass 32) resistance then what is the voltage of the source?
(c) H2 (molecular mass 2) (a) 1.5 V (b) 2.0 V (c) 2.5 V (d) 5 V
(d) CH4 (molecular mass 16) 22. Potentiometer wire of length 1 m is connected in
15. A container having 1 mole of a gas at a series with 490 resistance and 2 V battery. If
temperature 27°C has a movable piston which 0.2 mV/cm is the potential gradient, then
maintains at constant pressure in container of 1 resistance of the potentiometer wire is
atm. The gas is compressed until temperature
becomes 127°C. The work done is (CP for gas is (a) 4.9 (b) 7.9
7.03 cal/mol-K) (c) 5.9 (d) 6.9
(a) 703 J (b) 814 J (c) 121 J (d) 2035 J 23. A dipole is placed parallel to the electric field. If
16. An electron having mass (9.1 × 10–31 kg) and W is the work done in rotating the dipole by 60°,
charge (1.6 × 10–19 C) moves in a circular path of then work done in rotating it by 180° is
radius 0.5 m with a velocity 106 m/s in a magnetic
W
field. Find the strength of magnetic field. (a) 2 W (b) 3 W (c) 4 W (d)
2
5 6
(a) 1.13 10 T (b) 5.6 10 T 24. An electron of charge e moves in a circular orbit
of radius r around the nucleus at a frequency .
(c) 2.8 10 6 T (d) None of these
The magnetic moment associated with the orbital
17. A cylinder rolls down an inclined plane of motion of the electron is
inclination 30°, the acceleration of cylinder is
r2 e er 2
g g 2g (a) er 2 (b) (c) (d)
(a) (b) g (c) (d) e r
3 2 3
Mock AIIMS 3
25. A and B are two identically spherical charged 30. The ratio of frequencies of two pendulums are
bodies which repel each other with force F, kept 2 : 3, then their length are in ratio
at a finite distance. A third uncharged sphere of
the same size is brought in contact with sphere 2 3 4 9
(a) (b) (c) (d)
B and removed. It is then kept at mid point of A 3 2 9 4
and B. Find the magnitude of force on C. 31. The value of escape velocity on a certain planet
F F is 2 km/s. Then the value of orbital speed for a
(a) (b) (c) F (d) Zero
2 8 satellite orbiting close to its surface is
26. A composite rod made of copper (a) 12 km/s (b) 1 km/s
5 1
( 1.8 10 K ) and steel (c) 2 km/s (d) 2 2 km/s
32. The electrochemical equivalent of a metal is
( 1.2 10 5 K 1 ) is heated then it 3.3 × 10–7 kg/C. The mass of metal liberated at
(a) bends with steel on convex side cathode by 3 A current in 2 sec will be
(b) bends with copper on convex side
7 7
(c) does not expand (a) 19.8 10 kg (b) 9.9 10 kg
(d) data is insufficient
27. A wave has the equation y = 0.1 sin (c) 6.6 10 7 kg (d) 1.1 10 7 kg
[100 t kx ] and wave velocity 100 m/s, its 33. For a paramagnetic material, the dependence of
wave number is equal to the magnetic susceptibility, on the absolute
temperature is given as
(a) 1m 1 (b) 2m 1
1
(c) m 1 (d) 2 m 1 (a) T (b)
T2
28. Volume temperature graph at atmospheric
1
pressure for a monatomic gas (V in m 3 , T in °C) (c) (d) Independent
T
is 34. An optically active compound
(a) rotates the plane polarised light
V V
(b) changes the direction of polarised light
(c) do not allow plane polarised light to pass
(a) (b) through
(d) none of the above
35. Three particles A, B and C are thrown from the
T( C) T( C) top of a tower with the same speed. A is thrown
up, B is thrown down and C is horizontally. They
V V hit the ground with speeds VA, VB and VC
respectively.
(a) VA = VB = VC (b) VA = VB > VC
(c) (d) (c) VB > VC > VA (d) VA > VB = VC
36. The equivalent resistance between A and B is
T( C) T( C) A
R R
29. In X- ray experiment K , K denotes
B R
(a) characteristic lines
(b) continuous wavelength R
R
(c) , emissions respectively
(d) None of these 8R 5R 3R 7R
(a) (b) (c) (d)
5 8 8 8
EBD_7100
4 Topicwise AIIMS Solved Papers
37. The variation of maximum kinetic energy (a) 1 A (b) 1.5 A (c) 0.9 A (d) 0.6 A
photoelectrons with applied frequency ( ) is 42. Which of the following is false ?
(a) (b) (a) convex lens always forms image with m < 1
(b) a simple mirror produces virtual, erect and

K.E.max .
K.E.max .

same-sized image
(c) a concave mirror produces virtual, erect
and magnified image
(d) a convex lens can produce real and same-
(c) (d) sized image.
K .E.max .

K.E.max .

43. A star having wavelength is reaching with


velocity vs from earth. The apparent shift in
wavelength will be
38. The angle of projection for which range is vs
vs
equal to maximum height attained by projectile (a) (b) –
is c c
1 1
(a) tan 4 (b) tan 5
1 1 vs 2 vs 2
(c) tan 4 / 5 (d) tan 5 / 4 (c) – (d)
39. The range of projectile will be maximum, when c2 c2
angle of projection is
44. The deflection in a galvanometer decreases from
(a) (b) (c) (d) None 25 divisions to 5 divisions when a resistor of
3 2 4 20 is connected in series. Find resistance of
40. Forces of 4 N and 5 N are applied at origin along
x-axis and y-axis respectively. The resultant force galvanometer.
will be
(a) 4 (b) 5 (c) 6 (d) 7
(a) 1 5
41N, tan 45. A current source drives a current in a coil of
4
resistance R1 for a time t. The same source drives
1 4 current in another coil of resistance R2 for same
(b) 41N, tan
5 time. If heat generated is same, find internal
5 resistance of source
1
(c) 41N, tan
4 R1R 2
(a) (b) R1 R 2
R1 R 2
1 4
(d) 41N, tan
5 (c) zero (d) R 1R 2
41. Current in 2 resistor is (see given figure) 46. The waves used by artificial satellites for
2 communication is
(a) microwaves (b) radio-waves, AM
3
(c) radio-waves, FM (d) X-rays
1 47. The ratio of de-Broglie wavelengths of proton
and -particle having same kinetic energy is
(a) 2 :1 (b) 2 2 :1
6V 2
(c) 2 : 1 (d) 4 : 1
Mock AIIMS 5

48. The dimensions of Planck’s constant is Reason : A polaroid is capable of producing


plane polarised beams of light.
(a) M 2 L2T 1 (b) M 2 LT 2
54. Assertion : An induced current develop in a
(c) M L2 T 1 (d) M L2 T 2
conductor moved in direction parallel to the
magnetic field.
49. Which of these requires quantum nature of light
for their explanation? Reason : An induced current is developed when
the number of magnetic lines of force associated
(a) diffraction (b) polarisation
with conductor is changed.
(c) interference (d) black body spectrum
55. Assertion : If the length of the conductor is
50. If blue light is used in place of red light in a doubled, the drift velocity will become half of
diffraction experiment the original value (keeping potential difference
(a) diffraction pattern remains unchanged unchanged).
(b) fringes come closer Reason : At constant potential difference, drift
(c) fringes become broader velocity is inversely proportional to the length
of the conductor.
(d) none of these
56. Assertion : Circuits containing capacitors
should be handled cautiously even when there
DIRECTION : is no current.
NOTE : Instructions for Q. 51 to Q. 60 Reason : The capacitors are very delicate and
(a) Both Assertion and Reason are true and so quickly breakdown.
‘Reason’ is the correct explanation of ‘Assertion’ 57. Assertion : The absorbance of a perfect black
(b) Both Assertion and R are true and Reason is not body is unity.
the correct explanation of ‘Assertion’ Reason : A perfect black body when heated emits
(c) ‘Assertion’ is true but ‘Reason’ is false radiations of all possible wavelengths at that
temperature.
(d) Both ‘Assertion’ and ‘Reason’ are false
58. Assertion : The phase difference between two
(e) Assertion is false but ‘Reason’ is true
medium particle having a path difference is
51. Assertion : The dominant mechanism for motion
of charge carriers in forward and reverse biased 2 .
silicon p-n junction are drift in both forward and Reason : The phase difference is directly
reverse bias. proportional to path difference of a particle.
Reason : In reverse biased, no current flow 59. Assertion : The impurities always decrease the
through the junction surface tension of a liquid.
52. Assertion : The force of repulsion between Reason : The change in surface tension of the
atomic nucleus and -particle varies with liquid depends upon the degree of
distance according to inverse square law. contamination of the impurity.
Reason : Rutherford did -particle scattering 60. Assertion : Orbital velocity of a satellite is
experiment. greater than its escape velocity.
53. Assertion : The unpolarized light and polarized Reason : Orbit of a satellite is within the
light can be distinguished from each other by gravitational field of earth whereas escaping is
using polaroid. beyond the gravitational field of earth.
EBD_7100
6 Topicwise AIIMS Solved Papers
72. Ozonolysis is relation in which double bond is
SECTION II - CHEMISTRY
cleaved to give carboxyl group. Which of these
will gives formaldehyde as one of its product ?
61. Arene diazonium salt results from reaction of
nitrous acid with (a) CH 3CH CH CH 3
(a) 1° aliphatic amine
(b) 2° aromatic amine (b) CH 3 CH 2 CH 2 CH 3
(c) 1° aromatic amine CH 3
(d) 1° aromatic amide (c) C CH 2 CH 3
62. Collidal system constituting the liquid as CH 3
dispersed phase and solid as dispersion medium
is CH 3 CH 3
(a) gel (b) emulsion (d) C C
CH 3 CH 3
(c) solution (d) suspension
63. Calculate number of valance electrons in 73. Which of these reacts with Grignard reagent to
2 give carboxylic acid?
complex [Cr(H 2 O)5 SCN ]
(a) 17 (b) 15 (c) 16 (d) 19 (a) CO (b) CO 2
64. The ground state valance shell electronic (c) RCOOR (d) HCHO
5 1 74. Dehydration of tertiary alcohals is presence of
configuration of an element is 3d 4s . The metal
is an acid involves
(a) Cr (b) Fe (c) Mn (d) V (a) Formation of carbocation
65. X reacts with acid chloride to give ester X is (b) Formation of carbanion
(a) Phenol (b) Benzoic acid (c) Formation of free radical
(c) Methnoyl chloride (d) Acid anhydride (d) Formation of Transition state
66. Aldehyde which do not show Cannizarro
75. Reagent of Clemmenson’s reduction is
reaction
O (a) NH 2 NH 2 (b) Zn Hg / HCl
|| (c) Pred / HI (d) Pd / H 2 , BaSO 4
(a) H C H (b) (CH 3 ) 3 CHO
76. The number of structural isomer of
(c) C 6 H 5CHO (d) CH 3CHO CrCl3.6H2O are
67. Lucas test is used to distinguish between (a) 4 (b) 2 (c) 3 (d) 1
(a) 1°, 2° and 3° amines 77. How many atoms are present in bcc ?
(b) 1°, 2° and 3° alcohol (a) 3 (b) 4 (c) 2 (d) 1
(c) Aromatic and aliphatic acids 78. Which of following pair present pseudohalide
(d) 1°, 2° and 3° amides and polyhalide respectively ?
68. The number of radial nodes in 5d is (a) OCN– and BrI2– (b) RCOO– and BrI2–
(a) 1 (b) 2 (c) 3 (d) 4 (c) NNN– and IF5 (d) OCN– and IF5
69. Increasing order of electronegativity of hybrid 79. Units of rate constant of a given chemical
orbital is reaction is L mol–1s–1. What is order of reaction?
(a) sp sp 2 sp 3 (b) sp sp 2 sp 3 (a) 0 (b) 1 (c) 2 (d) 3
80. Which of following is least paramagnetic in nature?
(c) sp sp3 sp 2 (d) sp 3 sp sp 2
(a) Mn (b) Fe (c) Ni (d) Cu
70. Which is longest bond ? 81. Which among the following will have heighest
(a) N 2 (b) O 2 (c) I 2 (d) Cl 2 B.P at 1 atm. pressure ?
71. Hinsberg’s reagent is used to distinguish (a) 0.1 M NaCl (b) 0.1 M BaCl2
between (c) 0.1 M Sucrose (d) 0.1 M urea
(a) acids (b) alcohols 82. Which of the following has smallest ionic radii?
(c) amides (d) amine (a) Na+ (b) Mg+ (c) F– (d) Al+3
Mock AIIMS 7

83. Given compound is subjected to chemical 92. Which of following principle/experiment shows
analysis. Results are quantisations of energy in an atom ?
I. –ve test to Ninhydrin (a) Heisenberg’s uncertainty principle
II. +ve test to Benedict’s solution (b) Auf baus principle
Compound is (c) Pauli’s exclusion principle
(a) Lipid (b) Monosaccride (d) H-spectrum
(c) Protein (d) Amino acid 93. Conjugate acid of CH 3 NH 2 is
84. Iodoform test is show by
(a) Glycol (b) Propanaldehyde (a) CH 3 NH (b) NH 2
(c) Ethanol (d) Diethyl ether
85. The correct order of penetrating power is (c) CH 3OH (d) CH 3 NH 3
(a) (b) 94. Sublimation energy of I 2 (s) is 57.3 kL/ mol and
(c) (d) enthalpy of fusion is 15.5 kJ/mol.
86. Given electron would enter which of the
The enthalpy of vaporisation of I 2 is
following shells first
(a) n = 5, = 0 (b) n = 3, =2 (a) 41.8 kJ/mol (b) 72.8 kJ/ mol
(c) – 72.8 kJ/ mol (d) – 41.8 kJ/mol
(c) n = 6, = 0 (d) n = 5, =1
95. 2-butyne on reaction with Pd/ BaSO 4 gives
3
87. [Cr(SCN )(H 2 O)5 ] and (a) Cis – 2 – butene (b) Trans – 2 – butene
[Cr( NCS)(H 2 O) 5 ] 3 show (c) 1 – butene (d) 2 – hydroxy butene
(a) Ionization isomerism 96. If H is (–) and S is (+) G will be
(b) Hydration isomerism (a) (–) ve (b) (+) ion
(c) Link age isomerism (c) Zero (d) H T S
(d) Co-ordination isomerism 97. The given reaction has reagent X as
88. Buna – S is polymer of
(a) Butadiene X
CH 3 C C CH 3
(b) Butadiene and nitrile H 2O / Zn
(c) Butadiene and styrene
CH 3 C C CH 3
(d) Butadiene and isoprene || ||
89. Which of following has bond order zero ? O O
(a) CO (b) O 2 (c) F2 (d) Be 2
(a) O2 (b) HNO 3
90. Maximum enol content is in
(c) O 3 (d) KMnO4
O O O
|| || || 98. Which of following is most reactive towards
(a) (b) nucleophilic substituting reaction
(a) CH2 = CH – Cl
O O O (b) C6H5 – Cl
|| || ||
(c) CH3CH = CHCl
(c) H (d) H (d) ClCH2 – CH = CH2
91. Which of following reduces Benedict’s solution? 99. BaCO3 BaO + CO 2 is an endothermic
O O reaction formation of BaO is favoured by
|| || (a) Decrease in temperature
C (b) Decrease in pressure
(a) (b) R OH (c) Increase in concentration
(c) HCHO (d) R – O – R (d) Increase in pressure
EBD_7100
8 Topicwise AIIMS Solved Papers

100. A is 0.5M solution of Ca ( NH 3 ) 2 and B is 0.75 105. Which is not true about hydrogen peroxide ?
m solution of KOH (a) If acts as both oxidising and reducing agent
(b) If is pale blue liquid
Depression in freezing point is
(c) If can be oxidised by O3
(a) Greater for b due to more concentration (d) Two OH bonds lies in same plane
(b) equal in both A and B freezing point is less 106. Which is a true peroxide ?
then zero (a) CO2 (b) MnO2
(c) equal to 0°C in both as ionic concentration (c) Na2O (d) BaO2
is negligible 107. The conductance of Li salts is lowest of all group
(d) Greater for a because number of ions is greater I metals
(a) easy difusion of Li+ ion
101. Calculate the volume of H 2 gas at NTP (b) lower ability of Li+ ion to polarise water
obtained by passing 2 ampere through acidified molecules
water for 1 hour. (c) lowest charge to radius ratio
(a) 0.0836 L (b) 0.0432 L (d) high degree of hydration of Li +
(c) 0.1672 L (d) 0.836 L 108. Which of following has highest protective power
on lyophobic collids ?
102. In an antiflourite structure cation occupies
(a) gum arabic (b) sodium oleate
(a) Octahedral void (b) Tetra hedral void (c) gelatin (d) starch
(c) Centre of cube (d) Edges of cube
109. 0.1 M solution of [Ag ], [Ba ] and [Ca ] is
103. Which is not permissible ?
added to solution of sodium sulphate. K sp
1
(a) n = 3, = 1, m = – 1, s = values for these salts are : BaSO 4 11
2 10 ;
6 5
1 CaSO 4 10 ; Ag 2SO 4 10
(b) n = 2, = 2, m = – 1, s = – Which of those will precipitate Ist ?
2
(a) BaSO 4 (b) Ag 2SO 4
1
(c) n = 4, = 2, m = – 1, s = –
2 (c) CaSO 4 (d) All the above

1 110. Which is not a reducing agent ?


(d) n = 4, = 2, m = 2, s = –
2 (a) LiNH 4 (b) Na/liq ammonia
Lindlar 's (c) Lindlar’s reagent (d) SeO 2
104. A CH 3 C CH 3
reagent DIRECTIONS : In the following questions an
Assertion (A) is given followed by a Reason (R). Mark
Na / liq. NH3 your responses from the following options.
B; A and B are
(a) Both Assertion and Reason are true and Reason
(a) Cis, trans 2 butene
is the correct explanation of 'Assertion'
(b) Trans, trans 2 butene (b) Both Assertion and Reason are true and Reason
(c) Cis, cis 2 butene is not the correct explanation of 'Assertion'
(c) Assertion is true but Reason is false
(d) Trans, cis 2 butene
(d) Assertion is false but Reason is true
Mock AIIMS 9

111. Assertion (A) : A very dilute acidic solution of 119. Assertion (A) : If hydration energy is greater
Cd2+ and Ni2+ gives yellow precipitate of CdS than lattice energy the solid dissolves in liquid
on passing hydrogen sulphide. Reason (R) : The solubility of a solid in a liquid
Reason (R) : Solubility product of CdS is more depends upon lattice energy and hydration
than that of NiS. energy
112. Assertion (A) : [Ni(CN)4]2– has square planar
120. Assertion (A) : Reduction of m-dinitrobenzene
and [NiCl4]2– has tetrahedral shape.
with ammonium sulphide gives m-nitroaniline.
Reason (R) : [Ni(CN)4]2– is diamagnetic while
NiCl42– is paramagnetic. Reason (R) : m-Nitroaniline formed gets
113. Assertion (A) : Rusting of an iron is an example precipitated and hence further reduction is
of corrosion. prevented.
Reason (R) : Rusting of iron is decreased by
acid and electrolytes. SECTION III - BIOLOGY
114. Assertion (A): Trihydroxyglutaric acid (HO2C–
CHOH–CHOH–CHOH–CO2H) exists in four
121. Sequence of taxonomic categories is
stereoisomeric forms; two of which are optically
(a) Class – Phylum – Tribe – Order – Family –
active while the other two are meso-forms.
Genus – Species
Reason (R) : It contains two asymmetric and
(b) Division – Class – Family – Tribe – Order –
pseudo-asymmetric carbon atom.
Genus – Species
115. Assertion (A) : The endothermic reactions are
(c) Division – Class – Order – Family – Tribe –
favoured at lower temperature and the exothermic
Genus – Species
reactions are favoured at higher temperature.
(d) Phylum – Order – Class – Tribe – Family –
Reason (R) : When a system in equilibrium is
Genus – Species
disturbed by changing the temperature, it will
122. Genophore/bacterial genome or nucleoid is made
tend to adjust itself so as to overcome the effect
of
of change.
(a) Histones and nonhistones
116. Assertion (A) : For each ten degree rise of
(b) RNA and histones
temperature the specific rate constant is nearly
(c) A single double stranded DNA
doubled.
Reason (R) : Energy-wise distribution of (d) A single stranded DNA
molecules in a gas is an experimental function of 123. Claviceps purpurea is causal organism of
temperature. (a) Smut of Barley
117. Assertion (A) : HNO3 is a stronger acid than (b) Rust of Wheat
HNO2 (c) Ergot of Rye
Reason (R) : In HNO3 there are two nitrogen-to- (d) Powdery Mildew of Pea.
oxygen bonds whereas in HNO2 there is only 124. Chloroplast of Chlamydomonas is
one. (a) Stellate (b) Cup-shaped
118. Assertion (A) : A metal having negative (c) Collar-shaped (d) Spiral
reduction potential when dipped in the solution 125. Oxysomes of F0 – F1 particles occur on
of its own ions has a tendency to pass into the (a) Thylakoids
solution. (b) Mitochondrial surface
Reason (R) : Metal having negative reduction (c) Inner mitochondrial membrane
potential have large hydration energy. (d) Chloroplast surface
EBD_7100
10 Topicwise AIIMS Solved Papers
126. Two linked genes a and b show 20% 135. An ovule which becomes curved so that the
recombination. The individuals of a dihybrid nucellus and embryo sac lie at right angles to
cross between + +/ + + × ab/ab shall show the funicle is
gametes (a) Hemitropous (b) Campylotropous
(a) + + 80 : ab : 20 (c) Anatropous (d) Orthotropous
(b) + + 50 : ab : 50 136. Which of the following movement is not related
(c) + + 40 : ab 40 : + a 10 : + b : 10 to auxin level
(d) + + 30 : ab 30 : + a 20 : + b : 20 (a) Bending of shoot towards light
127. In Escherichia coli, lac operon is induced by (b) Movement of root towards soil
(a) Lactose (b) Promoter gene (c) Nyctinastic leaf movements
(c) -galactosidase (d) I-gene (d) Movement of sunflower head tracking the
128. If a diploid cell is treated with colchicine then it sun
becomes 137. An interesting modification of flower shape for
(a) triploid (b) tetraploid insect pollination occurs in some orchids in
(c) diploid (d) monoploid which a male insect mistakes the pattern on the
129. Syngenesious condition is found in orchid flower for the female of his species and
(a) Asteraceae (b) Labiate tries to copulate with it, thereby pollinating the
(c) Solanaceae (d) Fabaceae
flower. This phenomenon is called
130. Floral formula of Tomato/Tobacco is
(a) Mimicry
(a) + K4–5 A10G(2) (b) Pseudopollination
(b) + + K2+2 C4A2+4G1 (c) Pseudocopulation
(d) Pseudoparthenocarpy
(c) + + P2 A3G1
138. The most common indicator organism that
(d) + + K(5) C(5)A5 G(2) represents polluted water is
(a) E. coli (b) P. typhi
131. Ectophloic siphonostele is found in
(c) C. vibrio (d) Entamoeba
(a) Osmunda and Equisetum
139. In order to obtain virus-free plants through
(b) Marsilea and Botrychium
tissue culture the best method is
(c) Adiantum and Cucurbitaceae
(a) Embryo rescue (b) Anther culture
(d) Dicksonia and Maidenhair fern
(c) Meristem culture (d) Protoplast culture
132. If a cell A with DPD 4 bars is connected to cell B,
140. Which one among the following chemicals is
C, D whose OP and TP are respectively 4 and 4, used for causing defoliation of forest trees?
10 and 5 and 7 and 3 bars, the flow of water will be (a) Phosphon-D
(a) A and D to B and C (b) Malic hydrazide
(b) A to B,C and D (c) 2, 4 Dichlorophenoxy acetic acid
(c) B to A, C and D
(d) C to A, B and D (d) Amo-1618
133. The size of chlorophyll molecule is 141. Which of the following is not true for a
(a) Head 15 × 15 Å, tail 25 Å species?
(b) Head 20 × 20 Å, tail 25 Å (a) Members of a species can interbreed.
(c) Head 15 × 15 Å, tail 20 Å (b) Gene flow does not occur between the
(d) Head 10 × 12 Å, tail 25 Å populations of a species.
134. Terminal cytochrome of respiratory chain which (c) Each species is reproductively isolated
donates electrons to oxygen is from every other species.
(a) Cyt. b (b) Cyt. c (d) Variations occur among members of a
(c) Cyt. a1 (d) Cyt. a3 species.
Mock AIIMS 11

142. The catalytic efficiency of two different 149. If Henle's loop were absent from mammalian
enzymes can be compared by the nephron, which of the following is to be
(a) formation of the product expected?
(b) pH optimum value (a) The urine will be more dilute
(c) Km value (b) There will be no urine formation
(d) molecular size of the enzyme (c) There will be hardly any change in the
143. Fire bellied toad is quality and quantity of urine formed
(a) Amphiuma (b) Banbina (d) The urine will be more concentrated
(c) Necturus (d) Salamandra 150. Number of cervical vertebrae in camel is
144. American water plant that has become a (a) More than that of Rabbit
troublesome water weed in India is (b) Less than that of Rabbit
(a) Cyperus rotundus (c) Same as that of Whale
(b) Eichhornia crassipes (d) More than that of Horse
(c) Trapa latifolia 151. Which of the following cranial nerves can
(d) Trapa bispinosa regulate heart beat?
145. Characteristics of smooth muscle fibres are (a) X (b) IX
(a) Spindle-shaped, unbranched, unstriated, (c) VIII (d) VII
uninucleate and involuntary
152. Which one of the following pairs correctly
(b) Spindle shaped, unbranched, unstriped,
matches a hormone with a disease resulting from
multinucleate and involuntary
its deficiency?
(c) Cylindrical, unbranched, unstriped,
(a) Luteinizing - Failure of
multinucleate and involuntary
hormone ovulation
(d) Cylin drical, unbranched, striated,
(b) Insulin - Diabetes insipidus
multinucleate and voluntary
(c) Thyroxine - Tetany
146. An adolescent human below 17 years of age
normally has dental formula as (d) Parathyroid - Diabetes mellitus
hormone
2,1,3,2 2,2,3,2 153. The growth of corpus luteum is initiated by
(a) (b)
2,1,3,2 2,2,3,2 (a) Human chorionic gonadotropin
(b) Follicle stimulating hormone
2,1,2,0 2,1,2,2
(c) (d) (c) Luteinizing hormone
2,1,2,0 2,1,2,2 (d) Prolactin
147. In alveoli of the lungs, the air at the site of gas 154. Two opposite forces operate in the growth and
exchange, is separated from the blood by development of every population. One of them
(a) alveolar epithelium only relates to the ability to reproduce at a given rate.
(b) alveolar epithelium and capillary The force opposing it is called
endothelium (a) environmental resistance
(c) alveolar epithelium, capillary endothelium (b) morbidity
and tunica adventitia (c) fecundity
(d) alveolar epithelium, capillary endothelium, (d) biotic potential
a thin layer of tunica media and tunica 155. Red-green colour blindness in humans is
adventitia governed by a sex-linked recessive gene. A
148. Splenic artery arises from normal woman whose father was colour-blind
(a) Anterior mesenteric artery marries a colour blind man. What proportion of
(b) Coeliac artery their daughters is expected to be colour-blind?
(a) 3/4 (b) 1/2
(c) Posterior mesenteric artery
(c) 1/4 (d) All
(d) Intestinal artery
EBD_7100
12 Topicwise AIIMS Solved Papers
156. Genetic drift operates only in (d) If both the Assertion and Reason are incorrect.
(a) Smaller Populations (e) If the Assertion is incorrect but the Reason is
(b) Larger Populations correct.
(c) Mendelian Populations
(d) Island Populations
161. Assertion : Mango dipped in concentrated
157. Which one of the following statements about
sodium chloride solution will contract.
fossil human species is correct?
(a) Fossils of Homo neanderthalensis have Reason : Water goes out due to exosmosis in
been found recently in South America hypertonic solution.
(b) Neanderthal man and Cro-Magnon man did 162. Assertion : Nissl’s granules that are basophilic
exist for sometime together are present in the cyton.
(c) Australopithecus fossils have been found Reason : They are composed of RNA.
in Australia 163. Assertion : Phycobilins are destroyed by heat.
(d) Homo erectus was preceded by Homo Reason : They are protein linked and proteins
habilis are denatured due to heat.
158. In the silk worm, if no juvenile hormone (JH) is
164. Assertion : The inner mucosa coat has
present when it moults, it will innumerable finger like projections.
(a) die Reason : Absorption increases due to infolds.
(b) moult into another larval stage
165. Assertion : Vital capacity is the total volume of
(c) moult into pupa air that can be breathed out with minimum effort.
(d) moult into an adult Reason : Vital capacity represents the maximum
159. Which one of the following is correct match? capacity of an individual to renew air in the
(a) Reserpine — Tranquilizer respiratory system.
(b) Cocaine — Opiate narcotic 166. Assertion : Heart wood is non functional.
(c) Morphine — Hallucinogenic Reason : Duramen is plugged due to in-growth
(d) Bhang — Analgesic of collenchyma.
160. Test tube baby is one who 167. Assertion : Glycogen is called animal starch.
(a) is born out of artificial insemination Reason : Glycogen is stored in the liver and
(b) has undergone development in a test tube muscles of animals.
(c) is born out of the technique of fertilization 168. Assertion : Racemose Inflorescence is an
in vitro indeterminate inflorescence.
(d) has been developed without fertilization Reason : The Inflorescence shows definite
growth.
DIRECTIONS : These questions consist of two 169. Assertion : Mouth parts of cockroach, honey
statements, each printed as Assertion and Reason. bee & mosquito are analogus organs.
While answering these questions, you are required to Reason : These organs follow the same basic
choose any one of the following five responses. plan of organization during development.
(a) If both Assertion and Reason are correct and 170. Assertion : Crossing over occurs at four strand
the Reason is a correct explanation of the or tetrad stage.
Assertion. Reason : Parent strand and gene linkages
(b) If both Assertion and Reason are correct but disappear at two strand stage.
Reason is not a correct explanation of the 171. Assertion : Oncogenes transform normal cell
Assertion. into cancer cell.
(c) If the Assertion is correct but Reason is Reason : They integrate their DNA with RNA
incorrect. of the host cells.
Mock AIIMS 13

172. Assertion : Human immuno deficiency Virus-III 182. On July 18, 2017, the Supreme Court has allowed
are retroviruses. the Centre to replace the oversight committee
Reason : They exhibit reverse transcription. set up to supervise the functioning of the
173. Assertion : Genes are actual physical units of Medical Coucil of India (MCI) with a fresh panel
heredity. of how many eminent doctors?
Reason : Genes are in the chromosome in linear (a) 8 eminent doctors
order. (b) 7 eminent doctors
174. Assertion : Viruses are obligatory parasites. (c) 6 eminent doctors
Reason : They show host specificity and (d) 5 eminent doctors
multiply only inside living systems. 183. Who among the following is set to become the
175. Assertion : Allergens are generally weak United Nations' youngest-ever'Messenger of
antigens, which are glycogen molecules. Peace'?
Reason : Allergy means inappropriate reaction (a) Mark Zuckerberg
of a person. (b) Malala Yousafzai
176. Assertion : Interferons are antiviral proteins. (c) Selena Gomez
Reason : It is released from the infected and (d) Virat Kohli
dying cells. 184. Find out the correct sequence -
177. Assertion : Horticulture is the conscious raise (a) Pain, Doctor, Hospital, Drug
of Cereal crop. . (b) Hospital, Doctor, Drug, Pain
Reason : Growing of vegetables, fruits and (c) Hospital, Doctor, Pain, Drug
ornamental plants is Horticulture. (d) Pain, Hospital, Doctor, Drug
185. Human : Brain :: Computer : ?
178. Assertion : Viruses cannot metabolise outside
host cells and use host machinery to produce (a) USB (b) Monitor
own nucleic acids and proteins. (c) CPU (d) Internet
186. Which of these is a dwarf planet?
Reason : Viruses lack energy yielding and
biosynthetic machinery. (a) Neptune (b) Titan
(c) Eris (d) Hydra
179. Assertion : All aggregate and multiple fruits
187. M is son of P, Q is the grand-daughter of O,
are false.
who is the husband of P. How is M related to
Reason : They develop from other floral parts
O?
instead of the ovary
(a) Son (b) Daughter
180. Assertion : Casuarina and Betula show
(c) Mother (d) Father
chalazogamy.
188. How many meaningful English words can be
Reason : Pollen tube enters the ovule through formed with the letters URLE using each letter
micropyle end.
only once in each word?
SECTION IV - G. K. (a) None (b) One
(c) Two (d) Three
181. On August 12, 2017, Dr Vishwanath Karad MIT 189. Name the country that will host the 2018 Table
World Peace University, considered to be the Tennis Team World Cup.
first of its kind in India was inaugurated in: (a) South Africa (b) China
(a) Pune (b) Bhopal (c) USA (d) England
(c) Jaipur (d) Chennai
EBD_7100
14 Topicwise AIIMS Solved Papers

190. Select the related word/letters/ number from the 196. On July 21, 2017, Uttar Pradesh Assembly
given alternatives. adopted two resolutions on naming the airport
Psychology : Human Being :: Ornithology : ? terminals of which two cities?
(a) Birds (b) Volcanoes (a) Kanpur and Lucknow
(c) Insects (d) Reptiles (b) Lucknow and Bareilly
191. The 2017 International Day for th e (c) Kanpur and Meerut
Remembrance of the Slave Trade and its (d) Kanpur and Bareilly
Abolition is observed on which date? 197. Which country is accused of interfering and
(a) August 24 (b) August 25 hacking the US 2016 presidential elections?
(c) August 22 (d) August 23 (a) Russia (b) China
192. Who is the head of the 9-judge Constitution (c) Germany (d) Japan
bench of the Supreme Court (SC) to determine 198. On July 23, 2017, Arun Jaitley inaugurated 315th
whether privacy is a fundamental right or not Rest House of the Kendriya Sainik Board in:
under the Constitution? (a) Shimla (b) Dehradun
(a) Abhay Manohar Sapre (c) New Delhi (d) Amritsar
(b) J S Khehar 199. Arran ge the followin g steps of AIIMS
(c) Sanjay Kishan Kaul application form
(d) Fali Narima (1) City Choice
193. Which of the following is the correct description (2) Payment
of the term 'sex ratio' as used in context of the (3) Password received
census? (4) Registration
(a) Number of females per 1000 persons (a) 3,4,2,1 (b) 4,3,1,2
(b) Number of females in a sample of 1000 (c) 3,4,1,2 (d) 3,1,4,2
persons 200. Which of the following diagrams indicates the
(c) Number of males per 1000 females best relation between Pluto,Planets, Sun and
(d) Number of females per 1000 males Earth ?
194. In India, which city is also known as 'City of
Palaces'? (a) (b)
(a) Jaipur (b) Kolkata
(c) Gwalior (d) Udaipur
195. Heena Sidhu, who recently won a World Cup (c) (d)
Gold Medal for India, is associated with
(a) Shooting (b) Archery
(c) Weightlifting (d) Boxing
Mock AIIMS 15

SOLUTIONS
SECTION I - PHYSICS 3V
n1
4 1
1. (b) From the F.B.D.
N = mg cos 3V 3V
Given n1 n2 or
F = ma = mg sin – N 2 2 4 1
a g (sin cos ) 1 1
N 2 2
N
V2 HR
4. (c) H t or t
mg sin m g cos R V2
xmg The voltage, V remains same, H is also same
HR1 HR 2
Now using, v 2 u2 2as t1 2 ; t2
V V2
2
or, v 2 g (sin cos ) H( R 1 R 2 )
or t = t1 t 2
( = length of incline) V2
or, v = 2g (sin cos ) = 15 + 20 = 35 min
2. (b) Acceleration due to gravity at lattitude’ ’ 8
5. (a) Fluid resistance is given by R =
r4
is given by g g e R e 2 cos 2 When two capillary tubes of same size are
At equator, = 90° joined in parallel, then equivalent fluid
cos = cos 90° = 0 resistance is
or g = g e = g (as given in question) R S R1 R 2
3 8 8 2L 8 L 9
At 30°, g 30 g R 2 cos 2 30 g R 2 = =
4 R4 (2 R ) 4 R4 8
Rate of flow
3
or, g g 30 R 2 R4
4 P 8 8 PR 4
= = X as X
3. (a) For 3rd harmonic/2nd over tone of organ RS 8 L 9 9 8
pipe open at ends 6. (a) The charge is moving in an equipotential
2
line. So no work is done.
7. (d) Optical fibers carry immense no. of signals
as compared to other wires. There is no
EM interference in these fibres.
8. (c) An EMW is the one constituted by
/4 /4
oscillating electric and magnetic field which
3V oscillate in two mutually perpendicular
n2
2 2 planes. The wave itself propagates in a
direction perpendicular to both of the
For 1st overtone of organ pipe open at one
end directions of oscillations of electric ( E )
1
and magnetic fields ( B) , i.e. E B.
9. (a) The angular magnification,
angle subtended by the image at eye
M=
/2 /4
angle subtended at eye with object
in actual position
EBD_7100
16 Topicwise AIIMS Solved Papers

f o F1 13. (d) Volume of bigger bubble


For telescope, M = = volume of 27 smaller bubbles
f e F2
10. (c) As per Reynold’s formula critical velocity 4 4 3 D
D3 27 d d
of a liquid is defined as 3 3 3
K 1 1
vc vc & vc Initial surface energy Si 4 D2
r r
Where is coefficient of viscosity of the Final surface energy Sf 27 4 D 2
liquid, its density and r is the radius of
the tube. K is a dimensionless constant D
S Sf Si and using d
called the Reynold number. Thus critical 3
velocity increases when density and radius
of the tube decreases. D2
11. (b) Let the fundamental frequency of organ S 4 27 D2
9
pipe be f
2
Case I : f = 200 5 = 205 Hz or 195 Hz = 2D 4 8 D2
V1 M1 64
14. (a) 4
V2 M2 M1
or M1 4 i.e. He
15. (b) At constant pressure
W P(Vf Vi ) nR (Tf Ti )
Case II : frequency of 2nd harmonic of = 1× 8.14 (127 – 27 ) = 8.14 × 100 = 814 J
organ pipe = 2f (as is clear from the second
figure) 16. (a) mv 2
qvB
2f = 420 10 or f = 210 5 r
31
or f = 205 or 215 mv 9.1 10 10 6
Hence fundamental frequency of organ B= 19
qr 1.6 10 0. 5
pipe
= 205 Hz = 1.13 10 5 T
12. (a) The moment of inertia (I) of circular ring 17. (a) Remember that acceleration of a cylinder
whose axis of rotation is passing thought down a smooth inclined plane is
its center, I1 m1R 2 N N

Also, I 2 m 2 (nR) 2
mg sin m g cos
Since both rings have same density, xmg
m2 m1
2 (nR) A 2 2 R A1 g sin mR 2
a where I is the
Where A is cross-section of ring, I 2
1
A1 A 2 (Given) m 2 nm1 mR 2
moment of Inertia for cylinder
I 1 m1R 2 m1R 2
Given 1 = = g sin 30 g
1
I2 8 m 2 (nR) 2 nm1 (nR) 2 a 2 g
mR 2 1 1 3
1 1 1 1
or n=2
2 mR 2 2
8 3
n
Mock AIIMS 17
18. (b) According to Kepler’s third law, current flowing due to charge e. Further
2 orbital motion of electron is equivalent to a
R T 3 27 Te current
R3 T2 9
Re Te Te e
I e
19. (a) The gravitational field intensity at the T
centre (of an equilateral triangle), 1
equidistant from the three vertices due to 3 (where T is the time period)
equal masses will be zero. The vector sum
of the forces due to the 3 masses will be M = IA e r 2
zero. 25. (c) Initial force between the two spheres
20. (a) Kinetic energy = translational kinetic carrying charge (say q) is
energy + rotational kinetic energy
1 1 1 q2
K. E = mv 2 I 2 F
2 2 4 r2 0
2 (r is the distance between them)
Moment of inertia of sphere (I) = MR 2 Further when an uncharged sphere is kept
5
in touch with the sphere of charge q, the
2
1 1 2 V 7 q 0 q
K.E. mv 2 MR 2 mv 2 net charge on both become .
2 2 5 R 10 2 2
21. (c) V' = V × IR = 0.5 + 0.1 × 20 = 2.5 V Force on the 3rd charge, when placed in
0.5V 20
center of the 1st two
r/2 r/2
0.1A

1 3 2
q q/2 q/2
V

22. (a) Pot. gradient = 0.2mV/cm 2


q q
3
q
0.2 10 2 1 2 1 2
= 2 = 2 10 v/m F3
10 4 2 4 2
0 r 0 r
Emf of cell = 2×10–2×1m = 2 10 2 V 2 2
= 0.02 V
As per the condition of potentiometer 1 q2
0.02 (R + 490) = 2 (R) or 1.98 R = 9.8 [2 1] F
4 0 r2
9.8 26. (b) A bimetallic strip, on uniform heating, bends
R= = 4.9
1.98 in the form of an arc and the metal with
23. (c) Work done in rotating a dipole by an angle greater ‘ ’ lies on the convex side.
‘ ’ is 27. (c) General wave equation
pE y A sin( t kx)
W pE(1 cos ) pE(1 cos 60)
2 On comparing, we get 100
Again, W180 pE (1 cos 180) 100 1
Wave number, k = m
pE[1 ( 1)] 2pE 4W v 100
24. (a) Magnetic moment = M = IA, where A is 28. (c) For constant pressure,V T
the area of the orbit ( r ) and I is the 2 29. (c) K and K are characteristics line in the
X-ray line spectrum.
EBD_7100
18 Topicwise AIIMS Solved Papers
36. (b) The equivalent circuit can be redrawn as
30. (d) T 2 R R
g
R R
1 1 A B
Frequency, n R R
T length R R 2R
R R
n1 2 2 2 1 9
n2 3 4 2R / 3
1 1 2
R 5R / 3
31. (c) Ve 2gR and V0 gR 5R / 8
2
Ve 2 V0 V0 2 km/s 37. (b) As per Einstein’s photoelectric equation :
2
32. (a) m = Zit E= h wF KE max
7 7 i.e. till a certain valve of , KE remains 0, it only
m = 3.3 10 3 2 = 19.8 10
starts increasing once the Work function
C
33. (c) (as per Curie’s law) (WF) of the metal surface is achieved.
T
Paramagnetic materials obey Curies law. u 2 sin 2 u 2 sin 2
C = Curies constant 38. (a) R ; R max
34. (a) When the plane polarised light passes g 2g
through certain substance, the plane of
polarisation of the light is rotated about
the direction of propagation of light h max .
through a certain angle.
35. (a) For A: It goes up with velocity u will it
reaches its maximum height (i.e. velocity R
becomes zero) and comes back to O and sin 2
attains velocity u. Equating we get sin 2
2
Using v 2 u2 2as vA u2 2gh or 4 sin cos sin 2
tan 4 or 1
u tan 4
2
u sin 2
u Vch
39. (c) R will be maximum for
O g
u
sin 2 = 1 2 or = 45
h 2 4

VB VA u
40. (a) R 42 52 41N
Vch
5
VCV
VC
The angle will be given by tan
4
For B, going down with velocity u
vB u2 2gh
For C, horizontal velocity remains same, i.e. ‘u’. 5N R
Vertical velocity = 0 2gh = 2gh
4N
The resultant v C = v 2x v 2y = u2 2gh . 5
or tan
Hence v A vB vC 4
Mock AIIMS 19
41. (c) At steady state the capacitor will be fully V
charged and thus there will be no current in R 2 = I2 =
R R2
the 1 resistance. So the effective circuit
becomes Further, as heat generated is same, so
2
I12 R1t I 22R 2 t
I1 2 2
A B V V
I2 3 or R1 = R2
R R1 R R2

I R1 (R R 2 ) 2 = R 2 (R R1 ) 2

6V 2. 8 R 2 R1 R1R 22 2RR 1R 2
Net current from the 6V battery, 2
= R R2 R1 R 222
2RR 1R 2
6 6 3
I 1.5A 2
2 3 2.8 1.2 2.8 2 R (R1 R 2 ) R 1R 2 ( R 1 R 2 )
2 3 1
Between A and B, Voltage is same in both R = R 1R 2
resistances, 46. (a) Microwaves are used for communication
2I1 3I 2 where I1 I 2 I 1.5 in artificial satellites.
2I1 3(1.5 I1 ) I1 0.9A h
47. (c) de Broglie wavelength, =
42. (a) Convex lens can form image with m < 1, m > 2mEK.E
1 and m = 1 depending upon the positionof
the object. Convex lens forms magnified m 4m p
p
image (m > 1) when the object is pole and
2f, same size as the object (m = 1) when mp mp
the object is at 2f and smaller image (m < 1),
[ E K.E ( ) E K.E ( p) ]
when the object is beyond 2f.
43. (a) When the source (star) is moving towards
p 2
the observer (earth), =
Apparent wavelength, 1
' C Vs Vs Vs E ML2 T 2
= = 1 = 48. (c) h = = = ML T
2 1
C C C
T 1
' = apparent shift in wavelength 49. (d) Black body spectrum
Vs 50. (b) Fringe width . Also blue red
= (C = velocity of sound) Therefore, fringes come closer when blue
C
light is replaced by red light in diffraction
44. (b) Case - I : When resistor is not connected
pattern.
Using V = IR V = 25 (R G ) .............. (i) 51. (d) In p–n junction, the diffusion of majority
Case - II : When resistor is connected carriers takes place when junction is
V = 5(20 + R G ) = 100 + 5 R G ............. (ii) forward biased and drifting of minority
carriers takes place across the function,
From (i) and (ii), 20 R G = 100 when reverse biased. The reverse bias
RG = 5 opposes the majority carriers but makes the
minority carriers to cross the p–n junction.
45. (d) Let internal resistance of source = R
Thus the small current in µA flows during
Current in coil of resistance
reverse bias.
V 52. (b) In Rutherford’s –particle scattering
R1 = I1 =
R R1 experiments, some of -particles was found
Current in coil of resistance to be scattered at very large angles, inspite
EBD_7100
20 Topicwise AIIMS Solved Papers
of having very high kinetic energy. This capacitor is touched by someone, he may
shows that there are the –particle which feel shock due to large charge still present
will be passing very close to nucleus. on the capacitor. Hence it should handled
Rutherford’s confirmed the repulsive force cautiously otherwise this may cause a
on –particle due to nucleus varies with severe shock.
distance according to inverse square law
57. (b) A perfect black body is one which absorbs
and that the positive charges are
concen trated at the center and not heat radiation of all wavelengths, which fall
distributed throughout the atom. This is on it. Such a body neither reflects nor
the nuclear model of Rutherford. transmits any part of the incident heat
53. (a) When a polaroid is rotated in the path of radiation and hence appears black
unpolarised light, the intensity of light irrespective of the colour of the incident
transmitted from polaroid remains radiation. Obviously the absorbance of a
undiminished (because unpolarised light perfect black body is unity. The radiation
contains waves vibrating in all possible given out by a perfect black body are called
planes with equal probability). However, black body radiations or full radiation or
when the polaroid is rotated in path of plane total radiations.
polarised light, its intensity will vary from
maximum (when the vibrations of the plane 2
polarised light are parallel to the axis of the 58. (b) As we know, x . This is
polaroid) to minimum (when the direction
of the vibrations becomes perpendicular phase difference between two particles
to the axis of the crystal). Thus using whose path difference is x. If x = , then
polaroid we can easily verify that whether x = 2 . Thus, the phase difference between
the light is polarised or not. two medium particles having a path
difference is 2 , i.e., the particles are in
54. (e) An induced current develop in a conductor
cannot moved in a direction parallel to the same phase of oscillations.
magnetic field. This is because when the 59. (a) The presence of impurities either on the
conductor moved in a direction parallel to liquid surface or dissolved in it,
considerably affect the force of surface
magnetic field, amount of flux linked with
tension, depending upon the degree of
the conductor does not change. Thus the contamination. A highly soluble
induced current develops only when substance like sodium chloride when
conductor cuts the lines of magnetic force. dissolved in water, increased the surface
The direction of flow of induced current tension of water. But the sparingly soluble
can also be found by applying Fleming’s like phenol when dissolved in water
right hand rule, when the direction of reduces the surface tension of water.
motion of conductor inside the magnetic 60. (c) The orbital velocity, if a satellite close to
field and the direction of magnetic field
action on if are known. earth is V0 gR e , While the escape
55. (a) Drift velocity of free electrons is given by, velocity for a body thrown from the earth’s
eE surface is Ve 2gR e .
vd
m
V0 gRe 1
Potential difference V
Where, E = Thus V =
length e 2gRe 2
eV or Ve 2V0
vd
m i.e., if the orbital velocity of a satellite
1 eV revolving close to the earth happens to
i.e., v d where
is constant.
m increase to 2 times, the satellite would
56. (c) A charged capacitor, after removing the escape.
battery, does not discharge itself. If this
Mock AIIMS 21

SECTION II - CHEMISTRY 67. (b) (HCl (conc.) ZnCl 2 ) is Lucas reagent.


61. (a) Aromatic diazonium salts are generally Different alcohals react at different rate with
prepared by adding cold aqueous solution this reagent.
of sodium nitrite to solution/suspension ZnCl2
of 1° aromatic amine at 273 - 278 K ROH + HCl RCl + H 2 O
273 278K HCl / ZnCl 2
ArNH 2 NaNO 2 HX 1 alc.
HNO2
do not react at room temp.
ArN2 X NaX 2H 2O HCl / ZnCl 2
2 alc. turbidity appears in
62. (a) Some sols have a high concentration of
dispersed solid and change spontaneously 5 min.
into semisolid form on cooling these are 3 alc.
HCl / ZnCl
2 turbidity appears at
called gels thus they form liquid as
dispersed phase and solid as dispersion once.
medium. 68. (b) No. of nodes = n – – 1
5 1 For 5d = 5 – 2 – 1 = 2
63. (c) Cr has valance shell 3d 4s 69. (b) More is s-character more is
Number of valance electrons = 6 electronegativity of hybrid orbital..
H 2 O is monodentate sp has 50 % s-character
2 electrons from each H 2 O 2
sp has 33 % s-character
Total electrons contributed by H 2 O sp 3
has 25 % s-character
= 2 × 5 = 10 Order of electronegativity
SCN gives 2 electrons 3
Total valance electrons = 6 + 10 + 2 = 18 sp > sp 2 > sp
Over all charge on complex is + 2 70. (c) Bond length is in order as given
Valance electrons in complex are 16 Single bond > double bond > triple bond
64. (a) Valance shell is 3d 5 4s1 , It means inner N 2 has triple bond; O 2 has double bond
shells are 1s 2 2s 2 2p 6 3s 2 3p 6 I 2 and Cl2 have single bond
At no. = 24. Hence element is Cr. Out of these I 2 has longer bond length
O as, due to big atoms the inter nuclear
|| distance is large.
OH O C R 71. (d) C 6 H 5SO 2 Cl is Hinsberg reagent
O
||
65. (a) +R C X 1 amine 2 amine

Ester R NH 2 R2 NH
66. (d) Aldehyde which do not contain H C6 H 5SO 2 Cl C6 H 5SO 2 Cl
atom on treatment with alkali solution C6 H 5SO 2 NHR HCl C6 H 5SO 2 NR 2 HCl
undergo self oxidation - reduction reaction NaOH NaOH
(Cannizaro reaction) CH 3 CHO has C 6 H 5SO 2 N( Na )R No reaction
H Soluble
Cannot show Cannizaro reaction.
e.g. of Cannizaro reaction : 3 amine
HCHO + NaOH
CH 3OH + HCOONa
Do not react at all
EBD_7100
22 Topicwise AIIMS Solved Papers
76. (a) 4 Isomers are
72. (b) CH 3 CH 2 CH CH 2 O3
[Cr (H 2O) 6 ] Cl 3 ; [Cr ( H 2 O) 5 Cl] Cl 2 .H 2 O
O [Cr (H 2 O) 4 Cl 2 ] Cl .2H 2 O ;
C 2 H 5 CH CH 2 [Cr (H 2 O) 3 Cl 3 ].3H 2 O
77 (c) Contribution by 8 atoms present at corner
= 1/8 × 8 = 1
O O Contribution by atom present within the
H 2O / H body = 1
Numbers of atoms present per unit cell
C 2 H 5 CHO HCHO =1+1=2
78 (a) A few ions, consisting of two or more
electro negative atoms of which at least
73. (b) R Mg X O C O one is N and properties similar to halide
ions are called pseudohalides of NNN ,
R C O
OCN , CN etc.
H 2O
XMgO R C O (HO)MgX Halide ions often react with molecules of
halogens or interhalogen to form poly
HO halides like BrI 2 , where as IF5 is an inter
74. (a) 3° alcohals undergo reaction with SN 1
mechanism halogen compound. Note - RCOO is not
pseudohalide
R 3 C OH H R 3C O H 2 dx
79. (c) = K [Conc]n
dt
R 3C X
R 3C X dx 1 1 1
Carbocation k= × n
=
dt [Conc.] Time [ Conc.] n 1
R For 2nd order reaction,
R C is most stable carbocation.
1 1 –1
R k= × = sec –1 mol L
sec [ mol / L ]2 1
Note :- 1° alcohol show SN 2 mechanism

H 80. (d) 25 Mn 3d 5 , 4 s 2
R CH 2OH R CH 2 O H 2 Number of unpaired electrons = 5
X
X CH 2 OH 2 26 Fe 3d 6 ,4s 2
| Number of unpaired electrons = 4
R
T. S. 28 Ni 3d 8 , 4s 2
75. (b) Clemmenson’s reaction Number of unpaired electrons = 2
Zn / Hg
R CHO R CH 3 28 Cu 3d10 ,4s1
HCl
Number of unpaired electrons = 1
Note :- Wolf Kishner reduction is More is number of unpaired electrons
NH 2 NH 2 higher is paramagnetism.
R CHO RCH 3
81. (b) Elevation in B.P. is colligative property
Also R CHO P / HI R CH 3 which depends upon number of particles.
Rosenmund reaction :- Each BaCl 2 ion gives Ba 2 and 2Cl .
Pd / H 2 BaSO 4 Thus number of particles is highest out of
RCOCl RCHO
Lindlar reagent given compounds.
Mock AIIMS 23
82. (d) Size of ion depends upon nuclear charge. linkage isomerism occurs.
More is nuclear charge compared to number In this case SCN has S and N as donor
of electrons, Tightly are electrons held and atom which can link with the central atom.
thus smaller is size
HC CH 2
Nuclear charge No. of electrons
Na 11 10 88. (c) nCH 2 CH CH CH 2 + n
2 1, 3 - butadiene
Mg 12 10 Styrene

F 9 10
3 ( CH 2 CH CH CH 2 CH CH 2 CH 2 ) n
Al 13 10
In Al 3 13 units of positive charge tend Buna - S
to attract 10 electrons more towards
nucleus thus reducing sphere of electron 89. (d) CO has triple bond; B.O. must be 3
cloud around the nucleus i.e. size.
83. (b) Ninhydrin test is shown by proteins and O 2 has double bond, B.O. must be 2
amino acid. Negative result show absence F2 is single bonded, B.O. must be 1
of these two.
Benedict’s solution test is for aldehyde Be 2 does not exist, B.O. = 0
group which is present in monosaccrides Alternatively B.O.
and not lipids.
84. (c) Iodoform test is shown by compounds of Number of bonding No. of anti bonding e
=
aldehyde, ketone or alcohol which have 2
2 2
CH 3 gp at position like C H 3CHO , Be 2 2s 2 , * 0
2s ; B.O.=
2
90. (b) enolic form increases when there is H-
C H 3 COR , CH 3CR (OH) etc. Ethyl bonding. If this H-bonding is further
alcohol has CH 3CH(OH) thus shows stabilised by conjugation enolic form
iodoform test further increases.
O O
H || ||
| C C
C H3 C OH 4I 2 6 NaOH CHI 3
CH 3 CH3 CH 3 is max enolic form.
5NaI 5H 2O HCOONa
85. (d). Penetration power is more when particle is Tautomerism
small in size and is moving with high speed.
order of penetr ating power is O H O
||
X ray C C
86. (b) Order of filling follows (n ) rule, small is CH 3 CH CH 3
(n ) lower is energy so filling is first. H-bonding

If (n ) values are same than lower n is Conjugation


given preference for O H O
(a) n = 5, = 0, n + = 5
(b) n = 3, = 2, n + = 5 C C
(c) n = 6, = 0, n + = 6 CH 3 CH CH 3
(d) n = 5, = 1, n + = 6 91. (c) Benedict’s sol. test is shown by aldehydes.
n = 3, = 2, i.e. 3d is filled Ist Reason for easy oxidation of aldehyde to
87. (c) When more than one atom in a acid is presence of H- atom on carboxyl gp.
monodentate ligand can act as donor Due to which it acts as strong reducing
EBD_7100
24 Topicwise AIIMS Solved Papers
agent and can thus reduce weak oxidising Carbonyl compounds
reagents like Tollens, Benedict and Fehling H / H 2O
solution. C C O3 C C
Zn || ||
Note :– Benedict’s solution is alkaline O O
solution of Cu 2 complexed with citrate di ketones
ions. 98. (d) More easily Nucleophilc can replace halide
Fehling solution is alkaline solution of ion more is reactivity. In case of vinyl C =
Cu+2complexes with Rochelle salt (is Sod. C, C – Cl bond aquires some double bond
potassium tartrate). Tollens solution is character and become strong thus is not
ammonical silver nitrate (AgNO3 / NH4OH). easily replacable.
92. (d) H – Spectrum confirms quantisation of order is Allyl chloride > Vinyl > Chloro
energy within an atom. Benzene
93. (d) Conjugate acid and base differ by one
hydrogen only
CH3NH2 is a base its conjugate acid is [C C Cl C C Cl]
99. (b) According to Le Chatelier’s principal
CH 3 NH 3 (Which is +ve, electrons deficient (1) Increase in concentration of any reactant
species) being consumed during the reaction
favours the reaction.
94. (a) Given I 2 (s) I(g) ,
(2) For an endothermic reaction increase in
Hsub lim ation 57.3kJ / mol temperature favours reaction in forward
direction.
I 2 (s) I2 ( )
(3) High pressure is favourable for the reaction
, H fusion 15.5kJ / mol in which there is decrease in volume.
In given case faourable conditions are :
I2 ( ) I(g)
(a) Increase in conc. of BaCO3
, H vap H sub H fusion
(b) Increase in temparature
= 57.3 – 15.5 = 41. 8 kJ/mol
(c) Decrease in pressure
Pb / BaSO4
95. (a) HC C C CH 3 100. (b) Ionic concentration of A i.e.
Lindlar 's reagent
Ca ( NO 3 ) 2 0.5 3 1.5
CH 3 CH 3
C C
Ionic concentration of B i.e.
H H KOH = 0.75 × 2 = 1.5
Cis-2-butene Ionic concentration is same for A and B
96. (a) H = (–) i.e. exothermic reaction depression in freezing point will also be
S = (+) i.e. entropy factor also favours. same.
Thus for a spontaneous reaction G is 101. (d) Change Q = Current (Ampere) × time (sec)
always negative. Q = 2 × 60 × 60
G=( H ) T S = – (ve) 1
H2O H2 O2
Note : G (ve ) , reaction is non 2
spontaneous
; 2H 2e H 2 (1 mole is 22400cc
G 0 , reaction is in equlibrium.
97. (c) Ozone changes bond compound to at NTP) For 1 mol of H 2 liberated 2 faradays
carbonyl compounds are required 2F = 2 × 96500
H / H 2O 0.2 × 60 × 60 will give H2
C C + O3 C O
Zn 22400 2 60 60
= 835.64cc
+ C O 2 96500
= 0.836 L
Mock AIIMS 25
102. (b) In fluorite structure, cations form face
centered cubic a array and anions fit into Conc. of SO 4 is [SO 24 ] in Ag 2SO 4
tetrahedral void.
In antifluorite structure case is reversed,the 10 5 3
= 10
oxide ions fill half tetrahedral holes. 0.1 0.1
103. (b) Fon any n; n 1 , m = – to 0 to + Conc. of SO 4 is [SO 24 ] in CaSO 4
1
and s= 10 6
2 = 10 5
for n = 2, 2 0.1
104. (a) Lindlar catalyst is Pd/H2, BaSO4 poisoned Ionic product is minimum for BaSO4
with sulphur or quinoline It gives it must precipitate Ist.
us product Na / liq. NH3 (Birch reduction)
110. (d) SeO2 in alkaline/acidic medium
gives trans alkenes.
Note there are variety of other reducing dehydrogenates ketones to give ,
agents which gives trans or cis products
according to mechanism they follow. Some unsturated ketone. (Removal of hydrogen
are listed below is oxidation).
1.Wilkinson’s catalyst 111.(b) Cd2+ is a 2nd group radical and Ni2+ is a
RhCl ( H 2 )( PPh 3 ) cis addition 4th group radical. So solubility product of
2 . H y d r o b o r a t i o n NiS has to be more than CdS. Further Cd 2+
B2 H 6 cis addition gives yellow colour of CdS with H2S, but
Note : LiAlH4 and NaBH 4 normally Ni2+ gives black colour of NiS with H2S. So
donot reduce olefinic bonds. They are both assertion and statement are wrong.
selective towards carbonyl C = O double (d) is correct choice.
bond. 112.(b) In [NiCl4]2– the Cl– ligands present in the
105. (d) Value of dipole moment is not zero thus
H – O – O – H is not complex ion are less basic than CN–. As
supposed structure where both OH are in such no pairing of electrons in the 3d-
same plane. subshell takes place. This results in sp3
Dipole moment is some what near 2.1 D hybridisation and the complex so formed
Struture comes out to be
is tetrahedral. On the other hand in case of
H
[Ni(CN)4]2– the CN– ligands present in the
O O complex ion are more basic than Cl–. As
H such pairing of electrons can take place in
106. (d) In CO2 O has – 2 valency the 3d subshell. Due to pairing of electrons
In MnO2 O has – 2 valency in 3d subshell, one of the d-orbital becomes
In Na2O O has – 2 valency
vacant. This results in dsp2 hybridisation
In BaO 2 O is O 22 and the complex so formed in square planar.
it is true peroxide 113.(c) Rusting involves reduction of absorbed
[ Note : Ba 2 O 2 BaO is Barium oxide] oxygen to OH– ions and oxidation of iron
107 (d) Li+ due to small size have high degree of to Fe2+ ions. The two ions and oxidation of
hydration. iron to Fe2+ ions. The two ions combine to
108 (c) Gelatin has maximum protective power
starch has minimum protective power yield Fe(OH)2 which gets oxidised to give
Fe2O3.nH2O (rust). The presence of acid
109. (a) Conc. of SO 4 is [SO 24 ] in BaSO 4 helps dissolution of pure iron to ferrous
10 11 ions while electrolytes increase the
10
= 10 conductivity and assist cell action.
0.1
EBD_7100
26 Topicwise AIIMS Solved Papers
114.(a) R is the correct explanation of A. equilibrium in the forward direction of those
reactions which proceed with absorption
COOH of heat (endothermic reactions), and in the
* OH backward direction of those reactions
H
which proceed with the evolution of heat
H OH (exothermic reactions).” E is clearly true
H * OH again according to Le-chatelier principle.
116.(a) Assertion is correct as for every 10ºC raise
COOH in temperature, the specific rate constant,
meso-form K nearly doubles. (Although it is not
correct for all reactions. For some reactions
K even gets tripled for 10ºC raise). The
COOH statement is clearly true and it explains the
H * OH assertion, as the rate of collision among
the molecules doubles for 10ºC rise in
HO H temperature. So the answer is (a).
H * OH 117..(c) Among oxyacids, the acidic character
increases with increase in oxidation state
COOH
meso-form of the central atom. Hence assertion is
correct. Structure of HNO 2 :
MIRROR O H N O ; Structure of HNO3 :
O
COOH H O N
O
HO * H
The assertion is true but the reason is
H OH wrong as can be clearly seen from the
* above structures.
H OH
118.(b) Metal having negative reduction potential
COOH or positive standard oxidation potential has
(+)-form a tendency to get itself oxidised and pass
into the solution.
119.(a) Lattice energy is the amunt of energy
COOH required to dissociate one mole of an ionic
crystal into its ions and hydration energy
H * OH is the amount of energy released when one
mole of ions undergo hydration. So for the
HO H
solubility of a solid in liquid hydration
HO * H energy must be greater than lattice energy
120.(c) The correct reason is : The overall electron
COOH deficiency in m-nitroaniline is much less
(–)-form (due to –R-effect of NO2 group and +R-
effect of NH 2 group) th an in m-
The central carbon atom is pseudochiral dinitrobenzene (–R-effect of the two NO2
carbon atom. groups) and hence does not accept
115.(d) The statement is clearly wrong in context additional electrons from a weak reducing
to Le-chateliers principle, which states that agent such as (NH4)2S and thus further
“increase in temperature shifts the reduction is prevented.
Mock AIIMS 27

SECTION III - BIOLOGY 128. (b) Certain mitotic and meitotic irregularties
such as formation of restitution nucleus,
121. (c) With the discovery of more and more fusions of spindles lead to doubling of
organisms, sometimes, it becomes difficult the chromosomes in a cell. Chromosome
to adjust an organism to the traditional doubling is in troduced by high
categories. Therefore to make taxonomic temprature, X-ray treatments, callus
position of a species more precise. The formation hybridisation and chemical
various obligate categories in hierarchial treatment like chlorofrom, chloral hydrate
colchicine, auxins, gibberellins and
classification are explained below
nicotine sulphate etc. induce polyploidy.
Division — Class — order — family 129. (a) Asterceae constitutes the most advanced
— Genus — Species — Tribe family among dicots. The syngenesious
122. (c) The bacterial genome/nucleoid is made of condition of the stamen is an adaptation
double stranded DNA without histone for entomophily
protein. The bacterial genome thus 130. (d) Tomato and tobacco both belong to the
represented by a single circular double family Solanaceae.
helical DNA. The genome contains some 131. (a) The central pith is surrounded by xylem,
100 chemical sites or loci. Each locus phloem, pericycle and endodermis . The
contains many genes. E coli contains about phloem occurs only outside the xylem e.g
4000 genes. Equisetum, Osmunda
123. (c) The fungus that causes the disease ‘Ergot 132. (c) D.P.D. or suction pressure (S.P.)
of Rye’ is Claviceps purpurea. It contains = O.P. – T. P.
Hence D.P.D. for A = 4 bars, B = 0 bars, C =
many poisonous alkaloids. The
5 bars, D = 4 bars
hallucinogenic drug LSD is extracted from
We know that direction of movement of
this fungi. water is from lower D.P.D. (S.P.) to higher
• Rust of wheat is used by Puccinia D.P.D. (S.P.).
graminis flow will be from B to A, C, and D.
• Powdery Mildew of Pea is caused by 133. (c) Chlorophyll has a tetrapyrrole porphyrin
Erysiphe. head (15 × 15Å) and a long chain alcohol
124. (b) Shape and number of chloroplast in called phytol (20 Å).
different member of algae is different 134. (d) Cyt a3 posseses two copper centers. The
latter help in transfer of electron to oxygen.
Chlamydomonas – cup shaphed, 1/cell
135. (a) In campylotropous ovule, the body is
Zygnema – Stellate, 2/cell curved but the embryosac is straight eg.
Spiral – Spirogyra 1/ cell Capsella. In Anatropous, the body of the
Collar shaped - Ulothrix 1/cell ovule is inverted and gets fused with
125. (c) The inner membrane of mitochondria funiculus along its whole length on one
possess small tennis like particles called side (most of th e angiosperms). In
elementary particles, F0 – F1 particles or orthotropous condition the body of ovule
oxysomes. There are 1 × 104 – 1 × 105 lies straight and upright over the funicle.
oxysomes in a mitochondria. e.g. Piperaceae, Polygonaceae.
136. (c) Nyctinastic (sleeping) movements : The
126. (c) + + 40 : ab 40 : + a 10 : + b : 10
diurnal (chan ges in day & night)
127. (a) Lac operon is an inducible operon system
movements of leaves cause sleeping
which regulates genetic material. The
movements during night which are called
genetic material remains switched off
nyctinastic movements. It may be
normally but becomes operational in the
photonastic as in oxalis or thermonastic as
presence of inducer.
in tulip flower.
EBD_7100
28 Topicwise AIIMS Solved Papers
137. (a) Mimicry is the resemblance of an organism •
Smooth muscle tissue occurs within
to its natural surroundings, like initating a almost every organ, forming sheets,
non-living object or another organism for bundles or sheaths around other
conceding itself from its enemies. Flowers tissues.
of Ophrys muscifera resemble the female Smooth muscle differs from both skeletal
wasps of Colpa aurea so that the male and cardiac muscle tissues in structure
wasp tries to copulate with the flowers and and function. Sarcomeres or myofibrils are
pollinate them. not present and are therefore not striated,
138. (a) E. coli are found in the human intestine. If ie. smooth.
they are present in water it indicates that 146. (d) The permanent teeth appear completely by
the water is polluted. the age of 12 years, except for the last
139. (c) Meristem culture is done for the molars, which , if present, are formed after
development of virus free plants. the age of 18 years. The dental formula
Meristematic tissue cells can be taken before the wisdom teeth appear would be
either from shoot or root tip.
2 1 2 2
140. (c) 2, 4 Dichlorophenoxy acetic acid is used i , c ; pm , m
2 1 2 2
for causing defoliation of forest trees. after wisdom teeth appear
141. (b) Species is the basic unit of classification.
2 1 2 3
The term was coined by John Ray. Most i , c ; pm , m
taxonomists define species as 2 1 2 3
morphologically distinct and reproductively 147. (b) Alveoli are the site of the respiratory
isolated natural population or groups of exchange of gases. Oxygen from the
populations where individuals resemble one alveolar air diffuses through the alveolar
another more closely that with the members epithelium and the capillary endotheliun
of other species, interbreed freely and form into the capillary blood and carbon dioxide
a genetically closed system. Gene transfer diffuses in the opposite direction.
occurs between populations of a species 148. (b)
by gene flow i.e. emigration and immigration.
Hepatic artery Liver
142. (c) Km (Michealis Menten constant). It is
defined as that substrate concentration at Heart Carotico systemic arch Dorsal aorta Coeliac artery
which under optimum conditions the rate
Splenic (spleen) Lienogastric
of an enzyme catalysed reaction reaches
half the maximum rate. Km is inversely Gastric (stomach)
proportional to the affinity of enzyme for
its substrate. 149. (a) Henle’s loop is associated with
143. (b) Fire bellied toad (Bombina) is a member of concentration of the urine and production
order – Anura of the class Amphibia. of hypertonic urine.
144. (b) Eichhornia crassipes is an American origin 150. (c) All mammals have 7 cervical vertebrae in
plant and now a troublesome water weed their neck (it is one of the typical mammalian
in India. characters). Though they become long in
145. (a) Characteristics of smooth muscle cells camel & giraff but the number is 7 only. As
• range from 5 to 10 µm in diameter and whale is also a mammal it must possess the
30 to 200 µm in length same 7 cervical vertebrae.
• spindle-shaped 151. (a) Xth pair of cranial nerves (vagus nerves)
• single, centrally located nucleus has a motor branch called cardiac nerve
which innervate cardiac muscles.
Mock AIIMS 29

152. (a) Luteinizing hormone (LH) stimulates 160. (c) In test tube baby, fertilization of the sperm
ovulation. Deficiency of insulin causes and the ova is carried out in lab conditions
diabetes mellitus. Deficiency of ADH or (in vitro) and when the embryos have
vasopressin causes diabetes insipidus. reached the 32-celled stage, it is implanted
Deficiency of parathormone causes tetany. back into the uterus of the surrogate
Deficiency of thyroxine causes cretinism mother.
in infants and myxoedema in adults. 161. (a) Due to the process of exosmosis, plasmoly-
153. (c) Follicle stimulating hormone (FSH) sis takes place. Hence water goes out of
stimulates maturation of ovarian follicles. the mango and the mango shrinks. Hence,
the reason is the correct explanation for
Luteinizing hormone stimulates testes to
the assertion.
secrete testosterone. Prolactin stimulates 162. (b) Nissl’s granules are both basophilic and
development of mammary glands during also contain RNA. They are basophilic
pregnancy. Human chorionic gonadotropin since they stain with basic dyes. The rea-
released from the placenta also maintains son is therefore not the correct explana-
the corpus luteum during pregnancy. tion of the assertion.
154. (a) Biotic potential is the inherent ability of an 163. (a) Phycobilins are accessory photosynthetic
organism to reproduce. Resistance refers pigments present in algae. These pigments
are present along with chlorophyll – and
to the sum total of all the limiting factors
help in capturing light for photosynthesis.
that inhibit further growth of population.
Phycobilins are pigments which contain
155. (b) Normal woman whose father was colour proteins and proteins are easily denatured
blin d would have received the X by heat. Hence, the reason is the correct
chromosome from her father X cX , explanation for the assertion.
164. (d) The inner mucosa is mainly meant to
marries a colour blind man Xc Y , their increase the surface area and not for
progeny would be: absorption. Hence, both assertion and
reason are false.
Xc X XcY 165. (e) Vital capacity is the total volume of air that
can be breathed out with maximum effort.
progeny X c X c XcX X cY XY The assertion in this case is false.
colour blind normal colour blind normal son 166. (c) Heart wood is non functional but it is
daughter daughter son
plugged due to the growth of parenchyma
156. (a) Genetic drift can operate only in smaller that thickens later on. The reason in this
population where in fluctuation can be case is false, but the assertion is true.
observed in the proportion of allele 167. (a) Glucose which is the final product of car-
bohydrate digestion is converted to gly-
distribution in the presence of external cogen in the liver and stored both in the
disturbances. liver and muscles of animals. This process
157. (c) Cro-Magnon succeeded Neanderthal. takes place only in animals and hence it is
Fossils of Neanderthal man have been called as animal starch. Hence the reason
found from Eroupe, Asia and north Africa. is the correct explanation for the assertion.
Fossils of Australopithecus have been Electron micrograph of a section of a liver
found in Africa. cell showing glycogen.
158. (b) The presence of juvenile hormone is 168. (c) Racemose inflorescence shows an indefi-
necessary for metamorphosis in to adult nite growth, which is why it is said to be an
159. (a) Morphine is an opiate narcotic, Bhang is a indeterminate inflorescence. The reason
hallucinogen, Reserpine derived form given is false.
Rauwolfia, is used a tranquilizer, cocaine 169. (e) Mouth parts of cockroach, honey bee &
is a stimulant. mosquito are homologus organs. Therefore
the assertion is false.
EBD_7100
30 Topicwise AIIMS Solved Papers
170. (b) The crossing over takes place at the four 176. (b) The interferons are a special defensive
strand stage so that recombinations can mechanism produced by the infected cells.
take place when the chromosomes sepa- On reaching the nearby uninfected cells
rate as chromatids. The gene linkages do they help in the formation of certain pro-
disappear if crossing over takes place at teins that prevent the multiplication of the
two strand stage. The reason is therefore virus. Hence the reason is not the correct
not the correct explanation of the asser- explanation of the assertion.
tion. 177. (e) Horticulture is the conscious raise of Veg-
etables, fruits & Ornamental plants and not
of cereal crops. Hence the assertion is
false.
178. (a) Viruses are bodies which have either DNA
or RNA as a genetic material. For multipli-
cation, it is the genetic material that enters
into the host cells which then with the help
of the DNA of the host cell prepare their
A double crossing over. protein shield. The reason is the correct
171. (c) Oncogenes integrate their DNA with DNA explanation for the assertion.
of the host cells and not their RNA. The 179. (a) All aggregate and multiple fruits develop
reason in this case is false. from other floral parts like the thalamus,
172. (a) HIV is found in body fluids like blood & calyx, etc. Hence the reason is the correct
semen. It is a retrovirus which has RNA as explanation for the assertion.
its genetic material. It hence shows reverse 180. (c) In Casuarina & Betula the pollen tube enters
transcription. Hence the reason is the cor- through th e Chalaza and not the
rect explanation for the assertion. microphyle. Hence the reason is false.
173. (b) Genes although show a linear order but are
arranged in the DNA as a helical coiled SECTION IV - G.K.
structure. Hence the reason is not the cor- 181. (a) 182. (d) 183. (b)
rect explanation for the assertion given.
184. (d) 185. (c) 186. (c)
174. (a) The virus cannot multiply outside living
systems. They require a host in order to 187. (a) 188. (b) 189. (d)
multiply. Hence the reason is true for the 190. (a) 191. (d) 192. (b)
assertion. 193. (d) 194. (b) 195. (a)
175. (e) The allergens are actually not glycogen 196. (d) 197. (a) 198. (c)
molecules but are protein molecules. Hence 199. (b) 200. (d)
the assertion is false.

You might also like